You are on page 1of 304

ELEv ENTH EDITION

LA N G E q &A

PSYCHIATRY
Sean M. Blitzstein, MD
Staff Psychiatrist, Jesse Brow n VA Med ical Center
Director, Psychiatry Clerkship
Clinical Associate Professor of Psychiatry
University of Illinois at Chicago
Chicago, Illinois

N ew York Chicago San Francisco Athens Lond on Mad rid Mexico City
Milan N ew Delhi Singap ore Syd ney Toronto
Copyright 2017 by McGraw-Hill Education. All rights reserved. Except as permitted under the United States Copyright Act of 1976,
no part of this publication may be reproduced or distributed in any form or by any means, or stored in a database or retrieval system,
without the prior written permission of the publisher.

ISBN: 978-1-25-964395-8
MHID: 1-25-964395-6.

The material in this eBook also appears in the print version of this title: ISBN: 978-1-25-964394-1,
MHID: 1-25-964394-8.

eBook conversion by codeMantra


Version 1.0

All trademarks are trademarks of their respective owners. Rather than put a trademark symbol after every occurrence of a trademarked
name, we use names in an editorial fashion only, and to the bene t of the trademark owner, with no intention of infringement of the
trademark. Where such designations appear in this book, they have been printed with initial caps.

McGraw-Hill Education eBooks are available at special quantity discounts to use as premiums and sales promotions or for use in corpo-
rate training programs. To contact a representative, please visit the Contact Us page at www.mhprofessional.com.

Notice

Medicine is an ever-changing science. As new research and clinical experience broaden our knowledge, changes in treatment and drug
therapy are required. The authors and the publisher of this work have checked with sources believed to be reliable in their efforts to
provide information that is complete and generally in accord with the standards accepted at the time of publication. However, in view
of the possibility of human error or changes in medical sciences, neither the authors nor the publisher nor any other party who has been
involved in the preparation or publication of this work warrants that the information contained herein is in every respect accurate or com-
plete, and they disclaim all responsibility for any errors or omissions or for the results obtained from use of the information contained in
this work. Readers are encouraged to con rm the information contained herein with other sources. For example and in particular, readers
are advised to check the product information sheet included in the package of each drug they plan to administer to be certain that the
information contained in this work is accurate and that changes have not been made in the recommended dose or in the contraindications
for administration. This recommendation is of particular importance in connection with new or infrequently used drugs.

TERMS OF USE

This is a copyrighted work and McGraw-Hill Education and its licensors reserve all rights in and to the work. Use of this work is subject
to these terms. Except as permitted under the Copyright Act of 1976 and the right to store and retrieve one copy of the work, you may
not decompile, disassemble, reverse engineer, reproduce, modify, create derivative works based upon, transmit, distribute, disseminate,
sell, publish or sublicense the work or any part of it without McGraw-Hill Educations prior consent. You may use the work for your
own noncommercial and personal use; any other use of the work is strictly prohibited. Your right to use the work may be terminated if
you fail to comply with these terms.

THE WORK IS PROVIDED AS IS. McGRAW-HILL EDUCATION AND ITS LICENSORS MAKE NO GUARANTEES OR WAR-
RANTIES AS TO THE ACCURACY, ADEQUACY OR COMPLETENESS OF OR RESULTS TO BE OBTAINED FROM USING
THE WORK, INCLUDING ANY INFORMATION THAT CAN BE ACCESSED THROUGH THE WORK VIA HYPERLINK OR
OTHERWISE, AND EXPRESSLY DISCLAIM ANY WARRANTY, EXPRESS OR IMPLIED, INCLUDING BUT NOT LIMITED
TO IMPLIED WARRANTIES OF MERCHANTABILITY OR FITNESS FOR A PARTICULAR PURPOSE. McGraw-Hill Education
and its licensors do not warrant or guarantee that the functions contained in the work will meet your requirements or that its opera-
tion will be uninterrupted or error free. Neither McGraw-Hill Education nor its licensors shall be liable to you or anyone else for any
inaccuracy, error or omission, regardless of cause, in the work or for any damages resulting therefrom. McGraw-Hill Education has no
responsibility for the content of any information accessed through the work. Under no circumstances shall McGraw-Hill Education and/
or its licensors be liable for any indirect, incidental, special, punitive, consequential or similar damages that result from the use of or
inability to use the work, even if any of them has been advised of the possibility of such damages. This limitation of liability shall apply
to any claim or cause whatsoever whether such claim or cause arises in contract, tort or otherwise.
Co nte nts

Contributors ............................................................................................................................................................v

Preface ................................................................................................................................................................. vii

Acknow ledgments ........................................................................................................................................... viii

1. Child and Adolescent Psychiatry ................................................................................................................. 1


Qu estions ........................................................................................................................................................... 1
Answ ers and Explanations ........................................................................................................................... 17

2. Adult Psychopathology ............................................................................................................................... 33


Qu estions ......................................................................................................................................................... 33
Answ ers and Exp lanations ........................................................................................................................... 58

3. Somatic Treatment and Psychopharmacology ........................................................................................ 81


Questions ......................................................................................................................................................... 81
Answ ers and Explanations ......................................................................................................................... 103

4. Psychological Treatment and Management .......................................................................................... 123


Qu estions ....................................................................................................................................................... 123
Answ ers and Exp lanations ......................................................................................................................... 132

5. Legal and Ethical Issues in Psychiatry and Medicine ......................................................................... 141


Qu estions ....................................................................................................................................................... 141
Answ ers and Exp lanations ......................................................................................................................... 154

6. D ifferential D iagnosis and Management ............................................................................................... 165


Qu estions ....................................................................................................................................................... 165
Answ ers and Explanations ......................................................................................................................... 185

7. Practice Test 1 .............................................................................................................................................. 205


Qu estions ....................................................................................................................................................... 205
Answ ers and Explanations ......................................................................................................................... 225

iii
iv Conte nts

8. Practice Test 2 ............................................................................................................................................... 247


Qu estions ....................................................................................................................................................... 247
Answ ers and Explanations ......................................................................................................................... 268

Bibliography ...................................................................................................................................................... 287

Index .................................................................................................................................................................... 291


Co ntributo rs

Shetal M. Amin, MD Marika Inga Wrzosek, MD


Legacy Com m u nity H ealth Services Director, Med ical Stu d ent Ed u cation
Clinical Assistant Professor Assistant Professor of Clinical Psychiatry
Departm ent of Psychiatry and Behavioral Departm ent of Psychiatry and Behavioral
Sciences Med icine
Baylor College of Med icine Med ical College of Wisconsin
H ouston, Texas Milw au kee, Wisconsin

Jennifer Sprague, MD Alexander K. Yuen, MD


Psychiatry Resid ent Child and Ad olescent Psychiatry Chief Fellow
Departm ent of Psychiatry Institu te of Ju venile Research
University of Illinois at Chicago Departm ent of Psychiatry
Chicago, Illinois University of Illinois at Chicago
Chicago, Illinois
Kelley A. Volpe, MD
Child and Ad olescent Psychiatry Fellow
Institu te for Ju venile Research
Departm ent of Psychiatry
University of Illinois at Chicago
Chicago, Illinois

v
This page intentionally left blank
Pre fac e

Welcom e to Lange Q&A : Psychiatry, 11th ed ition. It is m y sincere p leasu re to p rovid e you w ith this ed ition
in su p p ort of you r ongoing ed u cation in p sychiatry. Psychiatric research and p ractice are at the forefront of
m ed icine, incorp orating p harm acology, p sychology, sociology, and neu rosciences; the resu lt is a genu ine
biop sychosocial ap p roach to p atient care. Su ch a sizeable integration yield s a broad field in w hich to stu d y
and test. Becau se of this, the 11th ed ition covers the gam u t of top ics, from ad u lt and child p sychiatry, to
psychopharm acology and ethics. All m ajor d iagnoses are covered , w ith the em phasis being on d ifferential
d iagnosis and treatm ent (w hether p sychological or som atic). In fact, this ed ition consists of eight chapters
w ith 810 qu estions, and it has been u pd ated to reflect the Am erican Psychiatric Associations Diagnostic and
Statistical M anual of M ental Disorders, Fifth Edition (DSM -5). As this book is geared tow ard m ed ical stu d ents
stu d ying for their Psychiatry clerkship exam inations, as w ell as for their United States Med ical Licensing
Exam ination (USMLE) Step 2 CK exam ination, each and every qu estion is clinically focu sed com p rising of
a p atient-centered vignette. Consistent w ith the USMLE form at, the answ ers are either m ultip le-choice or
extend ed m atching item s. In ord er to p rom ote self-assessm ent and form ative learning, all answ ers inclu d e
d etailed explanations regard ing why the incorrect choices are flaw ed . A list of references are also includ ed for
those stud ents w ho w ish to further research a particu lar issue.
Finally, a few su ggestions on how best to u se this book. This book shou ld be u sed as a su p p lem ent to
you r p rim ary learning. It neither rep laces read ing a basic text on Psychiatry, nor seeing live p atients d u ring
a rotation. Chap ters 1 throu gh 6 can be u sed to reinforce or com p lem ent certain areas. The final tw o chapters
(Chapters 7 and 8) are practice tests, each consisting of greater than 115 rand om ly ord ered qu estions that
sim u late an actu al board exam ination.

Sean M . Blitzstein, M D

vii
Acknowle dg me nts

Thank you to m y p atient, su p p ortive, and organized ed itors at


McGraw -H ill. I also sincerely thank m y contributors, and m y prior
m entors and m entees, w ho have all given m e so m u ch. Finally, to
Jackie and Anika, w ho are m y hom e.

Sean M . Blitzstein, M D

viii
CHAPTER 1

Child and Ado le s c e nt


Ps yc hiatry
Que s tions

DIRECTIONS (Questions 1 through 75): For each of d u ring her p reviou s m ajor d ep ressive ep i-
the multiple-choice questions in this section, select sod e, she d enies it at this visit. At this tim e,
the lettered answ er that is the one best response in she also d enies self-inju rious u rges. Which of
each case. the follow ing w ou ld be the m ost ap propriate
p lan for treatm ent?
Questions 1 and 2 (A) Ad m it her to the hosp ital given her
history and potential risk.
A 15-year-old girl w ith a history of m ajor d epressive
(B) Ask you r p sychiatric colleagu e to assess
d isord er (MDD) com es to you r p rim ary care office
her at their next available intake.
for a rou tine visit. When you w alk into the exam -
ining room , you notice that she is w ithd raw n w ith (C) Prescribe antid ep ressant m ed ications
her head bent d ow n. She d isplays p oor eye con- w ith a follow -u p ap p ointm ent in
tact and barely speaks d uring you r interview . She 2 w eeks.
ad m its that she u ses m arijuana ap proxim ately tw ice (D) Refer her to a social w orker for
per w eek and cu ts on her arm w hen stressed . H er p sychotherapy.
m other reveals that she is particu larly w orried as (E) Tell the m other you w ill follow u p w ith
the girls paternal u ncle com m itted su icid e 10 years her at you r next rou tine visit.
ago. You su spect that she is having a recu rrence of
d ep ressive sym ptom s and are concerned about her Questions 3 and 4
risk for su icid e.
A m other brings her 7-year-old son to you becau se
1. Which of the follow ing factors m ost increases she is w orried that he sits u p in bed in the m id d le of
this patients risk of com m itting suicid e? the night and scream s. She says that at those tim es
he is inconsolable but eventually falls back to sleep.
(A) Cu tting behavior
(B) Gend er 3. Which of the follow ing is the m ost likely
(C) Presence of d ep ression d iagnosis?
(D) Relative w ho com m itted su icid e (A) Interm ittent exp losive d isord er
(E) Su bstance u se (B) N arcolep sy
(C) N ightm are d isord er
2. Fu rther history is obtained , and the p atient
(D) N onrap id eye m ovem ent (N REM) sleep
reveals that she has, in fact, been feeling m ore
arou sal d isord er, sleep terror typ e
d ep ressed recently, w ith d ifficu lty sleep ing,
low ap p etite, fatigu e, and p roblem s con- (E) N REM sleep arou sal d isord er,
centrating. While she had su icid al id eation sleep w alking type

1
2 1: Child a nd Ad ole s c e nt P syc hia try

4. Du ring w hich stage of sleep d o these ep isod es Questions 7 and 8


m ost likely occur?
Parents bring their 6-year-old boy to the clinic at the
(A) Stage 1 requ est of the boys teachers. The teachers rep ort
(B) Stage 2 that he is quiet in class. When he d oes talk, he fre-
(C) Stages 3 to 4 quently m akes errors w ith verb tense. H is parents
(D) Rap id eye m ovem ent (REM) stage recall that his sp eech w as d elayed . On exam ination,
the boy is friend ly and cooperative. H is speech is
(E) Any stage
clear, but he u ses sim ple sentences w ith a lim ited
vocabulary. Otherw ise, his physical and laboratory
Questions 5 and 6
exam ination is norm al.
A 17-year-old girl w ith a history of asthm a p res-
ents for a p hysical exam ination p rior to entering 7. Which of the follow ing is the m ost likely
college. You note that she appears angry. Upon d iagnosis?
fu rther qu estioning, you learn that she has felt irri- (A) Child hood -onset flu ency d isord er
table for the past 6 m onths since breaking up w ith
(B) Langu age d isord er
her boyfriend of 2 years. She says she feels tired all
the tim e and com es hom e from school every d ay, (C) Social (p ragm atic) com m u nication
lies on the cou ch, and w atches You Tu be vid eos. H er d isord er
grad es have d rop p ed becau se she cannot concen- (D) Sp ecific learning d isability
trate. Despite her fatigue, she com plains of d ifficu lty (E) Sp eech sou nd d isord er
sleeping. She has lost 12 lb over the last 6 m onths.
She rep orts that she quit the senior celebration com - 8. By w hat age w ou ld failu re to sp eak 200 w ord s
m ittee, no longer hangs ou t w ith her friend s, can- be m ost consistent w ith a sp eech d elay in this
not im agine things w ill im p rove, and is consid ering patient?
not going to college. H er p hysical and laboratory
(A) 1 year
exam ination is norm al.
(B) 2 years
5. Which of the follow ing m ed ications w ou ld be (C) 3 years
the m ost app ropriate to treat this p atient? (D) 4 years
(A) Carbam azep ine (Tegretol) (E) 5 years
(B) Im ip ram ine (Tofranil)
Questions 9 and 10
(C) Lithiu m
(D) Olanzap ine (Zyp rexa) A 15-year-old boy w ithou t prior p sychiatric his-
(E) Sertraline (Zoloft) tory is on his school concert trip. H e is brought to a
local em ergency room becau se of the acu te onset of
6. Accord ing to the Diagnostic and Statisti- increased anger, agitation, and p aranoia. On inter-
cal M anual of M ental Disorders, Fifth Edition view , he reports feeling u nsafe because a govern-
(DSM -5), w hich of the follow ing sym p tom s m ent agency is spying on him .
in this p atient d ifferentiate her d isord er from
that in an ad u lt? 9. Which of the follow ing tests w ou ld be the
m ost im portant to ord er first?
(A) Anhed onia
(A) Electroencep halogram (EEG)
(B) Decreased concentration
(B) Glu cose tolerance test
(C) Insom nia
(C) Positron em ission tom ograp hy (PET)
(D) Irritable m ood
scan of his head
(E) Weight loss
(D) Thyroid fu nction tests
(E) Urine toxicology screen
Que s tions : 415 3

10. All approp riate laboratory evaluations and She rep orts that he ap pears extrem ely revved up
stu d ies com e back w ithin norm al lim its. Which and hyp eractive.
of the follow ing d iagnoses w ou ld be m ost
consistent w ith this patients presentation? 13. Which of the follow ing is the m ost likely
d iagnosis?
(A) Anorexia nervosa
(B) Bip olar d isord er, m anic (A) Attention-d eficit/ hyp eractivity d isord er
(C) Bord erline p ersonality d isord er (ADH D)
(D) Generalized anxiety d isord er (B) Bip olar d isord er, m ost recent ep isod e
m anic
(E) Major d ep ressive d isord er
(C) Major d ep ressive d isord er w ith
Questions 11 and 12 p sychotic featu res
(D) Panic d isord er
An 8-year-old boy is referred to you by a school (E) Schizop hrenia
nu rse because he has been com p laining of stom ach-
aches every m orning in school. On interview ing the 14. Ingestion of w hich of the follow ing su b-
boys m other, you learn that he d oes not like to go stances w ou ld m ost likely bring abou t sim ilar
to school, insists on com ing hom e im m ed iately after sym ptom s?
school each d ay, and sleep s in his parents bed at
(A) Alcohol
night. The m other d enies other com p laints.
(B) Cannabis
11. Which of the follow ing is the m ost likely (C) Cocaine
d iagnosis? (D) H eroin
(A) Post-trau m atic stress d isord er (PTSD) (E) Phencyclid ine (PCP)
(B) Reactive attachm ent d isord er
15. A 9-year-old boy w alks into you r office
(C) Sep aration anxiety d isord er accom panied w ith his mother. They are argu ing
(D) Social anxiety d isord er (social p hobia) abou t his w anting a new portable vid eo gam ing
(E) Sp ecific p hobia system . H is m other, exasp erated w ith her sons
behavior, tells you that they w ere late becau se
12. Which of the follow ing w ou ld you m ost likely it took him a long tim e to finally agree to get
find in this p atients history? into the car to come to the appointm ent w ith
you. Alone w ith you in you r office, he appears
(A) Low levels of p arental control
irritated and refu ses to answ er your questions
(B) Parent w ith an anxiety d isord er or look up from his lap. After 10 minutes, he
(C) Parents w ith a w arm and accep ting style tells you about the annoying kid s in his
(D) Secu re attachm ent class and how they mad e m e get in trouble.
(E) Tem p eram ent characterized by Fu rther history from the m other reveals that,
sociability and extroversion d espite the above behavior, he has never been
violent or d estructive, and he has not had any
Questions 13 and 14 legal problems. Which of the follow ing is the
m ost likely d iagnosis?
A 16-year-old boy is brou ght to the p ed iatric em er-
gency d ep artm ent by his grand m other w ho rep orts (A) Agorap hobia
he is not acting like him self. H e rep orts that he (B) Attention-d eficit/ hyp eractivity d isord er
feels incred ibly great and d oes not need a d octor (ADH D)
becau se he has p ow ers to heal him self of all sickness. (C) Cond u ct d isord er
H e sp eaks rapid ly and his grand m other rep orts he (D) Generalized anxiety d isord er (GAD)
has not slept m ore than 6 hou rs over the p ast 3 d ays. (E) Op p ositional d efiant d isord er (ODD)
4 1: Child a nd Ad ole s c e nt P syc hia try

Questions 16 through 18 into the street in front of a car on the w ay home from
school. H e is physically unharmed , but refused to
A 15-year-old girl w ho is a com p etitive figu re speak to the emergency room d octor. You are asked
skater presents w ith concerns about her w eight. She to consult as you are on your psychiatry rotation. The
believes that she w ou ld be a better skater if she cou ld emergency room doctor is suspicious that the boys
lose w eight and feels very u p set and fru strated that behavior reflected und erlying suicid al impulses.
she has failed in her attem pts. The girl relu ctantly
ad m its that she som etim es eats a w hole lot of food 19. Which of the follow ing op ening statem ents
at one tim e su ch as a qu art of ice cream , a large bag w ou ld be the m ost valu able in facilitating the
of potato chips, and a jar of peanu t butter. You also boys d iscu ssion of the situ ation?
notice abrasions on the back of her right hand .
(A) You su re w ere lu cky the car sw erved
16. Which of the follow ing opening statem ents at the last m inu te.
w ou ld be the m ost ap p rop riate? (B) You w erent trying to actu ally get hit
by the car, w ere you ?
(A) Tell m e abou t the scratches on you r
hand . (C) What were you thinking w hen you ran
into the street?
(B) Ive noticed the cu ts on you r hand . Are
you trying to hu rt you rself? (D) It sou nd s like qu ite a d ay. Can you tell
m e abou t w hat hap pened after school
(C) H ow d id the scratches hap p en?
tod ay?
(D) I see you have scratches on you r hand .
(E) Child ren w ho try to hu rt them selves
Do you have a cat?
are very confu sed . Are you confu sed ?
(E) Som etim es I see girls w ho m ake
them selves throw up. H ave you ever 20. Which of the follow ing sym p tom s of MDD
d one that? w ou ld be m ore likely in this p atient com p ared
to an ad olescent w ith MDD?
17. Which of the follow ing laboratory abnorm ali-
ties w ould you m ost likely find in this patient? (A) Dru g u se
(B) H op elessness
(A) Elevated iron
(C) H yp ersom nia
(B) Elevated p rotein
(D) Psychom otor agitation
(C) H yp erchlorem ia
(E) Weight change
(D) H yp okalem ia
(E) H yp onatrem ia 21. Which of the follow ing w ou ld be this child s
m ost likely m ethod of attem p ting suicid e?
18. Which of the follow ing typ es of p sychother-
ap y w ould likely be the m ost effective for this (A) Firearm s
p articular p atient? (B) H anging
(A) Cognitive-behavioral therap y (C) Ju m p ing from a significant height
(B) Fam ily therap y (D) Stabbing
(C) Grou p therap y (E) Su bstance ingestion
(D) Psychoanalysis
22. An 11-year-old boy w ith enu resis presents
(E) Psychod ynam ic p sychotherap y to the clinic for rou tine follow -up. H is bed -
w etting had not resp ond ed to behavioral
Questions 19 through 21 interventions, so you had p reviou sly initiated
An 8-year-old boy w ith a history of major depressive treatm ent w ith intranasal d esm opressin
d isorder (MDD) treated w ith fluoxetine (Prozac) is (Dd avp ) after com p letion of a fu ll p hysical
brought to the emergency department after running and laboratory exam ination. Which of the
Que s tions : 1627 5

follow ing signs/ sym ptom s w ou ld be the m ost m ake od d , rep etitive m ovem ents w ith her hand s,
likely ad verse effect? and p refers to line her toys u p . She becom es acu tely
d istressed w hen rou tines are changed . The child s
(A) H ead ache
physical exam ination is u nrem arkable, bu t you note
(B) H ypotension that she d oes not seem to cry w hen she trip s over
(C) Liver toxicity you r exam ination stool, keep ing her attention on
(D) Sed ation aligning you r colored p encils.
(E) Trem or
25. Which of the follow ing is the m ost likely
Questions 23 and 24 d iagnosis?

A 9-year-old boy w ith a history of p anic d isord er (A) Au tism sp ectru m d isord er (ASD)
treated w ith cognitive-behavioral therapy is brought (B) Intellectu al d isability (ID)
to your office by his m other becau se he has been (C) Rett synd rom e
irritable and d ep ressed . On p hysical exam ination, (D) Selective m u tism
the boy appears d epressed bu t otherw ise norm al. (E) Social (p ragm atic) com m u nication
Laboratory exam ination is norm al. d isord er
23. What w ou ld be the likelihood of this p atient 26. The m other asks you abou t p rognosis for her
having a com orbid m ajor d ep ressive d isord er d au ghter s cond ition. Which of the follow ing
(MDD)? is the m ost p ositive p rognostic factor?
(A) 5% (A) Com orbid ep ilep sy
(B) 15% (B) Engagem ent in social skills training
(C) 25% (C) Fu nctional language by 5 years of age
(D) 50% (D) Intellectu al d isability
(E) 75% (E) Presence of catatonic sym ptom s
24. After a thorou gh history and m ental statu s Questions 27 and 28
exam ination, you d iagnose the boy w ith MDD
and d ecid e to initiate treatm ent w ith flu ox- A 12-year-old boy is referred by the cou rt for evalu -
etine. You inform the boy and his m other of ation. H e skip s school, stays ou t late at night, and
possible ad verse effects of flu oxetine. Which verbally abu ses his parents. H e has run aw ay from
of the follow ing w ou ld be the m ost likely sid e hom e on three sep arate occasions, p rom p ting his
effect? p arents to call the p olice. H e has been cau ght shop-
lifting and has been in nu m erou s p hysical fights
(A) H yp otension
w ith his p eers.
(B) Liver toxicity
(C) N au sea 27. Up on fu rther history, w hich of the follow ing
(D) Sed ation w ou ld m ost likely be fou nd in this p atient?
(E) Weight gain (A) Absence of a biological father
(B) Absence of a biological m other
Questions 25 and 26
(C) Mother w ith an anxiety d isord er
A 30-month-old girl is brou ght to the clinic by her (D) Patient being an only child
m other for a rou tine visit. The m other tells you that (E) Parents w ho d o not u se corp oral
the girl seem s to avoid affection, often d oes not look p u nishm ent
her in the eye, has stopp ed speaking in preschool,
and d oes not really engage w ith her p reschool peers
nor w ith her 5-year-old brother. She continu es to
6 1: Child a nd Ad ole s c e nt P syc hia try

28. Which of the follow ing p ersonality d isord ers that at any time, w ithout w arning, the boy w ill make a
is this boy m ost likely to d evelop ? d isruptive sound or shout out in class. They d escribe
him as polite and neat but restless and jumpy.
(A) Antisocial p ersonality d isord er
(B) Avoid ant p ersonality d isord er 31. Which of the follow ing is the m ost likely
(C) Paranoid p ersonality d isord er d iagnosis?
(D) Schizoid p ersonality d isord er
(A) Cond u ct d isord er
(E) Schizotyp al p ersonality d isord er
(B) Op p ositional d efiant d isord er
Questions 29 and 30 (C) Panic d isord er
(D) Sep aration anxiety d isord er
A 10-year-old girl w ho has recently been d iagnosed (E) Tou rette d isord er
w ith d iabetes m ellitu s typ e I is referred to you by
her ped iatrician for an evalu ation. You notice that 32. Which of the follow ing m ed ications w ou ld be
she seem s sad . H er parents are concerned about her the m ost appropriate to prescribe initially?
being d ep ressed . Consid eration is given for d iag-
nosing ad justm ent d isord er w ith d epressed m ood (A) Bu p rop ion (Wellbu trin)
versu s m ajor d epressive d isord er (MDD). (B) Clonid ine (Catap res)
(C) H alop erid ol (H ald ol)
29. Which of the follow ing criteria for the d iagno- (D) Paroxetine (Paxil)
sis of ad ju stm ent d isord er m ost d istinguishes
(E) Venlafaxine (Effexor)
it from MDD?
(A) Sym p tom s cau se m arked d istress or Questions 33 and 34
significant im pairm ent in fu nctioning.
A 7-year-old boy w ith leukem ia is referred to you
(B) Sym p tom s d evelop follow ing an
becau se of concerns abou t his m ood . H is p ar-
id entifiable stressor.
ents rep ort that he flu ctu ates betw een app earing
(C) Sym p tom s d evelop w ithin 3 m onths of d epressed and acting angry. At tim es, he plays qui-
the onset of the stressor. etly in his room , bu t at other tim es he d isp lays anger
(D) Sym p tom s d o not p ersist for m ore than outbu rsts, often hitting his 4-year-old brother. H is
6 m onths follow ing term ination of the m other ad m its that she has d ecreased her expecta-
stressor. tions of him , and feels that since he is ill he should
(E) Sym p tom s d o not rep resent norm al not receive any p u nishm ents.
bereavem ent.
33. Which of the follow ing m ethod s w ould be the
30. Approxim ately w hat p ercentage of child ren m ost effective w ay to engage his m other in a
w ho are d iagnosed w ith d iabetes m ellitu s d iscussion regard ing the role of her actions on
typ e I d evelop ad justm ent d isord er follow ing the boys behavior?
their m ed ical d iagnosis?
(A) Acknow led ge her gu ilt and anxiety
(A) 1% abou t her sons illness and explain the
(B) 5% im p ortance of p rovid ing lim its and
(C) 10% stru cture for his em otional w ell-being.
(D) 33% (B) Em p athize w ith the trau m a of having a
sick child .
(E) 75%
(C) Refer her to a p arent su p p ort grou p .
Questions 31 and 32 (D) Sit qu ietly and m ake no com m ents
abou t her p arenting style.
A 9-year-old boy is referred to you for evaluation (E) Tell her that treating her son like a baby
because of repeated teasing at school related to his is hu rting him em otionally.
inappropriate peer interactions. The teachers report
Que s tions : 2838 7

34. It is d eterm ined that the boy is su ffering from Questions 37 and 38
m ajor d epressive d isord er. You d iscu ss both
p sychopharm acologic and psychotherapeu tic An 8-year-old boy w ith a family history of tic d isor-
treatm ent options, but she is concerned abou t d ers is referred to you for an evaluation of behav-
his ongoing leukem ia treatm ent. Which of ioral d ifficulties in school. H is teachers report that he
the follow ing approaches w ou ld be the m ost is unable to sit still, constantly fid gets, and is unable
ap propriate regard ing treating both his leu ke- to complete class w ork becau se he is so easily d is-
m ia and m ajor d epression? tracted . When he com pletes his w ork, it is often d one
carelessly and is frequently not tu rned in. The teach-
(A) Treat the d ep ression p rior to the ers also note that he blurts ou t replies and has a hard
leu kem ia. tim e d u ring quiet tim e. The boys m other reports that
(B) Treat the d ep ression after the leu kem ia. he has alw ays had a lot of energy. Preparing to leave
(C) Treat the d ep ression concu rrently w ith for school in the m orning is challenging because of
the leu kem ia. her sons d isorganization and forgetfulness. Other-
(D) Treat the leu kem ia first and the w ise, she has no complaints. She d enies that her son
d epression w ill resolve. prod uces any repetitive m ovem ents or sou nd s.
(E) Treat the leu kem ia as treating the
37. Which of the follow ing is the m ost likely
d epression w ill not be effective in the
d iagnosis?
setting of a m alignancy.
(A) Attention-d eficit/ hyp eractivity d isord er
Questions 35 and 36 (ADH D)
(B) Bip olar d isord er
An 8-year-old boy p resents to you r office for a rou -
tine visit. One m onth earlier, you d iagnosed him (C) Cond u ct d isord er
w ith Tou rette d isord er and p rescribed m ed ication. (D) Op p ositional d efiant d isord er
H e and his p arents rep ort that the m ed ication has (E) Unsp ecified d isru p tive behavior
been help fu l. d isord er

35. Which of the follow ing d isord ers w ou ld this 38. The p atient is su bsequ ently treated for the
p atient be m ost likely to d evelop ? above cond ition. H e retu rns to an app oint-
m ent after several w eeks, now w ith rep etitive
(A) Au tism sp ectru m d isord er
grim acing and blinking m ovem ents, w hich
(B) Bip olar d isord er have resu lted in his getting teased in school.
(C) Langu age d isord er Which of the follow ing classes of m ed ications
(D) Obsessive-com p u lsive d isord er (OCD) is m ost likely to be resp onsible?
(E) Sep aration anxiety d isord er (A) Benzod iazep ines
(B) D 2 antagonists
36. Prior infection w ith w hich of the follow ing
w ou ld be m ost likely fou nd in the history of (C) Monoam ine oxid ase inhibitors (MAOIs)
this patient? (D) Selective serotonin reu p take inhibitors
(SSRIs)
(A) Haemophilus influenzae
(E) Stim u lants
(B) Influ enza viru s
(C) Parainflu enza viru s Questions 39 and 40
(D) Resp iratory syncytial viru s
A 6-year-old boy is referred to you by his school to
(E) Streptococcus pyogenes
evalu ate his d ifficu lty w ith keep ing u p w ith read ing
and m ath d esp ite his above average intelligence. It
is su sp ected that he su ffers from a learning d isord er,
so further testing and evaluation is ind icated .
8 1: Child a nd Ad ole s c e nt P syc hia try

39. Which of the follow ing find ings w ou ld be (C) Methylp henid ate (Ritalin)
required for a d iagnosis of sp ecific learning (D) Paroxetine (Paxil)
d isord er w ith im p airm ent in read ing? (E) Trazod one (Desyrel)
(A) The child has an above average IQ
(intelligence qu otient) bu t below Questions 43 through 45
average read ing achievem ent.
A 10-year-old boy is referred to you becau se his
(B) The child has an average IQ and below ped iatrician su spects that he m ay have attention-
average read ing achievem ent. d eficit/ hyperactivity d isord er (ADH D). After a
(C) The child s read ing achievem ent is thorou gh history, physical exam ination, and labora-
substantially below the child s IQ. tory investigation you m ake the d iagnosis of ADH D.
(D) The child s read ing achievem ent is After d iscu ssing the ad verse effects of m ed ications,
substantially above the child s IQ. you p rescribe m ethylp henid ate to be taken in the
(E) The child s read ing achievem ent and IQ m orning and at lu nch on school d ays.
are both below average.
43. Which of the follow ing w ou ld be the m ost
40. What w ou ld be the ap p roxim ate risk of this likely sid e effect of m ethylp henid ate?
child having a com orbid psychiatric d isord er? (A) H yp otension
(A) 5% (B) Insom nia
(B) 10% (C) Liver toxicity
(C) 25% (D) Trem or
(D) 50% (E) Weight gain
(E) 75%
44. What is the likelihood that this patient w ill sig-
Questions 41 and 42 nificantly benefit from the m ethylp henid ate?

An 8-year-old child is referred to you for an evalu- (A) 10%


ation of bed -w etting. Several behavioral interven- (B) 25%
tions have been attem pted , inclu d ing eliminating (C) 33%
fluid intake in the evening, sched uled aw akenings at (D) 50%
night to use the bathroom, and a urine alarm (a bell (E) 70%
and pad ). These techniqu es have been u nsu ccessful,
and the child continues to u rinate in bed every night. 45. Despite education and reassurance, the mother
remains opposed to stimulant medications but
41. What is the likelihood of this p atient having a still w ishes her son to receive pharmacologic
com orbid m ental illness? treatment for his ADH D. Which of the follow -
(A) 5% ing med ications w ould be the most appropriate
(B) 10% to prescribe?
(C) 20% (A) Arip ip razole (Abilify)
(D) 50% (B) Atom oxetine (Strattera)
(E) 75% (C) Citalop ram (Celexa)
(D) Mixed am p hetam ine salts (Ad d erall)
42. Which of the follow ing m ed ications w ou ld be (E) Valp roic acid (Dep akote)
the m ost ap propriate to prescribe to treat this
p atient? Questions 46 and 47
(A) Benztrop ine (Cogentin)
Du ring a rou tine office visit, the m other of a
(B) Desm op ressin (Dd avp ) 37-m onth-old girl tells you that she is concerned
Que s tions : 3951 9

abou t her d au ghters behavior. Since the birth of 48. In which of the following areas would you most
her son 4 m onths earlier, the m other states that her likely expect additional difficulties in this patient?
d aughter has been m ore irritable and angry. The
(A) Attention
child has told her m other that she d oes not w ant the
baby anym ore and to take him back. The m other is (B) Fine m otor skills
esp ecially concerned becau se her d au ghter tried to (C) Gross m otor skills
bite the baby the w eek before. (D) Im aginary p lay
(E) Potty training p rogress
46. Which of the follow ing statem ents w ou ld be
the m ost approp riate response to this m other? 49. Which of the follow ing qu alities w ou ld be
(A) If you sim p ly ignore you r d au ghters m ost associated w ith a m ore favorable p rog-
behavior, it w ill p ass. nosis in this child ?
(B) It is u nd erstand able that you r d au ghter (A) Easily toilet trained
is angry and exp eriences jealou sy w ith (B) Interested in m echanical toys
the new baby joining the fam ily. (C) Organized in p lay
(C) The next tim e she tries to bite him , you (D) Recip rocal conversation
should bite her back so she know s how
(E) Reciting songs and p oem s from m em ory
it feels.
(D) Tell you r d au ghter she is being a very Questions 50 and 51
bad girl and you w ont love her if she
bites the baby. A 17-year-old girl com es to you r office for a rou tine
(E) Tell you r d au ghter that she need s to visit. She states that she feels fine and offers no com -
love the baby and be a w ond erfu l big p laints. On p hysical exam ination, you find that her
sister. w eight is 92 lb and her height is 65 in. One year ear-
lier, her w eight w as 126 lb, and height 65 in. After
47. Which of the follow ing tasks w ou ld this girl fu rther d iscu ssion, you learn that she is terrified of
be able to p erform at her cu rrent age? gaining w eight and believes that she is fat and need s
to lose m ore w eight. She ad d itionally reports that
(A) Acknow led ge her angry and she has not m enstru ated in the past 6 m onths.
com petitive feelings tow ard her sibling.
(B) Be able to state her age and gend er. 50. Which of the follow ing laboratory abnorm a-
(C) Cou nt to 50. lities are you m ost likely to find in this p atient?
(D) Recognize that w ater p ou red from one (A) H yp ercholesterolem ia
glass into another of a d ifferent size is
(B) H yp erkalem ia
the sam e volu m e.
(C) H yp ocarotenem ia
(E) Rid e a bicycle.
(D) Increased thyroid -stim u lating horm one
(TSH )
Questions 48 and 49
(E) Leu kocytosis
A 4-year-old boy is referred to you for evalu ation
becau se he has p oor social related ness. Up on inter- 51. Which of the follow ing com p lications w ould
view , he appears healthy and w ell-kem pt. H e grabs be the m ost likely ind ication for ad m itting this
you r office key off you r d esk. At you r office d oor, p atient to the hosp ital?
he takes the key and locks and u nlocks you r d oor
rep eated ly. Desp ite attem p ts to red irect and d istract (A) Anem ia
him , he rem ains preoccu pied w ith this task. After (B) Arrhythm ia
abou t 10 m inu tes, you attem p t to take the key aw ay (C) Brad ycard ia
from him and he becom es extrem ely u p set, m aking (D) H yp otension
an insistent, p iercing cry. (E) Lanu go
10 1: Child a nd Ad ole s c e nt P syc hia try

Questions 52 and 53 54. Which of the follow ing item s in his history
w ou ld be m ost consistent w ith you r p rovi-
An 8-m onth-old boy is brought to the clinic by his sional d iagnosis?
m other. She com plains that her son has been expe-
riencing scream ing and crying fits w hen she leaves (A) Alw ays social and ou tgoing
him w ith a babysitter. She says that in the p ast he (B) Father w ith schizotyp al p ersonality
d id not object to being left w ith a babysitter and asks d isord er
you w hy he becom es so u p set now , and w hat she (C) Parents getting d ivorced
can d o about it. (D) Recently transferred schools
(E) Recently u sed m ariju ana
52. Which of the follow ing statem ents w ou ld be
the m ost app ropriate response?
55. Which of the follow ing featu res w ou ld ind i-
(A) This behavior is characteristic of cate a poorer p rognosis in this patient?
au tistic child ren.
(A) Acu te onset
(B) It is p ossible that you r son has
(B) Affective sym p tom s
separation anxiety d isord er.
(C) Good p rem orbid ad ju stm ent
(C) This behavior su ggests that you re not
spend ing enou gh tim e w ith you r son. (D) Onset before the age of 13 years
(D) It sou nd s as thou gh you r son is overly (E) Well-d ifferentiated sym p tom s
attached to you .
Questions 56 through 58
(E) This behavior is norm al at you r sons
age and w ill p ass w ith tim e. A 9-year-old girl w ith a fam ily history of bipolar
d isord er is referred to you by her school because of
53. Which of the follow ing d iagnoses w ou ld be d isruptive behavior in class that has been w orsen-
m ost ap prop riate if the boy in the p reviou s ing over the past 3 m onths. H er teachers report that
qu estion (Qu estion 52) w ere an 8-year-old boy her energy level is high, and she is m arked ly more
w ith sim ilar behavior? d istractible. She is also sleeping more poorly and is
(A) Agorap hobia increasingly m ore intrusive into her siblings per-
sonal space. Both attention-d eficit/ hyp eractivity d is-
(B) N orm al behavior
ord er (ADH D) and a m anic episod e are consid ered .
(C) Obsessive-com p u lsive d isord er
(D) Sep aration anxiety d isord er 56. Which of the follow ing sym p tom s w ou ld
(E) Social anxiety (social p hobia) be m ore consistent w ith ADH D rather than
m ania?
Questions 54 and 55
(A) Distractibility
A 10-year-old boy is referred to you d u e to extrem e (B) Im p u lsivity
d ifficulties in school. H e has been held back a grad e (C) Low self-esteem
and is still not p assing his classes. Du ring the cou rse (D) Motoric hyp eractivity
of your evalu ation, you learn that the boy hears
(E) Pressu red sp eech
voices telling him that he is stupid and to leave the
classroom . Afraid to d isobey, he goes to the bath-
57. If this child s school is u nable to m anage her
room frequ ently. H e also has d ifficu lty falling asleep
behavior in her classroom d esp ite ou tp atient
at night becau se the voices keep him aw ake. In ad d i-
treatm ent and m ed ication, w hich of the fol-
tion, you learn that the boy believes others can read
low ing long-term op tions w ou ld be the m ost
his thou ghts. Physical and laboratory exam inations
optim al school placem ent?
are norm al. You su sp ect that the boy m ay be su ffer-
ing from schizop hrenia. (A) H om e schooling
(B) Parochial school
Que s tions : 5263 11

(C) School for child ren w ith learning Questions 61 and 62


d isabilities
A 12-year-old boy w ith Tou rette d isord er com es to
(D) Resid ential treatm ent
you r office for a rou tine visit. Tw o w eeks earlier,
(E) Therap eu tic d ay school you had p rescribed clonid ine for his illness. The
boy rep orts that his tics have su bsid ed slightly since
58. Further history and evaluation over time result starting the clonid ine, bu t he com plains abou t the
in the d iagnosis of bipolar d isord er, most m ed icine.
recent episod e manic. Which of the follow ing
med ications w ou ld be the m ost appropriate 61. Which of the follow ing ad verse effects is this
treatm ent? boy m ost likely exp eriencing?
(A) Bu p rop ion (Wellbu trin) (A) Dry m ou th
(B) Du loxetine (Cym balta) (B) H yp otension
(C) Methylp henid ate (Ritalin) (C) N au sea
(D) Mixed am p hetam ine salts (Ad d erall) (D) Sed ation
(E) Valp roic acid (Dep akote) (E) Trem or

Questions 59 and 60 62. The p arents bring in the boys 7-year-old


An 8-year-old boy is brou ght to you r office by his brother for evalu ation. After fu rther history
m other for evaluation of an u pp er resp iratory infec- is obtained , he is d iagnosed w ith attention-
tion. The m other m entions that her son has started deficit/ hyperactivity d isord er (ADHD). Which
w etting the bed again. In ad d ition, she m entions of the follow ing classes of med ications w ould
that the boys grand m other d ied recently and w on- be the most appropriate choice for the brother?
d ers if this is affecting him . (A) Antip sychotic
(B) Monoam ine oxid ase inhibitor (MAOI)
59. At w hich of the follow ing ages w ou ld a child
(C) Serotonin-sp ecific reu p take inhibitor
norm ally be able to ap preciate that d eath is
(SSRI)
irreversible?
(D) Stim ulant
(A) 2 years (E) Tricyclic antid epressant (TCA)
(B) 3 years
(C) 5 years 63. A 6-year-old boy is brou ght to the em ergency
(D) 7 years d ep artm ent by his m other, w ho reports that
(E) 12 years he w as p laying on som e step s in front of the
hou se w hen he slip p ed and fell. She tells
60. Which of the follow ing d efense m echanism s you that she is concerned that he m ight have
is the boy m ost likely em p loying w hen he is broken his arm . An x-ray of the boys arm
w etting the bed ? show s a fracture of the u lna, as w ell as signs of
several old fractures of varying ages. Which of
(A) Acting ou t the follow ing is the m ost app rop riate cou rse
(B) Denial of action?
(C) Regression (A) Recom m end calcium su pp lem ents and a
(D) Repression m ultivitam in d aily.
(E) Som atization (B) Refer the boy to an orthoped ist for
fu rther evalu ation.
(C) Set the cu rrent broken bone in a cast
and have the boy see his p ed iatrician for
follow -u p care.
12 1: Child a nd Ad ole s c e nt P syc hia try

(D) Tell the boy that you notice that he has (C) N eu rology consu lt
had m u ltip le broken bones and ask him (D) Pu lm onary function tests
how each of these fractures happened . (E) Routine laboratory stu d ies
(E) Tell the m other that you notice that
the boy has had m ultiple broken bones 67. A 5-year-old girl d iagnosed w ith lu pu s is seen
and recom m end that she lim it the boys by her fem ale p ed iatrician for a rou tine visit.
sports activities. After retu rning hom e from the clinic, the girl
asks her friend to p lay d octor. Which of the
64. Th e m oth er of a 6-year-old boy calls an d follow ing d efense m echanism s best d escribes
asks you for ad vice. She says th at h er son this behavior?
still su cks his th u m b, an d sh e is concerned
abou t th is beh avior. Wh ich of th e follow - (A) Displacem ent
in g su ggestion s for h er to d o is th e m ost (B) Dissociation
ap p rop riate? (C) Id entification
(A) Ask the d entist to construct a m outh (D) Rationalization
ap p liance that w ill d eter su cking. (E) Reaction form ation
(B) Coat her sons thum b in hot pepper
sauce. Questions 68 and 69
(C) Give him gu m frequ ently. A 10-year-old girl w ith a history of asthm a is
(D) Ignore the behavior. brou ght to the clinic after a recent increase in her
(E) Im p lem ent a behavioral system to asthm a sym p tom s. Du ring the visit, you learn that
rew ard stop p ing. she is being physically beaten by her m others boy-
friend on a regu lar basis.
Questions 65 and 66
68. Und er w hich of the follow ing circu m stances
A 14-year-old girl presents to her pediatrician com- d oes the law requ ire m and atory rep orting by
plaining that she has been freaking out. The girl a p hysician of su sp ected child abu se?
describes episodes of shaking, gasping for air, and
feeling like she is going to die. The feelings intensify (A) In all cases
for a few minutes and resolve spontaneously. These (B) Only in cases in w hich the child show s
episodes have occurred at various times, in various sit- behavioral m anifestations of abu se.
uations, and the girl is worried that she is going crazy. (C) Only w hen consent of a p arent or
A complete history and physical examination does not gu ard ian is obtained .
reveal any further relevant symptoms or signs. (D) Only w hen the p hysician believes it is in
a child s best interest.
65. Which of the follow ing is the m ost app rop riate (E) Only w hen the p hysician has exam ined
p harm acologic treatm ent? all child ren in the fam ily.
(A) Arip ip razole
(B) Carbam azep ine 69. Which of the follow ing m anifestations w ou ld
be the m ost likely ou tcom e of the abu se?
(C) Du loxetine
(D) Sertraline (A) Aggression
(E) Valp roic acid (B) Dissociative d isord er
(C) Generalized anxiety d isord er
66. Prior to prescribing m ed ications, w hich of the (D) Major d ep ressive d isord er
follow ing shou ld the p ed iatrician ord er next? (E) Post-trau m atic stress d isord er
(A) Electrocard iogram (ECG)
(B) Electroencep halogram (EEG)
Que s tions : 6475 13

70. A 4-year-old boy is referred to you becau se he 73. An 8-year-old boy is brou ght in by his m other
w ill not sp eak in p reschool. Over the cou rse of w ho com p lains that she cannot get her son to
abou t 2 m onths, he grad u ally stop ped talking. listen to her. She is fru strated becau se he fre-
H is m other rep orts that he initially objected quently ignores her requ ests and instru ctions.
to going to preschool, but now no longer Consid eration is given tow ard the d iagnosis of
com plains. She states that at tim es her son is oppositional d efiant d isord er (ODD). Which
qu iet and stays in his room , bu t that she has of the follow ing featu res w ou ld best sup port
not otherw ise noticed a significant change in the d iagnosis in this child ?
his sp eech or behavior. Which of the follow ing
(A) Aggression to p eop le
is the m ost likely d iagnosis?
(B) Dep ressive sym p tom s
(A) Major d ep ressive d isord er (MDD) (C) Disobed ience tow ard teachers
(B) Persistent d ep ressive d isord er (D) Lack of p articip ation in tasks requ iring
(d ysthym ia) attention
(C) Selective m u tism (E) Violation of ru les
(D) Sep aration anxiety d isord er
(E) Social anxiety d isord er (social p hobia) 74. A 13-year-old girl is seen by her p sychiatrist
1 year after an au tom obile accid ent. She d em -
71. A 2-year-old boy is referred to you for evalu - onstrates intact langu age ability and com plex
ation d u e to the su sp icion that the child is m otor skills. She has no id entifiable abnor-
the victim of abu se second ary to factitiou s m alities in the perception of stim uli, bu t she
d isord er im posed by another. Which of the has lost the ability to read since the accid ent.
follow ing fam ily m em bers is the m ost likely Which of the follow ing d eficits is she m ost
p erp etrator fabricating the illness? likely d em onstrating?
(A) Father (A) Agnosia
(B) Brother (B) Alexia
(C) Mother (C) Anom ia
(D) Sister (D) Ap hasia
(E) Uncle (E) Ap raxia

72. A fru strated m other brings her 14-year-old son 75. A 10-year-old girl w ithou t significant m ed ical
to a child p sychiatrist after he is expelled from history is brou ght by her father to the p ed ia-
three high schools in 1 year. She reports the trician for evaluation. Over the past school
boy has tried tw ice to set his school on fire, has year, she has been having increasing d ifficu l-
slashed school bus tires, and has broken into ties going to sleep. Althou gh she has alw ays
the principals office to steal athletic trophies. had bed tim e ritu als, they have extend ed in
In ad d ition, he has been su sp end ed nu m er- com plexity and length. Most of her tim e in
ou s tim es for getting into fights w ith other the evening is now spent going arou nd the
stu d ents. She shu d d ers and tearfully relates hou se nu m erou s tim es, locking and u nlock-
that she recently caught him singeing one of ing the d oors and w ind ow s. While she know s
the fam ily cats w ith a cigarette butt. Which the chances of a bu rglary are slim , she is
of the follow ing p ersonality d isord ers is this extrem ely anxiou s abou t her safety, and she
boy m ost at risk of d evelop ing in the fu tu re? cant stop the u rges to perform these behav-
iors. As a resu lt, she only obtains 5 hou rs of
(A) Antisocial
sleep, and she has been falling asleep in class
(B) Bord erline w ith d im inishing grad es. Which of the follow -
(C) H istrionic ing therap eu tic interventions is consid ered the
(D) Obsessive-com p u lsive first-line treatm ent for this d isord er?
(E) Schizotyp al
14 1: Child a nd Ad ole s c e nt P sychia try

(A) Cognitive-behavioral therapy (CBT) interp reting social cu es and u tilizes less com -
(B) Fam ily therapy p lex sp oken langu age, is able to retain basic
(C) Grou p therap y langu age for social com m u nication. H e can
care for his basic personal need s after exten-
(D) Short-term psychod ynam ic therapy
sive teaching, bu t w ou ld eventu ally requ ire
(E) Supp ortive therapy consid erable su pport from co-w orkers and
sup ervisors to m anage responsibilities in the
D IRECTION S (Questions 76 through 93): The fol- fu tu re.
low ing group of numbered items are preceded by
a list of lettered options. For each question, select
the one lettered option that is most closely asso- Questions 80 through 83
ciated w ith it. Each lettered option may be used Match the m ost likely d isord er w ith the appropriate
once, multiple times, or not at all. p atient.

Questions 76 through 79 (A) Anorexia nervosa


(B) Au tism sp ectru m d isord er
Match the severity level of intellectual d isability w ith
(C) Bu lim ia nervosa
the patients ad aptive functioning.
(D) Binge eating d isord er
(A) N o intellectu al d isability (E) Illness anxiety d isord er
(B) Mild intellectu al d isability (F) Interm ittent exp losive d isord er
(C) Mod erate intellectu al d isability (G) Obsessive-com p u lsive d isord er
(D) Severe intellectu al d isability (H ) Panic d isord er
(E) Profou nd intellectu al d isability (I) Pica
(J) Pyrom ania
76. A 14-year-old boy w ho has lim ited attainm ent
of conceptual skills, has sp oken language lim - (K) Tou rette d isord er
ited to single w ord s or p hrases, requ ires su p - (L) Trichotillom ania
p ort for all activities of d aily living, requ ires
sup ervision at all tim es, and w ho cannot m ake 80. A 14-year-old girl w ith ep isod es of palpita-
resp onsible d ecisions regard ing the w ell-being tions, chest p ain, shortness of breath, and
of others. d iaphoresis w ho has a norm al physical and
laboratory exam inations.
77. An 8-year-old girl w ho has no obviou s d eficits
in learning, is equ ally m atu re as her p eers, is 81. A 10-year-old overw eight girl feels a loss of
able to perform d aily living tasks w ithout su p - control w hen she qu ickly consum es large
p ort, and has sou nd ju d gm ent. am ou nts of food , d enies heavy exercise, vom -
iting or laxative u se afterw ard s, and feels
significant guilt and sham e regard ing her
78. An 11-year-old boy w ho has d ifficu lties in
behavior.
acad em ic skills of w riting, read ing, and m ath,
is im m atu re in social situ ations, and w ho
has d ifficulties w ith regu lating em otions and 82. An 8-year-old boy w ith erythem atou s,
behavior in age-ap p rop riate fashion. While he chapped hand s, and an otherw ise norm al
is able to p rovid e his ow n p ersonal care, he p hysical and laboratory exam ination.
requ ires su p p ort in m ore com p lex d aily living
tasks, ju d gm ent, and organization. 83. A 13-year-old girl w ith a bald p atch on the
back of her head and an otherw ise norm al
79. A 17-year-old boy w hose concep tu al skills p hysical and laboratory exam ination.
consistently lag behind those of his peers,
and , w hile he has d ifficu lties p erceiving or
Que s tions : 7694 15

Questions 84 through 87 90. Establishing tru st in the w orld throu gh


resp onsiveness and em p athy of a caregiver.
Match the m ost likely d isord er w ith the appropriate
patient.
91. Preoccup ation w ith su perheroes w ho rep re-
(A) Attention-d eficit/ hyp eractivity d isord er sent id ealized caregivers as a resu lt of con-
(ADH D) flicted feelings tow ard caregivers.
(B) Au tism sp ectru m d isord er
(C) Generalized anxiety d isord er 92. The d evelop m ent of the ability to think abou t
(D) Langu age d isord er and m anipu late id eas abstractly.
(E) Selective m u tism
(F) Social anxiety d isord er 93. The d evelop m ent of the ability to ap p ly rea-
(G) Social (p ragm atic) com m u nication soning so that the child is not lim ited only by
d isord er p ercep tions.

84. A 6-year-old girl whose parents are going DIRECTIONS (Questions 94 through 99): For each of
through a divorce will not speak while at school. the multiple-choice questions in this section, select
the lettered answer that is the one best response in
85. A 9-year-old boy w ho frequ ently blu rts ou t each case.
com m ents in class w ithou t w aiting his turn to
be called on. 94. A 16-year-old girl is brou ght to you by her
m other because of d ropping grad es, apathy,
and p oor m otivation. You learn that she has
86. A 7-year-old boy w ho p erform s w ell in school
recently started sm oking m ariju ana on a
althou gh seem s to talk as if reciting a m ono-
regu lar basis. Which of the follow ing p atterns
logu e rather than interacting in conversation
w ou ld m ost su p p ort the d iagnosis of cannabis
and generally avoid s other child ren, bu t d oes
u se d isord er, severe?
not d isplay rep etitive or restricted interests.
(A) Over 12 m onths, u sing m ore than
87. An 8-year-old boy w ho is having d ifficu lty in intend ed and being u nsu ccessfu l at
school and avoid s interactions w ith his class- stopp ing, w ith a loss of her sum m er job
m ates and others for fear of em barrassm ent. and legal consequ ences from her u se.
(B) Over 12 m onths, having cravings to
u se, spend ing significant tim e obtaining
Questions 88 through 93
m arijuana, and m issing school in ord er
Match the age range w ith the corresp ond ing d evel- to sm oke.
opm ental m ilestone. (C) Over 12 m onths, continu ing to use
d esp ite recu rrent argu m ents, being
(A) Infant (018 m onths)
high w hile d riving, u sing d espite
(B) Tod d ler (1836 m onths) know ing it is affecting her m otivation,
(C) Preschool age (36 years) need ing increased am ounts to get the
(D) School age (712 years) sam e high, and having ongoing
(E) Ad olescence (1317 years) cravings w hen not u sing.
(D) Over 18 m onths, sm oking by herself bu t
88. Focu s on follow ing the ru les. need ing to sm oke m ore to achieve the
sam e effect.
89. Establishing self as au tonom ou s, sep arate (E) Over 18 m onths, sm oking m ost d ays
from caregiver, by p racticing leaving and p er w eek, m ultip le tim es p er d ay, and
retu rning to the caregiver. d eveloping w ithd raw al sym ptom s once
the m ariju ana is stopped .
16 1: Child a nd Ad ole s c e nt P sychia try

95. As the school p sychologist, you are asked to (C) N orm al variant in acad em ic attainm ent
see a fou rth grad er w ho has been consistently (D) Post-trau m atic stress d isord er (PTSD)
acting out in class. H e often lies abou t things (E) Vision d eficit
he has d one in class, such as trying to cheat
on tests, and he physically bu llies you nger 98. A 7-year-old girl is brou ght to the p ed iatri-
child ren. You learn that at hom e he practices cian d u e to w eight loss. She has been grad u -
shooting his BB gu n at squirrels as w ell as ally refu sing food s over the p ast several w eeks
at the fam ily d og. H e exp resses no concern to the point she is now barely even d rinking
for these creatu res nor rem orse at his behav- su pplem ental shakes. This has resu lted in sig-
ior. Which of the follow ing is the m ost likely nificant w eight loss and fatigu e. The girls tells
d iagnosis? you she is aw are she is losing w eight, that she
(A) Au tism sp ectru m d isord er w ou ld like to fit into the ou tfits she has, bu t
(B) Bip olar d isord er that she is afraid of vom iting if she eats the
w rong thing. Physical exam ination reveals a
(C) Child hood onset schizop hrenia
child in the 5th p ercentile of w eight, w hile her
(D) Cond u ct d isord er baseline had p reviou sly been in the 30th per-
(E) Op p ositional d efiant d isord er centile, and basic laboratory stu d ies reveal a
m ild m icrocytic anem ia. Which of the follow -
96. An 8-year-old boy is brou ght to the p ed iatri- ing is the m ost likely d iagnosis in this case?
cian for concerns that he has significant tan-
tru m s. The tantru m s occur at least three tim es (A) Anorexia nervosa
p er w eek (bu t frequ ently m ore) and inclu d e (B) Avoid ant/ restrictive food intake
p hysical aggression tow ard inanim ate objects d isord er (ARFID)
to the extent that he has d estroyed property. (C) Binge eating d isord er
H is parents note that the fam ily w alks on egg- (D) Bu lim ia nervosa
shells to p revent setting him off, and they are (E) Sp ecific p hobia
starting to feel held hostage by his chroni-
cally irate m ood . They believe this change in 99. A 7-year-old boy is brou ght to the p ed iatri-
behavior started a year ago, bu t has m arked ly cian becau se, for the p ast 6 m onths, he has
w orsened in the p ast several w eeks. Which of been telling his m other that he is a girl. Ini-
the follow ing is the m ost likely d iagnosis? tially his parents felt this w as a p hase, bu t
(A) Bip olar d isord er the patient has becom e increasingly d istressed
(B) Disru p tive m ood d ysregu lation d isord er and m ore insistent that he hop es his breasts
(DMDD) w ill grow d u ring p u berty. H e has been p u t-
ting on his old er sister s clothing, and prefers
(C) Generalized anxiety d isord er
p laying w ith her d olls rather than engaging in
(D) Major d ep ressive d isord er (MDD) rou ghhou sing w ith his p eers. H e has rep eat-
(E) Op p ositional d efiant d isord er (ODD) ed ly stated he d islikes his p enis and w ishes he
never had one. Physical exam ination reveals
97. A healthy 16-year-old stu d ent, w ho started norm al m ale genitalia of the app rop riate Tan-
school in the United States after fleeing a ner stage. Based on this inform ation, w hich of
w ar-engulfed region 1 year ago, is struggling the follow ing is the m ost likely d iagnosis?
to keep up in her classes, m ore so w ith Eng-
lish than m ath. Which of the follow ing best (A) Bod y d ysm orp hic d isord er
explains w hy her d ifficu lty is not d u e to a (B) Delu sional d isord er
learning d isability? (C) Gend er d ysp horia in child ren
(A) H earing d eficit (D) N onconform ity to gend er roles
(B) N eu rocognitive d isord er d u e to (E) Transvestic d isord er
traum atic brain injury
Ans we rs a nd Expla na tions

1. (C) Su icid e is a consid erable risk in d ep ressed be ad m itted to a hosp ital, since she d oes not
ad olescents, and shou ld be sp ecifically ap p ear at im m inent risk of self-harm . Cer-
ad d ressed d u ring an interview w ith a p atient tainly, w aiting m ore than that w ithou t any
w ho ap p ears d ep ressed or agitated , or has a intervention w ou ld be inad equ ate and inap -
history of a su icid e attem pt. The ad olescent p ropriate. She w ou ld likely benefit from m ed -
su icid e rate has increased substantially d u r- ication m anagem ent, so a p sychiatrist w ou ld
ing the p ast few d ecad es. Male gend er, a p rior be p referable to a social w orker in this situ a-
su icid e attem pt, history of psychiatric illness, tion, bu t therapy w ou ld also be appropriate
fam ily history, and su bstance u se are all risk concurrent treatm ent.
factors for a com p leted su icid e. A history of
a p rior su icid e attem p t is the largest risk fac- 3. (D ) The Diagnostic and Statistical M anual of
tor for suicid e for both m ales and fem ales in M ental Disorders, Fifth Edition (DSM -5) recog-
all age grou ps, and the m ajority of those w ho nizes N REM sleep arou sal d isord ers of tw o
com plete su icid e have attem pted su icid e in types: sleep w alking typ e and sleep terror
the p ast. Greater than 90% of you th w ho com - type. Both are characterized by recu rrent epi-
m it suicid e have com orbid psychiatric illness, sod es of incom plete aw akenings from sleep,
frequ ently m ajor d ep ression. Althou gh cu t- u su ally in the first third of the m ajor sleep
ting behavior is concerning in ad olescents, it ep isod e, w ith am nesia for the ep isod es, no or
is not necessarily associated w ith the intent little recollection of d ream im agery, and resu l-
to kill oneself. More girls than boys d em on- tant im pairm ent or d istress. Sleep terror type
strate su icid al behavior and attem pts, bu t at involves rep eated ep isod es of su d d en aw ak-
least three tim es m ore teenage boys com p lete ening from sleep accom p anied by p anic sym p -
su icid e than d o teenage girls. This is becau se tom s that begin w ith a scream ; it is associated
boys m ore frequ ently u se gu ns and other w ith u nresp onsiveness to com fort or attem p ts
violent m ethod s to attem p t suicid e. to aw aken the child . Attacks typically last just
a few m inu tes and are often m ore d istressing
2. (C) Given the ad olescents history of d ep res- to the caregiver, as the patient d oes not recall
sion and risk factors of su bstance use and them in the m orning. N REM sleep arou sal
fam ily history, it is im p ortant to initiate d isord er, sleep -w alking typ e, involves the
ap prop riate treatm ent for her qu ickly. Given child sitting up or leaving the bed but is not
the severity of her d epression in the p ast, and accom p anied by terror or au tonom ic arou sal.
d ecline now , it is reasonable to initiate anti- N ightm are d isord ers occu r in the latter third
d ep ressant treatm ent at this visit w ith close of the night and d u ring REM sleep , in con-
follow -u p . While it w ou ld be op tim al for a trast to sleep terrors. When aw akened , the
p sychiatrist to evalu ate her as soon as possible ind ivid u al qu ickly becom es oriented , u nlike
and a referral is ap propriate, the w ait in get- other parasom nias w here the ind ivid ual is
ting to a specialist (psychiatrist) shou ld not d isoriented , confu sed , and d ifficu lt to arou se.
d elay care. She d oes not necessarily need to Interm ittent exp losive d isord er d oes not occu r

17
18 1: Child a nd Ad ole s c e nt P sychia try

d uring sleep and involves outbursts of anger acad em ic fu nctioning. There is no ind ication
and som etim es violent behavior. N arcolep sy that this child has d ifficulties w ith speech
is characterized by the triad of sleep attacks, p rod u ction (i.e., he can clearly articu late pho-
cataplexy (sud d en loss of m uscle tone), and nem es of sp eech), so d iagnosing sp eech sou nd
hyp nopom pic/ hypnagogic hallu cinations or d isord er w ou ld be inappropriate. As there are
sleep paralysis, as w ell as a hypocretin d efi- no d istu rbances in the flu ency and p attern of
ciency. speech, a d iagnosis of child hood -onset flu -
ency d isord er (stu ttering) is inap p rop riate.
4. (C) Ep isod es of sleep terror, as w ell as of sleep - Social (pragm atic) com m u nication d isord er is
w alking, occur d u ring d eep sleep (stages 34). characterized by persistent d ifficu lties in the
N ightm are d isord er occurs d uring REM sleep. social use of verbal and nonverbal com m u-
nication, for instance im p airm ents in greet-
5. (E) This patient is su ffering from a m ajor ings, inability to change com m u nication to
d ep ressive d isord er (MDD), single episod e. A a given context, d ifficu lties follow ing ru les
selective serotonin reu ptake inhibitor (SSRI) for conversation or story-telling, or d ifficu l-
such as sertraline is a first-line agent for MDD ties in u nd erstand ing w hat is inferred but not
in child ren and ad olescents. Tricyclic antid e- overtly stated . This child is able to appropri-
p ressants (TCAs) su ch as im ip ram ine cau se ately m od u late verbal com m u nication, bu t his
m ore ad verse effects than SSRIs typically d o, d ifficu lty is in the expression of verbal cues.
and in overd ose are m u ch m ore likely to be There m ay be a com orbid learning d isord er,
lethal. Mood stabilizers su ch as carbam aze- but at this time, there is insufficient information
p ine and lithiu m are u sed for bipolar d isor- to m ake that d iagnosis.
der and as adjuncts to the treatment of MDD
refractory to antidepressant medications alone. 8. (C) Sp eech d elay refers to exp ressive langu age
Antipsychotics such as olanzapine are usually d evelop m ent and nu m ber of w ord s spoken.
reserved for use as adjuncts when psychosis The average nu m ber of w ord s a 2-year-old
develops or in bipolar disorder. sp eaks is 200 w ord s. Therefore, the inability
to sp eak 200 w ord s by age 3 w ould constitu te
6. (D ) In the DSM -5, m ajor d epressive d isord er a speech d elay.
requ ires a d epressed m ood for at least 2 w eeks
for ad ults to receive the d iagnosis, but in the 9. (E) It is m ost likely that the boy u sed illicit
case of child ren and ad olescents, an irritable substances d u ring his trip that caused him
mood may substitute for having a d epressed to experience the acute psychotic sym ptom s.
mood . The required symptoms that involve While it is im p ortant to ru le ou t other m ed i-
sleep, appetite, anhed onia, concentration, and cal cau ses, su bstance use w ould be the m ost
psychomotor functioning are the same for likely. Seizu re d isord ers, head inju ries, d ia-
ad ults and ad olescents w hen diagnosing major betes, and thyroid d isease often w ou ld be
d epressive d isord er. accom p anied by other sym p tom s in ad d ition
to the new onset psychosis.
7. (B) This boy m ost likely su ffers from langu age
d isord er. Langu age d isord er is characterized 10. (B) While a su bstance-ind u ced state rem ains
by p ersistent d ifficu lties in the acqu isition and p ossible d u e to the large nu m ber of su bstances
u se of langu age d u e to d eficits in com prehen- that cannot be confirm ed by u rine toxicology
sion (receptive) or prod u ction (expressive). screens, it is im portant to consid er that this
Typ ical sym p tom s inclu d e having a m ark- instance of p sychosis is a m anifestation of an
ed ly lim ited vocabu lary, m aking errors in u nd erlying m ood d isord er, m ost likely bipolar
tense, and having d ifficulty recalling w ord s d isord er. In ad d ition to an elevated or irritable
or prod u cing sentences w ith d evelopm entally m ood , the d iagnosis of bip olar d isord er neces-
ap propriate length or com p lexity. In ad d i- sitates that ind ivid uals d isplay three of the
tion, the boys d ifficulties are interfering w ith follow ing (four if irritable m ood ): grand iosity,
Answe rs : 415 19

d ecreased need for sleep, increase in talkative- situ ations, w ith exp osu re causing intense
ness, objective or su bjective flight of id eas, anxiety. Finally, a d iagnosis of specific phobia
d istractibility, increase in goal-d irected behav- w ould require the display of marked and per-
iors, and engagem ent in p otentially high-risk sistent fear cued by the presence of anticipation
behaviors. Mania in bip olar d isord er can often of a specific object or situation.
p resent concu rrently w ith psychotic sym p -
tom s includ ing hallu cinations, d elu sions, and 12. (B) Child ren w ith p arents w ho have a history
d isorganized thinking. Anorexia nervosa (an of an anxiety d isord er are at increased risk for
eating d isord er) and generalized anxiety d is- d evelop ing an anxiety d isord er them selves.
ord ers d o not p resent w ith psychotic sym p- Other risk factors for d eveloping a child -
tom s. Ind ivid u als w ith bord erline personality hood anxiety d isord er inclu d e p arents w ho
d isord er m ay have p sychotic-like sym ptom s, have an anxiou s, overly controlling, or reject-
esp ecially w hen they are regressed , bu t they ing style, an insecu re attachm ent w ith ones
are m u ch m ore likely to p resent w ith d ep res- p rim ary caregiver, and an inhibited and shy
sion, suicid al thinking, and su bstance u se. tem peram ent.
While p sychotic featu res can accom p any
m ajor d ep ressive d isord er, the d ep ressive 13. (B) These sym p tom s are consistent w ith a
sym ptom s occu r first, and psychotic sym p- m anic ep isod e of bip olar d isord er. Attention-
tom s are m ore acu te and frequ ent in bipolar d eficit/ hyperactivity d isord er is associated
d isord er w ith hyp eractivity and im p u lsivity bu t not
grand iosity and d elu sional thinking. Major
11. (C) This boys behavior and sym p tom s are d ep ression m ay have psychotic sym ptom s
m ost consistent w ith sep aration anxiety d isor- if severe, bu t it w ou ld not p resent w ith an
d er, characterized by d evelop m entally inap- inflated m ood or increased energy. Panic
p rop riate and excessive anxiety concerning d isord er w ou ld present w ith recu rrent panic
separation from the hom e or from those to attacks and significant anxiety. Although
w hom the ind ivid u al is attached . Consistent schizophrenia can present w ith d elusional
w ith this d iagnosis, the boy d oes not like to sym ptom s, there are often other sym ptom s
go to school, com es hom e im m ed iately after such as grossly d isorganized speech and / or
school, sleeps in his parents bed at night, behavior, or flat affect; in ad d ition, this p atient
and has repeated physical sym ptom s w hen has significant m ood sym ptom s.
at school. The boy is not suffering from PTSD
becau se there is no evid ence of a trau m atic 14. (C) Cocaine is a stimu lant and can prod u ce
event that is p ersistently re-exp erienced or both manic and psychotic sym ptoms. Alco-
has cau sed sym ptom s of increased arou sal hol, cannabis, heroin, and PCP ingestion can
and avoid ance of associated stim u li. For a ind uce a psychotic state includ ing hallucina-
d iagnosis of reactive attachment d isord er, tions and paranoia, bu t it w ou ld not classically
the boy w ould need to have su ffered m ark- be accompanied by manic symptom s.
ed ly d istu rbed social related ness, in m ost
contexts, beginning before the age of 5 years, 15. (E) This boy likely has ODD. ODD is an exter-
and display a pattern of inhibited , w ithd raw n nalizing behavior d isord er that involves a p at-
behavior toward caregivers. A related disorder, tern of hostile and d efiant behavior. Child ren
d isinhibited social engagem ent d isord er, also w ith ODD are often angry, argu m entative,
stem s from significant neglect or d eprivation and easily annoyed by others. In ord er to con-
and results in child ren d isp laying a m ark- firm the d iagnosis, you w ou ld need to gather
ed ly overfam iliar ap p roach and com fort w ith ad d itional inform ation regard ing the length
strangers. The boys sym p tom s are not con- of tim e the behavior has been present (at least
sistent w ith social anxiety d isord er (social 6 m onths is requ ired ), as w ell as rule ou t a
p hobia) becau se he d oes not have a m arked m ood , p sychotic, or su bstance u se d isord er.
and p ersistent fear of social or p erform ance Althou gh anxiety d isord ers can p resent w ith
20 1: Child a nd Ad ole s c e nt P syc hia try

tension, irritability, and even noncom pliance behaviors su ch as ind u ced vom iting, laxative,
w hen a child is p laced in a new situ ation (su ch or d iu retic use. A hypokalem ic-hypochlore-
as a d octors office), this child d oes not ap p ear m ic alkalosis (d u e to vom iting) is a p ossible
to have anxiety in places w here escape m ay seriou s find ing that can contribute to a card iac
be d ifficu lt in the event of a p anic attack (su ch arrhythm ia. Anem ia (as op p osed to increased
as in agorap hobia), nor excessive w orry abou t iron) can be seen, as w ell as d ecreased p rotein.
a nu m ber of events (su ch as in generalized The red u ced chlorid e can lead to elevated
anxiety d isord er). While the p atients p resent- sod ium d u e to sod ium resorption. Becau se
ing sym p tom s are not p rim arily involving of these changes, it is im portant to m onitor
inattention, d istractibility, or hyp eractivity bu lim ic p atients for electrolyte and acid base
consistent w ith ADH D, there is consid erable im balances.
com orbid ity betw een ADH D and ODD. H is
lack of violence or seriou s violation of ru les 18. (A) Bulimia nervosa is most effectively treated
is not consistent w ith cond u ct d isord er; how - w ith cognitive-behavioral therapy. Family ther-
ever, ODD (esp ecially if u ntreated ) m ay lead apy, group therapy, and longer-term, insight-
to cond uct d isord er over tim e. oriented therapies (such as psychoanalysis and
psychod ynamic psychotherapy) may be used
16. (A) This patient appears to be suffering from as adjuncts to cognitive-behavioral therapy,
bulim ia nervous, an eating d isord er character- but they have not been demonstrated to be as
ized by episod es of bingeing and compensa- effective in changing the behaviors associated
tory behaviors (e.g., purging). Signs of bulim ia w ith bulimia nervosa.
nervosa includ e erosion of tooth enamel
caused by acid ic vomitu s as w ell as abrasions 19. (D ) When engaging a child in a clinical set-
on the d orsu m of the hand d u e to scraping by ting, it is im portant to choose w ord s carefu lly
the u pper teeth as the ind ivid ual pushes the in ord er to establish an op en, tru sting connec-
hand to the back of the throat to ind u ce vom it- tion, especially w ith a troubled and resistant
ing. As it is not a secret that the abrasions are child . It is p referable to initiate the interview
there, but the patient has not mentioned them, w ith m ore op en-end ed rather than closed -
a d irect bu t open-end ed statement, such as, end ed qu estions. The qu estions shou ld op ti-
Tell me about the scratches on your hand , m ally reflect em p athy for the child and their
is most likely to be helpful in this situation. situ ation w ithout ap pearing too sentim ental
Usually, a patient appreciates a d irect ques- or jud gm ental. Also, it is im portant to avoid
tion rather than w ond ering if you ignored or assu m ptions abou t the p atients feelings w ith-
d id not notice something obvious. Questions ou t checking w ith the patient first. The state-
su ch as, Do you scratch you rself? or, Do m ent, You su re w ere lu cky the car sw erved
you have a cat? are not open-end ed ; they at the last m inute assu m es the boy w anted
require yes or no answ ers and are unlikely to avoid being hit by the car, and also is a
to yield new information. Are you trying to com m ent that d oes not lead to any elabora-
hurt you rself? or H ow d id the scratches tion from the p atient. Statem ents (B) and (C)
happen? m ay sound accu satory; patients are are both ju d gm ental and m ore likely to shut
more likely to offer information if you seem d ow n conversation than facilitate it. Also, ask-
nonjud gmental. If this girl d oes not volu nteer ing a child if they are confu sed is a close-end ed
inform ation that confirm s to you that she is qu estion that com es across as cond escend ing.
not ind ucing vom iting, you may need to u se a
more d irect bu t reassuring statement su ch as, 20. (D ) Psychom otor agitation is m ore com m only
Som etimes I see girls w ho make them selves seen in child ren w ith MDD com pared to ad o-
throw up. H ave you ever d one that? lescents w ith MDD. Child ren w ith MDD m ay
ap p ear m ore anxiou s and irritable than sad
17. (D ) Med ical com p lications can arise for and d ep ressed . H yp ersom nia, hop elessness,
bu lim ic p atients w ho engage in com p ensatory w eight change, and illicit d ru g u se are m ore
Answe rs : 1627 21

com m only seen in ad olescents w ith MDD a separate criterion. While ASD is more com-
com pared to child ren w ith MDD. mon in males, it d oes exist in fem ales. ID m ay
be comorbid w ith ASD, but m ust be separately
21. (B) Suffocation (e.g., hanging) is the most com- assessed . ID is characterized by intellectu al and
mon m ethod for attem pting suicid e in ind ivid - functioning d eficits in conceptual, social, and
uals u nd er 15 years of age. Su bstance ingestion practical d omains. Rett synd rome is seen only
is another comm on m ethod . While firearms in fem ales and is characterized by normal pre-
are used less frequently, they are more often natal and perinatal d evelop ment, normal head
lethal. Other m ethod s of suicid e frequently circum ference at birth, and normal p sychom o-
attempted by child ren inclu d e stabbing, cut- tor d evelopment throu gh the first 5 m onths
ting, jum ping from build ings, ru nning in front of life. Betw een the ages of 5 and 48 months,
of vehicles, and gas inhalation. Some su icid e there is d eceleration of head grow th, loss of
attempts m ay be m istaken for accid ents, so hand skills w ith d evelop ment of stereotyped
it is im portant to d irectly ask child ren if they hand m ovem ents su ch as hand w ringing,
intend ed to hurt or kill themselves. loss of social interaction (w hich m ay improve
later), appearance of poorly coord inated gait
22. (A) H ead aches and nau sea are com m on or tru nk m ovem ents, and severely im paired
ad verse effects associated w ith the synthetic exp ressive and recep tive language d evelop-
antid iuretic horm one, Dd avp. H yp otension, ment w ith severe psychomotor retard ation.
liver toxicity, sed ation, and trem or are not Consequ ently, child ren w ith Rett synd rom e,
typically associated w ith d esm opressin. d uring the period of regression, m ay appear
to meet criteria for ASD. In this case, there is
23. (B) Approxim ately 10% to 15% of child ren no evid ence of the typical physical examina-
w ith anxiety d isord ers d evelop MDD. tion abnormalities. Of note, Rett synd rome no
longer has its ow n d iagnostic criteria in the
24. (C) The m ost com m on ad verse effects of flu ox- DSM -5 bu t rem ains d escribed in the d ifferen-
etine inclu d e gastrointestinal sym p tom s (e.g., tial d iagnoses for several d isord ers. Selective
nausea, loose stools), insom nia, agitation, and m u tism is characterized by a failu re to sp eak,
head aches. In general, hyp otension, liver tox- d esp ite norm al ability to d o so, in social situ -
icity, sed ation, and w eight gain are not sid e ations w here speaking is exp ected , su ch as at
effects associated w ith flu oxetine. school; ou tsid e of this social situation, they
d isplay norm al speech and interactions. Chil-
25. (A) This girls history and presentation are d ren w ith social (pragm atic) com m u nication
consistent w ith ASD. ASD is characterized by d isord er d isplay persistent d ifficu lties in how
d eficits in social comm unication and social they use verbal and nonverbal com m unica-
interaction, as w ell as restricted , repetitive pat- tion in social settings, bu t lack the restricted /
terns of behavior, interests, or activities (e.g., rep etitive p atterns of behaviors and interests
stereotyped or repetitive movements, inflex- that are present in ASD.
ible rigid ity to routines, fixated or restricted
interests, sensory input abnorm alities). Chil- 26. (C) ASD is d iagnosed four times more in boys
d ren w ith ASD have d ifficulties w ith social- than girls, and w hen it is present in girls, intel-
emotional reciprocity and have d ifficulty lectual d isability is often comorbid . The pres-
engaging in prosocial behavior. Fu rther, they ence of functional language by age 5 is a positive
often have imp aired eye contact and have d if- prognostic sign, and it also allow s child ren to
ficulty und erstand ing nonverbal com munica- engage more effectively in social skills training.
tion or cues. They also have abnormal social The remaining listed factors negatively affect
attachments, su ch as app earing to have no the prognosis of patients w ith ASD.
interest in peers (as in this case). The DSM -5 sig-
nificantly revised the category of ASD, so that 27. (A) This patients presentation is consistent
verbal language abilities no longer represent w ith conduct disord er (CD). Factors associated
22 1: Child a nd Ad ole s c e nt P syc hia try

w ith an increased risk of a child d evelop ing restless an d ju m p y beh avior is consistent
CD inclu d e a low IQ, low school achievem ent, w ith m isinterp retation of m otor tics. Tou rette
p oor parental su p ervision, p u nitive or erratic d isord er m ost com m only d evelop s in grad e
p arental d iscip line, child hood physical abu se, school-aged boys, an d the involu ntary tics
p arental conflict, the child s biological father m ay be m isinterp reted as p u rp osefu lly d is-
being absent, antisocial p arents, and having a ru p tive behavior. It is not u n u su al for these
large fam ily. child ren to have d ifficu lty w ith social and
p eer interactions. Of note, there is frequ ent
28. (A) Child ren and ad olescents w ho are d iag- com orbid ity w ith ADH D. Th is boy d oes
nosed w ith cond u ct d isord er are at increased not su ffer from con d u ct d isord er or op p osi-
risk for an tisocial p ersonality d isord er as tion al d efian t d isord er becau se he is p olite
ad u lts. Antisocial p erson ality d isord er is and d oes not d isp lay hostile, d estru ctive, or
not d iagnosed u ntil after th e age of 18, and angry behaviors. Th e ou tbu rsts are n ot typ i-
on e of the criteria is evid ence of con d u ct cal of p an ic d isord er, in w hich there are d is-
d isord er p rior to the age of 15. Earlier onset crete p an ic attacks, p eriod s of in tense fear,
of con d u ct d isord er is associated w ith an or d iscom fort, w ith p hysical m anifestations
increased risk of d evelop ing antisocial p er- su ch as p alp itation s and su bjective d ifficu lty
sonality d isord er. Cond u ct d isord er is not as breathing. There is no evid ence that this boy
closely associated w ith avoid ant p erson al- exp eriences d istress an d w orry w h en sep a-
ity d isord er, p aranoid p ersonality d isord er, rated from an im p ortant attach m en t figu re
schizoid p erson ality d isord er, or schizotyp al as in sep aration an xiety d isord er.
p ersonality d isord er.
32. (B) Clonidine has become the first-line treat-
29. (D ) By d efinition, sym ptom s of ad ju stm ent ment for Tourette disorder. It has a limited
d isord er d o not last longer than 6 m onths side-effect profile and helps control symptoms
after a stressor or the term ination of its con- of a frequently associated comorbid disor-
sequences. If d epressive sym ptom s persist, der, attention-deficit/ hyperactivity disorder
MDD m ay be d iagnosed . In both ad justm ent (ADHD). Tricyclic antidepressants (TCAs) have
d isord er and MDD, sym ptom s m ust cau se been shown to be effective in the treatment of
m arked d istress or significant im pairm ent in Tourette disorder, but other antidepressants
fu nctioning, m ay d evelop follow ing a stressor, such as bupropion, paroxetine, and venlafax-
m ay d evelop w ithin 3 m onths of the onset of ine are not known to be effective. High-potency
a stressor, and m u st not represent norm al antipsychotics such as haloperidol and pimo-
bereavem ent. Ad ju stm ent d isord er m ay also zid e were trad itionally the first-line agents for
be given sp ecifiers d ep end ing on the p res- Tourettes, but are more likely to cause signifi-
ence of anxiety, d ep ressed m ood , and cond u ct cant side effects. Recently, the newer atypical
d istu rbances. Ad ju stm ent d isord er follow ing (or second-generation) antipsychotics such as
bereavem ent m ay be given p rovid ed that risperid one and olanzapine have also been
the ind ivid ual d oes not m eet criteria for fu ll used to treat the disorder.
MDD, and the intensity of qu ality of the grief
reaction exceed s w hat is exp ected for cu ltu ral, 33. (A) It is not u nu su al for p arents of a seriou sly
religiou s, or age-m atched norm s. ill child to w ant to try to p rotect them from
any fu rther su ffering or d istress. H aving an ill
30. (D ) Follow ing a d iagnosis of d iabetes m ellitu s child is overw helm ing and anxiety-provok-
type I, approxim ately 33% of child ren d evelop ing. Parents m ay end u p treating su ch chil-
sym ptom s of ad justm ent d isord er. d ren as if they w ere you nger than their actual
age and m ay be m ore reluctant to set ap pro-
31. (E) Tou rette d isord er is the m ost likely p riate lim its. When calling a p arents attention
d iagnosis becau se the boys ou tbu rsts are to the potential harm of su ch interaction, a cli-
consistent w ith vocal tics, an d th e rep ort of nician m u st be tactfu l and em p athic. By gently
Answe rs : 2838 23

acknow led ging this m others d istress, she w ill Mood lability can be an associated sym p tom ,
be m ore recep tive to hearing feed back abou t althou gh the incid ence of bip olar d isord er is
her interactions w ith her child , inclu d ing that not p articularly increased . Langu age or sepa-
her relu ctance to set lim its and stru ctu re m ay ration anxiety d isord ers are also not increased
inad vertently be d etrim ental. While it is essen- in ind ivid u als w ith Tou rette d isord er.
tial, em p athizing alone w ill be insu fficient to
set the stage for a d iscussion of p arentchild 36. (E) A su bset of child ren w ith prior grou p A
interactions. Referring her to a p arent su p p ort beta-hem olytic strep tococcal infections have
grou p m ight be u sefu l follow ing you r initial d evelop ed Tou rette d isord er after the infec-
d iscu ssion w ith her. Telling her she is treat- tion. Other d isord ers associated w ith su ch an
ing her son like a baby is overly harsh and infection inclu d e OCD and Syd enham chorea.
m ore likely to m ake her d efensive rather that This grou p of d isord ers are referred to as p ed i-
recep tive. atric au toim m u ne neu rop sychiatric d isord ers
associated w ith strep tococcal infections, or
34. (C) A psychiatric illness in a child or ad oles- PAN DAS. The other infectiou s agents listed
cent w ith a m ed ical illness shou ld be treated are not associated w ith Tou rette d isord er.
aggressively u sing the type of treatm ent m ost
effective for the sp ecific p sychiatric illness, 37. (A) This boys history is typical of ADH D,
w hile keep ing in m ind p otential interactions com bined presentation. H e d oes not comp lete
of treatm ents. Med ical illnesses can exacerbate tasks, is careless, loses necessary item s, is d is-
p sychiatric illness and vice versa. As su ch, tracted and forgetful, is energetic, impu lsive,
effective treatm ent of a m ental illness m ay and cannot tolerate playing quietly. Symptom s
p ositively affect a m ed ical illness if im p rove- are p resent before age 12 and are occu rring in at
m ent of psychiatric sym p tom s enables the least tw o settings, and are causing im pairment
child to m ore fully participate in treatm ent of and d istress. Increased energy and im pulsivity
the m ed ical cond ition. In ad d ition, em otional can be a symptom of both ADH D and bipolar
state is related to im m u ne response. Althou gh d isord er, but to be an exacerbation of bipolar
there is a specific stressor in this case, it is d isord er, those symptom s m ust be associated
u nclear w hether the d ep ressive sym p tom s w ith energy and im pulsivity beyond base-
w ou ld resolve if the leu kem ia goes into rem is- line, inflated mood , grand iosity, increase in
sion; in ad d ition, given the im p airm ent in this goal d irected activity, and a d ecreased need
boys fu nctioning and that it is u nclear if/ for sleep. This boy d oes not d isplay excessive
w hen the leu kem ia w ou ld rem it, not treat- aggression, d estru ction of prop erty, d eceitfu l-
ing the d ep ression w ou ld be inap p rop riate. ness, theft, or serious violations of rules, as
Ap p rop riate p harm acologic treatm ent shou ld seen in cond u ct d isord er. H is behaviors are
be avoid ed only if there are sp ecific contrain- not negativistic, hostile, or d efiant, so he d oes
d ications based on the m ed ical treatm ent the not suffer from oppositional d efiant d isord er.
child is receiving. In ad d ition, the effective- Therefore, he d oes not have characteristics of a
ness of psychotherapy is related to the specific d isruptive behavior d isord er.
m ental illness, not to the p resence or absence
of a m ed ical illness. 38. (E) Stim u lant m ed ications are the first-line
treatm ent for ADH D bu t have been associ-
35. (D ) It is not u nu su al for child ren w ith Tou rette ated w ith an increased risk of d eveloping
d isord er to have com orbid p sychiatric d isor- tics. In general, if a child suffers from tics or
d ers. Obsessive-com pu lsive d isord er is a very has a fam ily history of tics, stim u lant m ed i-
com m only associated d isord er, often present- cations shou ld be avoid ed , and an alternate
ing in ad olescence. Other anxiety d isord ers, m ed ication shou ld be u sed to treat ADH D if
attentional d isord ers, and learning d isord ers necessary. Benzod iazep ines, D 2 antagonists,
can be seen as w ell. Autism spectru m d isord ers MAOIs, and SSRIs d o not com m only exacer-
are not com m only associated w ith Tou rettes. bate tics and are not u sed to treat ADH D.
24 1: Child a nd Ad ole s c e nt P syc hia try

39. (C) To be d iagnosed w ith a learning d isor- (Ad d erall) are u sed to treat ADH D, bu t they
d er w ith im p airm ent in read ing, there m u st are also stim u lants and w ill have sim ilar sid e
be evid ence of an ind ivid u als p erform ance effects to the m ethylp henid ate-based m ed i-
in a given acad em ic area (in this case, read - cations. Aripip razole is a second -generation
ing) falling below w hat is exp ected for age. (atyp ical) antip sychotic, citalop ram is an anti-
Psychom etric testing, su ch as IQ testing, is d ep ressant, and valp roic acid is a m ood stabi-
one m easu re to assess the d isparity betw een lizer u sed in bip olar d isord er; none are u sed to
exp ected achievem ent and actu al achieve- treat ADH D.
m ent. A learning d isord er can also be d iag-
nosed if the ind ivid ual is u sing significant 46. (B) The clinician can be help fu l to p arents
and excessive com p ensatory levels of effort or by exp laining and norm alizing the child s
su pp ort to sustain average achievem ent. behavior. H ow ever, it is still im p ortant
for the p arent to resp ond to or correct this
40. (D ) At least 50% of child ren w ith learning behavior. In the case of this you ng girl, jeal-
d isord ers have a com orbid p sychiatric d is- ou sy of her you nger sibling is entirely nor-
ord er. The m ost com m on com orbid cond i- m al and is m ore d ifficu lt for her, since as a
tions inclu d e ADH D, anxiety, and d epressive p reschooler her langu age skills m ay not be
d isord ers. d evelop ed su fficiently to articu late her d is-
tress. Once the m other accep ts this child s
41. (C) Most child ren w ith enu resis are boys, and behavior as u nd erstand able and in the nor-
overall anyw here betw een 20% and 40% have m al range, she can resp ond in an em p athic
a com orbid m ental d isord er. w ay w hile still setting lim its on her aggres-
sive behavior. Ignoring the behavior w ill not
42. (B) Of the choices p rovid ed , Dd avp is the treat- ad d ress the aggression and is not em p athic.
m ent of choice for enu resis; it is a variation Und er no circu m stances is biting the child
of the antid iu retic horm one vasop ressin and back help fu l. Threatening to w ithd raw
is given intranasally. Benztrop ine is u sed to love is m anip u lative and anxiety-p rovoking
p revent extrap yram id al sym p tom s cau sed to the child . By telling the child that she m u st
by antip sychotic (neu rolep tic) u se. Meth- love the baby, the m other is not tru ly em p a-
ylp henid ate is a stim u lant com m only u sed thizing w ith her and m ay elicit sham e and
in the treatm ent of ADH D. Paroxetine is a fu rther resentm ent tow ard the baby.
serotonin-sp ecific reu p take inhibitor u sed to
treat d ep ression and anxiety. Trazod one is an 47. (B) A 37-m onth-old girl is a you ng preschool
antid ep ressant m ost often u sed for insom nia. age child . At this stage, she shou ld be able
to state her age and gend er and be involved
43. (B) The m ost com m on ad verse effects of m eth- in or have recently com p leted p otty train-
ylp henid ate and other stim u lant m ed ications ing. Althou gh she has significant langu age,
inclu d e insom nia, d ecreased ap p etite, w eight it w ou ld be very u nu su al for her to u se her
loss, d ysp horia, and irritability. Trem or, hyp o- langu age sp ontaneou sly to id entify and artic-
tension, w eight gain, and liver toxicity are not u late her feelings abou t her you nger sibling.
com m on sid e effects. Althou gh a p recociou s 3-year-old m ay be able
to cou nt to 50, it w ou ld not necessarily be
44. (E) Stim u lant m ed ications, su ch as m ethylp he- exp ected of her u ntil she is 5 years. She is cu r-
nid ate, are effective in d im inishing sym ptom s rently consid ered to be in the p reop erational
of ADH D in ap p roxim ately 70% of p atients. stage of cognitive d evelopm ent and cannot
yet ap p ly concep ts of conservation of m atter
45. (B) Atom oxetine is a nonstim u lant norepi- (concrete op erations). H er gross m otor skills
nephrine reu ptake inhibitor, w hich has been and balance are likely not d evelop ed enou gh
fou nd to be help fu l in treating both child hood to rid e a bicycle. She shou ld be able to rid e a
and ad ult ADH D. Mixed am p hetam ine salts tricycle, how ever.
Answe rs : 3955 25

48. (D ) This p atients p resentation is su sp iciou s 52. (E) Stranger anxiety occurs as part of nor-
for au tism sp ectru m d isord er (ASD). ASD is m al child d evelopm ent and is evid ence of
characterized by d eficits in social com m unica- the d evelopm ent of a secure attachm ent; it
tion and social interaction as w ell as restricted , d oes not su ggest that a p arent is inatten-
rep etitive p atterns of behaviors, interests, or tive. Stranger anxiety u su ally ap p ears by 7 to
activities. Of the choices listed , the d ifficu lty 8 m onths and generally resolves w ith tim e.
w ith im aginary p lay rep resents a d eficit in Typ ically, stranger anxiety is stronger tow ard
u nd erstand ing relationships, w hich m ore com pletely u nknow n persons than tow ard
broad ly falls u nd er the d iagnostic criterion those w ho are m ore fam iliar. Becau se stranger
of d eficits in social interaction. The langu age anxiety at this age is d evelop m entally app ro-
im p airm ent criterion of the DSM -IV -TR w as priate, this boy cannot be d iagnosed w ith sep -
elim inated in the DSM -5. Attentional d ifficu l- aration anxiety d isord er, and he is not overly
ties, im pairm ent in m otor skill d evelopm ent, attached to his m other. Child ren w ith au tism
or d elay in potty training are not necessarily spectrum d isord er, in fact, often lack this
featu res of ASD. d evelop m ental m arker.

49. (D ) Child ren w ith au tism spectrum d isor- 53. (D ) This boy w ou ld be d iagnosed w ith sep a-
d er (ASD) have m ore favorable p rognoses if ration anxiety d isord er. While an 8-m onth-old
they are able to converse m eaningfu lly w ith child w ou ld typically d isplay stranger anxi-
others; though no longer p art of the DSM -5 ety, an 8-year-old w ou ld not. This behavior
d iagnostic criteria, the acqu isition of fu nc- w ou ld have norm ally gone aw ay for m ost
tional language by the age of 5 years rem ains a child ren by age 3 or 4 years. School age chil-
p ositive prognostic factor. It is not u nu su al for d ren are exp ected to be able to sep arate from
child ren w ith ASD to be easily toilet trained , the parent/ caregiver in ord er to attend school
enjoy m echanical toys, be rigid ly organized in and other activities. Ind ivid u als w ith agora-
their play, and be capable of m em orizing or phobia often refu se to leave their hom e d ue to
reciting p oem s, d ialogu e from a TV show or the fear of d eveloping a panic attack. Obses-
m ovie. These latter characteristics are not nec- sive-com pu lsive d isord er is characterized by
essarily associated w ith a good prognosis. obsessions and com pu lsions; the focu s is not
related to sep aration. Social anxiety d isord er
50. (A) This girl likely has anorexia nervosa. (social p hobia) involves anxiety in a social or
H yp ercholesterolem ia is com m on in anorexia perform ance situ ation.
nervosa. Other find ings associated w ith the
starvation state are m ild norm ocytic norm o- 54. (B) Ind ivid uals w ho d evelop schizophrenia
chrom ic anem ia and leu kop enia. If vom iting are m ore likely to have a p arent w ith a p sy-
is ind u ced , hyp okalem ia, hyp ochlorem ia, and chotic or schizotyp al d isord er. Ind ivid uals
m etabolic alkalosis m ay be seen. H ypercaro- w ho d evelop schizop hrenia are m u ch m ore
tenem ia, cau sing yellow ing of the skin, m ay likely to have a history of social w ithd raw al
be seen if m any carrots are eaten in an attem p t and introversion rather than extroversion.
to satisfy the appetite w ith a low -calorie food . Recent stressors su ch as p arental d ivorce or
TSH is not typ ically altered . m oving m ay contribu te to the onset or exac-
erbation of som e p sychiatric d isord ers bu t
51. (B) While all of the choices are complications not sp ecifically schizophrenia. Finally, recent
resulting from anorexia nervosa, a card iac d ru g u se w ou ld be m ore likely to su p p ort a
arrhythmia is consid ered a major com plica- d iagnosis of su bstance-ind u ced p sychosis.
tion and therefore alone justifies an inpatient
ad mission. If the patient has mu ltip le minor 55. (D ) Early-onset schizop hrenia occu rs before
com plications (e.g., anem ia, brad ycard ia, age 18 years, w hile child hood onset schizo-
hypotension, lanu go), strong consid eration for phrenia has an onset before the age of 15 years.
ad mission to the hospital shou ld also be given. Child hood onset of schizop hrenia is rare and
26 1: Child a nd Ad ole s c e nt P syc hia try

associated w ith a poorer ou tcom e. A better d eveloped the ability to have concrete think-
p rognosis is associated w ith an acu te onset, ing. Usually, this level of cognitive m aturity is
m ore affective sym ptom s, old er age at onset, achieved betw een the ages of 6 and 10.
good p rem orbid fu nctioning, w ell-d ifferenti-
ated sym ptom s, and lack of a fam ily history of 60. (C) This boy is displaying regression, a defense
schizophrenia. mechanism in w hich there is an attempt to
return to an earlier d evelopmental phase to
56. (C) Child ren w ith ADH D often suffer from avoid the tension and conflict at the present
low self-esteem , w hile child ren w ith m ania level of development (e.g., d istress over the
are m ore likely to be eu phoric. Distractibility, grandmothers death). Acting out is a defense
im p u lsivity, m otoric hyp eractivity, and p res- mechanism in w hich an unconscious w ish or
sured sp eech m ay be seen in child ren w ith impulse is expressed through action to avoid
both d isord ers. an accompanying affect. Denial is a d efense
mechanism in w hich the aw areness of a painful
57. (E) If a school is unable to manage a childs aspect of reality is avoided by negating sensory
behavioral or emotional difficulties despite d ata. Repression is a d efense mechanism in
ad ded support and special placement in a sepa- w hich an idea or feeling is expelled or w ithheld
rate classroom, the most appropriate next step is from consciousness. Somatization is the pres-
therapeutic d ay school placement. Therapeutic ence of physical symptoms that are a manifes-
schools are d esigned for stud ents w ith psychi- tation of emotional or psychological d istress.
atric disorders that are severe enough that their
home schools cannot keep the child or other 61. (D ) Sed ation is a frequ ent ad verse effect of
stud ents safe and manage the child in a learn- clonid ine u pon initiating treatm ent. With con-
ing environment. H ome schooling, a parochial tinu ed treatm ent, the sed ation usu ally sub-
school, or a school for stud ents w ith learning sid es. Dry m outh is a less com m on ad verse
d isabilities are not therapeutic in nature and d o effect. H yp otension is a p ossible sid e effect as
not have teachers and staff w ho are trained to clonid ine is also used as an anti-hypertensive
w ork w ith students w ith emotional and behav- m ed ication. The red u ction in blood pressu re
ioral disord ers. Resid ential placement involves u su ally d oes not resu lt in significant sym p -
the child resid ing at the school w hile receiving tom s, how ever. In general, nausea and trem or
therapeutic treatment and attend ing classes, are not seen.
and is reserved for students w hose needs can-
not be met by therapeutic schools. 62. (D ) Stim u lants are the m ost com m only u sed
m ed ication in the treatm ent of ADH D, and
58. (E) Valp roic acid is an anticonvu lsant, w hich althou gh they can u nm ask tic d isord ers,
is also a m ood stabilizer u sed to treat bip olar ADH D treatm ent gu id elines state that stim u -
d isord er. Du loxetine and bu propion are both lants can still be u sed in ind ivid u als w ith tics.
antid epressants and not ap propriate for treat- In fact, a recent stu d y fou nd that stim u lants
ing m ania as they m ay w orsen the sym p tom s d id not w orsen tics in those w ith tic d isord ers.
or prom ote rapid -cycling. Methylphenid ate Antip sychotics and MAOIs are not u sed for
and m ixed am phetam ine salts are stim u lants ADH D. SSRIs are infrequ ently u sed to au g-
u sed in attention-d eficit/ hyp eractivity d isor- m ent treatm ent in som e cases and are not con-
d er, and are contraind icated in m ania because sid ered a first-line treatm ent. Althou gh TCAs
they m ay actu ally exacerbate m ania. m ay be as effective as stim u lants, their p oten-
tial for ind u cing a card iac arrhythm ia or, less
59. (D ) Typ ically, child ren are 7 or 8 years old likely, su d d en d eath p reclu d es tricyclics from
w hen they begin to und erstand the irrevers- being u sed as first line for ADH D.
ibility of d eath. To und erstand that d eath is
irreversible, child ren m ust have com pleted the 63. (D ) The find ing of m u ltiple fractu res, espe-
cognitive stage of preoperational thou ght and cially w hen they are of d ifferent ages, is a
Answe rs : 5669 27

red flag for p hysical abu se. Even thou gh the the blood w ork is com plete. An EEG and refer-
boy m ay be scared to rep ort w hat hap p ened ral to a neu rologist is not necessary u nless one
for fear of punishm ent, it is im portant to try has su sp icions of a seizu re d isord er. A p u lm o-
to talk w ith him alone and find out as m u ch nary fu nction test is used to d iagnose asthm a,
as possible. Recom m end ing extra vitam ins or w hich is a m u ch less likely etiology of this
lim ited sp orts d oes not ad d ress the qu estion p atients sym ptom s.
of w hether the boy is safe at hom e. It is not
ap propriate to refer the boy to an orthop ed ist 67. (C) Identification is the process of adopt-
or his ped iatrician prior to investigating the ing other peoples characteristics. Identifica-
p ossibility of p hysical abu se. tion w ith a parent is important in personality
formation. This girls behavior may occur as
64. (E) In ad d ressing the d elicate task of helping a an attempt to imitate the doctor because she
child stop or alter their method of self-soothing, ad mires her, or it may represent an effort to
sensitive and collaborative approaches are much cope w ith anxiety about the d octor because she
more preferable to abrupt and harsh interven- fears her. Displacement is a d efense mechanism
tions. Encouraging a child to stop thumb suck- in w hich emotions are shifted from one idea or
ing with a more flexible reward system that is object to another that resembles the original but
enticing for the child engages the child in making evokes less distress. Dissociation is a d efense
choices about their own body, rather than impos- mechanism in w hich a persons character or
ing a harsh intervention. Both hot pepper sauce sense of id entity is temporarily but d rasti-
and a mouth appliance are punitive in nature. cally mod ified in ord er to avoid emotional d is-
However, more passive approaches like ignoring tress. Rationalization is a d efense mechanism
or offering gum are also not likely to be success- in w hich rational explanations are offered in
ful. Because prolonged thumb sucking can result an attempt to justify unacceptable attitudes,
in significant dental problems, helping a child beliefs, or behaviors. Reaction formation is a
stop thumb sucking is eventually valuable. d efense mechanism in w hich an unacceptable
impulse is transformed into its opposite.
65. (D ) This teenager is likely su ffering from p anic
d isord er. Serotonin-specific reu ptake inhibi- 68. (A) In all ju risd ictions in the United States, cli-
tors (SSRIs) su ch as sertraline are effective in nicians are m an d ated rep orters of su sp ected
the treatm ent of p anic d isord er. Antipsychot- child abu se and are requ ired to rep ort all
ics, su ch as arip ip razole, are not ind icated for cases. The law d oes not leave th e issu e to the
the treatm ent of panic d isord er. While d u lox- clin icians d iscretion or p arental/ gu ard ian
etine, a serotonin-norep inep hrine reu p take consent. Once the rep ort is m ad e, child p ro-
inhibitor, has an ind ication for the treatm ent tective services (CPS) m ay screen th e in ci-
of generalized anxiety d isord er in child ren d ent ou t, if the incid ent d oes not m eet the
over age 7 years, it has not been stu d ied exten- states d efinition of m altreatm ent or if insu f-
sively in p anic d isord er. Mood stabilizers, ficient inform ation is p rovid ed , CPS m ay
such as carbam azep ine and valproic acid , are screen th e case in, w hich p rom p ts fu rther
not as effective in p anic d isord er. investigation and p ossibly rem oving the
child from the hom e u ntil an investigation is
66. (E) Althou gh, the sym p tom s that the p atient com p leted .
d escribes are consistent w ith p anic attacks,
it is im p ortant to ru le ou t any other m ed ical 69. (D ) Although all of the listed choices are pos-
causes that could be em u lating that p resen- sible ou tcom es associated w ith child hood
tation (e.g., m etabolic problem s such as thy- abuse, d ep ression is the m ost com m on one. In
roid d ysfu nction, electrolyte abnorm alities, p articu lar, abu se of a chronic natu re can have
anem ia). An ECG w ou ld be help fu l if she a significant effect on p ersonality d evelop -
rep orted card iac sym p tom s as p art of her p re- m ent, and , in ad d ition, p red isp ose the child to
sentation. This optional test can be d one after significant psychop athology as an ad u lt.
28 1: Child a nd Ad ole s c e nt P syc hia try

70. (C) This boys history is m ost consistent CBT for OCD is the nonp harm acologic ther-
w ith selective m u tism . H e d oes not sp eak at ap eu tic treatm ent of choice. The CBT often
school but continu es to talk at hom e. Consis- involves exp osu re and resp onse p reven-
tent failure to sp eak in a specific social situ - tion to help extinguish com pulsions. Fam ily
ation d espite sp eaking in other situ ations is therapy can be help ful in ad d ressing fam ily-
not characteristic of MDD, persistent d epres- related issu es contribu ting to the child s
sive d isord er (d ysthym ia), sep aration anxiety sym ptom s. Grou p therapy can be p art of an
d isord er, or social anxiety d isord er (social OCD treatm ent, particu larly CBT. Short-term
p hobia). Social p hobia can be d iagnosed along p sychod ynam ic and sup portive therap ies are
w ith selective m u tism . not consid ered ad equ ate treatm ents alone,
althou gh they m ay be help fu l in ad d ress-
71. (C) The m other is the m ost com m on perpetra- ing related issu es su ch as self-esteem and
tor of intentionally p rod u cing physical or psy- relationship s.
chological sym ptom s in her child in ord er to
assu m e the sick role by p roxy (factitiou s d is-
7679. [76. (D ), 77. (A), 78. (B), 79. (C)] The DSM -5
ord er im posed on another). The victim is usu -
revised the intellectu al d isabilities (form erly
ally a preschool child , and im p osing an illness
m ental retard ation, MR) severity scale to be
on a child is consid ered to be physical abuse
based on the d egree of d eficits in fu nctional
w hich requ ires rep orting.
im p airm ent; su ch ad ap tive fu nctioning
72. (A) The case represents a child w ith cond u ct involved ad ap tive reasoning in the follow ing
d isord er. Ind ivid u als w ith cond u ct d isord er d om ains: concep tu al, social, and p ractical.
are at increased risk of later d evelop m ent of Ind ivid u als m ay fu nction at d ifferent levels
antisocial p ersonality d isord er; in fact, DSM -5 betw een each d om ain. Clinical assessm ent
criteria for antisocial personality d isord er (i.e., d egree of fu nctional ability) is necessary
requ ire evid ence of a cond u ct d isord er before to assess ad ap tive fu nctioning and therefore
the age of 15 years. severity of intellectu al d isability; p sychom et-
rics su ch as IQ testing shou ld be taken into
73. (C) Disobed ience tow ard authority figu res is a accou nt, bu t m ay not accu rately assess real-
characteristic of ODD. Aggression and violat- life fu nctioning and attainm ent of p ractical,
ing ru les are sym p tom s consistent w ith con- d aily tasks. The d eficits in ad ap tive fu nc-
d u ct d isord er. If the hostile and negativistic tioning m u st be related to the intellectu al
behavior consistent w ith ODD occu rs only in im p airm ents and occu r in the d evelop m ental
the context of a m ood d isord er, the child can- p eriod . The rep resentative abilities for m ild ,
not be d iagnosed w ith ODD. Relu ctance to m od erate, and severe intellectu al d isability
p articip ate in tasks that requ ire ongoing m en- are listed in the qu estion stem . Ind ivid u als
tal attention is a com m on sym ptom of ADH D. w ith p rofou nd intellectu al d isability gen-
erally are lim ited to concep tu al skills that
74. (B) Alexia is the inability to read . Agnosia is involve the p hysical w orld , rather than sym -
the inability to recognize objects d espite intact bolic p rocesses (concep tu al d om ain), have
senses. Anom ia is the sp ecific inability to nam e lim ited u nd erstand ing of sym bolic com -
objects even though the object is recognizable m u nication in sp eech or gestu res bu t m ay
and can be d escribed by the p atient. Ap hasia u nd erstand som e sim p le instru ctions (social
is m ore global than alexia and is an abnorm al- d om ain), and are d ep end ent on others of
ity in either the exp ression or the com p rehen- all asp ect of d aily p hysical care, health, and
sion of langu age. Apraxia is an inability to safety (p ractical d om ain). Of note, the range
p erform learned m otor skills d espite norm al of IQ for severe intellectu al d isability (for-
strength and coord ination. m erly MR) w as historically 25 to 39, for aver-
age (norm al) intelligence w as historically 90
75. (A) This p atients sym p tom s are consistent to 110, for m ild intellectu al d isability w as
w ith obsessive-com p u lsive d isord er (OCD). historically 55 to 70, for m od erate intellectu al
Answe rs : 7093 29

d isability w as 40 to 54, for p rofou nd intellec- p ersonality d isord er. Eye tics su ch as blinking
tu al d isability w as below 25, and bord erline and eye rolling are the m ost com m on initial
intellectu al fu nctioning w as consid ered an sym p tom s in Tou rette d isord er (K). Facial tics
IQ betw een 71 and 84. su ch as grim acing or licking m ovem ents and
vocal tics su ch as throat clearing or gru nting
8083. [80. (H), 81. (D ), 82. (G), 83. (L)] Palp itations, are the next m ost com m on initial sym p tom s.
chest pain, shortness of breath, and d iap horesis Whole-bod y tics, su ch as bod y rocking or
com m only occu r d u ring p anic attacks, a com - p elvic thru sting, and self-abu sive tics, su ch as
p onent of p anic d isord er (H). Binge eating hitting, m ay d evelop later.
d isord er (D ) is characterized by sham e and
loss of control over eating large am ou nts of 8487. [84. (E), 85. (A), 86. (G), 87. (F)] Selective m u t-
food , w ithou t com p ensatory behaviors, and ism (E) is rare, and child ren w ho su ffer from
w ithou t bod y im age d istortions. Chap p ed it are only m u te in certain situ ations (e.g.,
hand s and other d erm atologic p roblem s are school). It often involves a stressful life event
often p resent in obsessive-com p u lsive d isor- such as parents d ivorce and the child ren
d er (G) d u e to excessive w ashing w ith w ater are m u te by choice. Blurting ou t in class is a
or cau stic cleaning agents. Trichotillom ania sym ptom of im pu lsivity that is often seen w ith
(hair-p u lling d isord er) (L) is m u ch m ore com - ADH D (A). N o langu age im pairm ent is seen
m on in girls, w ith the onset frequ ently starting as p art of ADH D. Au tism sp ectru m d isord ers
w ith, or shortly after, p u berty. The rep eated (B) have both d ifficu lties in social interaction
p u lling ou t of hair resu lts in d ecreased or as w ell as restricted / rep etitive interests, w hile
com p lete loss of hair in a sp ecific area. The child ren w ith social (pragm atic) com m unica-
scalp , eyebrow s, and eyelashes are the sites tion d isord er (G) stru ggle w ith the practical
m ost com m only involved . Au tism sp ectru m ap p lications and nu ances of both verbal and
d isord er (B) is characterized by im p airm ents nonverbal com m u nication. Child ren w ith lan-
in social interactions and the d evelop m ent of gu age d isord ers (D ) u nd erstand social con-
stereotyp ed or rep etitive p atterns of behav- ventions, bu t stru ggle w ith the acqu iring and
iors, bu t w ithou t significant d elay in lan- u sing langu age d u e to problem s w ith com p re-
gu age skills or cognitive fu nction. Bu lim ia hension or p rod u ction. Child ren w ith gener-
nervosa (C), seen in 1% to 1.5% of ad olescent alized anxiety d isord er (C) d isp lay excessive,
girls and you ng w om en, is a bingeing and d ifficu lt to control w orry, abou t a m ultitu d e
p u rging typ e of eating d isord er; anorexia ner- of events; the anxiety or w orry also m anifests
vosa (A) is an eating d isord er w ith significant w ith at least one (in child ren) p hysical sym p -
w eight loss and fear of w eight gain. Illness tom . Social anxiety d isord er (social phobia)
anxiety d isord er (E), form erly hyp ochond ria- (F) is characterized by anxiety in situ ations
sis, is the p reoccu p ation that one has a seriou s w here one is exp osed to be scru tinized ,
d isease based on m isinterp retation of sym p - ju d ged , or observed .
tom s, d esp ite ap p rop riate m ed ical evalu a-
tion and reassu rance. Interm ittent exp losive 8893. [88. (D ), 89. (B), 90. (A), 91. (C), 92. (E), 93. (D )]
d isord er (F) is characterized by recu rrent As children grow , they pass through d ifferent
ou tbu rsts of behavior, m anifesting as either stages of d evelopment that represent emo-
verbal or behavioral aggression, that are ou t tional and cognitive maturation. A vital task
of p rop ortion to p rovocation or p recip itating of infancy (A) is to establish a secure sense of
stressors. Pica (I) is the eating of nonnu tri- trust that occurs in the relationship w ith ones
tional su bstances su ch as d irt or p aint. Pyro- responsive caregiver. Tod dlers (B) struggle
m ania (J) requ ires several criteria, inclu d ing w ith the increasing understanding that they
d eliberate and p u rp osefu l fire-setting and are a separate being from their caregiver, and
tension or affective arou sal before the act, they practice separating and reuniting w ith
w hich is not better accou nted for by con- their caregiver as a w ay to consolid ate their
d u ct d isord er, a m anic ep isod e, or antisocial sense of separateness and autonomy. Preschool
30 1: Child a nd Ad ole s c e nt P syc hia try

(C) child rens identification w ith strong and acts: aggression tow ard p eop le and anim als,
pow erful characters such as superheroes helps d estru ction of p rop erty, d eceitfu lness, and
them cope w ith their ow n feelings of smallness seriou s violations of ru les. There is nothing
and inadequacy (as compared to their parents). to su ggest an au tism sp ectru m d isord er, p ar-
This stage is also called the Oedipal phase; at ticu larly becau se it w ou ld have likely been
this time child ren typically experience long- d iagnosed m u ch earlier; m ore im p ortantly,
ings tow ard the parent of the opposite sex and his actions seem w illfu l and not rep etitive
jealousy and hostility tow ard the same sex par- or stereotyp ed . A child w ith bip olar d isor-
ent. School age (D ) child ren d evelop the abil- d er m ay d isp lay im p u lsivity and aggressive
ity to think more logically and concretely and behavior, how ever, the child m aintains an
thus can id entify that equal amounts of liquid overall sense of right and w rong and often
in tw o d ifferently shaped containers hold the exp eriences regret for the ou t-of-control
same amount; once they are able to grasp this behavior once sym p tom s are im p roved .
concept of conversation of liquid s, child ren Also, bip olar child ren have significant m ood
have progressed into the stage of concrete sym p tom s that u nd erlie their behavior. Chil-
operations, w here they can consistently hold d ren w ith cond u ct d isord er m ay also have a
onto more than one d imension at a time. Ad o- com orbid m ood d isord er, bu t m ood sym p -
lescents (E) further advance in cognitive abili- tom s are not necessarily part of the cond uct
ties, finally d eveloping the ability to think in d isord er constellation. There is no evid ence of
an abstract fashion (reaching the stage Piaget psychotic symptom s, su ch as hallu cinations
d escribed as formal operations). or d elusions that w ou ld be characteristic of
child hood schizophrenia. Oppositional d efi-
94. (C) The DSM -5 rates the severity of su bstance ant d isord er requ ires 6 months of negativistic,
u se d isord ers on the basis of how m any cri- hostile, and d efiant behavior d irected mostly
teria are p resent, w ith m ild severity need - at au thority figures. While it m ay be a p remor-
ing tw o to three criteria, m od erate severity bid cond ition of cond uct d isord er, it is not as
need ing four to five criteria, and severe su b- severe.
stance u se d isord er requ iring m ore than six
criteria. Substance u se criteria includ e trou ble- 96. (B) This child is exhibiting chronic irritability
som e behaviors and consequ ences associated that is im pairing to his fu nctioning, a hallm ark
w ith the u se, inclu d ing a failu re to m eet w ork of DMDD. The DSM -5 classifies DMDD, a new
or school expectations (su ch as failing grad es), d iagnosis, in the d epressive d isord ers seg-
p utting oneself at risk (su ch as d riving w hile m ent, and it is therefore im p ortant to d istin-
u sing) or legal p roblem s. The criteria also gu ish it from MDD. This p atient d oes not have
incorp orate evid ence of p hysical and / or p sy- a p red om inantly d ep ressed m ood and neuro-
chological d epend ence on the su bstance; this vegetative sym ptom s (e.g., insom nia, anergia,
inclu d es tolerance, w ithd raw al, heavier u se change in ap petite, poor concentration) that
than intend ed , and d isp laying an ongoing w ou ld be consistent w ith MDD. Criteria For
d esire or u nsu ccessful attem pts to cut back DMDD inclu d e that the sym p tom s have been
on su bstance u se. N either u sing a su bstance consistently p resent (3 or m ore per w eek) for
w hile alone nor the frequ ency of u se are at least 12 m onths, w ithou t a sym ptom -free
sym ptom s w hen d iagnosing a substance u se p eriod m ore than 3 w eeks; sym p tom s m ust be
d isord er; how ever, using a su bstance m ost p resent by age 10 years, and no earlier than
d ays per w eek and / or u sing alone ind icate age 6 years; sym p tom s m u st occu r in at least
the persons use is m ore extensive and poten- tw o of three settings and m u st be severe in at
tially m ore concerning, w hich shou ld prom pt least one. There is no evid ence of m anic sym p -
fu rther qu estioning. tom s (e.g., d ecreased need for sleep, elevated
m ood , p ressu red sp eech, flight of id eas, and
95. (D ) This child fu lfills criteria for cond u ct d is- grand iosity) consistent w ith bip olar d isord er.
ord er, nam ely, three of the fou r follow ing H e d oes not ap pear to exhibit anxiety over
Answe rs : 9499 31

d ifferent areas of his life to ind icate general- bod y im age concerns as in anorexia, and are
ized anxiety d isord er. While there is som e often aw are they are losin g w eigh t. Th ese
overlap betw een DMDD and ODD (both m ay p atients tend n ot to binge eat as in bin ge
be d iagnosed ), p atients w ith ODD d o not have eatin g d isord er, n or engage in com p ensa-
the pred om inate irritable m ood sym ptom s tory m echanism s to lose w eight, su ch as in
intersp ersed w ith the anger ou tbu rsts in m u l- bu lim ia nervosa. While this child d oes have
tiple settings. a fear of vom iting, w hich cou ld be a resu lt
of a sp ecific p h obia, the overarch ing clin ical
97. (C) This stu d ent is m ost likely struggling d ue concern is her n u tritional statu s, w orsened
to variations in her acad em ic environm ent, by the d istress of eating. For som e child ren,
inclu d ing external factors su ch as learning in th e avoid ance m ay be based on the food s
English (not her native langu age), likely hav- sen sory characteristics. Patients w ho lack an
ing a gap w ith d isru p tion of her schooling interest in eating or have a low ap p etite m ay
d ue to w ar, and possibly a lack of equ ivalent also qu alify for a d iagnosis of ARFID.
ed u cational op p ortu nity in her hom e cou n-
try. Learning d ifficu lties d ue to neu rological 99. (C) This child is struggling with gender d ys-
(e.g., stroke, traum atic brain injury) or sensory phoria of childhood. The core feature of gen-
(e.g., hearing or vision im pairm ent) cau ses are der dysphoria (formerly called gender identity
d istinct from specific learning d isord ers. It is disorder) is an experience of incongruence
im p erative that sensory d eficits are screened between ones experienced gender and ones
and treated before giving a psychiatric d iag- assigned gender. Presentations of exhibited
nosis; in this case, there is no ind ication of su ch behaviors differ pre- and post-puberty, but the
a d eficit, w hich, even if corrected , still w ou ld core features remain. In body dysmorphic dis-
not accou nt for the variance of this child s aca- order, the main feature is perception of a spe-
d em ic background . Learning d isord ers are cific body part as abnormal, not the sense that
not m u tu ally exclu sive from PTSD, bu t there one is the wrong gender. In transvestic disor-
is no evid ence of re-exp eriencing, arou sal, or der, cross-dressing behavior generates sexual
avoid ance sym p tom s seen in PTSD. excitement in heterosexual individuals (most
often) and does not challenge primary gender.
98. (B) This ch ild is exhibiting sym p tom s of Insistence on being of some other gend er is not
ARFID, w h ich is ch aracterized by avoid - considered delusional in the absence of other
ing food or restricting food intake w ith- psychotic symptoms. Nonconformity to gender
ou t intentional efforts to lose w eight and roles should be distinguished from gender dys-
w ithou t bod y shap e concerns, that resu lt phoria by the degree of distress and pervasive-
in nu tritional d eficiencies and w eight loss. ness of gender-variant activities in the latter,
Patien ts w ith ARFID d o not have the sam e driven by the desire to be a different gender.
This page intentionally left blank
CHAPTER 2

Adult Ps yc ho patho lo g y
Que s tions

DIRECTIONS (Questions 1 through 120): For each (D) 50%


of the multiple-choice questions in this section select (E) 100%
the lettered answer that is the one best response in
each case. Questions 3 and 4

A 40-year-old man w ith schizophrenia comes for his


Questions 1 and 2 regular outpatient med ication management appoint-
A 22-year-old single m an is referred to you for a ments. H e reports that over the last w eek his intestines
1-year history of strange behavior characterized and heart have been removed . H e has subsequently
by talking to the television, accu sing local p olice of w ithd raw n and been staying in his basement apart-
bu gging his room , and carrying on conversations ment, avoid ing friend s and family members. When
w ith him self. H is m other d escribes a 3-year history asked about his lack of getting out in the w orld , he
of progressive w ithd raw al from social activities, responds, What w orld? There is no w orld!
and reports the p atient d ropp ed ou t of college and
since has been living in his room at hom e. Attem pts 3. Which of the follow ing term s best d escribes
to hold a job as a bu sboy at a local restaurant have this sym p tom ?
abruptly end ed after d isp u tes w ith the em p loyers. (A) Cap gras synd rom e
(B) Cotard synd rom e
1. What is the p revalence of this patients likely
(C) Folie d eu x
illness in the general p op u lation?
(D) Fregoli d elu sion
(A) 0.1% (E) Major d ep ressive d isord er
(B) 1%
(C) 2% 4. Up on retu rning for a follow -u p visit 15 d ays
(D) 3% later, the p atient now claim s that cyborg
(E) 5% alien robots that look id entical to his p arents
have recently rep laced his m other and father.
2. The p atients m other inform s you that he has Which of the follow ing term s best d escribes
an id entical tw in brother. What is the likely this sym ptom ?
chance of the patients tw in also having the (A) Cap gras synd rom e
sam e illness? (B) Cotard synd rom e
(A) 1% (C) Delu sional d isord er
(B) 10% (D) Folie d eu x
(C) 20% (E) Fregoli d elu sion

33
34 2: Ad ult P sychop a thology

5. A 32-year-old w om an w ith a history of d ep res- her blood alcohol is und etectable. H er urine
sion as a teenager, now 6 d ays p ostp artu m , is toxicology results com e back positive for can-
brou ght into the em ergency room by her hu s- nabis, w hich she confirm s she started sm oking
band . She has not slep t for the p ast several abou t 3 m onths ago. Which of the follow ing
d ays, even w hile the new born is nap p ing. w ou ld be the m ost ap p rop riate p rovisional
She has been irritable, and has been pacing in d iagnosis?
the m id d le of the night and w eeping, w hile
(A) Brief p sychotic d isord er
talking to no one in particular. Yesterd ay she
began to ignore the infant, bu t tod ay she vol- (B) Schizop hreniform d isord er
u nteered that the child is the Antichrist and (C) Schizop hrenia
m ust be d estroyed . She ad m its to you that (D) Su bstance-ind u ced m ood d isord er
she w ants to sm other the infant in a hu m ane (E) Su bstance-ind u ced p sychotic d isord er
w ay to p revent the ap ocalyp se. Which of the
follow ing d iagnoses is the m ost likely? 8. She w as started on appropriate treatm ent. One
(A) Bip olar d isord er year later this w om an retu rns to you r office
w ith her m other for follow -u p . H er sym p tom s
(B) Delu sional d isord er
rem itted w ithin a m onth. H ow ever, she has
(C) Major d ep ressive d isord er (MDD) w ith not d one w ell in her freshm an year and for the
p sychotic featu res p ast several m onths has continu ed to exp eri-
(D) Schizoaffective d isord er ence w orsening social isolation and am otiva-
(E) Schizop hrenia tion. While she has not u sed any su bstances
since she last saw you , she relu ctantly ad m its
Questions 6 through 8 to occasionally hearing the d evil com m u nicat-
ing w ith her. She tries to ignore the com m u ni-
An 18-year-old college freshm an w ithou t p rior p sy- cation, and has taken to arranging her books
chiatric history is brought to the em ergency room in a certain m anner to p revent his controlling
after being fou nd on her d orm itory roof d ressed only her thoughts. On her m ental statu s exam ina-
in her u nd erw ear, d esp ite freezing tem p eratu res. tion she m akes poor eye contact and her affect
Cam pu s p olice report she w as flapping her hand s is blu nted . H er m other rep orts that the p atient
and clim bing the banister on the roof, stating to an now rarely calls hom e, thou gh before shed d o
u nseen other, I w ill d o as you com m and soar to so tw ice w eekly. Which of the follow ing is the
m y d eath to fu lfill your p rophecy! She accu sed the m ost likely d iagnosis?
p olicem en of being Satans horsem en and cu rsed
as she w as being taken d ow n. H er room m ates con- (A) Brief p sychotic d isord er
firm that for the p ast 5 w eeks the patient has been (B) Schizop hreniform d isord er
acting bizarrely, and her speech has been increas- (C) Schizop hrenia
ingly d isorganized . You p lan to obtain collateral (D) Su bstance-ind u ced m ood d isord er
inform ation from her fam ily.
(E) Su bstance-ind u ced p sychotic d isord er
6. Which of the follow ing features w ou ld you be
9. A 42-year-old w om an p resents to a therap ist
m ost likely to find in her history?
w ith a history of d ram atic m ood sw ings since
(A) H ead trau m a early ad olescence, w here she w ill qu ickly
(B) Low intelligence becom e d eep ly d ep ressed for hou rs to d ays,
(C) N eglectfu l m other u su ally in resp onse to sep aration from a
loved one. She also ad m its to rage attacks,
(D) Physical or sexu al abu se
w here she w ill break item s, scream , or scratch
(E) Progressive social w ithd raw al herself su p erficially on her arm s. She inter-
m ittently binge d rinks and has frequ ently
7. She is ad m itted to the psychiatric unit. H er engaged in u np rotected sexu al intercou rse
p hysical exam ination is unrem arkable, and w ith new p artners. Others d escribe her as
Que s tions : 513 35

reactive and intense. Which of the follow - 11. Which of the follow ing d iagnoses is the m ost
ing d efense m echanism s d oes this p atient likely?
m ost likely em p loy?
(A) Catatonia associated w ith m ajor
(A) Altru ism d ep ressive d isord er
(B) Intellectu alization (B) Catatonia associated w ith schizop hrenia
(C) Sp litting (C) H yp othyroid ism
(D) Sublim ation (D) N eu rolep tic m alignant synd rom e
(E) Und oing (E) Unsp ecified catatonia

10. The fam ily of a 26-year-old patient w ith 12. Recognizing the clinical situ ation in front of
schizophrenia brings him in for follow -u p. you , you ad m it the p atient to the p sychiatric
H e w as initially d iagnosed at age 25 after a w ard for inability to care for self. Which of the
psychotic break that, in retrosp ect, follow ed a follow ing treatm ents w ou ld be best started
protracted cou rse of increasing isolation and im m ed iately?
am otivation. Desp ite d ifficu lties, he w as able
(A) Am itriptyline
to grad u ate from college and hold a fu ll-tim e
job by age 23. H e reports interm ittent hallu ci- (B) Electroconvulsive therapy (ECT)
nations, bu t has been able to m aintain ind e- (C) Lithiu m
pend ent living and p art-tim e em p loym ent. (D) Lorazep am
On exam ination, he is a d isheveled m an w ho (E) Sertraline
articu lates a m u ltitu d e of d elu sional beliefs
w ith a sophisticated vocabu lary. Which of 13. A 49-year-old bank teller w ith no know n psy-
the follow ing characteristics in this patient is chiatric history is referred to your office by
m ost strongly associated w ith a better overall her internist for an evaluation. For the p ast
prognosis? 2 m onths, she has been increasingly convinced
(A) Age at presentation that a w ell-know n m usic star is in love w ith
her and that they have had an ongoing affair.
(B) Gend er of p atient
She is w ell-groom ed , and there is no evid ence
(C) Insid iou s sym p tom onset of thou ght d isord er or hallucinations. She has
(D) Pred om inantly p ositive sym p tom s been fu nctioning w ell at w ork and in other
(E) Prem orbid fu nctioning social relationships. Which of the follow ing is
the m ost likely d iagnosis?
Questions 11 and 12
(A) Brief psychotic d isord er
A 36-year-old patient w ith no previou s psychiatric (B) Delusional d isord er
history is brou ght to the em ergency room by his (C) Paranoid p ersonality d isord er
fam ily. For the p ast m onth he has not been eating (D) Schizop hrenia
regu larly and has isolated him self in the ap artm ent
(E) Schizop hreniform d isord er
w here he lives alone. Tw o m onths ago he stop p ed
fishing, and rep orted feeling like a bu rd en on his
Questions 14 and 15
fiance. On m ental statu s exam ination, he d isp lays
p sychom otor retard ation and rarely blinks. H e A 46-year-old d ivorced w om an w ith a history of
resp ond s in the negative to any qu estion asked . m ajor d ep ressive d isord er is ad m itted to you r
On physical exam ination, he app ears in no acute inp atient p sychiatric u nit follow ing an intentional
d istress, is afebrile, m arginally hypotensive, and acetam inop hen overd ose. She has had m u ltip le
m ild ly tachycard ic. H e resists you r m otions w ith psychiatric hospitalizations w ith similar presen-
strength proportional to w hat you exert and crud ely tations. The patient reports a 3-w eek history of
m im ics your m ovem ents. H is laboratory stu d ies are insom nia, d ifficulty concentrating, low energy,
u nremarkable. hop elessness, and a d ecreased app etite. She has been
36 2: Ad ult P sychop a thology

unable to w ork recently because of her d epression Questions 16 and 17


and has lost interest in activities she once enjoyed .
She has no history of manic episod es. H er past psy- A 34-year-old w hite m ale is referred by his p rim ary
chiatric history is significant for a prior episod e of care physician for d epression. Upon initial inter-
d epression after the birth of her second child . She view , he com plains of feeling blue. H is m ental
has u nd ergone treatment w ith several ad equ ate tri- statu s exam ination reveals a d isheveled ap pearance,
als of m ed ications, inclu d ing au gm entation w ith d ep ressed m ood , p sychom otor retard ation, and su i-
lithium, yet she has continued to have resid ual cid al id eation w ithout plan. H is thought processes
sym ptoms of d epression. She has never been treated are significant for thou ght blocking and som e slow -
w ith psychotherapy. In the past she has experienced ing. Deficits w ith rem ote and short-term m em ory
au d itory hallucinations w hen her d epression w as are noted . Ju d gm ent and insight are also im p aired .
most severe. H er med ical history is significant for You r provisional d iagnosis is m ajor d epressive
hyp othyroid ism , w hich is ad equately m anaged w ith d isord er.
levothyroxine. She also had prior surgery after a leg
fracture from a suicid e attem pt w here she jumped 16. Which type of sleep d istu rbance you w ould
ou t of a w ind ow . m ost exp ect to see in this patient?
(A) Decreased resp onse to sed ative d ru gs
14. Which of the following factors would be most (B) Early m orning aw akening
influ ential in you r recom m end ing electro-
(C) Increased rap id eye m ovem ent (REM)
convu lsive therapy (ECT) for this patient?
stage latency
(A) Concu rrent thyroid d ysfu nction w ith (D) Sleep ing too d eep ly (d ifficu lty being
ad equate treatm ent aw akened )
(B) H istory of associated p sychotic (E) Sleep ing too lightly (aw akened too
sym ptom s w ith prior d epression easily)
(C) H istory of p erip artu m d ep ression
(D) Severe d ep ression that has not 17. Which of the follow ing m etabolic changes
resp ond ed to several m ed ications w ou ld be m ost likely fou nd in this p atient?
(E) Treatm ent-resistant d ep ression w ith (A) Decreased m onoam ine oxid ase (MAO)
recu rrent su icid al id eation activity
(B) Increased catecholam ine activity
15. The ind ications for ECT are d iscu ssed w ith
(C) Increased cortisol secretion
the patient, as w ell as the risks, benefits, and
(D) Increased sex horm ones
sid e effects. She asks appropriate qu estions
and consents to ECT. Fu rther m ed ical history (E) Increased im m u ne fu nctions
is exp lored p rior to beginning the p roced u re.
Which of the follow ing cond itions w ou ld be a Questions 18 and 19
relative contraind ication to p roceed ? A 35-year-old m an is being treated for m ajor d ep res-
(A) Coronary artery d isease, w ith a sive d isord er w ith paroxetine. H e has m issed his last
m yocard ial infarction 2 years ago tw o appointm ents, leaving m essages telling you that
(B) Im p lanted p acem aker for p eriod ic Ive been sp ectacular! H is w ife has since called
arrhythm ia you to rep ort that her hu sband has been sp end ing
m oney on frivolou s item s and overd rew the cou -
(C) Incid entally fou nd frontal m eningiom a
p les bank account. She states that her hu sband now
m easu ring 10 cm in d iam eter
quickly becom es agitated and angry. During the
(D) Second -trim ester p regnancy interview in you r office, the p atient qu estions you r
(E) Trau m atic brain inju ry su stained in cred entials and accu ses you of being m ore loyal to
teenage years his w ife than to him . Most of the interview is spent
interru p ting the p atient as you try to d ecip her his
Que s tions : 1423 37

rap id sp eech. In his rant he threatens to cu t the brake 20. Which of the follow ing d iagnoses is m ost
lines on his m other-in-law s car becau se he feels she likely?
has been intru d ing in his m arriage.
(A) Bip olar I d isord er
18. Which of the follow ing is the next m ost (B) Bip olar II d isord er
ap propriate step ? (C) Cyclothym ic d isord er
(D) Dou ble d ep ression
(A) Discharge the p atient hom e as he
refu ses ad m ission, bu t see him (E) Persistent d ep ressive d isord er
tom orrow
21. The patient returns w ithin 4 w eeks, and he now
(B) Escort the p atient (w ith p olice assistance
d escribes w orsening d epression for the past
as need ed ) to the nearest em ergency
3 w eeks, w ith ongoing insomnia, poor appetite,
room
little energy, anhed onia, and poor concentra-
(C) Inform the m other-in-law that she is in tion. H e also ad mits to passive suicidal ideation
d anger w ithout plan. Which of the follow ing w ould be
(D) Inform the p olice that a threat has been the most likely d iagnosis?
m ad e against the m other-in-law
(A) Bip olar I d isord er
(E) Tell the w ife to have the m other-in-law
stay w ith her (B) Bip olar II d isord er
(C) Cyclothym ic d isord er
19. After ad d ressing the above situ ation, the (D) Dou ble d ep ression
p atient is su bsequently started on valp roic (E) Persistent d ep ressive d isord er
acid . Which of the follow ing ad d itional p har-
m acologic interventions w ould be the m ost Questions 22 and 23
ap prop riate?
A 26-year-old fem ale p resents to the psychiatric
(A) Check a seru m p aroxetine level em ergency d ep artm ent in an acu tely d istressed ,
(B) Cross tap er the p aroxetine to nervou s state. She com p lains of terrible anxiety,
nortrip tyline and the em ergency d ep artm ent staff is u nable to
(C) Discontinu e the p aroxetine calm her d ow n or gain an ad equ ate history from
(D) Initiate bu p rop ion the patient. On physical exam ination, she is slightly
(E) Initiate lorazep am d iaphoretic, tachycard ic, and her pup ils are m ild ly
d ilated . She is on no m ed ications.
Questions 20 and 21
22. Which of the follow ing tests w ould be the
A new patient com es to you r office for a p sychiat- m ost useful in d eterm ining the etiology of her
ric evalu ation. H e d escribes m any years w here he sym ptom s?
experiences episod es of 5 to 7 d ays of feeling very
(A) Blood glu cose
d epressed , w ith insom nia, low energy, and poor
concentration. H e d enies any suicid al id eation or (B) Catecholam ine m etabolites
prior attem p ts. Up on further history he reveals (C) Electrocard iogram (ECG)
ad d itional period s w here I feel the opp osite, w ith (D) Thyroid fu nction
d ecreased sleep, elevated energy, feeling on top of (E) Urine toxicology
the w orld , and increased sex d rive. These tim es last
1 to 2 w eeks, and he d enies any p aranoia, d elu sions, 23. Which of the follow ing su bstances w ou ld be
or hallu cinations. Desp ite these recu rrent ep isod es, m ost likely to app ear on her urine toxicology
he has alw ays been able to fu nction ad equ ately, resu lts?
although they rem ain d istressing to him . H e d enies
any d ru g or alcohol u se, and he has no significant (A) Caffeine
m ed ical p roblem s. (B) Cannabis
38 2: Ad ult P sychop a thology

(C) Cocaine the staff reports that she has becom e extremely d oc-
(D) Op iates ile and d isplays very little emotion. She has a large
(E) LSD appetite and compu lsively puts both food and non-
food item s in her mouth. She also d isplays sexual
Questions 24 through 26 d isinhibition, often w alking out of her room w ithout
her pants on.
A 46-year-old m an is ad m itted to the hospital for
elective cholecystectom y. On hosp ital d ay 4, he is 27. Which of the follow ing clinical cond itions best
noted to be afebrile, but acutely d iaphoretic, tachy- d escribes her behavior?
card ic, hypertensive, trem ulou s, and agitated . H e
(A) Arnold Chiari synd rom e
tears ou t his sutures and insists on leaving against
m ed ical ad vice (AMA). H e is ap parently hallu cinat- (B) Kl verBu cy synd rom e
ing, ju d ging from his insistence that he be allow ed (C) Mbiu s synd rom e
to squ ash those bu gs on the w all (there are none). (D) Pick d isease
(E) Pu nch-d ru nk synd rom e
24. Which of the follow ing d iagnoses is the m ost
likely? 28. N eu roim aging w ou ld m ost likely show d am -
(A) Brief psychotic d isord er age to w hich tem p oral lobe stru ctu re?
(B) Delirium (A) Am ygd ala
(C) Delu sional d isord er (B) H ip p ocam p u s
(D) Fu nctional neu rological sym p tom (C) Inferior horn of the lateral ventricle
d isord er (D) Insu la
(E) Postoperative sepsis (E) Su p erior tem p oral gyri

25. Which of the follow ing m ed ications w ou ld be Questions 29 and 30


m ost app rop riate to treat his cond ition?
A 22-year-old w oman presents w ith fatigue for 4
(A) Diazep am months. She also reports irritability and poor energy,
(B) Disulfiram w ith dismal sleep and poor concentration. She has
(C) Lorazep am maintained a rigorous exercise routine, as she states
(D) Phenobarbital it makes her feel good to run off her boyfriends cook-
(E) Phenytoin ing. Her oral intake has sharply d ropped and she has
lost a significant amount of w eight, but she explains
26. If u ntreated , w hat w ou ld be his m ost likely that she hasnt been as hungry d ue to increased
mortality rate? stress. H er thyroid stud ies are normal, her CBC
reveals anemia, and she is not pregnant, in fact stat-
(A) 5% ing that she hasnt had a period in several months.
(B) 10%
(C) 30% 29. Up on fu rther qu estioning, w hich of the
(D) 50% follow ing qu alities w ou ld you m ost exp ect to
find in her social history?
(E) 60%
(A) H er p arents likely p raise her su ccesses.
Questions 27 and 28 (B) She has a legal history significant for
A 62-year-old w om an presents to the nu rsing home reckless d riving.
w here you w ork as a consulting psychiatrist. She has (C) She has asp ired to be a m od el.
a history of a bilateral tem poral lobectomy for intrac- (D) She is of low socioeconom ic statu s.
table seizures. After a few w eeks at the new facility, (E) She is a scholastically gifted
in ad d ition to her short-term m em ory d ifficulties, perfectionist.
Que s tions : 2434 39

30. Which of the follow ing d iagnoses is the m ost (D) Major d ep ressive d isord er
likely? (E) N orm al reaction
(A) Anorexia nervosa
33. Seven m onths later, your patients m ood has
(B) Avoid ant/ restrictive food intake
w orsened , and he feels w orthless and hop e-
d isord er (ARFID)
less that he w ill never find another girlfriend .
(C) Bu lim ia nervosa H e has lost a few p ou nd s and has not resu m ed
(D) Exercise-ind u ced am enorrhea d ating, p referring to isolate him self in his
(E) Obsessive-com p u lsive d isord er (OCD) stu d io apartm ent. H e is still feed ing his d og,
bu t he has had to m iss increasing am ou nts of
31. A 23-year-old college stu d ent has been su f- w ork becau se he ju st cant get him self to get
fering w ith frequ ent ep isod es of feeling read y for class. Which of the follow ing is the
u tter d oom for the p ast 3 m onths. Du ring m ost likely d iagnosis?
these p eriod s, he also experiences trem u lou s-
(A) Acu te stress d isord er
ness, sw eating, d izziness, and tingling in his
extrem ities. H e rep orts having these attacks at (B) Ad ju stm ent d isord er
least once a w eek and is now becom ing fear- (C) Generalized anxiety d isord er
fu l of attend ing classes lest he has an ep isod e. (D) Major d ep ressive d isord er
Which of the follow ing m ed ications w ou ld be (E) N orm al reaction
the m ost appropriate for im m ed iate relief of
his sym ptom s? 34. A 21-year-old college stu d ent is brou ght to
(A) Alprazolam (Xanax) you r office by her p arents after com p leting
her spring sem ester w ith u ncharacteristically
(B) Chlord iazep oxid e (Libriu m )
low grad es. H er p arents rep ort that since
(C) Divalp roex sod iu m (Dep akote) the sp ring session end ed 2 m onths ago their
(D) Fluoxetine (Prozac) d au ghter has been staying in her room , bu t
(E) Phenelzine (N ard il) is irritable w hen they try to engage her in
conversation. She is not interested in fam ily
Questions 32 and 33 barbecu es or being a cou nselor for the p ark
d istrict soccer cam p . After her p arents leave
A 36-year-old grad u ate stu d ent com es to you r
the exam ination room , your patient d iscloses
office because of d ifficulty sleep ing since breaking
she has been feeling d epressed since a m onth
up w ith his fiance 5 m onths ago. H e is sp end ing
before final exam inations, and her concen-
at least 2 hours in bed thinking abou t his ex-fiance
tration and sleep have been horrible. She
and w hat he cou ld have d one d ifferently. H is con-
had su icid al thoughts after she got her report
centration is w orsening and hes having d ifficu lties
card back, but d enies them cu rrently. She also
com pleting his coursew ork. While he feels d ow n, he
ad m its to d rinking alcohol to m ake herself feel
is not su icid al, and he is seeking su p p ort of friend s
better, and her intake ranges from 3 to 4 beers
in the p ost-breaku p p eriod . H e is attend ing m ost
or shots of liqu or m ost d ays of the w eek. Asid e
classes. H e d escribes feeling tenser overall, espe-
from likely alcohol u se d isord er, w hich of the
cially w hen he is at a bar. H e is having m ore trouble
follow ing d iagnoses is the m ost ap p rop riate at
relaxing and has noted the new onset of low back
this tim e?
and should er pain.
(A) Acu te stress d isord er
32. Which of the follow ing is the m ost likely (B) Ad ju stm ent d isord er
d iagnosis? (C) Am otivation synd rom e
(A) Acu te stress d isord er (D) Major d ep ressive d isord er (MDD)
(B) Ad ju stm ent d isord er (E) Su bstance-ind u ced m ood d isord er
(C) Generalized anxiety d isord er
40 2: Ad ult P sychop a thology

35. A 37-year-old w om an, w ho w orks the night (A) Bipolar d isord er w ith psychotic features
shift at a local grocery store taking inventory, (B) Major d ep ressive d isord er w ith
reports that her child hood and college years p sychotic featu res
w ere u neventfu l bu t hap p y. She sp end s m ost (C) Schizoaffective d isord er
of her tim e alone w hen she is not at w ork.
(D) Schizop hrenia
She d oes not ventu re out of her house and
her social contacts are lim ited to w ork-related (E) Schizop hreniform d isord er
interactions w ith cow orkers. She is an avid
p lant lover, and she spend s m ost of her free 38. A 42-year-old bu siness execu tive p resents for
tim e taking care of her ind oor nursery. She his first contact w ith a m ental health provid er.
reports that she is quite content w ith her life. H e rep orts that for the last 4 m onths he has
Which of the follow ing is the m ost likely d iag- been feeling d ep ressed . H is low -energy level
nosis for this p atient? and p oor m otivation are affecting his job
p erform ance and the CEO of his com p any
(A) Agorap hobia ad vised him to take a cou p le of w eeks off.
(B) Au tism sp ectru m d isord er The p atient rep orts that he started feeling
(C) Avoid ant p ersonality d isord er d ow n w hen his w ife d iscovered that he w as
(D) Schizoid p ersonality d isord er involved in his third extram arital affair. Since
then he has m oved into a sm all apartm ent by
(E) Schizotyp al p ersonality d isord er
him self. H e is sleep ing alm ost 12 hou rs every
night, has a p oor app etite, and is exp erienc-
36. An 18-year-old , p regnant, hu m an im m u no-
ing financial d ifficu lty d u e to ind iscrim inate
d eficiency virus (H IV)p ositive w om an pres-
p u rchases. H e lam ents the loss of his form er
ents for the treatm ent of op iate u se d isord er.
self. H e reports that he u sed to have several
She reports u sing heroin for the last 8 m onths
p eriod s of tim e p er year, lasting for u p to
w ith su bstantial (bu t u nsu ccessfu l) efforts to
1 w eek, w hen he only need ed 4 to 5 hou rs
qu it for the last 4 m onths. She is now hom eless
of sleep, had large am ou nts of energy, spoke
and has recently been arrested for shop lifting.
qu ickly, and cou ld p arty all night and w ork
Which of the follow ing pharm acological ther-
all d ay. Which of the follow ing d iagnoses is
ap ies w ould be the m ost ap prop riate at this
m ost likely in this case?
tim e?
(A) Bipolar I d isord er
(A) Bu prenorphine
(B) Bip olar II d isord er
(B) Clonid ine
(C) Bord erline p ersonality d isord er
(C) H alop erid ol
(D) Major d ep ressive d isord er
(D) Methad one
(E) N arcissistic p ersonality d isord er
(E) N aloxone
39. A 52-year-old w om an w ho has been treated
37. A 47-year-old w om an transfers care to you r
w ith m ed ication for 3 years for a chronic m ood
office. She has a long history of recu rrent
d isord er reports d ry m ou th, trou ble u rinating,
d epression. She also has a history since her
and occasional d izziness w hen she gets ou t
early tw enties that is significant for interm it-
of bed . Which of the follow ing m ed ications is
tent com m and au d itory hallucinations and
she m ost likely being prescribed ?
p ersecu tory d elu sions that are often p res-
ent even w hen her affective sym p tom s are in (A) Divalp roex sod iu m
rem ission. She hold s a p art tim e job and lives (B) Flu oxetine
ind ep end ently. She has been com p liant w ith (C) Im ip ram ine
m ed ications and has not been hosp italized
(D) Lithiu m
since a su icid e attem p t over 7 years ago. Which
of the follow ing is the m ost likely d iagnosis? (E) Phenelzine
Que s tions : 3545 41

Questions 40 and 41 and lau ghs w hen she cannot recall any of the three
item s you ask her to rep eat. H er attention is intact.
A 72-year-old m an is brou ght in by his w ife to you r She has som e d ifficulties nam ing objects. She states
geriatric p sychiatry clinic. The p atients w ife is con- her m ood as good and her affect is euthym ic and
cerned abou t his progressive confu sion over the last fu ll range.
year. She is p articu larly d istressed that he rep eat-
ed ly asks the sam e qu estions throu ghou t the d ay. 42. Which of the follow ing is the m ost ap propriate
H er hu sband has becom e increasingly u nstead y on p rovisional d iagnosis?
his feet and need s to u se a w alker w hen they go ou t.
She w ond ers if these sym ptom s m ay be related to (A) Deliriu m
the m eningitis he su ffered from 3 years ago. (B) Major d ep ressive d isord er
(C) Major neu rocognitive d isord er
40. Which of the follow ing sym p tom s or signs (D) Mild neu rocognitive d isord er
w ou ld m ost likely be fou nd in this p atient?
(E) N orm al p ressu re hyd rocep halu s
(A) Elevated opening p ressu re u p on lu m bar
p unctu re 43. A w orkup is p erform ed . H er laboratory val-
(B) Frontal release signs u es are u nrem arkable, and a CT scan of her
(C) H istory of incontinence brain d em onstrates d iffu se cortical atrop hy
and norm al ventricles. Which of the follow ing
(D) Ocu lom otor d ifficu lties
d iagnoses is m ost likely?
(E) Perseveration
(A) Major neu rocognitive d isord er d u e to
41. Which of the follow ing w ou ld m ost likely Alzheim er d isease
be fou nd on neu roim aging w ith com p u ted (B) Major neu rocognitive d isord er d u e
tom ography (CT) in this p atient? to Lew y bod y d isease
(A) Cerebellar atrop hy (C) Major neu rocognitive d isord er d u e
to trau m atic brain inju ry
(B) Dilated lateral ventricles
(D) Major neu rocognitive d isord er d u e to
(C) Focal su bcortical hyp ointensities
vascular d isease
(D) Frontoparietal atrop hy
(E) Unsp ecified m ajor neu rocognitive
(E) Frontotem p oral atrop hy d isord er

Questions 42 through 44 44. If the p atients CT scan revealed a p rep ond er-
A 69-year-old w om an is brou ght to you r geriatric ance of atrophy in the frontal and tem p oral
psychiatry clinic by her hu sband . She d enies any- regions, w hich of the follow ing behavioral
thing is w rong, but he is concerned that for the past m anifestations w ou ld be m ost likely?
year or so she has been less able to recall things she (A) Decline in object nam ing
read s abou t in her m agazines. She u sed to be in
(B) Mem ory p roblem s
charge of grocery shopping, but over the past year
(C) Poor organization
or so she has been leaving things off the list, and
her husband has taken over the job since the p atient (D) Social d isinhibition
cant seem to organize it anym ore. She is no longer (E) Word find ing d ifficu lty
able to keep track of the cou p les finances, and there
have been several occasions w hen her son fou nd 45. A 48-year-old m an has been d rinking u p to
her w and ering ou tsid e, a nu m ber of hou ses aw ay. 6 beers p er night d u ring the w eek and u p to
She is otherw ise healthy, and takes a m ultivitam in 12 beers a night on the w eekend . A year ago,
d aily. H er physical exam ination is unrem arkable. he had his d river s license su spend ed for
Mental status exam ination reveals a healthy ap pear- d ru nk d riving. H is m arriage is failing becau se
ing w om an w ho is coop erative w ith you r qu estions, of these d ifficulties. Last m onth, he w as d iag-
nosed w ith a gastric u lcer as a result of alcohol
42 2: Ad ult P sychop a thology

consum ption. H e ad m its to an alcohol p rob- becom es ju m p y w henever she hears the
lem and has tried to stop on nu m erou s occa- train going by her hom e. Which of the follow -
sions. H e find s that he experiences insom nia if ing d iagnoses is the m ost ap p rop riate for this
he d oes not d rink for m ore than 2 d ays. Which p atient?
of the follow ing featu res of this case su ggests a
(A) Acu te stress d isord er
severe versus m od erate alcohol u se d isord er?
(B) Ad ju stm ent d isord er
(A) H igh qu antity of alcohol consu m ed on a (C) Generalized anxiety d isord er (GAD)
regu lar basis
(D) Posttrau m atic stress d isord er (PTSD)
(B) H istory of legal problem s
(E) Major d ep ressive d isord er (MDD)
(C) Inability to stop d rinking d esp ite
know ing the harm fu l effects Questions 48 and 49
(D) Marital conflict d u e to d rinking
(E) N u m ber of d ifficu lties he is A 40-year-old w om an w ith a 20-year history of
experiencing schizophrenia p resents to the p sychiatric em ergency
d ep artm ent after a su icid e attem p t by carbon m on-
46. A 26-year-old com pu ter p rogram m er w ithou t oxid e poisoning. She has ongoing com m and hal-
p reviou s p sychiatric history has been m ar- lu cinations to harm herself, and has acted on them
ried for 4 years. H is w ife is exp ecting their at least 10 tim es since her initial d iagnosis. She also
first child . She rep orts that 3 m onths ago the has p ersistent d elu sions that she is resp onsible for
p atient becam e p reoccup ied w ith the id ea w orld d isasters, w hich is w hy she m u st elim inate
that she becam e pregnant by another m an. herself as the sou rce of pain and su ffering in the
Du ring this tim e, he began m issing w ork and w orld . She has been tried on both typ ical and atyp i-
isolated him self in his bed room . H is affect has cal antipsychotics, thou gh none have been effective
p rogressively becom e m ore blu nted . Recently, in fu lly elim inating her p sychotic sym p tom s. H er
he believes that his w ife is carrying a child level of fu nctioning rem ains p oor, and she p resently
conceived by extraterrestrial forces. H e u rged lives in a grou p hom e.
her to have an abortion and she refu sed .
The p atient d enies any history of significant 48. Which of the follow ing is this p atients m ost
alcohol or illicit su bstance u se and his recent likely lifetim e risk of su icid e?
m ed ical evalu ation w as w ithin norm al lim its. (A) 1%
Which of the follow ing is the most app ropriate (B) 5%
d iagnosis?
(C) 10%
(A) Brief p sychotic d isord er (D) 30%
(B) Delu sional d isord er (E) 50%
(C) Major d ep ressive d isord er w ith
p sychotic featu res 49. Which of the follow ing m ed ications w ou ld
(D) Schizop hreniform d isord er be the m ost ap p rop riate to p rescribe for this
(E) Schizop hrenia p atient?
(A) Clozap ine
47. A 27-year-old w om an w as involved in a train (B) Flu p henazine
d erailm ent 2 w eeks ago. Since that event, she
(C) H alop erid ol
has felt d ow n, has not slept w ell, has exp e-
(D) Lithiu m
rienced rep eated and intru sive thou ghts of
the accid ent, and has recu rrent nightm ares. (E) Zip rasid one
Lately, she has changed her com m u te to avoid
the train, even thou gh this ad d s 3 hours to her 50. A 27-year-old internal m ed icine resid ent gen-
com m ute d aily. When on the train she has an erally d islikes w orking in an ou tp atient clinic.
acu te increase in her anxiety. She also often Tod ay, how ever, he is looking forw ard to his
Que s tions : 4655 43

clinical w ork becau se one of his app ointm ents 53. In this p atient, w hich typ e of p sychotherap y
is a follow -u p visit for a single, attractive w ou ld be the m ost efficaciou s?
31-year-old w om an w ho is finishing her anti-
(A) Cognitive-behavioral therap y (CBT)
biotic regim en for treatm ent of p neu m onia.
Which of the follow ing term s bests d escribes (B) Dialectical behavioral therap y (DBT)
this d octor s resp onse to his patient? (C) Grou p therap y
(D) Psychoanalysis
(A) Cou ntertransference
(E) Psychod ynam ic therap y
(B) Em pathy
(C) Id entification Questions 54 and 55
(D) Projection
(E) Transference A 34-year-old w om an p resents for the treatm ent of
her severe, m ed ication-refractory, m ajor d epressive
Questions 51 through 53 d isord er. After review ing her p ast p sychiatric his-
tory and interview ing the patient, you conclu d e that
A 31-year-old w om an w as ad m itted to a p sychiatric she w ou ld be app ropriate for ECT.
u nit after attem pting suicid e by overd ose. She had
recently broken u p w ith her boyfriend of 4 m onths. 54. In d iscu ssing the effects of ECT w ith the
She also d escribes episod es of m ood lability, m arked patient, w hich of the follow ing should you tell
by feelings of d ep ression and anger d irected tow ard her is the m ost likely sid e effect?
the psychiatric resid ent w ho com pleted the rota-
(A) Am nesia
tion 5 d ays after her ad m ission. When the resid ent
left, she rep orted that she w as having u rges to cu t (B) Asp iration
her w rists. She has had nu m erou s ad m issions for (C) Card iac arrhythm ias
su icid al gestu res and a lifelong history of tu m u ltu - (D) Convu lsion fractu res
ou s rom antic relationships. As the patient nears her (E) Psychosis
d ischarge d ate, she rep orts that all the staff hates
m e except for Dr. Johnson. Dr. Johnson, a m ed ical 55. For the best p ossible ou tcom e, how m any
stu d ent, had a recent d ifference of opinion w ith the treatm ents is this patient likely to requ ire?
nursing staff regard ing the patients d ischarge.
(A) 2
51. Which of the follow ing d iagnoses is the m ost (B) 4
likely for this patient? (C) 10
(D) 15
(A) Bord erline personality d isord er
(E) 20
(B) Cyclothym ic d isord er
(C) H istrionic p ersonality d isord er
Questions 56 and 57
(D) Major d ep ressive d isord er (MDD)
(E) Schizoaffective d isord er A 38-year-old w om an p resents to your clinic telling
you that she has had d istu rbing, recu rrent thou ghts
52. Which of the follow ing w ou ld be the m ost abou t harm ing her 7-m onth-old infant. She im ag-
ap p rop riate ou tp atient treatm ent for this ines u sing a knife to stab her child , bu t she has no
p atient? d esire to hu rt her child . As a resu lt of having these
d istressing thou ghts, she has rem oved all sharp
(A) Antid epressants objects from her kitchen. Because of this, she has not
(B) Benzod iazepines been able to p rep are m eals at hom e and has chosen
(C) Grou p p sychotherap y to buy fast food or take ou t for the fam ily m eals. She
(D) Ind ivid u al psychotherap y feels very anxiou s regard ing these, althou gh she has
(E) Mood stabilizers not shared these thou ghts w ith her hu sband .
44 2: Ad ult P sychop a thology

56. Which of the follow ing d iagnoses w ou ld be 59. The p atient is su bsequ ently ad m itted , bu t tries
the m ost likely for this patient? to hit several staff as they tend to her; she also
rep eated ly tries to get ou t of bed and d em and s
(A) Delusional d isord er
to be let go. Which of the follow ing w ou ld be
(B) Obsessive-com p u lsive d isord er the m ost appropriate im m ed iate pharm aco-
(C) Obsessive-com p u lsive p ersonality logic m anagem ent for this p atient?
d isord er
(A) Dip henhyd ram ine
(D) Schizophrenia
(B) H alop erid ol
(E) Schizotyp al p ersonality d isord er
(C) Lorazep am
57. Which of the follow ing w ould be the m ost (D) Phenobarbital
ap propriate first-line p harm acotherap y for (E) Valp roic acid
this cond ition?
60. Which of the follow ing w ou ld be the m ost
(A) Lithium
ap p rop riate long-term treatm ent of this
(B) Lorazep am p atient?
(C) Flu voxam ine
(A) Antibiotic therap y
(D) H alop erid ol
(B) Chest x-ray every 6 m onths
(E) N ortrip tyline
(C) CT head every 6 m onths
Questions 58 through 61 (D) Ind w elling Foley catheter
(E) Intravenou s flu id s
A 72-year-old w om an is brou ght to the em ergency
d epartm ent from a nu rsing hom e for poor oral 61. Which of the follow ing is the m ost likely
intake. She is afebrile, has a pu lse of 95, and a blood 6-m onth m ortality of the d iagnosis in Qu es-
pressu re of 90/ 60. Mental statu s exam ination (MSE) tion 58?
reveals an aw ake and alert, bu t frail, m alnou rished
and d ehyd rated w om an w ho is oriented to p er- (A) 5%
son only. She rep orts that the p resid ent is George (B) 15%
Bu sh. She is easily d istracted and cannot recall any (C) 20%
of three item s after a few m inu tes. She is irritable (D) 35%
and sw ings at the staff w hen they try to insert an IV.
(E) 45%
The team starts IV flu id s as blood and u rine are sent
to the laboratory. A chest x-ray is u nrem arkable, as
Questions 62 and 63
is the head CT. One hou r later, she is calm er and
rep orts the correct d ay, tim e, and p lace; she is less You are a research psychiatrist cond u cting a d ou ble-
d istractible. blind , p lacebo-controlled trial of a new antid ep res-
sant. You have enrolled 200 p atients in the stud y, all
58. Labs are rem arkable for leu kocytosis and d irty of w hom m eet the criteria for u ncom plicated m ajor
u rinalysis. Asid e from cystitis, w hich of the d ep ression. You p lan to rand om ize 100 p atients to a
follow ing is the m ost ap p rop riate d iagnosis p lacebo m ed ication and the other 100 p atients to the
for the p atient? exp erim ental antid ep ressant.
(A) Deliriu m
62. Of the 100 p atients taking the placebo,
(B) Major d ep ressive d isord er
ap p roxim ately how m any p atients w ou ld be
(C) Major neu rocognitive d isord er d u e to exp ected to im p rove after 6 w eeks?
Alzheim er d isease
(D) Substance-ind u ced neu rocognitive (A) 5
d isord er (B) 10
(E) Unsp ecified neu rocognitive d isord er (C) 30
Que s tions : 5668 45

(D) 50 66. A 32-year-old single su ccessfu l Wall Street


(E) 70 execu tive tells you that on w eekend s he likes
to visit a d om inatrix. H is regular, paid app oint-
63. Of the 100 p atients taking the exp erim ental m ent w ith this p erson is d escribed as hum ili-
antid ep ressant (assu m ing this d ru g is as effi- ating and som ew hat p ainfu l bu t also very
cacious as stand ard antid epressants), ap proxi- sexually arou sing. While he ad m its that this
m ately how m any p atients w ou ld be exp ected behavior m ay be w eird to som e p eop le, he
to im p rove after 6 w eeks? enjoys it, can afford it, and it d oesnt interfere
w ith his w ork or other hobbies. Which of the
(A) 10 follow ing d iagnoses is the m ost ap p rop riate?
(B) 30
(A) Fetishistic d isord er
(C) 50
(B) Frotteu ristic d isord er
(D) 60
(C) N o d iagnosis
(E) 80
(D) Sexu al m asochism d isord er
Questions 64 and 65 (E) Sexu al sad ism d isord er

You r p atient is a 38-year-old d ivorced Catholic Questions 67 and 68


m ale w ith a m onth long history of d ep ressed m ood ,
anhed onia, initial insom nia, low energy, and p oor A 28-year-old w om an com p lains of falling asleep
app etite. H e ad m its to su icid al id eation w ith a d u ring the d ay. This problem has been occu rring for
plan to overd ose for the past several d ays, and he 3 m onths and is now interfering w ith her w ork as a
has gathered p ills this m orning. Up on fu rther his- telephone operator as she falls asleep tw o or three
tory, he ad m its to sim ilar sym ptom s 5 years prior, tim es a d ay w hile speaking w ith cu stom ers. At tim es,
also w ith a p rior su icid e attem p t by overd osing. H e she find s herself falling asleep at her d esk, and she
d rinks 1 beer w eekly and d enies illicit d ru gs. H e is is aw akened w hen her head hits the com p u ter con-
on no other m ed ications except for a m u ltivitam in. sole in front of her. Od d ly enou gh, she reports, this
H e is su bsequ ently ad m itted after being m ed ically can happen w hen she becom es particularly stressed
cleared . ou t, for exam ple, if she is m anaging m any calls.
The p atient also states that this d istu rbance has not
64. What p ercentage of p atients w ith this illness im p roved d esp ite her sleep ing 8 hou rs each night.
eventu ally com m it su icid e?
67. Which of the follow ing is the m ost likely
(A) 1% d iagnosis?
(B) 5%
(A) Circad ian rhythm sleepw ake d isord er
(C) 10%
(B) H yp ersom nolence d isord er
(D) 20%
(C) Insom nia d isord er
(E) 30%
(D) N arcolep sy
65. Which of the follow ing characteristics of this (E) N ightm are d isord er
patient is the nu m ber one p red ictor of a fu tu re
com p leted su icid e? 68. Which of the follow ing is the m ost app rop ri-
ate p harm acotherap y for this cond ition?
(A) Age
(B) Gend er (A) Bu p rop ion (Wellbu trin)
(C) Previous attem pt (B) Flu oxetine
(D) Relationship statu s (C) Lorazep am
(E) Religion (D) Methylp henid ate (Ritalin)
(E) Phenelzine
46 2: Ad ult P sychop a thology

69. A 28-year-old w om an presents for her annu al 71. Which of the follow ing d ru gs is m ost likely to
gynecology ap p ointm ent. She com p lains cau se this p resentation?
that in the w eek before her period , she often
(A) Alcohol
exp eriences m arked anger and irritability
and argu es m ore w ith her boyfriend . She also (B) Cocaine
reports d im inished energy and concentration, (C) H eroin
and is sleep ing m ore than is u sual for her. (D) Inhalants
These sym p tom s, in ad d ition to breast tend er- (E) Phencyclid ine (PCP)
ness and head aches, alw ays rem it in the w eek
after her m enses is finished . Which of the 72. Which of the follow ing w ould be m ost im por-
follow ing is the m ost likely d iagnosis? tant to ad m inister im m ed iately?
(A) Major d ep ressive d isord er (A) Acetylcysteine
(B) N o d iagnosis (B) Deferoxam ine
(C) Persistent d ep ressive d isord er (C) Methad one
(D) Prem enstru al d ysp horic d isord er (D) Methylene blu e
(PMDD) (E) N aloxone
(E) Prem enstru al synd rom e (PMS)
73. A 30-year-old male veteran from Operation
70. A 37-year-old accou ntant presents to the p ri- Iraqi Freed om / Operation End uring Freed om
m ary care clinic w ith com p laints of insom - presents to the m ental health clinic at the urg-
nia. H e ad m its to feeling blu e for 6 w eeks ing of his w ife. While stationed in Iraq on his
since getting p assed over for prom otion. third d eployment, his u nit w as hit by a road sid e
Since that tim e, he has had poor sleep, often bom b. H e saw a fellow Marine killed , w hile he
aw akening early in the m orning. H e also has and several other members of the comm and
had a d ecreased ap p etite w ith a 15-lb w eight sustained nonlethal injuries. Since that time, he
loss, p oor energy, gu ilt over not being good has had chronic insom nia w ith ongoing night-
enou gh, and he has been d istracted at w ork. mares of the event as w ell as occasional flash-
H e ad m its to passive su icid al thoughts w ith- backs. H e d escribes alw ays feeling on ed ge,
ou t a plan. Which of the follow ing w ou ld be avoid ing crow d s, and becoming easily startled
the likely cou rse of this patients illness if not w ith loud noises. H e ad m its to regular alcohol
treated ? use, especially w hen his sym ptoms are w orse.
(A) 3 to 6 m onths H e feels d etached from events w ith his fam-
ily, preferring to isolate himself. Which of the
(B) 3 to 7 m onths
follow ing is the most appropriate treatment to
(C) 6 to 13 m onths begin for this patient?
(D) 9 to 15 m onths
(A) Atyp ical antip sychotic
(E) 12 to 18 m onths
(B) Benzod iazep ine
Questions 71 and 72 (C) Lithiu m
(D) Serotonin-sp ecific reu p take inhibitor
A 36-year-old m an is brou ght to the em ergency (SSRI)
d epartm ent in respiratory arrest. On exam ination,
(E) Valp roic acid
he is brad ycard ic and u nresponsive, w ith constricted
p u pils bilaterally. There are no other obviou s inju -
74. A 40-year-old m arried w om an is referred by
ries on the p atient, bu t no one is im m ed iately avail-
her internist to a p sychologist for fu rther treat-
able to p rovid e collateral history. There is susp icion
m ent. She p resents an 8-m onth history of recu r-
that the patients cond ition m ay be the result of an
rent bou ts of terror, associated w ith chest
overd ose.
p ain, tachypnea, trem ors, flushing, nausea,
and fears of im p end ing d oom . These ep isod es
Que s tions : 6979 47

last for ap proxim ately 15 m inu tes and d o not (C) Unconsciou s d esire to assu m e sick role
have a particu lar trigger. As a resu lt, she has (D) Unconsciou s d esire to avoid w ork
had increasing d ifficu lty traveling far from (E) Unconsciou s conflict p rod u cing
her hom e d ue to concerns over having fu rther sym p tom s
attacks in p u blic. Despite ad equate treatm ent
w ith sertraline, she rem ains sym p tom atic and Questions 77 and 78
in significant d istress. Which of the follow ing
psychotherapies would be the most appropriate A 32-year-old m an is brou ght to the psychiatric
for her cond ition? em ergency d epartm ent by the p olice after having
been arrested for p u blic nu d ity. On m ental statu s
(A) Cognitive-behavioral therap y (CBT)
exam ination, the p atient cannot sit d ow n and is
(B) Eye m ovem ent d esensitization and only partly cooperative. H e interrup ts the inter-
rep rocessing (EMDR) view several tim es d em and ing to contact his law -
(C) Insight-oriented therap y yer, becau se m y rights given to m e by God and
(D) Interpersonal p sychotherap y ord ained by the Jeffersonians have been infringed .
(E) Su p p ortive p sychotherap y H is sister ind icates that this p atient has been treated
for at least tw o ep isod es of m ajor d ep ression in the
Questions 75 and 76 p ast, one of w hich resu lted in a su icid e attem pt. H e
is cu rrently a com p u ter p rogram m er, bu t has been
You are treating a 48-year-old m arried fem ale on increasingly stressed at w ork. One m onth ago, his
the inpatient m ed ical u nit for pyelonephritis; she girlfriend broke u p w ith him , and since then he has
has resp ond ed w ell to app ropriate antibiotic ther- been increasingly irritable. Over the p ast 2 w eeks he
apy and has been afebrile for the last 24 hou rs. You has m axed ou t his cred it card from online gam bling
inform her of likely d ischarge if she continu es to sites, w hich he has continu ed to visit over the past
im p rove. The next m orning, how ever, she com - fou r consecu tive nights. Laboratory tests show a
p lains of feeling feverish and achy, and having d ys- negative d ru g screen and alcohol levels.
u ria again. The nu rsing staff reports that she has a
su d d en fever of 103F. You treat the fever w ith acet- 77. Which of the follow ing is the m ost likely
am inophen and p erform a physical exam ination, d iagnosis?
ord er chest x-rays, d raw blood , and ord er a u ri-
nalysis w ith cu ltu re. While you are aw aiting these (A) Ad ju stm ent d isord er
resu lts, the nu rse inform s you that she w itnessed (B) Bip olar d isord er
the patient d ipping her therm om eter into a hot cup (C) Brief p sychotic d isord er
of tea before her tem p erature w as taken. (D) Cyclothym ic d isord er
(E) Major d ep ressive d isord er (MDD)
75. Which of the follow ing d iagnoses m ost likely
accou nts for this w om ans behavior? 78. Which of the follow ing w ould be the m ost
(A) Factitiou s d isord er ap p rop riate pharm acologic treatm ent for this
(B) Fu nctional neurological sym ptom p atient?
d isord er (A) Carbam azep ine
(C) Illness anxiety d isord er (B) H alop erid ol
(D) Malingering (C) Lam otrigine
(E) Som atic sym p tom d isord er (D) Lithiu m
(E) Sertraline
76. Which of the follow ing is the m ost likely m oti-
vator of this patients behavior? 79. A 68-year-old m an w ithou t p rior p sychiatric
(A) Conscious d esire to assu m e sick role history, bu t w ith a history of hypertension,
(B) Consciou s d esire to avoid w ork hyperlip id em ia, and arthritis is ad m itted for a
new left m id d le cerebral artery stroke. Which
48 2: Ad ult P sychop a thology

of the follow ing psychiatric sym ptom s w ou ld 82. A 33-year-old male w ith a history of frequent
be the m ost likely as a resu lt? fighting, aggression, impulsivity, and sui-
cid e attempts is referred to a research facility.
(A) Anxiety
H e receives multiple tests and studies. Upon
(B) Depression cerebrospinal fluid (CSF) analysis, d ecreased
(C) Mania metabolites of w hich of the follow ing neu-
(D) Obsessions rotransmitters w ould most likely be seen as
(E) Panic attacks compared to ind ivid uals w ithout his problems?
(A) Dop am ine
80. A 45-year-old w om an w ith no p reviou s p sy-
(B) Gam m a-am inobu tyric acid (GABA)
chiatric history is ad m itted to neu rology for
the treatm ent of an acu te m ultiple sclerosis (C) Glu tam ate
flare. She d oes not sm oke, d rink alcohol, or (D) N orep inephrine
u se other illicit d ru gs. She is started on ap p ro- (E) Serotonin
p riate therap y and by the third d ay her initial
low er extrem ity w eakness has resolved . H ow - 83. A 38-year-old w om an w ith m u ltip le sclerosis
ever, she also begins to act strangely, and asks of m od erate severity has had sym ptom s of
you to change her room to p revent the nin- d ep ression and m em ory loss increasing over
jas ou tsid e from creeping in and stealing m y the last year. On m ental statu s exam ination,
sou l. She also tells you the nurses have been you notice a blu nted affect and d ecreased
p oisoning her m ed icine. Which of the follow - sp eed of m ental processing. A m agnetic reso-
ing d iagnoses w ou ld be the m ost likely? nance im aging (MRI) exam ination is m ost
likely to reveal w hich of the follow ing?
(A) Ad ju stm ent d isord er
(B) Bipolar d isord er (A) Global cerebral atrop hy
(C) Psychotic d isord er d u e to m u ltip le (B) Mu ltip le p laqu es in frontal w hite m atter
sclerosis (C) N orm al brain
(D) Schizophrenia (D) Periventricu lar lacu nar infarcts
(E) Substance-ind uced psychotic d isord er (E) Ventricular enlargem ent

81. A 42-year-old m an p resents w ith a history of 84. An 80-year-old w id ow ed w om an is ad m itted


recu rrent ep isod es of m ajor d ep ression, w ith to the hospital for confu sion. On exam ina-
one prior hospitalization. For the past several tion she is som nolent, thou gh earlier in the
m onths he has felt increasingly d ep ressed , m orning she had been alert and aw are. She
w ith insom nia, low ap p etite, little energy, cry- is u ncoop erative, hostile, and hallu cinating.
ing sp ells, and p oor concentration. H e is not H er insight and m em ory are p oor. The p ri-
taking any current m ed ications. H e states that m ary team w ishes to know if she is d eliriou s
he has been on fluoxetine, p aroxetine, sertra- or d em ented . Which of the follow ing signs/
line, and venlafaxine in the p ast, and , w hile sym ptom s in this p atient is the m ost sp ecific
theyve all been efficaciou s, he has stop ped for d eliriu m ?
them d u e to significant erectile d ysfu nction.
(A) Com bativeness
Which of the follow ing m ed ications w ou ld be
the m ost ap propriate to prescribe? (B) Flu ctu ating consciou sness
(C) Poor m em ory
(A) Aripip razole
(D) Psychosis
(B) Bup ropion
(E) Uncoop erativeness
(C) Citalop ram
(D) Lithium 85. A happily m arried grad uate stu d ent com es
(E) Sertraline to you r clinic w ith com p laints of insom nia.
She has been u nable to fall asleep because
Que s tions : 8088 49

she ru m inates abou t grad es, m oney, her rela- office reporting that his m ood has been d ow n
tionship , and her you ng child ren. She has in the d u m p s, and he feels like he m ay never
alw ays been concerned abou t these, bu t lately feel better. Recently, he has been thinking that
her w orries have gotten ou t of control. She is life w as not w orth living anym ore and has
u nable to relax and feels constant tension in p assive thou ghts of su icid e. H e reports ongo-
her m uscles. While she d enies sym p tom s of ing thou ghts of overd osing on p ills. Which of
p anic attacks, she has noticed an increase in the follow ing w ould be the m ost appropriate
head aches and gastrointestinal d istu rbances next step ?
over the p ast few m onths. She d enies any
(A) Call his p arents and arrange a fam ily
p roblem s w ith m ood . She d enies any recent
m eeting
stressors, changes to her rou tine, or changes to
her hu sband s rou tine. Which of the follow ing (B) Discontinu e his SSRI and have him
is the m ost likely d iagnosis? retu rn in 1 w eek
(C) Refer the p atient to the p sychiatric
(A) Ad ju stm ent d isord er em ergency d ep artm ent
(B) Generalized anxiety d isord er (D) Sched u le w eekly ou tp atient visits
(C) Obsessive-com p u lsive d isord er (E) Sw itch the SSRI and have him follow -u p
(D) Panic d isord er w ith you in 1 m onth
(E) Social anxiety d isord er (social p hobia)
88. A 36-year-old m arried w hite fem ale p resents
86. You are a research p sychiatrist w ho is stu d ying to the em ergency room w ith a 2-m onth his-
signs and sym p tom s associated w ith certain tory of d epression, term inal insom nia, fatigu e,
p sychiatric d isord ers, and notice a category d ecreased ap p etite, anhed onia, and exces-
of patients w ho have sensory gating d eficits, sive guilt. She feels hopeless and relu ctantly
short-term m em ory d ifficulties, and abnor- ad m its to su icid al id eation for the p ast w eek,
m alities in sm ooth-p u rsu it eye m ovem ents. w ith thou ghts of taking all of m y m ed icines.
Which of the follow ing d iagnoses is m ost After fu rther qu estioning, she states that I
likely to be fou nd in this p atient p op u lation? w ou ld never d o it as she is a d evou t Catho-
lic w ho attend s chu rch regu larly. Which of the
(A) Attention-d eficit/ hyp eractivity d isord er
follow ing characteristics m ost increases this
(ADH D)
p articu lar patients risk of su icid e?
(B) Major d ep ressive d isord er (MDD)
(C) Obsessive-com p u lsive d isord er (OCD) (A) Age
(D) Posttrau m atic stress d isord er (PTSD) (B) Gend er
(E) Schizop hrenia (C) H op elessness
(D) Marital statu s
87. A 23-year-old single m ed ical stu d ent com es (E) Religion
to you r office com p laining of d ifficu lty sleep-
ing, excessive w orry abou t his stu d ies, his Questions 89 and 90
relationship w ith his p arents, and that his girl-
friend of 2 years is going to break u p w ith him A 29-year-old m arried w hite w om an w ith a p ast
d espite being happ y w ith their relationship. m ed ical history of recu rrent m igraines is brou ght to
H e feels irritable at tim es, and has significant the psychiatric em ergency d epartm ent by her hu s-
m u scle tension. Because of these sym p tom s, band w ho rep orts that, d esp ite feeling d ep ressed
his grad es have su ffered . H e d oes not rep ort 2 m onths ago and being com p liant w ith p rescribed
a d epressed m ood . You d iagnose him w ith treatm ent, she now has been acting bizarre for sev-
generalized anxiety d isord er, and p rescribe a eral d ays. On initial interview , the p atient states, I
com bination of psychotherapy and a selective- feel su p erbly su p rem e, and you have no id ea w hat
serotonin reuptake inhibitor (SSRI). Three an am azing person I am ! The p atient is talking so
m onths later, the patient com es back to you r rap id ly that you cannot interru p t her. H er hu sband
50 2: Ad ult P sychop a thology

rep orts that the p atient has not slep t in over a w eek, (D) Retrograd e
and d u ring the sam e tim e p eriod , she has p u t a d ow n (E) Selective
p aym ent for a car, has p urchased a d iam ond tennis
bracelet, and has booked an extravagant vacation. Questions 93 and 94

89. Which of the follow ing is the m ost likely A 75-year-old man comes to you r office com plain-
d iagnosis? ing of poor sleep since his w ifes d eath 1 m onth ago.
Since that time, he has been unable to fall asleep,
(A) Anxiety d isord er and he has felt d ow n. H e is slightly more isolative
(B) Bip olar and related d isord er now , as many of the activities he enjoyed d oing w ere
(C) N eu rocognitive d isord er w ith his w ife. H is appetite is d ecreased , but he is still
(D) Psychotic d isord er bathing and cooking. H e sometim es feels guilty that
(E) Som atic sym p tom d isord er she d ied before him , and is angry w ith God that he
is alive w ith no soul mate anymore. H e sometim es
90. You suspect a medication is the cause for her hears the voice of his d eceased w ife encouraging
current condition, but neither she nor her hus- him to move on, and he d enies suicid al id eation.
band recall the medication prescribed 2 months
ago. Which of the following medications is the 93. Which of the follow ing is the m ost ap p rop riate
most likely etiology? d iagnosis at this tim e?

(A) Am itriptyline (A) Ad ju stm ent d isord er w ith d ep ressed


m ood
(B) Clonazep am
(B) Bereavem ent
(C) Flu oxetine
(C) Major d ep ressive d isord er
(D) Lithium
(D) Persistent com p lex bereavem ent
(E) Sertraline
d isord er
91. A 62-year-old m ale patient w ith schizop hrenia (E) Schizop hrenia
is brou ght into the em ergency d ep artm ent by
the police for trespassing. Up on interview, he 94. Which of the follow ing w ou ld be the m ost
tells the p hysician that over the past 3 w eeks app rop riate treatm ent for this p atient?
his television has been giving him cod ed (A) H osp italize the p atient im m ed iately
m essages. Which of the follow ing term s best (B) Prescribe an antid ep ressant
d escribes this p henom enon?
(C) Prescribe an antip sychotic
(A) H allu cination (D) Refer for brief su p portive therap y
(B) Id ea of reference (E) Refer for cognitive-behavioral therap y
(C) Illu sion
(D) Thou ght broad casting 95. When asked abou t his level of ed u cation, a
(E) Thou ght insertion 48-year-old m an w ith a history of schizoaf-
fective d isord er, d ep ressed typ e, sp end s the
92. A 45-year-old p atient tells her d octor that after next 5 m inutes d escribing his high school
hearing that her hu sband d ied , she cou ld not bu ild ing, friend s he had at the tim e, clu bs he
rem em ber leaving her office and going hom e. joined , and eventu ally his high school grad u a-
In every other resp ect, her m em ory is intact. tion cerem ony. H e conclud es by saying, And
Which of the follow ing types of am nesia is that w as the end of m y schooling. Which
this an exam ple of? of the follow ing term s d oes this response best
d em onstrate?
(A) Continuou s
(A) Circu m stantiality
(B) Generalized
(B) Loosening of associations
(C) Localized
Que s tions : 89100 51

(C) Perseveration 99. A 21-year-old w om an com p lains of recu r-


(D) Pressured sp eech rent ep isod es w here she has extrem e anxiety,
(E) Tangentiality along w ith p alp itations, d izziness, nau sea,
abd om inal cram p s, d iarrhea, tingling of her
96. In her p sychiatrists office, a p atient su d d enly fingers, shortness of breath, and fearing she
low ers herself to the floor, begins flailing m ay d ie. These p eriod s last u nd er 30 m inu tes,
abou t w ild ly, then flings a garbage p ail against and theyve been increasing in frequency. She
the w all, and runs out of the office. Im m ed i- is u nable to p red ict w hen they w ill occu r, bu t,
ately afterw ard , she returns. She is alert and as a resu lt, she is often w orried abou t hav-
oriented , yet d oes not rem em ber the incid ent. ing another attack, and she has been u nw ill-
Which of the follow ing typ es of seizu res is the ing to leave her ap artm ent for several w eeks.
m ost likely? She d rinks 1 to 2 d rinks of alcohol p er m onth
and d enies illicit d ru g u se. She has no m ed i-
(A) Com p lex p artial seizu re cal problem s, is only on oral contraceptives,
(B) Jacksonian seizure and her physical and laboratory w orku p are
(C) N onep ilep tic seizu re negative. Which of the follow ing w ould be the
(D) Tem p oral lobe seizu re m ost app rop riate treatm ent?
(E) Tonicclonic seizu re (A) Bu sp irone
(B) Lithiu m
97. A 26-year-old m an new ly d iagnosed w ith (C) Lorazep am
narcolep sy exp lains that he has ep isod es of
(D) Valp roic acid
falling d ow n w ithou t any loss of consciou s-
ness, p recip itated by lau ghter or anger. Which (E) Venlafaxine
of the follow ing term s best d escribes this
sym p tom ? Questions 100 and 101

(A) Catalepsy A 46-year-old m an w ith a history of hyp ertension


(B) Catap lexy and hyperlip id em ia p resents to his p rim ary care d oc-
(C) H yp ersom nia tor w ith com plaints that Im d rinking too m u ch.
(D) H yp nagogic hallu cinations H e gives a long history of binges over 4 to 5 d ays
at a tim e, w here he w ill d rink 1 to 2 pints of gin per
(E) Sleep p aralysis
d ay. H e has had several DUIs and is at risk of losing
his job. In ad d ition, his d rinking has put a strain on
98. A 21-year-old m an w ith new ly d iagnosed
his m arriage. Unfortunately he has been u nable to
schizophrenia has been com p liant w ith his
maintain sobriety for any significant period of tim e.
med ications and is less p sychotic. At his next
H e is stressed out because of the consequences of
follow -u p appointm ent, he is noted to be rest-
his d rinking, and he ad mits to chronic, intermittent
less and constantly m oving. H e states that
insom nia. H e has a good appetite, how ever, and his
he feels as if he has to be m oving all the tim e
energy is ad equate u nless he is significantly hun-
and is u ncom fortable if he sits still. Which
gover. H e feels helpless but d enies suicid al id eation.
of the follow ing sid e effects is he m ost likely
exp eriencing?
100. Which of the follow ing m ed ications w ou ld be
(A) Akathisia m ost appropriate to prescribe for this patient?
(B) Akinesia
(A) Acam p rosate
(C) Dystonia
(B) Bu p rop ion
(D) Rabbit synd rom e
(C) Citalop ram
(E) Tard ive d yskinesia
(D) Lithiu m
(E) Olanzap ine
52 2: Ad ult P sychop a thology

101. Tw o w eeks later this sam e patient is seen (A) Atrop hy of brainstem and cerebellu m
in the em ergency room after being p icked (B) Atrop hy of frontal and tem p oral lobes
u p by the police for d istu rbing the peace (C) Enlarged ventricles
and w and ering. On exam ination he app ears
(D) Global atrop hy
confu sed , has nystagm us, and an unstead y
gait. Which of the follow ing shou ld be ad m in- (E) Periventricu lar w hite-m atter lesions
istered im m ed iately?
104. Which of the follow ing w ou ld best d escribe
(A) Glucose his clinical course?
(B) H alop erid ol
(A) Continu ed im p rovem ent
(C) Lorazep am
(B) N o im p rovem ent or w orsening
(D) N altrexone
(C) Rap id d ecline
(E) Thiam ine
(D) Stead y d ecline
102. A 36-year-old man presents to the em ergency (E) Step -w ise d eterioration
d epartm ent after being found w ithout cloth-
ing in the street. H e has m ultiple excoriations 105. An 86-year-old w om an w ith m u ltip le m ed ical
all over his bod y, is paranoid and agitated , and problem s and a recent hip fractu re is ad m itted
states that bugs are craw ling all over him. H is to the intensive care u nit. While in the u nit, she
vitals are as follow s: T-99.1, BP-160/ 93, P-105, aw akens at night and m istakes her intrave-
R-20. Which of the follow ing substances w ould nou s (IV) p ole for a fam ily m em ber com ing for
be m ost likely found in his laboratory results? a visit. She then becom es agitated , attem p ting
to pu ll out her IV and get ou t of the hospital
(A) Alcohol bed . Which of the follow ing ap p roaches is the
(B) Cannabis m ost im p ortant in the u ltim ate m anagem ent
(C) Cocaine of this patient?
(D) Opiates (A) Ad m inister d ip henhyd ram ine
(E) PCP (B) Ad m inister halop erid ol
(C) Ad m inister lorazep am
Questions 103 and 104
(D) App ly soft restraints
A 72-year-old m arried m ale w ith a history of hyper- (E) Determ ine and correct the u nd erlying
tension, bord erline d iabetes, hyperlipid em ia, and cond ition
coronary artery d isease is brou ght in by his w ife.
Over the p ast year he has becom e increasingly for- 106. A 24-year-old grad u ate stu d ent in p hiloso-
getfu l, m isp lacing his keys, getting lost w hile d riv- p hy is referred by his stu d ent health center
ing, and starting to w and er. When confronted w ith for a p sychiatric evalu ation. Althou gh he
these situations he becom es d efensive, m aking up claim s to have had sim ilar bu t attenu ated
excu ses for his behavior. H is sleep is erratic, bu t he is sym p tom s in ch ild hood , since beginn ing
eating w ell and enjoys w atching his favorite m ovies his thesis, he d escribes an acu te w orsening
on the television. H is w ife d escribes extrem e m ood of fears that he w ill contract H IV. While he
lability, w here he w ill be lau ghing and then crying. u nd erstand s the m od es and risks of contrac-
H e d enies suicid al id eation. H e d oesnt d rink alco- tion and p ractices safe sex, he is u nable to get
hol and d enies d ru g u se. H e w as a p rior sm oker bu t rid of these thou ghts. As a resu lt, he feels
qu it 20 years ago. H is p hysical exam ination and labs com p elled to w ash his hand s m any tim es p er
are u nrem arkable. d ay, even to the p oint of their becom ing raw
an d bleed ing. Desp ite his insigh t that his
103. As p art of his w orku p , an MRI of his brain concerns are irrational, he is not able to stop
is ord ered . What w ou ld be the m ost likely the behaviors. A p ositron em ission tom ogra-
find ing? p hy (PET) scan of this p atien ts brain w ou ld
Que s tions : 101111 53

m ost likely d em on strate increased activity in easily agitated , striking at nu rsing staff and
w hich of the follow ing stru ctu res? his d au ghter. Which of the follow ing m ed ica-
tions w ou ld be m ost appropriate to prescribe
(A) Am ygd ala
in this case?
(B) Cau d ate nu cleus
(C) Cerebellu m (A) Am itriptyline
(D) H ip p ocam p u s (B) Clonazepam
(E) Parietal lobes (C) Dip henhyd ram ine
(D) Donep ezil
Questions 107 and 108 (E) H aloperid ol

A 70-year-old w id ow ed m an w ithou t significant 109. A 36-year-old w om an presents w ith com -


m ed ical history is brou ght in by his d au ghter d ue to p laints of a d epressed m ood for the p ast
concerns abou t his being able to ad equately care for m onth. She reports p oor sleep, little app etite
him self. H e has been fou nd lost in the d ow ntow n w ith w eight loss, low energy, d ecreased con-
area on several occasions, attem p ting to w alk into centration, and little libid o. She ad m its to feel-
strangers hou ses, as w ell. H e is unable to cook for ing hop eless and su icid al, althou gh she d enies
him self bu t is unable to calculate the correct am ou nt a sp ecific p lan or intent. She is su bsequently
for p u rchases. H is d au ghter has noticed that he begu n on p aroxetine 20 m g at bed tim e. Which
is d izzy w hen stand ing u p and has a w id e-based , of the follow ing sym p tom s w ou ld be m ost
slow w alk. Up on interview , he ap pears d isheveled likely to im p rove the earliest?
and smells of u rine. H e is friend ly and coop erative
(A) Decreased libid o
overall, althou gh easily d istracted and confu sed .
While he know s his nam e, he believes the year is (B) Dep ressed m ood
1989. When confronted abou t his m em ory d ifficu l- (C) H op elessness
ties, he states that there are sm all gnom es living in (D) Poor sleep
his furnitu re, w ho play tricks on him by m oving (E) Su icid al id eation
item s arou nd in his cond om iniu m . Accord ing to the
d au ghter, he rarely d rinks alcohol, d oes not sm oke 110. A 35-year-old m an com p lains to his therap ist
tobacco, and has never u sed illicit d ru gs. that his new p artner enjoys sexual activity
only w hen inflicting pain on him . This d is-
107. Which of the follow ing d iagnoses is m ost turbs and fru strates the patient. Which of the
likely? follow ing best d escribes the behavior exhib-
(A) Major frontotem p oral neu rocognitive ited by his p artner?
d isord er (A) Exhibitionistic d isord er
(B) Major neu rocognitive d isord er d u e to (B) Frotteu ristic d isord er
Alzheim er d isease (C) Sexu al m asochism d isord er
(C) Major neu rocognitive d isord er w ith (D) Sexu al sad ism d isord er
Lew y bod ies
(E) Transvestic d isord er
(D) Major vascu lar neu rocognitive d isord er
(E) Su bstance-ind u ced m ajor 111. An anxiou s 23-year-old Asian m ale u niver-
neu rocognitive d isord er sity stu d ent p resents to stu d ent health ser-
vices claim ing that his p enis is shrinking into
108. The above p atient is ap p rop riately d iagnosed his abd om en. Desp ite reassu rances from the
and then placed in an interm ed iate care facil- staff and the p hysician, he rem ains convinced
ity. Eight m onths later he is brou ght by the of this belief. Which of the follow ing syn-
d aughter becau se the nu rsing hom e is having d rom es is this p atient m ost likely su ffering
increasing d ifficu lty caring for him . H e is m ore from ?
confused , resp ond ing to internal stim uli, and
54 2: Ad ult P sychop a thology

(A) Cap gras synd rom e (A) Bip olar d isord er


(B) Koro (B) Delu sional d isord er
(C) Ku ru (C) Major d ep ressive d isord er w ith
(D) Taijin-kyofu sho psychotic featu res
(E) Zar (D) Schizoaffective d isord er
(E) Schizop hrenia
112. A 21-year-old m ale college stu d ent is evalu -
ated by his college stu d ent health center after Questions 114 and 115
being arrested for m astu rbating outsid e of a
sorority w ind ow late at night. H e ad m its to A 63-year-old Vietnam veteran w ith p osttrau m atic
having w atched a particu lar fem ale stu d ent stress d isord er (PTSD) p resents to the ou tpatient
insid e the build ing over a period of several m ental health clinic. H e has been prescribed nu m er-
m onths. Which of the follow ing is the m ost ous m ed ications over the years, and he has recently
likely d iagnosis? been taking citalop ram 40 m g d aily. H e d enies sid e
effects and strongly believes this has been the m ost
(A) Exhibitionistic d isord er
effective m ed ication he has taken. While he d enies
(B) Frotteu ristic d isord er intru sive thou ghts or overt flashbacks, he continu es
(C) Gend er d ysp horia to have severe nightm ares several tim es p er w eek
(D) Transvestic d isord er w hich significantly interfere in his sleep . H e rem ains
(E) Voyeuristic d isord er ju m p y at tim es, and becau se of this d oesnt attend
fu nctions w here there are large crow d s. H e has not
113. A 45-year-old sep arated fem ale is brou ght by u sed alcohol or d rugs in 18 years, and he d enies any
her brother into the em ergency room w ith the su icid al id eation.
chief com p laint of strange behavior. She
has been living w ith her brother for the p ast 114. Which of the follow ing m ed ications w ou ld
2 years, and she stop ped her m ed ications 12 be the m ost beneficial to p rescribe for
m onths ago. The brother states that over the au gm entation?
past 6 m onths she has becom e increasingly (A) Alp razolam
paranoid and d elu sional, believing that the
(B) Bu p rop ion
neighbors are sp ying on her. She insists that
the Chinese governm ent is using her as an (C) Prazosin
agent to com bat terrorism , and that they com - (D) Risp erid one
m unicate their instru ctions via a w ireless sig- (E) Valp roic acid
nal transm itted d irectly to her brain. She is
often noticed to be talking w hen no one else is 115. In ad d ition to m ed ications, this p atient w ishes
in her room . In ad d ition, over the past m onth to pursu e talk therapy. Which of the fol-
she has been staying u p m ost of the night, p ac- low ing form s of p sychotherap y w ou ld be the
ing arou nd the hou se and attem p ting to con- m ost app rop riate to recom m end ?
struct an anti-terrorism m achine, u sing p arts
(A) Behavior therap y
of various household electronics. She states
that it is im portant that she alone com p letes (B) Cognitive p rocessing therap y
her m ission, that she is the m ost senior sp y (C) Cognitive therap y
in the organization w ith a top-secret clear- (D) Dialectical behavioral therapy
ance that only the Presid ent and I p ossess. (E) Psychod ynam ic p sychotherap y
H er brother has had a hard tim e calm ing her
d ow n as she sp eaks too qu ickly to follow, goes 116. A 23-year-old grad u ate stu d ent com es to the
from one top ic to another, and w ont sit still. psychiatric em ergency room com p laining of
Which of the follow ing d iagnoses w ou ld be anxiety. She has never been seen by a psychia-
the m ost likely for this patient? trist before and is not taking any m ed ications.
Que s tions : 112120 55

H er vital signs are notable for a heart rate of 119. A 26-year-old m an w ith schizophrenia is
110 beats/ m in. She is also slightly d iap horetic being d ischarged from his third ad m ission
and has m ild ly d ilated p u pils. Given her p res- to the p sychiatric u nit in 3 years. This m ost
ent state, w hich area of this p atients brain recent stay w as p rom p ted by acu te exacerba-
w ou ld m ost likely d em onstrate increased tion of com m and aud itory hallu cinations and
activity? paranoia that resulted in bizarre behaviors
su ch as lying in the m id d le of the road and
(A) Am ygd ala
berating cu stom ers that cost him his restau -
(B) Basal ganglia rant job. H is sym p tom s are im p roved w ith
(C) H ip p ocam p u s pharm acotherap y, w hich he is tolerating, w ith
(D) Locu s ceru leu s m ild sid e-effects of sed ation that are grad u ally
(E) Thalam u s im p roving. Which of the follow ing tim e p eri-
od s m ost accu rately d escribes w hen his risk of
117. You are asked to give a p sychiatric consu lta- su icid e is the highest?
tion on a 28-year-old w om an w ith system ic (A) At his first p sychotic break
lu pu s erythem atosu s w ho w as ad m itted to
(B) Du ring his first hosp italization
the m ed ical service. After you see her, one of
you r m ed ical colleagu es tells you that she w ill (C) In the w eek p reced ing this
no longer sp eak to any of them becau se she hosp italization
hates all of them and now insists on seeing (D) In the w eek w hile he is hosp italized
you becau se you are the best d octor in the (E) In the w eeks follow ing hosp italization
hospital. Which of the follow ing term s best
d escribes the patients behavior? 120. You are asked to review neu rop sychological
testing for a 19-year-old p atient w ho is fail-
(A) Acting out
ing classes at his local com m u nity college. H is
(B) Externalization resu lts ind icate an intelligence qu otient (IQ)
(C) Regression of 55. H e is has a grou p of p eers bu t is often
(D) Sp litting im m atu re, and his friend s often p revent him
(E) Su blim ation from being taken ad vantage of as he is very
gu llible. H e takes care of his p hysical need s
118. A 42-year-old w oman w ith recurrent episod es bu t p arents ensu re that he has enou gh grocer-
of major d epression is ad mitted to a med i- ies and need to rem ind him to check his bank
cal unit after a car accident that rend ered her accou nt balance. Which of the follow ing is the
unconscious. The patient regains conscious- m ost ap propriate d iagnosis?
ness after 3 days and corroborates that she w as, (A) Mild intellectu al d isability
ind eed , on an antid epressant, but she says she
(B) Mod erate intellectu al d isability
also cannot remember w hich one. She is started
on paroxetine (Paxil) for her d epression. Tw o (C) N o d iagnosis (norm al intellectu al
days after beginning this med ication, she fu nctioning)
develops tachycard ia, d iaphoresis, and myo- (D) Severe intellectu al d isability
clonic jerks. The neurotransmitter most likely (E) Profou nd intellectu al d isability
associated w ith the above reaction is synthe-
sized in w hich of the follow ing central nervous D IRECTION S (Questions 121 through 145): The
system structures? follow ing group of numbered items are preceded
by a list of lettered options. For each question,
(A) Cau d ate nu cleu s
select the one lettered option that is most closely
(B) Locu s ceru leus associated w ith it. Each lettered option may be
(C) N u cleu s accu m bens used once, multiple times, or not at all.
(D) Rap he nu cleu s
(E) Su bstantia nigra
56 2: Ad ult P sychop a thology

Questions 121 through 130: Match each scenario 125. A 27-year-old grad u ate stu d ent w oke u p late
w ith its most likely description. and skip p ed breakfast. She now has a m assive
head ache and is irritable w hen she w alks into
(A) Alcohol intoxication
her first m orning class. She feels like falling
(B) Alcohol w ithd raw al asleep and as if she has the flu by the end of
(C) Am p hetam ine intoxication the m orning.
(D) Am phetam ine w ithd raw al
(E) Caffeine intoxication 126. A veterinarian technician is brou ght to the
(F) Caffeine w ithd raw al em ergency d epartm ent after attacking w hat
(G) Cannabis intoxication he thought w as a cougar (it w as a hou secat).
(H ) Cannabis w ithd raw al In the em ergency room he is febrile, ap p ears
(I) Cocaine intoxication p anicked , has nystagm u s, and d em onstrates
u nexpected strength and rage.
(J) Cocaine w ithd raw al
(K) H allu cinogen intoxication
127. A 35-year-old p ilot is brou ght in for evalu ation
(L) Inhalant intoxication
becau se he has not slep t for d ays, and now he
(M) Opioid intoxication is anxiou s, tachycard ic, trem u lou s, and u nable
(N ) Opioid w ithd raw al to give coherent history. H is pu pils are d ilated
(O) Phencyclid ine intoxication and his blood p ressure is high, d esp ite norm al
valu es 2 w eeks ago on a flight physical.
121. A 32-year-old single, m ale w ith injected con-
ju nctiva cant concentrate at w ork, lau ghs 128. A 54-year-old w om an w ith arthritis com p lains
read ily at his cow orkers d ood les, feels of yaw ning, d iarrhea, abd om inal cram ps, and
relaxed , and sp eaks slow ly w hile seem ingly nausea. H er pup ils are d ilated and she has
focu sed on the air in front of him . notable piloerection.

122. A 45-year-old separated male ad mitted for 129. A 23-year-old m an is fou nd u nresponsive w ith
d epression and suicid al id eation is irritable, slow ed breathing. H e has m ultiple scars on his
asks for extra food , and spend s m ost of the first arm s and he has severe m iosis.
d ay sleeping. The chest pain he had on ad mis-
sion has subsid ed and he has no ECG changes.
130. On the third postop erative d ay you r 63-year-
old patient becom es agitated , d em and s you
123. After recess, a ju nior high school stu d ent rem ove the snakes from his room , and asks
sm ells fu nny, is stu m bling, feels d izzy and w hy it is so lou d at night (it is d aytim e). H e is
nauseated , yet rem ains sm iling and says she tachycard ic, hypertensive, and trem u lous.
feels su ch a ru sh. By the m id d le of her next
class she has a head ache but otherw ise feels
like she d id this m orning. Questions 131 through 136: Match each scenario
w ith the most likely receptor responsible from the
follow ing list.
124. A 17-year-old high school stu d ent attend s a
college fraternity party hosted by his eld er (A) 5-H T2 A recep tor
brother. After several hou rs, he feels m ore (B) Alp ha recep tor
courageou s and approaches lad ies he w ou ld (C) Beta recep tor
norm ally be too shy to engage, his w ord s are (D) Dop am ine recep tor
slightly slu rred , and he has d ifficulties m ov-
(E) H istam inergic recep tor
ing in a straight line. H e is slightly flu shed and
notes m ild m em ory problem s for events ear- (F) Mu scarinic recep tor
lier in the night.
Que s tions : 121145 57

131. A 32-year-old m an on clozapine exp eriences 138. You r p atient is angry w ith you and claim s you
constipation. are the w orst d octor ever, as op p osed to her
form er clinician, w ho actu ally listened to her.
132. A 29-year-old w om an on qu etiap ine experi-
ences d izziness on stand ing. 139. You r bord erline p atient m isses seven ap p oint-
m ents in a row resu lting in you r term inating
133. A 52-year-old m an on risperid one exp eriences her treatm ent contract.
a m ilky d ischarge from his breasts.
140. After slip p ing in front of you r boss on a fro-
134. A 41-year-old m an on haloperid ol experiences zen pu d d le, you exclaim that in a form er life
resolu tion of his au d itory hallu cinations and I w as actu ally an elite figu re skater!
his d elu sions.
141. You r 69-year-old p atient states he ju st has a
135. A 22-year-old w om an on olanzap ine exp eri- bad cold after being d iagnosed w ith m eta-
ences significant sed ation and w eight gain. static lung cancer, and states all he need s is
som e hot tea and rest.

136. A 53-year-old m an on quetiap ine exp eriences


im p rovem ent and stabilization of m ood . 142. You argu e w ith the boss at w ork, and w hen
you com e hom e you harshly groom you r cat
so that she actu ally w riggles aw ay ou t of your
Questions 137 through 145: Match each scenario arm s.
w ith its most likely defense mechanism from the
follow ing list.
143. A 68-year-old w id ow volu nteers at the local
(A) Altru ism veterans nursing hom e.
(B) Denial
(C) Disp lacem ent 144. After rep eated ly failing to bring you r p or-
(D) H u m or tion of the grou p project to class, you accu se
(E) Projection the group lead er of forgetting to e-m ail you a
(F) Projective id entification rem ind er.
(G) Reaction form ation
(H ) Rationalization 145. After not m atching into you r chosen sp ecialty,
(I) Splitting you say that it w as fu ll of boring nerd s any-
w ay and that you really have w ay too m u ch
137. A 37-year-old married male is just told by his personality to be p art of them .
wife that she has been having an affair. He
immediately hugs her and tells her he loves her.
Ans we rs a nd Expla na tions

1. (B) The patients 3- to 4-year history of bizarre w ith psychotic featu res, the prominence of
behavior, d elu sions, and d ecline in social fu nc- psychosis w ith bizarre (not possible) d elusions
tioning strongly su ggest that he has schizo- and w ithou t clear d epressed m ood m akes this
p hrenia. The p revalence of schizophrenia in d iagnosis less likely. Further, though nihilis-
the general p op u lation is ap p roxim ately 1%. tic themes and negativism can be observed in
Schizophrenia is fou nd in all societies and d epression, the d elusions are usually nonbi-
geograp hical areas arou nd the w orld , and zarre (potentially feasible).
its p revalence is rou ghly equ al in m ales and
fem ales. 4. (A) Th e belief that p eop le h ave been rep laced
by im p osters is th e h allm ark of Cap gras
2. (D ) In tw in stu d ies, schizop hrenias m onozy- synd rom e. Delu sion al d isord er is ch aracter-
gotic concord ance is 40% to 50%, suggesting ized by th e p resen ce of (u su ally) n on bizarre
that there is a strong genetic com ponent to the d elu sion s. See exp lan ation s to Qu estion 3
illness. The p revalence of schizop hrenia in the for fu rth er d efin ition s.
follow ing p op u lations is: 10% to 15% in the
nontw in sibling of a patient w ith schizop hre- 5. (A) This w om ans presentation is consistent
nia, 12% in the child of one p arent w ith schizo- w ith p sychosis w ith p erip artu m onset, char-
p hrenia, and 40% in the child of tw o p arents acterized by d elu sions and hallu cinations d u r-
w ith schizop hrenia. ing p regnancy or w ithin 4 w eeks of d elivery.
She fu rther exhibits insom nia, irritability, and
3. (B) Cotard synd rome d escribes nihilistic d elu - m ood lability. This, cou pled w ith previou s
sional content; in ad d ition to lost possessions, early d ep ressive ep isod es, strongly su ggests
patients m ay feel they have lost blood , heart, an ep isod ic m ood d isord er. Thou gh p ostpar-
intestines, as w ell as believe that the w orld tu m p sychosis can be d ue to d ifferent etiolo-
beyond them has been red uced to nothing- gies, m ost cases eventu ally m anifest as bip olar
ness. This psychotic/ d elusional theme can be d isord er. H ow ever, only a sm all fraction of
seen in m any psychotic illnesses. Capgras syn- w om en w ith bip olar d isord er w ill m anifest
d rom e is a d elusion of d oubles characterized w ith p ostp artu m p sychosis. A less frequ ent
by the belief that people have been replaced by u nd erlying etiology of postpartu m psychosis
id entically appearing im posters. Folie d eux, is MDD. Prim ary p sychotic illnesses, su ch as
or shared psychotic d isord er, is w hen a sim ilar d elu sional d isord er, schizoaffective d isord er,
d elusion is aroused in one person by the close and schizop hrenia, are even m ore rare.
influence of another; both ind ivid uals are usu-
ally closely associated for a prolonged period 6. (E) Progressive social w ithd raw al is com -
of tim e. The Fregoli d elusion is a variation of m only seen as part of the prod rom e of schizo-
the d elusion of d oubles, and is the belief that p hrenia. All other choiceshead trau m a, low
fam iliar people assume the guise of strangers. intelligence, a neglectfu l m other, and abu se
While m ajor d epressive d isord er can manifest have not been p roven in any conclu sive w ay

58
Answe rs : 112 59

to be significantly linked to schizophrenia, projective id entification, and splitting. Splitting


althou gh early theories held that a history of is d ividing external objects (ind ivid uals) into
one or m ore w as a pred isposing factor. all good or all bad categories. Altruism
(living vicariously by helping others) and sub-
7. (E) While her history strongly su ggests a p ri- limation (gratifying urges in socially acceptable
m ary psychotic d isord er such as brief p sychotic w ays) are mature d efenses, w hile intellectual-
d isord er (lasting 1 d ay1 m onth), schizo- ization (using intellectual processes to avoid
p hreniform d isord er (lasting 16 m onths), or feelings) and und oing (acts performed to und o
schizophrenia, the presence of cannabis on her obsessional thoughts) are consid ered neurotic
toxicology screen preclud es su ch a d iagnosis d efenses.
at this tim e, especially since the onset of her
cannabis u se pred ates the positive sym ptom s 10. (E) Good p rem orbid fu nctioning p ortend s a
by a few w eeks. Unless her sym p tom s p ersist better p rognosis for this p atient. Other fea-
after sobriety from cannabis is attained , given tu res of schizophrenia that pred ict a better
the prom inence of her psychotic (rather than p rognosis inclu d e later age at p resentation,
m ood ) sym ptom s, her m ost ap prop riate d iag- fem ale gend er, acute and rap id onset of sym p-
nosis at this tim e is a su bstance-ind u ced (spe- tom s (as opposed to insid ious onset), and the
cifically cannabis-ind u ced ) p sychotic d isord er. p resence of m ood sym p tom s. While pred om i-
Fu rther, if her sym p tom severity rem ains ou t nantly p ositive sym p tom s also p red ict a m ore
of proportion, she m ay eventu ally m eet crite- favorable prognosis, this patient has signifi-
ria for a com orbid p rim ary p sychotic d isord er cant negative sym p tom s, as evid enced by his
as w ell as a cannabis u se d isord er. isolation and am otivation.

8. (C) The p atient m eets criteria A for schizo- 11. (A) This patient is presenting w ith several
p hrenia: for at least 1 m onth she has exhibited catatonic featu res, includ ing negativism,
hallu cinations, d elu sions, and d isorganized psychom otor slow ing, and echopraxia, w ith
sp eech. Fu rther her sym p tom s are occu r- slight vital sign flu ctuations. Catatonia is often
ring ou tsid e of an acu te m ood ep isod e and und erd iagnosed . While catatonia w as for-
are not d u e to another m ed ical cond ition. merly consid ered a subtyp e of schizophrenia,
They have p ersisted for over a year, d esp ite it is found more frequently in affective d isor-
sobriety from cannabis. Since her p sychotic d ers, especially m ajor d epressive d isord er.
sym p tom s have lasted longer than 6 m onths, This patients history of isolation, anhed onia,
her m ost ap p rop riate d iagnosis is schizo- psychom otor retard ation, d ecreased appetite,
p hrenia. Brief p sychotic d isord er is charac- and gu ilt (burd en on fiance) are m ore su g-
terized by p sychotic sym p tom s lasting 1 d ay gestive of major d epressive d isord er. While
to 1 m onth. Schizop hreniform d isord er refers hyp othyroid ism shou ld be ru led ou t, catato-
to sym p tom s lasting m ore than 1 m onth bu t nia is not a com mon presentation. There is no
less than 6 in the absence of concu rrent m ood evid ence to su ggest that he is faking symp-
d isord er, su bstance u se, or another m ed ical toms for second ary gain, as in m alingering.
cond ition. The lack of prior psychiatric history or psy-
chosis m akes a d iagnosis of catatonia associ-
9. (C) The patient meets the criteria for bord erline ated w ith schizophrenia unlikely. There is no
personality d isord er characterized by rapid evid ence of exposu re to antipsychotics nor
mood sw ings, efforts to avoid aband onment, significant autonomic instability, so neuroleptic
chronic feelings of emptiness, intense anger malignant synd rom e is u nlikely.
outbursts, impulsivity, fluctuations betw een
idealization and devaluation, and recurrent 12. (D ) The m ainstay of treatm ent for catatonia
self-mutilation or suicid ality. People w ith is p harm acotherap y w ith benzod iazep ines
this personality d isord er commonly employ or electroconvu lsive therap y. In this case, it
primitive d efense mechanisms, such as d enial, is reasonable to attem p t a trial of lorazep am
60 2: Ad ult P sychop a thology

and assess for resp onse; if the patient begins w ou ld sp u r the d ecision for ECT if she w ere
to becom e u nstable or requ ires high d oses of currently in the postpartu m state, acutely
lorazepam (>20 m g/ d ), or if his blood p res- suicid al, and / or a threat to her infant.
sure/ pu lse d o not tolerate the titration of
lorazep am , ECT w ou ld be the ap p rop riate 15. (C) There is no absolu te contraind ication to
cou rse of action. Am itrip tyline, sertraline, or ECT. H ow ever, a sp ace-occu p ying lesion can
lithium d ep end ing on w hether his d iagno- cause d angerou s increases in intracranial pres-
sis is u nipolar or bip olar d ep ressionw ould sure. Since ECT cau ses a tem porary increase
be ap p rop riate m aintenance therap y once the in cerebral p erfu sion and the sku ll is a fixed
catatonia is treated . space (volum e), the increase in blood vol-
u m e cau ses increased intracranial p ressu re;
13. (B) This w om ans presentation is m ost con- a sp ace-occu p ying lesion red u ces the p oten-
sistent w ith a d elusional d isord er. She has no tial reserve the skull has to offset the increase
other psychotic sym p tom s, her fu nctioning is in p ressu re. Du e to the transient tachycard ia
still good , and the d elu sion is isolated to one that occu rs w ith seizures, a recent m yocar-
specific belief. In ad d ition, this patients d elu - d ial infarction is also a relative contraind ica-
sion is nonbizarre (i.e., cou ld possibly occu r). tion to ECT becau se of excess d em and s on the
While d elu sional d isord er m ay consist of a m yocard iu m . After at least several m onths,
bizarre (cou ld not occu r) d elu sion, it is not as how ever, card iac risks of ECT are the sam e as
com m on. She d oes not exhibit other psychotic those for general anesthesia. Pacem akers and
sym ptom s, m aking brief psychotic d isord er a history of traum atic brain inju ry are not con-
u nlikely; fu rther, in brief p sychotic d isor- traind ications. Pregnancy is also not a contra-
d er, sym ptom s m u st resolve w ithin 1 m onth ind ication to ECT, and the risks are the sam e
w hereas hers have been occu rring for at least as those of general anesthesia. With recent
2 m onths. There is no evid ence she has a p ara- ad vances in anesthesia, fractu res are u ncom-
noid personality d isord er. Schizop hrenia and mon since paralysis is attained for the d uration
schizophreniform d isord er are unlikely since of the ind uced seizu re.
her sym p tom s are lim ited to one d elu sion,
and she has no hallu cinations, negative sym p - 16. (B) While m any sleep d istu rbances have
tom s, nor im p airm ent in fu nctioning. In ad d i- been noted in m ajor d ep ressive d isord er (as
tion, the tim e cou rse is insufficient for these d escribed by the other choices), early m orn-
d iagnoses. ing aw akening has been the m ost consistently
linked w ith m ajor d ep ression. The d ecrease in
14. (E) This w om an has a profou nd d epression REM sleep latency and slow w ave sleep d eficits
w ith m any lifetim e ep isod es, inclu d ing sev- often persist, even after treatment.
eral severe su icid e attem p ts. She has not
resp ond ed to several m ed ications or au gm en- 17. (C) Increased cortisol secretion in patients
tation strategies. The p ersistence of su icid al- w ith d epression w as one of the earliest obser-
ity in light of the treatm ent resistance is the vations in biological psychiatry, and has been
m ost pressing reason to p ursu e ECT. ECT can w ell borne out in subsequent stud ies. Levels of
ind u ce a rap id resp onse, and has excellent MAOs are unknow n. Catecholamines, sex hor-
rem ission rates (u p to 80%), m arked ly m ore mones, and imm une function are d ecreased in
than for m ed ication alone. If her thyroid d ys- d epression.
fu nction w ere untreated , this w ould be a rea-
sonable place to initiate treatm ent, how ever 18. (B) The m ost ap p rop riate next step is to ensure
her thyroid abnorm ality has been stabilized . the safety of the patient and others, and escort-
H er prior history of psychotic sym ptom s d oes ing the p atient to the em ergency room w ith
not necessarily im p ly she w ou ld be a good p olice assistance gu arantees this is p reserved .
ECT cand id ate, bu t it is fu rther evid ence of the Discharge is inap p rop riate at this tim e as the
severity of her illness. Perip artu m d epression p atient has m ad e an active threat; w hile this
Answe rs : 1324 61

p atient is likely in a m anic episod e and the w ou ld be p ersistent d ep ressive d isord er w ith
hom icid al threat is related to the d elu sion of interm ittent d ep ressive ep isod es.
his m other-in-law , the threat m u st be taken
seriously. States d iffer in the d u ty to w arn, bu t 21. (B) This patient is now experiencing a m ajor
at this tim e in ord er to m aintain safety, ad m is- d ep ressive episod e, w hich, cou pled w ith his
sion is necessary. If on d ischarge the threat p rior hyp om anic ep isod es, w ould be consis-
rem ains, the target and law enforcem ent m ay tent w ith the d iagnosis of bipolar II d isord er.
need to be notified (there is a d u ty to w arn). See exp lanation for Qu estion 20 for rem aining
It w ou ld be inap p rop riate to have the w ife d efinitions.
stay w ith the m other-in-law as this w ould be
p utting her in harm s w ay. 22. (E) In this you ng p atient on no m ed ications,
a u rine toxicology screen is m ost likely to
19. (C) This patient is su ffering from a m anic d eterm ine the cau se of her current state, p rob-
ep isod e, and given his anteced ent treatm ent ably d u e to d ru g intoxication. Blood glu cose
w ith an antid ep ressant, p aroxetine, it is likely is im p ortant to test since hyp oglycem ia can
the antid ep ressant ind u ced the m anic sw itch. p resent w ith m ental status changes and d ia-
In ad d ition to achieving m ood stabilization p horesis; how ever, it w ou ld be u nlikely in this
through valp roate, it is critical to stop the otherw ise healthy w om an. Catecholam ine
offend ing agent. While SSRIs are occasion- m etabolites w ou ld be p rem atu re at this tim e.
ally u sed in bipolar p atients w ith d ep ressive An ECG w ou ld be p art of a w orku p to ru le
ep isod es, their u se is not recom m end ed as ou t any arrhythm ias, bu t any abnorm alities
m onotherapy d ue to this p otential to ind uce w ou ld not necessarily id entify a sp ecific etiol-
m ania. Paroxetine levels are not u sed clinically. ogy or accou nt for her d ilated pu pils. Thyroid
N ortriptyline w ould be less appropriate than fu nction tests are im portant, as thyrotoxicosis
paroxetine, as tricyclic antid ep ressants are can m anifest w ith panic-type sym p tom s, bu t
more likely than SSRIs to ind uce mania. Bupro- again, in a healthy w om an w ith an acute onset
pion is also an antid epressant and w ould not of sym ptom s, the u rine d rug screen is m ore
be appropriate at this time. Lorazepam m ay likely to yield an etiology.
help w ith agitation, bu t the more imp ortant
step is to quickly stop the offend ing agent. 23. (C) This p atient is m anifesting signs and
sym ptom s of acute stim u lant (cocaine) intoxi-
20. (C) This p atient gives a history consistent cation, includ ing tachycard ia, d iap horesis,
w ith cyclothym ic d isord er, characterized by and p u p illary d ilation. Caffeine is not rou -
ep isod es of hyp om ania and d ep ressive ep i- tinely tested on urine toxicology screens and
sod es not m eeting criteria for MDD. Treat- w ou ld not cau se su ch a severe reaction. Can-
m ent is generally the sam e as for bip olar nabis intoxication m ay p resent w ith anxiety
d isord er. For a d iagnosis of bip olar I, only and p aranoia, as w ell as conju nctival injection,
a m anic ep isod e is necessary, thou gh m ost bu t it d oes not resu lt in d ilation of the p u p ils.
p atients w ill su ffer from d ep ressive ep isod es Intoxication w ith heroin and other op iates
as w ell. Bip olar II d isord er is characterized w ill u su ally p resent w ith sed ation, resp iratory
by m ajor d ep ressive ep isod es and hyp o- d ep ression, and constricted (pinpoint) pu pils.
m anic ep isod es. Persistent d ep ressive d isor- LSD w ill often cau se p rom inent hallu cinations
d er (d ysthym ia) is a chronic d ep ression that d uring intoxication.
m ay also m eet severity for a m ajor d ep res-
sive ep isod e; sym p tom s p ersist for 2 years, 24. (B) This p atient has acu te m ental statu s
w ith no m ore than 2 m onths of sym p tom - changes, associated w ith vital sign abnor-
free p eriod s. Dou ble d ep ression is an old er m alities and hallu cinations. This presentation
term referring to ind ivid u als w ith p ersistent is su sp iciou s for acu te d eliriu m , sp ecifically
d ep ressive d isord er d evelop ing an MDD, as d elirium trem ens (DTs) d ue to alcohol w ith-
w ell. Accord ing to the DSM -5, the d iagnosis d raw al. Untreated , DTs can be lethal. The
62 2: Ad ult P sychop a thology

likelihood for onset is highest in the third to m ajor neu rocognitive d isord er (d em entia) d u e
fifth d ay after the last d rink. While postopera- to Alzheim er d isease, in w hich the frontal and
tive sep sis is another cause of d eliriu m , lack of tem poral lobes are preferentially atrophied .
fever m akes this less likely than alcohol w ith- Pu nch-d ru nk synd rom e d escribes an acquired
d raw al. Brief psychotic d isord er, functional m ovem ent d isord er associated w ith trau m atic
neu rological sym ptom d isord er (conversion d am age to the su bstantia nigra, for exam ple,
d isord er), and d elu sional d isord er d o not fit from boxing.
the acu te onset of sym ptom s w ith alterations
in the level of consciou sness and vital sign 28. (A) Kl verBucy synd rom e is m ost closely
abnorm alities. In ad d ition, fu nctional neu ro- associated w ith severe d am age to, or d iscon-
logical sym p tom d isord er d oes not m anifest nectivity of, the am ygd alae bilaterally. The
w ith hallu cinations. hipp ocam p u s, although im p ortant to short-
term m em ory, is not d irectly involved w ith
25. (C) Benzod iazepines are the m ainstay of regu lating aggressive d rives, sexu al behav-
treatm ent for d eliriu m trem ens (DTs). Of iors, or fear resp onses. The lateral ventricle
the choices, only d iazepam and lorazepam is a CSF-containing sp ace that has no d irect
are benzod iazep ines; d iazep am is less id eal neu rop sychiatric fu nctional role. The insu la,
than oxazepam becau se it has active m etabo- fou nd d eep w ithin the central sulcus, is
lites and u nd ergoes extensive m etabolism in m ed ial to the tem p oral lobe and not involved
the liver. Oxazepam and lorazepam are not in Kl verBu cy synd rom e. Su p erior tem poral
d ep end ent on liver function for their m etabo- gyri are m ore generally involved w ith p ro-
lism ; they are therefore id eal to treat DTs in cessing com plex aud itory inform ation su ch as
p atients w ith u nd erlying liver d isease. Disu l- the u nd erstand ing of language.
firam is not appropriate in the acute treat-
m ent of DTs, bu t m ay have a role in su stained 29. (E) Th is p atien t is su fferin g from an eatin g
abstinence. Phenobarbital m ay be requ ired d isord er, p ossibly an orexia n ervosa. Man y
to ind u ce sed ation in p atients w ho d evelop p atien ts w ith eatin g d isord ers h ave stron g
seizu res d espite benzod iazepine treatm ent d rives for p erfection . Psych od yn am ically,
or those w ho requ ire high d oses of benzod i- p atien ts often feel a lack of con trol an d seek
azepines (e.g., enou gh to requ ire intu bation). to con trol th e on e th in g th ey p erceive th ey
Phenytoin d oes not have a role in acu te DTs can , th at is, food . Fu rth er, p atien ts also lack
treatm ent. a sen se of valid ation from caregivers. Th ey
d on t ch aracteristically h ave legal p rob-
26. (C) Deliriu m trem ens, if untreated , has a m or- lem s, an d often are of a h igh socioecon om ic
tality rate of close to 30%. Becau se of early statu s.
recognition and im p roved treatm ents, the
m ortality rate of DTs is now app roxim ately 30. (A) This patients m ost likely d iagnosis is
5%. anorexia nervosa. She exhibits intense food
restriction as w ell as com p ensatory behaviors
27. (B) Kl verBu cy synd rom e presents w ith of over exercising to m ake u p for any caloric
d ocility, lack of fear resp onse, anterograd e intake she m ay have had . Anem ia is not u ncom -
am nesia, hyp erphagia, and hypersexu ality. m on in patients w ith anorexia, and fatigu e
Arnold Chiari synd rom e d escribes a cond i- is a frequ ent com p laint. The p resentation is
tion w ith hyd rocephalu s and cerebellar ana- not consistent w ith avoid ant/ restrictive food
tom ic and functional abnorm alities. Mbiu s intake d isord er, w hich d escribes an eating d is-
synd rom e is a congenital absence of the tu rbance regard ing food s sensory characteris-
facial nerves and nu clei w ith resu lting bilat- tics or the concern abou t ad verse consequences
eral facial p aralysis. Pick d isease is a form of of intake not related to bod y im age. In bu li-
m ajor frontotem p oral neu rocognitive d isor- m ia nervosa, p atients engage in food binges,
d er (d em entia), often ind istinguishable from consum ing large qu antities of food and later
Answe rs : 2535 63

feeling intense guilt and sham e. Patients w ith is not u su ally threatened . While this p atient
bu lim ia m ay also engage in com p ensatory has an inability to relax an associated m uscle
m echanism s, such as laxatives, ip ecac, self- tension, sym p tom s com m only associated w ith
ind u ced vom iting, and over exercise to m ake generalized anxiety d isord er, his w orries are
u p for the caloric intake. Exercise-ind u ced lim ited to the recent relationship , and his
amenorrhea is possible, but less likely given this sym ptom s have not been long-stand ing. The
patients distorted necessity to compensate for p atient d oes not m eet criteria for m ajor d epres-
any food intake. Many patients with anorexia sive d isord er at this tim e, bu t if his sym ptom s
also have obsessive-compulsive tendencies (per- p ersist or w orsen, this d iagnosis shou ld be
fectionism), but there is no indication that the consid ered . This p atients sym ptom s are m ore
patient has OCD. than w hat w ou ld be exp ected for a norm al
reactionhis sleep has been d isru p ted and it
31. (A) Benzod iazep ines are effective in acu tely is starting to affect his schoolw ork for the p ast
stopping a p anic attack su ch as d escribed in 5 w eeks.
the vignette. Alp razolam has the shortest half-
life as w ell as the m ost rap id onset of action, 33. (D ) At this tim e the p atients cond ition has
and is therefore the agent m ost likely to abort w orsened , and he now has a p ervasive
the panic attack. H ow ever, because of these d ep ressed m ood and d istu rbance in sleep,
sam e qu alities, it also has significant abu se ap p etite, concentration as w ell as associated
potential and should only be used for short- hop elessness and w orthlessness. In ad d ition
term relief. Chlord iazepoxid e is a longer-acting to w orsening self-care, his sym p tom s are now
benzod iazep ine bu t d oes not have as rap id an significantly affecting his schoolw ork and
onset of action. Divalproate is not ind icated qu ality of life. H e therefore now m eets criteria
for this p atient in the absence of m ood insta- for m ajor d epressive d isord er, single episod e,
bility. Flu oxetine, an SSRI, and p henelzine, and w arrants ap p rop riate treatm ent, w hether
an MAOI, both are ind icated for the long- w ith m ed ication, therap y, or both.
term treatm ent of panic d isord er, bu t onset
of sym ptom relief m ay take w eeks. As su ch, 34. (E) While this p atients sym p tom s are consis-
m any clinicians w ill start an SSRI bu t w ill also tent w ith MDD, her ongoing alcohol use pre-
ad d an ad ju nctive benzod iazepine for the first clu d es the d iagnosis at this tim e; therefore,
few w eeks of treatm ent, until the SSRI starts to substance-ind u ced m ood d isord er is the m ost
take effect. likely d iagnosis. If she rem ains sober for a
p eriod of tim e and her d epressive sym ptom s
32. (B) At this tim e his d iagnosis is m ost consis- continu e, she w ou ld then be d iagnosed w ith
tent w ith an ad ju stm ent d isord er, specifically MDD. Am otivation synd rom e is a controver-
w ith anxiety. Ad ju stm ent d isord er notes the sial synd rom e associated w ith cannabis u se,
onset of m ood and behavioral changes fol- and is not app rop riate in this case given the
low ing an acu te stressor. The sym p tom s rem it lack of cannabis u se. In acu te stress d isord er,
w ithin 6 m onths after the stressor. Ad ju stm ent the patient is exp osed to a trau m atic event and
d isord er can be sp ecified as occu rring w ith exp eriences reexp eriencing (e.g., flashbacks),
d ep ressed m ood , w ith anxiety, w ith m ixed nu m bing, and increased arou sal sym p tom s
anxiety and d ep ressed m ood , w ith d istu r- for u p to 1 m onth after the event. Ad ju stm ent
bance of cond u ct, w ith m ixed d istu rbance of d isord er is characterized by the onset of m ood
em otions and cond u ct, and as w ell as rem ain- and behavioral changes follow ing an acu te
ing u nsp ecified . In acu te stress d isord er, the stressor that are out of proportion to a typical
p atient is exp osed to a trau m atic event and resp onse.
exp eriences reexp eriencing (e.g., flashbacks),
num bing, and increased arou sal sym p tom s 35. (D ) Persons w ith schizoid p ersonality d is-
for u p to 1 m onth after the event. A breaku p is ord er are d etached , reclusive, d em onstrate a
not typically trau m atic in the sense that life restricted range of affect, and d o not m ind the
64 2: Ad ult P sychop a thology

lack of social interaction. Agorap hobia is tied If the hallu cinations and d elu sions only
to the fear of having panic-type sym ptom s in occu rred in the context of her d ep ressive
p ublic. Su ch sym p tom s are not m entioned in ep isod es, the m ost likely d iagnosis w ou ld
this case. Autism spectru m d isord er is a d evel- be m ajor d ep ression w ith p sychotic featu res.
opm ental d isord er characterized by d eficits While schizop hrenia w ou ld be on the d iffer-
in social com m u nication and w ith rep etitive, ential, her affective (d ep ressive) sym p tom s
restricted behaviors and interests. Persons are p rom inent. Schizop hreniform d isord er
w ith avoid ant p ersonality d isord er are shy is characterized by sym p tom s of schizop hre-
and fearful of social rejection. H ow ever, their nia, w hich last from 1 to 6 m onths, so it is
lack of socialization is d istressing to them . not ap p rop riate given the chronicity of her
Schizotypal ind ivid uals can have schizoid fea- sym p tom s.
tu res but they also have bizarre thinking and
an od d or eccentric m anner. 38. (B) This p atients history is consistent w ith a
m ood p attern d efined by p eriod s of hyp om a-
36. (D ) Methad one m aintenance is the m ost nia (sym p tom s of m ania not severe enou gh
ap p rop riate p harm acological therap y for the to cau se occu p ational d ysfu nction or p sychi-
treatm ent of op iate (heroin) u se d isord er, atric hosp italization) and cu rrently a m ajor
even in p regnancy. Methad one m aintenance, d ep ressive ep isod e. H yp om ania w ith m ajor
esp ecially w hen com bined w ith p sychoso- d ep ression d efines bip olar II d isord er. In
cial services and obstetric m onitoring, sig- bip olar I d isord er, the m ania is m ore severe,
nificantly im p roves neonatal ou tcom es and cau sing notable occu p ational d ysfu nction,
obstetric ou tcom es for w om en ad d icted to p sychotic sym p tom s, or hosp italization.
heroin. The low est effective d ose shou ld be While this p atient is in the m id st of a m ajor
u sed d u ring p regnancy. There are no know n d ep ressive ep isod e, his history of hyp o-
teratogenic effects from m ethad one. Wom en m ania ind icates a bip olar d iagnosis; this is
m ay requ ire higher d oses of m ethad one in an im p ortant d istinction becau se im p rop er
the third trim ester d u e to increased m etabo- treatm ent w ith antid ep ressants can p recip i-
lism of the d ru g at that tim e. Bu p renorp hine tate a m anic ep isod e. Bord erline p ersonality
is a p artial op iate agonist that has a role in d isord er is characterized by a p ersistent p at-
m aintenance treatm ent, bu t its safety d u ring tern of u nstable relationship s, m ood states,
p regnancy and breastfeed ing has not been and self-im age. N arcissistic p ersonality d is-
d efinitively established , and its u se in p reg- ord er is characterized by a p ervasive p attern
nancy m ay lead to w ithd raw al sym p tom s in of grand iosity, need for ad m iration, and lack
op ioid -d ep end ent fetu ses. Clonid ine can be of em p athy.
u sed to treat hyp ertension and other sym p -
tom s d u ring acu te op iate w ithd raw al. While 39. (C) Dry m ou th, d izziness (associated w ith
halop erid ol (a first-generation antip sychotic) hypotension), and u rinary hesitancy are d u e
m ay be u sed safely in p regnancy, it has no u se to anticholinergic and antiad renergic effects
for the treatm ent of severe op iate u se d isor- of tricyclic antid ep ressants (TCAs) su ch as
d er, u nless agitation from acu te w ithd raw al im ip ram ine. Divalp roex sod iu m is a m ood
is p roblem atic. N aloxone is an op iate antago- stabilizer that m ay cau se gastrointestinal
nist, u sed for op iate overd ose, not ap p rop ri- u p set and is com m only associated w ith sed a-
ate in this case. tion and trem or at higher d oses. Fluoxetine, a
serotonin-sp ecific reup take inhibitor (SSRI),
37. (C) This p atient exhibits ep isod ic m ood sym p - is m ost often associated w ith gastrointestinal
tom s as w ell as chronic p sychotic sym p tom s u p set, sexu al d ysfu nction, and activation/
ou tsid e of those m ood ep isod es, m aking her agitation. Lithiu m is another m ood stabilizer
m ost likely d iagnosis schizoaffective d is- u sed in bip olar d isord er, m ost often caus-
ord er. There is no evid ence of p rior m anic ing p olyu ria, p olyd ip sia, trem or, and m en-
ep isod es, m aking bip olar d isord er u nlikely. tal confusion at higher d oses. Phenelzine, a
Answe rs : 3644 65

m onoam ine oxid ase inhibitor (MAOI), can be 43. (A) The most common type of major neurocog-
associated w ith hypotension bu t is less likely nitive d isord er (d ementia) is d ue to Alzheimer
to have anticholinergic effects. d isease, w hich demonstrates diffuse atrophy
on neuroimaging. Major neurocognitive d is-
40. (C) This patient likely has normal pressure ord er d ue to Lew y bod y d isease is clinically
hydrocephalus (NPH), one of the few poten- similar to Alzheimers, but also includ es hallu-
tially reversible causes of neurocognitive dis- cinations, parkinsonian features, and extrapy-
order (dementia). The classic triad of NPH is ramid al signs. Major neurocognitive disorder
confusion, gait apraxia, and incontinence. Ele- d ue to traumatic brain injury is characterized
vated opening pressure is not found in NPH by emotional lability, dysarthria, and impulsiv-
(thus the normal pressure part of the d iag- ity. Major neurocognitive d isord er due to vas-
nosis). Frontal release signs and perseveration cular d isease (formerly multi-infarct d ementia)
are nonspecific findings common in demented is the second most common type of demen-
patients but not specific to NPH. Oculomotor tia, and involves small w id espread lacunar
difficulties are a part of the WernickeKorsakoff infarcts. It is seen in those w ith hypertension or
syndrome. other card iovascular d isease; this patient d oes
not have either cond itions. Unspecified major
41. (B) In N PH , CT scan com m only reveals d ilated neurocognitive d isord er is reserved for patients
ventricles thought to be the resu lt of increased in w hom the precise etiology cannot be deter-
p ressu re w aves im p inging w ithin the ventric- mined w ith sufficient certainly to make the
u lar system . Cerebellar atrop hy is seen m ost appropriate attribution. All the major neuro-
often in congenital d isord ers and alcoholism . cognitive d isord ers must be d istinguishes from
H ypointensities found in subcortical areas are major d epression; pseud od ementia refers
often ind icative of lacu nar strokes. Frontopa- to a cond ition w here the patient demonstrates
rietal atrop hy is seen in m ajor neu rocognitive symptoms consistent w ith memory d ifficulties,
d isord er d ue to Alzheim er d isease. Fronto- but in the absence of a dementia; it is often seen
tem poral atrophy is fou nd in Pick d isease, an in d epression. Patients w ith pseud od emen-
u ncom m on type of frontotem p oral neu rocog- tia usually have more d epressive symptoms,
nitive d isord er (d em entia). have more insight into their symptoms than
d emented patients, and w ill often have a prior
42. (C) This p atient is m ost likely su ffering from history of d epression.
m ajor neu rocognitive d isord er (d em entia),
characterized by progressive im p airm ent of 44. (D ) Major frontotem p oral neu rocognitive d is-
cognitive fu nction in the absence of d eliriu m . ord er (form erly Pick d isease) is characterized
Mem ory im pairm ent and loss of fu nction in by p referential atrop hy of the frontotem p oral
at least one other cognitive d om ain are hall- region, as op p osed to the p arietotem p oral d is-
m arks. While som e loss of cognitive fu nction tribu tion seen in Alzheim er d isease. Patients
is exp ected w ith age (m ild neu rocognitive m ay exhibit either a behavioral variant or lan-
d isord er d oes not interfere w ith capacity for gu age variant, bu t m any have featu res of both.
ind ep end ence), it is too severe and this p atient Patients w ith the behavioral variant m ust
is no longer able to fu nction ind ep end ently. show three or m ore behavioral sym ptom s
Deliriu m is characterized by acu te m ental sta- (behavioral d isinhibition, ap athy or inertia,
tu s change w ith d eficits in attention (i.e., fluc- loss of sym p athy or em p athy, p erseverative/
tu ating levels of consciousness). While m ajor stereotyped / ritualistic behaviors, or hypero-
d ep ressive d isord er can present w ith neuro- rality and d ietary changes) or a p rom inent
cognitive d eficits (pseu d od em entia), there d ecline in social cognition and / or execu te
is no ind ication of d ep ression. This p atient abilities. Patients w ith the langu age variant
d oes not d em onstrate the gait apraxia and display a decline in language ability (difficulties
u rinary incontinence characteristic of norm al in speech production, naming, word finding,
p ressu re hyd rocephalu s. comprehension).
66 2: Ad ult P sychop a thology

45. (E) The DSM -5 d istingu ishes the severity of anxiety last for at least 6 m onths. The patient
alcohol (and other substance) u se d isord ers has no evid ence of a primary d epressive illness
on the basis of how m any criteria are m et. Cri- such as MDD.
teria A includ e evid ence of a problem atic pat-
tern of alcohol u se that resu lts in im pairm ent 48. (C) The lifetim e incid ence of su icid e in
and / or d istress. Patients m eeting 2 to 3 sym p - p atients w ith schizop hrenia is ap p roxim ately
tom s m eet criteria for m ild alcohol u se d is- 10%, compared w ith less than 1% in the general
ord er; those that have 4 to 5 sym ptom s m eet population.
criteria for m od erate alcohol u se d isord er;
those that m eet 6 or m ore of the possible listed 49. (A) Clozapine, the first atypical antipsy-
11 sym ptom s can be d iagnosed w ith severe chotic, has been show n to d ecrease su icid -
alcohol u se d isord er (form erly alcohol d ep en- ality in patients w ith schizop hrenia and is
d ence). The DSM -5 d oes not d istingu ish abu se p articu larly effective in treatm ent-refractory
from d ep end ence, and requ ires the nu m ber of p atients, such as the one in this exam p le. Clo-
p roblem atic sym ptom s to d eterm ine severity. zap ine requires regu lar blood m onitoring to
This p atient d rinks alcohol in large am ou nts, screen for the potentially seriou s sid e effect of
has had unsu ccessfu l efforts to cu t d ow n, has agranu locytosis. N one of the rem aining anti-
cravings, has had persistent interpersonal p sychotics, such as flu phenazine, haloperid ol,
p roblem s d ue to d rinking, has had recu rrent or ziprasid one, have been show n to be effec-
p hysical problem s d ue to d rinking, and has tive in treatm ent-refractory schizophrenia or
had w ithd raw al sym p tom s; he m eets at least to d ecrease su icid ality. While lithiu m has also
six criteria and therefore can be d iagnosed been show n to d ecrease su icid ality, its u se is
w ith alcohol u se d isord er, severe. not ind icated in this case given the lack of any
affective sym p tom s.
46. (D ) This p atient w ould m eet the criteria for
schizophrenia excep t that the d u ration of 50. (A) Feelings and attitudes originating from
his illness has been less than 6 m onths. Brief the clinician, evoked by the patient, are called
p sychotic d isord er m ay present w ith psycho- countertransference. Transference d enotes feel-
sis bu t the d uration of the d istu rbance m ust ings and attitud es about the physician coming
be 1 m onth or less. This patients d elu sions from the patient. Empathy is the ability of the
are too bizarre to typ ically be consistent w ith clinician to psychologically put oneself in the
d elu sional d isord er, and the age of onset for patients shoes and thereby und erstand the
d elu sional d isord er is u su ally m id d le age. patients thinking, feelings, or behavior. Id en-
There is no evid ence of a d ep ressive d isord er tification is a d efense mechanism characterized
at this tim e. by the unconscious incorporation of some-
one elses traits into ones ow n mannerfor
47. (A) This patient has sym p tom s consistent w ith example, ad olescents having hairstyles similar
PTSD or acute stress d isord er (i.e., trau m atic to ad mired rock stars. Projection is a primi-
event, intrusive m em ories of trau m a, avoid - tive d efense mechanism w hereby one assigns
ance of rem ind ers, hypervigilance), bu t the emotions to another person in an attempt to
fact that the trau m a w as only 3 w eeks p rior psychologically d efend against (cover up) the
m eans that she w ou ld be d iagnosed w ith acu te presence of those emotions w ithin oneself; for
d istress d isord er. For PTSD, there m u st also be example, the d octor in the case may tell his fel-
evid ence of negative cognitions or m ood s. If low resident that he believes that his female
her sym ptom s d id p ersist after 4 w eeks, then patient is attracted to him, w hen in actuality he
she w ou ld be d iagnosed w ith PTSD. Ad just- is attracted to her.
m ent d isord er m ost often represents a change
in m ood , anxiety, or cond u ct that hap p ens 51. (A) This patient d isplays symptom s of fear of
after a nontraum atic event. In GAD, there is loss and aband onment, intense interpersonal
no associated trau m a, and the sym ptom s of relationship s, recurrent suicid al behavior or
Answe rs : 4556 67

threats, affective instability, and d ifficulty w ith groups. DBT is an offshoot from CBT and
controlling intense anger, consistent w ith focuses on mind fulness and distress tolerance.
bord erline personality d isord er. Patients CBT is often a short course of therapy focused
w ith cyclothymic d isord er often present w ith on d ysfunctional thoughts w ith their result-
similar sym ptoms of mood changes d u e to ing behavioral and emotional manifestations.
hyp om anic ep isod es and m inor d epressive Group therapy is a component of DBT, but
episod es, bu t they are not u sually as frequent, should be used in conjunction w ith, not in place
short-lived , or intense, and they are not asso- of, ind ivid ual therapy. Psychoanalysis and
ciated w ith the other symptom s of bord erline psychod ynamic therapies utilize psychoana-
personality d isord er above. Patients w ith his- lytic principles and focus on formative child -
trionic personality d isord er, like those w ith hood experiences and the recurrent patterns of
bord erline personality d isord er, m ay d isplay behaviors in interpersonal relationships, unre-
excessive emotionality and attention seeking, solved feelings, and build ing of ego d efenses.
but their core symptoms center around super- While these therapies may be of benefit, the
ficial sed uctiveness and theatricality. While regression and intense emotions evoked may
patients w ith bord erline personality d isord er be too d ifficult for the patient to handle.
may have d ep ressive sym ptoms that d evelop
into MDD, this patients d epressive sym ptoms 54. (A) Amnesia is a comm on sid e effect of ECT,
are fleeting and in resp onse to stressors such as presenting in the form of short-term memory
perceived aband onment. Althou gh this patient d eficits. Aspiration, fractures, and arrhyth-
ap pears p aranoid that all the staff hates me, mias are rare sid e effects. Psychosis can be
there is no other evid ence for a primary psy- improved , rather than w orsened , w ith ECT; in
chotic illness su ch as schizoaffective d isord er. fact, ECT is a treatm ent of choice in patients
Of note, how ever, patients w ith bord erline w ith m ajor d epressive d isord er w ith psychotic
personality d isord er may experience tem po- features.
rary psychotic symptoms, such as paranoia, or
severe d issociation u nd er times of perceived 55. (C) For m ajor d ep ressive d isord er, 6 to 12
stress or substance u se. sessions are generally optim al for a good
riskbenefit ratio of p ositive treatm ent effects
52. (D ) Psychotherap y and stead y social su p p ort versus m em ory im p airm ent (w hich is m ore
rep resent the best long-term m ethod for the likely w ith greater than 20 treatm ents). For
m anagem ent of bord erline p ersonality d is- catatonic cond itions, tw o or fou r ECT treat-
ord er. Antid ep ressants, antip sychotics, and m ents m ay be effective. When ECT is u sed
m ood stabilizers d o have som e efficacy in for psychosis or m ania, a larger num ber of
treating target sym p tom s in som e p atients treatm ents m ay be necessary.
w ith bord erline p ersonality d isord er, bu t
the long-term m ainstay treatm ent is p sycho- 56. (B) This p atient has d istressing recu rrent and
therap y. Med ication can be a u sefu l ad ju nct intru sive thou ghts (obsessions). She attem p ts
in m anaging sym p tom s su fficiently so that to ignore or su p p ress her sym p tom s by not
the p atient can later engage m ore effectively exp laining to her hu sband how she feels and
in p sychotherap y. Benzod iazep ines m ay by getting rid of all the knives in her kitchen.
treat anxiety in these p atients, bu t they have Obsessions are the m ental acts, w hereas com -
p articu lar abu se liability in these im p u lsive p u lsions are p hysical actions p erform ed to
p atients. d ecrease anxiety associated w ith the obses-
sions. Obsessive-com p u lsive p ersonality d is-
53. (B) Psychotherapy is the mainstay of bord er- ord er is a p ervasive character style m arked by
line personality disord er treatment. Of the a p attern of p reoccu p ation w ith ord erliness,
available types of therapy, DBT has show n to p erfectionism , and m ental and interp ersonal
be particularly effective w ith borderline indi- control. These ind ivid u als are often d escribed
vid uals, especially w hen used in conjunction as control freaks by layp ersons and are
68 2: Ad ult P sychop a thology

noted to be p articu larly inflexible. Delu sional present w ith a more stable MSE than is illu s-
d isord er or schizop hrenia are inap p rop ri- trated in this case; in ad d ition, there is no evi-
ate d iagnoses becau se this p atient is not d ence that this w om an is d epressed . Although
p sychotic; her w orries are conceived of as a this p atient w ou ld be likely to have a neuro-
p rod u ct of her m ind and not as im p lanted cognitive d isord er (d em entia) as she cam e
m essages, as m ay occu r in schizop hrenia. from a nu rsing hom e, her flu ctu ating m ental
There is no m ention of a history of od d beliefs statu s is ind icative of a d elirium . Dem entias
or strangeness p rior to the birth of her child , are d iagnosed in the context of a relatively sta-
w hich w ou ld be consistent w ith schizotyp al ble set of d eficits on MSE, and the u nd erlying
p ersonality d isord er. causes are rarely associated w ith acu te m ed i-
cal insu lts. There is no evid ence of a su bstance
57. (C) Flu voxam ine and other serotonin-sp ecific or other cause of a m ajor neurocognitive d is-
reu p take inhibitors (SSRIs) (e.g., p aroxetine, ord er (d em entia). Im portantly, how ever, the
flu oxetine, sertraline, citalop ram ) are know n p resence of a m ajor neu rocognitive d isord er
for their antiobsessional effects as w ell as for (d em entia) d oes p red ispose a patient to a
their antid ep ressant and antianxiety effects. d elirium .
Patients m ay requ ire higher d oses than those
need ed for d ep ression. Lithiu m is a m ood sta- 59. (B) Polypharm acy can w orsen d eliriu m . Anti-
bilizer u sed for bip olar d isord er. Lorazep am , p sychotics su ch as halop erid ol can be used
a benzod iazep ine, m ay d ecrease anxiety bu t to calm an agitated patient w ith d eliriu m .
has no d irect antiobsessional effects. H alop er- Anticholinergic agents like d ip henhyd ram ine
id ol is a typ ical high-p otency antip sychotic shou ld be avoid ed as they can w orsen or cause
(neu rolep tic) that m ay be u sefu l in severe d elirium , and sed ating m ed ications should
form s of OCD in w hich there is a p sychotic be u sed at low est p ossible d oses. Benzod iaz-
com p onent. H ow ever, it d oes not am eliorate ep ines su ch as lorazep am shou ld be avoid ed
p u rely obsessional thinking. Becau se of the in m ost cases of d eliriu m (w ith the excep tion
im p ortance of serotonin in the treatm ent of of sed ative-hypnotic/ alcohol w ithd raw al) as
obsessive-com p u lsive d isord er, of the tricy- they can d isinhibit patients and w orsen the
clic antid ep ressants (TCAs), clom ip ram ine behavioral p roblem s. In cases of d eliriu m
w ou ld be p referred if this class w ere selected . w ith very severe agitation, p henobarbital m ay
H ow ever, TCAs are not first-line treatm ent be u sed , bu t this w ou ld not be the ap p rop ri-
for OCD. ate m anagem ent in this case. Valp roic acid ,
an anticonvu lsant and m ood stabilizer used
58. (A) This patients presentation and examina- in bip olar d isord er, is not ind icated in this
tion are m ost consistent w ith d eliriu m, as evi- p atient.
d enced by a fluctuating mental status w ith
attentional d eficits. Any num ber of m ed i- 60. (A) The d efinitive treatm ent for d eliriu m is to
cal cond itions can cau se d eliriu m, and the treat the und erlying cau se, in this case, treat-
eld erly are particularly p rone. This w oman is m ent of the u rinary tract infection w ith anti-
eld erly, d ehyd rated , and w ith an active infec- biotics. Mental statu s need s to be m onitored
tion; any one of these can increase the risk of carefu lly as som e antibiotics can also w orsen
d elirium. Urinary tract infections are a fre- d elirium . Sched u led chest x-rays or CT scans
qu ent cau se of altered m ental statu s in the have no role in treatm ent of this patients con-
eld erly. A chest x-ray can also elucid ate the d ition and carry u nnecessary rad iation expo-
presence of infection, and a head CT can rule sure. An ind w elling Foley catheter w ould
ou t acute intracranial pathology, both of w hich increase the risk of d evelop ing a u rinary
can cause d eliriu m. While major d epressive tract infection, and unless there is ind ication,
d isord er can cause cognitive d eficits on shou ld be avoid ed . Any associated electrolyte
MSEs that are reversible w ith antid epressant abnorm alities that m ay have resulted from the
treatment, these pseud od ementias usu ally p atients p oor oral intake need to be corrected ,
Answe rs : 5769 69

bu t intravenou s flu id s w ill not, by them selves, sexu al sad ism d isord er, the p atient is aroused
treat the infection or the d eliriu m . by giving such punishment (or fantasies of giv-
ing it) to others. Fetishistic d isord er involves
61. (C) Deliriu m carries a high-m ortality rate and sexu al arousal connected to nonliving objects.
is a poor p rognostic sign. The 1-m onth m ortal- In frotteuristic d isord er, the patient is sexually
ity rate of p atients w ho have had one ep isod e arou sed by touching or ru bbing u p against a
of d eliriu m is u p to 14%, and the 6-m onth nonconsenting person. All are examples of
m ortality rate is app roxim ately 22%. parap hiliaspersistent sexual interest other
than sexual interest in genital stimu lation or
62. (C) Abou t one-third of all p atients w ith preparatory fond ling w ith norm al, m atu re,
d ep ression respond to the su rp risingly pow - consenting hu man partners.
erfu l p lacebo effect. H ow ever, this resp onse is
not as w ell su stained as in p atients placed on 67. (D ) N arcolepsy is a d isord er often affecting
antid epressant m ed ications. p ersons in their teens or tw enties. It is a d is-
ord er of REM sleep m echanism s, character-
63. (D ) Stand ard antid epressant therapies, inclu d - ized by catap lexy (su d d en loss of m u scle tone
ing SSRIs and TCAs, have a p ositive resp onse follow ing intense em otion), hyp nopom pic or
in 55% to 65% of patients. hypnagogic hallu cinations, or sleep p aralysis
(an inability to p erform volu ntary m ovem ents
64. (C) This p atient likely has m ajor d ep ressive either at sleep onset or aw akening that can
d isord er. The rate of com pleted suicid e in be terrifying). Circad ian rhythm sleep w ake
hosp italized p atients w ith m ajor d ep ression d isord er is a d isord er cau sed m ost often by
is app roxim ately 10% to 15%. Most p atients sleep-sched u ling changes su ch as night-shift
w ith m ajor d ep ression w ill have som e d egree w ork. H yp ersom nolence d isord er d escribes
of su icid al id eation, and u p to 40% of p atients self-reported excessive sleepiness d espite
w ith m ajor d ep ression w ill attem p t su icid e. m ain sleep p eriod of su fficient tim e. Insom nia
Screening for su icid e shou ld be perform ed d isord er notes a com plaint of d issatisfaction
in any p atient w ith d ep ression, and stu d ies w ith sleep qu ality and qu antity. N ightm are
have show n that asking abou t su icid e d oes not d isord er is d iagnosed in the absence of other
p lant the thou ght in p atients; m ost p atients p red isp osing m ental illness (e.g., PTSD), in
w ho have attempted suicide have seen a doctor w hich the p atient has rep eated nightm ares
w ithin 4 w eeks of the attempt. causing significant d istress.

65. (C) The nu m ber one p red ictor of a com p leted 68. (D ) Methylphenidate, a stimulant, has been
suicid e is a previous attem pt. Old er single (or shown to be useful in inhibiting the onset
d ivorced ) m ales are m ore likely to attem p t. of narcoleptic episodes, likely owing to the
Religion, good su pport, and child ren can be capacity for the drug to enhance CNS arousal
p rotective factors. Concu rrent su bstance u se mechanisms that inhibit REM-related mecha-
and anxiety d isord ers can increase the risk of nisms. Antidepressant treatment with tricyclic
suicid e. antidepressants has been found to be useful
in combination with methylphenidate in some
66. (C) There is no evid ence of occupational or patients with narcolepsy. However, the efficacy
social d ysfu nction or marked d istress caused of bupropion, fluoxetine (a serotonin-specific
by this patients activities. Virtually all DSM -5 reuptake inhibitor), and phenelzine (a mono-
d iagnoses are qualified by these criteria. If the amine oxid ase inhibitor) has not been well stud-
patient d id d escribe su ch concerns, the d iag- ied in narcolepsy. Benzodiazepine regimens are
nosis w ould m ost likely be sexu al masochism not particularly effective for narcolepsy.
d isord er, in w hich patients are aroused by psy-
chologically or p hysically pu nishing acts by 69. (D ) This p atients sym p tom s fit DSM -5 crite-
another person (or fantasies of pu nishm ent). In ria for PMDD. The rem ittance of sym p tom s in
70 2: Ad ult P sychop a thology

the w eek after m enses is typical of PMDD. In consid ered to be the SSRIs. Antip sychotics,
ad ults, persistent d epressive d isord er is char- inclu d ing the atyp icals, shou ld not be u sed
acterized by su bthreshold d epressive sym p - as m onotherap y, althou gh m ay be used in
tom s, for m ore d ays than not, over a 2-year conjunction, especially if there are associated
p eriod . H er sym p tom s are not severe enou gh p sychotic sym p tom s. Benzod iazep ines shou ld
nor of sufficient d u ration (2 or m ore w eeks) be avoid ed in this p atient p op u lation not only
to d iagnose MDD, and her sym p tom s are becau se of the significant com orbid ity w ith
m ore d isru ptive and cau se m ore d istress than substance ad d iction bu t also as they have not
p urely PMS. been fou nd to be p articu larly effective. Mood
stabilizers, su ch as lithium and valproic acid ,
70. (C) This p atient is d isp laying sym p tom s con- are occasionally u sed in ad d ition to SSRIs in
sistent w ith m ajor d epressive d isord er, single ord er to target the m ood lability or aggression
ep isod e. The average u ntreated d ep ressive som etim es seen in PTSD, althou gh they are
episod e lasts anyw here from 6 to 13 m onths; not as effective as SSRIs.
w ith treatm ent, the d u ration d ecreases to
ap proxim ately 3 m onths. Treatm ent can also 74. (A) Th is case is an exam p le of p an ic d isord er
d ecrease the risk of recu rrence in the future. and agorap hobia inad equ ately treated w ith
ap p rop riate m ed ication, n am ely sertraline.
71. (C) H eroin overd ose is m ost likely to have Man y m ental illnesses, inclu d in g an xiety
caused the clinical situ ation d escribed , inclu d - d isord ers, are best treated w ith a com bina-
ing the d ecreased level of consciou sness, tion of p harm acology an d p sych oth erap y.
resp iratory d ep ression, and p inp oint (con- The best-stu d ied p sychotherap y for p anic
stricted ) pu pils. Alcohol intoxication and PCP d isord er is CBT. Eye m ovem ent d esensitiza-
m ay cause com a bu t both are associated w ith tion and rep rocessing (EMDR) is a sp ecific
nystagm u s rather than pu p illary size changes. th erap y that has been d evelop ed for PTSD.
Cocaine u se causes pu pillary d ilatation, not Insight-oriented therap y is a long-term
constriction, and is usu ally associated w ith d ynam ic therap y. Althou gh it m ay be u se-
agitation in large am ou nts. Inhalants m ay fu l in som e cases of p anic d isord er, it has not
rarely cau se com a bu t are not classically asso- been as valid ated as CBT. Interp ersonal ther-
ciated w ith pu pillary constriction. ap y ad d resses relation ship s as a contribu tor
of d ep ression and is u sed to treat ind ivid u -
72. (E) N aloxone is u sed to reverse the acu te effects als w ith m ajor d ep ression. The p u rp ose of
of opiate overd ose by blocking CN S opioid su p p ortive p sychoth erap y is to stren gthen
recep tors. Acetylcysteine is ad m inistered in th e p atien ts d efen se m ech anism s in ord er to
acetam inop hen overd ose and d eferoxam ine is retu rn them to a p reviou s level of fu n ction-
u sed in iron overd ose. Methad one is u sed for ing. It has not been ad equ ately stu d ied for
the long-term m aintenance of opiate ad d iction p anic d isord er.
and w ou ld only w orsen the sym p tom s of her-
oin overd ose. Methylene blue is u sed to treat 75. (A) This case d ep icts a patient w ho is feign-
m ethem oglobinem ia. ing or p rod u cing sym p tom s of an illness to
gain gratification by assu m ing the sick role.
73. (D ) This p atient is su ffering from sym p tom s Factitiou s d isord er is som etim es referred to
of posttrau m atic stress d isord er (PTSD), as Mu nchau sen synd rom e; w hen caretakers
m anifesting in som e p atients follow ing the intentionally cau se illness in their charges,
exp osu re to threatened or actu al d eath, severe such as child ren, the d isord er is called fac-
inju ry, or violence, and consisting of intru - titiou s d isord er im posed on another. In
sion sym ptom s, avoid ance of related stim uli, fu nctional neu rological sym ptom d isord er
negative alterations in m ood and cognitions, (conversion d isord er), p atients present w ith
and increased au tonom ic arou sal. The best neu rologic sym p tom s that are not consistent
stu d ied and m ost efficaciou s m ed ications are w ith know n anatom y or p hysiology, and the
Answe rs : 7080 71

concern is u sually w ith a specific sym ptom , 78. (D ) Lithium and valproic acid , along w ith
not having a p articu lar d isease. In illness anxi- atyp ical or second -generation antip sychot-
ety d isord er, p atients have few som atic com - ics (e.g., olanzap ine) are consid ered first-line
p laints bu t are p rim arily concerned w ith the treatm ents for acute m ania, as w ell as preven-
id ea of being sick and having or acqu iring a tion of fu tu re m ood episod es. Carbam azepine
serious illness; the intensity is not so severe as is also an effective m ood stabilizer, bu t d u e to
to w arrant a d iagnosis of d elu sional d isord er. sid e effects is not u su ally first line. While halo-
Som atic sym ptom d isord er is characterized by p erid ol can be u sed to treat the acu te agitation
m u ltip le, cu rrent, som atic sym p tom s that are and p sychotic sym ptom s in bip olar d isord er,
d istressing or result in significant d isruption. it is not as effective for the m ood com p onent
There is no reason to believe this act cou ld or in preventing ep isod es of m ania, and its
help her achieve financial or other m aterial sid e effects (e.g., extra-pyram id al sym p tom s,
gain as in m alingering. tard ive d yskinesia) lim it its use as m onother-
ap y. Lam otrigine and lithiu m are both u sed in
76. (C) Patients w ith factitious d isord er have an the treatm ent of acu te d epressive episod es of
u nconscious d esire to assu m e the sick role, bip olar d isord er. Antid ep ressants su ch as ser-
althou gh their sym p tom p rod u ction is fu lly traline should be avoid ed in this case as they
conscious. In m alingering, both sym ptom can w orsen m ania and likely prom ote rapid
p rod u ction and the d esire for second ary gain cycling of m ood episod es.
(w ork avoid ance) are consciou s. In functional
neu rological sym ptom d isord er, the sym ptom 79. (B) Classically, infarcts of the left frontal hemi-
p rod u ction is believed to be d u e to u ncon- spheres (part of left mid d le cerebral artery d is-
scious conflicts. tribution) present w ith d epression, w hereas
those of the right frontal hem isphere present
77. (B) This patient m eets criteria for a current w ith euphoria, inappropriate ind ifference, or
m anic episod e w ith sym p tom s of p sychom o- m ania (althou gh recent research tend s to refute
tor agitation, p ressu red speech, grand iose this). Obsessive-com pulsive behaviors pres-
d elusions, flight of id eas, and history of exces- ent occasionally after d iffu se bilateral frontal
sive spend ing as evid enced by his cred it card injury. Anxiety and panic sym ptoms have not
d ebt. H is history of good prem orbid function- been d escribed as having any particular asso-
ing and a rem itted m ajor d ep ression are also ciation w ith left mid d le cerebral artery strokes,
consistent w ith a d iagnosis of bipolar d isor- although su ch comorbid cases m ay exist.
d er. Ad justm ent d isord ers can p resent follow -
ing an id entifiable stressor and m ay m anifest 80. (E) This w om an has d eveloped psychosis
w ith m ild m ood , anxiety, or behavioral d istu r- m arked by hallu cinations, d elusions, and
bances, bu t not overt m ania. Brief p sychotic paranoia. She w as m ost likely given high-d ose
d isord er is characterized by psychotic sym p- steroid s to treat her acute multip le sclerosis
tom s lasting 1 d ay to 1 m onth, bu t w hich are flare, and the steroid s ind uced a psychotic
not better accou nted for by another m ood d is- state. H igh-d ose steroid s can cause m ood d is-
ord er w ith psychotic featu res (su ch as bipolar turbance in many patients, and florid psycho-
d isord er) or psychotic d isord er. Cyclothym ic sis in a sm aller subset. The symptom s u sually
d isord er is incorrect as this patient has had entirely remit once steroid s are stopped or
both m anic and d ep ressive ep isod es, w hereas tapered appropriately. While most symptom s
in cyclothym ia the highs d o not exceed a hyp o- occur at high d oses, such as w ith IV steroid s,
m anic ep isod e, and the low s d o not extend to steroid psychosis can occur in patients treated
a m ajor d epressive episod e. The patients cu r- w ith chronic low er level oral steroid s as w ell.
rent m anic ep isod e ru les ou t MDD; in fact, H er psychotic symptom s rule out an ad just-
althou gh he m ay have futu re m ajor d ep res- m ent d isord er, and her age and lack of psychi-
sive episod es, he should never be d iagnosed atric history w ould not be consistent w ith a first
w ith MDD. episod e of bipolar d isord er or schizophrenia.
72 2: Ad ult P sychop a thology

While m ultiple sclerosis can, by itself, prod u ce and u ncooperativeness m ay be seen in either
d epression and affective lability, it d oes not d elirium or major neurocognitive d isord ers
usu ally cau se psychotic sym ptoms. (d em entias), a fluctuation in the level of con-
sciou sness (i.e., from alertness to som nolence)
81. (B) Sexu al d ysfu nction is a com m on sid e effect w ith d eficits in attention are the hallm arks for
of m any psychotropics, especially antid epres- d elirium. Attention is usually intact in neuro-
sants such as SSRIs. Wellbu trin, a non-SSRI cognitive d isord ers (d em entias). Fu rther, the
antid epressant, likely cau ses a low er incid ence acute onset of sym ptoms is more su ggestive
of sexu al d ysfu nction. Given his num erous tri- of d eliriu m. It is important to remember that
als and incid ence of erectile d ysfu nction w ith having a major neu rocognitive d isord er pre-
SSRIs, it is d oubtfu l that retrying sertraline or d isposes a patient to d elirium , so both m ay be
sw itching to another SSRI (e.g., citalopram ) seen in the sam e patient.
w ill be less likely to cau se sexu al d ysfu nc-
tion. Aripip razole (an atypical antip sychotic) 85. (B) This p atient is su ffering from generalized
and lithium (a m ood stabilizer) can be u sed anxiety d isord er. H er sym ptom s inclu d e rum i-
in au gm enting p artially effective antid ep res- nations abou t m any asp ects of her life, d espite
sants, bu t w ould not be app ropriate first-line there not being any d isruption in those areas.
m onotherapy in this case. She also has physical sym ptom s associated
w ith anxiety. There is no p articu lar stressor
82. (E) The serotonin m etabolite d esignated to qu alify for ad ju stm ent d isord er. H er ru m i-
5-hyd roxyind ole acetic acid (5-H IAA) m ea- nations m ay have an obsessional qu ality, bu t
sured in the CSF has been show n to be low er they are not lim ited to a specific thou ght nor
in p ostm ortem analysis of victim s of su icid e associated w ith any com p ensatory com p u l-
and patients w ith im pu lsivity and violence sions, therefore m aking obsessive-com pu lsive
or aggression w hen com pared w ith control d isord er u nlikely. She d oes not have specific
grou p s. N o su ch associations have been estab- p anic attacks or fears of fu tu re attacks con-
lished w ith the other neu rotransm itters listed . sistent w ith panic d isord er. H er anxiety is
not related to fears of hu m iliation or scru tiny
83. (B) Depression is the m ost com m on present- in a p erform ance situ ation, w hich w ou ld be
ing p sychiatric sym p tom in p atients w ith consistent w ith social anxiety d isord er (social
m u ltiple sclerosis. It m ay present d u ring the p hobia).
cou rse of m ild progressive cognitive d ecline
that is consistent w ith a subcortical neuro- 86. (E) There are a nu m ber of stigm ata associated
cognitive d isord er (d em entia) (i.e., slow ing of w ith schizop hrenia that are not inclu d ed in
m ental p rocessing, m otor d ifficu lties, blu nted DSM -5 d iagnostic criteria. These inclu d e soft
or inappropriate affect, w ith relative preser- neu rologic signs su ch as short-term m em ory
vation of langu age-d epend ent skills). N eu- d eficits, u nstable sm ooth-p ursu it eye m ove-
roim aging of these p atients u su ally show s m ents, and d ecreased ability to habitu ate
d iffuse w hite m atter plaqu es affecting the to rep eated sensory stim u li (sensory gating
frontal lobes. Cerebral atrophy and ventricu- abnorm alities). In ad d ition, p atients w ith
lar enlargem ent are associated w ith the m ost schizophrenia have d ifficu lty in conceptual-
com m on form of m ajor neu rocognitive d isor- izing com p lex visu al com p ositions. Patients
d er (d em entia), Alzheim er d isease. Periven- w ith ADH D, MDD, OCD, and PTSD have also
tricu lar w hite-m atter changes are observed d em onstrated short-term m em ory d ifficu lties,
in m ajor vascu lar neu rocognitive d isord er bu t not the other signs m entioned .
(form ally m u lti-infarct d em entia).
87. (C) The m ost ap propriate next step is to send
84. (B) The patient appears to be suffering from the patient to the p sychiatric emergency
d elirium. Although aggressiveness or combat- d epartment. This patient is experiencing an
iveness, m emory d eficits, psychotic symptom s, acute episod e of major d epression and , becau se
Answe rs : 8192 73

of his suicid al id eation w ith active plan and processes and speech) in the absence of mood
means to overd ose, he should be referred to symptoms. H er primary symptoms d o not
the emergency d epartment for further assess- involve physical or somatic symptoms, w hich
ment w hile ensu ring his safety. Stopping his w ould be seen in a somatic symptom disord er.
prescription w ill not ad d ress the im med i-
ate d anger to self, and having him follow u p 90. (A) This p atient w as m ost likely p rescribed
in 1 w eek or for w eekly outpatient visits are an antid ep ressant since her initial presenta-
also inappropriate given the imm ed iate safety tion w as for a d ep ressive ep isod e 2 m onths
concerns. While his p arents may be called for ago. Since she also has m igraines, and tricyclic
collateral information in the fu tu re, the more antid ep ressants are often u sed in their treat-
pressing step is to ensu re the patients safety. m ent, this w as m ost likely the agent she has
H e may not have respond ed to the SSRI or been taking. While all antid ep ressants have
have been in the minority of you ng ad u lts w ho the potential to ind u ce a m anic sw itch in sus-
exp erience w orsening of su icid al thou ghts on cep tible patients, the rate is slightly higher for
the SSRIs; either w ay, a sw itch in pharmaco- tricyclic antid epressants than for serotonin-
therapy is likely necessary, bu t w ill not resolve specific reu ptake inhibitors or serotonin-
his acu te suicid ality, and is therefore inappro- norep inep hrine reup take inhibitors, such as
priate as the next imm ed iate step. fluoxetine or sertraline, respectively. Benzo-
d iazepines (e.g., clonazepam ) are not know n
88. (C) The w om an p resents w ith a classic p resen- to cau se m ood sw itches into m ania, and m ood
tation for m ajor d ep ressive d isord er. H aving stabilizers such as lithiu m are a treatm ent for
a d epressive d isord er significantly increases bip olar d isord er.
ones risk for suicid e, and feelings of hopeless-
ness have been fou nd to be a particu lar risk 91. (B) The specific phenomenon described here is
factor. Many other factors also increase the an id ea of reference. The patient interprets an
risk for com p leted su icid e su ch as w hite race, event as relating to him, even though it clearly
m ale gend er, old er age, single/ d ivorced , and d oes not. A hallucination is a perception in the
having poor social su p p ort. absence of a clearly defined stimulus, w hereas
an illusion is a misinterpretation of an evident
89. (B) The presentation exemplifies a manic epi- stimulus (for instance, IV tubing appearing like
sod e, w ith symptoms that includ e inflated snakes). Thought broad casting is a d elusional
self-esteem and grand iosity, pressured speech, belief that ones thoughts can be heard or some-
a d ecreased need for sleep, and an impulsive how know n by others w ithout d irect commu-
shopping excursion w ithout consid ering pos- nication. Thought insertion is another d elusion
sible consequences. The most likely d iagnosis w here the patient believes thoughts from some
is a type of bipolar d isord er, as this patient external entity are placed in his mind.
has a history of d epression and now is manic.
How ever, before diagnosing bipolar d isord er 92. (C) The choices in this qu estion d escribe d if-
type I, the contribution of mania ind uced by ferent patterns of am nesia. Localized am nesia
an antid epressant must be ruled out. If this refers to m em ory loss su rrou nd ing a d iscrete
manic episod e occurred because of an antid e- p eriod of tim e, typ ically occu rring after a trau -
pressant, a more appropriate d iagnosis w ould m atic event; in this case, the patients am nesia
be substance-induced mood disorder. Anxiety resu lts from learning of her hu sband s d eath.
d isorders and neurocognitive d isord ers do not Rarely, a patient m ay forget his or her entire
present w ith the classic signs of mania. While p reced ing life (generalized am nesia) or forget
this patient has psychotic symptoms (grand i- all events follow ing a trau m a, excep t for the
ose d elusions), a psychotic d isord er such as im m ed iate p ast (continu ou s am nesia). Ret-
schizophrenia cannot be d iagnosed as there is rograd e am nesia is any am nesia for events
no ind ication of prominent psychotic symp- that com e before a trau m atic event. Selective
toms (hallucinations, d isorganized thought am nesia involves the inability to recall certain
74 2: Ad ult P sychop a thology

asp ects of an event, thou gh other m em ories of p hrase, or id ea is perseveration. Pressured


the event m ay be intact. speech is a continuou s, usu ally rap id , u ninter-
ru p tible stream of id eas that m ay m ake sense
93. (B) This p atient is su ffering from bereavem ent. bu t is often hard to follow . While this p atients
H is w ife has d ied w ithin the last 2 m onths, speech m ay be pressured , this is unable to be
and his sym p tom s inclu d e an ap prop riately d eterm ined by the vignette alone. Tangen-
d ep ressed m ood , althou gh he is still able to tiality occu rs w hen the patient strays off the
fu nction and is taking care of his basic need s. p oint entirely, never retu rning to the original
It is not u ncom m on to have au d itory or visu al intention of the answ er. Generally, the sp eech
hallu cinations of the d eceased d u ring the griev- or thou ght p rocesses of a schizophrenic or
ing p eriod . Also, m any ind ivid u als exp erience schizoaffective patient m ay d em onstrate any
survivors gu ilt as part of their grieving. While of these characteristics.
this patient had a significant stressor (d eath of
w ife), his sym p tom s are better accou nted for 96. (C) A nonepileptic seizu re (formerly called
by bereavem ent rather than ad ju stm ent d is- p seu d oseizu re) is a psychogenically ind u ced
ord er, since the sym ptom s are not ou t of pro- behavior that resembles epileptic activity
p ortion to w hat w ou ld be expected . If he had externally, bu t the EEG is entirely norm al.
suicid al id eation, severe loss of functioning, Althou gh the synd rom e may in some cases be
significant d istressing au d itory hallu cinations motivated by second ary gain (as in m alinger-
or other sym ptom s of psychosis, a d iagnosis ing), it most often occurs in reaction to stress
of m ajor d epression w ith or w ithou t p sychotic and represents a type of functional neurologi-
featu res w ou ld be m ore appropriate. Persis- cal sym ptom (conversion) d isord er. The other
tent com plex bereavem ent d isord er is pro- choices, w hich each represents true epileptic
p osed as a cond ition for fu rther stu d y, and seizu res, rarely, if ever, includ e the purpose-
w ou ld d escribe severe bereavem ent p ersisting ful, com plex activity as is seen in this case.
over a year after d eath. Besid es the hallu cina- N ot uncomm only, patients w ith conversion
tions relating to the d eceased w ife, there is no d isord er (functional neurological symptom
other evid ence of a prim ary psychotic illness d isord er) w ith seizures also have a concurrent
such as schizop hrenia. It is im portant to note seizu re d isord er, and it is quite d ifficult to sort
that bereavem ent is not a psychiatric d isord er. ou t the epileptic seizu res from the nonepilep-
tic ones. Diagnosing a nonepileptic seizu re
94. (D ) The m ost ap p rop riate treatm ent at this often requ ires d emonstrating a norm al EEG at
stage is su pp ort. If sym ptom s persist, and he the time of the event. The prognosis for non-
begins to exp erience loss of fu nctioning or epileptic seizures is qu ite p oor, w ith frequent
significant d istress, an antid epressant m ay be exacerbations.
w arranted . H osp italization w ou ld be ind icated
if this p atient had active thou ghts of su icid e 97. (B) N arcolep sy is a sleep d isord er character-
and w as at risk of harm ing him self or oth- ized by the tetrad of hyp ersom nia (excessive
ers. At this tim e, he is exp eriencing su rvivors d aytim e som nolence), cataplexy (transient
gu ilt bu t is not actively su icid al. There is no loss of m otor tone associated w ith strong
ind ication for an antip sychotic, and cognitive- em otions, as d em onstrated in this case), sleep
behavioral therap y is not ind icated in this p aralysis (total or p artial p aralysis in sleep
case. w ake transition), and hyp nagogic or hyp -
nop om p ic hallu cinations (vivid d ream -like
95. (A) Circum stantiality involves a circuitou s, hallu cinations occurring in the w ake/ sleep
over-inclu sive answ er that only eventu ally transition). Catalepsy is a state of im m obility
gets to the p oint as the p atient d em onstrates som etim es seen in catatonic states.
in this exam p le. Loosening of associations is a
resp onse com p osed of a series of d isconnected 98. (A) All of the choices are possible sid e effects,
id eas. The p ointless rep etition of a w ord , either short term or long term, of antipsychotic
Answe rs : 93103 75

med ication. Akathisia is the su bjective sensa- thiam ine m ay w orsen the encep halopathy.
tion of motor and mental restlessness. Akinesia H aloperid ol, a typical antipsychotic, is not
is a d ysfunction of slow ed or absent m ovement ind icated as the p atient is neither p sychotic
that can be associated w ith p seud op arkinson- nor agitated , and w ill not help the u nd erly-
ism in the setting of antipsychotics (neurolep- ing cond ition. N either lorazep am , a benzo-
tics). A d ystonia, like akathisia and akinesia, d iazepine u sed in acu te alcohol w ithd raw al,
can occu r acutely and involves mu scle rigid ity nor naltrexone, an op iate antagonist u sed to
and spasticity. Rabbit synd rome is a late-onset red u ce alcohol craving, w ill ap p rop riately
sid e effect that involves fine, rhythmic m ove- treat Wernicke encephalopathy.
ments of the lips. Tard ive d yskinesia is another
late-onset neurologic effect of antipsychotics 102. (C) This p atient p resents w ith d isorganized
(neuroleptics) and can includ e choreoathetoid behavior, agitation, p aranoia, and form ication
movements of the trunk or limbs, as w ell as (a particu lar type of tactile hallu cination in
other areas su ch as the tongue and lips. w hich one has the sensation of bu gs craw ling
on or u nd er the skin), as w ell as elevated vital
99. (E) This p atient is likely su ffering from both signs and d ilated p upils. These are all consis-
p anic d isord er and agorap hobia. The m ost tent w ith acute cocaine intoxication. Intoxi-
effective and tolerated m ed ications for p anic cation w ith alcohol p resents w ith d ecreased
d isord er are SSRIs (e.g., sertraline) or SN RIs level of consciou sness, d ecreased vital signs,
(e.g., venlafaxine). Busp irone, a 5-H T1 A slurred speech, and poor coord ination. Can-
recep tor p artial antagonist, can be effective nabis intoxication can p resent w ith p aranoia
in generalized anxiety d isord er bu t not p anic and tachycard ia, althou gh not u sually agita-
d isord er. Lithiu m and valproic acid are m ood tion or form ication. Opiate use classically
stabilizers m ost often u sed in bipolar d isord er. p resents w ith a d ecreased level of consciou s-
Lorazep am and other benzod iazep ines can be ness, d im inished vitals, p inp oint p u pils, d ry
help ful in treating anxiety, bu t should be m in- skin, and constipation. Patients w ho use PCP
im ized long-term d u e to the risk of ad d iction. m ay be qu ite agitated and p aranoid , as w ell
as have signs of sym p athetic stim u lation,
100. (A) This patient is suffering from alcohol u se bu t they often p resent w ith ataxia, fever, and
d isord er. While rehabilitation is the m ainstay nystagm u s (vertical, horizontal, or rotary).
of treatm ent for ad d ictions, several m ed ica-
tions have been beneficial in red ucing alcohol 103. (E) Given his card iovascu lar history, risk fac-
u se. Acam p rosate, a glu tam ate recep tor m od - tors, and clinical presentation, this patient
u lator, has show n efficiency in red u cing crav- m ost likely has m ajor vascu lar neu rocogni-
ing and alcohol u se. Antid ep ressants (e.g., tive d isord er (d em entia), the second m ost
bu p rop ion, citalop ram ), m ood stabilizers (e.g., com m on cau se of d em entia after Alzheim er
lithiu m ), and antip sychotics (e.g., olanzap ine) d isease. Becau se the etiology is com m only
have not been d em onstrated to red u ce alcohol d ue to m u ltiple infarcts, periventricu lar w hite
u se and are not ind icated in the absence of a m atter (lacu nar) infarcts m ay be seen on MRI.
com orbid m ood or psychotic d isord er. Atrop hy of the brainstem and cerebellu m m ay
be seen in cases of Parkinson d isease. Fronto-
101. (E) This p atient ap pears to be experiencing tem poral neu rocognitive d isord er (d em entia),
Wernicke encephalop athy, a resu lt of chronic, com m only referred to as Pick d isease, is a
heavy, alcohol u se, cau sed by acu te thiam ine com m on cause of d em entia in you nger ind ivid -
d eficiency. It p resents classically w ith the u als and d isp lays a p referential atrophy of the
triad of confu sion, oculom otor abnorm alities frontal and tem p oral lobes on MRI. Enlarged
(e.g., nystagm u s), and ataxia. The treatm ent is ventricles on MRI of the brain are consistent
IV thiam ine. While glu cose is often given in w ith neu rocognitive d isord er (d em entia) d u e
an em ergency setting, as w ell, it is im p erative to norm al p ressu re hyd rocephalu s, w hereas
to give thiam ine first, as giving glucose before cases of m ajor neurocognitive d isord er d ue
76 2: Ad ult P sychop a thology

to Alzheim er d isease m ay show general or lobes, and cingu lu m . These d ifferences are can
global atrop hy of the cerebru m . be reversed after ad equ ate p harm acologic or
cognitive-behavioral therapy. The other areas
104. (E) The cou rse of m ajor vascu lar neu rocogni- of the brain are not believed to be as involved
tive d isord er (d em entia) is typically a step- in OCD.
w ise d eterioration, corresp ond ing w ith the
continu ed infarcts occu rring to the patient. 107. (C) This p atient likely has m ajor neu rocogni-
Most, although not all, d em entias are irre- tive d isord er (d em entia) w ith Lew y bod ies, a
versible, and so continued im p rovem ent is com m on cau se of d em entia that is sim ilar to
u nu su al; how ever, m ajor neurocognitive Alzheim er d isease. Com m on sym p tom s are
d isord er (d em entia) d ue to trau m atic brain typical for and overlap w ith Alzheim ers, but
inju ry m ay not show im p rovem ent nor w ors- as in this case they have ad d itional sym ptom s
ening over tim e (assu m ing no fu rther insu lts). of urinary incontinence, orthostatic hypoten-
Rapid d ecline m ay be seen in cases of m ajor sion, visu al hallucinations, and parkinsonism .
neu rocognitive d isord ers d u e to p rion d isease Major frontotem poral neurocognitive d isor-
(e.g., Creu tzfeld tJakob d isease, ku ru ), w hile d er (d em entia) w as originally called Pick d is-
stead y d ecline is typical for m ajor neurocog- ease. While not rare, it is a com m on cau se of
nitive d isord er (d em entia) d u e to Alzheim er early-onset d em entia, and often p resents first
d isease. w ith behavioral d ifficu lties, inclu d ing ap athy,
m ood lability, d isinhibition, and d epression.
105. (E) This p atient is exp eriencing d eliriu m . As ind icative of the nam e, im aging d em on-
Deliriu m is a synd rom e characterized by a strates a p referential atrophy of the frontal
d istu rbance in attention and cognition. While and tem p oral lobes. The above associated
d elirium is a m ed ical em ergency and behav- sym ptom s (e.g., visu al hallucinations and
ioral control is im p ortant, the m ost im p or- p arkinsonism ) are not u su ally seen. Patients
tant app roach is to id entify and correctly w ith m ajor vascu lar neu rocognitive d isord er
the u nd erlying cau se(s). Com m on etiologies (form erly vascu lar d em entia) have a history of
inclu d e m etabolic, infectiou s, hyp oxia, organ card iovascular d isease, hypertension, strokes,
failure, and d rug intoxication/ w ithd raw al. etc., and often have ad d itional neu rological
Behavioral control m ay be necessary w ith a find ings not consistent w ith Lew y bod y d is-
high-potency antip sychotic su ch as halop eri- ease. There is no evid ence in the history for
d ol or soft restraints. Dip henhyd ram ine and any su bstance-ind u ced m ajor neu rocognitive
other anticholinergic m ed ications shou ld be d isord er (d em entia).
avoid ed as they can w orsen d eliriu m . Benzo-
d iazepines shou ld also be avoid ed (excep t in 108. (D ) The pharm acologic behavioral m an-
cases of alcohol or sed ative/ hypnotic w ith- agem ent of m ajor neu rocognitive d isord er
d raw al-ind uced d eliriu m ) as they can cau se (d em entia) w ith Lew y bod ies is tricky. Anti-
d isinhibition and w orsening agitation in m any cholinesterase inhibitors su ch as d onepezil
p atients. have d em onstrated efficacy in these p atients
and are w ell tolerated . While antip sychotics
106. (B) This p atient m ost likely su ffers from are often u sed (d esp ite the increased risk of
obsessive-com pulsive d isord er (OCD), a d is- m ortality) to treat agitation and / or p sychosis
ord er characterized by recu rrent obsessions in p atients w ith m ajor neu rocognitive d isor-
(e.g., fear of contracting H IV) and / or com - d ers, high-p otency agents like haloperid ol
p ulsions (e.g., frequent hand w ashing), w hich are p articu larly concerning given the sensitiv-
cause significant d istress or im p airm ent in ity and p otential for w orsening p arkinsonian
fu nctioning. The neu rophysiologic nature of sid e effects. Med ications w ith anticholinergic
this illness can be d em onstrated throu gh neu- sid e effects, such as tricyclic antid epressants
roim aging, w hich show s increased activity (e.g., am itrip tyline) and antihistam ines (e.g.,
(m etabolism ) in the cau d ate nu cleus, frontal d iphenhyd ram ine) shou ld be avoid ed as they
Answe rs : 104114 77

m ay w orsen the d em entia. Clonazep am and teuristic d isord er is the synd rom e of recu r-
other benzod iazepines shou ld also be avoid ed rent, intense sexu al fantasies and behaviors
given the significant risk of d isinhibition. involving tou ching or ru bbing against a non-
consenting ad u lt. Gend er d ysphoria is a cond i-
109. (D ) While antid epressants su ch as parox- tion w here there is incongru ence and d istress
etine help all the sym p tom s of d ep ression, betw een ones exp ressed gend er and assigned
som e respond qu icker to m ed ications. Sleep, gend er. Transvestic d isord er involves being
energy, and ap p etite changes are u su ally d ressed in clothing of the opposite sex for
the first to resp ond , follow ed later by libid o, sexual excitem ent; it is often present in hetero-
hop elessness/ helplessness, and su icid al id e- sexual m en and d iffers from gend er d ysphoria
ation. Because of the d ifferential resp onse (transsexu alism ) in that the person is u su-
tim es, it is im portant to m onitor a suicid al ally com fortable and content w ith his gend er
p atient as their energy and m otivation m ay id entity. It is, how ever, p ossible to see these
im p rove before the d ep ression and su icid al- d isord ers concom itantly.
ity, and thu s, the p atient m ay have the energy
to com m it su icid e. 113. (D ) This p atients p resentation is m ost con-
sistent w ith schizoaffective d isord er, bip olar
110. (D ) Sexu al sad ism d isord er and sexu al m as- type, characterized by the presence of psy-
ochism d isord er are, respectively, the d eri- chotic sym p tom s (e.g., paranoia, d elu sions,
vation of sexu al pleasu re from causing or hallu cinations) concu rrent w ith a m anic epi-
receiving m ental/ p hysical abu se. Exhibition- sod e (e.g., d ecreased need for sleep, increased
istic d isord er is exp osu re of the genitalia in energy, grand iosity, increased goal-d irected
p ublic to an u nw illing p articip ant, and u su - activity, flight of id eas). In this p atient, her p sy-
ally occu rs in m en. Frotteu ristic d isord er, the chotic sym ptom s occu rred in the absence of
ru bbing of genitals against another to achieve m ood sym p tom s, as w ell, w hich d istinguishes
arou sal, is also u su ally seen in m en and p er- schizoaffective d isord er from a prim ary m ood
form ed in crow d ed places. Patients w ith trans- d isord er, su ch as bipolar or m ajor d epressive
vestic d isord er are arou sal by cross-d ressing. d isord er. Patients w ith d elu sional d isord er
also have a d elu sion, althou gh the d elu sion is
111. (B) Koro, taijin-kyofu sho, and zar are all u su ally m ore circu m scribed than in this case,
exam p les of cu ltu re-bou nd d elu sions. Koro is and they d ont have the au d itory hallucina-
the w orry that the penis is shrinking into the tions or significant affective sym ptom s (i.e.,
abd om en and is fou nd in South and East Asia. m anic ep isod e) as seen in this case. Patients
Taijin-kyofu sho is the belief that ones bod y is w ith schizop hrenia obviou sly have sim ilar
offensive to others, and zar is the d elu sional sym ptom s su ch as paranoia, d elu sions, hal-
belief of p ossession by a sp irit. Cap gras syn- lu cinations, and d isorganized behavior, bu t
d rom e is the d elu sional id ea that im posters the m ood sym p tom s are not as prom inent or
have replaced once fam iliar p ersons. Ku ru is a enou gh to be d iagnosed w ith a m anic ep isod e.
slow ly progressive neurologic d isease w hich
lead s to d eath, sim ilar to Creu tzfeld tJakob 114. (C) This p atient w ith PTSD continu es to be
d isease, cau sed by prions. sym p tom atic, sp ecifically w ith nightm ares,
insom nia, increased startle, and avoid ance.
112. (E) Paraphilias encom p ass a w id e variety of Given the p artial efficacy and tolerability
m alad ap tive sexu al behaviors and fantasies. of the citalop ram (an SSRI), au gm entation
Voyeu ristic d isord er is d eriving sexu al plea- w ith another agent m akes the m ost clinical
sure from w atching another person or per- sense. Alp ha-ad renergic blockers like p ra-
sons involved in the act of und ressing or other zosin are p articu larly efficaciou s in treating
sexu ally oriented activity. Exhibitionistic d is- PTSD sym p tom s, p articu larly nightm ares
ord er is a paraphilia involving exposing ones and insom nia. Benzod iazep ines, inclu d ing
genitalia to an u nsu sp ecting au d ience. Frot- alp razolam , have not d em onstrated efficacy
78 2: Ad ult P sychop a thology

in PTSD; in ad d ition, they shou ld be avoid ed others. Regression is a retu rn to patterns of


given the risk of d ep end ency, a p articu lar relating, thinking, or feeling that had com e
concern in this case. Bu p rop ion, an antid e- before ones cu rrent d evelop m ental stage; for
p ressant w ith norep inep hrine and d op a- exam p le, m any m ed ical stu d ents w ho retu rn
m ine reu p take blockad e activity, has not hom e act as if they are teenagers w ith regard
been show n to be beneficial in treating PTSD. to their parents or other hom etow n friend s.
Antip sychotics (e.g., risp erid one) and m ood Sublim ation is the channeling of d rives or con-
stabilizers (e.g., valp roic acid ) are often u sed flicts into goals that are gratifying but socially
ad ju nctively in the treatm ent of PTSD, esp e- accep table (e.g., ind ivid u als afraid of blood
cially for m ood lability, agitation, flashbacks, and hospitals w ho becom e hosp ital w orkers
and p sychotic sym p tom s, althou gh the evi- or physicians).
d ence for their benefit is not as conclu sive as
for p razosin. 118. (D ) This p atients reaction to p aroxetine is
consistent w ith serotonin synd rom e, a poten-
115. (B) Cognitive processing therapy is an evidence- tially life-threatening cond ition caused by the
based form of cognitive-behavioral therap y interaction of a serotonin-sp ecific reu p take
that has d em onstrated efficacy in the treat- inhibitor (SSRI), su ch as p aroxetine, w ith a
m ent of PTSD. It incorp orates both a cogni- m onoam ine oxid ase inhibitor (MAOI), w hich
tive com ponent and exposu re to the trau m a resu lts in increased am ou nts of both catechol-
through rem em bering, d iscussion, and cues. am ines and ind olam ines. Featu res of m ild
N either p ure behavior nor cognitive therapy serotonin synd rom e includ e tachycard ia,
alone is as effective. Dialectical behavioral flushing, fever, hypertension, ocular oscilla-
therapy is another form of CBT, specifically tions, and m yoclonic jerks. Severe serotonin
d evelop ed for treating bord erline personality synd rom e m ay result in severe hypertherm ia,
d isord er, and it u tilizes ind ivid ual therap y, com a, au tonom ic instability, convu lsions, and
grou p s, safety-m anagem ent, and hom ew ork. d eath; therefore, clinicians m u st w ait at least
Psychod ynam ic p sychotherapy has not been 14 d ays after d iscontinu ing an MAOI before
ad equately studied to recommend as a primary starting a serotonergic agent. Most serotonin
treatment for PTSD. in the CN S is synthesized in the d orsal and
m ed ial raphe nu cleu s of the brain stem . The
116. (D ) The locu s ceruleu s is the alarm center of caud ate nucleu s is located in the brain stem
the brain and is hyperactive in anxiety states. and concerned w ith m em ory form ation. The
It is the location of m ost of the norep inep hrine- nu cleu s accu m bens is located in the striatu m
containing neurons in the brain. The am yg- and thou ght to be involved w ith sensations
d ala and hippocam pu s, both part of the lim bic of rew ard and pleasu re, as w ell as ad d ictive
system , are involved in fear/ anger responses behaviors. The locu s ceru leu s synthesizes nor-
and m em ory form ation, resp ectively. The ep inep hrine, and the su bstantia nigra synthe-
basal ganglia coord inate m otor activity, w hile sizes d opam ine.
the thalam u s is the brains relay center.
119. (E) This m ans risk of suicid e is highest after
117. (D ) Sp litting is a prim itive d efense m echa- d ischarge from the hospital. Up to 10% of
nism , m anifested by view ing others as either p atients w ith schizop hrenia d ie by su icid e.
all good or all bad . It is com m on in p atients Up to 50% of su icid es am ong ind ivid u als w ith
w ith bord erline p ersonality d isord er althou gh schizophrenia occu rs shortly after d ischarge
there is no other evid ence of bord erline from the hospital. Many sink into a d epres-
p athology in this case. Acting out refers to sive-like illness follow ing stabilization of their
p erform ing an action in contrast to bear- p sychotic sym p tom s. Risk factors inclu d e
ing and m anaging the im p u lse to p erform it. being m ale, u nm arried , u nem p loym ent, social
Externalization rep resents the tend ency to isolation, d ep ression, and p reviou s su icid e
p roject ones ow n internal characteristics onto attem p ts.
Answe rs : 115130 79

120. (A) This patient likely su ffers from m ild intel- head ache, sleepiness, irritability, concentra-
lectu al d isability (form erly m ental retard a- tion problem s, vom iting, and m u scle aches/
tion). The DSM -5 specifies the level of severity stiffness. Phencyclid ine (PCP) (O) and other
on the basis of intellectu al and ad aptive fu nc- d issociative anesthetics can ind u ce vertical or
tion d eficits. Deficits occu r in the conceptu al horizontal nystagm us (rotary w ith ketam ine),
d om ain, the social d om ain, and the p racti- hyp ertension (at least m ild , bu t hypertensive
cal d om ain. Psychom etric testing, su ch as crisis can occur), tachycard ia, ataxia, nu m b-
m easu rem ent of IQ, is no longer su fficient to ness and high pain tolerance, ataxia, hyper-
d eterm ine severity of im p airm ent, bu t is often acu sis, hypertherm ia, m u scle rigid ity, seizu res
a com ponent to assess the concep tu al d om ain inclu d ing statu s ep ilep ticu s, and d eath. The
d eficit. This p atient has d eficits in acad em ic p resence of nystagm u s helps to d istinguish
skills, has im m ature social jud gm ent that PCP intoxication from other form s of psycho-
p laces him at risk for being taken ad vantage sis. Behavioral m anifestations can be severe,
of, and need s help w ith com plex d aily living and severe agitation, rage, and p anic cou pled
tasks (m oney and groceries) even thou gh he w ith d isinhibition m ay cau se a d angerou s
can care for his p hysical need s. In the past, an clinical situ ation. Am phetam ine intoxication
IQ of 50 to 69 w ould be sufficient to d iagnose (C) is characterized by both behavioral and
m ild intellectual d isability; betw een 35 and 49 p hysiological changes. Patients m ay exp eri-
as m od erate; betw een 20 and 34 as severe; and ence eu p horia, interp ersonal sensitivity, anxi-
less than 20 as profound . ety, tension, or anger, im p aired ju d gm ent, and
im p aired social and occu p ational fu nctioning.
121130. [121 (G), 122 (J), 123 (L), 124 (A), 125 (F), 126 Fu rther, they m ay have tachycard ia or bra-
(O), 127 (C), 128 (N), 129 (M), 130 (B)] Cannabis d ycard ia, p u p illary d ilation, insom nia, blood
intoxication (G) inclu d es increased heart p ressu re changes, sw eating or chills, nau sea
rate for several hou rs, p ostu re-d ep end ent or vom iting. Chronic u se m ay resu lt in d ry
changes in blood pressu re, injected conju ncti- skin, acne-like lesions, or chronic nose bleed s.
vae, a high w ith m ild eu phoria, feeling of Op iate w ithd raw al (N ) is characterized by
relaxation, p ercep tu al changes (tim e d istor- d ysp horia, nau sea, vom iting, m u scle aches,
tion, intensified ord inary experiences), and lacrim ation, rhinorrhea, p iloerection, sw eat-
increased sociability. Cocaine w ithd raw al ing, d iarrhea, yaw ning, fever, and insom nia.
(J) is characterized by onset of d ysp horic m ood , Op iates can be illegal or p rescrip tion, su ch as
hyp ersom nia, increased ap petite, and fatigue. narcotic analgesics. Sym p tom s of acu te intoxi-
Inhalants have becom e increasingly com m on cation (M) includ e pup illary constriction
d ru gs of abu se. Signs and sym p tom s of intoxi- (can be d ilation once anoxic brain inju ry has
cation (L) are inhalant-sp ecific, bu t gener- occurred from respiratory d epression), d row s-
ally inclu d e rapid onset of effects, d izziness, iness, slu rred sp eech, and p ossibly p u lm onary
nausea, then slow ing, ataxia, slu rred speech, ed em a. At higher d oses, com a or d eath w ith
and d isorientation. Bronchospasm m ay also resp iratory d ep ression m ay occu r. Alcohol
occur. Intoxication usu ally im p roves w ithin w ithd raw al (B) can start hou rs after the last
an hou r of abstinence because the su bstances d rink, and is characterized by au tonom ic
are u su ally volatile hyd rocarbons. Treatm ent instability (sw eating, tachycard ia), trem ors,
of inhalant w ithd raw al is su pportive. In acu te insom nia, nau sea/ em esis, transient hallu ci-
alcohol intoxication (A), as blood concentra- nations or illu sions, psychom otor agitation,
tion of alcohol increases, there is w orsening anxiety, and seizu res. Deliriu m trem ens can
m otor p erform ance, incoord ination and jud g- d evelop , and can be d ead ly. Am p hetam ine
m ent errors, m ood lability, nystagm u s, slu rred w ithd raw al (D ) is characterized by fatigue,
sp eech, potential blackou ts, altered vital signs, vivid d ream s, sleep d istu rbances, increased
and possibly d eath. Caffeine w ithd raw al ap p etite, and p sychom otor retard ation or
(F) can occu r w hen p eop le abru p tly stop agitation. A caffeine intoxication synd rom e
their accu stom ed intake. Sym p tom s inclu d e (E) is possible, especially in ind ivid u als w ho
80 2: Ad ult P sychop a thology

consum e a large am ou nt or w ho are u nac- to sed ation and cau se w eight gain. By block-
custom ed to it. In stud ies, participants report ing m uscarinic-1 receptors (F), antip sychotics
sym ptom s of jitteriness, d iuresis, irritability, cause anticholinergic sid e effects of sed ation,
insom nia, p sychom otor agitation, and nau sea. d ry m ou th, and constipation.
There is evid ence of a m ild cannabis w ith-
d raw al (H) synd rom e lasting several w eeks, 137145. [137 (G), 138 (I), 139 (F), 140 (D ), 141 (B),
characterized by d ecreases in m ood and appe- 142 (C), 143 (A), 144 (E), 145 (H)] Defense
tite, w ith increases in irritability, anxiety, and mechanisms are conceptualized as the mind s
tension. Cocaine intoxication (I) is sim ilar to (m ostly unconscious) w ay of d ealing w ith d is-
am phetam ine intoxication. At higher d oses, tressing w ishes/ d rives/ thou ghts. They can be
signs of toxicity m ay d evelop and includ e sei- subd ivid ed into im matu re, neu rotic, or mature
zu res, chest p ain, hyp erp yrexia, and d eath. d efenses. The low er functioning d efense mech-
Susceptible u sers m ay also experience para- anisms are as follow s: Denial (B) is w hen an
noia, ranging from m ild hypervigilance to ind ivid ual refuses to accept reality because it
frank paranoia and p ersecu tion. H allu cino- is too d istressing; splitting (I) is also an imma-
gens inclu d e su bstances su ch as LSD, m es- ture d efense w here the person sees others or
caline, and ecstasy. They prim arily affect other situ ations as all good or all bad and is
p erceptions and in intoxication (K) can result unable to tolerate positive and negative aspects
in m ood changes, p aranoia, id eas of reference, to be present w ithin one p erson; d isplacement
d ep ersonalization, hallucinations, and synes- (C) is the channeling of ones unacceptable
thesia. Au tonom ic sym ptom s m ay also occu r, w ishes or feelings into less anxiety prod uc-
and inclu d e blu rry vision, tachycard ia, sw eat- ing onesfor instance, jerking the d ogs leash
ing, trem ors, and p u p il d ilation. harshly w hen one w as really u pset at having
been w oken up by a spouse (truly being angry
131136. [131 (F), 132 (B), 133 (D ), 134 (D ), 135 (E), w ith the spouse); in projection (E) ones u nac-
136 (A)] Blockad e at the 5-H T2 A (serotonin) ceptable id eas or thoughts are seen as com ing
recep tor (A) is thou ght to im p rove cognitive from another (for instance, a cheating husband
and affective sym ptom s in schizophrenia and accuses his w ife of being unfaithfu l); projec-
other psychotic d isord ers. Blockad e at the tive id entification (F) can be thou ght of as a
alp ha-1 ad renergic recep tor (B) is resp onsible self-fulfilling prophecy, w herein the patients
for the d izziness, sed ation, and orthostatic unacceptable feelings are projected to another,
hyp otension associated w ith m any of the low - but the other (e.g., therapist) acts in such a w ay
p otency typical and atyp ical antip sychotics. that they become truefor instance, a patient
The antip sychotics are not know n to affect view s the w orld as fu ll of u nloving people (she
beta recep tors (C). Blockad e at the d op am i- hates herself), then acts to push her therapist
nergic D2 recep tors (D ) in the m esolim bic and to the breaking point w here he terminates her
m esocortical areas of the brain are responsible care, therefore confirm ing her belief that the
for red u cing the positive psychotic sym p - w orld is full of unloving people. Examples of
tom s in schizop hrenia. Blockad e of the D2 neu rotic d efenses are as follow s: rationaliza-
recep tor in other p athw ays is resp onsible for tion (H) is the process of making excu ses
som e u nd esirable sid e effects: blockad e in or cognitively reframing a situ ation to make
the nigrostriatal pathw ay is associated w ith it less anxiety or d istress provoking; reaction
m ovem ent d isord ers su ch as extrapyram id al formation (G) d escribes the process of turn-
sym ptom s, w hereas blockad e in the tubero- ing unacceptable d rives and d esires into their
infu nd ibu lar p athw ay is resp onsible for the opposite. The follow ing are m atu re d efense
p rolactinem ia seen w ith som e antipsychot- mechanisms: altru ism (A) involves acceptable
ics; d op am ine inhibits the release of p rolac- actions that serve others and bring pleasure to
tin, hence blockad e of the d opam ine receptor the ind ivid u al; hu mor (D ) involves expressing
resu lts in p rolactin release. Blockad e at the his- unpleasant thoughts or feelings in a w ay that
tam ine-1 receptor (E) is believed to contribu te brings enjoyment and pleasure to others.
CHAPTER 3

S o matic Tre atme nt and


Ps yc ho pharmac o lo g y
Que s tions

D IRECTION S (Questions 1 through 73): For each (D) O isthotonos


of the multiple-choice questions in this section (E) Serotonin syn ro e
select the lettered answ er that is the one best
response in each case. Questions 3 and 4

A 24-ye r-o w o n resents to her ri ry c re


Questions 1 and 2 hysici n for regu r checku . During the ex in -
A 32-ye r-o n is itte to gener hos it tion, her octor notes 15- b w eight g in. The tient
fter ingesting n u nknow n qu ntity of hene zine co ins th t she w ys fee s tire uring the y
in su ici e tte t. After g stric v ge n in- es ite s ee ing fro 8 pm to 9 a m i y. On further
istr tion of ch rco s u rry, he is tr nsferre to the questioning, the hysici n e rns th t the tient
e ic intensive c re u nit for onitoring. Tw enty- fee s s n e ty, often thinks bout e th, c nnot
fou r hours ter, he begins to see horses running in concentr te t w ork, n cks the energy to c re for
the h n u s out his intr venous (IV) ines. her tw o chi ren. H ow ever, her oo icks u w hen
her hysici n questions her bout her chi ren.
1. Which of the fo ow ing tre t ents w ou be
the ost i ort nt t this ti e? 3. Which of the fo ow ing e ic tions w ou be
the ost ro ri te to tre t her i ness?
(A) Ch or ro zine
(A) F u oxetine
(B) Cy rohe t ine
(B) N ortri ty ine
(C) Lor ze
(C) P roxetine
(D) Me eri ine
(D) Phene zine
(E) Phenytoin
(E) Tr zo one
2. Tw e ve ys fter his su ici e tte t, he
receives ven f xine (Effexor) to tre t his 4. H er hysici n initi tes tre t ent w ith n nti-
e ression. One hour fter ingestion of the e ress nt t ther eu tic ose. Which of the
ven f xine, he beco es t chyc r ic n i - fo ow ing ti es w ou be consi ere ini-
horetic n eve o s yoc onic jerks. Which u equ te tri for the e ic tion?
of the fo ow ing con itions h s he ost ike y (A) 1 w eek
eve o e ? (B) 2 w eeks
(A) Acute ystoni (C) 6 w eeks
(B) Ak thisi (D) 3 onths
(C) N eu ro e tic ign nt syn ro e (E) 6 onths

81
82 3: S oma tic Tre a tme nt a nd P syc hop ha rma c ology

Questions 5 and 6 bec u se she is w orrie th t her foo y be


oisone . U on interview, she is ys oor
Whi e on c in gener hos it , you re c e t eye cont ct, sycho otor ret r tion, n
4:30 a m to ev u te tient w ho w nts to be is- te rfu n constricte ffect. Which of the
ch rge . U on rriving on the f oor, the nurse te s fo ow ing w ou be the ost ro ri te
you he ju st u e ou t his thir IV of the y n e ic tion regi en for this tient?
st rte to sw ing the IV o e in the ir w hi e ye ing
rof nities. H e te s nu rses i e he u st c tu re (A) C oz ine
the tiger th t is oose in the rking ot. You qu ick y (B) F u oxetine n ris eri one
ook t his ch rt n see th t he is 55-ye r-o n (C) F u oxetine n c on ze
w ith history of coho u se isor er, itte 36 (D) F u oxetine onother y
hou rs e r ier for b o in in, n u se , n vo -
(E) Ris eri one onother y
iting. H is tr ns in ses w ere e ev te on is-
sion, bu t his he titis rofi e is sti en ing. H e is
8. A 40-ye r-o w o n w ith bi o r isor er
sche u e for u er en osco y in the orning.
resents to the e ergency roo 2 w eeks fter
st rting new e ic tion. She re orts th t
5. Which of the fo ow ing shou be the ost
she w s oing w e u nti she got vir
ro ri te e ic tion given t this ti e?
g stroenteritis n w s u n b e to e t for sev-
(A) Ch or i ze oxi e er ys. Whi e the vir sy to s reso ve ,
(B) Ch or ro zine she now co ins of n u se n vo iting,
(C) Disu fir t xi , n tre or. Which of the fo ow ing
stu ies shou be or ere first?
(D) H o eri o
(E) Ox ze (A) C rb ze ine eve
(B) De kote eve
6. You s e k brief y w ith this tient n re b e (C) H e CT
to sett e hi ow n. H is te er tu re is 102.1F, (D) Lithiu eve
u se is 130 be ts/ in, n b oo ressu re is
(E) Urine toxico ogy screen
220/ 120 H g. Which of the fo ow ing is
the ost ro ri te e ic tion n rou te of
9. A 25-ye r-o n resents to the e ergency
inistr tion to give?
e rt ent cco nie by his f i y w ho
(A) H o eri o intr u scu r y (IM) re ort th t the tient h s not eft his be -
(B) H o eri o intr venou s y (IV) roo in 3 w eeks. They re ort th t he s en s
(C) L bet o IV his y re ing science fiction nove s th t they
rovi e for hi . On ex in tion, you fin
(D) Lor ze IM
hi to be o orou s n isheve e . H e is
(E) Lor ze IV gu r e w hen you s e k w ith hi . H e oes
not ke eye cont ct n he voi s ooking
7. A 32-ye r-o sing e w o n resents to the t the te evision in the w iting roo bec u se,
e ergency e rt ent escorte by her f i y he s ys, he w nts to revent intru ers fro
fter she reve e to the her su ici e n to contro ing his thoughts. After co eting
t ke fu bott e of cet ino hen n rink the ex in tion, you it hi to the in -
bott e of w ine. She st tes th t she c nnot tient sychi tric u nit n begin ri i r zo e.
h n e w ork ny onger bec u se her boss is After 1 w eek, he is no onger fr i to ook t
trying to h ve her fire n her cow orkers re the te evision bu t he s en s uch of the y
he ing hi fin f u t in her w ork. She c i s cing the f oor, st ting th t he fee s rest ess.
they h ve even t e her hone n re h v- After re u cing the ri i r zo e to the ow est
ing her fo ow e . She re orts ifficu ty f ing ose you be ieve is ro ri te, he sti ces
s ee n oor concentr tion t w ork. She the h s. Which of the fo ow ing is the ost
h s ost 10 b over the st 2 onths, in rt ro ri te next ste ?
Que s tions : 513 83

(A) A benztro ine 11. Which of the fo owing is the ost ike y serious
(B) A i henhy r ine co ic tion this tient y ex erience?
(C) A or ze (A) Diffu se intr v scu r co gu tion
(D) A ro r no o (B) Myoc r i inf rction
(E) Discontinu e ri i r zo e n try (C) Pu on ry e bo is
nother nti sychotic (D) Res ir tory f i u re
(E) Rh b o yo ysis
10. A 35-ye r-o w o n resents to the office
st ting th t she is e resse . She w s i
12. Which of the fo ow ing ong-ter si e effects
off fro her w ork 5 w eeks go, n is h v-
of high- ose thiori zine ost ike y e his
ing trou b e fin ing new job. She escribes
hysici n to sw itch to h o eri o ?
fee ing very ow, w ith fee ings of ho e essness
bout the fu tu re, n ifficu ty getting herse f (A) Agr nu ocytosis
u n resse in the orning. She h s ost (B) H y er ro ctine i
5 b n is s ee ing oor y. She re orts one re- (C) Pri is
vious e iso e of e ression. A ition y, she
(D) Retin ig ent tion
escribes erio bou t ye r go in w hich
she st ye u for sever ys in row n- (E) T r ive yskinesi
ning big rty for her hu sb n s birth y,
c e ning the hou se n st rting nu erou s Questions 13 and 14
other rojects; t th t ti e, others co ente A 45-ye r-o u ne oye invest ent b nker res-
th t she w s t king qu ick y n cting ike ents to you r office cco nie by his w ife. H e h s
the energizer bu nny. Which of the fo ow ing been ou t of w ork for 1 onth bec u se he is fr i
e ic tions w ou be the ost ro ri te to t ke the tr in into w ork. H e never h ifficu ty
tre t ent for her e ression? tr ve ing u nti 2 onths go, w hen on tr in ur-
(A) Bu ro ion ing f i y v c tion, he su ffere chest in, i-
t tions, shortness of bre th, n use , ting ing in his
(B) F u oxetine
extre ities, n fe r of ying. These sy to s
(C) L otrigine
ste bou t 15 inu tes. A w orku by c r io o-
(D) V roic ci gist w s neg tive. H ow ever, these sy to s h ve
(E) Ven f xine continu e , so cco nie by intense fe r n
fee ing of being et che fro hi se f. H e h s h
Questions 11 and 12 roxi te y tw o tt cks er w eek bu t so h s
severe nxiety w hen just thinking bout returning
A 33-ye r-o inte ectu y is b e e resi ent
to w ork s he is fr i he w i ie on the tr in. H e
of grou ho e resents to the e ergency e rt-
te s you th t he cou not h ve co e to see you
ent escorte by st ff e bers w ho be ieve he h s
w ithou t his w ifes he .
beco e confu se n isoriente over the st few
ys. H e h s been resi ent there for sever ye rs
13. Which of the fo ow ing e ic tions w ou
w ithou t inci ent, bu t st w eek his thiori zine w s
be the ost ro ri te first choice for this
ch nge to h o eri o bec use of concerns bout
tient?
the ong-ter u se of high oses of thiori zine.
On ex in tion, you fin hi to be isoriente to (A) Bu ro ion
ce n ti e; he is i horetic w ith te er tu re (B) Bu s irone
of 105.8F, h s he rt r te of 130 be ts/ in, n (C) F u oxetine
res ir tory r te of 20 bre ths/ in. H is extre ities
(D) Pro r no o
re stiff. Rou tine bor tories reve w hite b oo
ce (WBC) cou nt of 15,000/ L, b oo u re nitro- (E) Tr zo one
gen (BUN ) 75 g/ L, cre tinine 1.2 g/ L, n
cre tinine hos hokin se 2,300 I/ L.
84 3: S oma tic Tre a tme nt a nd P syc hop ha rma c ology

14. The tient is nxiou s bou t w iting u nti the (A) Continu e ibu rofen t the rescribe
e ic tion t kes effect, n sks if you c n ose.
give hi so ething th t w i rovi e ore (B) Decre se the ose of ibu rofen.
i e i te re ief. You eci e to rescribe (C) Incre se the ose of ibu rofen.
benzo i ze ine in the interi . Which of the
(D) Sto the ithiu .
fo ow ing e ic tions w ou be the ost
otent? (E) Sw itch fro ibu rofen to s irin.

(A) A r zo Questions 17 and 18


(B) Ch or i ze oxi e
A 9-ye r-o boy is brou ght to the hysici n bec use
(C) Te ze
his rents h ve receive note fro his secon -
(D) Di ze gr e te cher co ining th t he is isru tive in
(E) Ox ze c ss. H is te cher be ieves he cou chieve better
gr es if he cou sit sti n y ttention. H is r-
Questions 15 and 16 ents re ort he h s w ys been n ctive chi bu t
oes h ve ifficu ty getting ong w ith other chi -
A 22-ye r-o sing e e co ege u sic stu ent
ren in the neighborhoo . In the ex in tion roo ,
re u ct nt y resents to you r office t the requ est of
the boy see s un b e to sit sti n h s ifficu ty
his rents. H is rents re w orrie bec u se he h s
co eting t sk you h ve ssigne . You eci e to
not been e ting, t ks const nt y, n never see s
rescribe hi ethy heni te.
to s ee . The tient st tes he h s never fe t bet-
ter, th t hes been co osing sever songs y,
17. Which of the fo ow ing is the ost co on
n requ ires on y few hou rs of s ee to fu nction.
si e effect of this tre t ent?
H e be ieves he w i be the next gre t rock st r n
w nts to finish the interview in or er to get b ck to (A) D yti e row siness
w riting, bu t he c nnot st y focu se ong enou gh to (B) Decre se in systo ic b oo ressu re
nsw er ny of you r qu estions. H e oes not h ve ny (C) Difficu ty f ing s ee
rior or f i y sychi tric history n enies u sing
(D) Incre se in etite
ny coho or i icit ru gs.
(E) S ow e grow th
15. You eci e to begin tri of ithiu . Before
initi ting tre t ent, w hich of the fo ow ing 18. Which of the fo ow ing sy to s w ou ost
w ou be the ini u bor tory infor tion ike y be ex cerb te by the e ic tion?
th t shou be obt ine ? (A) Be w etting
(A) Co ete b oo cou nt (CBC) (B) Myo i
(B) F sting g u cose n i i ne (C) Poor i u se contro
(C) Liver fu nction tests (D) Re ing ifficu ties
(D) Seru oni (E) Tics
(E) Seru cre tinine, BUN n e ectro ytes,
thyroi stu ies 19. An 8-ye r-o boy is brou ght to you r office
by his other bec u se of his h bits, w hich
16. One w eek fter st rting the ithiu , the tient h ve w orsene over the st ye r. She first
s r ins his nk e w hi e exercising. H is ri- notice the st ye r w hen sever ti es
ry c re octor ex ines hi n te s hi y he w ou re e te y b ink his eyes n
to use over-the-counter ibu rofen, w hich he frow n w hi e c e ring his thro t. H e continu es
w i h ve to t ke for ore th n 1 w eek. Which to o this es ite trying not to, n , in i-
of the fo owing wou be the ost ro ri te tion, he often sticks ou t his tongu e n s e s
course of ction? his shirt w hi e s e king w ith c ss tes.
These beh viors h ve resu te in his being
Que s tions : 1424 85

te se n osing frien s t schoo . Which 22. Sw itching to w hich of the fo ow ing e ic -


of the fo ow ing gents w ou be the ost tions w ou be the ost ike y to i rove
effective to tre t his isor er? these sy to s?
(A) A r zo (A) Ari i r zo e
(B) A itri ty ine (B) C oz ine
(C) C oni ine (C) H o eri o
(D) P roxetine (D) O nz ine
(E) Ris eri one (E) Ris eri one

20. A 45-ye r-o n w ith history of hy er- 23. A 24-ye r-o rrie secret ry h s co -
tension, yoc r i inf rction, n chronic ine of izziness, it tions, n sw e ty
inso ni is referre to you by his ri ry s for the st 10 onths. She so occ -
c re hysici n for ev u tion of his jor sion y h s extre e u sc e tension in her
e ressive e iso e. H e h s been tre te for neck th t occu rs both t w ork n t ho e.
3 onths w ith f u oxetine 60 g i y, w ith She h s ifficu ty f ing s ee n fee s
no i rove ent. H e h s no history of rior e gy ost of the y. In ition, she h s
jor e ressive e iso es. Which of the fo - h ifficu ty concentr ting t w ork resu t-
ow ing w ou be the ost ro ri te next ing in her king ist kes. She h s gone to
ste in his tre t ent? oc e ergency roo three ti es bec u se of
the it tions bu t nothing w s foun . H er
(A) A ithiu .
f i y r ctitioner ce her on hy og yce-
(B) A thyroi hor one (T3). ic iet n referre her to neu ro ogist w ho
(C) Sw itch to cit o r . fou n no foc bnor ities. She te s you
(D) Sw itch to nortri ty ine. she h s ny w orries bout her f i y but
(E) Sw itch to ven f xine. oes not fee rticu r y e resse . Which of
the fo ow ing e ic tions w ou rovi e the
Questions 21 and 22 ost i e i te re ief of her sy to s?

You h ve been tre ting 55-ye r-o n w ith (A) Bu ro ion


schizo hreni for 20 ye rs w ith h o eri o n (B) Bu s irone
benztro ine. Gener y, he h s one w e n h s (C) C o i r ine
not requ ire jor e ic tion ch nges or hos i- (D) F u oxetine
t iz tions. Abou t 2 ye rs go, you notice so e (E) Lor ze
i s cking n tongu e rotru sions th t i not
bother the tient; how ever, now he so h s o , 24. A 22-ye r-o w o n is itte to sychi-
irregu r ove ents of his r s w hich ke it tric u nit fter seriou s su ici e tte t. She
ifficu t for hi to e t. h s h ny su ici e tte ts in the st
w ith v rying severity. H er r s re sc rre
21. Which of the fo ow ing con itions is ost fro rior tte ts t hu rting herse f. She
ike y the c u se of these sy to s? h been goo stu ent u nti high schoo ,
(A) Anticho inergic toxicity w hen she took u w ith f st crow , beg n
(B) H u ntington ise se bu sing coho n riju n , n r n w y
fro ho e sever ti es. She h s h sev-
(C) Meige syn ro e
er intense, stor y re tionshi s w ith en.
(D) Sy enh chore
Ou t tient tre t ent h s ost y consiste
(E) T r ive yskinesi (TD) of her co ints to her ther ist bou t her
f i y. She u su y c s her ther ist iy
bou t ifferent crises; how ever, her ther ist
w s on v c tion u ring her ost recent crisis.
86 3: S oma tic Tre a tme nt a nd P syc hop ha rma c ology

She escribes fee ings of ho e essness n (C) N o e ic tion


he essness consistent y for the st sever (D) O nz ine
w eeks; in ition, she is s ee ing oor y, tire (E) To ir te
throu ghou t the y, istr cte , n h s ost
so e w eight ue to ck of etite. Which of 27. A 36-ye r-o n w ith his first e iso e
the fo ow ing e ic tions w ou be the ost of jor e ression h s been tre te for
he fu for this tient? 8 onths w ith roxetine n is cu rrent y in
(A) C on ze re ission. Bec u se of sexu ysfu nction, he
(B) F u oxetine eci e to sto t king the e ic tion 3 ys
go w ithou t first infor ing you . Yester y, he
(C) H o eri o
bec e cu te y irrit b e n even h crying
(D) Lithiu s e . H e so co ins of bo y ches, chi s,
(E) Zi r si one n gener eth rgy. Which of the fo ow ing
ste s w ou best evi te his sy to s?
25. A 30-ye r-o w o n, 10 w eeks regn nt w ith
her first chi , resents to you r office st ting (A) Rest rt roxetine.
th t she is e resse n is h ving oubts (B) St rt cet ino hen.
bou t beco ing other. She h s been ex e- (C) St rt bu ro ion.
riencing i y e esis n is h ving trou b e (D) St rt v roic ci .
fu nctioning t w ork. She re orts th t she w s (E) St rt ven f xine.
e resse throu ghou t her 20s, but iscontin-
u e her nti e ress nts w hen she bec e 28. A 20-ye r-o w o n is st rte on otrig-
regn nt bec u se of concerns bout h r ing ine for bi o r isor er, cu rrent y e resse .
her chi . She w ou ike you r he in ev u t- Tw o onths ter, she co es b ck re orting
ing risks n benefits of rest rting sertr ine, th t her sy to s re unch nge , es ite
w hich w orke w e for her in the st. Which her being co i nt w ith the e ic tion. You
of the fo ow ing risks w ou be the ost ike y review her e ic tion ist n iscover th t
to occu r? she w s recent y st rte on new e ic tion(s).
(A) Ebstein no y Which of the fo ow ing e ic tions w ou
(B) Gest tion i betes ost ike y be res onsib e for the ck of
effic cy?
(C) Inf nt eve o ent e y
(D) N eur tu be efects (A) Dros irenone n ethiny estr io
(E) Persistent u on ry hy ertension of (B) Ibu rofen
the new born (C) Lithiu
(D) V roic ci
26. A 19-ye r-o w o n is itte to n in - (E) Zi r si one
tient sychi tric f ci ity for her e ting isor-
er. She w eighs 82 b n st n s 5 ft 6 in t . Questions 29 and 30
She beg n ieting in high schoo to ose n
unw nte 5 b. Encour ge by co i ents A 68-ye r-o n resents to you r office on refer-
on her figu re, she continu e to ose nother r fro his ri ry c re hysici n for ev u tion
10 b. H er e ting h bits re ritu ize : She cuts of e ression. H e fee s e resse n ho e ess, n
her foo into s ieces n oves it rou n he h s ost his etite. H e no onger enjoys seeing
on her te. She h s n intense fe r of being his gr n chi ren. It t kes hi sever hours to f
overw eight. Which of the fo ow ing e ic - s ee t night, n he st ys s ee for on y bou t
tions w ou be the ost usefu for this tient? 3 hou rs. H e occ sion y oes not bu y foo bec u se
he is concerne bou t going broke; how ever, his
(A) Cy rohe t ine
f i y infor s you th t he is fin nci y w e -off.
(B) F uoxetine In ition, the tient fee s signific nt gu i t bou t
Que s tions : 2534 87

beco ing bu r en on his f i y. H is height is 6 ft (A) As irin


w ith w eight of 155 b, ecre se fro his u su (B) Decre se f u i int ke
175 b. (C) H y roch orothi zi e
(D) N itrog ycerin
29. Which of the fo ow ing e ic tions w ou be
the ost ro ri te for tre ting his inso ni ? (E) Pro r no o

(A) Bu ro ion Questions 33 and 34


(B) Di ze
A 47-ye r-o e nove ist h s fe rs of cont in -
(C) Di henhy r ine
tion fro nything he be ieves is irty. In rest u -
(D) F uoxetine r nts, he u ses his ow n stic u tensi s th t he c rries
(E) Tr zo one w ith hi . E sew here, he w e rs g oves or u ses er
tow e s to voi tou ching irty objects. U on
30. Which of the fo ow ing si e effects of this retu rning ho e, he then rocee s to w sh his h n s
e ic tion w ou be the ost i ort nt to 10 ti es before going bou t his ctivities. If he cci-
e u c te the tient bou t? ent y tou ches nything rior to the co etion
(A) Anorg s i of w shing his h n s, he ex eriences nxiety n is
then u n b e to erfor his u su night y t sks. As
(B) Gyneco sti
resu t of these sy to s he re ins u n b e to h ve
(C) I otence e ningfu soci ife.
(D) Pre tu re ej cu tion
(E) Pri is 33. Which of the fo ow ing e ic tions w ou be
the ost effective tre t ent for his con ition?
Questions 31 and 32
(A) C o i r ine
A 57-ye r-o n w ith bi o r isor er, w ho (B) C on ze
you h ve tre te su ccessfu y for 20 ye rs w ith ith- (C) N ortri ty ine
iu 300 g ti , is itte to the gener hos it (D) O nz ine
w ith chest in.
(E) Phene zine
31. Which of the fo ow ing fin ings w ou be
34. After 12 w eeks of ther eu tic ose of the
ost ike y seen on his e ectroc r iogr
e ic tion, the tient no onger nee s to
(ECG)?
w sh his h n s qu ite so often bu t sti h s
(A) First- egree trioventricu r (AV) b ock signific nt istress w hen he touches irty
(B) PR interv ro ong tion things. Which of the fo ow ing tre t ents
(C) QT interv ro ong tion w ou be the ost ro ri te next ste ?
(D) Sino tri b ock (A) A bu s irone.
(E) T-w ve e ression (B) A ithiu .
(C) A ris eri one.
32. The tient is eter ine to h ve su ffere (D) Ch nge to ris eri one.
yoc r i inf rction (MI) n qu ick y u n er-
(E) Refer for cingu oto y.
goes ngio sty. H e is eventu y st bi ize
n isch rge fro the hos it on ro r n-
Questions 35 and 36
o o , to ic nitrog ycerin, hy roch orothi -
zi e, n s irin, in ition to his ithiu . A 21-ye r-o co ege stu ent h s been ex eriencing
Prior to his MI, his ithiu eve w s u su y u itory h u cin tions w ith co n s te ing hi
0.8 to 0.9 o / L. N ow it is 1.3 o / L on to hurt hi se f for ore th n ye r. H e h s h tri-
the s e ose. Which of the fo ow ing is the s w ith the fo ow ing gents since th t ti e: qu e-
ost ike y c u se for this ch nge? ti ine 400 g bi for 2 onths, then h o eri o
88 3: S oma tic Tre a tme nt a nd P syc hop ha rma c ology

10 g bi for 6 w eeks, then o nz ine 30 g i y re ort th t she f i e sever c sses bec u se


for 2 onths. Des ite inor re u ction in the she h s not been going to c ss or tu rning in
voices, he re ins iso te fro f i y n frien s. ssign ents. She is not t king ny e ic -
tions n her toxico ogy screen is neg tive.
35. Which of the fo ow ing gents w ou be the On ent st tus ex in tion, her s eech is
ost ro ri te to begin? ressu re n her thou ght rocess is t ngen-
ti . You eci e to it her to the hos it .
(A) Ari i r zo e
Which of the fo ow ing e ic tions w ou be
(B) C oz ine the ost ro ri te to begin?
(C) Per hen zine
(A) Cit o r
(D) Ris eri one
(B) L otrigine
(E) Zi r si one
(C) O nz ine
36. Which of the fo ow ing rece tors is ost (D) To ir te
ike y res onsib e for the bove e ic tions (E) V roic ci
effic cy?
39. A 28-ye r-o w o n w ho you re tre t-
(A) D2
ing for bi o r isor er infor s you th t she
(B) D3 is 3 w eeks regn nt n is nxiou s bou t the
(C) D4 b bys he th. She is cu rrent y rescribe ith-
(D) 5-hy roxytry t ine-1 (5-H T1) iu 300 g bi , n she h s not h nic
(E) 5-hy roxytry t ine-3 (5-H T3) e iso e in 3 ye rs. Wh t w ou be best to
vise her reg r ing the u se of ithiu uring
37. A 65-ye r-o n w ith jor e ression regn ncy?
n hy ertension resents to the e ergency (A) Lithiu is highest risk u ring the thir
e rt ent bec u se he fe to the grou n fter tri ester of regn ncy.
rising fro ch ir. H e h s no signific nt c r-
(B) Lithiu incre ses the risk of neu r tu be
i c history n his ECG, e ectro ytes, n
efects.
neu ro ogic ex in tion re u nre rk b e.
H is f i y re orts th t his hysici n recent y (C) Lithiu c u ses eve o ent e y in
st rte new nti e ress nt bu t they o not ex ose inf nts.
know its n e. Which of the fo ow ing nti e- (D) Lithiu incre ses the risk of c r i c
ress nts w ou ost ike y be res onsib e for for tion.
his resent tion? (E) Lithiu incre ses the risk of c eft i
n te.
(A) Bu ro ion
(B) F u oxetine 40. You re c e to consu t on n 85-ye r-o
(C) I i r ine surgic tient w ho h s beco e co b tive,
(D) Mirt z ine ye ing, u nching st ff, n u ing ou t her
(E) N ortri ty ine IVs. She e n s to e ve bu t is too w e k
to get ou t of her hos it be . Which of the
38. A 21-ye r-o w o n w ith history of bu i- fo ow ing w ou be the ost ro ri te
i nervos resents to the e ergency intervention t this ti e?
e rt ent w ith her rents. She returne (A) Di henhy r ine
fro co ege st w eek, n since th t ti e,
(B) Done ezi
h s been st ying u very te, c ing frien s
roun the w or n t king nonsense (C) Lor ze
bout n internet st rt-u th t she is nning. (D) Orient tion to her su rrou n ings
She sks you to hu rry u w ith you r interview, (E) Ris eri one
since she h s w ork to get b ck to. H er rents
Que s tions : 3546 89

Questions 41 and 42 43. Which of the fo owing gents wou be the


ost ro ri te to inister for his git tion?
A 67-ye r-o w o n resents to your office for
ev u tion of 20- b w eight oss over 4- onth (A) Ch or ro zine
erio n inso ni for the st 4 w eeks. H er (B) H o eri o
ughter infor s you th t her other no onger (C) Lor ze
enjoys ny of her hobbies n h s been s e king (D) Ris eri one
te y bou t ying. They h ve re y consu te
(E) Thiori zine
n internist w ho erfor e n extensive e ic
w orku , inc u ing che istries, b oo cou nt, thy-
44. Which of the fo ow ing gents w ou be the
roi stu ies, g strointestin i ging, n en os-
ost ro ri te to give in or er to ini ize
co y, of w hich w ere nor .
the tients risk of f ing?
41. You w ou ike to begin n nti e ress nt (A) H o eri o
e ic tion bu t w ou ike to ini ize (B) Per hen zine
verse si e effects w hi e ex oiting other (C) Qu eti ine
si e effects. Which of the fo ow ing gents
(D) Ris eri one
w ou be the ost ro ri te?
(E) Thiori zine
(A) Bu ro ion
(B) F u oxetine Questions 45 and 46
(C) Mirt z ine
A 32-ye r-o n resents to your c inic fter osing
(D) N ortri ty ine his job bec use he w s intoxic te w hi e w orking. H e
(E) Sertr ine h s been rinking i y since he w s 16 ye rs o . H e
w s b e to co ete co ege n w ent to w ork fu -
42. U on fu rther history, you e rn th t she h s ti e right fter gr u tion, but h s ost sever jobs
h sever f i e tri s of nti e ress nts since. H is w ife h s thre tene to e ve hi if he oes
n w ou ike to st rt tricyc ic nti e res- not get he .
s nt. Which of the fo ow ing con itions w ou
be the ost ike y to rec u e u sing su ch n 45. Which of the fo ow ing e ic tions w ou
gent? be the ost u sefu s beh vior o ifier to
ecre se his coho use?
(A) Estrogen re ce ent ther y
(B) L cu n r inf rcts (A) Ac ros te
(C) Left bu n e br nch b ock on ECG (B) Disu fir
(D) Tre t ent w ith o i ine (C) F u zeni
(E) Urin ry retention (D) N oxone
(E) N trexone
Questions 43 and 44
46. Further history reve s th t he oes not h ve
A 72-ye r-o n is itte to gener hos i-
oo isor er. He te s you th t often he si y
t s intensive c re u nit bec u se of tere ent
c nnot contro his cr ving to h ve nother rink.
st tu s. H is e ic w orku h s reve e neu o-
Which of the fo owing gents wou be the
ni n congestive he rt f i u re (CH F). On the sec-
ost ike y to ecre se his cr vings for coho ?
on hos it y, he is git te n u s out his
IV ccess. H e so h s been note to s e k ou t ou (A) Bu ro ion
w ith no one in the roo . H is eve of consciou sness (B) Disu fir
see s to w x n w ne. H e oes not h ve sychi- (C) F u zeni
tric history n is not ergic to ny e ic tions. (D) N oxone
Besi es his CH F n neu oni , he oes not h ve
(E) N trexone
other co orbi con itions.
90 3: S oma tic Tre a tme nt a nd P syc hop ha rma c ology

Questions 47 and 48 bou t 1 onth go w hen he beg n h ving if-


ficu ty s ee ing. The st ff hysici n rescribe
A 36-ye r-o w o n is referre to you by her ri- or ze 2 g t be ti e; how ever, he w s
ry c re hysici n for n ge ent of nxiety sti u n b e to s ee . H e st rte w n ering
n fe r th t h s ersiste for sever onths. She u ring the y n h s since been restricte
re orts th t she w s r e n he host ge on to his roo . Bec u se he re ins git te u r-
bo t for sever ys by tw o r e en. Du ring ing the y, they h ve been giving hi or z-
this, she ex erience intense fe r for her ife. Since e 2 g every 6 hou rs for git tion. H is
then, she h s h intense stress w henever she is other e ic tions inc u e hy roch orothi -
ne r the w ter n h s frequ ent night res. She zi e 25 g/ , igoxin 0.125 g every other
c nnot rec et i s of the or e bu t tries to voi y, i ti ze 240 g su st ine re e se i y,
riving her c r w ithin sight of the oce n, w hich h s n tr ns er nitrog ycerin 0.4 g/ h s
been ifficu t bec u se of the oc tion of her ho e. nee e . H is e ectro ytes re nor exce t
She fee s et che fro her hu sb n n f iy for ot ssiu eve of 3.4 o / L. A recent
n h s b n one ns to u rsue her c reer s igoxin eve is not v i b e. Which of the
inter. She h s ifficu ty f ing s ee n is e si y fo ow ing interventions shou be consi ere
st rt e by hone c s. As resu t of her ex erience, first?
she no onger goes to u b ic ces one.
(A) Checking the hy roch orothi zi e eve .
47. Which of the fo ow ing e ic tions w ou be (B) Monitoring orthost tic vit s.
ost he fu in tre ting the sy to s she is (C) Re cing the ot ssiu .
ex eriencing? (D) Sto ing the roo sec u sion.
(A) A r zo (E) T ering the or ze .
(B) C rb ze ine
(C) F u oxetine Questions 50 and 51
(D) N trexone A 64-ye r-o n, cco nie by his w ife, res-
(E) Thiori zine ents to you r office co ining of e ory oss. H e
is retire stockbroker w ho c n no onger rec
48. She is tre te w ith the ro ri te e ic - stock qu otes ike he i sever ye rs go. H is w ife
tion n h s signific nt re u ction in ost h s beco e both concerne n nnoye bec u se
of her sy to s. H ow ever, she continu es he see s to i e i te y forget w h tever she te s
to ex eriences frequ ent, intense night res, hi . These e ory rob e s h ve s ow y ro-
resu ting in her h ving oor s ee n f tigu e gresse over sever ye rs. Recent y, he h iffi-
the next y. Which of the fo ow ing gents cu ty getting resse , n ore th n once, he u t
w ou be the ost benefici in re u cing these his u n erw e r on over his nts. H is w ife h s so
sy to s? note th t he s e ks u ch ess u ring inner th n
he once i . H e oes not h ve ny other signific nt
(A) Lor ze
e ic i nesses.
(B) N trexone
(C) O nz ine 50. Which of the fo ow ing e ic tions w ou be
(D) Per hen zine ost ike y to i rove his con ition?
(E) Pr zosin (A) A r zo
(B) Done ezi
49. You h ve been ske to ev u te n 80-ye r-
o e nu rsing ho e resi ent bec u se of (C) F u oxetine
ch nge in ent st tu s. The nu rsing st ff (D) Methy heni te
re orts th t he h s been tient there for (E) Tr zo one
bou t ye r w ithout ny rob e s, u nti
Que s tions : 4756 91

51. U on fu rther questioning, his w ife re orts th t 54. A 48-ye r-o w o n resents to you r r ctice
he oes not s ee w e t night n w n ers w ith co ints of e ression. H er other
rou n the hou se i ess y. Su bsequ ent y, he sse w y 6 onths go fro u ng c ncer
s ee s for rge rt of the y. Which of n her f ther ie 4 ye rs go fro the s e
the fo ow ing e ic tions w ou be the ost ise se. She h s seen ther ists to he w ith
u sefu to contro this beh vior? her grief but she continu es to h ve ongoing
s ness, ter in inso ni , ow energy, oor
(A) H o eri o
etite, n ifficu ty concentr ting. She
(B) Lor ze h s ssive thou ghts of su ici bu t no n
(C) Tr zo one or intent. She enies rior su ici e tte ts.
(D) Tri zo In ition, es ite sever tte ts to qu it
(E) Qu eti ine s oking, she sti s okes tw o cks of cig -
rettes y n h s h w orsening sth s
Questions 52 and 53 resu t. She h s no ition e ic rob-
e s. Which of the fo ow ing e ic tions
A 25-ye r-o n is brou ght into the e ergency w ou be the ost ro ri te to u se in this
e rt ent eth rgic n stu orou s. H e res on s tient?
on y to infu sti u i, w kes u brief y n ye s, then
goes b ck to s ee . A bu nce ersonne re ort (A) Bu ro ion
th t they fou n hi ne r hou se know n for ru g (B) Bu s irone
tr fficking. There is no evi ence of hysic inju ry. (C) I i r ine
(D) Phene zine
52. Which of the fo ow ing e ic tions shou he (E) Tr zo one
receive first?
(A) Dextrose n f u zeni 55. A 52-ye r-o n w ho h s oor y contro e
(B) Dextrose, f u zeni , n n oxone i betes is referre to you for ev u tion of
e ic tion nonco i nce. You e rn th t he
(C) Dextrose, f u zeni , n oxone, n
c nnot f s ee t night n h s no energy
thi ine
u ring the y. H is etite is gone n he
(D) Dextrose n n oxone
oes not enjoy ctivities s he once i . Over-
(E) Dextrose, n oxone, n thi ine he fee s e resse . A ition y, he h s
been ex eriencing chronic in in his feet,
53. The tient is su bsequent y tre te n u rine ee e to be u e to neu ro thic in, w hich
toxico ogy screen is ositive for the resence of is not re ieve by n gesics. Which of the fo -
o ioi s. When he is ore ert, he infor s you ow ing e ic tions w ou be the ost u sefu
th t he h s been using intr venous (IV) heroin in tre ting this tients sy to s?
i y for sever w eeks. H is st use w s bout
8 hours go. H is heroin u se esc te fro (A) A r zo
snorting to IV use fter he no onger fe t goo (B) A itri ty ine
high fro snorting. H e h s never etoxifie (C) Bu ro ion
before. H e h s no co orbi e ic con i- (D) Cit o r
tions n he oes not use coho . Which of (E) Du oxetine
the fo ow ing e ic tions w ou be the ost
ro ri te to etoxify this tient? 56. A 47-ye r-o w o n resents to your office
(A) Bu renor hine for sychi tric c re bec u se she h s ju st re o-
(B) Ch or i ze oxi e c te to you r tow n. She h s history of bi o-
r isor er, bu t her rior sychi trist recent y
(C) C oni ine
sto e her ithiu bec u se she eve o e
(D) N oxone
thyroi uto ntibo ies. She co ins of being
(E) Pro r no o irrit b e n not b e to s ee for sever weeks.
92 3: S oma tic Tre a tme nt a nd P syc hop ha rma c ology

She h s ots of energy bu t is king is- (D) Sw itch to o nz ine.


t kes t w ork s she is istr cte . She te s you (E) Sw itch to ris eri one ong- cting
th t her thou ghts re r cing n you h ve injection.
ifficu t ti e re irecting her nsw ers. H ow -
ever, she be ieves her biggest rob e is her Questions 59 and 60
f ci in, w hich w s i gnose s trige i-
n neur gi n h s not i rove w ith n - A 50-ye r-o n w ho h s fe t e resse for sev-
gesics. Which of the fo ow ing e ic tions er onths, resents to you r office for ev u tion.
w ou be the ost ro ri te in tre ting this He h s h oor s ee , ecre se etite w ith
tients sy to s? w eight oss, no interest in his hobbies, n c nnot
concentr te u ring you r cognitive ex in tion. H e
(A) A itri ty ine h s ssing thou ghts of ying bu t no ctive n or
(B) C rb ze ine intent. H e re u ct nt y its to frequ ent y he ring
(C) F u oxetine voices te ing hi th t he is going to ie soon. You
(D) G b entin su bsequ ent y eci e to st rt n nti e ress nt n
(E) Lithiu ris eri one t 0.5 g bi .

57. A 33-ye r-o w o n w ith history of schizo- 59. Which of the fo ow ing si e effects of ris eri-
hreni h s recent y st rte tre t ent w ith one w ou be the ost ike y?
o nz ine. She h s to er te the e ic tion (A) Agr nu ocytosis
w e n is iving in grou ho e, ntici t- (B) Anticho inergic si e effects
ing oving into her ow n rt ent. She no
(C) Leu kocytosis
onger he rs voices n no onger h s esire
to hurt herse f. Which of the fo ow ing si e (D) Orthost tic hy otension
effects w ou be ost ike y to interfere w ith (E) Weight oss
her co i nce?
60. As his ose of ris eri one is incre se , the
(A) Agr nu ocytosis risk of w hich of the fo ow ing si e effects w i
(B) Deve o ent of c t r cts so incre se?
(C) G ctorrhe
(A) Agr nu ocytosis
(D) Incre se excit bi ity
(B) Anticho inergic effects
(E) Weight g in
(C) Extr yr i sy to s
58. A 32-ye r-o e w ith schizo hreni is (D) Leu kocytosis
being tre te for n cu te ex cerb tion of his (E) Weight oss
u itory h u cin tions. As is his ttern, he
res on s very f vor b y to e ic tions w hi e 61. A 33-ye r-o rrie fe e w ith history
hos it ize , bu t w hen he is isch rge for- of bi o r isor er, w ith eri rtu onset,
gets to t ke the regu r y. This resu ts in h s been st b e since being int ine on ris-
cyc e of subsequent w orsening of his sy- eri one for the st sever ye rs. H er son is
chosis n rehos it iz tion. H e is current y oing w e , n the tient n hu sb n w ish
t king ris eri one 5 g/ or y, n he is to h ve nother chi . For the st ye r she
to er ting it w e w ith goo effic cy. Which of h s been tte ting to conceive w ithou t su c-
the fo ow ing w ou be the ost ro ri te cess. She is on no other e ic tions besi es
next ste in his tre t ent? the nti sychotic, n she h s not been u sing
birth contro . She h s no other e ic rob-
(A) Continu e the cu rrent regi en. e s, n she is current y sy to tic, w ith-
(B) Sw itch to c oz ine. ou t e ression or sychotic sy to s. She
(C) Sw itch to h o eri o . rinks 1 to 2 g sses of w ine er w eek, enies
i icit rugs, n oes not use tob cco ro ucts.
Que s tions : 5766 93

Invo ve ent of w hich of the fo ow ing br in 64. A 45-ye r-o fe e execu tive w ho tr ve s
thw ys is ost ike y res onsib e for her often to Eu ro e fro the Unite St tes res-
inferti ity? ents to you r office co ining of signific nt
inso ni n yti e f tigu e fter tr ns t n-
(A) Locus ceru eu s
tic f ights. Which of the fo ow ing e ic tions
(B) Mesocortic tr ct w ou be the ost ro ri te tre t ent for
(C) Meso i bic tr ct her co ints?
(D) N igrostri t tr ct
(A) A itri ty ine
(E) Tu beroinfu n ibu r tr ct
(B) Di henhy r ine
Questions 62 and 63 (C) Me tonin
(D) Qu eti ine
A 19-ye r-o co ege stu ent resents to the office (E) Te ze
t the u rging of her frien s, w ho re concerne
bout her w e -being. For the st sever onths, Questions 65 and 66
she h s been incre sing y reoccu ie w ith her
w eight n bo y i ge, n h s been regu r y A 60-ye r-o n resents to you r office co in-
c nce ing ns w ith frien s bec u se she oes not ing of the su en onset of it tions, sw e ting,
w nt to e t w ith other eo e. She st tes th t she is sh king, shortness of bre th, choking, n fee ing
f t n w ou ike to ose 10 b. When qu erie fu r- th t he is going cr zy. These sy to s st 5 to
ther, she its th t she is bingeing n throw ing 10 inu tes n h ve occu rre sever ti es er
u ost every y. She enies ny x tive use, w eek for the st 3 onths. H e enies ny rtic-
n she h s not been restricting her e ting. She is u r stressor or event w hich rovokes the tt cks,
5 ft 4 in n w eighs 110 b. H er ot ssiu is 3.4; thou gh he is const nt y fr i of h ving futu re
other e ectro ytes re w ithin nor i its. You e iso es. H e h s recent y been seen by his c r io o-
re ete her ot ssiu n refer her to sychi trist gist w ho h s ru e ou t c r i c etio ogy.
for fu rther ev u tion.
65. Which of the fo ow ing w ou be the ost
62. Which of the fo ow ing w ou be the ost ro ri te e ic tion to tre t this tients
effective e ic tion for her con ition? isor er?
(A) Bu ro ion (A) Bu ro ion
(B) F u oxetine (B) I i r ine
(C) L otrigine (C) Phene zine
(D) Lithiu (D) Qu eti ine
(E) Mirt z ine (E) Sertr ine

63. You ter e rn th t she w s tre te in high 66. If the tients sy to s w ere u e to c r-
schoo for the s e sy to s w ith sertr ine i c etio ogy n he su bsequ ent y bec e
n cit o r , both t high oses for over e resse , w hich e ic tion w ou ose the
3 onths e ch, bu t w ith itt e i rove ent in highest risk?
her sy to s. Which of the fo ow ing e i-
(A) A itri ty ine
c tions w ou be the ost ro ri te next
ste ? (B) Bu ro ion
(C) Methy heni te
(A) Bu ro ion
(D) Sertr ine
(B) Lithiu
(E) Ven f xine
(C) P roxetine
(D) To ir te
(E) V roic ci
94 3: S oma tic Tre a tme nt a nd P syc hop ha rma c ology

67. A 33-ye r-o w o n w ith i gnosis of of the fo ow ing si e effects o you infor hi
schizo hreni h s been int ine on h o- is the ost ike y?
eri o since she w s i gnose t ge 24. In
(A) He che
you r office, she s ys she h s not h her en-
stru erio in sever onths, h s i in- (B) H e ring oss
ishe sex rive, h s been isch rging i k (C) Me ory oss
fro her bre sts, n h s in u ring sexu (D) Seizures
intercou rse. Which of the fo ow ing w ou (E) Tooth in
ost ike y be incre se in this tient?
(A) A renocorticotro ic hor one (ACTH ) 70. You h ve been tre ting 33-ye r-o w o n
w ith bi o r isor er for 4 ye rs. In re r -
(B) Do ine
tion for nne regn ncy, you h ve gree
(C) N ore ine hrine to t er her e ic tions but see her fre-
(D) Pro ctin qu ent y in c se of nic e iso e. Six w eeks
(E) Serotonin into her regn ncy, she resents to you r office
w ith ressu re s eech n the be ief th t her
68. A 24-ye r-o n is hos it ize bec use b by is Jesu s Christ. H er hu sb n s ys he is
he h s been he ring voices for 6 onths te - concerne bec u se she h s sto e t king
ing hi to ki hi se f n now fe rs th t he her ren t vit ins n is cting biz rre y.
y ct on the . H e ost his job 2 onths go Which of the fo ow ing tre t ents w ou be
n on ent st tu s ex in tion e rs the s fest to tre t her isor er?
u nke t n isorg nize , w ith f t ffect.
(A) C rb ze ine
H e is begu n on ri i r zo e, n his sy -
to s eventu y ecre se enou gh for hi to (B) Di ze
be isch rge . On fo ow -u visit to you r (C) Div roex so iu
office, he re orts th t his h ucin tions h ve (D) E ectroconvu sive ther y (ECT)
i rove bu t th t he c nt sit sti n fee s (E) Lithiu c rbon te
ike he nee s to be const nt y in otion. The
tient inter ittent y st n s u n w ks 71. You re the consu t- i ison sychi trist on c
rou n you r office s you interview hi . for the tr u su rgery te . You re ske to
Which of the fo ow ing con itions ccount for ev u te 39-ye r o n w ho w s hit by
his new co ints? c r n severe y inju re . H e is un b e to give
(A) Acu te ystonic re ction history, bu t n cqu int nce w ho cco -
nie the tient to the hos it re orts th t he
(B) Ak thisi
is i y, he vy ru g u ser. On ex in tion,
(C) M nic e iso e the tient is note to h ve tr ck rks on his
(D) N eu ro e tic ign nt syn ro e r s. Which of the fo ow ing c sses of ru gs
(E) T r ive yskinesi o you te the surgeons is the ost ngerou s
if bru t y w ith r w n?
69. A 55-ye r-o n w ith ong history of jor
(A) C nn binoi s
e ression resents to your office fter u ti-
e f i e nti e ress nt tri s. Since the ge of (B) H u cinogens
20, he h s h equ te tri s of nortri ty ine, (C) O i tes
hene zine, f uoxetine, sertr ine, bu ro ion, (D) Se tive-hy notics
irt z ine, ven f xine, n u oxetine, w ith (E) Sti u nts
ini res onse. Other sychi trists h ve
suggeste ECT in the st, bu t he is concerne Questions 72 and 73
bout e ory oss. H e w ou ike to try tr n-
scr ni gnetic sti u tion (rTMS), n A 44-ye r-o n w ith history of chronic, severe
w nts to know ore bout si e effects. Which coho u se isor er is brou ght to the e ergency
Que s tions : 6781 95

roo . You get re ort fro f i y e ber th t st rte on cit o r n h s n incre se of


he h s been iving off booze n ru gs n nothing 11 sec in his QTc v u e on ECG.
e se for w eeks. The e ergency e rt ent nu rse
is bou t to go in to r w b oo fro the tient n 75. A 51-ye r-o w o n being tre te for schizo-
offer hi foo . hreni eve o s i betes.

72. Which of the fo ow ing is the ost ro ri te


next ste in the tre t ent of this tient? 76. An 11-ye r-o gir being tre te for jor
e ressive isor er re orts frequ ent su ici
(A) A inister or ze . i e tion.
(B) A inister thi ine.
(C) Obt in co ete b oo cou nt. 77. A 34-ye r-o w o n being tre te for bi o-
(D) Obt in iver fu nction tests. r I isor er eve o s cu te in in her b o-
(E) Obt in vit in B12 eve . en n is i gnose w ith ncre titis.

73. After he is itte to the e icine service, 78. The WBC count of 27-ye r-o n being
w hich of the fo ow ing e ic tions w ou be tre te for the tre t ent refr ctory schizo-
the ost ro ri te to initi te? hreni ro s to 3,000/ L.
(A) Ac ros te
(B) C oni ine 79. A 42-ye r-o n sto s his nti e ress nt.
(C) Disu fir Tw o ys ter, he ex eriences fee ing of
(D) Lor ze ho e essness, h s ifficu ty s ee ing, n fee s
qu ite nxious.
(E) N trexone

D IRECTION S (Questions 74 through 83): The fol- 80. A 36-ye r-o wo n being tre te for bi o r
low ing group of numbered items are preceded by isor er fin s herse f urin ting very frequent y.
a list of lettered options. For each vignette, select
the one lettered option that is most closely associ- 81. Sever onths fter st rting on new nti-
ated w ith it. Each lettered option may be used sychotic, 32-ye r-o tients tot cho es-
once, multiple times, or not at all. tero incre ses fro 200 to 250 g/ L.

Questions 74 through 81
Questions 82 and 83
For e ch vignette, choose the e ic tion ost ike y
For e ch vignette, choose the e ic tion ost
to be ssoci te w ith the si e effect.
ro ri te to rescribe.
(A) C rb ze ine
(A) A r zo
(B) C oz ine
(B) Cit o r
(C) Div roex so iu
(C) Div roex so iu
(D) H o eri o
(D) F u oxetine
(E) Lithiu
(E) H o eri o
(F) P roxetine
(F) Lithiu
(G) Per hen zine
(G) C oz ine
(H ) Pi ozi e
(H ) P roxetine
(I) Zi r si one
(I) Ris eri one
74. A 14-ye r-o boy being tre te h r co- (J) Sertr ine
ogic y for Tou rette isor er is ition y (K) Tr zo one
96 3: S oma tic Tre a tme nt a nd P syc hop ha rma c ology

82. A 34-ye r-o n w ith schizo ffective isor- (C) C o i r ine


er h s been nonco i nt w ith e ic tions (D) F u oxetine
in the st. You w nt to be b e to give hi (E) F u vox ine
e ot injection of n nti sychotic.
Questions 86 and 87
83. You re tre ting 45-ye r-o veter n w ith
You re tre ting 28-ye r-o n w ith 6-ye r
history of coho use isor er for osttr u -
history of schizo hreni w ho ives w ith his rents.
tic stress isor er (PTSD), n he sks you
The tient h s h nu ber of ebi it ting si e
for so ething to he hi s ee .
effects, su ch s severe extr yr i sy to s,
fro tr ition nti sychotics inc u ing h o eri-
D IRECTION S (Questions 84 through 105): For o . H is f i y is w orrie bou t w eight g in, w hich
each of the multiple-choice questions in this sec- the tient ex erience on o nz ine. Des ite the
tion, select the lettered answ er that is the one best nu erous e ic tions, his sychotic sy to s
response in each case. continu e. You iscu ss c oz ine w ith the tient
n his f i y n they gree to try the e ic -
Questions 84 and 85 tion. You h ve initi te tre t ent s ow y n he h s
A 28-ye r-o sing e w o n resents w ith 8 w eeks show n signific nt i rove ent; how ever, he h s
of e ression, inso ni , ow energy n f tigu e, been ro u cing rge ou nt of s iv n roo
istr ctibi ity, n oor etite. She h s no rior so king his shirts.
sychi tric history n no f i y sychi tric his-
tory. After further history n bor tory ev u - 86. Which of the fo ow ing is the ost ro ri te
tions re co ete , she is begu n on sertr ine, next ste ?
w hich is eventu y incre se to 100 g/ . After (A) A nt ine.
2 onths on th t ose, the tient re orts th t she (B) A ro r no o .
fee s b ck to nor n sks w hen she c n sto
(C) A su b ingu tro ine.
the e ic tion.
(D) Discontinu e c oz ine.
84. H ow ong shou you reco en her to (E) Incre se the ose of c oz ine.
continu e the e ic tion?
87. In ition to the bove, the tient co ins
(A) 1 onth of it tions. An ECG reve s sinu s t chy-
(B) 2 onths c r i , w hich is consistent w ith rior ECGs
(C) 4 onths erfor e over the st onth. Which of the
(D) 10 onths fo ow ing is the ost ro ri te next ste ?
(E) 2 ye rs (A) A benztro ine.
(B) A bet o .
85. She returns for fo ow-u visit 1 onth ter.
(C) A or ze .
Her sy to s h ve continue to re in in
(D) A ro r no o .
re ission, though she escribes recent con-
f ict with her boyfrien ue to h ving itt e sex (E) Discontinu e c oz ine.
rive. The tient st tes th t it h s gotten to the
oint where she h s eci e to sto the e ic - 88. A 21-ye r-o sing e fe e co ege stu ent
tion un ess so ething c n be one. She is wi - is brou ght to the u niversity he th service by
ing to switch to nother e ic tion es ite the her roo tes. She kes it c e r to you th t
risk of re se. Which of the fo owing gents she oes not think theres nything w rong
wou be the ost ro ri te choice? w ith her, bu t her roo tes te you th t they
c nt e w ith her ny ore bec u se she
(A) Bu ro ion never s ee s n never shu ts u . The tient
(B) Cit o r insists this is bec u se she h s iscovere n
Que s tions : 8293 97

i ort nt the tic roof th t she u st 91. The tient is 59-ye r-o fe e w ith schizo-
finish w riting. Shes convince th t the uni- hreni , s w e s oor y contro e i betes
versity w i i e i te y gr nt her tenure s e itu s, obesity, hy er i i e i , n hy er-
soon s they see the roof, thou gh she h s tension. She h s been tre te for over 35 ye rs
troub e st ying on to ic w hen you sk qu es- w ith v riou s first-gener tion nti sychotics,
tions bout it. She h s not seen sychi trist n thou gh she is on n equ te ose, she
before, n u rine toxico ogy screen, w hich sti h s resi u sychotic sy to s n h s
she consents to to rove I not cr zy, is now begu n to eve o invo u nt ry b inking
neg tive. Which of the fo ow ing is the ost n tongu e-ro ing ove ents. She is gree-
ro ri te gent for initi n inten nce b e to sw itching to n ty ic / secon -gen-
tre t ent of this tient? er tion nti sychotic. Which of the fo ow ing
e ic tions w ou be the ost ro ri te
(A) H o eri o
for this tient?
(B) L otrigine
(C) Lithiu (A) Ari i r zo e
(D) Lor ze (B) C oz ine
(E) O nz ine (C) O nz ine
(D) Qu eti ine
89. A 34-ye r-o e ectric engineer co ins to (E) Ris eri one
his octor th t he h s been fee ing e resse
for sever onths fter the bre ku of his 92. You ev u te 34-ye r-o ivorce , e, w ith
rri ge. Since th t ti e, he h s h signifi- history of coho use isor er w ho resents
c nt inso ni , ifficu ty concentr ting, h s w ith the chief co int of I c nt sto rink-
ost 25 b in the st 3 onths bec u se his ing, oc. He h s h sever reh bi it tion
etite isnt the s e, fee s f tigu e , h s sig- issions but is un b e to int in sobriety.
nific nt fee ings of gu i t reg r ing the se r - He is current y rinking one-fifth of vo k
tion, fee s ho e ess n he ess, n h s fe t ost i y. Whi e he is very iscour ge n
su ici w ithou t s ecific n. A i gnosis of gui ty bout his coho use, he is ho efu bout
jor e ressive isor er is e n he is getting better, n he enies erv sive e res-
begu n on n nti e ress nt. Which of the fo - sive sy to s or suici i e tion. H e h s
ow ing sy to s w ou be ex ecte to t ke rior history of signific nt coho w ith r w ,
the ost ti e to i rove? inc u ing one seizure n e iriu tre ens,
both requiring n ICU ission. Which of the
(A) Decre se etite
fo ow ing e ic tions w ou be ost ro-
(B) Decre se concentr tion ri te to rescribe for this tient?
(C) Decre se energy
(A) Disu fir
(D) Inso ni
(B) Meth one
(E) Su ici ity
(C) N trexone
90. A 58-ye r-o n w ith history of cirrho- (D) Sertr ine
sis, he titis C, n coho is w ishes to qu it (E) V roic ci
rinking bu t w nts to etox first. Which of
the fo ow ing e ic tions w ou be the ost 93. A 35-ye r-o sing e n resents to you r
ro ri te to rescribe in this tient? office co ining of inso ni . H e re orts
th t he is b e to f s ee t night, bu t st ys
(A) A r zo
s ee for on y 2 or 3 hou rs, then w kes u fre-
(B) Ch or i ze oxi e quent y throu ghout the night. H e is exh uste
(C) C on ze n w ou ike so ething to he hi s ee .
(D) Di ze Other th n fee ing fru str te n f tigue
(E) Lor ze u ring the y, he enies other e ressive
98 3: S oma tic Tre a tme nt a nd P syc hop ha rma c ology

sy to s or ny st sychi tric history. e ic tions w ou be the ost ro ri te to


Which of the fo ow ing e ic tions w ou be rescribe?
the ost ro ri te for this tient?
(A) Bu s irone
(A) A itri ty ine (B) F u oxetine
(B) R e teon (C) F u vox ine
(C) Tr zo one (D) Lor ze
(D) Z e on (E) Pro r no o
(E) Zo i e
97. A 30-ye r-o e resents w ith 4 w eeks
94. A 40-ye r-o rrie , regn nt w o n w ith of ow energy, oor s ee , oor etite,
history of hy ertension, yoc r i inf rc- ecre se interest in e su r b e ctivities,
tion, egener tive isc ise se, n jor n ow oo . H e re orts being hos it -
e ressive isor er w ith sychotic fe tu res ize s you ng u t for 2-w eek erio of
is being consi ere for e ectroconvu sive gr n iosity, hy ersexu ity, ecre se nee
ther y (ECT) fter f i ing u ti e nti e- for s ee , n eng ging in risky beh viors. H e
ress nt n nti sychotic tri s. Which of w s st rte on ithiu bu t i not to er te the
the fo ow ing con itions w ou be re tive e ic tion u e to eve o ent of i betes
contr in ic tion to ECT? insi i u s n h s not t ken e ic tion since.
H e w orks s n ccou nt nt, his BMI is 35, n
(A) Degener tive isc ise se
his st A1c w s 5.2. Which of the fo ow ing
(B) H y ertension e ic tions w ou be the ost ro ri te to
(C) Pregn ncy rescribe for this tient?
(D) Psychotic e ression
(A) Lithiu
(E) Recent yoc r i inf rction
(B) Lu r si one
95. You re c e to consu t on 70-ye r-o (C) Qu eti ine
w o n, hos it ize in the ICU fter yo- (D) Sertr ine
c r i inf rction. The tient is ye ing t (E) V roic ci
st ff, u ing ou t her IV n trying to get out
of be . She refuses to t ke ny e ic tion by 98. A 29-ye r-o w o n w ith history of sei-
outh. H er ECG show s QTc of 480. Which zu res is i gnose w ith bi o r isor er,
of the fo ow ing e ic tions w ou be best to ost recent e iso e e resse , n st rte
he her git tion? on otrigine. Abou t w hich of the fo ow ing
si e effects w ou be the ost i ort nt to
(A) Ari i r zo e IM
e u c te the tient?
(B) Di ze IV
(C) H o eri o IV (A) Acu te ystoni
(D) Lor ze IV (B) Ak thisi
(E) Ris eri one Const IM (C) A stic ne i
(D) R sh
96. A 30-ye r-o w o n w ith no reviou s sy- (E) Ren f i u re
chi tric history resents to you r office sking
for he in n ging erfor nce nxiety. 99. A 19-ye r-o n w ith bi o r isor er is
She oves to ct n sing in her co unity brought in to the e ergency roo by the
the tre, but she beco es terrifie w hen get- o ice. H e is gr n iose n e usion , s w e
ting on st ge in front of crow s. As resu t, s git te n co b tive. H e is given n
her voice sh kes n she h s been un b e intr uscu r injection of h o eri o . After
to get goo rts. Which of the fo ow ing 30 inutes, the tient is fou n rching for-
w r . On hysic ex in tion, stiffness of the
Que s tions : 94104 99

neck n b ck usc es re note . Which of the 102. A 58-ye r-o n w ith tri fibri tion,
fo ow ing ter s best escribes this si e effect? hy ercho estero e i , n hy ertension is
sent to you by his ri ry c re hysici n
(A) L ryngos s
for he in n ging his nxiety. H e re orts
(B) Ocu ogyric crisis th t for ny ye rs he h s w orrie bout
(C) O isthotonos everything, n th t he occ sion y h s to
(D) P eu rothotonos st y ho e fro w ork bec use he beco es so
(E) Tortico is fe rfu . H e often h s ifficu ty f ing s ee
n not u nco on y gets tension he ches
100. The tient is 59-ye r-o w o n w ith st w hen rticu r y stresse . H is e ic tions
e ic history of hy ercho estero e i , i - inc u e w rf rin, torv st tin, n hy ro-
betes, n i betic neu ro thy, w ho resents ch orothi zi e. H e oes not rink coho or
to her ri ry c re octor w ith the chief co - c ffeine. Which of the fo ow ing e ic tions
int of he ches. U on further interview, w ou be the ost ro ri te to tre t this
she st tes her sy to s beg n fter the e th tients sy to s?
of her hu sb n 5 onths go. Since th t ti e, (A) Bu ro ion
she h s fe t e ty, w ith oor s ee , f tigu e,
(B) Bu s irone
ecre se concentr tion, n w ishes th t she
w ou ie so I c n join hi . H er review of (C) Cit o r
syste s is re rk b e for ongoing sh r ins (D) C on ze
n nee es in her extre ities. H er hysic (E) Ven f xine
ex in tion is u nre rk b e, n her b oo
su g rs h ve been re tive y w e contro e . 103. A 40-ye r-o n is st rte on f u oxetine for
Which of the fo ow ing w ou be the ost jor e ressive isor er. H e notes i rove-
ro ri te tre t ent for her sy to s? ent in his oo , bu t h s ifficu ty s ee ing.
Tr zo one is su bsequ ent y e to his regi-
(A) Bu ro ion
en. Three ys ter, he c s co ining of
(B) Du oxetine infu erection th t h s not su bsi e fter
(C) F u oxetine sever hou rs. Which of the fo ow ing best
(D) Mirt z ine escribes this tients con ition?
(E) Re ssu r nce (A) Erecti e ysfu nction
(B) Ny ho ni
101. A 44-ye r-o rrie e tient w ith his-
tory of recurrent jor e ressive isor er (C) P r r xis
resents to sychi trist fter sto ing his ser- (D) Pri is
tr ine 1 onth go. A though it h s been effec- (E) S tyri sis
tive in the tre t ent of his e ression, he h s
h incre sing ifficu ty int ining n erec- 104. A 35-ye r-o rrie fe e is being fo -
tion, n this h s c use friction n conf icts ow e by sychi trist for her first e iso e
w ith his w ife. H e is concerne , how ever, th t of jor e ression. She is rescribe cit o-
he w i h ve re se of his e ressive sy - r 40 g i y n h s been in re ission
to s if not e ic te . Which of the fo ow ing for 3 onths. She now co ins of the new
w ou be the ost ro ri te h r co ogic onset of ecre se ibi o w hich is c u sing her
tre t ent for this in ivi u ? o er te ou nt of istress. Which of the
fo ow ing w ou be the ost ro ri te to
(A) Begin irt z ine.
to her cu rrent regi en?
(B) Begin roxetine.
(C) Begin ven f xine. (A) Bu ro ion
(D) N o e ic tion. (B) Desi r ine
(E) Rest rt sertr ine. (C) Lithiu
100 3: S oma tic Tre a tme nt a nd P syc hop ha rma c ology

(D) Tr ny cy ro ine 107. A 16-ye r-o boy is brou ght to you by his
(E) Ven f xine other. She st tes th t, es ite h ving high
IQ, he oesnt co ete his ho ew ork, h s
105. A 41-ye r-o w o n resents to you r office been f i ing in schoo , n h s sto e going
sking for he in n ging her binge e t- out w ith frien s. H e its to s oking tw o to
ing. She e ts rge ou nts of ju nk foo three three joints of riju n i y.
or fou r ti es w eek, hi es foo , n su bse-
qu ent y fee s gu i ty bou t her ctions. H er 108. A 21-ye r-o n is brou ght into the e er-
BMI is 31. After erfor ing thorou gh his- gency e rt ent by the o ice fter getting
tory n hysic ex in tion, you eci e to rreste for ste ing g so ine. The tient fee s
rescribe to ir te. Which of the fo ow ing isconnecte fro re ity n frightene . On
si e effects o you te her is ost ike y? hysic ex in tion, he is note to h ve
(A) I rove ttention r sh rou n his ou th.
(B) P ncre titis
(C) Po yu ri 109. A 54-ye r-o n w ith o i te u se isor er
re se on heroin fter sever onths of
(D) Ren stones
sobriety. Whi e he beg n u sing $10 to $20 er
(E) Weight g in
y t first, he qu ick y incre se his u se to $50
to $60 er y bec u se I ont get high now
D IRECTION S (Questions 106 through 109): The
u n ess I u se ore.
follow ing group of questions are preceded by a
list of lettered options. For each question, select
the one lettered option that is most closely asso- D IRECTION S (Question 110): Select the lettered
ciated w ith it. Each lettered option may be used answ er that is the one best response in this case.
once, multiple times, or not at all.
110. A 27-ye r-o n w ith schizo hreni notices
Questions 106 through 109 the eve o ent of bre sts, ecre se ibi o,
n not ej cu ting w hen he h s n org s .
For the fo ow ing vignettes, choose the correct ter Which of the fo ow ing e ic tions is ost
fro the ist be ow . ike y res onsib e for these si e effects?
(A) A otiv tion syn ro e (A) Ari i r zo e
(B) C nn bis (B) C oz ine
(C) For ic tion (C) Qu eti ine
(D) H u cinogen ersisting erce tion (D) Ris eri one
isor er (E) Zi r si one
(E) Inh nts
(F) Intoxic tion D IRECTION S (Questions 111 through 121): The
(G) Psycho ogic e en ence follow ing group of questions are preceded by a
(H ) Su bst nce u se isor er list of lettered options. For each question, select
the one lettered option that is most closely asso-
(I) To er nce
ciated w ith it. Each lettered option may be used
(J) With r w once, multiple times, or not at all.

106. A 45-ye r-o n w ith chronic coc ine u se Questions 111 through 121
isor er resents to the e ergency e rt-
ent w ith co ints of bugs cr w ing on his For the fo ow ing vignettes, choose the correct si e
skin. effect fro the ist be ow .
(A) Agr nu ocytosis
(B) Ak thisi
Que s tions : 105122 101

(C) B e h ros s gener ize rigi ity, i horesis, n tere


(D) Consti tion ent st tu s.
(E) G ctorrhe
(F) Gyneco sti 117. A 36-ye r-o n w ith schizo ffective is-
(G) N eu ro e tic ign nt syn ro e or er, e resse ty e h s been receiving f u -
hen zine ec no te 50 g intr u scu r y
(H ) Obstru ctive j u n ice
every 2 w eeks. Whi e his sy to s re w e
(I) O isthotonos
contro e , he h s recent y notice w hite
(J) Orthost tic hy otension isch rge fro his ni es, bi ter y.
(K) Pig ente retino thy
(L) Pseu o rkinsonis 118. A 26-ye r-o n being tre te w ith high-
(M) R bbit syn ro e ose thiori zine is fou n on o hth o ogic
(N ) Retrogr e ej cu tion ex in tion to h ve e ery ig ent tion
(O) T r ive yskinesi of the retin . Des ite iscontinu tion of the
(P) Tortico is e ic tion, the ig ent tion continu es n
the tient eventu y suffers tot b in ness.
111. A 35-ye r-o n w ith chronic schizo hreni
t king thiori zine notices th t, if he urin tes 119. A 22-ye r-o n co ins of severe rest-
fter sturb ting, his u rine e rs c ou y. essness n git tion fter st rting on ri i -
r zo e. H e co ins th t he c nnot sit sti
112. A 46-ye r-o C uc si n w o n w ith schizo f- n ces const nt y.
fective isor er, bi o r ty e h s been n-
ge for ny ye rs on ris eri one 3 g tw ice 120. A 26-ye r-o tient w ith schizo hreni is
i y or y. She enies ny co ints but on st rte on ch or ro zine. After 1 w eek, he
ent st tus ex in tion is note to h ve co ins of f u - ike sy to s. A CBC is
re etitive, r i ove ents of her ou th (bu t obt ine n is nor . After nother w eek,
not tongue) w ith s cking of her i s. the tient eve o s ye ow ish co or to his
skin n sc er .
113. A 22-ye r-o Afric n-A eric n n on c o-
z ine for 3 onths re orts sore thro t n 121. A 45-ye r-o tient w ith schizo hreni is
fever. A CBC is obt ine w ith WBC cou nt of st rte on high ose of ch or ro zine. The
3
1,000/ w ith 1% gr nu ocytes. next y, he co ins of being ight-he e
every ti e he st n s. On ex in tion, he is
114. A 43-ye r-o n w ith schizo hreni is note to h ve signific nt ecre se in b oo
on h o eri o . H e is note to h ve f exe ressu re n n incre se in he rt r te.
osture, festin ting g it, resting tre or, n
br ykinesi . D IRECTION S (Questions 122 and 123): For each of
the multiple-choice questions in this section, select
115. The s e 43-ye r-o n w s continu e the lettered answ er that is the one best response in
on h o eri o es ite his si e effects n each case.
fter sever ye rs eve o e choreo thetoi
ove ents of the trunk n i bs, ong w ith 122. You re c e in the i e of the night by the
i s cking n tongu e ove ents. su rgic service to see 52-ye r-o n w ho
w s itte to the hos it for n e ergency
en ecto y. Review of the ch rt ocu ents
116. A 21-ye r-o n w ith sychosis w s nu erou s st hos it iz tions for coho
st rte on h o eri o t high oses. Eight intoxic tion n w ith r w . The su rgic
ys into his tre t ent, he eve o e fever, te is concerne bout his w ith r w ing
102 3: S oma tic Tre a tme nt a nd P syc hop ha rma c ology

w hi e in recovery. H ow ong fter this tients tri s of ithiu , v roic ci , n o nz -


st rink w ou you ost ike y ex ect to see ine, he is st rte on oxc rb ze ine. Three ys
serious w ith r w sy to s eve o ? ter, he co ins of n u se n w e kness;
he e rs confu se bou t w hy he is in the
(A) 6 hou rs
hos it . Which of the fo ow ing bor tory
(B) 12 hou rs stu ies w ou be the ost i gnostic in this
(C) 1 y c se?
(D) 3 ys
(A) BUN n cre tinine
(E) 7 ys
(B) CBC
123. A 33-ye r-o n w ith history of bi o r I (C) E ectro ytes
isor er is itte to the in tient u nit w ith (D) Liver fu nction tests
cu te ni . Bec u se he h s f i e reviou s (E) Oxc rb ze ine eve
Ans we rs a nd Expla na tions

1. (C) Phene zine is ono ine oxi se inhibi- MAOI c n resu t in otenti y ife-thre ten-
tor, use (r re y now but frequent y in the st) ing ru g inter ction know n s serotonin syn-
for the tre t ent of severe e ression. Phene - ro e. Me ic tions th t b ock su ch reu t ke
zine, tr ny cy ro ine, n isoc rbox zi re inc u e the SSRIs, TCAs, bu s irone, n other
irreversib e b ockers of MAO-A ( ono ine nti e ress nts su ch s ven f xine. Fe tu res
oxi se-A) n MAO-B ctivity; se egi ine, of i serotonin syn ro e inc u e tri
given or y or in the for of skin tch, is of ent st tus ch nges, utono ic hy er-
n MAO-B inhibitor th t w s eve o e for ctivity, n neu ro u scu r bnor ities.
the tre t ent of P rkinson ise se; t higher A ition sy to s inc u e t chyc r i ,
oses, it is nonse ective MAO inhibitor n f ushing, fever, hy ertension, ocu r osci -
is use in the tre t ent of e ression. These tions, n yoc onic jerks. Severe serotonin
e ic tions re extre e y ngerous in over- syn ro e y resu t in seriou s hy erther i ,
ose, n , fter brief sy to tic erio co , u tono ic inst bi ity, convu sions, n
of 12 to 24 hours, y ro uce hy er yrexi e th; therefore, one u st w it t e st 14 ys
n utono ic excit bi ity sufficient enough to fter iscontinu ing n MAOI before st rting
c use rh b o yo ysis. Su ortive c re shou serotonergic gent. Acu te ystoni (inc u ing
be institute . If e iriu eve o s, s oses o isthotonos) occurs w ith the u se of nti sy-
of IV benzo i ze ines shou be use . Lor ze- chotic e ic tions, not MAOIs. Ak thisi ,
is referre bec use of its short e i in tion or the inner fee ing of rest essness, is nother
h f- ife. N euro e tics, es eci y short- cting extr yr i si e effect of neu ro e tic
gents such s ch or ro zine, shou be rugs. N eu ro e tic ign nt syn ro e is
voi e bec use of their ten ency to contrib- n i iosyncr tic re ction to neu ro e tic ru gs
ute to hy otension. If ventricu r rrhyth i s rese b ing ign nt hy erther i .
eve o , they c n be tre te w ith i oc ine.
Cy rohe t ine is 5-HT2 nt gonist th t is 3. (A) This tient is ike y su ffering fro
so eti es use in the tre t ent of the ost jor e ressive e iso e. Bec u se of her
severe c ses of serotonin syn ro e. Me eri- w eight g in, hy erso ni , n oo re c-
ine shou be voi e bec use it y con- tivity, her e ression h s ty ic fe tu res.
tribute to the renergic crisis. Phenytoin y A thou gh ty ic fe tu res h ve tr ition y
be use if seizures eve o . Other e ic tions been thou ght to res on best to MAOIs su ch
th t re contr in ic te w ith MAOIs inc u e s hene zine, these e ic tions re consi -
sti u nts, econgest nts, ine recursors ere thir - ine gents bec u se of the otenti
such s l - o n l -try to h n, n the nti- risk of serotonin syn ro e n hy ertensive
hy ertensives ethy o , gu nethi ine, n crisis w hen e ting cert in tyr ine-cont ining
reser ine. foo s. SSRIs such s f uoxetine n roxetine
y be s effic cious s MAOIs in tre ting ty -
2. (E) B ocking reu t ke of c techo ines n ic e ression; how ever, roxetine shou
in o ines in tients re y u sing n be voi e given this tients hy erso ni

103
104 3: S oma tic Tre a tme nt a nd P syc hop ha rma c ology

n w eight g in. Bec u se of their f vor b e bsor tion. H o eri o c n be u se for its
s fety rofi e, SSRIs h ve beco e first- ine se tive effect bu t it shou not t ke the ce
tre t ent in jor e ression. Tricyc ic nti- of benzo i ze ine. Si i r y, IV bet o
e ress nts su ch s nortri ty ine re ess y be u sefu in the intensive c re unit bu t it
effective th n MAOIs n SSRIs for ty ic w i not n ge the u n er ying w ith r w
e ression. Tr zo one is n extre e y se t- n shou not re ce benzo i ze ine.
ing nti e ress nt th t is gener y u se for
ju nctive tre t ent of inso ni . 7. (B) This tient is su ffering fro jor e res-
sion w ith sychotic fe tu res. Best r ctice
4. (C) Most tients w ho re going to h ve reco en s co bin tion of n nti e res-
res onse to n nti e ress nt o so w ithin s nt n n nti sychotic e ic tion, such s
the first 6 w eeks. H ow ever, it c n t ke u to 12 f uoxetine n ris eri one. E ectroconvu sive
w eeks to chieve fu res onse. Few er th n ther y, not iste ong the nsw er choices,
50% of eo e w ith jor e ressive isor er is nother effective tre t ent for sychotic
chieve fu re ission fro their first nti e- e ression. N either nti e ress nt ono-
ress nt tri . ther y nor nti sychotic onother y is
s effective s the co bin tion. C oz ine,
5. (E) A coho w ith r w shou be strong y secon -gener tion nti sychotic, is reserve
consi ere in this tient. Benzo i ze ines for the tre t ent of tients w ith schizo hre-
re the ru g of choice for contro of coho ni w ho h ve f i e tw o or ore nti sychotic
w ith r w sy to s s w e s for ro hy- tri s. C on ze , benzo i ze ine, y
xis g inst w ith r w seizu res n the rovi e so e re ief fro nxiety, bu t w i not
otenti y ife-thre tening e iriu tre ens. tre t the u n er ying con ition.
Long- cting benzo i ze ines su ch s ch or-
i ze oxi e n i ze re ro ri te; 8. (D ) This tient is ex eriencing sy to s of
how ever, both re extensive y et bo ize ithiu toxicity, the severity of w hich is eter-
by the iver. Bec u se this tient h s e ev te ine by checking ithiu eve . A ther eu -
tr ns in ses, ox ze or or ze , w hich tic ithiu eve is 0.8 to 1.2. At eve s rou n
on y u n ergo g u cu roni tion ( n re there- 1.2, tients ex erience tre or, n u se , i r-
fore not e en ent on iver fu nctioning) rior rhe , n t xi ; this is fo ow e by seizu res
to e i in tion, re refer b e. The ne onic t eve s 1.5 to 2.0, then cu te ren f i u re
LOT (Lor ze , Ox ze , Te ze ) (requ iring i ysis) t eve s gre ter th n 2.0,
c n be he fu in re e bering w hich ben- w ith su bsequ ent co n e th t eve s
zo i ze ines re et bo ize in this w y. A bove 2.5. Bec u se ithiu h s su ch n rrow
high- otency nti sychotic gent su ch s h - ther eutic in ex, ehy r tion c n e to
o eri o y be necess ry to he contro this toxicity. Whi e c rb ze ine n e kote
tients git tion, how ever, n nti sychotic re both u se to tre t bi o r isor er, they
w i not revent coho w ith r w , n ow - h ve u ch w i er ther eutic in exes. A he
otency nti sychotics, su ch s ch or ro - CT is not in ic te in this c se, given th t the
zine, shou be voi e bec u se they y ost ike y c u se of the tients sy to s is
ctu y ow er the seizure thresho . Disu fi- ithiu toxicity. Si i r y, there is nothing in
r is u se to tre t coho u se isor er, not the vignette to su ggest subst nce intoxic tion
cu te coho w ith r w . s c u se of her sy to s.

6. (E) This tient h s e iriu tre ens, w hich 9. (D ) Ak thisi , ove ent isor er th t is
is e ic e ergency. H e shou be tr ns- frequ ent si e effect of nti sychotic e i-
ferre to n intensive c re setting w here IV c tion, c n be ifficu t to istingu ish fro
benzo i ze ines c n be inistere w ith nxiety in sychotic in ivi u ; how ever,
gre ter s fety. The IV rou te is referre in new -onset rest essness in tient recent y
this situ tion to ensu re equ te n r i st rte on nti sychotic e ic tion suggests
Answe rs : 415 105

this i gnosis. A thou gh ore co on w ith of the other iste co ic tions h ve been
first-gener tion nti sychotics, it c n be seen re orte in N MS bu t w ith ess frequ ency.
w ith secon -gener tion nti sychotics, s
w e , t r tes s high s 20%. Onset u su y 12. (D ) Retin ig ent tion is know n to occu r
begins betw een 5 n 60 ys fter initi - w hen thiori zine is u se in high oses
tion of tre t ent bu t c n occu r fter just one (>1,000 g/ ). It y not re it w hen thio-
ose. First, the nti sychotic (neu ro e tic) ri zine is iscontinu e n c n eventu y
ose shou be re u ce s u ch s ossib e. e to b in ness. The other choices c n occu r
If sy to s of k thisi continu e, in the w ith nti sychotics (neu ro e tics) n re
bsence of other extr yr i si e effects, not s ecific to thiori zine.
ny c inici ns w ou then st rt tre t ent
w ith bet b ocker su ch s ro r no o . In 13. (C) This tient is suffering fro nic isor-
gener , nticho inergic gents su ch s ben- er n gor hobi . There re ny e ic -
ztro ine re u se to tre t extr yr i tions th t h ve e onstr te effic cy in nic
sy to s su ch s ystoni or rkinsonis . isor er, inc u ing TCAs, SSRIs, MAOIs, n
Di henhy r ine is he fu in cu te ysto- high- otency benzo i ze ines. Of the gents
ni s bu t not in k thisi . Lor ze , ben- iste , on y f u oxetine, n SSRI, is one of the .
zo i ze ine, is n ro ri te secon - ine Bu ro ion, n nti e ress nt w ith o -
tre t ent for k thisi if sy to s re sti ine ctivity, n bu s irone, rti 5-H T1
not n ge or the tient c nnot to er te nt gonist w ith nti nxiety ro erties, re
bet b ocker. Given the tients i rove ent not effective in nic isor er. Pro r no o ,
of his sychotic sy to s, it is refer b e to bet - renergic b ocker, y he to evi te
continu e tre t ent w ith ri i r zo e us the hysic sy to s of nic su ch s t chy-
the bove junctive e ic tions r ther th n c r i , but it oes not revent the tt cks.
sw itching to nother nti sychotic. Tr zo one, n nti e ress nt w ith ixe
serotonergic effects, h s so shown conf icting
10. (C) This tient is su ffering fro bi o r II resu ts.
e ression, w ith history ch r cterize by
current jor e ressive e iso e n his- 14. (A) A r zo is the ost otent benzo i-
tory of hy o ni . The tre t ent of bi o r ze ine iste . Using 1 g of or ze for
e ression is controversi . Most tre t ent the equ iv ent ose in i igr s, the otency
gu i e ines reco en ithiu , otrigine, re tionshi of so e co on benzo i ze-
u r si one, or qu eti ine s first- ine o tions. ines is s fo ow s: r zo 0.5 g, i z-
V roic ci y be re son b e secon - ine e 10 g, ox ze 20 g, te ze
choice. Monother y with nti e ress nts (e.g., 20 g, n ch or i ze oxi e 25 g.
bu ro ion, f uoxetine, ven f xine) is not rec-
o en e ue to the risk of in ucing ni , 15. (E) Bec u se ithiu is grou IA onov -
n the use of nti e ress nts in conjunction ent ion, the ki ney h n es it u ch s it
with oo st bi izers re ins eb t b e. oes so iu . N inety-five ercent of ithiu
is excrete u nch nge throu gh the ki neys.
11. (E) This tient is ex eriencing neu ro e tic Therefore, e ectro ytes, cre tinine, n BUN
ign nt syn ro e (N MS), r re bu t seri- re requ ire to check b se ine ki ney func-
ou s si e effect of nti sychotic e ic tions. tion. Thyroi stu ies re so requ ire bec u se
Rh b o yo ysis is the ost co on nger- ithiu inhibits the synthesis of thyroi hor-
ou s co ic tion of N MS, occurring in u to one n its re e se fro the thyroi . A CBC
25% of tients in one series. Di ysis y be is o tion , thou gh ithiu y c u se
requ ire to rotect tients fro ren f i - benign e ev tion in the WBC. F sting g u cose
u re. The w hite b oo ce count n cre tinine n i i s shou be checke rior to initi t-
hos hokin se re often e ev te in N MS bu t ing tre t ent w ith nti sychotic gents. Liver
there re no s ecific bor tory fin ings. A fu nction tests shou be erfor e rior to
106 3: S oma tic Tre a tme nt a nd P syc hop ha rma c ology

initi ting tre t ent w ith iv roex so iu ; 20. (C) Few er th n h f of tients chieve fu
r re c ses of f t he totoxicity h ve been re ission fro e ression fter tre t ent
re orte w ith iv roex so iu . In ition, w ith n SSRI. Best r ctice w ou be to sw itch
iv roex so iu y so e ev te seru to nother SSRI (cit o r ) or n SN RI (ven-
oni eve s in r re c ses. f xine); how ever, bec u se of his history of
hy ertension, ven f xine, w hich c rries w ith
16. (E) In so e in ivi u s, nonsteroi nti- it 5% to 7% r te of hy ertension, is not the
inf tory gents (NSAIDs) c use n incre se best choice. N ortri ty ine, tricyc ic nti e-
in ithiu eve s. Therefore, these gents shou ress nt, is otenti y ngerou s choice in
be use w ith c ution in tients t king ithiu , tient w ith history of yoc r i inf rc-
n voi e if ossib e. As irin n su in c, tion. Au g ent tion w ith ithiu n thyroi
how ever, o not ffect ithiu eve s n re hor one re re son b e choices for tient
therefore s fe for use w ith ithiu . Given his w ith partial res onse, bu t re not s ro-
bi o r isor er, the tients oo st bi izer ri te w hen tient h s no res onse to
shou not be iscontinue , n the other tre t ent.
choices re not ro ri te in this c se.
21. (E) The ost ike y syn ro e is TD, ove-
17. (C) Difficu ty f ing s ee n ecre se ent isor er th t y occu r fter ong-ter
etite re the tw o ost co on si e tre t ent w ith nti sychotic e ic tions
effects of ethy heni te. Sti u nts h ve (es eci y first gener tion) su ch s h o eri-
been re orte to s ow grow th; how ever, it is o . In tients on first-gener tion nti sy-
be ieve to be ess co on w ith ethy he- chotics, TD occu rs t n ver ge r te of bou t
ni te n ose re te . In ition, w hen 5% er ye r, w ith bou t 20% eve o ing TD
rug ho i ys re given (e.g., su erti e), w ithin 5 ye rs, n 40% eve o ing it w ithin
grow th reboun is seen. Methy heni te 20 ye rs. TD consists of nu ber of bnor-
y so c u se n incre se in systo ic b oo n invo unt ry ove ents su ch s i
ressu re. s cking, f ci gri cing, n choreo the-
toi ove ents of the i bs n trunk. Anti-
18. (E) Sti u nts h ve been frequ ent y re orte cho inergic toxicity, w hich u su y resents s
to ex cerb te the tics ssoci te w ith Tou rette e iriu , is not resent. In H u ntington is-
isor er, n so e stu ies h ve w rne th t e se, n u toso o in nt genetic isor er,
they shou not be given to chi ren w ith tics the invo u nt ry ove ents re cco nie
or f i y history of Tourette. H ow ever, this is by rogressive e enti . Meige syn ro e is
co ic te by the f ct th t so e stu ies h ve n or -f ci ystoni invo ving b inking n
i entifie frequ ent co orbi ity of ttention- chin thru sting, i u rsing or tongu e ove-
eficit hy er ctivity isor er (ADHD) n ents, n occ sion y shou er ove ent.
Tourette isor er. The other sy to s re not Sy enh chore , ssoci te w ith rheu tic
incre se by or ssoci te w ith sti u nt use. fever, occu rs in chi ren.

19. (E) This tient is su ffering fro Tou rette 22. (B) Whi e t r ive yskinesi y be rogres-
isor er. Ph r co ogic tre t ent is strict y sive, ost tients st bi ize n so e y
sy to tic n not cu r tive. Do ine (D 2) ctu y i rove, even if they continue t k-
rece tor nt gonis w ith nti sychotic e i- ing the nti sychotic. Whi e nti sychotics
c tion such s ris eri one rovi es the gre t- c use TD, secon -gener tion nti sychotics
est sy to re ief in chi ren su ffering fro re ess ike y th n first-gener tion nti sy-
Tou rette isor er. C oni ine, n h -2- chotics to c u se TD. H ow ever, c oz ine,
gonist, is refer b e in the tre t ent of i secon -/ thir - ine nti sychotic th t requ ires
Tou rettes s it oes not h ve the s e ong- regu r w hite b oo ce cou nt onitoring, is
ter si e effects s nti sychotics. The other the e st ike y to c u se TD n y, in f ct,
e ic tions iste re not u se for Tourettes. i rove sy to s.
Answe rs : 1627 107

23. (E) This tient ike y h s gener ize nxi- of PPH N in w o en ex ose to SSRIs te in
ety isor er (GAD). Benzo i ze ines re the regn ncy. Ebstein no y, for tion
ost effective e ic tions for qu ick y re u c- of the tricu s i v ve, is seen w ith ithiu
ing sy to s of GAD. A benzo i ze ines ex osu re. Gest tion i betes is not ssoci-
re effective n the choice shou be b se te w ith nti e ress nt u se. Inf nt eve -
on otency, h f- ife, n si e effects. Bu ro- o ent e y c n be seen w ith ex osu re
ion is n nti e ress nt w ith o iner- to v roic ci . N eu r tube efects re seen
gic n nor renergic ro erties, bu t is not w ith ex osu re to oo st bi izers su ch s c r-
u sefu in the tre t ent of GAD. Bu s irone b ze ine n v roic ci .
is effective for re u cing sy to s of GAD
bu t requ ires sever w eeks for signific nt 26. (C) Un ess there is co orbi sychi tric
i rove ent. The tricyc ic gent c o i - isor er su ch s e ression or obsessive-
r ine n SSRIs su ch s f u oxetine h ve co u sive isor er, h r cother y gen-
nxio ytic ro erties bu t so o not w ork s er y h s i ite ro e in the tre t ent of
qu ick y s benzo i ze ines. norexi nervos . Re orts on the usefu ness
of cy rohe t ine to sti u te etite h ve
24. (B) This tient ike y h s the i gnosis of bor- been ixe . Gener y, these tients re y
er ine erson ity isor er (BPD) with co or- h ve goo etite, bu t w ork h r not to
bi jor e ressive isor er. Bor er ine give in to their ow n hu nger. F u oxetine, n
erson ity isor er is ri ri y tre te by sy- SSRI, n to ir te, n nticonvu s nt, h ve
chother y, rticu r y i ectic beh vior both show n to be effective in the tre t ent
ther y. However, e ic tion c n rovi e use- of bu i i nervos , bu t not in the tre t ent
fu junctive tre t ent. Given her co orbi of norexi . The secon -gener tion nti sy-
e ressive sy to s, f uoxetine wou be the chotic o nz ine c n in u ce w eight g in, but
ost ro ri te tre t ent. However, recent bec u se of this, ost tients w ith norexi
et - n ysis of h r cother y of BPD w i not gree to t ke it.
showe th t oo st bi izers (e.g., v roic
ci , otrigine, to ir te) n secon -gen- 27. (A) This tient ike y h s SSRI iscontinu -
er tion nti sychotics (e.g., ri i r zo e, o n- tion syn ro e, w hich y occu r w ithin 1 to
z ine, zi r si one) y be the ost effective 3 ys of bru t y sto ing n SSRI. The ost
e ic tions for this con ition (es eci y ffec- co on hysic sy to s re izziness,
tive regu tion n i u sivity) in the bsence n use n vo iting, f tigu e, eth rgy, n
of co orbi e ression. f u - ike sy to s. Psycho ogic sy to s
of nxiety, irrit bi ity, n crying s e s re
25. (E) Decisions bou t w hether to continu e nti- so not u nu su . P roxetine y be ore
e ress nts u ring regn ncy re high y ike y to c u se this bec u se of its short h f-
in ivi u ize , b se on w eighing risks n ife n ck of n ctive et bo ite. F u ox-
benefits for e ch in ivi u w o n. In one etine h s ong h f- ife (24 ys) s w e s
stu y of regn nt w o en w ith histories of n ctive et bo ite w ith very ong h f- ife
jor e ression, 68% of those w ho iscon- (715 ys), n therefore, oes not require
tinu e e ic tion re se , versu s 26% of t ering. The iscontinu tion syn ro e is
those w ho continu e e ic tion. One of the not know n to be eth ; how ever, rest rting
ost seriou s otenti risks of SSRI ex osure the roxetine n then t ering it is re -
u ring regn ncy is ersistent u on ry son b e choice. Acet ino hen is un ike y
hy ertension of the new born (PPH N ), r re to signific nt y he . Bu ro ion is n nti-
con ition (12 er 1,000 ive births) in w hich e ress nt w ithou t sexu si e effects, n
new borns eve o res ir tory f i ure u e to w ou be re son b e choice for the tre t-
ostn t ersistence of e ev te u on ry ent of e ression, thou gh w ou not he
v scu r resist nce. Whi e it is not c e r, there w ith the iscontinu tion sy to s. De k-
is be ieve to be n incre se risk (23 ti es) ote is tre t ent for bi o r isor er, w hich
108 3: S oma tic Tre a tme nt a nd P syc hop ha rma c ology

this tient oes not h ve. Ven f xine is e ression or inversion, bu t the ch nges re
serotonin-nore ine hrine reu t ke inhibitor u su y not c inic y signific nt. First- egree
w ith si i r w ith r w si e effects. AV b ock n sinu s no e ysfu nction re r re.
Lithiu toxicity c n r re y c u se sino tri
28. (A) Concu rrent u se of otrigine n or b ock, AV b ock, AV issoci tion, br y r-
contr ce tives c n ffect eve s of both e i- rhyth i s, ventricu r t chyc r i , n ven-
c tions. The c e r nce of otrigine is sig- tricu r fibri tion, bu t these re usu y u e
nific nt y gre ter in w o en ex ose to or to u n er ying c r i c ise se.
contr ce tives; ow er eve s of the e ic -
tion w ou ccou nt for the tients ck of 32. (C) Thi zi e iu retics su ch s hy roch oro-
i rove ent. A ition y, otrigine y thi zi e c n incre se ithiu eve s ow ing to
in u ce the et bo is of or contr ce tives, ecre se ithiu c e r nce. Other e ic -
otenti y ren ering the ess effective. Ibu - tions th t c n incre se ithiu eve s re the
rofen, ithiu , n zi r si one o not ffect iu retics eth crynic ci , s irono ctone,
otrigine et bo is . H ow ever, v roic n tri terene; N SAIDs (exce t s irin n
ci decreases the c e r nce of otrigine, su in c); n the ntibiotics etroni zo e
necessit ting ow er otrigine oses w ith n tetr cyc ine. Nitrog ycerin n ro r no o
concurrent u se. o not interfere w ith ithiu c e r nce.

29. (E) Tr zo one is n nti e ress nt e ic tion 33. (A) C o i r ine, ixe serotonin n nor-
w hich ffects the serotonin syste by w e k e ine hrine reu t ke inhibitor (bu t c ssifie
reu t ke inhibition n nt gonist ctiv- s tricyc ic nti e ress nt) w s rove
ity t 5-H T1 , 5-H T1c, n 5-H T2 rece tors. It for the tre t ent of obsessive-co u sive is-
h s se tive effect, ike y ro uce by both or er (OCD) in the Unite St tes in 1989. Tri-
h - renergic n hist ine b ock e. It s co ring SSRIs to c o i r ine re ort
is extre e y se ting t nti e ress nt oses equ effectiveness; how ever, the si e effect
(300500 g), bu t is often u se t s er rofi e f vors the SSRIs, w hich re the ost
oses (50200 g) s n ju nct to other nti- co on first- ine tre t ent in c inic r c-
e ress nts to he w ith s ee . Both i ze tice. Other tricyc ic gents (su ch s nortri -
( benzo i ze ine) n i henhy r ine ( n ty ine) re not s effective s c o i r ine
ntihist ine) re u se s se tives, bu t nei- bec u se they re not s serotonergic. C on ze-
ther is n nti e ress nt, n both c n c u se y be he fu for the nxiety ssoci te
e iriu in the e er y. Both f uoxetine n w ith OCD, bu t it is not effective in tre ting the
bu ro ion re ore ctiv ting nti e res- u n er ying isor er (i.e., obsessions or co -
s nts n y initi y w orsen inso ni . u sions). O nz ine is secon -gener tion
nti sychotic w hich y be u se for u g en-
30. (E) Pri is , infu , ro onge erection, is t tion in severe c ses of OCD, bu t it is not rec-
r re si e effect of tr zo one. The nuf c- o en e s onother y. Phene zine is
tu rer h s re orte the inci ence of ny bnor- ono ine oxi se inhibitor (MAOI), use
erecti e fu nction to be bout 1 in 6,000 very r re y for e ression, bu t not u se in
en. This si e effect u su y occu rs w ithin OCD.
the first onth of tre t ent but c n occu r
t ny ti e. Any bnor erecti e fu nction 34. (C) Just ike in the tre t ent of refr ctory
shou ro ti e i te iscontinu tion of e ression, tre t ent of refr ctory OCD is
the e ic tion. The other sy to s re not rge y b se on c se re orts n c inic
co on or ngerous enou gh to w rr nt references. There re very few contro e
infor ing the tient. b in e tri s in refr ctory OCD. Pr ctice
gu i e ines reco en u g enting SSRIs
31. (E) Lithiu c uses ECG ch nges in bou t 20% w ith secon -gener tion nti sychotic (su ch
to 30% of tients, ost co on y T-w ve s ris eri one) for tients w ith o er te
Answe rs : 2839 109

res onse to the SSRI. For tients w ith no inc u ing usc rinic, h -1- n h -2-
res onse, one shou consi er sw itching to renergic, o inergic, n hist inergic,
ifferent SSRI or to c o i r ine. In c ses, to v rying egrees. As c ss, their b ock e
sychother yin rticu r, ex osure ther- of h -1-rece tors is gener y be ieve to be
y w ith res onse reventionshou be rec- res onsib e for ny orthost tic hy otension th t
o en e . Bus irone n ithiu y h ve y occur, es eci y in the e er y. I i r -
so e u ti ity s u g enting gents, bu t their ine is terti ry ine which b ocks u ti e
evi ence b se is ess th n for nti sychot- rece tors n therefore h s ny si e effects;
ics. Anti sychotics re not gener y u se s nortri ty ine, secon ry ine, h s re tive y
onother y in OCD. Cingu oto y, surgi- ess h -1-b ocking otency n ten s to be
c roce u re, shou be reserve for severe, better to er te in gener . Bu ro ion, f uox-
refr ctory c ses. etine, n irt z ine re not ssoci te with
orthost tic hy otension.
35. (B) This tient is su ffering fro co n
u itory h u cin tions, w hich h ve not su f- 38. (E) This tient is resenting w ith bi o r is-
ficient y res on e to equ te tri s of or er, current y nic. First- ine tre t ent for
one first-gener tion nti sychotic e ic - ni inc u es v roic ci or ithiu , w ith
tion (h o eri o ) or tw o secon -gener tion or w ithou t concu rrent nti sychotic gent.
nti sychotics (qu eti ine n o nz ine). Anti e ress nts su ch s cit o r shou
Therefore, tri of c oz ine is w rr nte . not be u se in the setting of cu te ni , s
C oz ine w s the first secon -gener tion they y w orsen the con ition. L otrigine
nti sychotic n h s been fou n to be su e- is n nticonvu s nt, u se for bi o r e res-
rior to other nti sychotics in re u cing both sion n inten nce; it is not s n effective
ositive n neg tive sy to s. It y so nti nic gent. O nz ine is re son b e
i rove cognitive eficits in tients w ith tern tive to v roic ci , bu t ue to the
schizo hreni . It is not u se s first- ine tre t- risk of w eight g in, y not be the o ti
ent bec u se of 1% risk of gr nu ocytosis. choice for this tient w ith n e ting isor-
Ari i r zo e, ris eri one, n zi r si one er. To ir te is n nticonvu s nt use in
re secon -gener tion nti sychotics th t the tre t ent of bu i i nervos ; how ever it
re not s effective s c oz ine for tre t ent- is not consi ere effective for the tre t ent of
refr ctory schizo hreni . Per hen zine is bi o r isor er.
first-gener tion nti sychotic ike h o eri o .
39. (D) A of the co on y use oo -
36. (C) C oz ine cts t ny rece tors, inc u - st bi izing e ic tions c rry so e risk of fet
ing D 1, D 2, n D 4; hist ine-1; u sc rinic; for tions or otenti e eterious effect
h 1- renergic; n serotonin ty es 5-H T2, on ter cognitive eve o ent. Use of ithiu
5-H T2c, n 5-H T3. Efforts to i entify the ex ct uring the first tri ester incre ses the risk of
ech nis of the nti sychotic ction of c o- Ebstein no y, for tion of the tricus-
z ine h ve reve e t e st tw o ossibi i- i v ve, fro 1:20,000 to 1:1,000 tients. If
ties. Un ike first-gener tion nti sychotics, fetus h s been ex ose to ithiu , fet echo-
c oz ine h s uch ore otent nt gonis c r iogr shou be one between weeks 16
t the D 4 rece tor co re to the D 2 rece - n 18 of regn ncy. Lithiu y be use in
tor, es eci y in the i bic syste . This h s the secon n thir tri esters with c refu
e to s ecu tion th t the D 4 rece tor y ttention to equ te hy r tion. C rb ze-
e i te sychotic sy to s. C oz ine so ine c n c use cr niof ci efects (11%), finger-
h s ctivity t the 5-H T2 rece tor, ctivity th t n i hy o si (26%), neur tube efects, n
first-gener tion nti sychotics ck. eve o ent e y (20%). Div roex so iu
is ssoci te with neur tube efect r tes of
37. (C) Tricyc ic nti e ress nts, inc u ing i i r - roxi te y 2%, n y c use intr uterine
ine n nortri ty ine, b ock v rious rece tors, growth ret r tion. Benzo i ze ines such s
110 3: S oma tic Tre a tme nt a nd P sychop ha rma c ology

i ze y be ssoci te with c eft i n 42. (C) Tricyc ic nti e ress nts (TCAs) ro u ce
te, though this re ins controversi . sever c r iov scu r si e effects, the ost
signific nt being qu ini ine- ike effect s ow -
40. (E) It is not u nu su for e iriou s tients to ing c r i c con uction. So e c inici ns voi
beco e hosti e or co b tive, osing risk TCAs if there re ny ECG ch nges. Cert in y,
to the se ves or to hos it st ff. Low - ose if there re ch nges in con uction, such s
ty ic n ti sych otics, su ch s ris eri one, ro onge QT interv , w i ening of the QRS
re very effective in re u cin g git tion in co ex, or AV con uction bnor ities,
e iriou s tients. Di henhy r ine shou TCAs shou be voi e . In over ose, they c n
be voi e s the nticho inergic effects y w i en the QRS co ex, c use bun e br nch
w orsen the e iriu n confu sion. Done e- b ock, n c use t chy rrhyth i s; how ever,
zi is n n ticho inester se inhibitor u se for even t ther eu tic concentr tions, they y
jor neu rocognitive isor ers ( e enti s). h ve verse effects on c r i c con uction.
A thou gh benzo i ze ines ike or ze Estrogen re ce ent ther y n o i ine
c n be u se for git tion in e iriu , th ey re not contr in ic tions. TCAs c n be u se
shou be voi e in e er y tien ts u e to in tients w ith rior cerebr inf rctions.
the risk of w orsening e iriu n oten ti Whi e TCAs y ex cerb te urin ry retention
isinhibition. Orient tion to su rrou n ings is fro their nticho inergic effects, this is not
often ition y he fu in e iriu , bu t it contr in ic tion to their use.
w i not i e i te y c the tient.
43. (B) This tient is ike y e iriou s n ro t
41. (C) Mirt z ine is n nti e ress nt w hose i entific tion n tre t ent of the u n er ying
h r co ogic rofi e is ifferent fro other c use is in ic te . To he contro the git -
v i b e gents. It is centr h -2- ren- tion th t y cco ny e iriu , ow - ose
ergic nt gonist n n nt gonist of 5-H T2 h o eri o is frequ ent y u se . H o eri o
n 5-H T3 rece tors, s w e s H 1 rece tors. oes not tre t the e iriu , how ever. It is u se
B ock e of h -2-rece tor e s to enh nce ost frequent y bec u se it is the ost otent
serotonin re e se; how ever b ock e of 5-H T2 of the ty ic nti sychotics, therefore requ ir-
n 5-H T3 e s to re tive enh nce ent of ing ow er oses, w ith few er nticho inergic or
5-H T1 ctivity giving irt z ine ifferent orthost tic si e effects. It so h s the v n-
si e effect rofi e th n SSRIs. For ex e, t ge of being v i b e in or , IM, or IV for .
irt z ine ten s to incre se etite n Low - otency gents (su ch s ch or ro zine
c use w eight g in co re to cebo. In or thiori zine) re not on y ssoci te w ith
ition, its H 1 nt gonis c u ses se tion. orthost tic hy otension n nticho inergic
H ow ever, t higher oses, h -2- rener- si e effects, bu t so w ith ro ong tion of the
gic b ock e so e s to incre se nore i- QT interv . Lor ze y he se te the
ne hrine re e se, w hich y cou nter ct the tient bu t it w i not he his sychosis n
H 1- e i te se tion. In this tient, u se of cou c use isinhibition n w orsening of
irt z ine (to ex oit its otenti incre se his e iriu . Ris eri one is not v i b e in
etite n se tion si e effects) shou be renter for , n is therefore not s usefu
consi ere . Bu ro ion is n nti e ress nt in n git te tient w ho requ ires IV or IM
w hose si e effect rofi e inc u es inso - e ic tion.
ni n w eight oss, neither of w hich w ou
be benefici in this tient. F u oxetine n 44. (A) Of the first-gener tion nti sychotics
sertr ine re SSRI nti e ress nts w hich, iste (h o eri o , er hen zine, n thio-
w hi e effective, y initi y c u se ctiv tion. ri zine), h o eri o is the ost otent n
N ortri ty ine is tricyc ic nti e ress nt th t h s the e st ctivity t h -1 rece tors.
is eth in over ose n w hose nticho in- Therefore, it is the e st ike y to c u se ortho-
ergic ro erties c n e to e iriu in the st tic hy otension. The secon -gener tion
e er y. gents iste (ris eri one n queti ine)
Answe rs : 4050 111

h ve ctivity t h -1 rece tors n re both re u sefu in re u cing the core sy to s of


ssoci te w ith orthost tic hy otension. PTSD (reex eriencing, voi nce, nu bing,
n hy er rou s ). Benzo i ze ines such
45. (B) The et bo is of ethy coho invo ves s r zo re not benefici in tre ting
tw o-ste enzy tic rocess. The first these sy to s. Anticonvu s nts re not
enzy e, coho ehy rogen se, et bo izes rou tine y u se ; o en- be stu ies of iv -
eth no to cet ehy e, w hich is qu ick y roex, c rb ze ine, n to ir te h ve
et bo ize by ehy e ehy rogen se. show e ixe resu ts. N trexone, n o ioi
Disu fir inhibits ehy e ehy rogen se, nt gonist, h s not been extensive y stu ie
resu ting in n ccu u tion of cet ehy e. in PTSD. Anti sychotics su ch s thiori zine
Acet ehy e c u ses f ci f u shing, t chy- h ve gener y not been u sefu , though the
c r i , hy otension, n u se n vo iting, secon -gener tion nti sychotics y be u se-
n hysic isco fort. Therefore, tient fu s ju nctive e ic tions, es eci y if the
on isu fir h s n incentive to re in tient h s co orbi sychotic sy to s.
bstinent. Obviou s y, isu fir w orks on y
if tients continu e to t ke it. Ac ros te, 48. (E) Pr zosin is n h -1- renergic rece tor
GABA-ergic gonist, n n trexone, n b ocker, tr ition y use to tre t hy ertension
o ioi nt gonist, re both u se to re u ce n benign rost tic hy ertro hy, w hich h s
cr vings for coho . F u zeni is benzo- e onstr te effic cy in tre ting night res
i ze ine nt gonist u se in benzo i ze ine n other sy to s in tients w ith PTSD.
over ose. N oxone is n o ioi nt gonist Se tive-hy notic gents, such s or ze ,
u se in e ergency n ge ent of o i te re not rticu r y he fu in tre ting PTSD
intoxic tion n over ose. n h ve rger otenti for hysio ogic
n sycho ogic e en ence. N trexone h s
46. (E) Mu ti e neu rotr ns itter syste s h ve not been extensive y stu ie in PTSD. O n-
been investig te in tte ts to contro co- z ine, secon -gener tion nti sychotic, n
ho cr vings. One syste th t ike y ys er hen zine, first-gener tion nti sychotic,
ro e in the rew r thw y is the o ioi sys- y be usefu in severe PTSD w ith sychotic
te . N trexone is n o ioi nt gonist th t sy to s, but they c rry risk of w eight g in
h s been show n to ecre se the nu ber of n et bo ic si e effects, s w e s t r ive
ys erson w ith coho use isor er rinks yskinesi over the ong ter .
n to incre se the ti e before re se of
he vy rinking. Tre t ent of e ression w ith 49. (E) This tient is ike y ex eriencing e ir-
n gent su ch s bu ro ion y he contro iu , s evi ence by his cu te ch nge in
rinking if co orbi e ression exists, bu t ent st tu s, w ith confu sion, isorient tion,
u se of nti e ress nts in the bsence of oo n git tion. E er y tients c n be rticu -
isor er h s not been effective in re u cing r y sensitive to the verse effects of ben-
cr vings. Disu fir is n coho -sensitizing zo i ze ines. This tient is t king 8 g/
gent w hich eters tients fro rinking. of or ze , w hich is rge ose. S ow y
F u zeni is benzo i ze ine nt gonist t ering the or ze , w hi e investig ting
u se in benzo i ze ine over ose. N oxone other otenti c u ses of e iriu , is re son-
is n o i te nt gonist th t is u se to cu te y b e first ste . H y roch orothi zi e eve s re
reverse the effects of o i te intoxic tion. not e sure , n the other choices w i not
ress or he to n ge his e iriu or the
47. (C) This tient is ike y su ffering fro ost- ike y u n er ying c u se.
tr u tic stress isor er (PTSD). First- ine
h r co ogic tre t ent for PTSD is the 50. (B) This tient is ike y su ffering fro jor
se ective serotonin reu t ke inhibitors (e.g., neu rocognitive isor er ( e enti ), ue to
f uoxetine, roxetine, sertr ine), w hich sev- A zhei er ise se. H e h s c e r cognitive
er r n o ize contro e tri s h ve show n ec ine, n , thou gh reversib e c uses of
112 3: S oma tic Tre a tme nt a nd P syc hop ha rma c ology

cognitive i ir ent shou be investig te , requ ire . Sever tre t ents th t shou be
there is no in ic tion th t ny reversib e c use i e i te y consi ere inc u e IV extrose,
is resent. There re five e ic tions cu r- u su y D 50, to tre t hy og yce i ; thi ine
rent y v i b e to tre t A zhei er ise se: to gu r g inst the eve o ent of Wer-
the nticho inester se inhibitors one ezi , nicke ence h o thy in n coho ic tient
g nt ine, riv stig ine n t crine, n w ith thi ine eficiency; n n oxone, n
the N - ethy -d - s rt te (N MDA) rece tor o ioi nt gonist, to reverse the effects of
nt gonist, e ntine. Of these e ic tions, o ioi intoxic tion. F u zeni is benzo i-
t crine is r re y u se u e to the risk of he tic ze ine nt gonist th t shou not be u se
f i ure. A r zo is short- cting benzo i- before obt ining fu rther history bec u se it
ze ine w hich y w orsen confu sion. There y cu te y ow er the seizu re thresho .
oes not e r to be n in ic tion for f u ox-
etine in this tient (su ch s e ressive 53. (A) Bu renor hine is one of the ost effective
isor er). Tr zo one is serotonergic nti e- e ic tions for o ioi etoxific tion. Given
ress nt often u se to tre t inso ni ; it w i in sub ingu for , it is ixe o ioi go-
not i rove cognitive bi ity. Methy heni- nist nt gonist w hich su resses o ioi w ith-
te is sti u nt th t oes not e r to be r w n b ocks the effects of other o ioi s.
in ic te in this c se. Benzo i ze ines such s ch or i ze oxi e y
be he fu s juncts to contro nxiety, but
51. (C) Tr zo one is serotonergic gent fre- they re not sufficient for o i te w ith r w .
quent y use in tients to he with s ee , n C oni ine, centr h -2- gonist, su -
in this tient y he w ith his w n ering resses the sy thetic res onse to the heroin
beh vior. It c n c u se orthost tic hy oten- w ith r w n he s to contro git tion n
sion n shou therefore be u se w ith c u - utono ic inst bi ity, such s e ev te b oo
tion in those w ith risk f ctors for f s. H o ressure n he rt r te; how ever, it w i not
n qu eti ine re nti sychotic e ic tions ress the heroin cr ving n is ess effective
th t shou both be u se c refu y in the in n ging w ith r w th n bu renor hine.
e er y. H o eri o is high- otency first- N oxone wou not be ro ri te beyon
gener tion nti sychotic th t is so eti es the initi resuscit tion efforts. Pro r no o is
u se to n ge severe git tion, bu t it y not use for heroin etoxific tion.
c u se extr yr i sy to s. Qu eti ine
is secon -gener tion nti sychotic; how - 54. (A) Bu ro ion ( rkete s Zyb n) is n nti-
ever, given the b ck box w rning bou t the e ress nt th t h s so been show n to be
risk of su en e th in e er y tients w ith effective s rt of s oking cess tion ro-
e enti -re te sychosis, nti sychotics gr . The ech nis of ction is u nc e r, bu t
shou be u se s ring y in the geri tric o - it is be ieve to h ve n effect on o iner-
u tion. Benzo i ze ines su ch s or ze gic tr ns ission. Giving u cig rettes y be
n tri zo shou so be u se w ith c u - extre e y ifficu t for tient w ho is so
tion in the e er y. They y ctu y w orsen suffering fro jor e ression; how ever,
the beh vior n c u se e iriu . If necess ry, st rting bu ro ion u ring the e ression
st rting oses shou be u ch ow er th n y he her qu it s oking in the fu tu re. The
st n r osing n s ow y t ere u . other nti e ress nts iste h ve no i ct
on s oking cess tion.
52. (E) P tients w ho resent w ith severe y tere
eve s of consciou sness nee to be e ic y 55. (E) Du oxetine is serotonin-nore ine hrine
ev u te n qu ick y tre te for sever reu t ke inhibitor (SN RI) th t is use in the
reversib e c u ses. These inc u e hy og yce- tre t ent of both jor e ression n in,
i , o ioi over ose, n coho intoxic - inc u ing neuro thic in n fibro y gi .
tion. Airw y rotection n onitoring of A r zo is benzo i ze ine not ro-
ir exch nge n c r iov scu r st tu s re ri te for tre ting either jor e ressive
Answe rs : 5161 113

isor er or in. Whi e bu ro ion is use- erb tions u e to oor co i nce, he is goo
fu nti e ress nt, it so w i not he w ith c n i te for ong- cting e ic tion. Ris -
the tre t ent of in. Sever tricyc ic nti- er Const (Ris eri one) is ong- cting for
e ress nts, inc u ing itri ty ine n nor- of ris eri one given in bi onth y injections.
tri ty ine, re so u se to tre t chronic in. Continu ing his cu rrent regi en or sw itching
H ow ever, tricyc ic nti e ress nts re ore to nother or e ic tion su ch s o nz ine
eth in over ose th n new er e ic tions ike w i ike y on y e to continu tion of the
se ective serotonin reu t ke inhibitors (SSRIs) viciou s cyc e of oor co i nce n re se.
n SN RIs, n they gener y h ve ore si e C oz ine is ro ri te for tients w ho
effects, rticu r y nticho inergic n c r io- h ve f i e tw o or ore nti sychotic e ic -
v scu r ones. SSRIs su ch s cit o r y tions, w hich is not the c se in this tient, n
y ro e in chronic in n ge ent, bu t the regu r b oo onitoring so requ ires
the t re not s robust s w ith u oxetine. co i nt tient. Whi e h o eri o is so
v i b e in e ot for , it is not s goo
56. (B) C rb ze ine is n nticonvu s nt th t choice for this tient, given his rior res onse
is u se to tre t both bi o r ni n tri- to ris eri one.
ge in neu r gi . A itri ty ine n f u ox-
etine re both nti e ress nts, w hich re 59. (D ) Ris eri one is n ty ic or secon -gen-
not ro ri te in this tient w ho is in the er tion nti sychotic gent th t h s otent
i e of nic e iso e. G b entin is n 5-H T2 nt gonist ro erties s w e s b ock-
nticonvu s nt th t h s e onstr te effic cy ing t D 2 n h -1-rece tors. Bec u se of the
in the tre t ent of trige in neu r gi ; how - h -1-b ocking ctivity, it y c use ortho-
ever, it is not consi ere effective s n nti- st tic hy otension. Agr nu ocytosis is con-
nic gent. Lithiu shou not be rest rte cern w ith c oz ine, not ris eri one. A benign
ue to concerns bou t its rior effect on her eu kocytosis c n occu r w ith ithiu u se, bu t
thyroi fu nctioning, n it so oes not tre t not w ith ris eri one. Ris eri one y c use
trige in neur gi . w eight g in r ther th n w eight oss, n it oes
not c u se reci b e nticho inergic effects.
57. (E) As n nti sychotic w ith oo -st bi izing
ro erties, o nz ine is usefu for both sy- 60. (C) Secon -gener tion nti sychotic gents, ike
chotic isor ers, ike schizo hreni , s w e s c oz ine n ris eri one, re consi ere ty i-
bi o r isor er. Weight g in is si e effect of c bec use of their ecre se ro ensity for
ost nti sychotic e ic tions, both first n c using extr yr i si e effects (EPS) n
secon gener tion; how ever, it is rticu r y resu b y re uce (but not zero) risk of t r-
concerning w ith o nz ine. In one stu y of ive yskinesi . Unfortun te y, on y c oz ine
nti sychotic u se in eo e w ith schizo hre- e rs to tru y obey this ru e. The other sec-
ni , over 45% of tients on o nz ine g ine on -gener tion nti sychotic gents e r
ore th n 7% of their bo y w eight; o nz ine to h ve so e egree of EPS, with ris eri one s
ten s to in uce ore w eight g in th n other the e ing cu rit; t oses higher th n 6 g/ ,
nti sychotic gents. Agr nu ocytosis is r re ris eri one c uses EPS t r tes co r b e to
bu t otenti y f t si e effect of c oz ine. h o eri o . Agr nu ocytosis is concern with
O nz ine is not ssoci te w ith the eve - c oz ine, not ris eri one. A benign eukocy-
o ent of c t r cts or incre se excit bi ity. tosis c n occur with ithiu use, but not with
O nz ine is ike y to c use se tion n is ris eri one. Ris eri one oes not c use re-
u su y inistere in the evening. E ev te ci b e nticho inergic effects, n usu y c uses
ro ctin n otenti g ctorrhe re si e weight g in r ther th n weight oss.
effects of first-gener tion nti sychotics n
ris eri one, but ess co on w ith o nz ine. 61. (E) Anti sychotic gents b ock o ine
58. (E) Given th t the tient to er tes n rece tors in sever thw ys in the br in.
res on s to or ris eri one bu t su ffers ex c- B ock e of o ine rece tors in the
114 3: S oma tic Tre a tme nt a nd P sychop ha rma c ology

tu beroinfu n ibu r tr ct resu ts in incre se rhyth s ee w ke isor ers, e ye s ee


ro ctin re e se ( o ine cts s n inhibi- h se ty e. Me tonin is gener y benign
tor of ro ctin re e se), w hich y c use intervention w ithou t risks of bu se, e en-
inferti ity, enorrhe , g ctorrhe , n ence, or res ir tory su ression. A itri -
gyneco sti . Ris eri one c u ses ore ro- ty ine n qu eti ine re se ting but h ve
ctin incre se th n other nti sychotic e i- seriou s si e effects n shou not be u se for
c tionsin f ct, it y r ise ro ctin eve s ri ry inso ni . Di henhy r ine is u se
u to 100-fo . Do ine nt gonis in the for inso ni bu t c n e ve tients fee ing
esocortic n eso i bic tr cts is res onsi- row sy the next y. Short- cting benzo i z-
b e (in rt) for the nti sychotic effect. B ock- e ines su ch s te ze c n be u sefu in the
ing o ine rece tors in the nigrostri t tre t ent of inso ni ; how ever, these e i-
syste c uses the extr yr i sy to s. c tions c rry w ith the risk of e en ence
The ocu s ceru eu s, oc te in the br in ste , is n rebou n inso ni .
the ost i ort nt nor renergic nuc eu s in
the br in. 65. (E) This tient ike y h s nic isor er.
Sertr ine is se ective serotonin reu t ke
62. (B) This tient is su ffering fro bu i i ner- inhibitor (SSRI) rove for the tre t ent of
vos . First- ine h r co ogic tre t ent is nic isor er. A thou gh other e ic tions
n SSRI su ch s f u oxetine, w hich ecre ses re effective, SSRIs re the s fest in over ose
sy to s of bingeing n u rging in e en- n h ve few er si e effects. Bu ro ion, n
ent of ny oo isor er. Bu ro ion is con- nti e ress nt w ith o inergic ech -
tr in ic te in tients w ith e ting isor ers nis , is not effective in the tre t ent of nic
ue to n incre se risk of seizure. L otrig- isor er. I i r ine is tricyc ic nti e res-
ine is n nticonvu s nt u se in the tre t- s nt n hene zine is ono ine oxi se
ent of bi o r e ression n inten nce. inhibitor; w hi e both re effective in tre ting
Lithiu is so u se to tre t bi o r isor er. nic isor er, SSRIs re f r better to er te
Mirt z ine is n nti e ress nt w hich fre- n s fer. Qu eti ine is n nti sychotic e -
qu ent y c u ses w eight g in, n is therefore ic tion th t w ou not be ro ri te for this
oor choice for tient w ith bo y-i ge tient.
issu es.
66. (A) The tricyc ic nti e ress nts, such s
63. (D ) To ir te is n nticonvu s nt u se in itri ty ine, h ve been show n to incre se
the tre t ent of bu i i n binge-e ting is- ort ity in c r i c tients bec u se of their
or er. In sever contro e tri s, to ir te qu ini ine- ike effects n ten ency to both
ecre se the frequency of binge e iso es n incre se he rt r te n ecre se b oo res-
e to w eight oss. Lithiu n v roic ci sure. Bu ro ion is n nti e ress nt th t is
re both use to tre t bi o r isor er n not s fe in c r i c tients. As c ss, het-
ro ri te for bu i i . Bu ro ion is n nti- ines, su ch s ethy heni te, h ve been
e ress nt but is contr in ic te in tients fou n to be s fe in tients w ith c r i c is-
w ith e ting isor ers u e to n incre se risk e se, thou gh re not ro ri te for tre t-
of seizure. P roxetine is n SSRI th t c n be ing jor e ression in this tient. As there
u se in the tre t ent of bu i i ; how ever, y be inor ch nges in b oo ressu re n
this tient h s re y f i e tw o SSRI tri s, u se, tients shou be onitore c ose y.
n is u n ike y to benefit fro thir ; in i- Sertr ine is so s fe for tients w ith c r i c
tion, it ten s to incre se w eight g in w hich ise se n ossesses few ru g rug inter-
w ou ke it oor choice in this tient. ctions. Ven f xine c n incre se the b oo
ressu re n ver ge of 8 to 10 H g bu t
64. (C) Me tonin, hor one invo ve in cir- this effect is ose re te . At oses ess th n
c i n rhyth , h s been show n to be effec- 150 g/ , c inic y signific nt ch nges re
tive in the tre t ent of jet g n circ i n re tive y r re.
Answe rs : 6271 115

67. (D ) The sy to s the tient is ex erienc- (ECT), w hich requires gener nesthesi ,
ing re co on effects of e ev te ro ctin rTMS c n be erfor e in the office, w ithou t
eve s. H o eri o s ech nis of ction is ny nesthesi . The ost co on si e effect
o ine rece tor b ock e (D 2). Do ine is he che t the site of the ic tion. The
nor y inhibits ro ctin secretion in the tre t ent is very ou n tients re ske
tuberoinfun ibu r thw y, but bec use h - to w e r e r u gs; how ever, he ring oss h s
o eri o b ocks o ine, ro ctin secretion not been re orte . A so, u n ike w ith ECT,
is u no ose . H o eri o oes not signifi- there is no e ory oss ssoci te w ith the
c nt y incre se ACTH , o ine, nore ine h- roce u re. Seizu res n tooth in re r re
rine, or serotonin. co ic tions.

68. (B) This tient ost ike y h s k thisi , w hich 70. (D ) Decisions bout e ic tions u ring reg-
is n nti sychotic(neu ro e tic)-in uce si e n ncy re high y in ivi u ize , b se on
effect. P tients gener y ex erience su bjective w eighing risks n benefits for rticu r
fee ings of rest essness n c n be seen sw ing- in ivi u . ECT h s been u se in regn ncy
ing their egs, rocking b ck n forth w hi e sit- for ore th n 50 ye rs n its s fety n effi-
ting, cing, n r i y tern ting betw een c cy is w e ocu ente . V roic ci n
sitting n st n ing. Acute ystoni s re ch r- ithiu (in ition to secon -gener tion nti-
cterize by infu contr ction of usc es sychotics) re in ic te s first- ine gents
resu ting in bnor ove ents or ostu res, for the tre t ent of nic e iso es, w ith c r-
such s s s s of the j w , bnor osi- b ze ine s goo tern tive; how ever,
tioning of the he , or ifficu ty sw ow ing. three re ssoci te w ith n incre se
The onset of cu te ystoni u su y eve o s risk of congenit bnor ities w hen t ken
w ithin the first w eek of initi ting or incre sing uring the first tri ester. Benzo i ze ine
neuro e tic e ic tion. Ak thisi c n so e- such s i ze re not first- ine tre t ent
ti es be ist ken for nxiety or ni , but in for ni , n y be ssoci te w ith n
this c se there re no other signs (such s e u- incre se risk of or c efts w ith first-tri ester
sions of gr n eur, ecre se nee for s ee , ex osu re.
incre se energy, or hy ersexu ity) to in i-
c te nic e iso e. N euro e tic ign nt 71. (D ) Cess tion or re u ction in se tive, hy -
syn ro e is ife-thre tening co ic tion notic, or nxio ytic e ic tions th t h ve
invo ving u sc e rigi ity n ystoni , s w e been u se he vi y or for ro onge erio
s u tono ic sy to s su ch s e ev te te - of ti e y resu t in w ith r w syn ro e
er ture, incre se he rt r te, n incre se ch r cterize by sy to s th t eve o
b oo ressu re. T r ive yskinesi is ong- w ithin hou rs to few ys fter cess tion or
ter consequence of nti sychotic use, n re u ction. Au tono ic hy er ctivity, ortho-
it is ch r cterize by invo u nt ry, choreo th- st tic hy otension, usc e w e kness, tre or,
etoi ove ents of the bo y, es eci y in the inso ni , n u se , vo iting, u itory/ visu /
extre ities. It is not infu , thou gh y be t cti e h ucin tions, git tion, or nxiety y
so ew h t is b ing, n is often irreversib e. occur. By f r, the ost serious seque e re
gr n seizu res or e iriu . De en ing on
69. (A) Tr nscr ni gnetic sti u tion is new the otency, ou nt, frequ ency, n ength
tre t ent for e ression, rove in 2008 of u se, u to 75% of tients c n ex erience
by the FDA for tre t ent of jor e res- gr n seizu res on the secon or thir y
sion th t h s f i e t e st one nti e ress nt of w ith r w , n ny h ve ore th n one
tri . The tre t ent invo ves ying series seizu re. De iriu y eve o betw een the
of e ectro gnetic u ses to the eft orbit thir n eighth y of w ith r w . Minor
refront cortex; fu tre t ent consists of sy to s y ersist for u to 4 w eeks.
t e st 20 i y sessions, e ch sting bou t With r w fro other e ic tions iste is
40 inu tes. Un ike e ectroconvu sive ther y u n e s nt, bu t is not ife thre tening.
116 3: S oma tic Tre a tme nt a nd P sychop ha rma c ology

72. (B) Chronic coho ics often t ke in c ories bi o r isor er, on y v roic ci c rries
fro itt e e se besi es coho n re thu s w rning of n incre se risk of ncre titis.
t risk for thi ine eficiency. If thi ine- A thou gh c rb ze ine n c oz ine c n
eficient tient is given foo (g u cose), he both c u se gr nu ocytosis, c oz ine is ore
or she c n eve o Wernicke ence h o - ike y to be u se ( n is FDA rove ) for
thy ( e iriu ) fro the bo ys tte ts to the tre t ent of refr ctory schizo hreni .
et bo ize g u cose in the bsence of thi ine P roxetine is the on y nti e ress nt iste
yro hos h te. A though the other choices n h s very short h f- ife su ch th t it ro-
re i ort nt concerns, none re of the s e u ces signific nt w ith r w syn ro e fter
cu ity s thi ine eficiency. ju st 2 ys. Whi e c rb ze ine, iv roex
so iu , n ithiu ( s w e s nti sychot-
73. (D ) This n wi rob b y su ffer fro co- ics) y be u se to tre t bi o r isor er,
ho w ith r w . Benzo i ze ines n coho on y ithiu c uses both o yu ri ( benign
h ve si i r o es of ction in their o u - si e effect) n ne hrogenic i betes insi i-
tion of GABA rece tors in the br in. This u s, resu ting in frequ ent u rin tion. Whi e
si i rity kes benzo i ze ine su ch s nti sychotics h ve the risk of incre sing cho-
or ze sensib e n co on choice for estero , of those iste , c oz ine h s the ost
the tre t ent of coho w ith r w . Ac - signific nt effect.
ros te, isu fir , n n trexone re use
for re se revention in tients w ith coho 82. (I) H o eri o , c oz ine, n ris eri one
u se isor er, bu t not for n ging coho re u sefu in the tre t ent of schizo hre-
w ith r w . C oni ine, n h -2- gonist ni , bu t on y h o eri o n ris eri one re
th t o u tes utono ic inst bi ity, is u se v i b e in e ot injection. Ris eri one
to n ge sy to s of o i te w ith r w , w ou be refer b e bec u se it is secon -
n ctu y y b ock ( sk) the u tono ic gener tion nti sychotic w ith better si e
w rning signs of coho w ith r w . effect rofi e.

7481. [74 (H), 75 (B), 76 (F), 77 (C), 78 (B), 79 (F), 80 83. (K) Tr zo one, serotonergic nti e res-
(E), 81 (B)] Psychi tric ru gs c n h ve seriou s s nt, is n effective s ee ing i th t is not
si e effects, so e of w hich re the subjects of h bit for ing. Whi e benzo i ze ines such s
b ck box w rnings on rescribing infor- r zo y he w ith inso ni , it is w ise
tion ckets. Pi ozi e, first-gener tion to voi rescribing the in tient w ith
nti sychotic, inter cts w ith nu ber of history of coho use isor er bec u se of their
e ic tions, inc u ing cit o r ; the co bi- cross-re ctivity w ith coho .
n tion of these tw o e ic tions c n ro ong
the QTc. A thou gh c oz ine, h o eri o , 84. (D ) The reco en e ength of tre t ent
er hen zine, i ozi e, n zi r si one re of first e iso e of u ni o r e ression is t
nti sychotics u se to tre t schizo hreni , e st 6 onths n u su y on the sc e of 9 to
c oz ine is the ost ike y to c u se hy er- 12 onths, n ossib y onger, e en ing on
g yce i n e to the eve o ent of i - tient f ctors such s f i y history of oo
betes. After controversi set of he rings on isor er, severity n u r tion of the e res-
the subject of chi ren n nti e ress nts sive e iso e, n co orbi sychi tric sy -
in the Unite King o n Unite St tes in to s such s nxiety n subst nce buse.
2003 n 2004, regu tors eci e there w s Discontinu tion w ithin the first 16 w eeks of
enou gh evi ence to show th t so e of the tre t ent is ssoci te w ith high risk of
SSRIs, inc u ing roxetine, y incre se re se.
the risk of su ici thoughts in chi ren. This
w rning is now exten e to you ng u ts, u 85. (A) Bu ro ion is the one nti e ress nt iste
to ge 24. A thou gh c rb ze ine, iv - th t oes not c use sexu ysfu nction, n
roex so iu , n ithiu re use to tre t is so eti es e to SSRIs in or er to he
Answe rs : 7291 117

w ith sexu ysfu nction. Cit o r , f u ox- in conju nction w ith ithiu or iv roex
etine, n f u vox ine re SSRIs th t h ve so iu initi y; how ever, u e to their si e
been re orte to c u se v rying egrees of effects n risk of t r ive yskinesi , they re
sexu ysfunction. C o i r ine is tricy- not rou tine y u se s inten nce tre t ent.
c ic nti e ress nt th t h s gre ter inci ence L otrigine is n nticonvu s nt th t is u se
of sexu ysfunction th n other TCAs, ike y for the tre t ent of bi o r e ression n
ue to its otent serotonin reu t ke inhibiting inten nce tre t ent; it is not u sefu u r-
ro erties. ing n cu te nic e iso e. Lor ze c n
be u se initi y for se tion n s ee , bu t
86. (C) Si orrhe is co on si e effect of it is not effective for tre ting or int ining
tre t ent w ith c oz ine (u to 30% of tients w ith bi o r isor er.
tients). It c n be extre e y botherso e to
the tien t n oth ers bu t oes n ot u su y 89. (E) The neu roveget tive sy to s (ch nges
requ ire iscontin u tion of tre t en t. Often , in etite, oor concentr tion, nergi , n
beh vior e su res re su fficient, su ch s s ee isturb nces) ten to res on first w ith
the u se of ozenges or cin g tow e on nti e ress nt tre t ent, often rior to sig-
the tien ts i ow t n ight. Anticho in er- nific nt oo ch nges or re u ction in su i-
gic gents su ch s tro ine ro s often he ci ity. This is w hy it is co on w is o in
to re u ce the vo u e of s iv . A n t in e sychi try th t tients w ho begin to recover
y be h e fu in th e tre t ent of rkin - fro e ression y be t gre ter risk for
soni n sy to s n ro r n o o is u sefu suici e th n w hen they re t the height of
in the tre t ent of k thisi . Incre sing the e ression. As they st rt to function better
ose w ou not n ro ri te ro ch in hysio ogic y (w ith i rove s ee , energy,
this situ tion . n otiv tion), they y be ore c b e of
c rrying out suici i u se.
87. (D ) P tients on c oz ine often eve o er-
sistent sinus t chyc r i th t oes not requ ire 90. (E) In c ses of su s ecte iver i ir ent, it is
cess tion of tre t ent. Often, the t chyc r i vis b e to u se benzo i ze ine ini y
reso ves w ithou t necessit ting fu rther inter- et bo ize by the iver (e.g., or ze n
vention; how ever, shou it ersist, it y be ox ze ). These benzo i ze ines o not go
tre te w ith the bet - nt gonist ro r no- through oxi tion, bu t r ther on y throu gh
o . Benztro ine is n nticho inergic gent g u cu roni tion. The other benzo i ze ines
th t w i not he the t chyc r i , n y, iste re et bo ize ost y by the iver
in f ct, w orsen it. C oz ine h s signific nt n cou qu ick y bu i to toxic eve s in
h -rece tor b ock e, w hich often c u ses cirrhotic tient.
orthost tic hy otension, requ iring gr u
titr tion of the ose; bec use of this, bet o 91. (A) The tient h s ike y eve o e t r ive
w ou not be ro ri te s it y ex cerb te yskinesi (TD). A of the secon -gener tion
this si e effect. Lor ze is benzo i z- nti sychotics h ve ow er r tes of TD co -
e ine th t yc the tient, bu t w i not re to the first-gener tion nti sychotics.
irect y ffect he rt r te. Sto ing the c oz - A thou gh they h ve w rnings reg r ing
ine w ou not be necess ry or ro ri te in the incre se risk of i betes n hy er i -
this c se. i e i ( et bo ic syn ro e), they c u se it t
ifferent r tes, so this is often f ctor in choos-
88. (C) This tient resents w ith signs n ing rticu r nti sychotic e ic tion. The
sy to s su ggestive of first nic e iso e. e st ike y to c u se et bo ic syn ro e re
Lithiu n iv roex so iu re in the ri i r zo e n zi r si one, fo ow e by
inst y of initi n inten nce tre t ent ris eri one n queti ine. C oz ine n o n-
of bi o r I isor er. Anti sychotics, such z ine h ve the highest r tes of weight g in,
s h o eri o n o nz ine y be u se hy er i i e i , n insu in resist nce.
118 3: S oma tic Tre a tme nt a nd P sychop ha rma c ology

92. (C) N trexone, u-o ioi rece tor b ocker, rob e s reso ve, so e y ersist in efi-
exerts its h r co ogic effect through o - nite y.
u ting coho s reinforcing effects. It h s
been show n to signific nt y re u ce the risk of 95. (A) This tient h s e iriu , co on co -
he vy rinking in tients. Another v n- ic tion in the ICU. Low - ose nti sychot-
t ge is th t it c n be converte to onth y ics su ch s ri i r zo e c n be very he fu if
intr u scu r injection, w hich c n incre se u se ju icious y. H o eri o is often use in
co i nce. Whi e isu fir , n versive c inic r ctice; how ever, IV h o eri o c n
gent th t inhibits ehy e ehy rogen se, engthen the QTc n therefore shou not be
c n so be use , ore recent stu ies h ve re - u se for this rticu r tient. Benzo i z-
eg te it to secon - ine e ic tion. N one of e ines su ch s i ze n or ze c n
the other e ic tions iste re effective for w orsen e iriu in the e er y n shou
coho use isor er: Meth one is ong- be voi e . Ris eri one (Ris er Const )
cting o ioi gonist u se in o i te u se isor- is the ong- cting for of ris eri one, n
er. N either sertr ine ( n nti e ress nt) nor w ou not be ro ri te in this setting.
v roic ci ( oo st bi izer) tre ts coho
u se isor er in the bsence of n u n er ying 96. (E) The tient ike y suffers fro erfor nce
oo isor er. nxiety, for of soci nxiety isor er
(soci hobi ). Pro r no o , bet b ocker,
93. (D ) A of the e ic tions iste re s ee i s; is very he fu in n ging erfor nce
how ever, on y z e on, nonbenzo i ze ine nxiety. T ken i e i te y before the fe re
hy notic w ith h f- ife of on y 4 hou rs, is event, it ecre ses he rt r te, b oo ressu re,
ro ri te for i e inso ni (f ing n tre or, n consequent y ecre ses nxi-
s ee bu t frequ ent y w kening u ring the ety. Bu s irone is n nxio ytic gent th t is
night). A of the tern tives iste re ore he fu in the tre t ent of gener ize nxi-
u sefu for eo e w ho h ve initi inso ni ety isor er. F uoxetine n f u vox ine
( ifficu ty f ing s ee ) n requ ire e ic - re nti e ress nts u se in the tre t ent of
tion t be ti e. A itri ty ine n tr zo one gener ize nxiety isor er n nic isor-
re both se ting nti e ress nts. R e teon er. Lor ze is benzo i ze ine th t c n
is e tonin-rece tor gonist. Zo i e is be he fu in n ging cu te nxiety, bu t its
nother nonbenzo i ze ine hy notic w ith cco nying se tion y be rob e tic
onger h f- ife th n z e on. in erfor nce context.

94. (E) ECT h s re tive y few con tr in ic tions 97. (B) The tient resents w ith history of bi o-
n in so e c ses is referre for its r i r isor er n is cu rrent y e resse . Tre t-
onset of ction. H ow ever, bec u se of the ent for bi o r e ression is co ic te ;
c r iov scu r effects of ECT, history of how ever, the cu rrent best r ctice ro ches
recen t yoc r i inf rction (w ithin the inc u e u r si one, qu eti ine, n ithiu s
st 6 onths) is re tive con tr in ic - onother ies. The tient h s re y f i e
tion. Another re tive contr in ic tion is the ithiu n w ou be better serve by iffer-
resence of c inic y sign ific nt in tr cr - ent tre t ent. Lu r si one is w eight neu tr
ni s ce-occu ying esion u e to the risk ty ic nti sychotic th t h s been show n to
of br in ste herni tion. ECT y be er- be effective for the tre t ent of bi o r e res-
for e in tients w ith egener tive isc sion. Queti ine is so n effective tre t ent,
ise se or hy ertension, n u ring reg- how ever it is ssoci te w ith w eight g in n
n ncy. ECT is s fe n effective tre t ent the tient is re y overw eight. Sertr ine
for sychotic e ression. The ost co on n other SSRIs h ve ixe evi ence for bi o-
co ints tients h ve fo ow ing ECT re r e ression, n they shou never be re-
i ir en ts in both nterogr e n ret- scribe s onother y s they incre se the
rogr e e ory. A th ou gh ost e ory r te of fu tu re nic e iso es. V roic ci
Answe rs : 92109 119

is secon - ine tre t ent for bi o r e res- his jor e ression. Rest rting sertr ine
sion; it is ore effective for cu te ni th n w ou be ike y to c u se the s e rob e s
cute e ression. n e to non herence.

98. (D ) A thou gh there re v riou s si e effects 102. (D ) This tient h s sy to s of gener ize
of otrigine, the ost ngerou s is th t nxiety isor er. A of the e ic tions iste ,
of severe r sh, w hich c n eve o into exce t c on ze , benzo i ze ine, c n
StevensJohnson syn ro e, otenti y ife- inter ct neg tive y w ith w rf rin. Bu ro ion
thre tening co ic tion. Beginning t ow is so ike y to w orsen nxiety. Bu s irone n
ose n s ow y incre sing re u ces the risk cit o r re re son b e choices for his nxi-
signific nt y. Acu te ystoni n k thisi re ety, bu t his PT/ PTT n IN R w ou h ve to
both si e effects of nti sychotic e ic tions. be onitore c refu y. Ven f xine y so
A stic ne i is r re si e effect of c rb - e ev te b oo ressu re.
ze ine. Ren f i u re c n occur w ith ithiu .
103. (D ) Pri is , infu , ro onge erection,
99. (C) A of these re ty es of ystonic re ctions is r re bu t seriou s si e effect of tr zo one.
th t re in u ce by the u se of nti sychotics Erecti e ysfu nction refers to ifficu ty obt in-
(neu ro e tics). O isthotonos, so know n s ing or int ining n erection, co on
arc de cercle, is s s of the neck n b ck si e effect of SSRIs. N y ho ni is ins ti -
th t c uses the tient to rch forw r . The b e sexu esire in w o n w hi e s tyri sis
ost r ing of these ystoni s is ryn- is its cou nter rt in en. A r r xis, or s i
gos s , w hich is the s s of the u sc es of the tongu e, is so know n s Freu i n
contro ing the tongue n the thro t; this c n s i .
e to res ir tory istress. Ocu ogyric cri-
sis is s s of the extr ocu r u sc es, often 104. (A) Sexu ysfunction (e.g., ecre se ibi o,
resu ting in the tient ooking u n u n b e erecti e ysfunction, norg s i ) is not unco -
to ook ow n. P eu rothotonos, so know n s on y seen w ith SSRIs n other nti e res-
Pis syn ro e, is e ning ostu re in u ce s nts (such s those iste ). Bu ro ion is one of
by the s s of the torso u sc es. Tortico - the few nti e ress nts w hich c uses itt e to
is is s s of the neck u sc es th t u su y no sexu ysfunction, n , in f ct, y reverse
brings the neck to one si e or nother bu t y the si e effect w hen e to tre t ent. In
so u forw r or b ckw r . ition, ny other sychotro ic e ic -
tions inc u ing ithiu n nti sychotics c n
100. (B) A thou gh ny of the iste nti e res- so c use sexu ysfunction.
s nts w i tre t her e ression equ te y,
on y u oxetine, co bine serotonin- 105. (D ) To ir te is n nticonvu s nt u se
nore ine hrine reu t ke inhibitor, h s been in the tre t ent of bu i i n binge e t-
rove in the tre t ent of both e ression ing isor er. It c rries w ith tw o to fou r
n neu ro thic in. Given her ike y jor ti es incre se risk of ren stones. To i-
e ressive isor er, re ssur nce w ou not r te often c u ses cognitive i ir ent,
be ro ri te or effic ciou s. rticu r y w or -fin ing ifficu ty, short-
ter e ory rob e s, n rob e s w ith
101. (A) With the exce tion of irt z ine, bu ro- ttention. P ncre titis is r re si e effect of
ion, n nef zo one (now off the rket v roic ci , n o yu ri is seen w ith ith-
ue to risk of he tic f i ure), nti e res- iu u se. To ir te c u ses w eight oss, not
s nts c n c u se signific nt sexu ysfu nction, w eight g in.
inc u ing ecre se ibi o, erecti e ysfu nc-
tion, n norg s i . Given the recurrent 106109. [106 (C), 107 (A), 108 (E), 109 (I)] For ic -
n tu re of his i ness, e ving hi w ithou t tion is the fee ing of bu gs cr w ing on the skin,
e ic tion w ou ike y e to re se of u su y ssoci te w ith coho w ith r w ,
120 3: S oma tic Tre a tme nt a nd P syc hop ha rma c ology

bu t so w ith ru g intoxic tion, rticu r y the n tu r o ine inhibition of ro ctin


w ith coc ine. A otiv tion syn ro e is re e se fro the nterior itu it ry c u sing
ck of rive or otiv tion u su y ssoci- g ctorrhe (E) n gyneco sti (F). The
te w ith chronic ru g u se, es eci y c nn - si e effects th t rise fro o ine b ock-
bis. Inh nts, inc u ing g so ine, re vo ti e e in the b s g ng i c u se v riou s ys-
subst nces th t h ve r i -onset intoxic ting toni s, inc u ing b e h ros s (s s of
effects. To er nce is the nee for higher oses the eye i s) (C), o isthotonos (s s of the
of ru g in or er to chieve the s e effect. neck n b ck c u sing n rche ostu re)
M riju n is c ssifie s c nn bis. H u ci- (I), n tortico is (s s of the sternoc ei o-
nogen ersisting erce tion isor er is so stoi ) (P). The b ock e of D 2 rece tors in
know n s f shb ck. It is ch r cterize by the nigrostri t thw y c n so c u se other
istressing retu rn of erce tu ch nges extr yr i syn ro es su ch s k thisi
w ithou t the ingestion of h u cinogens, su ch (su bjective fee ing of rest essness) (B), seu -
s LSD. Intoxic tion is re resente by beh v- o rkinsonis (L) w hich rese b es P rkin-
ior , cognitive, or erce tu ch nges t ken son ise se, ch r cterize by u sc e rigi ity
in re tionshi to ru g ingestion. Psycho ogi- n short festin ting g it, r bbit syn ro e
c e en ence is cr ving or nee to con- (M) ( r re si e effect of ong-ter nti sy-
tinu e t king rug, frequent in those tients chotic u se ch r cterize by invo u nt ry, fine,
w ith su bst nce u se isor ers. Su bst nce u se rhyth ic otions of the ou th n i s), n
isor er enco sses revious i gnoses of t r ive yskinesi (O), isor er of bnor-
buse n e en ence n w ys inc u es invo u nt ry ove ents c use by ro-
continu e u se of su bst nce (e.g., coho or onge u se of nti sychotics. An i iosyncr tic
i icit ru gs) in s ite of verse consequ ences bu t r re co ic tion of nti sychotics (neu -
re te to the su bst nce (fu nction i ir- ro e tics) is neu ro e tic ign nt syn ro e
ent). With r w is set of signs n sy - (G). This is ch r cterize by hy erther i ,
to s, hysic n sycho ogic , th t re ch nge in ent st tu s, n incre se u s-
ssoci te w ith coho or ru g cess tion. c e tone. It c n e to rise in u sc e bre k-
ow n ro u cts (cre tine kin se eve s re
110. (D ) Anti sychotic gents, in ition to b ock- ty ic y e ev te ), ren f i u re fo ow ing
ing o ine rece tors in the nigrostri t ehy r tion, u on ry co ic tions, n
thw y (c u sing extr yr i sy to s) e th. C oz ine c n c u se gr nu ocytosis
n the eso i bic thw y ( ecre sing h - (A), ngerou s si e effect th t so eti es
u cin tions), so b ock o ine in the tu ber- c n resent w ith sore thro t n fever; if
oinfu n ibu r syste . Do ine nor y this occu rs, the ru g u st be sto e i e-
inhibits ro ctin re e se in this thw y, i te y. Consti tion (D ) is co ic ting
n therefore o ine b ock e incre ses si e effect of so e nti sychotics, rticu -
ro ctin eve s, c u sing sexu ysfu nction, r y those w ith nticho inergic effects. This
gyneco sti in en, n g ctorrhe in c n often be re ieve w ith u se of x tive.
w o en. The ost ike y e ic tions to c u se Obstru ctive j u n ice (H) is r re n occu rs
these si e effects re first-gener tion nti sy- in y w ith ch or ro zine. Sy to s
chotics n ris eri one. inc u e fever, n u se , ise, n ru ritu s.
Orthost tic hy otension (J), su en ro in
111121. [111 (N ), 112 (M), 113 (A), 114 (L), 115 (O), b oo ressu re u on st n ing, is u e to h -
116 (G), 117 (E), 118 (K), 119 (B), 120 (H), 121 1- renergic b ock e. It is es eci y trou b e-
(J)] Anti sychotics, so know n s neu ro e - so e w ith ch or ro zine n c oz ine
tics, c n c u se w i e v riety of si e effects. n best h n e by r ising oses s ow y.
These si e effects c n be broken ow n into Pig ente retino thy (K) is observe w ith
so e of the fo ow ing c tegories for n e s- high oses of thiori zine n best etecte
ier ro ch. The o inergic si e effects w ith goo o hth o ogic ex in tion. An
inc u e those th t re u e to the b ock e of ition si e effect of so e nti sychotic
Answe rs : 110123 121

e ic tions th t y ecre se co i nce zu res re r re y foc n u su y occu r w ithin


is retrogr e ej cu tion (N ); org s y be 48 hou rs fter coho consu tion ce ses.
chieve bu t ej cu tion is bnor , w ith De iriu tre ens ost often occu rs w ithin
retrogr e ro u sion of the se en, resu ting 72 hou rs of cess tion of rinking, thou gh
in c ou y e r nce of the u rine. y e k for sever ys fu rther.

122. (D ) A coho w ith r w c n t ke the for of 123. (C) This tient is exhibiting sy to s of
inor w ith r w sy to s su ch s sw e t- hy on tre i , re tive y co on si e effect
ing, f u shing, n tre u ou sness, or ore of oxc rb ze ine, so obt ining seru e ectro-
serious consequ ences inc u ing w ith r w ytes w ou be critic . The other tests w ou
seizu res or w ith r w e iriu , so know n not he in i gnosing hy on tre i , n
s e iriu tre ens. Minor w ith r w sy - oxc rb ze ine eve s re not rou tine y checke
to s c n st for sever ys. With r w sei- (un ike eve s of c rb ze ine).
This page intentionally left blank
CHAPTER 4

Ps yc ho lo g ic al Tre atme nt and


Manag e me nt
Que s tions

DIRECTIONS (Questions 1 through 42): For each of (C) Ego strengths


the multiple-choice questions in this section, select (D) Projections
the lettered answ er that is the one best response in (E) Unconsciou s thou ghts
each case.
3. A 43-year-old m an is chastised at w ork. When
Questions 1 and 2 he com es hom e, his friend asks him how his
d ay w ent. H e respond s angrily saying that a
A 38-year-old m ale thinks that he is a failu re and real friend w ou ld nt be so nosy. Which of
w ill never find a p artner w henever he feels rejected . the follow ing d efense m echanism s m ost likely
H e often find s him self getting d epressed and isolat- characterizes this reaction?
ing him self w hen he thinks like this. H is therap ist
thinks that it w ould be a good id ea for him to m ake (A) Denial
a log of the situ ations, his im m ed iate thou ghts, his (B) Disp lacem ent
im m ed iate feelings, alternate resp onses to those (C) H u m or
thoughts and feelings, and re-rate his feelings after (D) Intellectu alization
com pleting the exercise so they can be exam ined (E) Isolation of affect
d uring their sessions.
4. A 20-year-old w om an d iagnosed w ith bord er-
1. Which of the follow ing types of therapeu tic line p ersonality d isord er thinks of her thera-
ap proaches is the therapist m ost likely u sing? p ist as the best person Ive ever know n. The
(A) Cognitivebehavioral next w eek, the therapist announces that he
(B) Exp ressive w ill be going on vacation. The patient becomes
enraged and states that he is the cru elest
(C) H yp notherap y
d octor in the w orld ! Which of the follow ing
(D) Psychoanalysis d efense mechanism s best characterizes this
(E) Psychod ynam ic reaction?

2. Which of the follow ing term s refers to the (A) Devalu ation
im m ed iate thou ghts (e.g., I am a failu re) (B) Id ealization
that the p atient exp eriences and record s in this (C) Intellectu alization
type of therapy? (D) Repression
(A) Au tom atic thou ghts (E) Sp litting
(B) Core beliefs

123
124 4: P syc holog ica l Tre a tme nt a nd Ma nag e me nt

5. A 16-year-old boy has just lost his mother and m ind . Which of the follow ing analytic tech-
father in a car accident. In your office, he talks niques d oes this best rep resent?
philosophically about death and its implica-
(A) Free association
tions. When asked how he feels about his par-
ents death, he responds by saying that it is the (B) H yp nosis
nature of things to pass away. Which of the fol- (C) Rep ression
lowing defense mechanisms best d emonstrates (D) Thou ght record s
this patients inability to talk directly about his (E) Transference
emotional experience concerning personal loss?
(A) Denial 9. A psychotherapist has been m eeting regu larly
w ith a 10-year-old boy for 18 months. During
(B) Intellectu alization
one of the therapy sessions, she find s herself
(C) Projection feeling very angry at the patient after he reports
(D) Su ppression beating up his little brother. Which of the
(E) Und oing follow ing phenomena is the psychotherapist
most likely exhibiting?
Questions 6 and 7
(A) Countertransference
A 44-year-old w om an w ith schizop hrenia is stru g- (B) Extinction
gling w ith p aranoia, au d itory hallu cinations, and (C) Interp retation
d elu sions. She lives w ith her m other but has a poor (D) Resistance
relationship w ith her. She tells you that everyone
(E) Transference
w ants her to sp end a lot of m oney and bu y d ru gs.
10. In an acu te inp atient p sychiatric u nit, the
6. Which of the follow ing resp onses w ou ld be
resid ent p sychiatrist is in charge of lead ing a
the m ost usefu l?
grou p consisting of p atients new ly ad m itted
(A) Well, d ecid ing for you rself is best. to the w ard . Du ring group d iscu ssion she asks
(B) If you bu y d ru gs, Ill call the p olice. tw o participants to act out a scenario, d escribe
(C) Perhap s w e shou ld look at w hat you r their feelings abou t the situ ation, and then
m other w ou ld think abou t that. exp lore the ind ivid u al conflicts w hich arose.
Which of the follow ing techniques of p sycho-
(D) Why d o you think everyone w ants you
therapy d oes this m ost represent?
to d o that?
(E) N o, they d ont. (A) Feed back
(B) Free association
7. You d ecid e that su p p ortive therap y w ou ld be (C) Go-arou nd
helpfu l to this patient. Which of the follow ing (D) Psychod ram a
w ou ld be the m ost ap p rop riate goal of su p -
(E) Resistance
p ortive therap y for this p atient?
(A) Correcting fau lty id eas Questions 11 and 12
(B) Exp loring the feeling of m eaninglessness
A 28-year-old fem ale p atient w ho is ad d icted to opi-
in life
oid analgesics states that she has been up set w ith
(C) Investigating the freed om of ind ivid u als fam ily for not continu ing to su p p ort her financially
(D) Personality change and allow ing her to stay w ith them . She has been
(E) Strengthening of d efenses u nem ployed for m any years since leaving college.
They recently told her that she shou ld be m ore ind e-
8. A 32-year-old p sychiatry resid ent is in p sycho- p end ent; they w ant her to start w orking and find a
analysis 4 d ays p er w eek. H e is encouraged to p lace to live. In her next therap y session, she tells
lie on the cou ch and say w hatever com es to her therapist that she u nd erstand s w hy they w ou ld
Que s tions : 516 125

say that, bu t then goes on to d escribe a fight w ith her 14. A 34-year-old male is referred by his job because
boyfriend , stating she w ants to break u p w ith him of ongoing interpersonal conflicts. During
becau se he is not su p p ortive of her. H er therap ist the interview, he appears very focused on his
asks if it is p ossible she is u pset at her parents for health, his attractiveness to others, and his suc-
w anting her to be m ore ind ep end ent bu t exp ressing cess at work. When confronted with his diffi-
this anger tow ard her boyfriend instead . culties, he becomes defensive, blaming others,
and accusing them of being jealous of me. He
11. Which of the follow ing therapeu tic techniqu es describes himself as better than his friends
best d escribes the therap ists resp onse? and colleagues and admits to taking advantage
of others in order to get what I deserve. Which
(A) Cou ntertransference
of the following treatment modalities would be
(B) Em p athy the most appropriate for this patient?
(C) Interp retation
(A) Anger m anagem ent
(D) Parad oxical intervention
(B) Com bined ind ivid u al and grou p
(E) Working throu gh
therapy
12. The p atients fam ily m em bers and close (C) Psychoanalysis
friend s w ork w ith the therap ist and join the (D) Psychopharm acotherap y
p atient in a session. Du ring the m eeting, they (E) Social skills training
tell her about her m alad aptive behaviors and
how they have affected her and her fam ily 15. A 29-year-old w om an has been d epressed for
negatively. They then give her an u ltim atu m , 2 months prior to seeking med ical attention.
w hereby if she d oes not get treatm ent for her She believes that nobod y likes her even though
ad d iction they w ill not continu e to su pp ort she is alw ays cord ial, and that there is noth-
her. Which of the follow ing techniqu es best ing she can d o to change the w ay other people
d escribes this scenario? perceive her. Therefore, w hen she is around
peop le she feels tense and u ncomfortable and
(A) Behavioral therap y
has started avoid ing leaving the hou se. Which
(B) Fam ily therap y of the follow ing cognitivebehavioral therapy
(C) Grou p therap y (CBT) interventions w ould most d irectly target
(D) Intervention the physical feelings she has around people?
(E) Relap se p revention (A) Cognitive restru cturing
(B) Exp osu res
13. A 24-year-old female patient with a history of
generalized anxiety disorder presents for therapy. (C) Keep ing a m ood d iary
In a previous session she reluctantly discussed (D) Keep ing a thou ght record
how much she did not like her new step-mother (E) Progressive m u scle relaxation
and didnt approve of her father marrying her.
This session she reports that her step-mother is 16. After several weeks, a 40-year-old female patient
in the hospital and she has been spending most who is in psychodynamic psychotherapy begins
of the past few days at the hospital caring for her, to show up late and miss appointments; each
buying many get-well gifts and inserting herself time this happens she has a reason for doing
in her step-mother s care. Which of the following so. However, the therapist begins to feel that
concepts best describes this behavior? her behavior is interfering with her treatment.
The therapist is not sure if the patient under-
(A) Abreaction
stands that her behavior is inappropriate and
(B) Dep end ency potentially damaging to her therapy. Which of
(C) Projection the following topics would be the most impor-
(D) Reaction form ation tant to review with the patient before making
(E) Sp litting an interpretation of her behavior?
126 4: P syc holog ica l Tre a tme nt a nd Ma nag e me nt

(A) Bou nd aries and ru les d efining the w ay (D) Interp retation
in w hich therap y is cond u cted (E) Refram ing
(B) Focu s on the therapeu tic goals
(C) Method u sed to change the p atients Questions 19 and 20
m alad aptive thou ghts
An 18-year-old , single m an p resents to you r office
(D) Patients need for attention com plaining that he cannot pass a m ovie theater
(E) Reasons the p atient is late to the w ithou t stop p ing, going insid e, and bu ying cand y.
ap pointm ents This behavior is trou blesom e to him and interferes
w ith his d aily activities, resu lting in his being con-
17. A 45-year-old m an p resents to you r ou tp atient stantly late to other app ointm ents. You d ecid e to
clinic hand icap ped by a fear of parking lots help the p atient w ith behavior therap y.
and field s. The fear started 4 m onths p rior to
this visit. At the beginning of cognitive ther- 19. Which of the follow ing w ould be the m ost
ap y, he tells you that his behavior is constantly ap p rop riate focu s of the therapy?
being scru tinized and criticized by other
p eop le. H e claim s that he cannot change his (A) Analyzing the p atients relationship s
behavior, becau se if he d oes, others w ill think (B) Decreasing the m alad ap tive behavior of
he is a fool. Which of the follow ing w ou ld be stopping by m ovie theaters
the m ost appropriate response? (C) Exam ining the p atients negative eating
habits
(A) Thats silly. People w ill not think you
are a fool. (D) Exp loring the p atients child hood
trau m as
(B) Yes, I can see that.
(E) Working on resolving the p atients
(C) What m akes you think th at others
u nconsciou s conflicts
are con stantly scru tinizing you r
beh avior?
20. Which of the follow ing behavioral interven-
(D) H ow can you possibly think other tions w ould be the m ost help fu l in this case?
p eople care enou gh abou t you to
constantly scru tinize your behavior? (A) Aversion therap y
(E) Well, m aybe you are a fool. H ave you (B) Environm ental m od ification
ever thou ght of that? (C) Exp osu re and resp onse training
(D) Mod eling
18. A 32-year-old fem ale in therap y d escribes (E) Relaxation training
how she feels that she should have a life p art-
ner; and since she is not currently d ating, she 21. A 39-year-old m an p resents to you r office
is u nlikely to ever get m arried and is therefore com plaining of chronic stress, anxiety, and
u nlovable. H er therapist points ou t to her that p oor sleep. H e also ad m its to interm ittent
she has had a few m eaningful long-term rela- head aches, w hich you d iagnose as tension
tionships in the past and that she has d ated as head aches. You recom m end behavioral ther-
recently as a cou p le of m onths ago. The thera- ap y for red u ction of his head aches. Which of
p ist also explains that m any p eople m ay not the follow ing behavioral techniqu es w ould be
d ate, bu t this d oes not m ean they are u nlov- m ost appropriate for this patient?
able or w ill never get m arried . Which of the
follow ing term s best d escribes the therap eu tic (A) Aversive stim u li
intervention d escribed above? (B) Biofeed back
(C) N egative reinforcem ent
(A) Clarification
(D) Stim u lu s control
(B) Confrontation
(E) System atic d esensitization
(C) Em p athy
Que s tions : 1726 127

Questions 22 and 23 (C) Com p lete hom ew ork frequ ently to


better learn abou t their issu es.
A 32-year-old m ale com p lains of ongoing d epression
(D) Prevent conflicts from arising betw een
for the past m onth w hich he attribu tes to the break-
the therapist and p atient.
up w ith his fiance. H e has been having insom nia,
d ecreased appetite, and low energy, althou gh he (E) Requ ire m ore intensive treatm ent d u e to
d enies anhed onia or problem s concentrating. H e severity of m ental illness.
also d enies any su icid al id eation. H e has ju st started
to d ate som eone else w ithin the last w eek. After d is- 25. Which of the follow ing w ou ld be the m ost
cussion w ith the patient, it is d ecid ed to u tilize both im p ortant tool in his p sychoanalysis?
m ed ication and interp ersonal psychotherap y for his (A) Altering cognitive d istortions
d ep ression. (B) Altering m alad ap tive behaviors
(C) Interp retation of transference
22. Which of the follow ing goals w ou ld be the
m ost likely focu s of his p sychotherapy? (D) Interp reting d ream s
(E) Solving interp ersonal p roblem s
(A) Character change
(B) Clarifying com m u nication p atterns 26. A m ale th erap ist h as been w orkin g w ith a
(C) Strengthening d efenses fem ale p atient for over 3 years, enabling her
(D) Interpreting transference to im p rove her self-esteem and confid ence,
(E) Pointing ou t resistance and to eventu ally d ivorce her em otion ally
abu sive h u sban d 1 year ago. The p atient con-
23. Which of the follow ing techniqu es w ou ld tinu ally thanks the therap ist and exp lains
be m ost likely u tilized d u ring this p atients that she is sexu ally attracted to the therap ist.
therapy? H e is initially taken aback and then som e-
w hat flattered by her sexu al interest. H ow -
(A) Dream interp retation ever, he also realizes that he feels a strong
(B) Defense analysis attraction to her and feels the n eed to care
(C) Free association for her. Which of the follow ing w ou ld be the
(D) H ypnosis m ost ap p rop riate n ext step for the th erap ist
to take?
(E) Role p laying
(A) Act cold , show less em p athy, and set
Questions 24 and 25 m ore rigid bou nd aries d u ring sessions.
A 22-year-old college stud ent presents com plaining (B) Exp lore the p atients feeling in ord er to
of having no goals in his life, not d oing as w ell as he increase her erotic transference.
could in school, and d esiring but having no serious (C) Ignore the p atients d isclosu re of her
long-term romantic relationships. H e has not chosen sexu al attraction.
a m ajor and states that his ind ecision is paralyzing (D) Seek su p ervision w ith a m entor or
him from m oving on w ith the rest of m y life. After colleague.
d iscussing various form s of psychotherapy w ith the (E) Term inate or fire the p atient from his
patient, you recomm end psychoanalysis fou r times care.
per w eek.
Questions 27 and 28
24. Which of the follow ing is the m ost im p ortant
reason for the frequ ency of p sychoanalysis? You are seeing a 25-year-old man in cognitivebehav-
ioral therapy w ho suffers from generalized anxiety
(A) Allow the p atient to get enou gh su p p ort d isorder, as w ell as epilepsy. H e often talks about
d u ring the course of their treatm ent. the many dreams he has w hile in his sessions, and
(B) Allow transference to bu ild betw een he is having a hard time w ith the restructuring of
p atient and therapist.
128 4: P syc holog ica l Tre a tme nt a nd Ma nag e me nt

malad aptive thoughts. You d ecid e that hypnosis may relationships. He blames the problem on his
be helpful as an adjunctive therapy for this patient, sabotaging things when the women start to
as he has been hypnotized before w ith good results get serious. He is clearly ambivalent about his
and you have a skillful hypnotist in your clinic. The desire to settle down with someone. Accord-
patient is hopeful that hypnosis w ill be beneficial for ing to Erik Erikson, which of the following stages
him again. of development is the patient most likely in?
(A) Tru st versu s m istru st
27. Which of the follow ing characteristics is the
best p red ictor of resp onse to hyp nosis in this (B) Au tonom y versu s sham e
p atient? (C) Initiative versu s gu ilt
(D) Intim acy versu s isolation
(A) Patient has a lot of d ream s.
(E) Id entity versu s role confu sion
(B) Patient is resp onsive to su ggestion.
(C) Patients d iagnosis. 31. A 9-year-old boy w ith attention-d eficit/
(D) Patient w ith low seizu re threshold . hyperactivity disorder (ADHD), predominantly
(E) Trained hyp notist w ill be d oing his hyp eractive/ im p u lsive presentation, presents
treatm ent. to you r office accom panied by his m other. H e
is cu rrently on m ethylp henid ate (Ritalin) and
28. The p atient is started in hyp nosis, bu t there they both w ond er if there is som ething else
is a concern as to w hether he is in a trance that can be d one to help the patient. Despite
d u ring the sessions. Which of the follow ing som e benefit and tolerability, the child contin-
ind icators w ou ld be m ost likely increased in a u es to have d ifficulty w ith com pleting hom e-
trance state? w ork and follow ing instru ctions at school
and at hom e. H e has also recently gotten
(A) Am nesia
into a m inor p hysical fight w ith a p eer. Par-
(B) Pain p ercep tion ent m anagem ent training is recom m end ed for
(C) Pu lse the parents. Which of the follow ing aim s best
(D) Reflexes illu strates how p arent m anagem ent training
(E) Resp irations w ill help the fam ily?
(A) Focu sing on m alad ap tive
29. You are treating an 11-year-old boy for op p osi- com m unication in the fam ily
tional d efiant d isord er and have heard from
(B) Focu sing on the fam ilys interp ersonal
the p arents that his d isord er has had a sig-
relationship s
nificant im p act on the fam ily d ynam ics. They
have tw o old er child ren w ho often feel left (C) Increasing p arental u se of p ositive
ou t and feel d istanced from the fam ily u nit. reinforcem ent to increase d esired
You recom m end a trial of fam ily therap y. behaviors
In this case, w hich of the follow ing w ou ld (D) Interp reting p arents negative cognitions
be the m ost ap p rop riate focu s of the fam ily abou t the p atient
therap y? (E) Teaching parents to p ositively refram e a
negative exp erience
(A) Assigning roles in a hou sehold
(B) Early child hood exp eriences 32. A 14-year-old boy w ith attention-d eficit/ hyper-
(C) Im p u lsive behavior activity d isorder (ADH D) is frequently late for
(D) Relationship p atterns school and forgets to d o his chores around the
(E) Unconsciou s conflicts house d ue to his d isorganization. H is parents
are fed up w ith him and ask you w hat they can
30. A 32-year-old man presents for psychother- d o to help change his behavior. Which of the
apy. He denies pervasive depression or anxi- follow ing represents the most helpful behav-
ety symptoms, but he has had a string of failed ioral tool for patients w ith ADH D?
Que s tions : 2737 129

(A) A chart of behaviors to be rew ard ed by d iagnosed w ith schizop hreniform d isord er.
p arents H e recovers after 4 m onths bu t has another
(B) Aversion therapy relap se 6 m onths later in the absence of an
(C) Interp retation of m alad ap tive behaviors elevated , irritable, or d ep ressed m ood . H e is
then given a d iagnosis of schizop hrenia and
(D) Positively refram ing a negative
referred for m ed ication m anagem ent and
exp erience
therap y. After a few w eeks of therap y, the
(E) Pu nishm ent p atient exp eriences a w orsening of his p sy-
chotic sym p tom s. Which typ e of therap y w as
Questions 33 and 34 he m ost likely referred to?
A 25-year-old u nm arried m an in his fou rth year of (A) Behavioral therap y
m ed ical school presents to you r office com p laining
(B) Fam ily therap y
of not being able to rem ain in a relationship w ith
(C) Grou p p sychotherap y
a girlfriend for longer than 3 m onths. H e says that
he w ould like an insight-oriented therap y, and you (D) Psychoanalytic p sychotherap y
agree that this form of therapy w ou ld help him . You (E) Su p p ortive p sychotherap y
decide to treat this patient with brief psychod ynamic
p sychotherap y. 36. A 50-year-old m an is in p sychoanalysis
becau se he feels stagnant in his p ersonal
33. Which of the following factors would best predict life. H e notes he is not having any p roblem s
a positive outcome with brief psychodynamic w ith his w ork or relationship w ith his w ife,
psychotherapy in this particular patient? althou gh he feels that he has never had m ore
than a p latonic relationship w ith her. While he
(A) Age feels cap able and confid ent in m ost sp heres
(B) Gend er of his life, inclu d ing his relationship s, he feels
(C) Marital statu s like he is at a road block in therap y and d oes
(D) Motivation for change not know how to p roceed . H e has, how ever,
(E) Socioeconom ic statu s m entioned a reoccu rring d ream about his
w ife tu rning into his m other and taking care
34. Which of the follow ing asp ects best d istin- of him . The therapist thinks that interpreting
gu ishes his sp ecific typ e of p sychotherap y his d ream m ay be a good tool to u se in ther-
from other form s of therap y? ap y. Which of the follow ing u ses of his d ream
w ou ld be the m ost ap p rop riate in this case?
(A) Correcting cognitive errors
(B) Id entification of a focal conflict (A) Interp reted by the analyst to
su blim inally influ ence the m ind of the
(C) Interp reting d ream s
p atient.
(D) Interp reting resistance
(B) Provid e inform ation abou t p sychic
(E) Mod ifying m alad ap tive behaviors conflicts.
(C) Rep resent the consciou s fram ew ork for
35. A 20-year-old m an is cu rrently in college and
behavior.
has com p leted the first sem ester of his sop ho-
m ore year. H alfw ay into the second sem ester, (D) Used to alter cognitive errors.
his grad es d rop from As to Fs over a 3-m onth (E) Used to change behavior.
p eriod . H e becom es increasingly isolated
and p aranoid , believing that the governm ent 37. You have been assigned to lead a d ialectical
is after him becau se he has solved all theo- behavioral therap y grou p for p atients w ith
logical p roblem s throu gh d irect com m u ni- bord erline p ersonality d isord er. Du ring the
cation w ith God . H e is started on olanzap ine grou p one of the p atients relates a story abou t
10 m g by m ou th (PO) d aily and is tentatively getting into a fight w ith her m other. Afterw ard
she w as so d espond ent she had thou ghts
130 4: P syc holog ica l Tre a tme nt a nd Ma nag e me nt

abou t cu tting herself. Instead of resp ond - Which of the follow ing stages of grou p form a-
ing, she w atched her favorite com ed y m ovie. tion is this grou p m ost likely in at this tim e?
Which of the follow ing skills d id the p atient
(A) Form ing
m ost likely em p loy?
(B) Storm ing
(A) Cognitive restructu ring (C) N orm ing
(B) Distress tolerance (D) Perform ing
(C) Em otion regu lation (E) Ad jou rning
(D) Interpersonal effectiveness
(E) Mind fu lness 41. A 38-year-old law yer is referred for behavioral
therapy because of extrem e social anxiety
Questions 38 and 39 w hich is interfering in his ability to litigate. In
his early therap y sessions, he is told to m ake
A 33-year-old w om an p resents to the em ergency a hierarchy of situ ations that m ake him anx-
d epartm ent saying that she w ants to kill herself. She iou s. The therap ist then begins w orking w ith
has felt increasingly d epressed for m any m onths, him by first exposing him to the least anxiety-
w ith late insom nia, p oor ap p etite, w eight loss, d is- p rovoking item s on the list and then grad u ally
tractibility, and fatigu e. She cu rrently feels hopeless increasing the severity. Which of the follow ing
and help less, and has a p lan to overd ose on p ills therapeu tic techniques is the therapist m ost
that she has stockpiled at hom e. likely u tilizing?

38. Other than her feelings of helplessness and (A) Aversion


hopelessness, w hich of the follow ing cognitive (B) Flood ing
d ifficulties w ou ld she m ost likely also exhibit? (C) Mod eling
(A) Begging the question (D) Su ggestion
(B) Circu lar reasoning (E) System atic d esensitization
(C) Personalization
42. A 47-year-old , d ivorced m ale has been
(D) Rigid , black-or-w hite thinking
attend ing p sychotherap y for 8 m onths d u e to
(E) Selective abstraction ongoing stress and conflicts at w ork. Du r-
ing m ost of his therap y sessions, the p sychia-
39. After hospitalization, w hich of the follow ing trist resp ond s to his com m ents w ith em p athic
form s of p sychotherap y w ou ld be the m ost resp onses. Which typ e of p sychotherap y is
ap p rop riate to help m inim ize this patients the p sychiatrist m ost likely u tilizing in this
su icid ality? case?
(A) Behavioral therapy (A) Behavioral
(B) Cognitive therap y (B) Cognitive
(C) H yp nosis (C) Dynam ic
(D) Interpersonal p sychotherap y (D) Existential
(E) Parad oxical therap y (E) Su p p ortive

40. In the context of a w om ens grou p , one m em - D IRECTION S (Questions 43 through 47): The fol-
ber has been frequ ently sid e talking. As low ing group of questions is preceded by a list of
th e facilitator, you bring this u p d u ring the lettered options. For each patient scenario, select
grou p . Th e m em ber states she only d oes this the one lettered option that is most closely asso-
becau se you d ont give enou gh tim e for each ciated w ith it. Each lettered option may be used
m em ber to talk. The other grou p m em bers once, multiple times, or not at all.
agree w ith h er.
Que s tions : 3850 131

(A) Biofeed back 46. To train a 4-year-old boy to say thank you,
(B) Cou ntertransference his m other gives him a sw eet w henever he
(C) Flood ing d oes.
(D) H ypnosis
(E) N egative reinforcem ent 47. To train a 4-year-old boy not to bite his class-
m ates, his m other p uts soap in his m ou th
(F) Positive reinforcem ent
w henever he d oes.
(G) Projection
(H ) Projective id entification
D IRECTION S (Questions 48 through 50): The fol-
(I) Punishm ent
low ing group of questions is preceded by a list
(J) Real relationship of lettered options. For each question, select the
(K) Schem a one lettered option that is most closely associated
(L) Sp litting w ith it.
(M) Therap eu tic alliance (A) Josef Breu er
(N ) Transference (B) Jean-Martin Charcot
(O) Und oing (C) Erik Erikson
(D) Sigm u nd Freu d
43. You are seeing a 26-year-old w om an w ho com -
plains of m igraines. She is otherw ise healthy. (E) H einz Kohu t
You exp lain to her that you w ou ld like to try a (F) Franz Anton Mesm er
techniqu e in w hich you give her inform ation
on m u scle tension and tem peratu re. 48. Mad e m any contributions to p sychiatry and
neu rology w ith a sp ecial interest in d escribing
hysteria.
44. You are seeing a 45-year-old m an in p sycho-
analysis fou r tim es p er w eek. Du ring one ses-
sion he appears less com fortable talking. H e 49. Know n as the inventor of the cathartic treat-
eventu ally tells you that you r new glasses m ent or talking therap y.
rem ind him of his au nt w ho cared for him as a
child bu t w as cru el to him . 50. Know n as the d evelop er of self-p sychology.

45. A p atient w ho is afraid of clow ns is taken to


the circus in the beginning of her therapy in
ord er to get over her fear.
Ans we rs a nd Expla na tions

1. (A) Cognitivebehavioral therap y refers to a others. Unconscious thoughts are instinctual


variety of techniqu es that focus on the construc- thoughts, d rives, and need s that are ou t of
tion and reconstruction of peoples cognitions, ones aw areness.
emotions, and behaviors. Cognitivebehavioral
therapists use a range of m od alities to help 3. (B) Disp lacem ent is a d efense that transfers
p atients assess, recognize, and d eal w ith prob- a feeling abou t, or a resp onse to, one object
lem atic and d ysfu nctional w ays of thinking, (p erson) onto another object (p erson). Denial
em oting, and behaving. In this case, a thou ght is a d efense that keep s ou t of consciou s aw are-
record is being u sed to exam ine the p atients ness an asp ect of external reality or subjective
feelings and beliefs. Exp ressive therap y u ses d istress that is too u ncom fortable for the p er-
d ifferent form s of exp ression (e.g, art, m ove- son to accept. H u m or is consid ered a m atu re
m ent, m usic, or w riting) to have a patient d efense m echanism that em p hasizes the am u s-
exp ress his or her feelings and w ork throu gh ing or ironic asp ects of a stressor. Intellectu -
them . H ypnotherapy uses hypnosis to u ncover alization is a d efense in w hich the ind ivid u al
u nd erlying conflicts and hypnotic su ggestion favors abstract thinking over d ealing w ith the
in ord er to change behaviors. Psychoanaly- d istu rbing feelings of an id ea or exp erience.
sis encou rages patients to verbalize all their Isolation of affect involves d etachm ent of
thoughts, includ ing free associations, fanta- feelings from a p articu lar id ea or exp erience.
sies, and d ream s. The analyst u ses these to
u nd erstand the u nconsciou s conflicts causing 4. (E) Splitting occu rs w hen an ind ivid u al is
the patients sym ptom s or character problem s. u nable to see others m od erately, w ith both
Psychod ynam ic therap y attem pts to d iscover p ositive and negative qualities (i.e., peoples
the unconscious content of a patients psyche actions are either all good or all bad ). Devalu -
and then w ork on the issu es in therap y; it has ation is a d efense that attribu tes excessive
its root in p sychoanalysis. negative qu alities to another. Id ealization attri-
bu tes excessive p ositive qu alities to another.
2. (A) Au tom atic thou ghts are the im m ed iate Intellectu alization is a d efense that u tilizes
thoughts that com e to m ind in any situation excessive abstract thinking or generalizations
that are based on underlying core beliefs; these to m anage threatening em otions. Rep res-
are d eep ly held id eas regard ing the w orld sion expels d isturbing w ishes, thoughts, or
and oneself that d rive a patients em otional exp eriences from consciou s aw areness.
resp onses and behavior. Ego strength is the
ability of the ego to cope w ith com p eting 5. (B) This is an exam p le of intellectu alization, a
conflicts betw een the id , ego, and su perego d efense that u tilizes excessive abstract think-
w hich help one m anage stress and m aintain ing or generalizations to m anage threatening
stability. Projections are d efense m echanism s, em otions. Denial is a d efense that keep s out
w hereby an ind ivid u al u nconsciou sly attri- of consciou s aw areness an aspect of exter-
bu tes u nd esirable asp ects of oneself onto nal reality or subjective d istress that is too

132
Answe rs : 110 133

u ncom fortable for the person to accep t. Pro- their unconscious is explored . Rep ression is
jection is a d efense w hereby a thou ght, feel- a d efense m echanism that exp els d isturbing
ing, or id ea that is u naccep table to the p erson w ishes, thou ghts, or exp eriences from con-
is falsely attribu ted to another. Su p p ression scious aw areness. Thou ght record s refer to a
is w here one intentionally avoid s thinking of p atient record ing situ ations, feelings, au to-
d istressing thou ghts, feelings, id eas, or experi- m atic thou ghts and alternative responses then
ences. Und oing is a ritu alistic act com p leted re-rating the initial feelings; this techniqu e
in ord er to red u ce gu ilt abou t an event that is com m only u sed in cognitivebehavioral
occurred in the past. therapy. Transference, in strict term s, is the
re-exp eriencing of p ast exp eriences w ith the
6. (D ) The best strategy in any com m u nication is analyst in the setting of p sychoanalytic psy-
to try to gain clarity. Asking this patient w hy chotherapy. Cou ntertransference is the ana-
she believes w hat she d oes m ay lead now here lysts resp onse to this. H ow ever, these term s
bu t it m ay help to u nd erstand the p atients have com e to m ean the transferring of em o-
concerns. Choice (A) m ay be helpful in som e tions and feelings that one has from ones past
situ ations and certainly has a positive ring to the physician or care provid er in the case
to it. H ow ever, this patients com m ent is an of transference and the p hysician tow ard the
im p licit qu estion. She is asking for help and p atient in countertransference.
genu inely d oes not know w hat to d o. H id ing
behind p ithy m axim s is not u su ally help fu l. 9. (A) Cou ntertransference is the consciou s or
Choices (B) and (C) are coercive and block u nconsciou s em otional reaction of the thera-
com m u nication. Giving supp ortive guid ance p ist to the p atient. Transference is the con-
is ap p rop riate in this situ ation, bu t coercion scious or u nconsciou s em otional reaction of
is never ap p rop riate in a therap eu tic rela- the p atient to the therapist based on a past
tionship. Choice (E) is not coercive but also relationship the p atient had w ith a signifi-
blocks com m u nication. While the p atient m ay cant other person (e.g., a p arent). Extinction
requ ire reality testing, the resp onse m ay sacri- is a term frequ ently u sed in learning theory,
fice a m ore insightfu l, em p ow ering rep ly. w hich refers to the red u ction in frequ ency of a
learned resp onse as a resu lt of the cessation of
7. (E) The goal of su p p ortive therap y is the reinforcem ent. Interp retation is insight offered
strengthening of current d efense m echanism s. by the therap ist regard ing p atterns of thou ght
Throu gh su p p ortive care, the p atient can learn or behavior. Resistance is the patients u ncon-
to cope w ith d ifficu lt problem s u sing alread y scious opposition to full d isclosure of feelings
established abilities. Correcting fau lty id eas or id eas.
is the exp ressed goal of cognitive therap y.
Exp loring the feeling of m eaninglessness in 10. (D ) Psychod ram a is a techniqu e in w hich
life and investigating the freed om of ind ivid u - grou p m em bers are u sed as the au d ience
als are goals of existential p sychotherap y. Per- and the cast in reenactm ents of scenarios and
sonality change is a goal of psychod ynam ic conflict. The content of the d ram a is u sed to
p sychotherap y, w here d efenses are exam ined exp lore ind ivid u al p roblem s in a grou p setting.
grad u ally in ord er to achieve a m ore op tim al Feed back, a card inal featu re of grou p p sycho-
level of fu nctioning. therapy, com es from peers as w ell as from the
lead er. Feed back from p eers in the grou p can
8. (A) This is an exam p le of free association. be a p ow erfu l stim u lu s for insight and change.
Freu d believed that all m ental activity is cau s- Free association is u sed in p sychoanalytically
ally related , m eaning that free associations are oriented grou ps. These grou ps are largely
not truly rand om . They follow logically, bu t ou tpatient grou ps w hose m em bers can toler-
the m issing connection is being held d ow n or ate and effectively u se this techniqu e. Mem -
rep ressed u nconsciou sly by d efenses. H yp no- bers are encou raged to be sp ontaneou s w ith
sis is w here a p atient is placed in a trance and their thou ghts and feelings, and the therap ist
134 4: P syc holog ica l Tre a tme nt a nd Ma nag e me nt

assists in p ointing ou t com m on them es and Abreaction is w hen a p atient relives an em o-


p ossible connections. Go-arou nd is a com m on tional or traum atic experience to get rid of or
techniqu e w here each grou p m em ber is asked p u rge negative em otions and m ove forw ard .
to introd u ce them selves, state how they are Dep end ency is w here one need s another p er-
feeling, or otherw ise specifically respond ; this son or ad d ictive su bstance in ord er to m aintain
gives each ind ivid u al a chance to p articip ate. a level of em otional or p hysical fu nctioning.
Resistance is the u nconsciou s op position to Projection is a prim itive d efense consisting of
fu ll d isclosu re of thou ghts or feelings. attribu ting ones ow n und esirable thou ghts,
d esires, or em otions onto som eone or som e-
11. (C) Interp retation is u sing a hyp othesis to thing else. Sp litting is a d efense m echanism
exp lain the p sychological m eaning of a behav- w here one view s them selves and / or others as
ior or sym p tom s. Cou ntertransference is w hen all good or all bad . It is com m only seen in
the therap ists feelings and em otions are p atients w ith bord erline personality d isord er.
d irected tow ard a p atient. Em p athy is w hen
a therap ist id entifies and u nd erstand s a 14. (B) This ind ivid u al has featu res of narcissistic
p atients situ ation or feelings. Parad oxi- p ersonality d isord er. Althou gh p atients w ith
cal intervention is a therap eu tic techniqu e narcissistic p ersonality d isord er have d iffi-
w here a therap ist instru cts a p atient to hold culty w ith confrontation, it is necessary to help
onto a sym p tom in ord er to get the p atient them w ork tow ard change; therefore, group
to u nd erstand that the sym p tom is not help - therapy is a good setting for this to take p lace.
fu l. Working throu gh refers to thinking abou t In ad d ition, in ind ivid u al therap y p atients
and d iscu ssing a p roblem in ord er to lessen can process these situ ations and exam ine how
its intensity or im p act. p ossible insu lts to the ego can be d ealt w ith, as
w ell as ad d ressing the other ind ivid u al issu es.
12. (D ) Intervention is an orchestrated attem p t Anger m anagem ent as w ell as social skills
by one or m any to get an ind ivid u al to accep t training can be helpfu l, but are not enough
p rofessional help or treatm ent, u su ally for an to exam ine the d eep u nd erlying issu es and
ad d iction or crisis. Behavioral therap y focu ses d efenses that patients w ith narcissistic p erson-
on changing a patients m alad aptive behaviors ality d isord er have. Sim ilarly, p sychoanalysis
or actions; its pu rpose is not to get a patient to alone w ou ld not be enou gh to help patients in
com m it to treatm ent. Fam ily therap y involves w hich m any of their relationship s and social
an entire fam ily in therapy together w ork- interactions are affected by their p ersonality
ing to change the d ynam ics, d evelop m ent, d isord er. Pharm acotherapy is often not help -
and system s of interaction; fam ily therap y fu l w ith personality d isord ers unless severe
d oes not usu ally focu s on the problem s of im p u lsivity or p sychosis is p resent, or u nless
one m em ber. Grou p therapy is w hen a group a p atient has a com orbid p sychiatric d isord er
of ind ivid uals w ork on issu es in conju nction (e.g., m ajor d ep ressive d isord er).
w ith each other; it can be help fu l becau se the
d ifferent backgrou nd s and problem s can be 15. (E) Progressive m u scle relaxation is a relax-
looked at together in ord er to help each p er- ation technique u sed in CBT to control and
son w ith their ow n d ifficu lties. Relapse pre- m inim ize m u scle tension. Of the op tions, it is
vention is u sed in ad d ictions treatm ent, w here the only one d irectly targeted at the p atients
ind ivid u als are tau ght self-control and how to u ncom fortable feelings w hile arou nd other
m aintain changes in behavior, as w ell as how p eop le. Cognitive restru ctu ring and keep ing
to anticipate and cope w ith relap se. a thou ght record are techniqu es w hich m ay
im p rove the sym p tom s by id entifying and
13. (D ) This p atient is d em onstrating the concep t m inim izing au tom atic thou ghts. Exp osu re
of reaction form ation, w hich is w hen u nac- therapy is a techniqu e used to d ecrease the
ceptable or anxiety-provoking em otions are avoid ance behaviors by exp osing p atients to
d ealt w ith by acting in the opposite m anner. the feared stim u li in ord er to show they can
Answe rs : 1119 135

tolerate it. It is com m only u sed in treating have the best ou tcom e in any form of therap y,
anxiety d isord ers. A m ood d iary is a techniqu e there m u st be a good relationship or alliance
u sed for m onitoring ones m ood to d eterm ine betw een the p atient and therap ist. Choice (A)
triggers, au tom atic thou ghts, and treatm ent m ay also be harm fu l as a p atient shou ld not
p rogression. be called silly. Choice (B) is am biguou sto
w hich p art of the qu estion are you resp ond -
16. (A) In this case, it w ou ld be im p ortant for ing? The p atient m ay believe that you are
the therap ist to review the fram e for ther- insu lting him rather than attem p ting to be
ap y. In p sychod ynam ic p sychotherap y the sup portive.
fram e refers to the bou nd aries and ru les
d efining the w ay the therap ist and p atient 18. (E) Refram ing is a techniqu e com m only u sed
interact (e.g., the m eeting tim e, p lace, length, in cognitivebehavioral therap y. First, a ther-
other d etails d ow n to w here the therap ist and ap ist w ill exam ine how p atients thou ghts or
p atient u su ally sit, how fees for service are beliefs abou t them selves or the w orld m ay
collected , w hat hap p ens w hen a p atient starts be skew ed . Then the thou ghts or beliefs are
m issing ap p ointm ents). The fram e is im p or- refram ed by com p aring them to a m ore nor-
tant as it gives p sychotherap y a stru ctu re m alized p ersp ective. Clarification is w hen
and sets u p a sp ecific d ynam ic betw een the a therap ist asks for fu rther d etails abou t a
p atient and therap ist. Therefore, if bou nd ar- p atients exp erience or situ ation they are
ies or ru les are challenged the therap ist can d escribing. Confrontation is w hen a thera-
u se this to better u nd erstand the reasons as p ist d irectly asks p atients abou t their m al-
w ell as d eterm ine any transference issu es at ad ap tive behaviors or sym p tom s. Em p athy
p lay. In this case the therap ist su sp ects the is w hen a therap ist show s u nd erstand ing or
p atient m ay not u nd erstand the fram e u sed agrees w ith p atients, show ing them that they
for her therap y sessions and w hy it is im p or- are able to p u t them selves in the p atients
tant. Review ing the therap eu tic goals and shoes or see their p ersp ective. Interp retation
the typ e of therap y p rovid ed m ay both be is w hen a therap ist p resents the p atient w ith
reasonable top ics for the therap ist to d iscu ss; a hyp othesis regard ing the reason(s) (often
how ever, it w ou ld not ad d ress her reasons for u nconsciou s) a p atient has certain behaviors
being late and not show ing u p for ap p oint- or sym p tom s.
m ents. Interp reting the p atients behavior as a
need for attention m ay not only be incorrect, 19. (B) Behavior therap y is focu sed on behaviors,
bu t it m ay be exp erienced by the p atient as an not on p ossible und erlying factors that cause
attack or hu rtfu l, p articu larly if she d oes not the behaviors. In this case, ad d ressing the
u nd erstand the therap eu tic fram e correctly. negative or m alad ap tive behavior (e.g., stop -
Sim ilarly, looking at the ind ivid u al reasons p ing by m ovie theaters to bu y cand y) w ould
w hy she is late avoid s the su bject of w hy this be ad d ressed . Behavior therap y is a here-
p attern of behavior is p resent. and -now ap p roach to m alad ap tive behaviors.
It m ay u tilize inform ation su rrou nd ing the
17. (C) One reason this is the best resp onse is that em ergence of a p articu lar m alad ap tive behav-
it is a qu estion. Patients w ith cognitive d istor- ior in ord er to constru ct a behavior m od ifi-
tions often take their view of the w orld as a cation p lan. Althou gh behavior therapy can
given. They d o not qu estion their p ersp ectives. and d oes change neu robiology, this is not
Asking a qu estion invites p atients to exam ine its exp ressed goal. It d oes not focu s on the
their perspectives and thereby becom e open p atients relationship s or seeking the sou rces
to the p ossibility that their thoughts m ay be for the p atients negative eating habits. This
inaccu rate. Again, gaining clarity is alw ays a is also w hy child hood issu es su ch as trau m a
good first step in com m u nication. Choices (D ) are not typ ically ad d ressed in behavioral
and (E) are also qu estions bu t are au thoritar- therapy. Resolving und erlying conflicts in an
ian and p u nitive, not to m ention insu lting; to attem p t to change behavior is a d escrip tion of
136 4: P syc holog ica l Tre a tme nt a nd Ma nag e me nt

the goals of psychod ynam ic p sychotherapy or are id entified in the cou rse of therapy. These
p sychoanalysis. com m u nication p atterns are then altered ,
for instance, by role playing or asking the
20. (C) In this case, exp osu re and resp onse train - p atient to try ou t d ifferent form s of com m u -
ing w ou ld be the m ost help fu l in d ecreas- nication d u ring the session. Character change,
ing the p atients m alad ap tive behavior of interp reting transference, and p ointing ou t
stop p ing at m ovie th eaters. It w ou ld teach resistance are m ore the focu s of p sychod y-
m ore ad ap tive behaviors throu gh exp osin g nam ically oriented p sychotherap y. Strength-
him to situ ations w h ere h e cou ld p ass by a ening of d efenses is the goal of su p p ortive
m ovie th eater an d en gage in an alternative p sychotherap y.
behavior. Aversion therap y is w hen a nega-
tive (p ainfu l) stim u lu s is p aired w ith a m al- 23. (E) Interp ersonal p sychotherap y focu ses on
ad ap tive behavior in ord er to extin gu ish the the patients role and com m unication style
behavior; it is not com m only u sed and often w ithin im p ortant relationship s and how his
the beh avior retu rn s over tim e. In this case, or her position m ay be cau sing or com p li-
en vironm en tal m od ification w ou ld be less cating the illness (e.g., d epression). If one or
help fu l as the p atient w ill likely encou n ter tw o specific w ays of com m u nicating can be
situ ation s w here he m ay p ass m ovie the- altered so that a relationship im p roves, the
aters in th e fu tu re; in ad d ition , changing th e d ep ressive illness also im proves. This ther-
en vironm en t d oes n ot ad d ress the behav- ap y also has the ad d ed benefit of d u rability;
ior. Mod eling is w here ad ap tive behaviors once a com m unication pattern is im p roved
are tau ght by d em onstration, w hich w ou ld and the p atient better u nd erstand s how his or
be d ifficu lt to teach in this p articu lar case. her role in a relationship m ay create d istress,
Relaxation train ing is u sed to help a p atient the d epressive illness is not as likely to recu r.
to han d le an xiety-p rovoking situ ations, not Variou s techniqu es u sed in interpersonal
chan ge an u nh ealthy beh avior. p sychotherap y are role p laying, su p portive
listening, clarification, and com m u nication
21. (B) Biofeed back and stress red u ction are u sed analysis. Interpreting d ream s, analyzing p sy-
to red u ce the frequency and severity of ten- chic d efenses, and free association are used
sion head aches. They are equ ally effective in in m ore p sychod ynam ically based therap ies,
accom plishing this goal. Su bstance w ithd raw al such as psychoanalysis. H ypnosis is a sleep-
synd rom es are exam ples of aversive stim uli, like state ind u ced by a therap ist, w hich has
and the resu ltant negative reinforcem ent theo- been of som e u se in behavioral m od ification
retically lead s to a d ecrease in su bstance u se. such as sm oking cessation. It is not a goal or
Stim u lus control is u sed in the treatm ent of com ponent of interpersonal p sychotherapy.
insom nia: The negative association betw een
m u ltip le failed attem p ts at falling asleep and 24. (B) Psychoanalysis tend s to be am ong the lon-
the specific environm ent w here they occurred gest of therap ies and involves the p rocess of
is elim inated by red u cing d istractions and free association, in w hich the patient lies on
creating an environm ent m ore cond ucive to a couch, facing aw ay from the therap ist, and
sleep (e.g., taking televisions and com pu ters says w hatever com es to m ind . It takes four to
ou t of the bed room ), as w ell as teaching good five sessions a w eek to su ccessfully com p lete
sleep hygiene. System atic d esensitization is a an analysis becau se it is d ifficu lt for patients
behavioral techniqu e w hich can be u sed in to bu ild , su stain, and w ork through their
asthma, where the patient is exposed to provok- transference and d efensive stru ctu re; less fre-
ing stimuli in a progressive manner. qu ent sessions d ecrease the form ation of a
transference neurosis and increase d efenses.
22. (B) The m ajor focus of interpersonal psycho- Psychoanalysis is based on an exp ressive or
therapy is comm unication analysis. Ind irect, exp loratory m od el of therap y, as op p osed to
unclear interpersonal commu nication patterns a su p portive m od el aim ed at stabilizing crisis
Answe rs : 2030 137

and reestablishing a baseline level of fu nction. 27. (B) While a skilled hyp notist is very im por-
H om ew ork is a m ethod u sed on cognitive- tant, far and aw ay the m ost im portant factor in
behavioral therap y and not p sychoanalysis. hyp nosis is a p atient w ho is highly resp onsive
In p sychoanalysis, conflicts betw een therap ist to suggestion, as ind icated by the patients
and patient are analyzed and d iscussed as they prior p ositive exp erience w ith hyp nosis and
often represent a recreation of child hood con- belief that it w ill be help fu l. Som e ind ivid u als
flicts and relationship s. There is no correlation are able to alter their su bjective experience in
betw een severity of m ental illness and length resp onse to su ggestion m ore than others; som e
of therapy. Patients w ho are in psychoanalysis patients can be tau ght self-hypnosis. N one of
are not m ore likely to be severely ill and w ill the other characteristics is as im p ortant in the
need a certain level of insight and com petence su ccess of hypnosis.
to gain the benefits of p sychoanalysis w ithout
encou ntering severe p roblem s. 28. (A) There are nu m erou s ind icators of a trance
state, su ch as am nesia, anesthesia, and red u ced
25. (C) The m ajor tool of p sychoanalysis is the p ulse, reflexes, and respiration. Other changes
carefu l interp retations of the transference inclu d e changed voice qu ality, eye closu re,
neu rosis. Altering cognitive d istortions is lack of bod y m ovem ent, p u p illary changes,
the m ain goal of cognitive therap y. While red u ced blinking, and tim e d istortion.
psychoanalysis m ay accom p lish m any tasks,
inclu d ing changing m alad aptive thou ghts 29. (D ) Fam ily therap y focu ses on the relation-
and behaviors and solving interp ersonal ships betw een m em bers and how to m ake
problem s, these are the consequ ences of anal- them m ore ad aptive to the situations. In this
ysis, not the instru m ents of change. Althou gh case, fam ily therapy w ill help the p arents
interpreting d ream s can lead to insights into a and child ren u nd erstand w hy there are con-
patients u nconsciou s thou ghts and feelings, it flicts betw een them and w ork w ith the fam ily
is an ad ju nctive tool. as a u nit; the p u rp ose is neither to blam e nor
change the brothers behavior. Fam ily therapy
26. (D ) In this case, the therap ist is having an inap - d oes not stress assigning roles to each m em -
p rop riate attraction to his p atient and shou ld ber. Unlike p sychoanalysis or p sychod ynam ic
seek su pervision in ord er to u nd erstand the therapy, fam ily therapy d oes not focu s on any
reasons for his (and the p atients) sexu al feel- ind ivid u als early child hood exp eriences or
ings. In ad d ition, a su p ervisor can help to traum a, u nconsciou s conflicts, or im pu lses
m ake sure that the transference or counter- that led to the fam ily m em bers pathology.
transference is not significantly interfering in
the therapy. N ot ad d ressing the situ ation, or 30. (D ) This p atient is in the Erikson stage of inti-
acting less em pathic or cold w ould be evasive m acy versus isolation w hich occu rs in you ng
and confu sing to the p atient, m aking it d iffi- ad u lts from the ap p roxim ate age of 20 to
cult for the patient to m ove forw ard in ther- 40 years. This is the p eriod w hen ad u lts are
ap y. The therap ist shou ld not try to elicit m ore struggling w ith becom ing em otionally close
behaviors or continu e to d eep en any erotic and physically intim ate w ith another versu s
transference that cou ld lead to a greater pow er feelings of loneliness. Tru st versu s m istru st
d ifferential or m ore sed uctive behavior; this occu rs from birth to 1 year w hen infants learn
cou ld p ut the relationship in jeopard y or cross w hether or not p rim ary caregivers can satisfy
bou nd aries inap p rop riately. Althou gh the basic need s. The au tonom y versu s sham e/
therapist can fire or term inate the patient, the d ou bt stage occu rs in tod d lers from ages 1 to
situ ation m ay not be seriou s enou gh to w ar- 3 years w hen child ren attem pt to m aster tasks
rant su ch severe actions; it m ay also send the on their ow n (as w ell as their ow n im pu lses),
m essage to the p atient that her sexu al feelings and , if su p p orted (as op p osed to p u nished ),
are d angerous and that the therap y w as not a grow confid ent and secu re in their ability to
safe place to d iscuss them . su rvive. Initiative versus gu ilt occu rs from age
138 4: P syc holog ica l Tre a tme nt a nd Ma nag e me nt

3 to 5 years w here p reschool child ren gain a interp reted in so far as they relate to the id en-
sense of w ell-being from d isplaying initiative tified focal conflict.
w ithou t feeling gu ilty. The id entity versu s
role confu sion stage occu rs d u ring ad oles- 35. (D ) Psychoanalytic p sychotherap y or psy-
cence from ages 11 to 19 years w hen teenagers choanalysis is not generally ind icated in
attem p t to d eterm ine w ho they are and w hat p atients w ho have d ifficu lty w ith reality
they w ish to accom plish in life. testing, such as those w ith schizophrenia.
The inevitable regression, introsp ection, and
31. (C) Parent m anagem ent training is a type of self-exam ination required in these intense,
therapy that focu ses on giving parents tools to insight-orientated p sychotherap ists are u su -
m anage p roblem behaviors in child ren w ith ally overw helm ing to p sychotic p atients and
d isrup tive behaviors. The goal is to increase actu ally m ay cau se m ore harm than good ,
the u se of positive reinforcem ent to increase w ith a resu ltant w orsening of p sychosis. All
d esired behaviors at hom e and school. This is of the other types of psychotherapies can be
typically d one w ith behavior charts and earn- helpfu l in the treatm ent of schizophrenia or
ing p rivileges. The other choices apply m ore other psychotic d isord ers.
to other types of fam ily therapy.
36. (B) Dream s p rovid e inform ation abou t p sychic
32. (A) A behavioral tool th at is u sed qu ite effec- conflicts. In this case, d ream interpretation
tively is a beh avior ch art. It can serve as a m ay help the p atient learn m ore abou t his cu r-
p ositive incen tive to good beh avior w h en rent u nhap p iness and conflict in his m arried
th e rep ort is p ositive and is then rew ard ed . life. Dream s are u sed to u ncover p sychic con-
Th e rew ard s can be ad ju sted for th e d egree flicts, w hich, w hen interp reted app ropriately
of p ositive beh avior obtain ed . N either aver- and consciou sly (not su blim inally), can lead to
sion therap y nor p u n ish m en t is help fu l for a p rod u ctive change in behavior. Accord ing to
teachin g p atien ts w ith ADH D n ew , m ore p sychoanalytic theory, d ream s rep resent the
constru ctive behaviors. In terp retive strate- u nconsciou s w orkings of the m ind , not the
gies su ch as a w ell-tim ed in terp retation of conscious ones, and may be symbolic in con-
m alad ap tive beh aviors or p ositively refram - tent. Interpreting d reams may eventually lead
in g a n egative exp erien ce are n ot beh avioral to an alteration in a patients cond uct, but
tools. reducing cognitive errors or changing behavior
is not its primary purpose.
33. (D ) Motivation for change goes beyond a
d esire for rem oval of sym ptom s and inclu d es 37. (B) Dialectical behavioral therap y (DBT) w as
a w illingness to tolerate d iscom fort and to d evelop ed to treat bord erline personality d is-
take risks in a search for u nd erstand ing. The ord er. It is often p erform ed in both grou p and
other d em ographics d o not play as significant ind ivid u al settings to m inim ize the intense
a role. transferences often d eveloped by patients
w ith bord erline p ersonality d isord er in ind i-
34. (B) In brief p sychod ynam ic therap y a single, vid ual therapy. The focu s is on teaching skills
focal area of conflict is id entified , and d u ring to m inimize m ood d ysregulation and self-
the therapy the tim e is sp ent actively interpret- harm. Distress tolerance focuses on d istracting
ing transferences as they p ertain to this id en- from and tolerating negative em otions. Cog-
tified focal conflict. Many types of therapy nitive restru cturing is a concept from cogni-
correct cognitive errors and mod ify malad ap- tive therap y that is som etim es em ployed in
tive behaviors, although cognitivebehavioral DBT. It refers to recognizing and changing
therapies are specifically concerned w ith these m alad ap tive thou ghts. Em otion regu lation
ou tcom es. Interpretations of d ream s and resis- focu ses on recognizing and regulating m ood
tance m ay certainly take place in brief psy- changes. Interp ersonal effectiveness relates to
chod ynam ic therapy; how ever, they are only interp ersonal skills and assertiveness training.
Answe rs : 3143 139

Mind fulness is based on trad itional Bu d d hist is the next stage w here m em bers w ork hard
p ractice of being in the m om ent w ithou t to achieve goals. Ad journing is the final stage
ju d gem ents. w here the w ork is com p lete and / or the grou p
end s.
38. (D ) Patients w ho are su icid al characteristi-
cally have severe hopelessness and think 41. (E) System atic d esensitization is the process
abou t events in rigid , black-and -w hite w ays. of exposing a patient to feared objects or situ -
They see very few op tions and becom e over- ations and teaching them to d ecrease their
w helm ed w ith their feelings of hop elessness anxiety and fear in ord er to overcom e them .
w hich are exacerbated by seeing no w ay System atic d esensitization is som etim es called
ou t. Begging the qu estion and circu lar rea- grad u ated exp osu re therap y and can be u sed
soning are both logical fallacies. They occur w ith m any typ es of anxiety d isord ers, inclu d -
w hen an id ea is p roven to be tru e w hen ing social anxiety d isord er (p hobia) and sp e-
assu m ed im p licitly or exp licitly in the p rem - cific phobias. Aversion therapy is w hen a
ise (e.g., only w eak p eop le p u t them selves in stim ulu s is paired w ith som ething negative
m y position, therefore I am a w eak p erson). or d iscom forting in ord er to get a patient to
Personalization is ones tend ency to relate avoid or stop a certain m alad aptive behavior.
events to oneself w ithou t any reason for d oing Flood ing is w hen a p atient is exp osed to the
so. Selective abstraction is term from cognitive m ost feared situ ation all at once. Mod eling is
therapy w here d etails of a scenario are taken w hen observed behaviors are im itated . Su g-
ou t of context and believed w hile everything gestion is w hen an id ea or behavioral change
else contrary abou t the scenario is ignored . is consciou sly su ggested to a p atient in ord er
to change the behavior.
39. (B) Cognitive therap y is the p sychotherap y
that w ould be m ost ap prop riate to help this 42. (E) Su pportive psychotherap y attem p ts to
p articu lar p atient solve problem s m ore p ro- fortify p sychological d efenses by provid ing
d uctively. Behavioral therapy is focu sed on em p athic reassu rance as op p osed to p rob-
behaviors, not on the p ossible u nd erlying ing into a p atients p sychological conflicts.
factors that cause behaviors. H ypnosis is a Behavioral therap ies u tilize op erant cond i-
sleep-like state ind uced by a therapist, w hich tioning, relaxation training, and exposure
has been of som e u se in u ncovering psychod y- techniqu es (am ong others) in ord er to change
nam ic processes and in behavioral m od ifica- d ysfunctional behavior, m ostly in the treat-
tion such as stopp ing sm oking. Interp ersonal m ent of anxiety d isord ers. Cognitive therapy
therapy is aim ed at und erstand ing com m uni- is often com bined w ith behavioral therap y; it
cation patterns and roles in relationship s. In w as d evelop ed by Aaron Beck and is u sed for
p arad oxical therap y, d evelop ed by Bateson, d ep ressive and anxiety d isord ers. It is based
the therapist su ggests that the patient engage on the belief that the w ay that p atients think
in the behavior w ith negative connotations and act are d ue to cognitive schem as w hich
(e.g., a phobia or com pulsion). d erive from child hood exp eriences and tem -
p eram ental factors. Dynam ic psychotherapy
40. (B) Storm ing is the second stage of Tu ckm an d elves into cu rrent and p ast conflicts in ord er
stages of group d evelopm ent w hich includ es to bring unconscious m aterial into conscious-
p ow er stru ggle and bou nd ary testing; grou p ness. Existential psychotherap y is based on
m em bers m ay have internal conflict w ith each existential p hilosop hy and em p hasizes p er-
other or rebel against the facilitator. Form - sonal feeling and exp erience over rational
ing is the first stage in w hich grou p m em - thinking.
bers are oriented to the grou p and roles are
form ed . N orm ing is the third stage in w hich 43. (A) Biofeed back is the p rocess w hereby a
grou p m em bers get to know each other bet- p atient receives inform ation on physiologic
ter and resolve their d ifferences. Perform ing variables in ord er to alter those variables and
140 4: P sycholog ica l Tre a tme nt a nd Ma nag e me nt

u ltim ately the corresp ond ing behavior. For tion that the other ind ivid ual is ind u ced to
exam p le, p atients w ith m igraine head aches behave in su ch a w ay as to confirm the p rojec-
obtain inform ation on m u scle tension or tem - tions. The real relationship (J) is consid ered to
p erature. They w ill then use this inform ation be the relationship betw een a p sychoanalyst
to d ecrease the intensity of the head aches by and p atient w hich is not influ enced by trans-
d irectly trying to relax and alter the know n ference. A schem a (K) refers to an organized
p aram eters. p attern of thou ghts or behaviors relating to
oneself, others, and the environm ent arou nd .
44. (N ) Transference (and transference neu ro- Splitting (L) is another d efense m echanism
sis) can be und erstood as a general phenom - characterized by view ing feelings, others,
enon in w hich the p atient d isp laces onto and situ ations in all good or all bad term s.
others (inclu d ing the therapist) the feelings, Therap eu tic alliance (M) refers to the healthy
thoughts, and conflicts from the patients past relationship betw een a p hysician/ therap ist
(p articu larly early child hood ). and p atient. Und oing (O) is a d efense m echa-
nism w here a p atient attem pts to u nd o an
45. (C) Flood ing is a techniqu e u sed in behavior u nw anted thou ght or action (e.g., an obses-
therapy w here a patient is exp osed to a feared sion) by engaging in contrary behavior (e.g., a
or anxiety-provoking stim u lus all at once to com pu lsion).
help him or her overcom e the fear or anxiety.
It is u sed m ost frequ ently in sp ecific p hobias 48. (B) Jean-Martin Charcot is known for his contri-
as in this case. butions to neurology, including the description
of many medical diseases including Charcot
46. (F) Positive reinforcement is a behavioral tech- Marie tooth disease, amyotrophic lateral scle-
nique w here a rew ard (a sw eet) is given after a rosis (ALS), Charcot joint, Charcot aneurysms,
specific behavior (saying thank you) in ord er and Charcot triad. He was also known for his
to increase the frequency of that behavior. descriptive work on hysteria and other neuroses.

47. (I) Punishm ent is presenting an aversive 49. (A) Josef Breu er is know n for his u se of the
stim ulu s (soap in the m ou th) after a behavior cathartic treatm ent or talking therap y in the
(biting) in ord er to d ecrease the frequency of fam ous case of Anna O, w hich w as su bse-
that behavior. Countertransference (B) is the qu ently follow ed closely by Sigm u nd Freu d .
com pilation of a therapists feelings tow ard
a patient. H ypnosis (D ) is a sleep -like state 50. (E) H einz Kohu t is m ost w ell-know n for his
ind u ced by a therap ist, w hich has been of d evelop m ent of self-psychology. It w as a
som e u se in u ncovering u nconsciou s pro- very influ ential school of thou ght w ithin
cesses and in behavioral m od ification su ch as p sychoanalysis and help ed d irect m od ern
sm oking cessation. N egative reinforcem ent ap p roaches to therap y, p articu larly concern-
(E) is an operant cond itioning techniqu e w here ing the treatm ent of narcissism . Erik Erikson
an aversive stim u lu s is rem oved in ord er to (C) w as a p sychoanalyst w ho p osited eight
increase the frequ ency of a d esired behavior. stages of d evelopm ent, from infancy to old
For exam p le, p u tting on you r seat belt so the age. Sigm u nd Freu d (D ) w as the fou nd er of
car w arning stops beeping. Projection (G) is a p sychoanalysis, best know n for his theories of
d efense m echanism w here u nw anted feelings, the u nconsciou s m ind . Franz Anton Mesm er
attitu d es, and thou ghts are u nconsciou sly (F) is consid ered the father of m od ern hyp no-
attribu ted to others. Projective id entification sis; the w ord m esm erism is d erived from his
(H) is sim ilar to p rojection, bu t w ith the ad d i- nam e.
CHAPTER 5

Le g al and Ethic al Is s ue s in
Ps yc hiatry and Me dic ine
Que s tions

DIRECTIONS (Questions 1 through 51): For each of (E) Resp ect the p atients confid entiality
the multiple-choice questions in this section, select and w ait u ntil you r next sched u led
the lettered answ er that is the one best response in ap p ointm ent w ith him to d iscuss his
each case. feelings.

2. The legal p reced ent that gu id es the ap p rop ri-


Questions 1 and 2
ate cou rse of action in this case is w hich of the
The case m anager for a 35-year-old m an that you are follow ing?
treating contacts you to d iscuss concerns regard ing (A) Durham v. United States
a change in your p atients behaviors. You learn from
(B) OConnor v. Donaldson
the case m anager that the patient has been m ak-
ing p rovocative sexu al com m ents tow ard her over (C) Rogers v. Commissioner of the Department
the past m onth. Du ring your session, the p atient of M ental Health
exp resses concern that his case m anager is som ehow (D) Rouse v. Cameron
conspiring against him and he plans to d o som e- (E) Tarasoff v. Regents of University of California
thing abou t it. On fu rther qu estioning, he becom es
increasingly irritable and abru p tly storm s ou t of Questions 3 through 5
you r office. You hear him in the hallw ay exclaim ing,
That w om an, Im going to stab her and she w ont Tw o years ago, you w ere the anesthesiologist
bother m e anym ore! involved in a cesarean d elivery of a baby born w ith
cerebral palsy. You had heard from your obstetrical
1. You r first cou rse of action shou ld be w hich of colleagu e that the fam ily w as planning to sue, and
the follow ing? tod ay a process server d elivers papers notifying you
that the fam ily has brought an action against the
(A) Attem p t to contact the p atient over the p hysicians involved in the su rgery, inclu d ing you.
next several hou rs to d iscu ss the intent
of his threat tow ard his case m anager. 3. In m ed ical m alp ractice cases, the p laintiff
(B) Contact the p atients fam ily and let m ust establish w hich of the follow ing?
them know abou t his threats tow ard the
(A) Burd en of p roof beyond a reasonable
case m anager.
d ou bt
(C) Inform the p olice of the content of you r
(B) Clear and convincing evid ence of
p reviou s sessions and his recent threat
w rongd oing
tow ard the case m anager.
(C) Crim inal intent
(D) N otify the case m anager of the p otential
d anger. (D) Crim inal m ischief
(E) H arm or d am age

141
142 5: Le ga l a nd Ethica l Is s ue s in P sychia try a nd Me d ic ine

4. Which of the follow ing behaviors best p rotects 7. The p atients fam ily hires an attorney to chal-
a p hysician against a m alp ractice su it? lenge the valid ity of her w ill. Which of the
follow ing item s, if p resent, w ou ld u nd erm ine
(A) H aving the patient sign a no harm
this patients com petence to m ake a w ill?
contract.
(B) H iring a m alp ractice attorney to (A) Inability to read and w rite.
exam ine the p hysicians treatm ent (B) Paranoid d elu sions regard ing the
p ractices. p atients fam ily.
(C) Maintaining a good therap eu tic alliance (C) Patients nond elu sional exp lanation of
w ith the p atient. w hy she w ants to d onate her estate to
(D) Refusing to prescribe m ed ications that the local hum ane society.
m ay have ad verse effects. (D) Presence of a d iagnosable p sychiatric
(E) Seeing the p atient in clinic on a w eekly d isord er.
basis. (E) Refu sal to u nd ergo treatm ent
recom m end ed by her p rim ary care
5. When d eterm ining m ed ical m alpractice, the p hysician.
term stand ard of care refers to the use of
m ed ical and p sychiatric treatm ents that are: Questions 8 and 9
(A) End orsed by the Am erican Med ical A 23-year-old m an w ith no p rior p sychiatric history
Association is charged w ith m u rd ering his next-d oor neighbor.
(B) Evid ence based H is friend s note that he becam e increasingly iso-
(C) Safe and free from any p otential lative and su sp iciou s of others in the w eeks p rior
negative sid e effects to the crim e. The exam ining psychiatrist reported
(D) The m ost cu rrent treatm ents available that the m an harbored paranoid d elu sions regard -
ing his neighbor and noted that his thou ght p rocess
(E) Used by average, reasonable
w as too d isorganized to com p lete the exam ination.
p ractitioners in sim ilar circu m stances
8. Which of the follow ing legal stand ard s is
Questions 6 and 7
the cu rrent basis for establishing an insanity
An 80-year-old w id ow w ith a h istory sign ificant d efense?
for schizop hrenia w as recently d iagnosed w ith (A) Am erican Law Institu te test (Mod el
end -stage hep atic cancer. She is concerned abou t Penal Cod e)
the d isp osition of her estate and d oes not w ant
(B) Du rham ru le
her fam ily to receive any of h er m on ey. Sh e p lans
to d onate her entire estate to the local hu m ane (C) Irresistible im p u lse ru le
society. (D) MN aghten ru le
(E) Prod u ct ru le
6. Which of the follow ing is m ost im p ortant in
establishing a legally valid w ill? 9. Crim inal resp onsibility requ ires d em onstra-
tion of the crim inal act along w ith w hich of the
(A) Conservatorship of estate
follow ing elem ents?
(B) Inform ed consent
(C) Lack of m ental illness (A) Dim inished m ental cap acity
(D) Testam entary cap acity (B) Mens rea
(E) Witnesses at the signing of a w ill (C) Mod u s op erand i
(D) Psychiatric d isord er
(E) Witness to the act
Que s tions : 413 143

Questions 10 and 11 12. A 19-year-old m an w ith a history significant


for bip olar I d isord er is charged w ith assau lt-
An obese 54-year-old w oman presents via ambu- ing a p olice officer. This is the m ans fifth arrest
lance to the emergency d epartment complaining of in the p ast 4 m onths for aggression tow ard an
severe substernal chest pain lasting 40 minutes, pro- au thority figu re. The assigned attorney has
fuse sw eating, and nausea. Her vital signs are blood inform ed the forensic p sychiatrist involved
pressure 195/ 96 mm H g, heart rate 63 beats/ min, in the case that the p atient has been nonco-
respiratory rate 18 breaths/ min, temperature 98.8F. operative and belligerent w hile attem p ting
An electrocardiogram reveals 3-mm ST-segment ele- to prepare for his d efense. The attorney also
vations in lead s V4, V5, and V6. Cardiac enzymes and questions w hether the patient has an und er-
laboratory w orkup are pend ing. You suspect a lateral stand ing of the severity of his crim e given his
w all myocardial infarction and recommend imme- acu te m ental state and lack of form al ed uca-
d iate thrombolytic therapy. The patient exclaims, tion. Which of the follow ing item s w ou ld be
Youre crazy if you think Im going let some intern consistent w ith the assertion that the patient is
care for me! My family w ill d rive me across tow n to incom p etent to stand trial?
the private hospital! The patient then jumps from
the gurney and begins w alking tow ard the exit. (A) Failu re to com p rehend the crim inal
charges.
10. Which of the follow ing is the m ost ap prop riate (B) H istory of p rior assau lt charges.
next step?
(C) Inability to p rovid e inform ed consent.
(A) Ad m it the patient to the card iology (D) Inability to read and w rite.
service on a physician em ergency (E) Presence of a d iagnosable m ental
certificate. illness.
(B) Allow the p atient to leave against
m ed ical ad vice. 13. A 35-year-old m an is brou ght to the em er-
(C) Detain the p atient u ntil the resu lts of the gency room by his fam ily for threatening to
card iac enzym es are available. shoot his m other in the chest. Urine toxicology
(D) Detain the p atient u ntil you can assess is negative for cocaine, p hencyclid ine (PCP),
her ability to p rovid e inform ed consent. or op ioid s, and his blood alcohol level is 0.04.
(E) Sed ate the p atient and begin You learn from the p atients fam ily that for
throm bolytic therapy. several d ays he has been threatening to kill his
m other and has even w ritten notes about how
11. On further evalu ation, the patient d em on- he w ou ld execu te his p lan. On m ental statu s
strates a thorou gh u nd erstand ing of the infor- exam ination (MSE), the p atients sp eech is
m ation you have given to her and ap p reciates lou d , p ressu red , and threatening tow ard the
the consequences of not being treated im m e- em ergency d ep artm ent staff. Fam ily history
d iately. The ap propriate next step is w hich of is significant for a bip olar sp ectru m d isor-
the follow ing? d er. The patient states that he has not d one
anything w rong and d em and s to be released
(A) Ad m it the p atient to the card iology im m ed iately. Which of the follow ing next
service on a physician em ergency steps w ou ld be the m ost appropriate?
certificate.
(B) Allow the p atient to leave against (A) Arrange for a p olice hold given the
m ed ical ad vice. p atients p otential for violence.
(C) Detain the p atient u ntil the resu lts of the (B) Involu ntarily ad m it the p atient to an
card iac enzym es are available. inp atient p sychiatric w ard .
(D) Detain the p atient so that her fam ily can (C) Med icate the p atient w ith a m ood
m ake d ecisions on her behalf. stabilizer and arrange for outpatient
follow -u p .
(E) Sed ate the p atient and begin
throm bolytic therap y.
144 5: Le ga l a nd Ethica l Is s ue s in P sychia try a nd Me d ic ine

(D) N otify the police abou t the patients (C) Confid entiality
hom icid al threats. (D) Ju stice
(E) Warn the patients m other about the (E) N onm aleficence
p atients threats.
16. A 36-year-old woman with delusional disord er
Questions 14 and 15 is brou ght to the em ergency room by p olice
You are the psychiatrist on d uty w hen a man, arrested after she w as fou nd tresp assing on the Gov-
by the police for assaulting his w ife, is brought into ernor s p rivate estate. She claim s she has
the emergency room for an evaluation. The mans been having an affair w ith the Governor and
behavior is w ild and unpredictable. H e refuses to d em and s that he acknow led ge he is the father
answ er your questions and accuses you of having of her 5-year-old son. This is the third tim e the
an affair w ith his w ife. At one point he picks up a p atient has been caught stalking the Gover-
chair and looks menacingly at a nurse before being nor. The patient is su bsequ ently ad m itted to
subd ued by police officers. Detectives tell you that the inp atient unit and d ischarged several d ays
the man has been arrested in the past for aggression later to her p arents w ith w hom she lives. The
tow ard family members, and that the only chance of Governor u ltim ately d ecid es to pu rsue charges
his avoid ing jail time that night is to be ad mitted to against the w om an. The w om ans attorney
the psychiatric unit. H ow ever, the man makes it clear em p loys a forensic p sychiatrist to evalu ate the
that he d oes not w ant to stay in the hospital. w om an and rend er an op inion abou t her m en-
tal status at the tim e of the crim e. Which of
14. Which of the follow ing shou ld ju stify the the follow ing statem ents regard ing her incar-
p hysicians d ecision to involu ntarily ad m it ceration is m ost accu rate if fou nd not guilty by
the patient? reason of insanity?

(A) H istory of prior p sychiatric (A) She w ill not serve any tim e and w ill
hosp italization. be released w ith close p sychiatric
(B) Im m inent threat to others. follow -u p .
(C) Med ication noncom p liance. (B) She w ill serve half her crim inal sentence
incarcerated and the other half in a
(D) Presence of a d iagnosable m ental
locked p sychiatric facility.
d isord er.
(C) She w ill serve her original crim inal
(E) Refu sal to follow -u p w ith p sychiatric
sentence once she is d eem ed sane.
ou tpatient services.
(D) She w ill serve less tim e than if fou nd
After being ad m itted to the psychiatric unit, the gu ilty.
patient becom es increasingly aggressive tow ard (E) She w ill serve m ore tim e in a locked
staff. H e ad am antly refu ses all m ed ications offered p sychiatric u nit than she w ou ld in
to help calm his agitated behaviors. The patient at p rison if she w ere fou nd gu ilty.
one point accu ses a staff m em ber of stealing m oney
from his w allet and attem p ts to p u nch him . You Questions 17 and 18
place a stat ord er for a H ald ol 5 m g intram u scu lar A 9-year-old girl is brou ght to the p ed iatric em er-
(IM) injection to sed ate the p atient and p revent fu r- gency d ep artm ent by her p arents for evalu ation of
ther escalation of his com bative behaviors. a p ersistent cough. She is w ithd raw n and com plains
of a scratchy throat. Vital signs are stable and the
15. Ad m inistering the H ald ol w ithou t the p atient is afebrile. On exam ination, the lungs are
p atients consent d irectly challenges w hich of clear to auscultation bilaterally and the posterior
the follow ing ethical principles? oropharynx is clear. There are m ultiple bruises on
(A) Au tonom y her bu ttocks and back. A chest x-ray d em onstrates
(B) Beneficence several rib fractu res in d ifferent stages of healing.
H er p arents rep ort that their d au ghter is qu ite active
Que s tions : 1421 145

w ith her you nger siblings and often gets into fights (D) Find ou t the boyfriend s nam e and
w ith them . The p atient agrees w ith her p arents. telephone num ber to confirm the
history.
17. Which of the follow ing is the m ost likely (E) Maintain confidentiality by disclosing no
d iagnosis? information, but encourage the patient to
(A) Age-app ropriate rou gh p lay d iscuss this issue w ith her parents.
(B) Major d ep ressive d isord er
20. At the p atients next follow -u p visit, she
(C) Physical abu se requ ests to be tested for H IV. She rep orts that
(D) Sexu al abu se her boyfriend recently tested negative, bu t
(E) Som atic sym p tom d isord er given her ad m itted p rom iscu ity, she w ond ers
if she has contracted H IV. Which of the follow -
18. Which of the follow ing is the m ost ap p rop ri- ing shou ld be d one p rior to ad m inistering the
ate next step ? H IV test?
(A) Confront the siblings abou t their (A) Directly inform the p atients boyfriend
behavior. that he is at risk for H IV.
(B) Discharge the p atient w ith follow -u p in (B) Exp lain to the p atient how the test is
1 w eek to reevalu ate her bru ises. p erform ed and interp reted , along w ith
(C) N otify Child Protective Services. inform ation on confid entiality and how
(D) Refer for fam ily therapy to ad d ress the you w ill p roceed if the resu lt is p ositive.
issu e of rou gh p lay. (C) Inform the d ep artm ent of p u blic health
(E) Treat the p atient w ith p enicillin. of you r intent to ad m inister an H IV test.
(D) N otify her p arents abou t the patients
Questions 19 and 20 requ est for an H IV test.
(E) Perform a p regnancy test.
A 17-year-old girl p resents to you r office com plain-
ing of a bu rning vaginal d ischarge. She inform s you
21. A 44-year-old m an w ith insu lin-d ep end ent d ia-
that she has been involved in a consensual sexu al
betes m ellitu s and end -stage renal failu re has
relationship w ith her boyfriend and is w orried that
been on d ialysis for several years w hile aw ait-
she has contracted a sexu ally transm itted d isease
ing a kid ney transp lant. H e feels as thou gh he
(STD). You start the p atient on antibiotics. Three
has w aited long enough and d oes not w ant
days later, you discover that the culture of her vaginal
to continu e living tied to a m achine. After
d ischarge grew N eisseria gonorrhoeae.
several fam ily m eetings and consultations
w ith other p hysicians, he inform s you that he
19. A few d ays later, you receive a call from the
no longer w ishes to be d ialyzed . You obtain a
p atients p arents d em and ing to know w hy
p sychiatric consu lt that conclu d es no evid ence
their d au ghter w as seen in you r office. Which
of a m ood or thou ght d isord er. The fam ily is
of the follow ing is the m ost appropriate next
u p set w ith the p atients d ecision and d em and s
step?
that you continue to ad minister d ialysis u ntil a
(A) Alert the parents that their d au ghter is transplant is available. Which of the follow ing
at risk for hu m an im m u nod eficiency is the m ost ap p rop riate next step ?
virus (H IV) and shou ld be tested .
(A) Coerce the p atient into continu ing
(B) Ask the p arents abou t their d au ghters treatm ent.
sexu al history.
(B) Continu e d ialysis u ntil you convince the
(C) Exp lain to the p arents that their ethics com m ittee to su p p ort the fam ilys
d au ghter has contracted an STD and d ecision.
requ ires im m ed iate antibiotic treatm ent.
(C) Encou rage the fam ily to d issu ad e the
p atient from w ithhold ing treatm ent.
146 5: Le ga l a nd Ethica l Is s ue s in P sychia try a nd Me d ic ine

(D) Resp ect the patients w ishes and 24. A 23-year-old w om an w ith a know n history
d iscontinu e d ialysis. of heroin u se is ad m itted for intravenou s (IV)
(E) Tell the patient that a transplant w ill antibiotic treatm ent for infective end ocard itis.
arrive soon and encou rage him to The nu rse inform s you that the p atient w as
rem ain in treatm ent. accid entally given the w rong antibiotic bu t
has su ffered no ad verse reaction. Which of the
22. A 40-year-old su rgical attend ing is ad m itted to follow ing is the m ost ap p rop riate next step ?
the m ed ical u nit after d eveloping severe right (A) Encourage the p atient to seek legal
flank pain. Fu rther w orkup confirm s a d iag- action becau se a critical m istake has
nosis of nep hrolithiasis. One of the su rgical occu rred .
resid ents asks you abou t the attend ings con-
(B) Enforce the no harm , no d isclosu re
d ition in ord er that ad equ ate coverage can be
ru le.
arranged . Which of the follow ing is the m ost
ap prop riate next step? (C) First rep ort the m istake to the hosp ital
ad visory com m ittee.
(A) Arrange a conference betw een you r (D) Inform the p atient that she w as given
m ed ical attend ing and the su rgical the w rong m ed ication.
hou se staff.
(E) Notify the patients family of the mistake.
(B) Inform the su rgical resid ent of the
attend ings cond ition and length of stay. 25. A 38-year-old w om an is ad m itted to the oncol-
(C) Reveal only the estim ated length of stay. ogy u nit w ith severe aplastic anem ia. She
(D) Tell the su rgical resid ent that you w ill ap p ears p ale and w eak. Vital signs ind icate a
not say anything, bu t she cou ld take a blood p ressu re of 110/ 75 m m H g and a p u lse
look in the attend ings chart. of 110/ m in. H em atocrit is 18%. On MSE, cog-
(E) Tell the su rgical resid ent to ad d ress all nition is intact and there is no evid ence of a
concerns d irectly to the attend ing. m ood or p sychotic d istu rbance. The p atient
states that she is a Jehovahs Witness and
23. A 43-year-old m an is referred to you for refu ses any blood transfu sion on the basis of
continued treatm ent of d epression after his her religiou s beliefs. Which of the follow ing is
release from jail. The cou rt m and ated p sychi- the most appropriate next step in the treatment
atric treatm ent w hile he is on p robation. The of this patient?
p atients probation officer calls you for infor- (A) Ad m inister p acked red blood cells.
m ation regard ing his cond ition, p rogress, and
(B) Exp lain the im p lications of no treatm ent
treatm ent com pliance. Which of the follow ing
bu t resp ect the p atients refu sal for
is the m ost ap p rop riate resp onse?
treatm ent.
(A) Corresp ond w ith the p robation officer (C) Persu ad e the p atient that she m u st
only through w ritten d ocum ents. accep t the transfu sion.
(B) Discuss the case with the probation officer (D) Refer the case to the hosp itals ethics
because the treatment is court mandated. com m ittee.
(C) Ignore the requ est altogether becau se (E) Refer the p atient for involu ntary
p sychiatric treatm ent bears no relation p sychiatric treatm ent based on her
to law enforcem ent. life-threatening d ecision.
(D) Lim it you r d iscussion w ith the
p robation officer to only treatm ent 26. The p arents of a new born w ith Dow n syn-
com pliance because the rest of the d rom e find their d au ghter to be lethargic
inform ation is confid ential. and m inim ally resp onsive. Med ical evalu a-
(E) Obtain a confid entiality w aiver from the tion is significant for the follow ing cerebro-
p atient before speaking to the probation spinal flu id find ings: opening pressu re of
officer. 100 m m H g, w hite blood cell cou nt of 5,000/ L
Que s tions : 2229 147

(p red om inantly neu trop hils), protein m ore p hysician for years, is d iagnosed w ith hepato-
than 40 m g/ d L, glu cose content m ore than cellu lar carcinom a. The patient d oes not have
40 m g/ d L, and Gram stain p ositive for bac- an ad vance d irective and never d esignated a
teria. You suspect grou p B streptococcal m en- p ow er of attorney. Psychiatric consultation
ingitis and recom m end IV antibiotic therap y. conclud es that the patient is unable to m ake
The p arents feel as thou gh their child w ill u lti- an inform ed d ecision regard ing treatm ent
m ately have a p oor quality of life and requ est options, and you tu rn to his fam ily for gu id -
that treatm ent be w ithheld . Which of the ance. The p atients old est son is ad am ant that
follow ing is the m ost app rop riate next step ? his father receive chem otherapy, w hile the
tw o you nger d au ghters feel that he shou ld
(A) Refer the case to the ethics com m ittee
not su ffer the ad verse effects of chem otherapy
for review at their next sched u led
especially becau se hes so d em ented . Which
m eeting.
of the follow ing is the next appropriate step?
(B) Rep eat the lum bar punctu re to verify
the d iagnosis. (A) Abid e by the d aughters w ishes becau se
(C) Resp ect the p arents w ishes becau se the patients quality of life is already poor.
they are the prim ary d ecision m akers. (B) Abid e by the sons w ishes becau se he is
(D) Start intravenous antibiotics against the the old est.
p arents w ishes. (C) Ask the p atient w hich fam ily m em ber
(E) Threaten to rep ort the p arents to child he w ou ld like to d esignate p ow er of
p rotective services unless they change attorney.
their d ecision. (D) Consu lt the hosp itals ethics com m ittee.
(E) Let the p atient d ecid e w hether or not to
27. You are a p sychiatrist w ho hosts a m orning p roceed w ith treatm ent.
rad io show d ed icated to ed u cating the general
p u blic abou t m ental illness. Du ring a qu es- 29. A 27-year-old w om an w ho is 3 m onths post-
tion and answ er segm ent, a caller p hones in p artu m is brou ght to the em ergency room by
to ask abou t you r op inion regard ing a prom i- her hu sband and m other w ith concerns that
nent politician w hose recent erratic behaviors the patient is not acting like herself. The
have gained significant m ed ia attention. The hu sband inform s the consu lting p sychiatrist
caller asks you d irectly w hether you believe that for the past few w eeks the p atient has
the p olitician has a bipolar sp ectru m d isord er. been increasingly irritable, w ithd raw n, and
Which of the follow ing resp onses is the m ost crying alm ost d aily. H e d oesnt u nd erstand
ap prop riate? w hy the p atient is behaving like this given
that the p regnancy w as planned and the
(A) Com m ent in w ritten form only.
p atient w as looking forw ard to having a fam -
(B) Offering a p sychiatric d iagnosis in su ch ily. The p atient tearfu lly ad m its that this is her
instances is u nethical. first child and that she is overw helm ed w ith
(C) Provid e an off-the-record or the responsibilities of being a new m other.
anonym ou s opinion. On fu rther qu estioning, she hesitantly con-
(D) State you r d iagnosis bu t ind icate that fesses to intru sive thou ghts of su ffocating
other problem s m ay account for the her child and som etim es w orries abou t being
sym p tom s. alone w ith the baby. She d enies su icid al id e-
(E) Stating you r op inion p u blicly is ation or aud itory/ visu al hallu cinations. The
legitim ate as long as the p olitician is not p atient is w illing to seek ou tp atient treatm ent
you r p atient. bu t ad am antly refu ses volu ntary ad m ission.
H er m other states that the p atient is ju st
28. An 84-year-old w id ow ed m an w ith severe exhau sted and that everything w ill be fine
m ajor neu rocognitive d isord er (d em entia), after she gets som e rest. Which of the follow -
for w hom you have been the p rim ary care ing is the m ost ap p rop riate next step ?
148 5: Le ga l a nd Ethica l Is s ue s in P sychia try a nd Me d ic ine

(A) Ad m inister a stat d ose of lorazep am flattered by the p atients sexu al interest and are su r-
2 m g IM and reevalu ate w hen the p rised by you r ow n interest in the patient.
p atient is m ore calm .
(B) Ad m it the patient involuntarily. 31. Which of the follow ing is the most appropriate
(C) Call Child Protective Services becau se cou rse of action?
the infant is at risk of harm . (A) Decline p articip ation in the relationship
(D) Discharge the p atient and inform becau se sex w ith a form er p sychiatric
the hu sband and m other to bring p atient is u nethical.
the patient back to the hosp ital if her (B) Engage in sexu al relations becau se sex
sym ptom s d o not im prove in the next w ith a cu rrent or form er p sychiatric
few w eeks. p atient is ethical.
(E) Give the p atient a 2-w eek sup ply of (C) Engage in sexu al relations becau se sex
flu oxetine to treat her d ep ression and w ith a form er p sychiatric p atient is
then arrange ou tpatient follow -up. p erm issible only if you d o not exp loit
you r p ast p osition of au thority.
30. A physician is at a comm unity fair w ith her (D) Engage in sexu al relations becau se there
spouse w hen a patient approaches the p sychi- is no established p rofessional cod e of
atrist to say hello. The spou se d oes not recog- ethics regard ing sex w ith p sychiatric
nize the patient. The patient d oes not introd u ce p atients.
him self to the sp ou se, nor d oes the physician
(E) Inform the p atient that the p rofessional
acquaint the tw o. After a brief conversation,
cod e of ethics requ ires that you w ait
the patient politely excu ses him self and leaves.
1 year after term ination before you can
On the w ay hom e, the spouse asks, Who w as
ethically engage in sexu al relations.
that man you w ere talking to earlier? The
physician should d o w hich of the follow ing?
32. Which of the follow ing w ou ld be the m ost
(A) Answ er the sp ou ses qu estion tru thfu lly. ap p rop riate cou rse of action if the above
(B) Ask the sp ou se to gu ess the id entity of p atient w ere you r m ed ical or su rgical p atient?
the person. (A) Decline particip ation in the relationship
(C) Inform the sp ou se that he m u st first becau se sex w ith a cu rrent p atient is
p rom ise not to reveal the id entity of the u nethical.
p atient before answ ering the qu estion. (B) Engage in sexu al relations becau se sex
(D) Inform the sp ou se that revealing w ith a cu rrent nonp sychiatric p atient is
su ch inform ation w ou ld com prom ise ethical.
confid entiality. (C) Engage in sexu al relations becau se sex
(E) Lie to the sp ou se to p rotect the id entity w ith a cu rrent nonp sychiatric p atient is
of the patient. p erm issible only if you d o not exp loit
you r p ast p osition of au thority.
Questions 31 and 32 (D) Engage in sexu al relations becau se there
You have been treating a fem ale patient w ho has is no established p rofessional cod e of
been seeing you for p sychod ynam ic p sychotherap y ethics regard ing sex w ith nonp sychiatric
for app roxim ately 6 m onths. N ear the end of the p atients.
su ggested cou rse of treatm ent, the p atient reports (E) Inform the p atient that the p rofessional
that she feels m arked ly better abou t her progress cod e of ethics requires that you w ait
and attributes her im p rovem ent to you r exp ertise. 1 year after term ination before you can
Prior to the last session, she confesses that she has ethically engage in sexu al relations.
alw ays found you attractive and that she w ou ld like
begin an intim ate relationship w ith you . You feel 33. A 52-year-old man, for w hom you have been
the primary care physician for the last 20 years,
Que s tions : 3036 149

w as recently diagnosed w ith amyotrophic (B) Ensu re that the p atient is established
lateral sclerosis. The disease has rapid ly pro- w ith a casew orker to su p ervise the
gressed and he has experienced multiple respi- p atient in the com m unity before
ratory complications that likely w ill require a d ischarge.
tracheotomy. Severe muscle w eakness and atro- (C) Establish follow -u p at the p atients
phy are apparent in all limbs. The patient states outpatient com m u nity m ental health
that there is no meaning in continuing w ithout facility p rior to d ischarge.
his physical capacities. H e asks for your help (D) Keep the p atient on the u nit as long as it
in ending his life in a humane and d ignified is m ed ically necessary.
manner. H is MSE is unremarkable and there
(E) Sp eak w ith the inp atient social w orker
is no evidence of any psychiatric d isorder. You
to d eterm ine w hether the patient is
d iscuss the patients request w ith his family and
eligible for a loan.
they unanimously support his d esire to end
the suffering. Which of the follow ing w ould
Questions 35 and 36
be the most appropriate course of action?
You receive a subp oena from an attorney rep resent-
(A) Ignore the p atients requ est.
ing a p arty that has filed a law su it against one of
(B) Provid e the p atient w ith inform ation
you r p atients. The su bp oena p ertains to releasing
regard ing how to effectively end his
the m ed ical record s of you r patient.
life.
(C) Provid e the p atient w ith enou gh 35. Which of the follow ing is the m ost ap p rop riate
m ed ication refills to p rovid e a lethal next step ?
d ose.
(D) Refu se to p articip ate in assisting the (A) Contact the attorney w ho obtained
p atient w ith su icid e and focu s on the su bpoena to d iscu ss the p rocess of
resp ond ing to the p atients end -of-life releasing the m ed ical inform ation.
issu es. (B) Do not release the inform ation and
(E) Resp ect the p atients w ishes by help ing contact your patient regard ing the
him end his life in a p ainless and su bpoena.
resp ectfu l m anner. (C) Release the m ed ical record s d irectly to
the presid ing jud ge.
34. You are an inp atient p sychiatrist treating a (D) Release the m ed ical record s u p on
p atient w ith bip olar I d isord er. The p atient receiving the su bp oena.
has a long history of m ed ication noncom p li- (E) Requ est that the p atient sign a release
ance resu lting in severe, persecu tory d elu - of inform ation form and tu rn over the
sions d u ring his m anic ep isod es. After 1 w eek record s.
of treatm ent, you receive a p hone call from
hospital ad m inistration inform ing you that 36. A cou rt hearing has been organized by you r
the p atients insu rance w ill not cover the cost p atients attorney to qu ash the su bpoena you
of an ad d itional inpatient stay. You are encou r- have been issu ed . At the hearing, the ju d ge
aged to d ischarge the p atient so that the hosp i- rules that you should release the entire m ed i-
tal w ill not have to incu r these costs. You feel, cal record even thou gh your patient has not
how ever, that the p atient requ ires m ore tim e consented to the release of information. Which
on the inpatient u nit because of safety con- of the follow ing w ould be the most appropriate
cerns. Which of the follow ing actions w ou ld next step?
be the m ost ap p rop riate?
(A) Ap p eal to the states Sup rem e Cou rt to
(A) Contact the insu rance com pany w ithou t block the release of the m ed ical record .
the patients perm ission and request (B) Refu se to d isclose the p atients m ed ical
coverage for ad d itional d ays. record regard less of the cou rt ord ers.
150 5: Le ga l a nd Ethica l Is s ue s in P sychia try a nd Me d ic ine

(C) Release only inform ation that w ill not (A) Becau se you r d ep ression is im p roved ,
incrim inate you r p atient. it w ou ld be ap p rop riate for u s to be
(D) Release the com p lete m ed ical record to bu siness p artners.
the jud ge. (B) I can invest w ith you only w hen ou r
(E) Work ou t a p lan of legal action w ith treatm ent is nearing its end .
you r p atient. (C) I have to d ecline; it p otentially
m ay interfere w ith ou r treatm ent
37. A cou rt-ap pointed forensic psychiatrist is relationship .
evalu ating a m an charged w ith arm ed robbery (D) It is p robably a bad id ea; Im alread y
to d eterm ine w hether he is com petent to stand com m itted in other investm ents.
trial. Du ring the interview, the d efend ant con- (E) Thank you for thinking abou t m e. Id
fesses to m u rd ering a w om an 3 years ago be honored to invest w ith you .
and hid ing her bod y in an u nd isclosed area.
Which of the follow ing actions w ou ld be the Questions 40 and 41
m ost appropriate?
A 23-year-old Cau casian Catholic w om an w ith a
(A) Avoid d etails of the d efend ants prior
history of m ajor d epressive d isord er and m u ltiple
crim inal history in the report.
m ed ical illnesses p resents to your office after argu -
(B) Encou rage the d efend ant to sp eak w ith ing w ith her hu sband . She is a stay-at-hom e m other
his attorney about the m u rd er. of fou r child ren and ad m its that she is u nable to
(C) Im m ed iately notify au thorities of the w ork becau se of her m ed ical lim itations. She rep orts
d efend ants confession. that her hu sband d oes not allow her to visit w ith
(D) Inclu d e this d etail in the final rep ort. friend s or fam ily w ithou t his consent and generally
(E) Withd raw from the case. d oes not help w ith child care or other household
d u ties and obligations. As a result, she is becom -
38. A 28-year-old w om an w ith m ajor d ep ressive ing increasingly resentfu l, d ep ressed , and isolative.
d isord er has been seeing you for w eekly psy- Du ring the session, she ad m its to thou ghts of killing
chotherap y and has failed to p ay her bill for herself. Of note, she has a p ast history of an over-
2 m onths. Which of the follow ing is the m ost d ose attem pt w ith aspirin.
ap p rop riate next step ?
40. Which of the follow ing is her strongest risk
(A) Contact the patients family to d etermine
factor for su icid e?
if the patient is financially stable.
(B) Contract w ith a billing collector to (A) Age
d em and im m ed iate p aym ent. (B) Gend er
(C) Inform the p atient that you w ill not see (C) Marital statu s
her if she d oesnt pay for her treatm ent. (D) Past history of su icid e attem p t
(D) Inqu ire as to the reasons she has been (E) Term inal m ed ical illness
avoid ing p aym ent at the p atients next
visit. 41. Which one of the follow ing, if d ocu m ented ,
(E) Term inate the treatm ent. w ould mostly likely legally protect a physician
in the event of a patient suicid e?
39. A wealthy 46-year-old male banker is in psy-
(A) A w ritten no self-harm contract
chotherapy with you for treatment of a single
signed by the p atient.
episode of major depression. After significant
improvement in his symptoms, he offers you the (B) An assessm ent of su icid e risk and
opportunity to take part in one of his financial p rotective factors.
ventures. The investment appears to be sound (C) The p atients m issed ap p ointm ents.
and fairly lucrative. Which of the following is (D) The p atients refu sal to consid er
the most appropriate response to the banker? p harm acological treatm ents.
Que s tions : 3745 151

(E) The patients verbal p rom ise to seek ative on the u nit. H e becom es intru sive, not able to
m ed ical attention if feeling su icid al. be red irected , and d em and s im m ed iate release. You
exp lain to him that you feel he is gravely d isabled
42. A 36-year-old m an w ith a history of bip olar I and u nable to care for him self. H e d isagrees w ith
d isord er is brought to the em ergency d epart- you and d em and s d u e p rocess.
m ent by p olice after stabbing a patron in a
bar room braw l. H is blood alcohol level w as 44. On w hich of the follow ing legal princip les is
0.320 u pon arrival, and the p atient requ ired the patients request for a hearing based ?
intram u scu lar (IM) H ald ol for agitation. The (A) Actu s reu s
p atient has no recollection of the event, and
(B) H abeas corp us
the victim d ied 2 hours later. The p atient has a
(C) Mens rea
history of assau lt w hile being m anic. H e cu r-
rently p articip ates in w eekly p sychotherap y (D) Parens p atriae
sessions and attend s his m ed ication m anage- (E) Rights u nd er the Fou rth Am end m ent
m ent ap pointm ents every m onth. H is law yer
has chosen to assert an insanity p lea in d efense 45. The cou rt agrees that the p atient is severely
of the patient. Which of the follow ing factors d isabled and in need of acu te m ed ical m anage-
is m ost likely to u nd erm ine his assertion of the m ent. You initiate treatm ent w ith qu etiapine
insanity d efense? and his sym ptom s of m ania m arked ly d im in-
ish. The p atient requ ests im m ed iate d ischarge
(A) Inability to recall the event. and agrees to follow u p w ith a p artial hospi-
(B) Mental d isease or d efect. talization program . You feel that he w ou ld
(C) Prior history of assau lt. benefit from fu rther inp atient treatm ent bu t is
(D) Voluntary intoxication. no longer gravely d isabled or a threat for self-
(E) Violent nature of the crim e. harm . Which of the follow ing is the next m ost
ap p rop riate step ?
43. You are a forensic p sychiatrist hired as an
(A) Ap pease the patient by increasing
exp ert w itness by the d efense attorney of a
sm oking privileges.
m entally ill p atient charged w ith crim inal m is-
(B) File for another cou rt hearing to d etain
cond u ct. The attorney is seeking you r help in
the patient fu rther.
convincing the ju ry that the patient w as m en-
tally ill at the tim e of the crim e. Which of the (C) Ignore the p atients requ est becau se he
follow ing is your prim ary resp onsibility as an has been com m itted by the court.
exp ert w itness? (D) Persu ad e the patient to stay for a few
m ore d ays.
(A) Cou ntering evid ence of crim inal
(E) Release the patient to the partial
resp onsibility.
hosp italization p rogram .
(B) Establishing reasonable d ou bt.
(C) Evalu ation, d iagnosis, and initiation of Questions 46 and 47
treatm ent of the accu sed .
(D) Obtaining a not guilty by reason of You are consu lted by the m ed ical team to evalu -
insanity verd ict. ate a w om an on the m ed ical u nit w ho su ffers from
a m ajor neurocognitive d isord er (d em entia). The
(E) Rend ering an opinion based on
internist believes that she requ ires a central line for
reasonable m ed ical certainty.
flu id s and m ed ication, bu t is u nsu re if she is able to
Questions 44 and 45 fu lly com p rehend the risks and benefits of the p ro-
ced ure. The team is requ esting help in d eterm ining
A 48-year-old m an is involu ntarily ad m itted to the her capacity to give inform ed consent.
hosp ital after an acute m anic episod e. The patient
is hyperactive, d em and ing, and increasingly talk-
152 5: Le ga l a nd Ethica l Is s ue s in P sychia try a nd Me d ic ine

46. Which of the follow ing com ponents is the d enies any hallu cinations or d elu sions. Which
m ost im portant in obtaining inform ed consent of the following items from your evaluation of
in this case? the patient most raises your suspicion regarding
a d iagnosis of m alingering?
(A) Ability to read and w rite.
(B) Absence of m ental illness. (A) Du ration of sym p tom s.
(C) Involving fam ily m em bers in this (B) Flashbacks w hile aw ake.
d iscu ssion. (C) H istory of m ajor d ep ressive d isord er.
(D) Petitioning a court to establish the (D) Lack of physical inju ries.
p atients com petence. (E) Prior incarcerations.
(E) Raising alternative treatm ent op tions.
49. One of you r p atients consistently m isses
47. The fam ily is su bsequ ently contacted abou t ap p ointm ents w ithou t giving you ad vance
her cond ition, and they requ est a m eeting notice. After num erou s failed attem p ts at
w ith the treatm ent team . Du ring the m eeting, resolving this issu e w ith the patient, you real-
the patients eld est son notifies you that the ize that the p atients behavior is not changing
p atient d id create a living w ill ap proxim ately and you d ecid e to d ischarge the p atient from
1 year ago, but they are unsure whether it would you r care. Which of the follow ing m ost ap p ro-
be useful. Which of the follow ing w ould you p riately d escribes w hat you shou ld d o?
tell them best d escribes the pu rpose of a living
(A) Contact the patients fam ily/ friend s
w ill?
to requ est their help in im p roving the
(A) Absolves personal resp onsibility. p atients attend ance.
(B) Arranges for fu neral services and (B) N otify the p atients insu rance com p any
d istribu ting her estate. that she is d ischarged from you r care.
(C) Establishes p ersonal p references (C) N otify you r staff that the p atient is not
regard ing end -of-life issu es. to be given fu rther appointm ents.
(D) Prevents the p atient from changing her (D) Write a letter to the p atient notifying her
m ind about life su pport if she becom es that she has been d ischarged effective
term inally ill. im m ed iately d u e to her failu re to
(E) Requ ests p hysician-assisted su icid e if com ply w ith treatm ent.
she becom es term inally ill. (E) Write a letter to the p atient stating that
she w ill be d ischarged in 60 d ays d ue to
48. A 28-year-old m an involved in a m otor vehicle her failu re to com p ly w ith treatm ent.
accid ent brings a law su it against the d river.
Em ergency d ep artm ent record s d o not show 50. You are asked by the cou rt to p rovid e a foren-
any p hysical inju ries, but the p atient is claim - sic evaluation for a child cu stod y case. The
ing to su ffer from p osttrau m atic stress d is- child had been living w ith his biological
ord er (PTSD). You are asked to evalu ate the m other and stepfather since birth. Recently,
p atients sym p tom s. H e com p lains of d istress- the child s stepfather passed aw ay from lu ng
ing d ream s of the accid ent, having flashbacks cancer leaving the u nem ployed m other alone
w hile aw ake, and avoid ing the intersection to raise the child . The child s biological father,
w here the accid ent occu rred . H is sym p tom s w ho is a renow ned orthop ed ic su rgeon in the
have lasted for 3 m onths. Past psychiatric his- com m unity, is requ esting full cu stod y becau se
tory is significant for m ajor d epressive d isor- he asserts that he is better able to financially
d er, im p u lsivity, and violent behavior. H e has su pport the child . The child s m other refu ses
been incarcerated nu m erou s tim es, show ing to relinquish cu stod y d u e to concerns that
a blatant d isregard for the law. On MSE, the leaving her hou sehold w ill em otionally im p act
p atient is likable and coop erative. H is m ood is the child . She also im p lies that the father has
rep orted as being d ep ressed and he cu rrently anger p roblem s and voices su sp icion as to
Que s tions : 4651 153

w hether he is, in fact, the biological father. that your colleagu e has been m aking increas-
Which of the follow ing factors w ou ld be the ingly careless m istakes and m issing im p ortant
m ost im p ortant in d eterm ining w hich p arent m eetings and d iscu ssions. Occasionally, you
shou ld get custod y? observe that his hand s shake w hen he is d ocu -
m enting his notes or hold ing his charts. You
(A) Who can p rovid e for the best interests
are concerned that he m ay have an alcohol use
of the child ?
d isord er. Which of the follow ing is the m ost
(B) Who has the highest level of ed u cation? ap p rop riate cou rse of action?
(C) Who is in the best m ental and p hysical
health? (A) Confront the colleagu e and d em and that
he seek ad d iction cou nseling services.
(D) Who is the m ost financially stable?
(B) Contact the Dru g Enforcem ent
(E) Who is the biological p arent?
Ad m inistration to rescind his license.
51. You are an internal m ed icine resid ent w ho has (C) Do nothing so as to avoid p ersonal
been assigned to rotate throu gh the inp atient liability.
m ed ical service for the next 2 m onths. Shortly (D) N otify the hosp itals com m ittee for
after you begin, you notice that one of you r im p aired p hysicians.
colleagu es consistently com es to w ork late and (E) N otify the local p olice.
sm ells of alcohol. Som etim e later, you notice
Ans we rs a nd Expla na tions

1. (D ) A physician or therap ist has a d uty to w arn. The California Su p rem e Cou rt revised
p rotect id entifiable victim s from im m inent its p reviou s hold ing in Tarasoff II (1982) w ith
d anger. This obligation m ay be fu lfilled by a d u ty to p rotect. Durham v. United States
d irectly contacting the p arty at risk, notify- held that an ind ivid u al is not crim inally
ing the p olice, or taking other ap p rop riate resp onsible if the u nlaw fu l act w as the p rod -
m easu res to p rotect the victim . Contacting the u ct of m ental d isease or m ental d efect. The
p atients fam ily fails to protect the intend ed d ecision resu lted in the p rod u ct ru le of crim i-
victim . Although notification of the police is an nal responsibility (insanity d efense) and u lti-
accep table intervention in this case, d isclosing m ately w as rep laced by the Am erican Law
u nrelated confid ential inform ation abou t the Institu te stand ard . In Rogers v. Commissioner
p atient is inap prop riate. When notifying third of the Department of M ental Health, the court
p arties, care should be taken to release only exp ou nd ed op inions regard ing the treatm ent
inform ation necessary to p rotect the p otential of involu ntarily com m itted patients. OConnor
victim . The rem aining choices fail to protect v. Donaldson and Rouse v. Cameron relate to
the potential victim . involu ntary hosp italization.

2. (E) Tarasoff v. Regents of University of California 3. (E) The fou r Ds of establishing m ed ical m al-
is the land m ark legal p reced ent establishing p ractice cases involve proving that a deviation
liability to third p arties. It ru led that m ental of du ty directly cau sed a dam age. The plaintiff
health p rofessionals have the d uty to p rotect m ust establish the presence of a fid u ciary or
id entifiable, end angered third p arties from d octorpatient relationship (a d u ty), negli-
im m inent threats of seriou s harm m ad e by gence (d eviation from stand ard of care), and
their ou tpatients. Therefore, the physician m ay d am ages d irectly cau sed by that negligence.
ethically break confid entiality in cases involv- Typ ically, the bu rd en of p roof in m ed ical m al-
ing im m inent harm to others. In the Tarasoff practice cases (civil cases) is by a p rep ond er-
case, a stud ent at the University of California ance of evid ence (greater than 50%). Proof
(Prosenjit Pod d ar) exp ressed intentions to beyond a reasonable d ou bt refers to crim i-
kill a fellow stu d ent (Tatiana Tarasoff) to his nal cases. Clear and convincing evid ence is
therapist. Pod d ar tragically follow ed throu gh reserved for sp ecial cases d ecid ed by the cou rt.
on his threat and m u rd ered the you ng w om an N either crim inal intent nor crim inal m ischief
at her hom e. Tarasoff v. Regents of University is requ ired to p rove m ed ical m alp ractice.
of California hold s legal preced ent only in the
state of California; how ever, the case has been 4. (C) As a broad generalization, a good p atient
ad op ted by p hysicians/ therapists as the stan- d octor relationship is key for a p hysician to
d ard of care and has led to nationw id e legis- potentially avoid or d eflect a law su it. It is the
lative changes. There have been tw o ru lings physicians d u ty as a health care p rovid er to
regard ing the Tarasoff case. The original d eci- facilitate op en com m u nication regard ing the
sion, Tarasoff I (1976), established a d u ty to patients illness, treatm ent recom m end ations,

154
Answe rs : 18 155

risks versu s benefits of su ch treatm ents, and 7. (B) Com p etence is d eterm ined by an ind ivid -
to acknow led ge any errors that the physician u als ability to m ake d ecisions in accord ance
m ay have m ad e. Requ iring that a p atient sign w ith ones ow n goals, concerns, and valu es.
a no harm contract is not reasonable nor Althou gh p hysicians often rend er opinions
d oes it stand u p as a legal d ocum ent in a court on an ind ivid uals com petence, only a ju d ges
of law . H iring a m alpractice attorney, seeing ru ling d eterm ines w hether an ind ivid u al is
the p atient on a m ore frequ ent interval, or com petent or incom petent. The p resence of
avoid ing prescribing m ed ications w ith p oten- p aranoid d elu sions regard ing the p atients
tially ad verse sid e effects d o not d ecrease a fam ily qu estions her ability to m ake rational
p hysicians risk for m alp ractice su its. d ecisions, and significantly u nd erm ines her
assertion that she w as com p etent to m ake a
5. (E) The standard of care for determining medi- w ill. The inability to read and w rite, the p res-
cal malpractice is based on how a similarly qual- ence of a d iagnosable psychiatric d isord er,
ified practitioner w ould have performed und er and the refu sal to u nd ergo treatm ent recom -
the same or similar situation. Usually the testi- m end ed by her prim ary care physician d o not
mony of an expert w itness (a physician quali- au tom atically u nd erm ine a p atients com p e-
fied by evidence of his/ her expertise, training, tence to m ake a w ill as long as the criteria for
and special know ledge to provid e an opinion testam entary cap acity are m et. An exp lana-
about the case) is necessary to d emonstrate the tion of w hy the p atient w ants to d onate her
stand ard of care in med ical malpractice suits. estate to the local hu m ane society d oes not
Treatments that are endorsed by the American u nd erm ine the p atients com p etence to m ake
Med ical Association, evid ence based , free from a w ill u nless su ch an exp lanation d em on-
ad verse effects, or the most current are not suf- strates that the p atient is u nable to m eet any
ficient to establish the stand ard of care. of the sp ecified criteria.

6. (D ) Testam entary capacity refers to ind ivid u als 8. (A) The Am erican Law Institu te test is the
com p etence to m ake a legally valid w ill. Case m od ern stand ard for the insanity d efense. It
law and statu tes d iffer across states. H ow - hold s that a person w ith a m ental d isease or
ever, the central elem ents of testam entary d efect shou ld not be held crim inally resp onsi-
cap acity inclu d e: (1) com p rehension of the ble for an act if the p erson (1) cou ld not app re-
act of w riting or signing a w ill, (2) know led ge ciate the w rongfu lness of the cond uct and (2)
of p otential heirs, and (3) u nd erstand ing cou ld not conform the cond uct to the require-
the extent of ones assets or p rop erty. Con- m ents of the law . The Am erican Law Institu te
servatorship of estate refers to an ind ivid - test expand ed u p on the 1843 MN aghten rule,
u als ability to m anage his or her finances. w hich held that an ind ivid u al w as not gu ilty
Inform ed consent is the p rocess of obtaining by reason of insanity if, as a resu lt of a m ental
p erm ission for m ed ical treatm ent. Im p ortant d efect, the p erson w as u naw are of the natu re,
elem ents inclu d e the ability to m ake a vol- qu ality, and consequence of his or her act or
u ntary d ecision regard ing treatm ent, exp la- d id not know that the act w as w rong (right-
nation of risks and benefits, and cap acity to w rong test). The Du rham ru le arose from a
m ake an inform ed d ecision. The p resence or 1954 District of Colu m bia case in w hich the
absence of a m ental illness d oes not neces- cou rt held that an accu sed is not crim inally
sarily im p air an ind ivid u als com p etency to resp onsible if his u nlaw fu l act w as the p rod -
m ake a w ill. Althou gh w itnesses are neces- u ct of a m ental d isease or m ental d efect. This
sary at the signing of a w ill in ord er for the is also know n as the p rod u ct ru le of crim inal
w ill to be enforceable in a cou rt of law , the resp onsibility. The irresistible im p u lse ru le is
w itness m u st first ensu re that the ind ivid u al an antiqu ated stand ard for p roving an insan-
has testam entary cap acity in the event the ity d efense. Established in England in 1922,
w ill is challenged and the w itness is asked to the irresistible im p ulse rule states that an ind i-
attest to the ind ivid u als com p etence. vid u al is not resp onsible for a crim inal act if
156 5: Le ga l a nd Ethica l Is s ue s in P sychia try a nd Me d ic ine

he w as unable to resist that act d u e to a m ental is im p ortant in m aking d ecisions related to


illness. m ed ical treatm ents bu t has little bearing on
ones com petency to stand trial. A person can
9. (B) Crim inal resp onsibility requires (1) crim i- suffer from a m ental illness and be com petent
nal intent (m ens rea) and (2) a crim inal act to stand trial.
(actu s reu s). H aving either d im inished m ental
capacity or a know n psychiatric d isord er can 13. (B) In volu n tary h osp italization is u sed w h en
serve to avoid crim inal responsibility, bu t are a p h ysician believes th at an in d ivid u al is an
not required to p rove crim inal resp onsibility. im m in en t th reat to th em selves or oth ers, or
Mod u s operand i is the pattern in w hich an is oth erw ise u n able to ap p rop riately care
action or crim e is execu ted by a p erson. A w it- for th em selves. Arran gin g a p olice h old is
ness to the crim e also is not requ ired to p rove in correct as p olice h old s are issu ed w h en
crim inal responsibility. an in d ivid u al w h o is arrested requ ires a
p sych iatric evalu ation . If th e arrested in d i-
10. (D ) Before discharging or transferring the vid u al d id n ot requ ire in p atien t p sych iatric
patient, it is important to determine whether she care, h e w ou ld be released back in to th e
can make an informed decision regarding her cu stod y of th e p olice. Arran gin g for ou tp a-
care. Informed consent requires that the patient tien t treatm en t w h en th e p atien t is actively
(1) comprehend the issues and/ or choices pro- h om icid al is an im p rop er cou rse of action .
vided by the physician, (2) appreciate the con- N otification of th e p olice or w arn in g th e
sequences of making a certain decision, and (3) p atien ts m oth er is n ot im m ed iately n ec-
arrive at a voluntary decision after weighing the essary if th e p atien t is w ell con tain ed in a
facts. At this point in the vignette, there is not closed p sych iatric u n it.
enough information to establish the patients
ability to make an informed decision regard - 14. (B) Involu ntary ad m ission to the hosp ital is
ing her care. Exceptions to informed consent ju stified w hen the p atient is d eterm ined to
include incompetence, emergency situations be (1) an im m inent d anger to self or others or
(e.g., comatose or unresponsive patient), a com- (2) gravely d isabled or unable to care for self.
petent patients waiver of informed consent, The criteria for involu ntary hosp italization
and therapeutic privilege. The other choices are is d erived from the d octrine of parens patriae
incorrect because this patients ability to make w hich allow s the state to intervene and act as
an informed decision has not been assessed. a su rrogate parent for those w ho are unable to
care for them selves or m ay harm them selves.
11. (B) A com p etent patient has the right to refu se State law s d iffer on the length of tim e a person
treatm ent, even if the consequ ences are life can be com m itted to the hospital on the basis
threatening. In the extension of this vignette, of either criterion. A history of prior psychi-
the patient has ad equately d em onstrated the atric hosp italization, m ed ication noncom p li-
elem ents of an inform ed d ecision. The other ance, presence of a p sychiatric d isord er, or
choices are incorrect becau se they w ould refu sal to follow -u p w ith ou tp atient services
p revent a com petent patient from m aking an are all im p ortant factors w hen d eterm ining
inform ed d ecision in her treatm ent. w hether an ind ivid u al shou ld be hosp italized ,
bu t are not requ isite elem ents for involu ntary
12. (A) In Dusky v. United States (1960), the Su p rem e hosp italization.
Cou rt ru led that com petency to stand trial
requ ires the crim inal d efend ant to be able to 15. (A) Forcing treatm ent on a patient d irectly
(1) rationally consu lt w ith ones attorney and challenges his au tonom y. In general, an ad u lt
(2) rationally and factu ally u nd erstand the w ith d ecisional cap acity has the legal right
p roceed ings against him . A history of p rior to d eterm ine w hat m ay be d one to his bod y.
assau lt charges or illiteracy d oes not constitu te H ow ever, a physician m ay m ed icate a patient
grou nd s for incom p etency. Inform ed consent against his w ill and w ithou t a cou rt hearing
Answe rs : 920 157

if it is d one to p revent a p atient from cau sing PTSD, ADH D, and op p ositional d efiant d is-
im m inent harm to him self or others. In the ord er. Age-appropriate rou gh play is not a
vignette, the patient is grossly im paired and likely cau se for this child s inju ries. The his-
p oses an im m ed iate threat to others. Benefi- tory and physical exam ination d o not provid e
cence refers to a physicians d uty to d o good sufficient evid ence to d iagnose either a m ajor
for the p atient. Confid entiality, or physician d ep ressive d isord er or som atic sym ptom d is-
p atient privilege, is a legal concep t that p ro- ord er. There is not enou gh evid ence to su g-
tects com m u nications betw een a p atient and gest sexu al abu se in this p articu lar case.
his or her d octor from being d isclosed to a third
p arty. Justice refers to the concep t of rew ard 18. (C) Every state requ ires m and atory rep orting
and pu nishm ent and the im p artial d istribu - of abu se or neglect to Child Protective Ser-
tion of social benefits (e.g., w hether resou rces vices. In cases in w hich there is a reasonable
shou ld be d istributed equ ally to those in need suspicion of abu se, a report should be filed
vs. w hether they shou ld go w here they w ou ld even if the p atient and fam ily d eny the alle-
have the greatest im pact on the ind ivid u al or gations. In cases of su sp ected abu se, step s to
society). N onm aleficence refers to a physi- ensu re the safety of the m inor shou ld be taken
cians intention to not harm or bring harm to im m ed iately. Referring the fam ily to therap y
the patient. or d ischarging the patient w ith follow -u p to
reevaluate her bruises d isregard s the child s
16. (E) N ot gu ilty by reason of insanity (or gu ilty safety and p otentially subjects the minor to
but m entally ill in som e state) is a verd ict ren- continu ed abuse. Consid ering the extent of the
d ered in a crim inal case w hich find s that the physical injuries, it is u nlikely that the patients
d efend ant w as insane at the tim e of com m itting younger siblings inflicted them ; consequently,
the crim e. This is d eterm ined by the applica- neither confronting the siblings about their
tion of the test for insanity used in the particu- behavior nor treatm ent w ith penicillin serves
lar jurisd iction. Contrary to popular belief, the any pu rp ose.
insanity d efense (1) is seld om asserted , (2) is
u su ally u nsu ccessfu l, and (3) typically lead s 19. (E) Accord ing to the Am erican Med ical Asso-
to a greater num ber of years that the d efen- ciation (AMA) Cod e of Med ical Ethics, confi-
d ant spend s in a m ental hospital. Ind ivid uals d entiality in the treatm ent of m inors shou ld
found not gu ilty by reason of insanity are usu- be m aintained u nless sp ecifically p rohibited
ally com m itted to a m ental facility rather than by state law (e.g., in cases of abortion) or w hen
serving tim e incarcerated . Typ ically, the com - the parents involvem ent is necessary to m ake
m itm ent is not for a set am ount of tim e bu t complicated or life-threatening treatment d eci-
rather u ntil the ind ivid u al is d eem ed not to sions. The other choices all constitute violations
be a threat to society. Releasing an ind ivid ual of confid entiality to your patient.
found not guilty by reason of insanity w ithou t
ap p rop riate psychiatric treatm ent is u nethical 20. (B) Inform ed consent for H IV testing involves
and fails to p rotect society. a thorou gh exp lanation of the tests interp re-
tation, inform ation regard ing confid ential-
17. (C) Typical signs of physical abuse inclu d e ity, and fu rther evalu ation need ed if the test
bu rns w ith p ecu liar p atterns (e.g., cigarette is p ositive. The p atient shou ld be aw are of
m arks, geom etric d esigns, bilateral im m er- the risk of false positives and negatives, and
sion patterns), bruises in low -trau m a areas the need for confirm atory testing if the test
(e.g., bu ttocks, genital areas, or back), reti- resu lt is p ositive. Choices (A) and (D ) are
nal hem orrhages, and m ultiple fractu res at incorrect becau se they violate p atient confi-
d ifferent stages of healing. Child abu se can d entiality. When a patient tests positive for
occur in all socioeconom ic levels, and physi- H IV, state law s m ay requ ire notification of the
cally abu sed child ren can d isplay a range sexual partner (this is u sually d one w ithou t
of p sychop athology inclu d ing d ep ression, revealing the p atients id entity) in ad d ition to
158 5: Le ga l a nd Ethica l Is s ue s in P sychia try a nd Me d ic ine

the d epartm ent of p ublic health. There is no 25. (B) Exp laining the im p lications of no treat-
reason to notify the states health d ep artm ent m ent bu t resp ecting the p atients refu sal for
or perform a p regnancy test before ad m inis- treatm ent is the m ost ap p rop riate cou rse of
tering an H IV test. action. This allow s the p atient to m ake an
inform ed d ecision w ith regard to her treat-
21. (D ) The American Medical Association (AMA) m ent. The elem ents integral to inform ed con-
Cod e of Ethics states that the principle of patient sent inclu d e (1) cap acity to m ake a d ecision;
autonomy requires that physicians respect the (2) an ad equ ate exp lanation of the risks, ben-
d ecision to forego life-sustaining treatment of a efits, rationale for treatm ent, and alternative
patient w ho possesses d ecision-making capac- treatm ents (inclu d ing no treatm ent at all);
ity. In this case, the patients d esire to stop and (3) a volu ntary d ecision. In this vignette,
d ialysis should be respected because adults there is no evid ence that the p atient su ffers
w ith d ecisional capacity are able to determine from a cond ition that m ight im p air her d eci-
w hich treatment they should accept or deny. sional cap acity. Fu rther, her stated religiou s
The remaining choices und ermine the patients belief is consistent w ith a Jehovahs Witnesss
right to make an informed decision. refu sal of hu m an blood p rod u cts. Ad m in-
istering p acked red blood cells infringes on
22. (E) Physicianpatient confidentiality should be the p atients right to refu se a blood transfu -
maintained except in certain cases (e.g., d uty to sion and consequ ently is incorrect. Strongly
protect others, m and atory rep orting, or em er- p ersu ad ing the p atient to arrive at a d ecision
gencies). Thus, the physician should not reveal for treatm ent is coercive and cou ld p revent
confid ential information w ithout the express the p atient from m aking a volu ntary d eci-
consent of the patient. The remaining choices sion. It is inap p rop riate to refer the case to
violate physicianpatient confid entiality. the hosp itals ethics com m ittee if the p atient
has d ecisional cap acity and d ecid es to refu se
23. (E) Confid entiality betw een patient and phy- treatm ent. It is also inap p rop riate to invol-
sician should be respected even in cases of u ntarily hosp italize this p atient if she has
cou rt-mand ated treatm ent. Obtaining a confi- arrived at her d ecision in com p liance w ith the
d entiality w aiver from the patient is essential elem ents of inform ed consent.
before releasing any inform ation to the proba-
tion officer. Ignoring the probation officers 26. (D ) Accord ing to the AMA Cod e of Med i-
request is inappropriate in such circumstances. cal Ethics, treatm ent d ecisions m u st be m ad e
The other choices are incorrect because they in the best interest of the child or neonate
violate physicianpatient confid entiality. regard less of the d esires of the p arents. Treat-
m ent can be w ithheld or w ithd raw n only
24. (D ) The patient shou ld im m ed iately be in cases in w hich the risks far ou tw eigh the
inform ed that she w as accid entally given the benefits, there is low p otential for su ccess, or
im p rop er m ed ication. Evalu ating the p atient w hen treatm ent extend s the child s su ffering
for any ad verse reaction and exp laining the w ithou t p otential for a m eaningfu l existence.
m istake is the ap p rop riate first step . Encou r- In this p articu lar case, the neonate su ffers
aging the p atient to seek legal action is both from a seriou s, bu t treatable, central nervou s
alarm ist and inap p ropriate in su ch a case. The system infection and treatm ent shou ld not
fact that the patient d id not suffer harm is be w ithheld . The lu m bar p u nctu re resu lts
not a reason to conceal the m ed ication error. u nequ ivocally d em onstrate bacterial m enin-
Althou gh m any hosp itals requ ire d ocu m en- gitis. Withhold ing treatm ent u ntil the next
tation or reporting of su ch to a p articu lar regu larly sched u led ethics m eeting w ou ld
p erson or com m ittee, this shou ld not be the likely resu lt in the neonates d em ise. Threat-
first course of action. N otifying the patients ening to rep ort the p arents to child p rotective
fam ily of the m istake is a breach of confid enti- cu stod y u nless they revoke their d ecision is
ality and inap propriate. coercive and inap p rop riate.
Answe rs : 2132 159

27. (B) It is u nethical to offer sp ecific p sychiatric Peripartum mood episod es can present either
d iagnoses for ind ivid u als that you have not w ith or w ithout psychotic features. Infanticide
p ersonally exam ined . Moreover, com m ent- is most often associated with psychotic epi-
ing on an ind ivid u al that you have evalu ated sod es (e.g., command hallucinations to harm
w ou ld be a breach of confid entiality. the infant, d elusions that the infant is evil, etc.).
Although the patient in the vignette denied
28. (D ) Consu ltation w ith the ethics com m ittee overt psychotic symptoms, she d emonstrated
shou ld be sought w hen the p atients prior a significant clinical d ecline and acknow l-
w ishes are u nclear or if fam ily m em bers are edged that she had thoughts of harming her
u nable to agree on treatm ent options. Com - infant. Discharging the patient und er any cir-
p etent ad u lts m ay form ulate p references cumstances is inappropriate because the infant
(ad vance d irectives) regard ing their m ed i- remains at risk of harm. Child Protective Ser-
cal treatm ent in the event that an inju ry or vices should be notified in cases of suspected or
illness cau ses severe im p airm ent or loss of known child abuse or neglect. Administering
d ecisional cap acity. Ad vance d irectives can lorazepam and reassessing the patient do not
be established by d ocu m enting the ad u lts ad d ress the issue of the babys safety.
m ed ical preferences and goals in a living w ill
or by d esignating a health care proxy (d u rable 30. (D ) Most physicians w ill somed ay find them -
p ow er of attorney for health care) to m ake selves in situations w here they encounter their
treatm ent d ecisions on the patients behalf. patients in the com mu nity. Patientphysician
Fam ily and / or close associates shou ld be con- confid entiality d oes not cease once a physician
su lted to aid in reaching treatm ent d ecisions leaves the hospital and m ust be maintained
if there is no ad vance d irective or d esignated regard less of w ho is making the inquiry. Confi-
p ow er of attorney. Ethics com m ittee consulta- dentiality can only be broken when the physician
tions are also u seful w hen the pow er of attor- has a d uty to protect others, in cases of m and a-
neys d ecision is unreasonable or contrary to tory reporting, or in med ical em ergencies. The
the patients prior w ishes. It w ou ld be inap- remaining choices violate physicianpatient
p ropriate to allow a severely incapacitated confid entiality.
p atient to d esignate a pow er of attorney or
m ake treatm ent d ecisions. Choosing sid es in 31. (A) Sexu al or rom antic relationship s w ith cu r-
a fam ily conflict w ou ld be arbitrary; consu lta- rent or form er p sychiatric p atients (regard -
tion w ith the hosp ital ethics com m ittee w ou ld less of the tim e elap sed since treatm ent) are
help clarify issu es in a com p licated case such alw ays consid ered u nethical, m aking the
as this. other choices incorrect. Ad d itionally, p sychi-
atrists shou ld avoid situ ations in w hich their
29. (B) The most appropriate course of action is to interactions w ith cu rrent or form er p atients
involuntarily ad mit the patient. Major d epres- cou ld be m isinterp reted as an intim ate rela-
sive d isord er with peripartum onset is a form of tionship .
clinical d epression that d evelops in 3% to 6% of
w omen during pregnancy or the weeks follow- 32. (A) Sexu al relations w ith cu rrent patients are
ing d elivery. Approximately 50% of postpar- u nethical and constitute sexual m iscond u ct.
tum d epressive episod es actually begin prior to The AMA Cod e of Med ical Ethics states that
delivery, thus episod es are collectively referred sexu al and / or rom antic relations w ith form er
to as peripartum episod es. Postpartum depres- p atients are u nethical if the physician u ses or
sion usually refers to episod es that start w ithin exp loits tru st, know led ge, em otions, or influ -
4 w eeks after d elivery. Symptoms can last ence d erived from the p reviou s p rofessional
from several months to a year. It is imperative relationship . At the very least, the p hysician
that examining physicians inquire about the has an ethical d u ty to term inate the physician
mothers thoughts tow ard her child when con- p atient relationship prior to initiating any
cerns for peripartum d epression are present. rom antic or sexu al relationship s.
160 5: Le ga l a nd Ethica l Is s ue s in P sychia try a nd Me d ic ine

33. (D ) Accord ing to the AMA Cod e of Med ical the party requesting the subpoena and releas-
Ethics, p hysician-assisted su icid e is fu nd a- ing the med ical record s w ithout consent from
m entally incom patible w ith the physicians the patient constitute unethical breaches of
role as healer, and the p hysician shou ld physicianpatient confidentiality. Releasing the
aggressively respond to a p atients end -of- m ed ical information d irectly to the jud ge at
life need s rather than assist in su icid e. Phy- this stage makes no sense and w ould likew ise
sician-assisted su icid e is d efined as the act of be a breach of confid entiality. Requesting that
facilitating a p atients d eath by p rovid ing you r patient sign a release of information is
the necessary m eans and / or inform ation to inappropriate w ithout first d iscussing w hether
enable the p atient to perform the life-end ing the patient consents to releasing the record s.
act. Sim p ly ignoring the p atients requ est is
inap p rop riate becau se it fails to recognize and 36. (D ) Turning over m ed ical record s to com ply
ad d ress p otential end -of-life issu es. End -of- w ith a d irect cou rt ord er is ethical and legal.
life issu es that shou ld be ad d ressed w ith the To d efy a ju d ges ord er can p otentially p lace a
p atient includ e ad equ ate pain control, em o- p hysician in contem p t of court. When the d is-
tional/ fam ily su pport, com fort care, respect closu re of confid ential inform ation is required
of patient autonom y, and appropriate m u l- by law or cou rt ord er, p hysicians shou ld d is-
tid isciplinary referrals (hospice, cou nseling, close the m inim al inform ation requ ired by
religiou s issu es, etc.). The rem aining choices law . H ow ever, in this case releasing p artial
are consistent w ith the d efinition of p hysician- record s violates the cou rt ord er. Working ou t
assisted su icid e and are therefore u nethical. a p lan of legal action or ap p ealing to the states
Sup rem e Cou rt is not the role of the physician,
34. (D ) The treating p hysician is u ltim ately legally bu t rather the p atients attorney.
resp onsible for d eterm ining w hether a p atient
is ap p rop riate for d ischarge. Discharging a 37. (A) In the United States crim inal ju stice sys-
m ed ically unstable or u nsafe patient is con- tem , a com petency evaluation is an assess-
sid ered negligence and can potentially lead m ent of the d efend ants ability to u nd erstand
to a law suit if there is an ad verse outcom e. A and rationally p articip ate in a cou rt process
p sychiatric patient can be consid ered for d is- (Dusky v. United States, 1960). The p sychia-
charged as long as he/ she d oes not m eet the trists goal in assessing for com petency to
criteria for involu ntary ad m ission (i.e., im m i- stand trial involves evaluating w hether the
nent risk to self or others or grossly u nable to d efend ant has a rational und erstand ing of the
care for self). The patient in the vignette is not crim inal cou rt proceed ings, the capacity to
consid ered safe to leave the hospital, w hich u nd erstand the d ifferent potential outcom es of
m akes the other answ er choices incorrect. the cou rt proceed ings (e.g., plead ing gu ilty vs.
Contacting the insu rance com p any w ithou t not guilty), and has a su fficient ability to com -
p erm ission violates patient confid entiality. m u nicate w ith and assist the attorney in the
Regard less of w hether the p atient has insu r- d efense. The exam iner should d ocu m ent that
ance or qualifies for a loan, the p hysician he or she has fu lly inform ed the d efend ant of
shou ld continu e to treat the patient as long as the purpose and nature of the evaluation pro-
it is m ed ically necessary. ced u re and the extent of nonconfid entiality. In
situations such as this w here confid entiality is
35. (B) When a physician is served a subpoena to lim ited the p sychiatrist m u st m ake every effort
release m ed ical information, the appropriate to m aintain confid entiality w ith regard to any
first step is to contact the patient to find out inform ation that d oes not bear d irectly u p on
if he or she consents to the release. Discussing the legal purpose of the evaluation. There is no
the implications of the law suit w ith your need for the exam iner to w ithd raw from the
patient and clarifying w hether the patient case in the event of a crim inal confession, nor
has the proper legal cou nsel for the law su it it is app rop riate for the exam iner to p ersu ad e
are also important. Contacting the attorney of the d efend ant to act in any particular m anner.
Answe rs : 3343 161

38. (D ) Inquiring abou t possible reasons the stance use), em ploym ent status (unem ployed
patient m ay be avoid ing or d eferring payment or retired > em ployed ), su pport system (living
m ay help u ncover important treatment issues. alone > living w ith others), and health statu s
Such inquiry often lead s to how the patient (seriou s m ed ical cond ition or term inal illness
feels about the therapy and / or the therapist. > good health).
These feelings can be u tilized to ad d ress fac-
tors related to her symptoms. Ad d itionally, 41. (B) Suicidal behavior is among the most stressful
concerns may arise regard ing the financial sit- emergencies encountered by mental health pro-
uation of the patient that otherw ise w ou ld not fessionals. There is no algorithm or scoring tool
have been d iscussed in the sessions. Exploring that is able to consistently identify a patients
such issues should be d one in a sensitive m an- risk for suicide. If a malpractice claim is brought
ner to avoid alienating the p atient. Contacting up against a psychiatrist, adequate documenta-
the patients family violates physicianpatient tion of the patients suicide risk and protective
confid entiality. Requesting that a billing collec- factors assist the court in determining the scope
tor d em and s im med iate payment and term i- and complexities of the patients treatment. Fail-
nating treatment w ould be far too aggressive at ure to document these assessments and inter-
this point. Inform ing the patient that you w ill ventions potentially gives the court a reason
not see her if she d oes not im med iately pay her to believe they were not done. No self-harm
bill may be perceived as confrontational and contracts do not hold up as legal documents in a
threatens the therapeutic alliance. court of law. The remaining choices are not suf-
ficient enough to legally protect a psychiatrist in
39. (C) Becom ing involved in the business ventu re the event of a patient suicide.
of a psychotherapy patient could have signifi-
cant ad verse effects on the patients transfer- 42. (D ) An ind ivid u al w ho com m its a crim e w hile
ence (feelings and m em ories exp erienced by voluntarily intoxicated cannot assert an insan-
the patient that are aroused by the therap ist) ity d efense becau se the basis of su ch behavior
as w ell as on the p hysicians cou ntertrans- stem s from a rational d ecision to d rink. Invol-
ference (feelings arou sed in the p hysician u ntary intoxication m ay be u sed effectively as
w hile w orking w ith the p atient). Therefore, an insanity d efense if the ind ivid u al had no
it shou ld be avoid ed . Bu siness relationship s choice in becom ing intoxicated (e.g., the per-
w ith form er p atients are often p erceived as son w as unknow ingly d rugged ). A m ental
p roblem atic, and business relationships w ith d isease or d efect or inability to recall the event
current patients are consid ered u nethical. m ight be used to enhance an insanity d efense,
Anger, d istru st, and gu ilt are som e of the feel- not u nd erm ine it. Prior history of assau lt or
ings that m ay occu r if a bu siness ventu re su ch the natu re of the crim e shou ld have no bear-
as the one d escribed w ere to fail. Fu rtherm ore, ing on an insanity d efense. H ow ever, if p rior
a physicians bu siness involvem ent w ith a violent behavior has been associated w ith a
p atient m ight affect the neu tral stance essen- m ental illness and the inability to appreciate
tial to effective psychotherapy. The central the crim inality of ones cond uct or conform to
concern that involvem ent in a business ven- the law , this cou ld be u sed to strengthen an
tu re m ight interfere w ith treatm ent shou ld be insanity d efense.
exp lained to the p atient honestly.
43. (E) Med ical expert w itnesses are called to
40. (D ) The m ost significant pred ictor of su icid e p resent their professional opinions based
is a p ast history of su icid e attem p t. Other on reasonable m ed ical certainty. An expert
su icid e risk factors includ e: ad vancing age w itness shou ld have cu rrent and su bstan-
(>45), gend er (M > F), m arital status (sep a- tial experience and know led ge in the area in
rated or d ivorced > m arried ), alcohol u se d is- w hich they testify, and m ay be called to testify
ord er (su icid e rate is 50% higher in those w ith in both civil and crim inal trials. A p hysician
alcohol u se d isord er vs. those w ith no sub- w ho p rovid es exp ert testim ony is exp ected to
162 5: Le ga l a nd Ethica l Is s ue s in P sychia try a nd Me d ic ine

p erform an im p artial evaluation and p rovid e volu ntary choice w ithout coercion or d u ress,
an ind epend ent op inion. Exp ert testim ony and an ad equ ate u nd erstand ing of the ratio-
shou ld be based upon current scientific d ata nale, risks, benefits, and alternative treat-
and a stand ard of care that is accep ted am ong m ent op tions. The rem aining choices are not
those w ho p ractice in the respective field . It is requ ired for inform ed consent.
not the responsibility of the exp ert to ensu re a
p articu lar ou tcom e for either sid e. Unlike the 47. (C) Living w ills, also know n as ad vance d irec-
trad itional p hysicianpatient relationship , the tives, preserve patient autonom y by establish-
exp ert w itness d oes not initiate treatm ent of ing p ersonal p references regard ing end -of-life
p atients and has lim its on confid entiality. issu es. If p atients lose their d ecisional cap acity
in the fu tu re, ad vance d irectives ou tline w hat
44. (B) H abeas corp u s refers to the U.S. Constitu -
p roced u res they d o or d o not d esire w ithou t
tions clau se that the state shall not d eprive
relying on p hysician or fam ily m em ber op in-
any person of life, liberty, or prop erty, w ithou t
ions. Patients are able to change their living
d ue process of law . Und er habeas corp us, cit-
w ills if d esired . Physician-assisted su icid e is
izens have the right to p etition the cou rt w hen
not consid ered ethical. The rem aining choices
d etained to d ecid e w hether they are being held
are not characteristics of a living w ill.
law fu lly. Establishing crim inal resp onsibility
requ ires the p resence of both actu s reu s and
m ens rea. Actu s reu s refers to the volu ntary 48. (E) Malingering is characterized by intentional
act of com m itting a crim e, and m ens rea refers feigning of sym ptom s for second ary gain (i.e.,
to crim inal intent. Both choices are incorrect financial com p ensation, d isability benefits,
as they have nothing to d o w ith the patients avoid ing crim inal p rosecu tion or m ilitary
right to d u e p rocess. Parens p atriae refers to d uty, etc.). A d iagnosis of m alingering shou ld
the states fu nction as parent for those unable be su sp ected w hen there is the p resence of
to care for them selves. This is often u sed for one or m ore of the follow ing factors: (1) liti-
ju stification of involu ntary com m itm ent. The gation (esp ecially w hen the p atient is referred
Fou rth Am end m ent p rotects citizens from by a law yer), (2) d iscrep ancy betw een objec-
u nreasonable searches and seizu res. tive find ings and the p atients com p laints, (3)
a d iagnosis of antisocial p ersonality d isord er
45. (E) It w ou ld be u nethical to involuntarily (w hich his history su p p orts), and (4) a failu re
d etain a patient in treatment if he or she is to com ply w ith the evaluation and recom -
no longer d eemed to be gravely d isabled or a m end ed treatm ents. The d u ration of sym p-
threat to self or others. In this case, the patient tom s in this case has no bearing on a d iagnosis
shou ld be p rom ptly d ischarged to the partial of m alingering. The lack of physical inju ries or
hospital program . Appeasing the p atient by the presence of flashbacks in a claim of PTSD
increasing sm oking privileges fails to ad d ress is not su fficient to raise su sp icions of m alin-
the key issue in this question. Filing for another gering. PTSD occu rs in ind ivid u als exp osed to
cou rt hearing is approp riate only if the patient actu al or threatened d eath or inju ry and con-
is still gravely d isabled or a d anger to self or sequently m ay not involve p hysical injuries. A
others. Althou gh the patient w as initially com- p ast history of m ajor d epression d oes not raise
mitted by the court, it w ould be unlaw ful to a su sp icion of m alingering.
ignore the patients request for d ischarge if he
no longer meets criteria for involuntary hospi- 49. (E) Accord ing to the AMA Cod e of Med ical
talization. Persu ad ing the patient to stay a few Ethics, p hysicians are obligated to p rovid e
more d ays even thou gh he w ishes to leave the continu ity of care for their patients. When
hospital and no longer m eets criteria for invol- d ischarging a patient from you r care, ad vance
untary com mitment is inapp ropriate. notice of the p end ing d ischarge m u st be given
to allow the patient su fficient tim e to obtain
46. (E) The key elem ents to inform ed consent another p hysician. Contacting the p atients
inclu d e the cap acity to m ake a d ecision, a fam ily breaches confid entiality and is not the
Answe rs : 4451 163

ap propriate proced ure for d ischarging the 51. (D ) Physicians are ethically obliged to rep ort
p atient. N otifying the insu rance com p any fails colleagues w ho are im paired , incom petent,
to inform the patient of her d ischarge and d oes or engaging in unethical cond uct. The d u ty
not provid e her tim e to obtain a new p hysi- to report stem s from physicians obligation
cian. The rem aining choices fail to provid e the to protect patients against harm . Reporting
p atient w ith su fficient notice to obtain prop er p roced u res vary from state to state; how ever,
m ed ical follow -u p . the AMA Cod e of Med ical Ethics suggests ini-
tial reporting to a hosp itals im p aired physi-
50. (A) The prevailing concep t in d eterm ining cian p rogram . If no su ch program exists, the
child custod y is based on ind ivid u al(s) w ho chief of staff or clinical service d irector shou ld
can provid e for the best interests of the child . be notified . For p hysicians w ithou t hosp ital
Factors that can be used to determine the best p rivileges, an im p aired physicians p rogram
interests of the child includ e assessing the ind i- ru n by a m ed ical society or the state board
vid uals biologic relation, financial stability, licensing com m ittee shou ld be m ad e aw are of
level of ed u cation, and health. H ow ever, it is the im paired physician. The other choices d o
the aggregate rather than any single factor not m eet the AMA gu id elines and fail to ini-
w hich d eterm ines w hat is in the best interest tiate proper treatm ent and sup ervision of the
of the child . im p aired p hysician.
This page intentionally left blank
CHAPTER 6

Diffe re ntial Diag no s is and


Manag e me nt
Que s tions

D IRECTION S (Questions 1 through 60): For each of (C) H lop erid ol (H ld ol) nd p erp hen zine
the multiple-choice questions in this section, select (Tril fon)
the lettered answ er that is the one best response in (D) Mirt z p ine (Re eron) nd cit lop r
each case. (Celex )
(E) Zip r sid one (Geod on) nd sertr line
Questions 1 and 2 (Zoloft)

A 43-ye r-old w o n p resents to you r office tell- Questions 3 and 4


ing you th t recently she h s been exp eriencing n
incre se in the volu e of voice th t she h s been A 29-ye r-old w o n w ho ju st d elivered 3 w eeks
he ring for ye rs. It const ntly criticizes her beh v- go is referred bec u se her obstetrici n noticed th t
iors reg rd less of her ood . She lso notes th t for she ppe red to be d isheveled . Upon initi l inter-
ost of the p st ye r, her ood h s been very low . view , the p tient tells you th t she h s been feel-
She is no longer ble to get ny ple su re fro w tch- ing d ow n since d elivering her son. She tells you
ing television. H er sleep is p oor nd her energy is th t w hile she continu es to c re for her son, she is
low . In d d ition, she d escribes 20-lb w eight loss in grow ing incre singly d ep ressed bec u se she d oes
the p st ye r bec u se she no longer feels the need to not get ny p le sure fro t king c re of hi . She is
e t. She d enies the u se of ny d ru gs or lcohol. u n ble to sleep nd h s not been e ting uch either.
She feels tired ll the ti e. She lso tells you th t t
1. Which of the follow ing d i gnoses best ti es, she w ill he r b by in the b ckground cry-
ccou nts for this p tients sy p to s? ing, bu t w hen she checks on her son, he is sleep ing
sound ly. She d enies ny thoughts of w nting to hu rt
(A) Bip ol r II d isord er her son or herself.
(B) M jor d epression w ith psychotic fe tures
(C) Schizo ffective d isord er 3. Which of the follow ing d i gnoses is the ost
(D) Schizop hreni likely?
(E) Schizoid p erson lity d isord er (A) Ad ju st ent d isord er
(B) Brief p sychotic d isord er
2. Which of the follow ing ed ic tion co bin -
(C) M jor d ep ressive d isord er w ith
tions w ou ld best tre t this p tients sy pto s?
p erip rtu onset nd p sychotic fe tu res
(A) Div lp roex sod iu nd lor zep (D) Postp rtu obsessive-co p u lsive
(Ativ n) d isord er
(B) Flu oxetine (Proz c) nd d i zep (E) Schizo ffective d isord er, d ep ressed typ e
(V liu )

165
166 6: Diffe re ntia l Diag nos is a nd Ma na g e me nt

4. Which of the follow ing is the ost pp rop ri- Questions 7 and 8
te first step in her tre t ent?
A 62-ye r-old w o n p resents to you r office long
(A) Flu oxetine (Proz c) nd qu eti p ine w ith her d u ghter. H er d u ghter exp l ins to you
(Seroqu el) th t l tely, her other h s been extre ely con-
(B) H lop erid ol (H ld ol) cerned bout her bod y od or. Despite ultiple re s-
(C) H osp it lize the p tient su r nces fro her d ughter th t she d oes not s ell,
(D) Lithiu she continues to be concerned th t other people find
her bod y od or extre ely offensive. The p tient tells
(E) Su p p ortive ther p y
you th t she know s th t she s ells bec u se she c n
s ell her ow n bod y od or ll the ti e d espite t king
Questions 5 and 6
u ltip le show ers throu ghou t the d y. Despite being
A 64-ye r-old w o n is brou ght to the e ergency concerned bout her od or, she continues to w ork
d ep rt ent by her neighbor, w ho s ys y friend fro ho e s she h s d one for the p st 30 ye rs
isnt cting right. The p tient requ ires the su p p ort nd she continu es to sp e k w ith friend s on the tele-
of nurse w hile w lking to n ex in tion t ble. p hone. She p ys her onthly bills w ithou t the help
Ex in tion reve ls th t she c nnot correctly id en- of her f ily. H er thinking otherw ise see s logic l
tify the se son or the tow n she is in. She d oes not nd pp rop ri te.
recognize her neighbor. She is in ttentive nd see -
ingly p thetic to the ctivity rou nd her. She d ozes 7. Which of the follow ing is the ost likely
off repe ted ly d uring the interview , bu t e ch ti e d i gnosis?
is rou s ble nd resu es nsw ering qu estions. H er (A) Delu sion l d isord er
nsw ers re illogic l nd inconsistent.
(B) M jor d ep ressive d isord er w ith
Vit l signs re w ithin nor l li its nd she
p sychotic fe tu res
is neither tre u lou s nor d i p horetic. N eu rologic
ex in tion find s bil ter l sixth nerve p lsy nd (C) Obsessive-co p u lsive d isord er
horizont l nyst g u s. Urine toxicology screen nd (D) P r noid p erson lity d isord er
blood lcohol level re neg tive. (E) Schizop hreni

5. Which of the follow ing is the ost likely 8. Which of the follow ing w ou ld be the ost
d i gnosis? p p rop ri te tre t ent?
(A) Acu te su bd u r l he to (A) C rb zep ine (Tegretol)
(B) Alcohol w ithd r w l (B) Electroconvu lsive ther p y (ECT)
(C) Folic cid d eficiency (C) Flu oxetine (Proz c)
(D) N or l p ressu re hyd rocep h lu s (N PH ) (D) Lithiu (Esk lith)
(E) Wernicke encep h lop thy (E) Ol nz p ine (Zyp rex )

6. Which of the follow ing is the ost i p ort nt Questions 9 and 10


first step in n ging this p tient?
A 29-ye r-old n w ith history of chronic schizo-
(A) Ad inistr tion of benzod i zep ine p hreni co es to the e ergency d ep rt ent w ith
(B) Ad inistr tion of folic cid te p er tu re of 102.9F, l bile blood p ressu re rising
(C) Ad inistr tion of thi ine to 210/ 110 H g, p u lse of 110/ in, nd resp i-
(D) Co p u ted to ogr p hy (CT) sc n of the r tory r te of 22 bre ths/ in. This p tients ed ic -
he d tions includ e h lop erid ol, benztrop ine (Cogentin),
nd clon zep (Klonop in). H e c nnot correctly
(E) Intr venou s (IV) flu id s nd observ tion
id entify the d y, d te, or ye r, nd believes hi self
to be in city fro w hich he oved 10 ye rs go.
A f ily e ber ind ic tes th t 3 d ys go he w s
Que s tions : 413 167

he lthy nd co pletely oriented nd th t he h s no 11. Which of the follow ing shou ld be the ost
signific nt ed ic l or surgic l history. i ed i te n ge ent?
Physic l ex in tion reve ls th t he is in cute
(A) Ad ission to ed ic l u nit
d istress w ith hypertonicity. L bor tory ex in tion
reve ls cre tinine phosphokin se (CPK) of 45,000 (B) Ad ission to p sychi tric u nit
IU/ L, w hite blood cell cou nt of 15,000/ L nd no (C) Med ic tion nd d isch rge to close
left shift, sod iu of 145 Eq/ L, nd cre tinine of f ily e ber
2.5 g/ d L. Lu b r p u nctu re prod u ces cle r fluid (D) Referr l to n ou tp tient p sychi trist
w ith slightly elev ted p rotein cou nt. (E) Restr ints nd ed ic tion in the
e ergency d ep rt ent
9. Which of the follow ing is the ost likely
d i gnosis? 12. Which of the follow ing ed ic tion(s) w ou ld
(A) Anticholinergic synd ro e be the ost p p rop ri te?
(B) Centr l nervou s syste (CN S) infection (A) Benzod i zep ine only
(C) M lign nt hyp erther i (B) Lithiu nd selective serotonin
(D) N eu rolep tic lign nt synd ro e (N MS) reu p t ke inhibitor (SSRI)
(E) Prolonged i obiliz tion (C) Lithiu only
(D) SSRI nd n ntip sychotic
10. With ppropri te tre t ent, the p tient recov- (E) Tricyclic ntid ep ress nt (TCA) only
ers co pletely nd returns ho e. In onths
ti e, he co es to the e ergency dep rt ent Questions 13 and 14
st ting th t the voices in the w lls re telling
hi to kill hi self. He h s t ken no edic tions A 22-ye r-old n is brou ght to the e ergency
since he left the hospit l. His vit l signs re st - d ep rt ent by p olice fter n ep isod e in w hich he
ble nd edic l workup is neg tive. Which of r ns cked the office w here he w orks looking for
the following ther pies should be initi ted first? evid ence. H e st rted this job 2 onths go fter
gr d u ting fro college. H e lives w ith fou r roo -
(A) ECT tes, nd he believes they re je lous of hi
(B) H lop erid ol d ep ot injections bec u se of his job nd h ve therefore been p oison-
(C) Ol nz p ine ing his food . H is f ily reve ls th t once before
(D) Physic l restr ints w hen he beg n college he w ent throu gh p eriod
(E) S fety onitoring only of cting cr zy but got better w ithout tre t ent
nd h s d one w ell since. In the e ergency d ep rt-
Questions 11 and 12 ent, he is shou ting th t he h s been u p for w eek
w riting cl ssic book bou t ccou nting bu t th t
A 39-ye r-old w o n co es to the e ergency so eone t the office stole it fro hi . H e need s
d ep rt ent nd co pl ins th t since her boyfriend to be p hysic lly restr ined by e ergency d ep rt-
broke u p w ith her 3 onths go, she h s been sleep - ent secu rity. Physic l ex in tion nd co plete
ing nd e ting p oorly, h s lost ll interest in her l bor tory w orku p nd toxicology screen p rove to
w ork, nd feels gu ilty th t she d rove her boyfriend be neg tive.
w y. In the p st onth, she h s begu n to feel hop e-
less, help less, nd th t life y not be w orth it. In 13. Which of the follow ing ed ic tion(s) w ould
the p st 2 w eeks, she h s d eveloped belief th t be the ost p p rop ri te to initi te?
r re d ise se is rotting her he rt, nd over the p st
(A) Antip sychotic nd benzod i zep ine
w eek, voice h s been telling her she is no good nd
should t ke n overd ose of he rt ed ic tion she is (B) Bu sp irone (Bu Sp r)
prescribed . At first she w s ble to ignore the voice, (C) C rb zep ine (Tegretol)
how ever, she is now t the point th t she believes (D) Lithiu
she shou ld ct on it. (E) SSRI
168 6: Diffe re ntia l Diag nos is a nd Ma na g e me nt

14. Three onths l ter, the p tient sees his d octor (A) Acu te stress d isord er
for follow -u p . H e is t king lithiu nd h lo- (B) Ad ju st ent d isord er
p erid ol. H e is d oing w ell, except he co pl ins (C) MDD
of p inful u scle cr p ing. H is lithiu level
(D) P nic d isord er
is 0.8 Eq/ L. Which of the follow ing w ou ld
be the ost p p rop ri te next step in his (E) PTSD
n ge ent?
17. A p tient w ith history of bip ol r d isord er is
(A) Decre se the h lop erid ol d ose. d itted to p sychi tric hosp it l in n cute
(B) Decre se the lithiu d ose. nic ep isod e. H er ed ic tions inclu d e n
(C) Incre se the h lop erid ol d ose. SSRI nd benzod i zepine, w hich re both
(D) Incre se the lithiu d ose. d iscontinu ed on d ission. An ntipsychotic
nd ood st bilizer re st rted . Tw o d ys
(E) St rt b clofen (Liores l).
fter d ission, she c lls the nu rsing st ff to
her bed . She is extre ely frightened nd co -
Questions 15 and 16
p l ins excited ly th t she c nnot stop looking
A 29-ye r-old w o n tells her d octor th t bou t u p . On ex in tion, her eyes re noted to be
3 w eeks go she w s c rin g for ch ild w ho r n d evi ted up w rd , bil ter lly. Which of the fol-
into the street nd w s killed by bu s. Since then, low ing sid e effects is ost consistent w ith her
she c nn ot get th e i ge of the ccid ent ou t of h er p resent tion?
ind . Even in sleep , she d re s bou t it nd it p re- (A) N MS
vents her fro sleep ing ore th n few hou rs t
(B) Ocu logyric crisis
nigh t. She u sed to t ke bu s to w ork, bu t she n ow
d rives bec u se she c nnot be r to be ne r bu ses s (C) Retrocollis
this c u ses her to think bou t the ccid ent. In the (D) Torticollis
p st w eek, sh e h s begu n issin g w ork bec u se (E) Tris u s
she is u n co fort ble le ving her h ou se. Sh e feels
extre ely gu ilty, believing the ccid ent w s h er Questions 18 and 19
f u lt.
A 59-ye r-old w o n w ith long history of gener-
15. Which of the follow ing is the ost likely lized nxiety d isord er (GAD) tells her p ri ry c re
d i gnosis? d octor th t 2 d ys go w hile in crow d ed su p er-
rket she felt d izzy, long w ith ssoci ted he rt
(A) Acu te stress d isord er p lp it tions, p ressu re on her chest, nd fright-
(B) Ad ju st ent d isord er ening sense of d oo . Shortly there fter, she fell
(C) M jor d ep ressive d isord er (MDD) u nconsciou s nd w oke u p inu tes l ter to crow d
(D) P nic d isord er nd gor p hobi rou nd her. She felt so ew h t better nd rejected
others d vice th t n bu l nce be c lled . She
(E) Posttr u tic stress d isord er (PTSD)
qu ickly d e her w y ho e.
16. The p tient d ecid es g inst ny ed ic tion
18. Which of the follow ing w ou ld be the ost
bu t follow s u p w ith p sychother p y. A ye r
p p rop ri te next step in her n ge ent?
l ter, lthou gh she is no longer h ving d is-
tressfu l sy pto s rel ting to the ccid ent, she (A) Cognitive-beh vior l ther p y
feels s d nd te rfu l ost of the ti e, is h ving (B) Electroc rd iogr (ECG)
trou ble e ting, h s lost interest in g rd ening, (C) Re ssu r nce th t her cond ition is benign
nd w kes u p t 4 a m every orning, u n ble
(D) Short- cting benzod i zep ines
to get b ck to sleep. She const ntly feels tired
throughout the d y. Which of the follow ing is (E) SSRI
the ost likely d i gnosis?
Que s tions : 1424 169

19. Which of the follow ing d i gnoses is the ost to feel d epressed , helpless, nd hopeless, nd
i p ort nt to ru le ou t first? he su bsequ ently overd oses on his ed ic tion.
H is w ife find s hi u nconsciou s nd p roceed s
(A) Acu te stress d isord er
to c ll 911. Which of the follow ing w ou ld be
(B) C rd iov scu l r d ise se the ost likely c u se of d e th?
(C) GAD
(A) C rd i c rrhyth i
(D) Illness nxiety d isord er
(B) Resp ir tory f ilu re
(E) P nic tt ck
(C) Seizu re
20. A 28-ye r-old le p resents to you r office (D) Shock
bec u se he w nts to stop p rticu l rly d is- (E) Stroke
turbing beh vior. H e tells you th t he often
goes on p u blic tr ins w ith the intention of try- Questions 23 and 24
ing to ru b his genit ls on other p eop le w ithou t
their consent. H e continu es to h ve f nt sies A 19-ye r-old n w ith no p reviou s psychi tric his-
bou t this, bu t he w ishes to stop bec u se he tory is noted by his college roo te to be cting
is fr id of getting into trou ble. Which of the biz rrely for the p st onth nd h lf, h ving con-
follow ing is his ost likely d i gnosis? vers tions w ith people w ho re not there, w lking
rou nd the d or itory roo n ked , nd ccusing the
(A) Exhibitionistic d isord er roo te of c lling the Dep rt ent of H o el nd
(B) Frotteu ristic d isord er Secu rity to h ve hi onitored . The p tients vit l
(C) Sexu l sochis d isord er signs re ll w ithin nor l li its nd his neurologic
(D) Sexu l s d is d isord er ex in tion show s no d eficits or bnor lities.
(E) Voyeu ristic d isord er
23. Which of the follow ing tests w ou ld be ost
usefu l in the initi l d i gnosis of this p tient?
Questions 21 and 22
(A) Co p lete blood cou nt (CBC)
A 53-ye r-old n presents to your office co pl in-
(B) Erythrocyte sed i ent tion r te (ESR)
ing of w orsening d epression. H e st tes th t he no lon-
ger enjoys spend ing ti e w ith his f ily, he is only (C) Liver fu nction tests
getting few hours of sleep t night, nd his ppetite (D) N oncontr st CT sc n of the br in
is uch low er th n it used to be. H e feels tired ll d y (E) Toxicology screen
long. You d ecid e to initi te n SSRI.
24. If the bove test w ere neg tive, w hich of the
21. Which of the follow ing sid e effects is he follow ing w ou ld be the ost likely d i gnosis?
ost likely to exp erience fter 3 onths of
(A) Delu sion l d isord er
tre t ent?
(B) M jor d ep ression w ith p sychotic
(A) Di rrhe fe tu res
(B) H e d ches (C) Schizop hreni
(C) N u se (D) Schizop hrenifor d isord er
(D) Sed tion (E) Su bst nce-ind u ced p sychosis
(E) Sexu l d ysfu nction
Questions 25 and 26
22. The p tient retu rns few ore ti es, nd
d u ring e ch visit, you incre se the d os ge of You h ve been sked by the su rgery te to ev lu -
the SSRI. H ow ever, he d oes not report ny te 35-ye r-old n w ho h d surgery to rep ir his
d ecre se in his d epressive sy pto s. Bec use fr ctu red right w rist 24 hou rs go nd is now co -
of his l ck of response to n SSRI, you d ecid e to pl ining of nxiety. The p tient h s been in the hos-
initi te tricyclic ntid ep ress nt. H e continu es pit l for 2 d ys. H is he rt r te is 120 be ts/ in, nd
170 6: Diffe re ntia l Dia g nos is a nd Ma na g e me nt

his blood pressure is 160/ 106 H g. H e is febrile 28. Which of the follow ing w ou ld be the ost
nd reports th t he h s never su ffered nything like pp rop ri te tre t ent for this p tient?
this before. H e is not in ny p in nd h s no pre-
(A) A low -sti u lu s environ ent
viou s psychi tric history. H is ed ic tion inclu d es
only cet inophen for p in control. You note th t (B) Benztrop ine
he is d i phoretic, flu shed , nd tre u lou s. (C) Methylp henid te (Rit lin)
(D) Phenytoin (Dil ntin)
25. Which of the follow ing is the ost p p rop ri te (E) Prop r nolol (Ind er l)
ed ic tion to tre t this p tient?
(A) C rb zep ine 29. A 40-ye r-old d ivorced w o n is brought
in to the e ergency d ep rt ent fter being
(B) Clonid ine (C t p res)
fou nd sleep ing in p ile of le ves. Initi lly, she
(C) Lor zep is d ifficu lt to rou se nd h s trou ble nsw er-
(D) Meth d one ing you r qu estions d u e to u bling. She
(E) N ltrexone (ReVi ) d oes not ppe r to be in respir tory d istress,
bu t her p u p ils re noted to be p inp oint. After
26. Withou t the bove tre t ent, w hich of the few hou rs, she beco es ore inter ctive nd
follow ing co p lic tions is he ost t risk for is co p l ining of d iffu se, cr p y bd o in l
d evelop ing? p in nd sy p to s of the flu . H er pu p ils
re now slightly d il ted , nd she is y w ning.
(A) Abd o in l p in
H er vit l signs re te p er tu re of 99.3F,
(B) Cirrhosis he rt r te of 99 be ts/ in, nd blood pressu re
(C) F tty liver of 142/ 90 H g. Cu rrently, w hich of the
(D) Mu scle cr p s follow ing is the ost likely d i gnosis?
(E) Seizu res (A) C nn bis bu se
(B) Coc ine intoxic tion
Questions 27 and 28
(C) Coc ine w ithd r w l
You re w orking in the psychi tric e ergency (D) Op i te intoxic tion
d ep rt ent of l rge etropolit n hospit l. A (E) Op i te w ithd r w l
20-ye r-old n w ith unknow n p sychi tric his-
tory is brou ght in by the p olice fter being fou nd Questions 30 and 31
stu bling n ked rou nd loc l college c p u s.
H e is rked ly git ted , p cing, nd pp e rs to be A 70-ye r-old w id ow is d itted for n ev lu tion
resp ond ing to intern l sti u li. On ex in tion, you of d epression nd nxiety. She tells you th t for
note th t he is t chyc rd ic w ith he rt r te in the the p st 15 ye rs her f ily d octor h s prescribed
110s, he h s tics or sp s s in his f ce, nd he h s so e p ills th t h ve help ed her sleep nd feel less
vertic l nyst g u s. nervou s. She s ys th t she r n ou t of the yesterd y
nd since th t ti e h s felt incre singly nxiou s nd
27. Which of the follow ing tests w ou ld be ost jittery. She lso notes th t shes now h ving tre ors
helpfu l in the d i gnosis? in her h nd s th t h ve not been there before.
(A) CBC
30. Which of the follow ing ed ic tions w ou ld be
(B) Electroencep h logr (EEG) the ost d ngerou s to su d d enly d iscontinue?
(C) M gnetic reson nce i ge (MRI) of the
br in (A) Flu p hen zine (Prolixin)
(D) N oncontr st he d CT (B) I ip r ine (Tofr nil)
(E) Toxicology screen (C) N ortrip tyline (P elor)
(D) Thiorid zine (Mell ril)
(E) Tri zol (H lcion)
Que s tions : 2535 171

31. Which of the follow ing w ould be the ost Questions 34 and 35
d ngerou s sid e effect of bru p tly stop p ing the
bove ed ic tion? A 34-ye r-old fe le p resents to you r office s she
h s been feeling d epressed nd ngry for the p st
(A) Au tono ic hyp er ctivity cou ple of d ys bec u se she believes th t her boy-
(B) Seizu res friend is going to le ve her. As resu lt, she h s been
(C) H llu cin tions c lling hi every hour ju st to confir th t he is not
(D) Worsening nxiety le ving her. She tells you th t she h s h d ny rel -
tionships in the p st w hich h ve lw ys been very
(E) Vo iting
rocky, c u sing her e otions to const ntly go u p nd
d ow n. When ny of these rel tionships end ed ,
Questions 32 and 33
she cu t on her r s sup erfici lly to relieve stress.
A 73-ye r-old n is d itted to the hosp it l for At this ti e, she tells you th t she d oes h ve so e
co u nity- cqu ired pneu oni nd d ehyd r - thou ghts of w nting to cut herself, bu t no thoughts
tion. On d y 2 of his hosp it liz tion, you re sked of w nting to end her life.
to ev lu te the p tient for d ep ression; the st ff h s
noted th t he see s very w ithd r w n. H e is not e t- 34. Which of the follow ing is the ost likely
ing or sleeping w ell. The nu rsing st ff reports th t d i gnosis?
l st night he w s ngry nd requ ested to le ve the (A) Bip ol r d isord er
hospit l. You t lk w ith the p tients f ily nd find
(B) Bord erline p erson lity d isord er (PD)
th t the p tient h s no previou s psychi tric history.
Prior to the onset of this illness 5 d ys go, he h d (C) H istrionic PD
no d ep ressive sy pto s nd no d ifficu lties w ith (D) M jor d ep ressive d isord er
cognition. On ex in tion, his vit l signs re te - (E) N rcissistic p erson lity d isord er
per tu re of 98.2F, he rt r te of 87 be ts/ in,
blood pressure of 130/ 86 H g, nd p erip her l 35. Which of the follow ing is the ost pp rop ri-
oxygen s tur tion of 95% on roo ir. H e is d row sy te tre t ent for this d i gnosis?
nd oriented only to p erson. H is Mini-Ment l St te
(A) Arip ip r zole (Abilify)
Ex in tion (MMSE) score is 23/ 30.
(B) Cit lop r (Celex )
32. Which of the follow ing is the ost likely (C) Cognitive-beh vior l ther p y
d i gnosis? (D) Di lectic l beh vior l ther p y
(E) Psycho n lysis
(A) Anxiety d isord er
(B) Deliriu Questions 36 and 37
(C) F ctitiou s d isord er
(D) M jor d ep ressive d isord er (MDD) A 65-ye r-old w o n w ith p st ed ic l history
of noninsu lin-d ep end ent d i betes ellitu s nd
(E) M jor neu rocognitive d isord er
d epression is d itted w ith incre singly d epressed
(d e enti )
ood over the l st onth. She is u n ble to co -
plete her crossw ord pu zzles bec use of d ifficu lty
33. The ed ic l te sks for ed ic tion reco -
concentr ting. She h s trouble f lling sleep nd
end tions if the p tient beco es git ted .
lso w kes u p in the id d le of the night. She d enies
Which of the follow ing w ou ld be the ost
su icid l id e tion, but d oes feel gu ilty th t she is
pp rop ri te ed ic tion to reco end ?
d epressed . Prior to this episod e, she w s d oing w ell
(A) Benztrop ine nd w s ctively eng ged in co u nity volunteer
(B) Dip henhyd r ine grou p s. In the l st onth, she h s lost 13 lb d ue to
(C) Lor zep poor int ke. When sked w hy she is not e ting, she
st tes she is w orried th t she w ill beco e infected
(D) H lop erid ol
w ith b cteri . She h s been to her p ri ry p hysici n
(E) Thiorid zine
for n ev lu tion, bu t she cl i s everything w s
172 6: Diffe re ntia l Dia g nos is a nd Ma na g e me nt

nor l. H er hu sb nd confir s th t the p tient h s looks frightened nd sks, Wh t is h p p ening to


been very w orried bou t getting d ise se to the e? H is ood is OK nd his ffect is fl t. H e is
p oint w here she w ill e t only food in se led con- fu lly oriented . H e s ys th t he he rs nu ber of
t iners. H e lso confid es in you th t she h s been p eople t lking to hi , ybe in y he d , s ying
w orried th t she ight h ve c ncer, nd , d esp ite b d things bou t hi .
re ssu r nces fro her p ri ry c re p hysici n, she
continu es to voice her concerns to her hu sb nd th t 38. Which of the follow ing d i gnoses is ost
y intestines re not w orking. likely?
(A) Brief p sychotic d isord er
36. Which of the follow ing is the ost likely
d i gnosis? (B) Delu sion l d isord er
(C) Schizop hreni
(A) M jor d ep ressive d isord er (MDD)
(D) Schizop hrenifor d isord er
w ithou t p sychotic fe tu res
(E) Su bst nce-ind u ced p sychotic d isord er
(B) MDD w ith p sychotic fe tu res
(C) Obsessive-co p u lsive d isord er (OCD) 39. Which of the follow ing l bor tory tests or pro-
(D) Persistent d ep ressive d isord er ced u res w ould be the ost helpfu l in n rrow -
(d ysthy i d isord er) ing the bove d ifferenti l?
(E) So tic sy p to d isord er
(A) CBC
37. Which of the follow ing ed ic tion(s) w ou ld (B) EEG
be the ost p p rop ri te tre t ent for this (C) Electrolytes
p tient? (D) N oncontr st he d CT
(A) Div lp roex sod iu (E) Toxicology screen
(B) Lithiu nd sertr line
Questions 40 and 41
(C) N ortrip tyline nd lor zep
(D) Sertr line A 35-ye r-old w o n w ith no previous person l or
(E) Sertr line nd risp erid one f ily psychi tric history is brought to the e er-
gency d ep rt ent by her husb nd, w ho reports th t
Questions 38 and 39 his w ife w s tte pting to kill herself by cutting her
w rists. H er husb nd tells you th t 6 onths go the
An 18-ye r-old n is brou ght to the p sychi tric p tients gr nd other d ied . Since th t ti e, her hus-
e ergency d ep rt ent by his p rents for ev lu tion b nd believes th t the p tient h s been beco ing
of his beh vior. Three onths go, the p tient st rted ore d epressed . She h s d ifficulty f lling sleep nd
cl sses t the st te university loc ted in d ifferent h s lost 15 lb in 2 onths. She w s recently fired fro
city, lthough he w ou ld co e ho e e ch w eekend her job s p r leg l bec use she w s un ble to con-
to visit. H is p rents h ve noticed th t over the p st centr te nd d e frequent ist kes. She feels guilty
3 w eeks he h s beco e incre singly w ithd r w n nd th t she is un ble to feel better, nd she end orses feel-
d oes not see to be t king c re of hi self. The p r- ings of hopelessness nd w orthlessness. She believes
ents w ere recently c lled by the p tients roo te th t her only w y out of this is to kill herself.
w ho infor ed the th t the p tient h s not been
going to cl sses for the l st w eek, h s not been e ting 40. Which of the follow ing w ou ld be the ost
or b thing, nd h s been spe king bout how peo- p p rop ri te tre t ent?
p le in his cl sses re trying to kill hi . The p tient
(A) Flu zenil (Ro zicon)
d enies ny d ru g u se nd is not on ny ed ic tion.
On ex in tion, the p tients vit l signs re w ithin (B) Flu oxetine
nor l p r eters. H is physic l (inclu d ing neu ro- (C) Flu p hen zine
logic ex in tion) is u nre rk ble. H e is p cing (D) Lor zep (Ativ n)
nd ppe rs to be resp ond ing to intern l sti uli. H e (E) Phenelzine (N rd il)
Que s tions : 3645 173

41. After being begu n on the p p rop ri te ed ic - the d e n of her college is concerned bout her nd
tion, w hich of the follow ing sid e effects y w orried th t she y be d ep ressed . The p tient tells
i ic w orsening sy pto of her illness? you th t d esp ite being f r w y fro her ho e for
the first ti e, she w s enjoying school nd her new
(A) Ak thisi
friend s u ntil 2 onths go, w hen she le rned th t
(B) Constip tion her p rents w ere getting d ivorced . Since th t ti e,
(C) Di rrhe her gr d es h ve gone fro As nd Bs, to Bs nd Cs
(D) Inso ni bec u se she is concerned bou t her p rents nd her
(E) N u se sister t ho e. She feels bu ed ou t ost of the
ti e, nd her friend s note th t she see s unh ppy
Questions 42 and 43 nd occ sion lly beco es te rfu l w hen t lking
bou t her f ily. She d enies d ifficu lty sleep ing or
A 46-ye r-old n w ith no previou s p sychi tric his- ch nges in ppetite or w eight, nd she continu es to
tory p resents w ith co pl ints of feeling d epressed enjoy d ily trip s to the gy to exercise. She d enies
since his w ife d ied 2 w eeks go fro p ncre tic c n- ny suicid l id e tion.
cer. H e sh res th t lthough he know s she is d e d ,
he so eti es he rs her c lling his n e. This h s 44. Which of the follow ing is the ost likely
occu rred sever l ti es nd usu lly h pp ens ore d i gnosis?
often t night w hen he is f lling sleep. H e d enies
ny visu l h llu cin tions, p r noi , or d elu sion l (A) Acu te stress d isord er
beliefs. H e expresses gu ilt bou t not sp end ing ore (B) Ad ju st ent d isord er w ith d ep ressed
ti e w ith his w ife w hen she w s live bu t d enies ood
ny thou ghts of w nting to end his life. H e is now (C) GAD
sleeping only bou t 3 to 4 hou rs t night nd h s (D) MDD
h d slightly d ecre sed pp etite since her d e th.
(E) N or l grief
Desp ite the bove, he h s still been ble to go to
w ork nd focu s on his job.
45. Which of the follow ing tre t ents w ou ld be
ost p propri te?
42. Which of the follow ing is the ost likely
d i gnosis? (A) Di lectic l beh vior l ther p y
(A) Ad ju st ent d isord er (B) Flu oxetine (Proz c)
(B) M jor d ep ressive ep isod e (C) N ortrip tyline
(C) N or l grief (D) Psychother p y
(D) Persistent d ep ressive d isord er (E) Risp erid one (Risp erd l)
(d ysthy ic d isord er)
Questions 46 and 47
(E) Schizop hreni
A 26-ye r-old w o n w ith no p reviou s psychi tric
43. Which of the follow ing w ou ld be the ost history is referred to you by her p ri ry c re p hysi-
pp rop ri te tre t ent for this p tient? ci n for ev lu tion of nxiety tt cks. She tells you
(A) A itrip tyline (El vil) th t p p roxi tely 2 onths go she beg n h ving
period s l sting 10 or 15 inu tes d u ring w hich, she
(B) Flu oxetine (Proz c)
s ys, I feel like I going to d ie. Du ring these epi-
(C) H lop erid ol (H ld ol) sod es, her he rt r ces, she feels s thou gh she c nnot
(D) Lithiu c tch her bre th, she is d izzy nd fr id she y p ss
(E) N o intervention t this ti e out, nd she h s tingling nd tre ors in her h nd s.
She is concerned bec use she is now h ving d iffi-
Questions 44 and 45 cu lty le ving her hou se d ue to w orry th t these ep i-
sod es w ill occu r, king it i p ossible for her to get
You re sked to see n 18-ye r-old w o n w ith
ho e. She is u n ble to id entify ny triggers le d ing
no p reviou s psychi tric or ed ic l history bec u se
174 6: Diffe re ntia l Dia g nos is a nd Ma nag e me nt

to the episod es. She is not on ny ed ic tion nd (C) Obsessive-co p u lsive p erson lity
h s no ed ic l proble s. d isord er (OCPD)
(D) P nic d isord er
46. Which of the follow ing is her ost likely (E) So tic sy p to d isord er
d i gnosis?
(A) GAD 49. Which of the follow ing tre t ents w ould be
(B) P nic d isord er nd gor p hobi ost ppropri te for this p tient?
(C) Sep r tion nxiety d isord er (A) Cognitive-beh vior l ther p y
(D) Soci l nxiety d isord er (soci l p hobi ) (B) ECT
(E) Sp ecific p hobi (C) Lithiu
(D) Psychod yn ic p sychother p y
47. Which of the follow ing w ou ld be the ost (E) Risp erid one
p propri te tre t ent to initi te?
(A) Alp r zol 50. A 47-ye r-old secret ry co es to your office
(B) Lithiu co pl ining, I lw ys w orried . She s ys
th t she w orries bout her job, her kid s, her
(C) Prop r nolol
hou sew ork, nd her hu sb nd . She is seeking
(D) Sertr line (Zoloft) help now bec u se she h s been h ving n
(E) Tr nylcyp ro ine (P rn te) incre singly d ifficu lt ti e concentr ting t
w ork nd h s been ore irrit ble w ith p eo-
Questions 48 and 49 ple rou nd her. H er sleep h s been ok y,
bu t she d oes not feel rested w hen she gets u p
A 36-ye r-old w o n w ith no for er p sychi t-
in the orning. She h s been ore w re of
ric history is referred to you by d er tologist
these feelings over the l st 2 ye rs nd they
for ev lu tion of her chronic lly ch pped h nd s.
occu r l ost every d y. She d enies ny d is-
She s ys she h s been seeing her d er tologist for
crete p nic tt cks. Which of the follow ing
p proxi tely 5 ye rs for this proble nd tre ted
d i gnoses is the ost likely in this p tient?
w ith v riety of top ic l gents w ith li ited su c-
cess. Over the p st 3 w eeks, her h nd s h ve beco e (A) GAD
w orse, to the p oint w here they re lw ys cr cked (B) OCD
nd bleed ing. Relu ct ntly, she confid es in you th t (C) Soci l nxiety d isord er (soci l p hobi )
she h s h d longst nd ing fe r of ger s, bu t since
(D) P nic d isord er
her colle gu e t w ork h s been sick, she h s been
w shing her h nd s t le st 40 ti es p er d y bec u se (E) Schizop hreni
she is fr id of contr cting the d ise se. She lso
refu ses to tou ch nything th t ight infect her w ith- 51. A 28-ye r-old w o n w ith no p revious psy-
ou t u sing h nd kerchief. She d its to being very chi tric or ed ic l history is d itted to the
tid y t ho e, s w ell. She spend s bou t 2 hou rs neu rology service for ev lu tion of cu te onset
in the orning getting show ered . She re lizes her of nu bness nd w e kness of the right sid e of
fe rs of cont in tion re irr tion l, but every ti e her f ce nd right r nd leg. Physic l ex -
she tries to stop w shing her h nd s she beco es in tion show s sy etric l 2/ 4 reflexes in
incre singly nxiou s. ll d istribu tions, d ow ngoing p l nt r reflexes
bil ter lly, nd 2/ 5 strength in the right u p per
48. Which of the follow ing d i gnoses w ou ld be nd low er extre ity in ll u scle group s. N o
ost likely? trop hy or f scicul tions re noted . H er g it
is t xic nd st ggering w ith extre e ex g-
(A) GAD ger ted ove ents of her r s; how ever, she
(B) Obsessive-co p u lsive d isord er (OCD) d oes not f ll w hen bu l ting w ithou t ssis-
t nce. Given the severity of her d eficits, she
Que s tions : 4655 175

see s u nconcerned by her level of d is bility. As. On ex in tion, she is c l , thin w o n


Which of the follow ing d i gnoses w ou ld be d ressed in very b ggy jogging suit. You co -
the ost p propri te? ent on her thinness, nd she tells you she p rid es
herself on her pp e r nce nd tries to st y sli by
(A) Conversion d isord er
exercising bou t 4 hou rs d y long w ith good
(B) F ctitiou s d isord er d iet. She d enies proble s w ith e ting too u ch or
(C) M lingering too little, nd cl i s th t only d iet nd exercise help
(D) So tic sy pto d isord er her to control her w eight. Desp ite you r concerns,
(E) Und i gnosed neurologic d ise se she d its th t she w ou ld like to lose few ore
p ou nd s. Up on fu rther qu estioning, she infor s you
52. A 54-ye r-old w o n w ith p st ed ic l she h s not h d regu l r enstru l period for over
history of hypothyroid is is d itted w ith ye r.
septic right knee. The surgery te sks you
to ev lu te the p tient bec use they fou nd 53. Which of the follow ing w ou ld be the ost
th t the flu id sp ir te fro the knee w s likely w orking d i gnosis?
grow ing p thogen fou nd p ri rily in the (A) Anorexi nervos
hu n ou th. They su sp ect the p tient w s
(B) Anxiety d isord er
injecting s liv into her knee. You ev lu te the
p tient nd find her to be p le s nt nd coop - (C) Bod y d ys orp hic d isord er
er tive. She tells you th t she h s h d very (D) Bu li i nervos
tough ti e l tely bec use her husb nd h s (E) MDD
recently been sick. Fortu n tely, she is nu rse
nd h s been ble to c re for hi t ho e. 54. Which of the following tests would be ost help-
L tely, she d its to feeling overw hel ed ful in supporting your provision l di gnosis?
nd not pp reci ted . She h s no id e w h t h s
(A) ECG
c used the proble w ith her knee. You t lk to
the f ily nd they tell you the p tient is in (B) Seru yl se
no fin nci l d ifficu lty nd continu es to enjoy (C) Seru gnesiu level
w ork s nu rse. After w orking closely w ith (D) Seru p ot ssiu level
you for sever l w eeks, she eventu lly d its (E) Weight nd height
to injecting her knee lthou gh c nnot u nd er-
st nd w hy she d id it. Which of the follow ing 55. A 5-ye r-old boy is referred to you by his ped i-
is the ost p p rop ri te d i gnosis? trici n for ev lu tion of ggressive beh vior.
(A) Conversion d isord er H is ed ic l nd extensive neu rologic w orku p
w s neg tive. An interview w ith the p tient
(B) F ctitiou s d isord er
reve ls restless boy w ho is ble to eng ge in
(C) Illness nxiety d isord er convers tion. H e tells you he gets ngry nd
(D) M lingering fru str ted in school s he thinks its boring.
(E) So tic sy p to d isord er H is p rents rep ort th t he is cu rrently rep e t-
ing kind erg rten d u e to p oor p erfor nce
Questions 53 and 54 nd d ifficu lty soci lizing w ith other child ren.
H is ggressive ou tbu rsts t school see to
An 18-ye r-old w o n in her first ye r of college occu r t ti es w hen he d oes not und erst nd
co es to see you for ev lu tion of d epression, fter the schoolw ork. H is other tells you th t she
her roo tes encour ged her to seek help . She still h s to help hi pick ou t his clothes for
rep orts d ifficu lty f lling sleep nd e rly orning school nd get d ressed . H is f ily history is
w kenings, poor concentr tion, f tigue, nd nxi- p ositive for tw o p tern l u ncles w ith le rn-
ety over the p st onth since rriving for the f ll ing d is bilities. Which of the follow ing tests
se ester. She tells you th t her p rents re very strict w ou ld be ost help fu l in the ev lu tion of
nd she is w orried th t she w ill not get str ight this p tient?
176 6: Diffe re ntia l Dia g nos is a nd Ma na g e me nt

(A) Good enough-H rris Dr w -A-Person p resents w ith od d beh vior, confu sion,
Test blood p ressure of 128/ 85 H g, nd he rt
(B) Kohs Block Test r te of 68 be ts/ in. H is p rents rep ort th t
(C) Minnesot Mu ltip h sic Person lity t ti es they h ve observed hi repetitively
Inventory-2 (MMPI-2) tou ching his sto ch nd r pid ly blinking his
eyes. On ex in tion, he p p e rs d zed nd
(D) Pe bod y Voc bu l ry Test
u n ble to concentr te. Which of the follow ing
(E) Wechsler Intelligence Sc le for Child ren is the ost likely d i gnosis?
(WISC)
(A) A p het ine toxicosis
Questions 56 and 57 (B) Ep ilep sy
(C) H yp oglyce i
An 8-ye r-old boy is brou gh t in to you r office
by h is oth er bec u se sh e is con cern ed th t h e (D) MDD
c nnot stop blinking his eyes nd shru ggin g his (E) So tic sy p to d isord er
sh ou ld ers. Sh e tells you th t this occu rs l ost
every d y n d h s been th is w y for the p st ye r 59. A 50-ye r-old n w ith long history of IV
nd h lf. Sh e tells h er son to stop d oin g this d rug u se is brought to the hosp it l by p olice
w h ile h e is in you r office, n d h e d oes stop . You fter loc l ho eless shelter w orker noted
notice, h ow ever, th t h e st rts b rkin g n d cou gh- hi to be confu sed nd w lking funny.
in g rep e ted ly. While t the shelter, he bec e very su spiciou s
of the w orkers nd ccu sed the of t king his
56. Which of the follow ing ed ic tions w ou ld be belongings. On ex in tion, you observe th t
the ost ppropri te in tre ting the p tients he h s left p u p il th t cco od tes but d oes
cond ition? not re ct, d epressed d eep tend on reflexes in
ll d istribu tions, nd loss of p osition sense t
(A) Ato oxetine the gre t toes bil ter lly. Althou gh the p tient
(B) Clonid ine w s tre ted w ith ntip sychotic ed ic tions in
(C) L otrigine the p st, he d enies cu rrently t king ny ed i-
(D) Lor zep c tion. Which of the follow ing d i gnoses best
(E) V lp roic cid ccou nts for this p tients sy p to s?
(A) Antip sychotic-ind u ced d yskinesi
57. You begin the p p rop ri te ed ic tion nd
(B) Kors koff p sychosis
2 onths l ter the boy nd his other retu rn
(C) N eu rolep tic lign nt synd ro e
for follow -u p visit. She tells you th t w hile
the eye blinking nd should er shrugging h ve (D) N eu rosyp hilis
gotten better, he is now f lling sleep in his (E) Wernicke encep h lop thy
cl sses t school. Wh t is the next ost p pro-
p ri te step in the n ge ent? 60. A 57-ye r-old w o n w ith no p revious p sy-
chi tric history co pl ins of incre sing nxi-
(A) Ad d nother ed ic tion to keep the boy ety over the l st 2 onths. Tod y she reports
w ke. th t it bec e very b d . She lso notes th t
(B) Ed u c te the other bou t p rop er sleep w ith these p eriod s of nxiety she gets p ou nd -
hygiene. ing he d che nd once f inted . She continu es
(C) Discontinu e the ed ic tion. to feel sh ky. When you check her vit l
(D) Red u ce the d os ge. signs, her he rt r te is 170 be ts/ in nd her
(E) Sw itch to nother ed ic tion. blood p ressu re is 230/ 130 H g. She is d i -
p horetic nd tre u lou s. Given this p tients
58. A 17-ye r-old boy w ith h istory of tten - sy p to s, w hich of the follow ing cond itions
tion-d eficit/ h yp er ctivity d isord er (ADH D) is ost likely?
Que s tions : 5666 177

(A) Acu te lcohol intoxic tion incre singly te rfu l, irrit ble, restless, s w ell
(B) H yperc lce i s h ving d ifficu lty p ying bills nd b l nc-
(C) H yp othyroid is ing her checkbook. She h s occ sion l hot
fl shes. On ex in tion, she continuou sly
(D) Pheochro ocyto
ch nges position in her se t. She h s d ifficulty
(E) Posterior circu l tion stroke w ith si p le rith etic nd short-ter e -
ory. H er d eep tend on reflexes re brisk sy -
D IRECTION S (Questions 61 through 66): For each etric lly throu ghou t. She h s fine resting
patient vignette, select the one lettered option tre or of her h nd s nd d ifficu lty rising fro
that is most closely associated w ith it. Each let- se ted p osition.
tered option may be used once, multiple times, or
not at all.
64. A 33-ye r-old w o n p resents w ith 1-ye r
(A) Conversion (fu nction l neu rologic l
history of tension he d ches, recently w ors-
sy pto ) d isord er
ening nd now ssoci ted w ith blu rry vision.
(B) H ep tic enceph lop thy So f r, n extensive neu rologic ev lu tion h s
(C) H u n i u nod eficiency viru s (H IV) been u nreve ling. She volu nteers th t she h s
(D) H yp erthyroid is been sick ost of her life, beginning t ge 16
(E) H yp oglyce i w hen she h d su rgery for p resu ed end o-
(F) Pheochro ocyto etriosis, w hich h s left her u n ble to h ve
sexu l rel tions. Since the su rgery, she h s
(G) So tic sy p to d isord er
h d inter ittent bd o in l cr ping, blo t-
(H ) Syste ic lu p u s erythe tosu s (SLE) ing, nd d i rrhe . Review of her ch rt show s
th t 3 onths go, she s w rheu tologist
61. A 55-ye r-old n is d itted to the hosp it l. for knee, b ck, nd eye p in.
H is w ife reports th t hes ch nged over the
l st ye r, h s beco e very forgetfu l, nd
h s period s w hen he beco es very u p set. 65. A 30-ye r-old w o n is brou ght to the hos-
On ex in tion, he h s signific nt e ory p it l by her fi nc. H e s ys th t over the l st
i p ir ent, sterixis, p l r erythe , nd 2 w eeks, she h s been tot lly d ifferent p er-
l rge ecchy otic re on his right sc p u l . son. She h s not been e ting or sleeping, nd
H e w s initi lly cooper tive, bu t now is very h s been irrit ble nd ngry. E rlier in the
git ted nd d e nd s to le ve. onth, she co pl ined of he d ches, p in in
her h nd s nd feet, nd fever. On ex in -
tion, she looks tired ; she is fully oriented bu t
62. A 52-ye r-old n w ith history of IV d ru g h s d ifficu lty rel ting the events of the l st
u se is brou ght to the e ergency d ep rt ent onth. Occ sion lly, she see s to beco e
by his soci l w orker fro ho eless shelter. confused .
She h s know n the p tient for 10 ye rs bu t h s
seen d r stic ch nge in hi over the l st ye r.
While previou sly jovi l nd inter ctive, he is 66. A 42-ye r-old w o n in the e ergency d ep rt-
now d iseng ged nd subd u ed . H e h s been ent pp e rs confu sed nd w ill not llow
incre singly forgetfu l, tod y h ving d ifficu lty blood to be d r w n. She is not oriented to ti e
u sing his e ting u tensils. H e h s lost t le st or pl ce. She t kes ed ic tions for cond i-
40 lb in the l st 6 onths nd co pl ined tion, bu t c nnot el bor te fu rther. The only
bou t feeling w e k nd losing his b l nce. H e vit l signs th t w ere t ken show he rt r te
scores 20/ 30 on the MMSE. of 140 be ts/ in nd blood p ressure of 172/
98 H g. Physic l ex in tion is re rk-
ble for fine tre or of her h nd s bil ter-
63. A 33-ye r-old w o n is brou ght to the e er- lly, nd d i phoresis. Five inu tes l ter, the
gency d ep rt ent by her hu sb nd , w ho tells p tient h s seizu re.
you th t over the l st few onths she h s been
178 6: Diffe re ntia l Dia g nos is a nd Ma na g e me nt

DIRECTIONS (Questions 67 through 72): For each 70. An 86-ye r-old fe le is brou gh t to th e ED
patient vignette, select the one lettered option that is fter the nu rsing ho e st ff noticed su d d en
most closely associated w ith it. Each lettered option ch nge in h er beh vior over the l st sever l
may be used once, multiple times, or not at all. d ys. On history nd p hysic l ex in tion, it
(A) Deliriu is d ifficu lt to elicit infor tion s the p tient
(B) MDD (p seu d od e enti ) keep s f lling sleep . H er vit l signs show
te p er tu re of 101.2F n d he rt r te of
(C) M jor neu rocognitive d isord er d u e to
101 be ts/ in. Urin lysis is p ositive for
Alzhei er d ise se
ketones.
(D) M jor neu rocognitive d isord er d u e to
frontote por l lob r d egener tion
(Pick d ise se) 71. A 59-ye r-old fe le is brou ght in by her son
w ith w ho she lives. H e st tes th t over the
(E) M jor neu rocognitive d isord er d u e
p st cou p le of onths his other h s been
to H u ntington d ise se
isol tive, no longer sp end s her ti e re d -
(F) M jor neu rocognitive d isord er d ue to
ing w hich she u sed to enjoy. She w ill re in
Lew y bod y d ise se
w ke l te t night nd e rly in the orning,
(G) M jor neu rocognitive d isord er d u e nd her p p etite h s d ecre sed . H e is lso con-
to Prion (Creutzfeld tJ cob) d ise se cerned bec use she w ill so eti es re e ber
(H ) M jor v scu l r neu rocognitive d isord er inor d et ils fro the d y before, bu t other
ti es w ill not rec ll w h t she h d for bre k-
67. A 71-ye r-old fe le is brou ght to the ER by f st. Ment l st tu s ex in tion is signific nt
her hu sb nd . Over the p st few ye rs, he h s for incre sed sp eech l tency, d ep ressed ood
noticed th t she h s been h ving e ory nd ffect, nd trou ble rec lling 2/ 3 ite s
p roble s. She often forgets w here she p u t fter sever l inu tes.
her keys or pu rse. H e no longer lets her d rive
bec u se she gets lost e sily. On MMSE, she h s
72. A 39-ye r-old le w ith no p reviou s p sychi-
p rticul r p roble s w ith the d y of the w eek,
tric history p resents w ith co p l ints of feel-
d te, three-ite rec ll, nd n ing pencil.
ing very d ep ressed l tely. H is f ily notes
th t he h s been u ch ore irrit ble recently,
68. A 68-ye r-old le w ith no p sychi tric history often yellin g w ith little p rovoc tion nd
is brou ght by his d u ghter bec u se, d esp ite see ingly t lking to hi self. On MSE, he is
u ltiple re ssu r nces, he continu es to insist noted to h ve qu ick, su d d en, bu t involu nt ry,
so eone else is in the house. The p tient st tes jerking ove ents of his r s.
he sees s ll n every d y ju st insid e of
the d oor. H is d ughter h s lso noticed th t
D IRECTION S (Questions 73 through 98): For each
he h s proble s w ith his e ory. On ex i-
of the multiple-choice questions in this section,
n tion, he h s shu ffling g it nd blu nted
select the lettered answ er that is the one best
ffect, w hich the d ughter st tes beg n t the
response in each case.
s e ti e s the visu l h llu cin tions.
73. A 56-ye r-old n p resents to you r office t
69. A 55-ye r-old le is brou ght by his w ife the requ est of his w ife, w ho s ys th t he d rinks
bec u se she h s noticed ch nges in his beh v- too uch. Wh t w ould be the ost i port nt
ior. L tely, his food p references h ve ch nged str tegy in ev lu ting this p tient for lcohol
nd he is now cu rsing lou d ly in pu blic. On u se d isord er?
ex in tion, he is e sily git ted . N eu rologic
ex in tion d e onstr tes p ositive B binski (A) Ascert in how often he d rinks.
resp onse nd snou t reflex. An MRI show s (B) Ask hi how frequ ently he gets d ru nk.
trophy in the front l nd te p or l lobes. (C) Ask hi w h t his f ily nd friend s
s y bou t his d rinking.
Que s tions : 6778 179

(D) Perfor co plete l bor tory Questions 77 and 78


investig tion.
A 55-ye r-old le w ith history of lcohol u se
(E) Qu ntify the ver ge ou nt he d rinks.
d isord er presents to you r office bec use he w ould
like to stop d rinking lcohol. H e believes he d rinks
74. A 42-ye r-old d ivorced le p resents to you r
bec u se he is d ep ressed nd w ishes to lso get tre t-
office st ting th t for the p st 7 onths he
ent for his d epression. After fu rther history is
h s been w orrying bout fin nci l nd w ork-
obt ined , it is reco end ed th t the first step is to
rel ted p roble s. H e st tes th t ore often
bst in fro ll lcohol s it c n c u se d ep ression.
th n not, he is w orrying bou t these issues
H e grees to this pl n.
throu ghou t the d y. H e h s d ifficu lt ti e
st ying sleep, he feels tired ll d y long,
77. Wh t is the likelihood of his re ining
nd he c n no longer concentr te d equ tely
d ep ressed if he is ble to refr in fro u sing
t w ork d u e to the w orrying. H e lso tells
lcohol for 1 onth?
you th t he feels very tense in his shou ld er
nd b ck. Which of the follow ing is the ost (A) 5%
p propri te d i gnosis? (B) 15%
(A) Depend ent person lity d isord er (C) 25%
(B) Gener lized nxiety d isord er (GAD) (D) 33%
(C) M jor d ep ressive d isord er (MDD) (E) 50%
(D) P nic d isord er
78. The bove p tient retu rns fter 6 w eeks of
(E) Soci l nxiety d isord er
int ining sobriety fro lcohol. H e tells
you th t he h s continu ed to feel d ep ressed .
Questions 75 and 76
H e h s p roble s sleep ing t night, feels tired
You re interview ing 54-ye r-old rried w o n throughou t the d y, h s poor pp etite, nd he
w ho h s been u rged to see shrink by her f - no longer d erives ple su re fro pl ying w ith
ily. She d escribes sy p to s of feeling ineffectu l, his d ogs? Which of the follow ing ed ic tions
believing th t the w orld is lw ys hostile to her, nd w ou ld be the ost p p rop ri te to begin t this
know ing th t things w ill never ch nge. ti e?
(A) A itrip tyline (El vil)
75. This tri d of sy p to s is ost ssoci ted
(B) Bu sp irone (Bu Sp r)
w ith w hich of the follow ing d isord ers?
(C) Lithiu (Esk lith)
(A) Dep ressive d isord ers (D) Lor zep (Ativ n)
(B) Dissoci tive d isord er (E) Sertr line (Zoloft)
(C) GAD
(D) P nic d isord er Questions 79 and 80
(E) Schizop hreni
A 52-ye r-old le w ith longst nd ing history of
lcohol use d isord er is brought into the ED intoxi-
76. Which of the follow ing tre t ents w ou ld
c ted w ith lcohol. B sic l bs re d r w n nd he is
ost likely t rget these sy p to s?
found to be d ehyd r ted , so he is given IV fluid s
(A) Beh vior l ther p y long w ith glu cose. Abou t n hou r l ter, the physi-
(B) Cognitive ther p y ci n reev lu tes the p tient nd find s th t his speech
(C) Cou p les ther p y is ore slurred , he is very confused , nd he now h s
nyst g us.
(D) Interp erson l ther p y
(E) P r d oxic l ther p y
180 6: Diffe re ntia l Diag nos is a nd Ma na g e me nt

79. Which of the follow ing is the next ost (A) Ad ju st ent d isord er
p propri te step in the n ge ent of this (B) Bip ol r d isord er
p tient? (C) MDD
(A) Ad inister ntibiotics. (D) Persistent d ep ressive d isord er
(B) Ad inister benzod i zep ine. (E) Schizop hreni
(C) Ad inister hep rin.
(D) Ad inister thi ine. 83. A 45-ye r-old n is in the e ergency dep rt-
ent bec use of di betic foot ulcer. In g ther-
(E) Ad inister tissu e p l s inogen
ing history, the physici n le rns th t this n
ctiv tor.
lives lone nd works nights s security gu rd.
He s ys he h s no friends but th t this does not
80. After d in istr tion of th e correct ed ic -
bother hi . He h s never been hospit lized
tion the p tien t i p roves signific ntly. H is
or received ny psychi tric help. On MSE, his
f ily rrives th e n ext orn in g n d th ey
ffect is fl t. Although he nswers questions
tell you th t he h s recen tly been st rting
nd see s to trust the judg ent of the doctors,
to ke u p f cts bou t h is life. You ord er
he h s little interest in the interview. He exhibits
n MRI of his br in nd d iscover lesion s in
no signs or sy pto s of psychosis or depres-
h is ill ry bod ies. Wh ich of th e fol-
sion. Wh t is the ost likely di gnosis?
low in g d i gn oses w ou ld best exp l in th ese
sy p to s? (A) MDD
(A) Alcohol-ind u ced jor neu rocognitive (B) P r noid p erson lity d isord er
d isord er (Kors koff synd ro e) (C) Schizoid p erson lity d isord er
(B) Conversion d isord er (D) Schizop hreni
(C) F ctitiou s d isord er (E) Schizotyp l p erson lity d isord er
(D) M jor neu rocognitive d isord er d u e to
Alzhei er d ise se 84. The id entified p tient is 30-ye r-old sep -
r ted fe le brou ght into the e ergency roo
(E) Wernicke encep h lop thy
by her id entic l tw in sister. The p tients his-
tory is not ble for prior episod e of d epres-
Questions 81 and 82
sion 5 ye rs go su ccessfu lly tre ted w ith
A 27-ye r-old n co p l ins th t he h s felt d ow n venl f xine. The p tient h s been st ying
in the d u ps for onths nd is feeling gu ilty w ith her sister since her sep r tion 1 onth
bec u se he h s been h ving n extr rit l ff ir. go. For the p st 2 w eeks, she h s been p cing
In recent w eeks, he h s st rted to believe th t his rou nd the hou se, not sleep ing ore th n 2 to
w ife is p oisoning his food nd the rest of his f ily 3 hou rs per night. Desp ite her feeling s d
is involved in n el bor te plot to d rive hi fro i ed i tely fter the sep r tion, the p tient
the hou se. now feels w ond erfu l, like I c n cco p lish
nything! In f ct, she h s been tte p ting to
81. Assu ing his thinking is d elu sion l, how re od el her sister s b throo , even thou gh
w ou ld his d elu sions be best ch r cterized ? she h s no tr ining or experience. H er sister
h s been extre ely concerned bou t her, bu t
(A) Biz rre she h s been u n ble to t lk to her bout it s,
(B) Ego-syntonic I c nt get w ord in ed gew ise, nd she
(C) Mood congru ent d oesnt lw ys ke sense. The p tient is
(D) Mood incongru ent only t king or l contr ceptives nd o epr -
(E) So tic zole for cid reflux. H er sister is concerned
th t she, herself, y eventu lly d evelop this
82. Which of the follow ing d i gnoses w ou ld ost illness. Wh t is her p p roxi te risk of d evel-
likely be resp onsible for the bove d elu sions? oping this d ise se?
Que s tions : 7989 181

(A) 0% to 10% ed ic tions w ou ld be the best choice for this


(B) 10% to 20% p tient?
(C) 20% to 30% (A) Alp r zol (X n x)
(D) 50% to 70% (B) Cod eine (Brontex)
(E) 80% to 90% (C) Dip henhyd r ine (Ben d ryl)
(D) Qu eti p ine (Seroqu el)
85. A 33-ye r-old w o n w ith history of
(E) Zolp id e (A bien)
schizop h reni tells you she h s been h e ring
voices for the p st 5 ye rs. Sh e h s b se-
88. An 8-ye r-old girl p resents to her p ed i tri-
line u d itory h llu cin tion th t she c nnot
ci n w ith her p rents second ry to incre sed
u nd erst n d ost of the ti e. Du ring p eri-
p roble s t school. She recently entered the
od s w here the voices beco e w orse, she
third gr d e nd h s been h ving d ifficu lties
typ ic lly he rs co nd u d itory h llu ci-
w ith beh vior in cl ss. At ho e, her p r-
n tions. Fro s fety st n d p oint, w hich of
ents d escribe her s ctive nd lw ys ov-
the follow ing is the ost i p ort nt infor-
ing fro one ctivity to the next. At school,
tion to obt in reg rd ing her u d itory
the child is u n ble to sit still in her ch ir nd
h llu cin tions?
frequ ently interru p ts the lessons. On ex i-
(A) H ow long she h s been he ring voices. n tion, you notice th t she is fid gety nd
(B) H ow lou d the voices re. const ntly p l ying w ith her h nd s. When
(C) Wh t the voices re s ying. she tte p ts to co p lete t sk she is e sily
d istr cted . Which of the follow ing w ou ld be
(D) Whether she recognizes the voices.
the ost effective p h r cother p y for this
(E) Whether the voices co e fro insid e or p tient?
ou tsid e her he d .
(A) Clonid ine
86. A 38-ye r-old n co p l ins th t for the (B) I ip r ine
p st 2 ye rs he h s requ ired sever l n p s (C) Methylp henid te
over the cou rse of the d y; he find s the n ps (D) Risp erid one
qu ite refreshing, but sees his d octor bec use
(E) Sertr line
l tely, s he is w king u p , he feels o en-
t rily p r lyzed . H e is cu rrently not on ny
89. A 30-ye r-old le co es to n urgent c re
ed ic tions. H e d enies the u se of ny illicit
f cility 8 hou rs fter the onset of u scle ches,
su bst nces. Which of the follow ing is the ost
ru nny nose, runny eyes, d ifficu lty sleeping,
likely d i gnosis?
nd loose stools th t he ttribu tes to flu
(A) Circ d i n rhyth sleep w ke d isord er, viru s. H e lives in d ifferent city nd is visit-
d v nced sleep -ph se type ing for conference. H e rep orts th t he forgot
(B) Circ d i n rhyth sleep w ke d isord er, his ed ic tions t ho e. H e t kes orphine
d el yed sleep -p h se typ e IR every 6 hou rs for chronic low b ck p in,
(C) H yp erso nolence d isord er d ip henhyd r ine occ sion lly to help w ith
sleep , venl f xine 150 g d ily for d ep res-
(D) N rcolepsy
sion nd Lisinopril for blood pressu re control.
(E) P r so ni On ex in tion, he is sw e ting, y w ning, h s
d il ted pu pils nd is ss ging his u scles.
87. A 45-ye r-old le w ith history of op i te H is blood p ressu re is 160/ 95 H g, pu lse
u se d isord er p resents to you r office co p l in- of 102 bp , nd he is febrile. When sked to
ing of p roble s w ith sleep . H e cl i s th t cont ct his pri ry c re p hysici n he beco es
he h s been sober fro heroin for the p st incre singly git ted , st ting they w ou ld not
10 ye rs. H e sks to be st rted on ed ic - contribute ny infor tion to this visit. L b
tion for his inso ni . Which of the follow ing v lues re ll w ith in nor l li its. Which
182 6: Diffe re ntia l Diag nos is a nd Ma na g e me nt

of the follow ing ost likely exp l ins his etfor in, u ltivit in, n OTC herb l su pp le-
sy pto tology? ent, tr d ol for chronic b ck p in, nd recent
ntibiotic prescription for linezolid . H is vit l signs
(A) Anticholinergic toxicity
re: BP of 160/ 92, he rt r te of 110 bp , nd te -
(B) H ypertensive crisis p er tu re of 101.2F. On ex in tion, he d i p horetic
(C) Influ enz nd h s clonu s in his bil ter l low er extre ities.
(D) Op i te w ithd r w l
(E) Serotonin synd ro e 92. Which of the follow ing is his ost likely
d i gnosis?
Questions 90 and 91
(A) Bipol r d isord er
A 28-ye r-old fe le w ith p st ed ic l history (B) H ypertensive crisis
of hypothyroid is nd syste ic lu pu s erythe - (C) Migr ine w ith u r
tosus (SLE) is hospit lized for lupu s fl re requ ir- (D) N eu rolep tic lign nt synd ro e (N MS)
ing IV steroid s. You re consu lted for the p tient
(E) Serotonin synd ro e
s she is co pl ining of ud itory h llu cin tions
nd h s been beco ing incre singly git ted . H er
93. Which of the follow ing is the best next step in
ed ic tions inclu d e IV ethylp red nisone, u lti-
the n ge ent of this p tient?
vit in, nd levothyroxine. On ex in tion, the
p tients ood is l bile nd she end orse u d itory (A) Continu e ll ho e ed ic tions.
h llucin tions. H er vit l signs re st ble. H er TSH is (B) Discontinu e linezolid , tr d ol,
0.7 IU/ L nd her T4 is 7.2 u g/ d L. p henelzine.
(C) Discontinu e etfor in.
90. Which of the follow ing is the ost likely
(D) Discontinu e over the cou nter herb l
d i gnosis?
sup ple ent.
(A) Bipol r d isord er d u e to hyperthyroid is (E) Incre se risp erid one.
(B) Brief psychotic d isord er
(C) MDD w ith p sychotic fe tu res 94. A 58-ye r-old le w ith no p st ed ic l his-
(D) Schizop hreni tory is brou ght to the d octor s office by friend .
(E) Steroid -ind u ced p sychotic d isord er The friend rep orts the p tient h s ccu u -
l ted too u ch stu ff in his one-story ho e
91. Which of the follow ing is the best next step in nd h s not been ble to le ve his ho e. The
the n ge ent of this p tient? p tient cl i s th t he y need these ite s t
so e point in the futu re nd refu ses to throw
(A) Decre se steroid d os ge nything w y. When sked if he h s ever
(B) Incre se levothyroxine tried to cle n out his hou se he gets d efensive,
(C) Initi te ECT fid gety nd st tes he cou ld bsolutely never
(D) Initi te lithiu throw nything ou t! H is friend brings in p ic-
tu res of the p tients ho e show ing ound s
(E) Initi te sertr line
of ite s in every roo . The p tient h s been
u ne p loyed for 1 ye r, expl ining th t he
Questions 92 and 93
need s the ti e to t ke c re of his ho e. Which
A 38-ye r-old n w ith p st ed ic l history of of the follow ing is the ost likely d i gnosis?
chronic low er b ck p in nd hypertension presents (A) GAD
w ith friend to the e ergency roo co p l ining of
(B) H o rd ing d isord er
git tion nd he d che for 2 d ys. Upon fu rther
qu estioning, he d its to p st p sychi tric history (C) OCD
of MDD w ith psychotic fe tures. Soon fter p resen- (D) Obsessive-co pu lsive person lity
t tion he beco es confu sed nd nonresponsive. H is d isord er (OCPD)
ed ic tion list inclu d es p henelzine, risp erid one, (E) P nic d isord er
Que s tions : 90100 183

95. A 31-ye r-old fe le is fou nd in D ll s, Tex s 98. A 35-ye r-old le is u rged to go to psy-
sitting in coffee shop for n entire d y. When chi trist by cow orker bec u se of frequent
sked to le ve the coffee shop u p on closing, nger ou tbu rsts. H e rep orts th t s long s
the p tient c nnot rec ll ny id entifying infor- he c n re e ber he h s h d hot te p ter.
tion or w here she is fro . The p olice re L tely, he h s been getting into to trou ble t
c lled , w ho escort her to n e ergency d ep rt- w ork d u e to u ltip le verb l confront tions
ent. When se rched , her ID c rd is fou nd w ith cow orkers, w hich occ sion lly esc l te
in her p u rse, d ocu enting th t the p tient is to the p tient throw ing v rious office objects
fro Michig n. Fu rther investig tion reve ls cross the roo . H e feels very gu ilty fter, but
her f ily rep orted her issing 2 w eeks go. he is u n ble to p revent his beh vior. H e h s no
Of note the p tient w s d op ted t you ng signific nt leg l proble s otherw ise. Which of
ge second ry to child bu se. Wh ich of the the follow ing is the ost likely d i gnosis?
follow ing is the ost likely d i gnosis?
(A) Antisoci l p erson lity d isord er
(A) Bord erline p erson lity d isord er (B) Bip ol r II d isord er
(B) Dep erson liz tion d isord er (C) Bord erline p erson lity d isord er
(C) Dissoci tive nesi (D) Cond u ct d isord er
(D) Dissoci tive id entity d isord er (E) Inter ittent exp losive d isord er
(E) M jor neu rocognitive d isord er d u e to
Alzhei er d ise se D IRECTION S (Questions 99 through 108): For
each of the follow ing patients, choose the most
Questions 96 and 97 appropriate diagnosis. Each lettered option may
be used once, multiple times, or not at all.
A 26-ye r-old fe le p resents to her pri ry c re
physici n w ith d ifficu lty sleep ing. She co pl ins (A) Antisoci l p erson lity d isord er
of const ntly fid geting in bed w ith const nt urge
(B) Avoid nt p erson lity d isord er
to ove her legs. She gets u p nd w lks rou nd in
ord er to help relieve it. This occu rs ost nights of (C) Bord erline p erson lity d isord er
the w eek nd h s gre tly interfered w ith her bil- (D) Dep end ent p erson lity d isord er
ity to sleep . Physic l ex in tion is neg tive for ny (E) H istrionic p erson lity d isord er
bnor lities. H er l b w ork show s low MCV, low (F) N rcissistic p erson lity d isord er
ferritin, nd he oglobin of 9.0 g/ d L. (G) Obsessive-co p u lsive p erson lity
d isord er
96. Which of the follow ing is the ost likely
(H ) P r noid p erson lity d isord er
d i gnosis for this p tient?
(I) Schizoid person lity d isord er
(A) GAD (J) Schizotyp l person lity d isord er
(B) Inso ni d isord er
(C) N on-REM sleep rou s l d isord er 99. A 45-ye r-old w o n recently is rele sed fro
(D) Obstru ctive sleep p ne p rison for r ed robbery. She h s nu erous
(E) Restless legs synd ro e j il sentences for ste ling, bribery, nd rson.
She end orses th t she s w p sychi trist s
97. Which of the follow ing is the ost p p rop ri- child for beh vior l proble s, getting into
te step in the tre t ent of this p tient? trou ble t school nd t ho e. Upon further
questioning, she rec lls p eriod of ti e
(A) Initi te hyd rocod one. torturing her f ilys p et c t s child .
(B) Initi te iron rep l ce ent.
(C) Initi te lor zep . 100. A 57-ye r-old n presents to his pri ry
(D) Initi te el tonin. p hysici n w ith co p l ints of bd o in l p in.
(E) Initi te risp erid one. H e is cco p nied by his other, w ith w ho
184 6: Diffe re ntia l Diag nos is a nd Ma na g e me nt

he lives. The p tient ppe rs to d efer to his see the Chief of the d ep rt ent, nd how he
other to nsw er ny of the qu estions d u r- c nnot be bothered by low -level e ployees.
ing the interview. The p tient t lks bou t how
he c lls his other throughou t the w orkd y 105. A 46-ye r-old n is reco end ed to see
to sk her trivi l qu estions nd so she c n help e p loyee he lth s he continu ou sly believes
hi ke ny, everyd y d ecisions. H e h s his colle gu es t w ork re ou t to s bot ge
recently been ore nxiou s s he fe rs th t she his e p loy ent. As resu lt, he h s w ritten
is old er nd y d ie soon, w hich w ou ld thre tening e ils to his peers nd su pervi-
le ve e lone, help less, nd u n ble to cop e. sors. When confronted w ith the f ct th t he
h s been invited to t ke p rt in soci l ctivi-
101. A 30-ye r-old n w orks in lighthou se d u r- ties w ith others, he st tes th t he is being
ing the overnight shift by hi self. H e h s tricked into co p l cency nd pl ns to sue
no friend s, prefers to be lone in his sp re the org niz tion.
ti e, nd d oes not d esire to ke person l
or inti te rel tionships. H e presents d ue to 106. A 21-ye r-old w o n w ho is in her third ye r
incre sed stress now th t he h s cow orker t of college p resents to ther p ist u p set th t
his job. she is u n ble to ke friend s. She exp resses
th t she w ou ld like to join school clu b to
102. A 34-ye r-old w o n h s just gone throu gh eet new p eop le nd ke friend s bu t she is
her second d ivorce nd is cu rrently in too shy. She d oesnt go ou t to p rties d esp ite
tu u ltu ous rel tionship w ith n she et being sked , bu t they w ou ld nt like e
1 onth go. H er ex in tion reve ls sc rs nyw y.
on her w rist, w hich she expl ins re fro cut-
ting herself w hen I get ngry or up set. She 107. A 34-ye r-old w o n is new ly d i gnosed w ith
often feels e pty insid e bu t d its to intense hyp ertension. Upon interview, she d its
ch nges of ood , qu ickly beco ing d espon- th t she st ys l te t her job on regul r b sis
d ent or ngry. She reports it is ll her f u lt she bec use she spend s n excessive ount of
feels this w y. ti e on e ch project, ensuring everything is
in ex ct ord er nd properly for tted . She
103. A 45-ye r-old w o n presents to her p ri ry triple-checks not only her ow n but ll of her
c re ppoint ent p rovoc tively d ressed for colle gues w ork. She d oesnt believe th t oth-
rou tine ex in tion. She exu ber ntly exp l ins ers re c rrying their lo d but believes she is
her cu rrent rel tionship w ith so eone she h s the only one w ho excels t her job. She is d is-
et only online nd not in person. She goes ppointed , how ever, bec use her fellow col-
into gre t d et il reg rd ing his erits, cl i ing le gues re ll ble to le ve t the ppropri te
th t he is the one. H ow ever, t the end of the ti e.
interview she sks the p hysici n if he is single.
108. A 32-ye r-old w o n p resents to her OB/
104. A 32-ye r-old n p resents to the e ergency GYN for routine c re. She is noted to be
d ep rt ent fter suffering l cer tion to his d ressed w ith ulticolored sc rves, b ngles,
left ind ex finger. When ed ic l stu d ent nd l rge nu ber of cryst l jew elry. She
tte p ts to ev lu te hi he rebu kes hi , st tes she is p l re d er s profession,
excl i ing I only t lk to ttend ing p hysi- believing she h s the tools to u nlock p er-
ci ns. H e is noted to be t lking lou d ly on his sons fu tu re. She c e to her p point ent
telephone bout how su ccessful he h s been fter re d ing her strologic l forec st, bu t w s
in the fin nci l rket, how he is su re he w ill concerned bec u se of the color of y ur .
Ans we rs a nd Expla na tions

1. (C) The ost likely d i gnosis in this c se is 3. (C) The p tient ost likely h s d i gnosis
schizo ffective d isord er. The p tient h s pro - of jor d ep ressive d isord er, w ith p erip r-
inent psychotic sy pto s, includ ing ud itory tu onset nd p sychotic fe tu res, since the
h llu cin tions, w hich re ru nning co en- d ep ression st rted w ithin 4 w eeks of giving
t ry of her beh viors, bu t lso concu rrent birth to her son. H e ring child crying is
ood sy p to s, includ ing d epressed ood , co on p sychotic sy p to of p ostp rtu
d ecre sed sleep, nhed oni , nd poor ppetite. d ep ression. Althou gh the p tient h s been
Also i p ort nt, the p tients h llu cin tions h ving p sychotic sy p to s, d i gnosing her
h ve occurred in the bsence of ood sy p- w ith brief p sychotic d isord er w ou ld not
to s, nd her ood sy pto s ppe r to h ve fu lly ccou nt for her ood sy p to s. In
been present for jority of the tot l d ur - ord er to eet the d i gnosis of schizo ffec-
tion of her illness. The presence of pro inent tive d isord er, p tient u st h ve p sychotic
psychotic sy p to s in the bsence of ood sy p to s d u ring p eriod w hen there re no
sy pto s kes jor d epression w ith psy- ood sy p to s p resent. Ad ju st ent d is-
chotic fe tures unlikely. A d i gnosis of schizo- ord er w ou ld not ccou nt for her signific nt
phreni lone w ou ld not d equ tely ccou nt ood or p sychotic sy p to s. Postp rtu
for this p tients ood sy pto s. Bipol r II obsessive-co p u lsive d isord er w ou ld u su -
d isord er is not v lid choice since there is no lly p resent s other w ith signific nt
cle r history of hypo nic episod e. Schizoid ou nt of nxiety d u e to ego d ystonic, obses-
person lity d isord er is unlikely bec u se th t sive thou ghts.
d i gnosis w ould not ccount for the p tients
overt psychotic nd ood sy pto s. 4. (A) The p tient w ou ld best be tre ted by initi-
ting n ntid epress nt (su ch s n SSRI) nd
2. (E) In schizo ffective d isord er, it is i p ort nt n ntip sychotic (to d d ress her h llu cin -
to tre t both the ood nd psychotic sy p- tions). H osp it lizing this p tient w ou ld be pre-
to s. Bec u se the p tients sy pto s inly tu re bec use the other is still t king c re
consist of d ep ression nd p sychosis, co bi- of her child nd is not h ving ny thou ghts of
n tion of n ntip sychotic nd ntid epress nt w nting to hu rt the child or herself. Lithiu
ed ic tion w ould be re son ble ppro ch. is ind ic ted s ood st bilizer for so eone
Mirt z pine, cit lopr , flu oxetine, nd ser- w ho h s d i gnosis of bip ol r d isord er. H lo-
tr line re ntid epress nt ed ic tions, w hile perid ol lone w ou ld help tre t her p sychosis,
zipr sid one, h lop erid ol, nd p erp hen zine bu t w ou ld d o nothing to d d ress her d ep res-
re ntipsychotic ed ic tions. Lor zep sion. Provid ing su pportive ther py could help
nd d i zep re benzod i zepines. Div l- the p tient, how ever, consid ering the severity
p roex sod iu is ood st bilizer nd is u sed of her illness, ed ic tions w ould need to be
for bip ol r d isord er. initi ted .

185
186 6: Diffe re ntia l Diag nos is a nd Ma na g e me nt

5. (E) This p tient presents with the sy pto s w ere longst nd ing p roble th t ffected
nd signs of Wernicke enceph lop thy, which u ltip le sp ects of the p tients life but never
results fro thi ine (vit in B1) deficiency. re ched p sychotic (d elu sion l) level. In this
It is ch r cterized by bil ter l bducens nerve c se, the only psychotic sy pto d escribed is
p lsy, horizont l nyst g us, t xi , nd bod y od or. In ord er to eet criteri for d i g-
glob l confusion cco p nied by p thy. E ch nosis of schizop hreni , the sy p to s w ou ld
of the other choices is possible nd should be l st for t le st 6 onths, the d elu sion w ou ld
ctively ruled out. An cute subdur l he - likely be biz rre, nd she w ou ld h ve other
to would typic lly present fter tr u p sychotic sy p to s su ch s u d itory h llu -
to the he d. Depending on the loc tion of the cin tions, d isorg nized speech, d isorg nized
he to , p tients c n h ve proble s with beh vior, nd / or fl t ffect.
speech, ltered ent l st tus, he iplegi , nd/
or nonre ctive pupil. Alcohol withdr w l 8. (E) Delu sion l d isord ers re typ ic lly tre ted
usu lly presents with unst ble vit l signs, tre - w ith n ntip sychotic. Of the choices listed ,
ulousness, nd git tion. Folic cid deficiency the only ntipsychotic is ol nz pine. C rb -
presents with di rrhe , cheilosis, nd glossi- zep ine is n ntiepilep tic th t is lso u sed
tis; neurologic bnor lities re usu lly not to tre t bipol r d isord er. ECT c n be used for
seen. NPH is ssoci ted with the cl ssic tri d of the n ge ent of cu te psychosis, but h s
de enti , incontinence, nd g it disturb nce. not been show n to be benefici l in the tre t-
ent of d elu sion l d isord er. Flu oxetine is n
6. (C) If Wernicke encep h lop thy is su sp ected , SSRI nd is ost co only used for d epres-
e ergent d inistr tion of IV thi ine is sive nd nxiety d isord ers. Lithiu is ood
essenti l, bec u se ny sequ el e of thi ine st bilizer nd is not used s tre t ent for
d eficiency re reversible w ith this tre t ent; d elusion l d isord er.
ist ken d inistr tion of thi ine is r rely
h r ful. A CT sc n of the he d should be 9. (D ) N MS is ch r cterized by severe (le d
ord ered i ed i tely in this c se; opti lly, p ip e) rigid ity, ch nge in ent l st tus, uto-
the IV thi ine w ould be st rted s the CT w s no ic inst bility, elev ted CPK, nd elev ted
being rr nged. The re ining choices do not w hite blood cou nt; slight elev tion in cere-
d dress the urgent need to replete thi ine. It brosp in l flu id p rotein cou nt is p ossible. N MS
should lso be noted th t giving this p tient ny y be ind u ced by ny ntipsychotic ed i-
glucose prior to d inistering thi ine c n c tions (neuroleptics), inclu d ing the typic l
result in w orsening of the Wernicke enceph - ntip sychotics. N o one sy p to is necess ry
lop thy (the et bolic conversion of glucose to for the d i gnosis; inste d , constell tion of
pyruv te requires thi ine s cof ctor). sy pto s nd their severity, in setting of
ntip sychotic exp osure, ke the d i gnosis
7. (A) Delu sion l d isord ers p resent w ith d elu - ore or less likely. Anticholinergic synd ro e,
sion (typic lly not biz rre) for t le st 1 onth. resu lting fro overd osing on nticholinergic
These p tients otherw ise h ve n org nized ed ic tions, d oes not p rod u ce rigid ity nd n
thou ght p rocess nd function w ell in other elev ted CPK. A CN S infection w ou ld be bet-
spects of their life. This p tient d oes not ter su pported by ch r cteristic find ings on the
h ve ny ch r cteristics of bip ol r d isord er or lu b r p u nctu re nd CT or MRI sc n; n ele-
d ep ression nd therefore w ould not be given v ted CPK w ou ld be possible if the CN S infec-
d i gnosis of schizo ffective d isord er. P tients tion c u sed seizu res. M lign nt hyp erther i ,
w ith obsessive-co p u lsive d isord er h ve n cute uscu l r p thologic p rocess, rese -
nxiety over p rticu l r thou ghts th t they bles N MS, bu t follow s the d inistr tion of
know re not r tion l (obsessions), nd they inh led nesthetic gents, s in gener l su r-
p erfor ritu lized beh viors to relieve their gery. Prolonged i obiliz tion cou ld resu lt
nxiety (co p u lsions). P r noid p erson lity in n elev ted CPK, bu t w ou ld not ccou nt for
d isord er cou ld be d i gnosed if the p r noi the other find ings.
Answe rs : 515 187

10. (C) Co nd u d itory h llucin tions re boyfriend , co nd u d itory h llu cin tions,
p sychi tric e ergency nd ost clini- nd feelings of help lessness.
ci ns w ou ld gree th t this p tient should
be rest rted on n ntip sychotic ed ic tion. 13. (A) The p tient p resents w ith p sychotic sy p -
ECT h s so e effic cy in tre t ent-resist nt to s nd git tion, long w ith qu estion ble
schizophreni bu t is not first-line ther py. nic sy p to s (e.g., d ecre sed sleep , gr n-
Bec u se they re long cting (i.e., w ith long d iosity, incre sed p rod u ctivity). The d iffer-
h lf-life) nd given his risk of fu ture N MS, enti l d i gnosis inclu d es bip ol r d isord er,
d ep ot injections of neu rolep tic re not first schizo ffective d isord er, nd schizop hreni
choice. As this p tient is not violent, p hysic l ( ong others). Initi l ed ic tions in this c se
restr ints shou ld be void ed . re i ed t the t rget sy p to s of git tion,
d elu sion l thinking, nd d isru p tive beh vior
11. (B) This w o n likely su ffers fro jor nd w ou ld re son bly inclu d e n ntip sy-
d ep ressive d isord er w ith p sychotic fe tu res. chotic nd benzod i zep ine. Bu sp irone is n
Co nd u d itory h llu cin tions to co - nxiolytic u sed ostly in gener lized nxi-
it suicid e, such s overd osing on ed ic - ety d isord er. It is not ind ic ted for bip ol r
tions, re psychi tric e ergency nd u su lly d isord er nd w ou ld not d equ tely d d ress
requ ire psychi tric d ission. Therefore, d is- the t rget sy p to s seen in this c se. While
ch rging to n outp tient setting w ould not be ood st bilizers su ch s c rb zep ine nd
reco end ed . While she w ill need co plete lithiu w ou ld tre t bip ol r d isord er, they
ed ic l w orku p to rule out ed ic l c uses of w ou ld not r p id ly tre t his p sychosis nd gi-
d epression, there is no ind ic tion for d itting t tion. SSRIs u st be u sed w ith extre e c u -
her to ed ic l unit t this ti e; the w orku p tion in bip ol r illness bec u se they c n incite
c n usu lly be cco plished on psychi t- or ex cerb te ni .
ric unit. There is no ind ic tion, su ch s vio-
lent beh vior, th t w ould necessit te physic l 14. (A) When the p tient retu rns for his first
restr ints. follow -u p visit, his lithiu level is ther p eu tic
t 0.8 Eq/ L nd is not in need of d ju st ent.
12. (D ) This p tient requ ires n ntid epress nt H ow ever, his u scle stiffness is likely n
nd ntip sychotic ed ic tion. In f ct, nei- extr -p yr id l sy p to (EPS) of h lop eri-
ther n ntid ep ress nt nor n ntip sychotic d ol, n ely cu te d ystoni . Other EPS inclu d e
lone tre ts jor d ep ressive d isord er w ith p rkinsoni n sy pto s (e.g., br d ykinesi ,
p sychotic fe tu res s w ell s both together. cogw heel rigid ity, pill-rolling tre or, sk-
Althou gh benzod i zep ine y help this like f ce). EPS is nor lly tre ted by low ering
p tients irrit bility, s sole gent it is p oor the d ose of the ntipsychotic or st rting nti-
choice for tre ting her d ep ressive nd p sy- cholinergic ed ic tions su ch s benztropine
chotic sy p to s. Lithiu d oes h ve so e (Cogentin), not w ith u scle rel x nt su ch
ntid ep ress nt effects, bu t in the bsence of s b clofen. The p tient w ould h ve to be fol-
convincing evid ence th t this p tient h s low ed closely for ree ergence of p sychotic
bip ol r d isord er, lithiu is not the p p rop ri- sy pto s.
te choice (If this p tient h s only p rti l
resp onse to the SSRI, lithiu y t so e 15. (A) Acu te stress d isord er is re ction th t y
p oint in the fu tu re be d d ed for u g ent - occur in p tient w ho w itnesses or experiences
tion.). A TCA w ou ld not be the first choice. tr u tic event. The event ust involve
First, there is su ggestion th t this p tient d e th, thre tened d e th, serious injury, or sex-
h s so e c rd i c history nd the tricyclics c n u l viol tion. The typic l sy pto s includ e
h ve p ro rrhyth i effect. Second , u nlike hyper rou s l st tes, d issoci tive st tes, void -
SSRIs, tricyclics c n be leth l in overd ose nce beh viors, nd intrusive reexperiencing
nd this p tient h s risk f ctors for su icid e of the event (e.g., fl shb cks, night res).
tte p t, su ch s recent bre ku p w ith her PTSD is si il r in ost respects to cu te
188 6: Diffe re ntia l Diag nos is a nd Ma na g e me nt

d istress d isord er, but in PTSD the sy pto s inclu d e rrhyth i , hyp oglyce i , ngin ,
persist for t le st 4 w eeks follow ing the seizu re, nd orthost tic hypotension.
tr u ; shorter d ur tion suggests d i g-
nosis of cu te d istress d isord er. The p tient 19. (B) Reg rd less of p tients p st p sychi tric
d oes not ppe r to be suffering fro ny of d i gnosis, ed ic l c u ses of physic l signs
the typic l criteri for d just ent d isord er or nd sy p to s u st lw ys be ru led ou t. The
MDD, except for gu ilt. Althou gh she h s nxi- nxiety d isord ers, lthou gh intensely d istress-
ety nd void nce sy pto s si il r to gor - ing nd u nco fort ble to p tients, r rely, if
phobi , there is no evid ence of p nic tt cks or ever, resu lt in u nconsciou sness. M ny sp ects
p nic d isord er, nd her sy pto s re better of the p tients story see to be rel ted to nx-
ccounted for by her cute stress d isord er. iety nd cou ld , in f ct, occu r d u ring p nic
tt ck; how ever, this p tients p resent tion
16. (C) While the p tient initi lly h d sy p to s is ore consistent w ith ische ic he rt d is-
consistent w ith cu te stress d isord er, ll of e se w hich shou ld p ro p t c rd i c w orku p .
these sy p to s resolved w ith p sychother- Illness nxiety d isord er is the w orry of h ving
p y. H d these sy p to s p ersisted over or obt ining serious illness d espite re ssur-
1 onth, she w ou ld h ve et criteri for nce by ed ic l p rovid ers.
PTSD. When she p resents g in ye r l ter,
she d isp l ys sy p to s consistent w ith MDD 20. (B) P r p hilic d isord ers d enote ny intense
su ch s d ep ressed ood , nhed oni , p oor nd p ersistent sexu l interest other th n
sleep nd p p etite, nd feeling tired ll d y genit l sti u l tion or fond ling w ith nor l,
long. Ad ju st ent d isord er shou ld not be tu re, or consenting hu n p rtners. P r -
d i gnosed w hen d i gnosis of MDD c n be p hili s re fu rther d ifferenti ted b sed on
d e. The p tient d oes not p p e r to h ve w h t beh vior p rovid es the u rges or f nt sies.
ny p nic tt cks nd she d oes not d escribe Frotteu ristic d isord er is obt ining rou s l
fe rs of h ving nother p nic tt ck w hich fro tou ching or ru bbing g inst noncon-
w ou ld be consistent w ith d i gnosis of p nic senting p erson. Exhibitionistic d isord er is the
d isord er. d esire to exp ose ones genit ls to noncon-
senting p erson. Sexu l sochis d isord er is
17. (B) Ocu logyric crisis is sp ecific ex p le of the d esire to be hu ili ted by nother p erson,
n cu te d ystonic re ction in w hich there is w hile sexu l s d is d isord er is the d esire to
sp s of the u scles of extr ocu l r otion. inflict p hysic l or p sychologic l su ffering
N MS, lso re ction to ntip sychotic ed i- on nother p erson. Voyeu ristic d isord er is
c tions, is ch r cterized by rigid ity, fever, rou s l fro w tching u nsu sp ecting p eop le
leu kocytosis, u tono ic inst bility, nd u nd ressing, n ked , or h ving sex. A few other
d eliriu . Torticollis nd retrocollis re d d i- p r p hili s inclu d e: p ed op hilic d isord er, sex-
tion l d ystonic re ctions th t refer to u scle u l f nt sies, u rges, or cts involving sex w ith
sp s s th t c u se bnor l p ositioning of p rep u bescent child ren; tr nsvestic d isor-
the he d , w hile tris u s is sp s of the j w d er, sexu l rou s l fro cross-d ressing; nd
u scles. fetishistic d isord er, w hich is sexu l rou s l
fro the u se of nonliving objects or nongeni-
18. (B) Benzod i zep ines, SSRIs, nd cognitive- t l bod y p rts.
beh vior l ther p y e ch h ve p l ce in tre t-
ent of the nxiety d isord ers. In this c se, 21. (E) While ny of the sid e effects listed y
w hile the p tient d oes h ve history of GAD, occu r w ith the u se of SSRIs, ost of the
she d oes not h ve history of p nic tt cks. w ill d i inish or resolve over ti e. Sexu l
It w ou ld be u nco on for 59-ye r-old d ysfu nction, either nifesting s d el yed
w o n to su d d enly d evelop p nic tt cks. org s or d ecre sed libid o, c n re in
Therefore, fu rther ed ic l w orku p is neces- sid e effect in 25% to 80% of p tients fter
s ry. A short d ifferenti l d i gnosis list w ould 3 onths of tre t ent.
Answe rs : 1627 189

22. (A) The ost co on c use of d e th w ith f ct th t since being hospit lized 2 d ys go
tricyclic ntid epress nt overd ose is c rd i c (his l st d rink) he h s h d n incre se in his
rrhyth i . The rrhyth i is d u e to the blood p ressu re, he rt r te, nd h s incre sed
d ru gs block d e of sod iu ch nnels in the nxiety, s w ell s d i p horesis nd tre ors.
yoc rd iu le d ing to c rd i c cond u ction Oftenti es, p tients c n lso beco e febrile.
slow ing. Seizu res, shock, resp ir tory f ilure, Benzod i zep ines re the tre t ent of choice
nd co c n lso occu r, bu t re not the ost for lcohol w ithd r w l. Any benzod i zep ine
co on c u ses of d e th. Strokes re typ ic lly c n be u sed the choice d ep end s on the
not c used by overd osing on tricyclic ntid e- d esired rou te of d elivery or the p resence of
p ress nts. liver d ysfu nction. With d ecre sed liver fu nc-
tion, lor zep (Ativ n) or ox zep (Ser x)
23. (E) The key p oints in this c se p resent tion re is p refer ble bec u se they re p ri rily
the bsence of previou s psychi tric history et bolized nd eli in ted ren lly inste d
nd the d u r tion of sy pto s. Before d i g- of throu gh the liver ( s re the other benzo-
nosing pri ry ent l illness, it is necess ry d i zep ines). Ad d ition lly, lor zep y be
to ru le ou t the effects of su bst nce. A u rine chosen bec use it c n be given IV, intr u scu -
toxicology screen usu lly includ es opi tes, l rly (IM), or by ou th (PO). C rb zep ine
coc ine, c nn binoid s, p het ines ( nd nd / or p ropr nolol h ve been u sed in so e
so eti es PCP). A phet ines, coc ine, settings for lcohol w ithd r w l, but neither
nd PCP intoxic tion c n i ic sy p to s re reco end ed for severe lcohol w ith-
of schizophreni . CBC, ESR, nd liver fu nc- d r w l, d eliriu tre ens, or seizu res. Cloni-
tion tests re u nlikely to id in the d i gnosis. d ine nd / or eth d one re u sed for op i te
Althou gh CT sc n is help fu l to ru le ou t w ithd r w l. N ltrexone h s been u sed to help
ss lesion or bleed ing, these re not likely int in bstinence fro lcohol bu t h s no
given his ge nd the nonfoc l neu rologic effic cy in w ithd r w l.
ex in tion.
26. (E) Alcohol w ithdr w l c n be life thre tening.
24. (D ) Delu sions, h llu cin tions, nd d isor- Tre ors begin bout 6 to 8 hours fter cess -
g nized beh vior re so e of the h ll rk tion, follow ed by perceptu l bnor lities
sy pto s of schizophreni . Bec use the (h llucinosis) w ithin 8 to 12 hours. Seizures
sy pto s h ve been present for ore th n y occur nyw here fro 12 to 72 hours fter
1 onth bu t less th n 6 onths ( nd there cess tion of lcohol. Deliriu tre ens (DTs)
is no p reviou s p sychi tric history), the ost y occur ny ti e in the first w eek of bsti-
p p rop ri te d i gnosis is schizop hrenifor nence. In the p st, before i proved d i gnosis
d isord er. Delu sion l d isord er u su lly ni- nd n ge ent, untre ted DTs h d ort l-
fests in id d le- ged ind ivid u l w ho, w hile ity r te of pproxi tely 15%. Abd o in l p in
fu nction l in society, exp eriences circu - nd uscle cr ps re co on sy pto s of
scribed d elusion of the je lou s, eroto nic, opi te w ithd r w l. Cirrhosis nd f tty liver re
gr nd iose, p ersecu tory, or so tic typ e. As long-ter consequences of lcohol use.
this p tient is lso su ffering fro pro inent
u d itory h llu cin tions, this w ou ld exclu d e 27. (E) This p tient is exp eriencing cu te intoxic -
the d i gnosis of d elusion l d isord er. There is tion w ith PCP s illu str ted by his git tion,
no history su ggestive of ood d isord er nd p sychosis, vit l sign inst bility, nd tics. A
thu s jor d epression w ith psychotic fe tu res toxicology screen is the best w y to d i gnose
is not s likely. Since the toxicology screen is this cond ition. Urine toxicology screens co -
neg tive, the d i gnosis of su bst nce-ind u ced only includ e op i tes, coc ine, c nn bis nd
p sychotic d isord er is less likely. PCP, but this v ries d epend ing on ind ivid u l
institu tions. A CBC w ou ld not id in the d i g-
25. (C) The p tient is ost likely exp eriencing nosis of this p tients su bst nce-ind u ced psy-
lcohol w ithd r w l s d e onstr ted by the chotic d isord er. There is no ind ic tion for n
190 6: Diffe re ntia l Diag nos is a nd Ma na g e me nt

EEG t this ti e bec u se no seizu re history hyp notic, or nxiolytic gent, su ch s benzo-
is elicited nd the p tient d oes not p p e r d i zepines, c n c use signific nt w ithd r w l
p ostict l. Althou gh structu r l br in lesions sy pto s, includ ing seizu res. Flu phen zine
c n c u se psychotic sy pto s, subst nce- nd thiorid zine re ntipsychotics, nd i ip-
ind u ced sy p to s re ore co on in this r ine nd nortriptyline re TCAs. N either
ge grou p. As in this c se, nonfoc l neu ro- ntipsychotics nor TCAs c use life-thre tening
logic ex in tion w ou ld d ecre se the likeli- w ithd r w l sy p to s.
hood th t i ging stu d ies w ou ld lter the
d i gnosis or tre t ent pl n. 31. (B) As with the sudden discontinu tion of
lcohol, benzodi zepine or b rbitur te with-
28. (A) Unless the p tient is extre ely git ted dr w l c n be life thre tening. The ost con-
nd t risk for hurting hi self or others, the cerning discontinu tion side effect is seizures.
best choice for tre t ent w ou ld be to p l ce Other withdr w l sy pto s include tre ors,
hi in qu iet, d rk (low sti u l tion) roo . nxiety, uditory, visu l, nd t ctile h llucin -
If the p tient is violent nd / or p sychotic, tions, nd utono ic hyper ctivity. The onset
co bin tion of ntipsychotics (neu rolep tics) of sy pto s depends on the h lf-life of the
nd benzod i zepines cou ld be u sed to keep edic tion, the dose, nd dur tion of use. Vo -
the p tient s fe. Methylphenid te is sti u- iting is not co on side effect of sed tive-
l nt, w hich w ou ld likely w orsen his git tion. hypnotic withdr w l.
Benztrop ine, u sed to p revent extr p yr id l
sy pto s ssoci ted w ith ntipsychotic use, 32. (B) The p tient is exhibiting typ ic l signs
ight w orsen the p tients d eliriu . Bec u se of d eliriu , includ ing n i p ired level of
the p tient is not su ffering fro seizures, p he- consciousness nd d ifficu lty w ith cognition,
nytoin is not ind ic ted . Bet blockers su ch s nifested s d row siness, p oor ttention,
p rop r nolol re u sed for blood pressure n- nd d isorient tion. Cl ssic lly, these sy p-
ge ent s w ell s k thisi c u sed by nti- to s y fluctu te d u ring the d y. P tients
p sychotics. There is no ind ic tion for its u se in w ho eet criteri for d eliriu y ppe r
this p tient. d ep ressed w ith d ecre sed level of rou s l
or y p pe r overly ctive nd energetic,
29. (E) The first p rt of the vignette illu str tes so e w ith the d ep ressed typ e occu rring ore
of the fe tures of op i te intoxic tion, n ely co only. P tients lso show cognitive nd
p up ill ry constriction, d row siness, i p ired e ory i p ir ents, w hich y be ni-
ttention nd e ory, nd slu rred sp eech. fested s d isorient tion or d ecre sed c p city
In the second inter ction, the p tient show s to register, ret in, nd rec ll infor tion. An
signs nd sy pto s of opi te w ithd r w l. MMSE y lso be low d u e to n und erly-
Y w ning, u scle ches, d i rrhe , l cri tion ing jor neu rocognitive d isord er (d e en-
or rhinorrhe , nd fever re ll typic l of opi- ti ) or d epressive illness. In this setting, it
te w ithd r w l. Ad d ition lly, ild ly elev ted w ou ld be i p ort nt to g ther infor tion
vit l signs y occu r; how ever, signific nt fro f ily or other ind ivid u ls w ho know
t chyc rd i nd hypertension shou ld lw ys the p tient in ord er to ev lu te for p os-
sign l possible lcohol w ithd r w l. Ph r - sible coexisting d e enti or jor d epres-
cologic n ge ent of opi te w ithd r w l c n sion. Risk f ctors for d eliriu re extre es
inclu d e clonid ine, centr lly cting lp h - of ge, prior history of d eliriu , lcohol use
2- gonist, nd eth d one, synthetic long- d isord er, sensory i p ir ent ( u d itory or
cting opi te. Lor zep c n be u sed to tre t visu l), nd pre-existing br in d ge. There
elev ted vit l signs nd lop er id e c n be is bro d etiologic d ifferenti l for d eliriu . In
u sed to tre t d i rrhe . this c se, c us tive f ctors y be infection,
hypoxi , et bolic bnor lities, nd cu rrent
30. (E) Tri zol is short- cting benzod i ze- ed ic tions. It is d ifficu lt to d i gnose n
p ine. Su d d en d iscontinu tion of ny sed tive, u nd erlying d e enti or MDD in d eliriou s
Answe rs : 2838 191

p tient. There is no ind ic tion of n nxi- nd SSRIs su ch s cit lop r , c n be effective


ety d isord er or p tient p u rp osely ind u cing in tre ting bord erline p erson lity d isord er;
sy pto s s in f ctitious d isord er. how ever, ed ic tions tend to be ore use-
fu l in conjunction w ith psychother py nd
33. (D ) A high-potency ntipsychotic such s h lo- shou ld be t ilored to specific sy pto s.
perid ol is the d rug of choice for n ging the
git ted or confused p tient w ith deliriu . 36. (B) This p tient is exhibiting signs nd sy p-
Benztropine, d iphenhyd r ine, nd thiorid - to s consistent w ith MDD w ith p sychotic fe -
zine ll h ve nticholinergic properties th t tu res. For ore th n 2 w eeks, the p tient h s
y ctu lly w orsen the d eliriu . Lor zep , d ispl yed d ep ressed ood , loss of interest in
especi lly in older p tients, y h ve p r - p le su r ble ctivities, w eight loss, in bility to
d oxic l d isinhibiting effect nd ex cerb te gi- concentr te, poor sleep, nd guilt bou t her
t tion nd confusion. H loperid ol in low d oses, current d epressive st te. She is lso su ffering
such s 0.5 g every 8 hours, helps to n ge d elusion l concerns bout bod ily fu nctioning
the git ted p tient. It lso h s the d v nt ge nd infection. If the p tient were co pl ining of
of IM, IV, or PO d inistr tion. It is gener lly nond elu sion l cont in tion fe rs (obses-
reco ended to obt in n EKG to ev lu te for sions) nd perfor ing ultiple co pu lsive
QTc prolong tion prior to initi tion. t sks to rid herself of the p thogens, d i g-
nosis of OCD cou ld be consid ered . Persistent
34. (B) The cluster B person lity d isord ers re d ep ressive d isord er (for erly d ysthy ic d is-
ch r cterized s the d r tic nd e otion l ord er) presents w ith chronic od er te level
p erson lity grou p. Bord erline PD p tients d is- of d ep ressive sy p to s for t le st 2 ye rs;
p l y longst nd ing p ttern of inst bility in how ever, this p tient is d escribing n cu te,
ny sp ects of their lives. They see things in severe, ep isod ic d eco pens tion over the l st
either bl ck or w hite, w hich contribu tes to onth. A p tient w ith so tic sy p to d is-
gre t d e l of ffect d ysregu l tion. Self- u ti- ord er presents w ith t le st one physic l sy p-
l tion su ch s su p erfici l cu tting or bu rning is to th t c uses signific nt d istress, resu lting
co on. H istrionic PD p tients typic lly d is- in soci l or occu p tion l d ysfu nction.
pl y dr tic bu t su p erfici l e otions, w ith
over-sexu lized beh viors. N rcissistic PD 37. (E) Bec use of the d elu sion l qu lity to her
p tients d isp l y n overv lued sense of self, p resent tion, the tre t ent of choice is both
entitle ent, nd r ge. Bip ol r d isord er nd n ntid ep ress nt nd n ntip sychotic, or
jor d epression re ffective d isord ers th t electroconvu lsive ther p y (ECT). Antid ep res-
p resent s h ving p eriod s of good fu nctioning s nts lone w ith or w ithout benzod i zepines
long w ith p eriod s of i p ir ent or d istress w ou ld not help w ith her p sychotic sy p to s.
d uring the cu te episod es (e.g., d u ring nic Div lp roex sod iu nd lithiu re u sed s
or d epressed ph ses). ood st bilizers in bip ol r d ep ression or
ni , bu t not co only in the tre t ent of
35. (D ) Di lectic l beh vior l ther p y is con- u nip ol r d ep ression.
sid ered the tre t ent of choice for p tients
w ith bord erline p erson lity d isord er. Cogni- 38. (A) Brief psychotic d isord er is d i gnosed w hen
tive beh vior l ther p y is p roble -solving sy pto s h ve been persistent for ore th n
p sychother p y specific lly u sed for d istorted 1 d y bu t less th n 1 onth. Schizop hreni
ssu ptions nd neg tive feelings th t c n w ou ld be d i gnosed if the p tients sy p to s
ex cerb te p sychi tric sy p to s like d ep res- occu rred for longer th n 6 onths, nd schizo-
sion nd nxiety. Psycho n lysis c n be used p hrenifor d isord er w ou ld be d i gnosed if
for d ifferent psychi tric illnesses; how ever, these sy pto s persisted for gre ter th n
it is not consid ered first-line tre t ent for 1 onth bu t few er th n 6 onths. Delu sion l
bord erline p erson lity d isord er. Med ic tions, d isord er is cond ition usu lly beginning in
inclu d ing ntip sychotics su ch s rip ip r zole, id d le ge in w hich the p ri ry sy p to is
192 6: Diffe re ntia l Diag nos is a nd Ma na g e me nt

gener lly nonbiz rre d elu sion. Delu sion l w hich is lso co on sy p to of d ep res-
d isord er is further cl ssified into subtypes, sion. Most of the bove sid e effects i prove
inclu d ing gr nd iose, je lou s, p ersecu tory, fter sever l d ys. Ak thisi (inner sense of
so tic, nd eroto nic. There is no cle r his- restlessness) nd constip tion re co on
tory or evid ence on physic l ex in tion of sid e effects of ntipsychotics.
d rug use consistent w ith su bst nce-ind u ced
p sychotic d isord er. 42. (C) N or l grief c n ppe r si il r to jor
d ep ressive ep isod e (MDE), bu t there re
39. (E) The ost helpful test would be urine toxi- w ys to d istingu ish betw een the tw o. While
cology screen, since intoxic tion with v riety bere ve ent is not p sychi tric d i gno-
of subst nces, including coc ine, PCP, c nn bis, sis, it is psychosoci l stressor th t c n pre-
or phet ines y i ic the sy pto s of cipit te n MDE in vu lner ble ind ivid u l.
schizophreni . It is necess ry to first rule out Nonbere ve ent-rel ted depression responds
these intoxic tion syndro es prior to offici lly to the s e ed ic tions nd tre t ents s
king di gnosis of pri ry psychotic ill- bere ve ent-rel ted d ep ression. MDE shou ld
ness, such s schizophreni . There is no specific be consid ered if the d ep ressive sy p to s l st
d i gnostic u tility for CBC or electrolytes for longer th n 2 w eeks, nd ssoci ted w ith
for ny of the p ri ry p sychotic d isord ers. p sycho otor ret rd tion, ch nge in pp etite/
Bec u se the p tient d oes not p resent w ith w eight, d ecre sed energy nd concentr tion,
signs or sy p to s of seizu re d isord er, n nd thou ghts of su icid e. The resp onse to nor-
EEG is u nlikely to p rovid e infor tion th t l bere ve ent exclu d es the d i gnosis of
w ill i p ct the d i gnosis. A CT sc n y d ju st ent d isord er. A d i gnosis of p ersistent
help to ru le ou t ss lesion or bleed , bu t d ep ressive d isord er shou ld be consid ered for
his ge nd nonfoc l neu rologic ex in - p tients w ho h ve d epressive sy pto s ore
tion ke this n u nlikely contribu tion to his often th n not for 2 ye rs. Schizophreni w ou ld
p resent tion. not be n p prop ri te d i gnosis s the p tient
d oes not h ve previou s psychi tric history,
40. (B) The p tient h s d i gnosis of MDD s d oes not h ve other p sychotic sy pto s, nd
ch r cterized by the d ep ressed ood , p oor the h llu cin tions re better ccou nted for by
concentr tion, p oor sleep , p oor p p etite, nd bere ve ent.
gu ilty feelings. Althou gh it p p e rs th t the
p tients d ep ression w s triggered by d e th 43. (E) At this ti e, the p tient is exp eriencing
in the f ily, she nevertheless eets criteri nor l bere ve ent nd no fu rther interven-
for MDD. There re tw o choices for ntid e- tions re necess ry. Shou ld the sy p to s
p ress nts in this grou p . Flu oxetine, n SSRI, continu e for longer period of ti e, w orsen,
is the best choice t this ti e. Phenelzine, n or the p tient beco es less fu nction l, ini-
MAOI, is not the first choice of ntid ep res- ti ting n ntid ep ress nt su ch s flu oxetine
s nt s the SSRIs h ve ore f vor ble sid e ( n SSRI) or, less frequ ently, itriptyline (
effect p rofile, re s fer in overd ose, nd h ve TCA) w ou ld be p p rop ri te. The p tients
si il r effic cy in the tre t ent of MDD. Flu - u d itory h llu cin tions re still consid ered to
zenil is benzod i zep ine nt gonist u sed be nor l p rt of bere ve ent t this ti e,
for the tre t ent of benzod i zep ine over- so n ntipsychotic like h loperid ol should
d ose, flu p hen zine is n ntip sychotic, nd not be initi ted u nless the voices beco e ore
lor zep is benzod i zep ine. N one re d istressing nd p erv sive. Lithiu is ood
p p rop ri te first-line tre t ents for MDD. st bilizer ind ic ted for bipol r d isord er.

41. (D ) SSRIs h ve nu ber of sid e effects, 44. (B) Ad ju st ent d isord er is ch r cterized by
inclu d ing d i rrhe , constip tion, n u se , the onset of e otion l or beh vior l d istu r-
he d che, sexu l d ysfunction, nd git tion. b nces w ithin 3 onths of signific nt life
A frequ ent, initi l sid e effect is inso ni , event th t y nifest s rked ch nge in
Answe rs : 3948 193

n ind ivid u ls bility to fu nction in school, of h ving n tt ck or being u n ble to esc pe


w ork, or interp erson l rel tionship s. The d uring n tt ck force p tients to re in in
d istu rb nces re not so severe, how ever, s f ili r pl ces. The ed ic l w orku p for p nic
to su ggest the d i gnosis of nother d isord er d isord er inclu d es CBC, electrolytes, f sting
such s MDD. The d ifferenti l d i gnosis for glu cose, c lciu , liver fu nction tests, BUN ,
d ju st ent d isord er inclu d es ood d isord ers cre tinine, u rin lysis, toxicology screen, ECG,
such s MDD, nd nxiety d isord ers su ch nd thyroid fu nction stu d ies. A c refu l su b-
s PTSD, GAD, nd cute stress d isord er. In st nce u se nd c ffeine int ke history should
this c se, the p tient y be su ffering fro lso be obt ined . GAD occu rs in ind ivid u ls
d ep ressed ood bu t d oes not h ve other neu - exp eriencing continu ed nxiety nd w orry
roveget tive signs of d ep ression. Acu te stress over sever l events l sting for gre ter th n
d isord er is d i gnosed w hen n ind ivid u l 6 onths. In contr st to p nic d isord er, there
w ho h s exp erienced n extre e stressor h s re not u su lly d iscrete ep isod es in GAD. Sep -
d issoci tive exp eriences nd nxiety shortly r tion nxiety d isord er is u su lly d i gnosed
fter the tr u tic experience. In this c se, the in you ng child ren w ho d isp l y excessive
p tient h s exp erienced signific nt psycho- nxiety w hen sep r ted fro their pri ry
soci l stressor, bu t it is not life-thre tening c regiver. Soci l nxiety d isord er occu rs in
event s is requ ired for the d i gnosis of cu te the context of pu blic p pe r nces or p erfor-
stress d isord er. In d d ition, this p tient is not nces, w here ind ivid u ls exp erience nxiety
d escribing these sy pto s. The p tient is not nd fe r. These feelings c u se gre t d istress
suffering nxiety bout nu ber of events for nd y c u se p eop le to void these settings.
over 6 onths consistent w ith GAD. Grief is This d isord er d iffers fro p nic d isord er by
the nor l response to the loss or d e th of n h ving n id entifi ble trigger. Specific phobi s
ind ivid u l close to the p tient. lso h ve n id entifi ble trigger.

45. (D ) The d i gnosis of d ju st ent d isord er 47. (D ) A v riety of ed ic tions, inclu d ing SSRIs
d oes not u su lly require ph r cother peu tic (sertr line), TCAs, MAOIs (tr nylcypro ine),
intervention, bu t inste d y resp ond to su p - nd benzod i zepines ( lpr zol ) h ve been
p ortive, ind ivid u l, or grou p psychother p y. used to tre t p nic d isord er. Althou gh ll h ve
Di lect l beh vior l ther p y is the first-line show n so e effic cy in the tre t ent of p nic,
tre t ent for bord erline person lity d isord er SSRIs re usu lly the first gent of choice
p tients, help ing d ecre se self-d estru ctive bec use they re gener lly w ell toler ted , w ith
beh viors nd i p roving ffect regu l tion. few er sid e effects th n TCAs or MAOIs, nd
Flu oxetine is n SSRI, nd nortrip tyline is l ck the d epend ency potenti l of benzod i z-
TCA, both ntid ep ress nts. Risp erid one is n epines. Co on sid e effects inclu d e sexu l
typic l or second -gener tion ntip sychotic. d ysfunction, g strointestin l d isturb nces, nd
inso ni . Lithiu is used in the tre t ent of
46. (B) This p tient is d escribing sy p to s bipol r d isord er, nd propr nolol c n be used
ssoci ted w ith p nic d isord er. This d isor- to n ge the p hysic l sy pto s of nxiety in
d er begins in l te d olescence or e rly d ult- soci l nxiety d isord er (soci l phobi ).
hood nd is ore co on in w o en th n in
en. Peop le typic lly d escribe recu rrent nd 48. (B) The p tient likely h s obsessive-co pulsive
u np rovoked p nic tt cks, consisting of feel- d isord er (OCD) s she su ffers fro recu r-
ings of fe r, d re d , or intense d isco fort sso- rent, intru sive, u nw nted thou ghts or i ges
ci ted w ith v riety of so tic sy pto s, (obsessions) nd repetitive beh viors or en-
inclu d ing t chyc rd i , chest p in, shortness t l cts (co pu lsions) th t re p erfor ed in
of bre th, tre u lousness, d i phoresis, n u- resp onse to these obsessions. P tients re l-
se , fe r of d ying, p resthesi s, light he d ed - ize th t the obsessions nd co p u lsions re
ness, nd hot or cold fl shes. The d i gnosis of u nre son ble, nd they c u se rked i p ir-
gor p hobi is d d ition lly d e w hen fe rs ent in their lives. M ny p tients w ith OCD
194 6: Diffe re ntia l Diag nos is a nd Ma na g e me nt

hid e their sy p to s nd y first co e to the sy pto s of nxiety nd w orry re in


the ttention of other ed ic l speci lists. excess of w h t w ou ld be exp ected , they d o not
GAD is suit ble d i gnosis in ind ivid u ls re ch the gnitu d e ch r cteristic for OCD,
w ho h ve w orries nd concerns th t re not nor re ny sp ecific obsessions or co pu lsions
li ited to one p rticu l r sp ect of their lives, elicited . There is no evid ence of p sychosis in
bu t w ith the nxiety signific ntly i p iring this p tient consistent w ith schizophreni .
their bility to fu nction. OCPD is person l-
ity d isord er in w hich p tients beco e p reoc- 51. (A) The d i gnosis of conversion d isord er
cup ied w ith perfectionis nd ord erliness. (fu nction l neurologic l sy pto d isord er)
H ow ever, there re no recu rrent obsessions is ost likely given th t the p ri ry sy p -
or co pu lsions, nd the sy pto s re ego- to s re neurologic nd the w orkup is neg -
syntonic. P nic d isord er nifests s recur- tive for evid ence of p thology th t w ould
rent, d iscrete ep isod es of extre e nxiety ccount for the d eficits. N eu rologic d eficits
nd fe r th t occu r unexp ected ly nd re it in conversion d isord er involve either otor
spont neously. So tic sy pto d isord er or sensory od lities nd re believed to be
p resents w ith t le st one p hysic l co p l int resu lt of u nd erlying, u nconsciou s p sychologi-
th t c nnot be fully expl ined by the results of c l conflicts or stressors. H d the sy pto s
ed ic l investig tions. been intention lly p rod u ced , the d i gnosis of
f ctitious d isord er or lingering w ould be
49. (A) A w id espre d nd effective tre t ent for p p ropri te. If the p ri ry incentive for the
OCD is cognitive-beh vior l ther py, d u r- beh vior w s to ssu e the sick role, the d i g-
ing w hich the p tient is exp osed to the thre t nosis w ou ld be f ctitiou s d isord er. If the sy p -
or fe r (obsession) w ith the response (h nd to s re feigned or ex gger ted in ord er to
w shing) p revented for p rogressively longer g in onet ry rei bu rse ent or to void j il
p eriod s of ti e u ntil the beh vior is extin- or w ork, the d i gnosis w ou ld be lingering.
gu ished . SSRIs h ve lso been su ccessfu l in So tic sy pto d isord er is less ppropri te
the tre t ent of OCD. Lithiu is u sed in the in this c se bec u se this p tients sy p to s
tre t ent of bipol r d isord er nd h s not been rent d istressing to her nd re solely neu ro-
show n to be effective in the tre t ent of OCD. logic co p l ints. An u nd i gnosed neu rologic
Psychod yn ic p sychother p y is intensive, cond ition is possibility, but given the p re-
long-ter ther p y th t y be help fu l for sent tion nd physic l ex in tion, it is less
p eople w ho w ish to better u nd erst nd the likely. More recent stu d ies h ve show n only
n tu re of their rel tionship w ith others, bu t 4% isd i gnosis of conversion d isord er w hen
it h s not d e onstr ted signific nt effic cy in there is n u nd erlying neu rologic l d isord er
OCD. Risp erid one is n typ ic l ntip sychotic c using the sy pto s.
th t is u sed to tre t psychotic d isord ers nd is
not first line for tre ting OCD. 52. (B) F ctitiou s d isord er (Mu nch u sens) is n
ppropri te d i gnosis w hen physic l or psy-
50. (A) Sy pto s of excessive w orry nd nxi- chologic l sy pto s re intention lly pro-
ety bou t sever l sp ects occu rring for over d uced in ord er to ssu e the sick role. It ust
6 onths re best ch r cterized by the d i g- be cle r th t these sy pto s re not intention-
nosis of GAD. Other sy pto s of GAD lly prod uced for other re sons, such s void -
inclu d e sleep d ifficu lties, irrit bility, nd d if- ing w ork, g ining onet ry co pens tion, or
ficulty concentr ting. P tients w ith GAD y void ing leg l issues. If ny of these re sons re
lso su ffer fro u scle tension, f tigu e, nd present, the d i gnosis is lingering. So tic
restlessness. Althou gh this p tient d oes d it sy pto d isord er presents w ith t le st one
to nxiety, it d oes not occu r in resp onse to or ore so tic sy pto s, w hich c n be p in.
p nic tt cks or in response to p ublic p erfor- The d i gnosis of illness nxiety d isord er is not
nces, s w ith p nic d isord er or soci l nxi- ppropri te s this p tient is not preoccupied
ety d isord er, resp ectively. Si il rly, lthou gh w ith the id e of h ving severe d ise se th t
Answe rs : 4958 195

is unrecognized d espite repe ted re ssur nces c ring for hi self, s w ell s f ily history
by ed ic l personnel. Cl ssic lly, people w ith of le rning d is bilities, you shou ld su spect
f ctitious d isord er re ore likely to be fe le the d i gnosis of intellectu l d is bility (ID).
nd to h ve experience in he lth c re. Often, ID is d efined by severely i p ired cognitive
these p tients w ill h ve nu erous d issions nd soci l fu nctioning. The severity of intel-
to ny d ifferent hospit ls w ith v riety of lectu l d is bility is d eter ined now by d p -
sy pto s. The p sychologic l f ctors u nd erly- tive fu nctioning r ther th n IQ score. Either
ing this d isord er re not w ell und erstood . N o the WISC or the St nford -Binet Intelligence
effective for of psychother py or ph r co- Sc le is n d equ te instru ent for esti ting
ther py h s been id entified for the tre t ent IQ. The Good enou gh-H rris Dr w -A-Person
of f ctitiou s d isord er. Test nd Kohs Block Test ssess visu l- otor
coord in tion. The MMPI-2 is frequ ently u sed
53. (A) The ost likely d i gnosis in this p tient to ssess person lity stru cture. The Pe bod y-
is norexi nervos nd is su p p orted by the Voc bu l ry Test is u sed w hen p tients su ffer
f ct th t she p pe rs very u nd erw eight, exces- fro l ngu ge b rrier.
sively exercises nd continu es to feel th t
she need s to lose ore w eight, thu s d e on- 56. (B) This p tient su ffers fro Tou rette d isor-
str ting her d istu rbed bod y i ge. Buli i d er, synd ro e of ultiple otor nd voc l
nervos c n be exclu d ed bec u se she d oes tics th t occu r d ily for t le st 1 ye r, w ith the
not h ve ny binge e ting. While the p tient onset before ge 18. Tou rette d isord er occu rs
d oes h ve so e nxiety, it is not cle r th t her ore co only in boys th n girls. First-line
nxiety h s re ched to level th t it is c u sing tre t ent for ild to od er te Tou rette d is-
ny typ e of d ysfu nction in her d ily ctivities, ord er is n lp h -2 gonist, su ch s gu nf cine
consistent w ith GAD. P tients w ith bod y d ys- or clonid ine. An ntipsychotic c n be used to
orphic d isord er gener lly focu s on one sp e- tre t the d isord er, bu t is u su lly reserved for
cific d efect r ther th n so ething s gener l p tients w ith ore severe c ses. Ato oxetine
s w eight. While the p tient d oes h ve sy p - is selective norep inep hrine reu p t ke inhibi-
to s su ggestive of MDD, su ch s proble s tor th t h s been used for the tre t ent of
w ith sleep , p oor concentr tion, nd f tigu e, ADH D; it c n be effective for child ren w ith
she d oes not ention feeling d ep ressed or ADH D w ho d evelop tics fter u sing sti u -
h ving nhed oni , w hich re requ ired for l nts. L otrigine nd v lp ro te cid re
d i gnosis of MDD. ood st bilizers u sed for the tre t ent of
bip ol r d isord er. Lor zep is benzod i z-
54. (E) Obt ining height nd w eight is cruci l for ep ine nd is typ ic lly u sed for cu te nxiety
est blishing the d i gnosis of norexi nervos , d isord er nd git tion.
s p tients w ith norexi nervos re u n ble
to chieve or int in gre ter th n 85% of 57. (D ) Given the signific nt i p rove ent d u e to
exp ected bod y w eight for height nd fr e the ed ic tion, it w ou ld not be ppropri te
stru ctu re. Ad d ition lly, they h ve contin- t this ti e to d iscontinu e or sw itch ed ic -
u ed concerns bout thinness nd bod y i ge tions. Sed tion is the ost co on sid e effect
d esp ite being u nd erw eight. In contr st, ost of clonid ine nd w ill often go w y fter few
p tients w ith bu li i nervos re of nor l w eeks. Gu nf cine, the other lph -2 gonist
height nd w eight or re slightly overw eight. is nother op tion if the sed tion continu es or
While the other tests listed y, ind eed , be interferes in fu nctioning. A d os ge red u ction
bnor l in norexi ( nd bu li i ), they re shou ld be tte pted before king ny of the
not necess ry or su fficient for the d i gnosis. other ch nges listed .

55. (E) Given the p tients history of nger nd 58. (B) Ep ilep sy is co p lex set of d isord ers
ggressive outbursts w hen not und erst nd ing th t y initi lly co e to the ttention of
schoolw ork, d ifficulties w ith soci liz tion nd p sychi trist bec u se of u nu su l beh viors.
196 6: Diffe re ntia l Diag nos is a nd Ma na g e me nt

Co p lex p rti l seizures, in p rticul r, y u nlikely s they d o not p resent w ith the bove
be ssoci ted w ith w id e r nge of sy p- yri d neu rologic l sy p to s nd s the
to s, includ ing sensory ur s nd uto tic p tient is not cu rrently t king n ntip sychotic
beh viors. Prior to the seizure, the p tient ed ic tion. Although this p tient h s so e
y notice b d s ell, n u nu su l t ste, or sy pto s consistent w ith Kors koff p sycho-
g strointestin l sy pto s. During the event, sis nd Wernicke enceph lop thy (both d u e
the p tient y look d zed or frightened nd to thi ine d eficiency fro long-ter lcohol
y exhibit nu ber of uto tis s su ch u se), neither d i gnosis lone w ou ld ccou nt
s lip s cking, eye blinking, nd fu bling for this p tients constell tion of neu ropsychi-
w ith his or her clothes. Ad d ition lly, there is tric sy p to s.
i p ir ent of consciousness. Co plex p rti l
seizu res re usu lly ssoci ted w ith te por l 60. (D ) Pheochro ocyto is tu or of the chro-
lobe epilepsy, but c n lso result fro lesions ffin cells, usu lly occu rring in the d ren l
elsew here in the br in. Another d i gnosis to ed u ll . Sy pto s y d iffer ccord ing to
rule out w ould be phet ine intoxic tion. the pred o in nt c techol ine rele sed . As
A phet ines re co only u sed to tre t norep inep hrine-secreting tu ors re the ost
ADH D nd y be d ru gs of buse in this ge co on, b seline hypertension w ith p rox-
group. Intoxic tion w ith phet ines y ys l ex cerb tions re frequ ent. Anxiety,
present s seizu res nd c n be e sily ru led ou t fe r, nd p lpit tions y cco p ny ex c-
by urine toxicology screen, but the nor l erb tions of hyp ertension. Bec u se this y
vit l signs ke this less likely. The other d i g- i ic sy pto s of p nic tt ck or GAD, it
noses w ou ld not ccount for this p resent tion. is essenti l to consid er this in the d ifferenti l
d i gnosis w hen p tients present w ith nxiety
59. (D ) Althou gh syphilitic infections h ve been cou pled w ith physiologic ch nges. Alcohol
on the d ecline since World W r I, they ust w ithd r w l, r ther th n intoxic tion, w ou ld
still be consid ered in the d ifferenti l for w id e p resent w ith nxiety, tre ors, nd elev ted
v riety of psychi tric d isord ers, especi lly vit l signs. H yperc lce i inclu d es sy pto s
w hen co bined w ith neu rologic bnor li- such s kid ney stones (stones), bone p in/
ties. In this c se, tw o for s of neurosyphilis fr ctu res (bones), d ep ression/ irrit bility
(t bes d ors lis nd gener l p resis) re co - ( o ns), nd n u se / vo iting (gro ns).
bined in one p tient. P tients d e onstr te Sy pto s of hypothyroid is re d epression,
sensory t xi w ith w id e-b sed g it, w eight g in, constip tion, ed e , d ry skin,
p ositive Ro berg sign, nd loss of vibr - ho rseness, nd thinning h ir. A posterior
tory nd proprioceptive senses initi lly in circu l tion stroke w ou ld present w ith cute
the low er extre ities. Deep tend on reflexes vision loss, confusion he d che, p resthesi s,
re d ecre sed nd pu pil bnor lities re w e kness, nd d izziness.
co on, lthou gh the fu ll ArgyllRobertson
p up il ( pu p il th t cco od tes bu t d oes not 61. (B) H ep tic encep h lop thy c n p resent
re ct to light) is infrequ ent. Gener l p resis is in ny w ys nd y h ve flu ctu ting
u su lly ssoci ted w ith d e enting p rocess cou rse, king it d ifficu lt to d i gnose w ith-
long w ith neu rologic sy p to s su ch s p u pil ou t p erip her l stig t . N eu rologic lly, the
bnor lities, tre ors, d yscoord in tion, nd p tient y exhibit sterixis, w hich, lthou gh
sp sticity in the low er extre ities. Testing for ch r cteristic for hep tic encep h lop thy,
neu rosyp hilis shou ld inclu d e blood test for is not sp ecific for the d isord er. P tients y
VDRL nd flu orescent trep one l ntibod y d isp l y 6- to 9-H z tre ors, ild ly incre sed
bsorp tion test (FTA-ABS). Consid er tion d eep tend on reflexes, nd n ltered senso-
shou ld lso be given to p erfor ing lu b r riu . Typ ic l p sychi tric sy p to s y
p u nctu re to test cerebrosp in l flu id for evi- inclu d e ch nges in p erson lity, bru p t ood
d ence of trep one l infection. A d i gnosis sw ings, nd ch nges in cognitive bility. In n
of ntipsychotic-ind uced d ystoni or N MS is cu te d eco p ens tion, d ep ression, c t toni ,
Answe rs : 5967 197

p sychosis, nd d eliriu y d evelop . Physi- occurrence of physic l sy pto s, w hich y


c l stig t reflecting d ecre sed liver fu nc- be p red o in ntly p in, th t re d istressing
tion inclu d e ecchy osis, p erip her l ed e , nd c u se soci l or occu p tion l d ysfu nction.
scites for tion, p l r erythe , nd sp i- It is i p ort nt to d eter ine th t these sy p -
d er ngio s. to s re not intention lly p rod uced or feigned
for other pu rposes. Si il rly, it is i port nt to
62. (C) H IV c n p rod u ce spectru of neuropsy- ensu re th t the p tient receive the p p rop ri te
chi tric nifest tions s resu lt of opp ortu - ed ic l w orku p , bec u se nu ber of d ise se
nistic infections, neopl s s, or d irect inv sion p rocesses y present w ith constell tion of
of the CN S by the viru s. M jor neurocogni- sy pto s. B sic tests th t shou ld be ord ered
tive d isord er (d e enti ) d ue to H IV infection inclu d e blood tests, ECG, colonoscop y, or
is one of the ost co on nifest tions gynecologic l ex in tions.
of pri ry CN S inv sion. Criteri for jor
neu rocognitive d isord er (d e enti ) includ e 65. (H) Depression is co on sy pto of
signific nt cognitive d ecline in one or ore SLE. Although neuropsychi tric cond itions
cognitive d o ins (le rning nd e ory, y rise t ny ti e d u ring the cou rse of the
ttention, executive function, l ngu ge, per- d ise se, they re ost frequent d uring cute
ceptu l- otor, or soci l cognition). It y ex cerb tions or l te in the d ise se process.
present in v riety of w ys, includ ing otor Other sy pto s inclu d e d eliriu , person lity
bnor lities, cognitive d ecline, or beh vior l ch nges, nxiety, nd psychosis. P tients y
ch nges. This c se d escribes typic l beh vior l lso exhibit cognitive i p ir ent on neuro-
ch nges, for ex ple, leth rgy nd soci l w ith- psychologic l testing. N eu rologic lly, p tients
d r w l. M jor neurocognitive d isord er d ue c n present w ith seizu res, stroke, cr ni l nerve
to H IV infection y lso includ e neurologic bnor lities, nd he d che.
bnor lities such s w e kness, i b l nce, or
t xi , s w ell s irrit bility nd p thy. The 66. (E) This p tient is exhibiting signs nd sy p -
results of the MMSE support the d i gnosis of to s consistent w ith d eliriu second ry to
jor neurocognitive d isord er. hyp oglyce i . Signs of hyp oglyce i ni-
fested by this p tient inclu d e t chyc rd i ,
63. (D ) This c se illu str tes present tion of tre or, hyp ertension, nd seizu re. Deliriou s
hyp erthyroid is , w ith d ep ression, irrit bility, st tes shou ld receive i ed i te w orku p for
nd cognitive d ysfu nction. H yperthyroid is n etiology s they c rry signific nt r te of
y present w ith sy pto s consistent w ith orbid ity nd ort lity. A st nd rd w orku p
v riety of psychi tric d isord ers, inclu d ing consists of CBC w ith d ifferenti l, electro-
d ep ression, nxiety, psychosis, ni , nd , t lytes, BUN , cre tinine, VDRL, vit in B12,
n extre e, d eliriu . P tients y rep ort hot fol te, u rin lysis, thyroid -sti u l ting hor-
fl shes nd / or incre sed sensitivity to he t. one (TSH ), c lciu , gnesiu , p hosp ho-
H yperthyroid is y lso c u se ild cogni- ru s, glu cose, u rine toxicology screen, liver
tive d eficits in c lcu l tion nd recent e ory. fu nction tests, p erip her l oxygen s tu r tion,
N eurologic lly, p tients y d e onstr te chest x-r y, ECG, s w ell s ent l st tu s nd
fine 8- to 12-H z tre or, lid l g, brisk d eep ten- p hysic l ex in tion.
d on reflexes, proxi l yop thy w ith uscle
w sting, nd y lgi s. 67. (C) P tients w ith jor neurocognitive d isor-
d er d ue to Alzhei er d ise se typic lly present
64. (G) So tic sy p to d isord er is ch r cter- w ith e ory proble s th t gr d u lly w orsen,
ized by t le st one so tic or p hysic l co - s w ell s one other cognitive d o in (e.g.,
p l int th t c nnot be verified by l bor tory p h si , pr xi , gnosi , or d istu rb nces in
nd p hysic l find ings. The d isord er h s its executive functioning). In this c se, the p tient
onset before ge 30 nd is ore co on in de onstr tes e ory proble s nd n gnosi
w o en th n in en. Di gnosis is b sed on the (un ble to n e pencil).
198 6: Diffe re ntia l Diag nos is a nd Ma na g e me nt

68. (F) M jor neu rocognitive d isord er d u e to typic lly c uses p sychi tric d istu rb nces su ch
Lew y bod y d ise se typ ic lly p resents w ith s d ep ression or nxiety, h llu cin tions, e -
w ell-for ed visu l h llu cin tions (p ed u ncu - ory proble s, nd choreifor ove ents.
l r h llu cinosis or Lillip u tionis ) l te in life On i ging, p tients y h ve boxc r ventri-
w ithou t p reviou s p sychi tric d istu rb nces. cles th t ppe r s h ving corners r ther th n
Motor d istu rb nces re si il r to th t of rou nd ed ed ges.
P rkinson d ise se bu t in Lew y bod y d ise se M jor neurocognitive d isord er d ue to Prion
occu r l ter th n the cognitive d eficits. Motor (Creu tzfeld tJ cob) d ise se is d e enti
p roble s inclu d e cogw heel rigid ity, shu ffling c used by prion, su ch s in d cow d is-
g it, nd red u ced r sw ing. The typ ic l p ill- e se. Ind ivid u ls p resent w ith cognitive d efi-
rolling tre or of P rkinson d ise se is less cits, t xi , bnor l ove ents, nd often
co on. st rtle reflex. It h s r pid p rogression. M jor
v scu l r neurocognitive d isord er (for lly
69. (D ) Pick d ise se is ch r cterized by ch nges referred to v scu l r or u lti-inf rct d e en-
in p erson lity e rly in the illness. Pri itive ti ) is c u sed by cerebrov scu l r events, such
reflexes re seen, su ch s the su ck, B binski, s strokes or e boli. In d d ition to the cog-
nd snou t. Other cognitive fe tu res of the nitive d eficits, it is not u nco on to d ispl y
d e enti typ ic lly occu r l te in the d ise se. p erson lity nd ood ch nges. In contr st to
I ging is u su lly ch r cterized by p referen- Alzhei er d ise se, jor neu rocognitive d is-
ti l trop hy of the front l nd te por l lobes. ord er d ue to v scu l r d ise se cl ssic lly p res-
Confir tion of the d i gnosis is d e vi ents w ith step -w ise d eterior tion.
biop sy or u top sy nd reve ls Pick inclu sion
bod ies. 73. (C) Bec u se the d efense ech nis of d eni l is
p ri ry in lcohol u se d isord ers, often the best
70. (A) Deliriu is synd ro e consisting of p p ro ch in ev lu ting p tient is to explore
flu ctu tion in the level of ttention nd cogni- how lcohol ffects his life, r ther th n d irect
tion, sy p to s of w hich c n w x nd w ne. qu estions bou t d rinking beh vior. H e ring
It is essenti l to look for one or ore u nd erly- bou t his d rinking fro his friend s nd f -
ing c u ses, su ch s electrolyte bnor lities, ily y p rovid e ore ccu r te d escrip tion
infections, nd sid e effects of ed ic tions. of p tients lcoholis th n the ind ivid u l
In contr st to jor neu rocognitive d isord er w ou ld p rovid e of hi self. Althou gh l bor -
(d e enti ), d eliriu h s n cu te onset of tory w ork ight provid e clu es to the presence
sy pto s nd is typic lly reversible once the of lcohol use d isord er, it c nnot be relied on
u nd erlying etiology is corrected . to ke the d i gnosis.

71. (B) Pseudode enti is ter referring to the 74. (B) GAD is d efined by excessive nxiety or
cognitive disturb nces (de enti ) c used by w orry occu rring for t le st 6 onths. Asso-
depressive illness, such s jor depression. ci ted sy pto s includ e feeling restless,
P tients typic lly present with depressed ffect, f tigu ed , or irrit ble; u scle tension; d ifficu lty
neuroveget tive sy pto s, nd e ory prob- w ith concentr tion; nd sleep d istu rb nce.
le s. During ex in tion, p tients de on- Dep end ent p erson lity d isord er rep resents
str te poor concentr tion nd effort such th t p ttern of the p erv sive need to be t ken
they h ve difficulty co pleting questioning. c re of, le d ing to su b issive, clinging beh v-
Pseudode enti is reversible once the depres- ior th t begins in e rly child hood . Althou gh
sion is tre ted. d ep ression nd p ersistent d ep ressive d isord er
sh re so e sy pto s w ith GAD, the d epres-
72. (E) M jor neu rocognitive d isord er d ue to sive sy p to s re ore severe in the ood
H untington d ise se is n utoso l d o in nt d isord ers th n w ith the nxiety d isord ers. The
d ise se c used by CAG repe t on the short onset of nxiety w ith p nic d isord er is ore
r of chro oso e nu ber 4. The d ise se sud d en nd intense, w ith the fe r of choking
Answe rs : 6880 199

or d ying co on, nd the p nic tt cks re the d epressive episod e h s l sted longer th n
recu rrent nd u nexp ected . Soci l nxiety d is- 4 w eeks fter the cess tion of lcohol points
ord er requ ires nxiety in soci l situ tions w ith to n und erlying (co orbid ) ood d isord er
fe rs of scru tiny or hu ili tion by others, such s jor d epressive d isord er (MDD)
such s perfor nce nxiety or fe rs of pu blic r ther th n su bst nce-ind u ced ood d is-
spe king. ord er. Sertr line is n SSRI nd is first-line
gent for the tre t ent for MDD. While i-
75. (A) N eg tive thoughts reg rding the self, the triptyline ( tricyclic ntid epress nt) is n
w orld, nd the future re know n s the Beck option, bec use of its sid e effect profile nd
cognitive tri d . Beck, w ho d eveloped the Beck leth lity in overd ose, it is not u sed s first-
Depression Inventory, pioneered the use of line gent for d ep ression. Bu sp irone is n nx-
cognitive ther py for d epressive d isord ers. iolytic u sed for the tre t ent of gener lized
This tri d of sy pto s is helpful in both und er- nxiety d isord er. Lithiu is ood st bilizer
st nd ing nd tre ting d epression through cog- typic lly u sed for bip ol r d isord er ( ni )
nitive ther py. The tri d is not ch r cteristic lly nd not co only for u nip ol r d epression.
seen in the other d isord ers listed . Lor zep ( benzod i zep ine) is lso n nx-
iolytic, bu t it w ou ld not d d ress the p tients
76. (B) Cognitive ther py c n help to t rget these d ep ressive sy p to s nd c n c use physi-
neg tive w ys of perceiving the self nd the ologic l d epend ence, esp eci lly in this p tient
environ ent. While often u sed in conju nction w ith history of lcohol d d iction.
w ith cognitive ther p y (i.e., cognitive-beh v-
ior l ther p y), p u re beh vior l ther p y is 79. (D ) This p tient is experiencing n cu te Wer-
only focu sed on beh vior od ific tion, not nicke enceph lop thy (d eliriu ) c u sed by
on the u nd erlying f ctors (such s cognitive d inistr tion of glu cose w ithou t thi ine.
errors) th t c u se or contribu te to the p rob- In p tients w ith long-ter lcohol u se d isor-
le tic beh viors. Cou p les ther p y w ou ld be d er w ho re likely to h ve low levels of thi -
less effic ciou s th n cognitive ther p y given ine, giving glu cose p rior to giving thi ine
this p tients sy pto s. The jor focus of c n often precipit te this cond ition, s glucose
interp erson l p sychother p y is the n lysis of oxid tion requires thi ine, thu s d epleting
co u nic tion styles nd ones role in rel - the re ining stores in the bod y; this d eple-
tionship s. In p r d oxic l ther py, d evelop ed tion c n then c use r p id d rop in the thi -
by B teson, the ther p ist su ggests th t the ine levels w hich c uses the encep h lop thy.
p tient eng ge in the beh vior w ith neg tive
connot tions (e.g., phobi or co pu lsion). 80. (A) Alcohol-ind u ced jor neu rocognitive d is-
ord er (Kors koff synd ro e) is n nterogr d e
77. (A) Abou t 40% of those w ith lcohol u se d is- nestic d isord er c u sed by chronic thi ine
ord er w ill beco e severely d epressed w ithin d eficiency, w hich cl ssic lly presents w ith
the first w eek of bst ining fro lcohol. conf bul tion, in w hich the p tients ke up
H ow ever, if they re ble to bst in fro lco- sp ects of their life th t they c nnot re e -
hol for tot l of 4 w eeks, only 5% of p tients ber. It is c u sed by d ge to the ill ry
w ill continu e to h ve signific nt sy p to s of bod ies th t is u su lly irreversible. Conversion
d ep ression. d isord er shou ld only be d i gnosed w hen there
re sy pto s of ltered neurologic l fu nction
78. (E) This p tient continu es to experience (sensory or otor) not d equ tely expl ined
d ep ressed ood nd neu roveget tive sy p - by recognized neu rologic l or ed ic l cond i-
to s (inso ni , nergi , poor concentr tion) tions. F ctitious d isord er w ou ld be d i gnosed
6 w eeks fter the cess tion of lcohol. Dep res- if the p tient f bric ted his sy p to s in the
sive sy pto s d uring the first 4 w eeks of bsence of extern l rew rd s (i.e., to ssu e
stopping the u se of lcohol re ore likely to the sick role). M jor neu rocognitive d isord er
be su bst nce ind u ced . H ow ever, the f ct th t d ue to Alzhei er d ise se, w hich d oes present
200 6: Diffe re ntia l Dia g nos is a nd Ma na g e me nt

w ith e ory d isturb nces, does not usu lly ind ifference y rese ble the neg tive sy p-
present so cutely, nd the history of lcohol to s of schizophreni , nd ind eed , th t is in
use d isord er ke Alzhei ers less likely eti- the differenti l here; but it would be difficult to
ology. Wernicke enceph lop thy, lso c used ke d i gnosis of schizop hreni w ithout
by thi ine deficiency, is gener lly reversible evid ence of psychosis. Although this p tient
cond ition resulting in t xi , confusion, nd y, ind eed , be d epressed nd eet criteri
ocul r bnor lities. Wernickes y progress for MDD, w e re given no d irect evid ence of
to Kors koff synd ro e. this. H e exhibits none of the istrust nd sus-
piciousness seen in p r noid person lity d is-
81. (C) Mood -congru ent d elu sions re co p t- ord er. This p tient l cks the ctively od d nd
ible nd consistent w ith the st te of ind of pecu li r nneris s or eccentric w y of think-
the p tient. In this c se, the p tient feels gu ilty ing seen in schizotyp l person lity d isord er.
nd p resu bly believes he d eserves p u nish-
ent. H is d elu sions express these thou ghts. 84. (D ) Bip ol r d isord er h s one of the l rgest
If he d escribed d elu sions of gr nd iosity, th t genetic co p onents of ll the ent l illnesses.
w ou ld be n ex p le of ood -incongru ent H ving onozygotic tw in w ith bipol r d is-
d elusion. Biz rre d elusions d escribe circu - ord er incre ses the risk of d eveloping bipol r
st nces th t re i possible to be tru e, for d isord er u p to 70%.
ex p le, schizop hrenic p tient believing
co pu ter chip is i pl nted in his br in. The 85. (C) Fro s fety st nd point, the ost i por-
d elusions in this qu estion, how ever u nlikely, t nt fe tu re of h llucin tions of p tient w ith
re in so e sense conceiv ble. Ego-syntonic schizophreni is the n ture of w h t the voices
d elusions re experienced by the su fferer s re s ying. Co nd u d itory h llu cin -
ccept ble; for ex ple, the nic p tient tions telling p tient to h r or kill hi or
believing he is the gre test ctor in the w orld . herself or so eone else re often psychi t-
This p tients d elu sions re ego-d ystonic, ric e ergency, u su lly requ iring hosp it liz -
exp erienced s u n ccep t ble nd u np le s nt. tion. A possible excep tion y be w here n
So tic d elu sions focus on bod ily fu nctions exp erienced clinici n know s the p tient w ell
nd integrity. nd believes the situ tion to be s fe. All of the
other choices in this qu estion spe k to i por-
82. (C) Often, the n tu re of the d elu sion nd its t nt ch r cteristics of u d itory h llu cin tions
rel tion to ood st te c n p rovid e clu es to th t shou ld be ev lu ted , bu t the p resence of
d i gnosis. Delu sions rising out of severe co nd u d itory h llu cin tions requ ires
jor d epressive episod e, s in this c se, re i ed i te ttention.
frequently ood congru ent. Bipol r d isord er,
nic cl ssic lly p rod uces d elu sions th t re 86. (D ) N rcolepsy is ch r cterized by p ersis-
gr nd iose, rel ting to the infl ted self-estee . tent d yti e hyperso nolence th t is te -
The d elu sions of schizop hrenic p tients c n be por rily relieved by brief n ps. N rcolepsy is
biz rre nd re often u nrel ted to ood st te. often cco p nied by pheno en co only
The p resence of d elu sions ru le ou t d ju st ent ssoci ted w ith in ppropri te REM sleep,
d isord er nd persistent d epressive d isord er s includ ing hypn gogic or hypnopo pic h l-
these ffective illnesses d o not re ch severity lucin tions (vivid h llucin tions upon f lling
th t includ es d elu sion l thinking. sleep or w king up), c t plexy ( sud d en
dr tic loss of uscle tone, usu lly follow ing
83. (C) P tients w ith schizoid person lity d isord er n intense e otion l re ction), sleep p r lysis
re typic lly soci lly isol ted nd d istinctly ( loss of volunt ry uscle tone t the begin-
inept t for ing nd c rrying on interperson l ning or end of sleep, s seen in this c se), nd is
rel tionships. They often co e to the ttention ssoci ted w ith hypocretin d eficiency. H yper-
of physici ns s result of so e other issue, so nolence d isord er is lso ch r cterized by
s seen in this c se. Their pp rent p thy nd persistent sleepiness bu t c n be d istingu ished
Answe rs : 8192 201

fro n rcolepsy by the f ct th t the n ps fevers, d il ted p u p ils, d ry skin, nd confu sion.
cco p nying it re often long nd unrefresh- H yp ertensive crisis occu rs w hen p tient
ing. Circ d i n rhyth sleepw ke d isord er, is on n MAOI nd consu es tyr ine-rich
d v nced sleep-ph se type refers to ind ivid u- food s resu lting in elev ted blood p ressu re,
ls w hose soci l/ occup tion l functioning is he d ches, n u se , nd vo iting. Influ enz
d isturbed by their d row siness d uring the eve- is viru s resu lting in fevers, y lgi s, nd
ning, w here s d el yed sleep-ph se type refers resp ir tory sy p to s. Serotonin synd ro e is
to those w hose soci l/ occup tion l function- life-thre tening cond ition d u e to i trogenic
ing is d isturbed by d row siness in the orning serotonin excess; this p tient is only t king
nd lertness in the evening. P r so ni s re one serotonergic ed ic tionvenl f xine.
beh viors th t re w orsened or only occur d ur- Serotonin synd ro e is ch r cterized by leth-
ing sleep nd includ e night re d isord er nd rgy, confu sion, flu shing, clonu s, co , nd
non-REM sleep rous l d isord ers (e.g., sleep- p ossibly d e th.
w lking or sleep terrors).
90. (E) P tients on high-dose steroids re t risk for
87. (C) The p tient in this qu estion h s p reviou s developing steroid-induced psychotic disorder
opi te u se d isord er. Bec u se of this, choosing or ni . This p tient is h ving uditory h l-
non ddictive edic tions would be preferred lucin tions nd is thus experiencing psycho-
over ddictive edic tions. Diphenhydr ine sis r ther th n ni . Bipol r disorder due to
is hist ine blocker nd is not d d ictive. nother edic l cond ition (hyperthyroidis )
It c n be help fu l for p tients w ith inso - is less likely given the p tients nor l thyroid
ni . Alpr zol nd zolp id e , w hile they studies. In order to h ve di gnosis of brief
c n be benefici l, d o h ve n d d ictive poten- psychotic disorder, edic tion or edic lly
ti l. Cod eine is n opi te, nd , w hile one sid e induced psychosis ust be ruled out; steroids
effect is sed tion, it shou ld be u sed for the re well-known c use of psychosis. MDD with
relief of p in. Qu eti p ine is n ntip sychotic psychotic fe tures requires pri ry ood
w hich c u ses sed tion. H ow ever, d u e to its disorder which this p tient does not h ve. This
sid e effect profile su ch s ove ent d isor- p tient h s cute onset of uditory h llucin -
d ers, w eight g in, incre sed cholesterol, nd tions nd does not ppe r to h ve pri ry
incre sed f sting su g rs, it shou ld not be u sed thought disorder, such s schizophreni .
s first-line gent for inso ni .
91. (A) As steroid -ind uced psychosis is co only
88. (C) The p tient in this vignette h s d i gno- seen in p tients requ iring high-d ose steroid s.
sis of ADH D, w ith sy pto s of in ttention The first step in n ge ent of this p tient
nd hyp er ctivity. Tre t ent for ADH D is w ould be to d ecre se the steroid d os ge. H er
u lti od l nd inclu d es ed ic tions nd TSH nd T4 levels re nor l nd she d oes not
beh vior l od ific tions. First-line p h r - require d just ent in her levothyroxine. There
cother py is ethylphenid te, sti ul nt. is no ind ic tion to st rt nd SSRI or ood st -
Clonid ine, n lp h -2 gonist, is second -line bilizer. ECT c n be used for the n ge ent
ed ic tion for ADH D. I ip r ine, TCA, of cute psychosis, how ever, w e h ve n id en-
nd sertr line, n SSRI, re ntid ep ress nts tifi ble c use of this p tients psychosis nd
u sed to tre t nxiety nd d ep ression. Risperi- w ould first d ecre se the steroid s.
d one is n typ ic l ntip sychotic.
92. (E) Serotonin syndro e is life-thre tening
89. (D ) This vignette illu str tes p tient w ho cond ition d ue to i trogenic serotonin excess.
is show ing signs of ild to od er te op i te This p tient is on ultiple serotonergic ed i-
w ithd r w l. Sy p to s of op i te w ithd r w l c tions, includ ing phenelzine, n MAOI, tr -
inclu d e l cri tion, rhinorrhe , y w ning, d ol, nd linezolid . Together these edic tions
u scle ches, bd o in l cr p ing, nd d i r- c used the sy pto s of leth rgy, confusion,
rhe . Anticholinergic toxicity p resents w ith flushing, clonus, di phoresis, co , nd y
202 6: Diffe re ntia l Dia g nos is a nd Ma na g e me nt

c use d e th. Bipol r d isord er is pri ry self or other u tobiogr p hic l infor tion.
ood d isord er; w hile the p tient h s p revi- These p tients often h ve signific nt ou nt
ou s d i gnosis of MDD w ith psychotic fe tu res, of tr u or stressfu l event in their history.
there is no history or evid ence of ni . H yper- Bord erline p erson lity d isord er is p erv -
tensive crisis occurs w hen p tient t king n sive d isord er w ith ffect d ysregu l tion, p t-
MAOI consu es tyr ine-rich food s resu lt- terns of unst ble rel tionships, nd su icid l/
ing in rked ly incre sed BP, he d ches, n u- self- util tion beh viors. Deperson liz tion
se , vo iting, nd confu sion. Migr ines w ith d isord er is d escribed s recu rrent feelings of
ur re type of he d che w ith sy pto d et ch ent fro self s w ell s h ving occ -
( ur ) preced ing the he d che. N euroleptic sion l ou t of bod y experiences. Dissoci tive
lign nt synd ro e is n id iop thic re ction id entity d isord er, for erly know n s u ltip le
fro ntipsychotic ed ic tions. It is psychi- p erson lity d isord er, is d i gnosis of h ving
tric e ergency ch r cterized by confusion tw o or ore d istinct person lities. Multiple
or d eliriu , fluctu ting vit l signs, nd le d ch nges of id entity usu lly co-occu r w ith
pipe rigid ity. e ory d eficits in p erson l d t nd d ily
events. M jor neu rocognitive d isord er d u e
93. (B) Multiple ed ic tions in d d ition to psy- to Alzhei er d ise se is d e enti d efined
chotropic ed ic tions (e.g., SSRIs, TCAs, by gr d u l d ecline in cognitive fu nctions of
MAOIs) c n incre se serotonin nd re i por- e ory, le rning, nd l ngu ge.
t nt to be w re of. These ed ic tions includ e
tr d ol, linezolid , nd tript ns. They shou ld 96. (E) Restless legs synd ro e nifests s
ll be d iscontinu ed in p tient w ith serotonin u nco fort ble sens tions in the legs, w hich
synd ro e. The other choices re either not re relieved w ith ove ent, ostly t rest nd
i ed i te or not p p rop ri te. in the evening. This cond ition is p ro inent
c use of sleep d istu rb nce. Iron d eficiency is
94. (B) This p tient d e onstr tes ho rd ing d is- know n risk f ctor ssoci ted w ith restless legs
ord er, n in bility to d isc rd p ossessions synd ro e. GAD is d i gnosis inclu d ing sig-
reg rd less of v lu e, so u ch so th t he h s nific nt nxiety over sever l re s of life th t
cre ted n u nliv ble situ tion in his ho e. i p irs fu nctioning. Inso ni d isord ers re
The p tient find s hi self u n ble to w ork sec- collection of sy pto s th t d isru p t sleep
ond ry to incre sing stress levels. First-line d ur tion nd qu lity. N on-REM sleep rou s l
tre t ent for ho rd ing d isord er is CBT. GAD d isord ers inclu d e sleep w lking nd sleep
is d i gnosis reserved for p tients w ho h ve terrors. While they y includ e ove ents,
incre sed nxiety over u ltip le d o ins, sig- they re p rt of beh viors such s w lking,
nific ntly i p iring their bility to fu nction e ting, nd sitting u p in bed . Obstru ctive sleep
in soci l or occu p tion l settings. P tients p ne , chronic sleep cond ition, is ch r cter-
w ith OCD h ve recu rrent, intru sive thou ghts ized by p neic ep isod es resu lting in excessive
(obsessions) resu lting in rep etitive beh v- d yti e sleepiness.
iors (co p u lsions) s resp onse. OCPD is
p erson lity d isord er in w hich p tients re 97. (B) Iron d eficiency is know n c u se of rest-
rigid nd p erfectionistic, w ith no recu rrent less legs synd ro e. Rep leting iron w ou ld be
obsessions or co p u lsions. P nic d isord er the first step in n ging this p tient before
involves recu rrent, u np rovoked p nic tt cks consid ering other ph r cother py options.
nd p eriod s of extre e nxiety l sting t le st
1 onth. 98. (E) The p tient in this vignette eets criteri
for inter ittent explosive d isord er, d i gno-
95. (C) The p tient in this vignette eets criteri sis d escribed by frequent verb l or physic l
for d issoci tive nestic, w ith d issoci tive ou tbu rsts tow rd people or p roperty. These
fu gue, w hich is ch r cterized by the sud d en ou tbu rsts of ggression re ou t of proportion
tr vel fro ho e cco p nied by forgetting to the situ tion or stressor nd c u se d istress
Answe rs : 93108 203

to the p tient. P tients w ith ntisoci l p er- tion, nd often d e onstr te recu rrent suicid l
son lity d isord er h ve p ttern of viol ting thre ts, tte pts, or self- u til tion.
the rights of others nd the f ilure to confor
to soci l nor s. While this p tient d ispl ys 103. (E) H istrionic p erson lity d isord er inclu d es
nger ou tbu rsts, there re no other viol - p tients w ith excessive e otion lity nd
tion of rights nor leg l proble s consistent ttention-seeking beh viors, w ith focu s on
w ith ntisoci l p erson lity d isord er. Bip ol r p p e r nce nd ttr ctiveness. These inclu d e
II d isord er inclu d es ep isod es of n MDE and sed u ctive beh viors, over-ex gger ted e o-
hyp o ni ; ood d isord er is not d escribed tion l resp onses, nd f lse p ercep tions s to
in this vignette. Bord erline p erson lity d isor- the extent of inti cy of their rel tionship s.
d er p tients d escribe p ttern of inst bility
d e onstr ted throu gh poor ffect regu l tion, 104. (F) P tients w ith n rcissistic p erson lity d isor-
self- util tion beh viors, nd unst ble rel - d er h ve n overinfl ted sense of self-w orth,
tionship s. Cond uct d isord er is d i gnosed sense of entitle ent, nd requ ire excessive
in child hood / d olescence nd involves n d ir tion.
irrit ble/ ngry ood , d efi nt beh vior, nd
vind ictiveness th t y progress to ntisoci l 105. (H) P r noid p erson lity d isord er involves
p erson lity d isord er in d ulthood . p tient th t h s consistent d istru st of others
w ithou t su bst nti l re soning. P tients often
99. (A) Person lity d isord ers re ego-syntonic, h ve d oubts of loy lty nd perceive tt cks on
l d p tive, p erv sive beh viors th t c u se their ch r cter. While not overtly p sychotic,
signific nt i p ir ent in soci l or occu p - they c n be extre ely p r noid nd litigiou s.
tion l fu nctioning. Antisoci l person lity d is-
ord er requ ires p tient to h ve history of 106. (B) Avoid nt p erson lity d isord er is d i g-
cond u ct d isord er prior to d ulthood . These nosed in p tients w ho re extre ely sensitive,
ind ivid u ls h ve no reg rd for b sic hu n often void ing situ tions (occu p tion l or
rights, re frequ ently violent, often f il to soci l) w here they re exp ected to for new
bid e by the l w , ke i p u lsive d ecisions, rel tionship s or friend ship s. These p tients
nd re reckless. They h ve little to no re orse re extre ely shy nd feel inferior to others.
for their ctions. This c u ses the to be relu ct nt to eng ge in
new ctivities for fe r of e b rr ss ent or
100. (D ) Depend ent person lity d isord er d escribes rejection.
p tient w ith n excessive need to be t ken
c re of resulting in d ifficulty king everyd y 107. (G) P tients w ith obsessive-co pulsive per-
d ecisions nd the need for support fro others. son lity d isord er re const ntly preoccupied
w ith p erfectionis nd control t the exp ense
101. (I) Schizoid p erson lity d isord er p tients re of efficiency nd flexibility. This results in
soci lly w ithd r w n nd d o not d esire close u nw illingness to d eleg te, excessive d evotion
rel tionship s. They p refer solit ry ctives nd to w ork, nd in bility to d pt.
p pe r to h ve restricted e otions th t sp n
w id e r nge of environ ents. 108. (J) P tients w ith schizotyp l person lity d isor-
d er re often looked t s eccentric nd h v-
102. (C) P tients w ith bord erline p erson lity d is- ing od d beh vior. While not d elusion l, they
ord er h ve chronic p tterns of i pu lsivity h ve unu su l beliefs, such s gic l thinking,
nd u nst ble rel tionship s, ffect d ysregu l - cl irvoy nce, or the p r nor l.
This page intentionally left blank
CHAPTER 7

Prac tic e Te s t 1
Que s tions

D IRECTION S (Questions 1 through 107): For each Questions 3 and 4


of the multiple-choice questions in this section,
select the lettered answ er that is the one best A 44-year-old w om an p resents to her p rim ary care
response in each case. d octor w ith m u ltip le com p laints, inclu d ing w eak-
ness in her low er extrem ities, bloating, head aches,
interm ittent loss of ap p etite, and back p ain. A care-
Questions 1 and 2 fu l review of sym p tom s reveals m any other vagu e
sym p tom s. H er com p laints d ate back to ad oles-
A 29-year-old m an w ith a history of bip olar d isor-
cence and she has seen m any d octors. Thorou gh
d er presents to the psychiatric em ergency d ep art-
w orku p s, inclu d ing an exp loratory lap arotom y,
m ent saying that he is the king of Pum bar and
have failed to u ncover any clear, anatom ic, or
need s everyones allegiance for the u p com ing w ar
p hysiologic cau se.
w ith the Martians. In the p ast few d ays, he has slep t
a total of 3 hou rs bu t says that he is not tired . H e has
3. Which of the follow ing is the best ap proach to
spent all of his m oney soliciting phone sex. N ow , he
this patient?
is agitated , d em and ing, and threatening.
(A) Tell her any physical w orku p is
1. Which of the follow ing is the best treatm ent u nnecessary.
for this p atient in the acu te setting? (B) Tell her to com e back in 1 m onth and , if
(A) Carbam azep ine (Tegretol) the sym ptom s are still present, you w ill
initiate a p hysical w orku p .
(B) Divalp roex sod iu m (Dep akote)
(C) Tactfu lly ask her w hy she is inventing
(C) H alop erid ol (H ald ol)
sym ptom s.
(D) H yd roxyzine (Atarax)
(D) Assess her for other p sychiatric
(E) Lithiu m illnesses.
(E) Initiate a p hysical w orku p and arrange
2. After treating the p atient acu tely, a m ed ica-
for follow -u p in a years tim e.
tion is requ ired for ongoing treatm ent of his
bip olar d isord er. You find that he has a history
4. Which of the follow ing p ersonality d isord ers
of agranulocytosis. Which of the follow ing
w ou ld m ost likely be com orbid in the above
m ed ications w ou ld be the best choice for his
p atient?
treatm ent?
(A) Avoid ant
(A) Antip sychotic m ed ication
(B) Bord erline
(B) Carbam azep ine
(C) Obsessive-com p u lsive
(C) Divalp roex sod iu m
(D) Schizoid
(D) Lithiu m
(E) Schizotyp al
(E) Lorazep am (Ativan)

205
206 7: Pra c tic e Te s t 1

Questions 5 and 6 7. Which of the follow ing sym p tom s is consi-


d ered a negative sym ptom in this patients
A 50-year-old sin gle m an h as in creasin gly becom e p resentation?
a con cern to his n eigh bors. H e w orks at a com ic-
book store, d resses in od d , ou td ated cloth es, an d (A) Au d itory hallu cinations
d isp lays p oor eye con tact. Althou gh h e tend s (B) Delu sions
to keep to him self, h e h as told neighborhood chil- (C) Flat affect
d ren th at th ere are w itch es w h o live d ow n th e (D) Loose associations
street.
(E) Paranoia
5. Which of the follow ing is the m ost likely 8. The p atient is started on m ed ication and
d iagnosis? m any of her sym p tom s im p rove. She begins
(A) Bipolar d isord er a new job and d oes w ell. One year later, she
(B) Bord erline p ersonality d isord er is brou ght to her d octor florid ly p sychotic,
actively hearing voices, and extrem ely p ara-
(C) Schizoid p ersonality d isord er
noid . She believes that her boss is trying to kill
(D) Schizop hrenia her. She has an u p p er respiratory viral illness
(E) Schizotyp al p ersonality d isord er that she believes to be the w ork of a foreign
governm ent. She d iscontinu ed her m ed ica-
6. The patients brother brings him to a d octor tion 4 w eeks ago becau se she felt too sed ated .
because, since the d eath of their mother, the In the p ast year, her cigarette sm oking habit
patients paranoia has cau sed him to qu estion has d ecreased to one p ack p er d ay. What is the
his neighbors activities. H e has since m oved m ost likely cau se of her exacerbation?
into a hotel that he could not afford in ord er
to get aw ay from the spies living next d oor. (A) A reaction to the viral illness
Which of the follow ing is the most appropriate (B) Decreased cigarette sm oking
intervention? (C) Med ication noncom p liance
(A) Antip sychotic (D) Stress from w ork
(B) Benzod iazep ine (E) Treatm ent refractory illness
(C) N o treatm ent
Questions 9 and 10
(D) Psychod ynam ic p sychotherapy
(E) Selective serotonin reu p take inhibitor A 44-year-old man complains to his doctor that he is
(SSRI) alw ays tired and is having d ifficulty getting out of
bed in the morning. Upon questioning, he reveals he
Questions 7 and 8 has three or four d rinks each night and perhaps more
on the w eekend s, but d enies he has any problem w ith
A 25-year-old fem ale college grad u ate is brou ght alcohol. A d iagnosis of alcohol use d isord er is mad e
to her d octor by her m other. Described as od d and the patient comes to your office in acute alcohol
since she lost her job a year ago, the p atient has w ithd raw al. He subsequently has a w ithd raw al sei-
com p lained of hearing voices and believes that zure and is ad mitted to the intensive care unit.
her bod y is receiving Wi-Fi com m u nications for a
cou nterterrorist op eration. H er m other notes she 9. Which of the follow ing laboratory find ings
has been isolating herself in her room . She is alert w ou ld be m ost likely fou nd in this p atient?
and oriented bu t su sp iciou s and gu ard ed on exam -
ination. H er affect is flat and her sp eech reveals (A) Decreased p rothrom bin tim e
loose associations. A com p lete m ed ical w orku p is (B) Elevated or d ep ressed liver enzym es
negative. (C) H igh blood -alcohol level
(D) H yp erm agnesem ia
(E) Throm bocytosis
Que s tions : 514 207

10. Which of the follow ing m ed ications w ou ld be to w orsening d epressive sym ptom s and acute sui-
m ost im portant to ad m inister? cid ality. She has had m u ltiple trials of m ed ications
w ithou t significant im p rovem ent. A cou rse of elec-
(A) Diazepam
troconvulsive therap y (ECT) is d eterm ined to be the
(B) H alop erid ol next ap p ropriate step.
(C) Lorazep am
(D) Phenytoin 12. While u nd ergoing the ECT, the treatm ent
(E) Valp roic acid team w ishes to m onitor im provem ent in her
d epression. Which of the follow ing tests has
11. A 28-year-old woman is brought to the emer- the greatest reliability and valid ity for this
gency department for active suicidal ideation p u rp ose?
with a plan to overdose on acetaminophen. (A) Beck Dep ression Inventory
She has no history of a psychiatric illness but
(B) Draw -A-Person Test
endorses many criteria for major depressive
disorder (MDD), including poor sleep for the (C) H alstead -Reitan N eu rop sychological
past 2 weeks. She recently lost her job and Battery
fears that she may not be able to pay her rent. (D) Rorschach Test
Attempts to obtain collateral information have (E) Them atic Ap p ercep tion Test (TAT)
been unsuccessful. You believe the patient
requires inpatient evaluation but her insurance 13. After one of her treatm ents, the p atient com -
company denies authorization for inpatient p laints of m em ory im p airm ent. Which of the
care, alternatively authorizing eight outpatient follow ing tests w ou ld be the m ost ap p rop riate
visits with a psychiatrist. You speak to the to assess her com plaint?
weekend on-call physician-reviewer and report
(A) Beck Dep ression Inventory
that the patient remains unsafe and wishes to
be discharged. Upon learning that the patient (B) Brow n-Peterson Task
does not have a history of psychiatric illness, the (C) Bu lim ia TestRevised
reviewer fails to authorize inpatient care, despite (D) Eating Disord er Inventory 2 (EDI-2)
your assessment. Which of the following is the (E) State-Trait Anxiety Inventory
most appropriate intervention?
(A) Ad m inister an antipsychotic m ed ication 14. A 67-year-old w oman w ith a history of
and reevalu ate the p atient in 1 hou r. d epression presents to your office for evaluation.
Her symptoms of poor appetite, insomnia,
(B) Ad m it the p atient on an em ergency
low energy, and feelings of hopelessness have
certificate to an inpatient facility.
w orsened recently. She has been on several
(C) Begin antid ep ressant therap y and d ifferent serotonin-specific reuptake inhibitors
arrange for outp atient follow -u p the (SSRIs), w hich you learn have not resulted in
next d ay. complete remission of her symptoms. Which of
(D) Explain to the p atient that her the follow ing med ications w ould be the most
insu rance com p any d id not au thorize appropriate to prescribe?
hosp italization and d ischarge her w ith
follow -u p care. (A) Citalopram
(E) Prescribe a m ed ication to help her (B) Flu voxam ine
sleep, arrange for follow -up care, and (C) Paroxetine
d ischarge her from the hospital. (D) Sertraline
(E) Venlafaxine
Questions 12 and 13

A 34-year-old w om an suffering from severe m ajor


d epressive d isord er is ad m itted to the hospital d u e
208 7: Pra c tic e Te s t 1

Questions 15 and 16 (D) H yp ochond riasis


(E) Panic d isord er
A 29-year-old w om an p resents to the em ergency
d ep artm ent w ith her 3-year-old child reporting that
18. Which of the follow ing m ed ications is the m ost
the child suffered a seizu re w hile at hom e. H ospi-
efficaciou s in the treatm ent of this illness?
tal record s verify that this is the third em ergency
d ep artm ent visit in as m any w eeks for the sam e (A) Clonid ine (Catap res)
p resentation. N eu rologic w orku p for seizu re d is- (B) H alop erid ol
ord er w as negative. Initiation of an anticonvu lsant (C) Lithiu m
has been ineffective. The m other becom es very fru s-
(D) Prop ranolol
trated , d em and ing that her son be ad m itted to the
hosp ital for further testing. (E) Sertraline (Zoloft)

15. Based on the above p resentation, w hich of 19. A w om an being treated for m ajor d ep ression
the follow ing d iagnoses is m ost likely in the is brou ght to the em ergency d ep artm ent after
child ? being fou nd u nconsciou s by a neighbor. The
neighbor states that over the p ast few d ays
(A) Conversion d isord er the w om an had been com plaining of severe
(B) Factitiou s d isord er head aches. She also m entions that the w om an
(C) N o d iagnosis often enjoys red w ine. The w om ans blood
(D) Seizu re d isord er pressu re is record ed as 220/ 110 m m H g.
Which of the follow ing shou ld be ad m inistered
(E) Sep aration anxiety d isord er
im m ed iately?
16. The m other is ad d itionally m ost likely to su f- (A) Alp ha-blocker
fer from w hich of the follow ing? (B) Beta-blocker
(A) Bipolar d isord er (C) Brom ocrip tine (Parlod el)
(B) Ep ilep sy (D) Calciu m channel blocker
(C) Major d ep ressive d isord er (E) Dantrolene sod iu m (Dantriu m )
(D) Posttrau m atic stress d isord er (PTSD)
Questions 20 and 21
(E) Schizop hrenia
A 38-year-old m an w ith schizop hrenia has had
Questions 17 and 18 nu m erou s hosp italizations and m any trials w ith
variou s antip sychotic m ed ications. H e continues to
A 26-year-old m an is being evalu ated in the em er-
be sym p tom atic, w ith d erogatory au d itory hallu ci-
gency d ep artm ent for su d d en onset of chest p res-
nations, p aranoia regard ing his neighbors, and poor
sure and d yspnea. This is his third em ergency
self-care. H e d enies su icid al or hom icid al id eation.
d ep artm ent visit for sim ilar sym p tom s for w hich he
H e has recently been started on clozap ine (Clozaril),
rep orts I feel like Im going to d ie. An electrocar-
w ith significant im p rovem ent in his cond ition. H e
d iogram (ECG) and stress test w ere norm al. Urine
d oes not d rink alcohol or u se illicit d rugs, and he
toxicology w as negative. The p atient d enies risk
d enies ad d itional m ed ical history.
factors for heart d isease and d oes not have a fam ily
history of heart d isease.
20. Which of the follow ing ad verse effects is
associated w ith this m ed ication?
17. Which of the follow ing is the m ost likely
d iagnosis? (A) Brad ycard ia
(A) Acu te m yocard ial infarction (B) Galactorrhea
(B) Acu te stress d isord er (C) H yp ertension
(C) Deliriu m (D) Seizu res
(E) Weight loss
Que s tions : 1527 209

21. Which of the follow ing blood tests w ill requ ire (C) Paranoid p ersonality d isord er
frequ ent, regu lar m onitoring for this p atient? (D) Schizoid p ersonality d isord er
(A) Calciu m level (E) Schizotyp al p ersonality d isord er
(B) Com p lete blood cou nt w ith d ifferential
25. A 28-year-old sep arated w om an is referred
(C) Electrolytes
from her p rim ary care d octor for evalu ation
(D) Thyroid fu nction tests for d ep ression. While she ad m its to occasional
(E) Urinalysis p eriod s of d ysp horia, she claim s to alw ays
feel em p ty insid e. Up on fu rther qu estion-
Questions 22 and 23 ing she d em onstrates a p ervasive p attern
of u nstable relationship s, p oor self-im age,
You ad m it an 83-year-old w id ow ed fem ale for
im p u lsiveness, and irritability. Which of the
fu rther evalu ation as she is no longer able to care
follow ing d iagnoses is the m ost ap p rop riate
for herself at hom e. She has lost 30 lb in the p ast
to consid er?
year, has p oor hygiene, and ad m its to increasing
forgetfulness. (A) Antisocial p ersonality d isord er
(B) Bord erline p ersonality d isord er
22. Which of the follow ing tests w ou ld best help (C) Dep end ent p ersonality d isord er
to m ake the correct d iagnosis?
(D) H istrionic p ersonality d isord er
(A) Blessed Rating Scale (E) Schizoid p ersonality d isord er
(B) Folstein Mini-Mental State Exam ination
(MMSE) Questions 26 and 27
(C) Geriatric Rating Scale
A 28-year-old single m an w ith a 10-year history of
(D) Glasgow Com a Scale schizophrenia has been taking his m ed ications reg-
(E) Mental Statu s Exam ination (MSE) u larly. H e now p resents w ith w orsening hallucina-
tions and prom inent thought d isorganization.
23. Which of the follow ing d isord ers w ou ld be
m ost im p ortant to ru le ou t as a cau se of her 26. Which of the follow ing neu rop sychological
clinical p resentation? tests w ou ld best d eterm ine his ability to
(A) Generalized anxiety d isord er organize and correctly p rocess inform ation?
(B) Major d ep ressive d isord er (A) Bend er Gestalt Test
(C) Obsessive-com p u lsive d isord er (B) Draw -A-Person Test
(D) Panic d isord er (C) Lu ria-N ebraska N eu rop sychological
(E) Posttrau m atic stress d isord er Battery
(D) Mini-Mental State Exam ination
24. A 35-year-old patient is encou raged to seek (E) Wisconsin Card Sorting Test (WCST)
p rofessional help by his cow orkers. H e
d enies pervasive d epression or anxiety, bu t 27. The above test m easu res fu nctioning at w hich
u pon interview he is od d ly d ressed , expresses of the follow ing lobes of the brain?
u nu sual beliefs and thinking, som e paranoia
regard ing his cow orkers m otivations, and (A) Cerebellar
has few close friend s. H e d enies d elu sions (B) Frontal
or hallu cinations, and there is no su icid al or (C) Occip ital
hom icid al id eation. Which of the follow ing (D) Parietal
d iagnoses is the m ost appropriate to consid er? (E) Tem p oral
(A) Avoid ant p ersonality d isord er
(B) N arcissistic p ersonality d isord er
210 7: Pra c tic e Te s t 1

Questions 28 and 29 31. The p atient retu rns 1 m onth later for a follow -
u p exam ination and rep orts th at he exp e-
A 30-year-old w om an w ithout prior p sychiatric his- rienced a gen eralized seizu re. Laboratory
tory is brought to the em ergency d epartm ent by the investigation reveals that the p henytoin level
p olice after being arrested for breach of the p eace. is 6.5 m g/ d L (norm al, 1020 m g/ d L). Which
The w om an w as observed acting irrationally at a of the follow ing is the m ost ap p rop riate
local bu siness w here she d em and ed to sp eak w ith intervention at this tim e?
the presid ent of the com p any claim ing that she had
new id eas for prod u ct d evelopm ent. The patient (A) Ad d a benzod iazep ine.
rep orts that she has not slep t for d ays and that (B) Ad d p henobarbital.
her m ood is fabulou s. Urine hum an chorionic (C) Assess com p liance.
gonad otrop in is p ositive. Illicit su bstances w ere not (D) Discontinu e p henytoin and begin
d etected . d ivalproex sod ium .
(E) Increase the p henytoin d ose.
28. Which of the follow ing ad d itional find ings
w ou ld m ost likely be p resent in her history or
32. A 24-year-old w om an w ith a history of
m ental status exam ination (MSE)?
schizophrenia tells you that she w ou ld like
(A) Daytim e sleepiness to becom e p regnant. H er husband has no
(B) Dep ressed affect history of m ental illness. What d o you tell
(C) Racing thou ghts her is the chance of her offsp ring d evelop ing
schizophrenia?
(D) Visu al hallu cinations
(E) Weight loss (A) 1%
(B) 2%
29. Which of the follow ing m ed ical d isord ers can (C) 5%
also present w ith sim ilar sym ptom s? (D) 8%
(A) Cirrhosis (E) 12%
(B) Diabetes m ellitu s
(C) H yp erglycem ia 33. A 28-year-old m arried fem ale p atient is ad m it-
ted to the hospital w ith bizarre behavior and
(D) Rheu m atoid arthritis
d isorganized thinking. Consid eration is given
(E) Thyroid d isord er to a d iagnosis of schizophreniform d isor-
d er. To help w ith the d iagnosis, the patient is
Questions 30 and 31 ad m inistered a test that consists of view ing a
A 24-year-old m an w ith a history of seizu re d is- set of 10 inkblots sequ entially. The exam iner
ord er, alcohol, and cocaine u se d isord er has been scores the patients responses to the blots by
incarcerated for assau ltive behavior. The p atient is noting the content of the p ercep tion, the area
evaluated by a neurologist, who prescribes phenytoin of the blot that form s the basis of the response,
(Dilantin). and the asp ects of the area that are u sed to
form the resp onse.
30. Which of the follow ing sid e effects is m ost In w hich of the follow ing classes d oes this
likely associated w ith this m ed ication? projective test belong?

(A) Ebstein anom aly (A) Associations


(B) Gingival hyp erp lasia (B) Choice of ord ering
(C) H ep atic failu re (C) Com p letions
(D) H yp ertension (D) Constru ctions
(E) Leu kocytosis (E) Self-exp ression
Que s tions : 2839 211

34. A m other and her 17-year-old son p resent to 36. An 89-year-old m arried w om an w ith no p rior
you r p sychiatric clinic seeking fam ily therap y. p sychiatric history bu t a history of m u ltiple
The m other rep orts that she and her son have m ed ical p roblem s is ad m itted for failu re to
a tum ultu ou s relationship and are constantly thrive. Upon history, the patient d oes not seem
argu ing w ith each other. The son ad m its that acclim ated to her su rrou nd ings. Which of the
abou t a year ago he d isclosed to his m other follow ing tests w ou ld best help to d eterm ine
that he is gay. H e feels that because of his her level of confu sion?
m other s strong religiou s beliefs, she has not
(A) Fargo Map Test
accep ted his sexu ality and this has been the
sou rce of tension in their relationship. The (B) Sp atial Orientation Mem ory Test
m other firm ly believes that her son is ju st (C) Stroop Test
confu sed and need s cou nseling. She has (D) Tem p oral Orientation Test
heard of a particular therapy that can change (E) Wisconsin Card Sorting Test
the sexu al orientation of an ind ivid u al and
asks that you cond u ct this on her son. Which 37. A Malaysian m an is brou ght into cu stod y by
of the follow ing is the appropriate next step? the police after m u rd ering his friend . While
(A) Ask the son if he is w illing to p u rsu e he d oes not rem em ber com m itting the act, he
this treatm ent. d oes recall being insu lted by him at an earlier
tim e. Which of the follow ing cond itions is
(B) Engage the son in this therap y based off
m ost associated w ith this presentation?
on w hat you know .
(C) Inform the m other that su ch therap y is (A) Am ok
u nethical. (B) Dhat
(D) Learn m ore abou t the treatm ent first (C) Ganser synd rom e
and ask the fam ily to retu rn w hen you (D) Koro
are know led ge able enou gh to cond u ct (E) Latah
this.
(E) Refer the fam ily to a therap ist w ho Questions 38 and 39
specializes in such treatm ents.
A 26-year-old w oman is d iagnosed w ith schizophre-
35. An 18-year-old m an is referred to his college nia. The psychiatrist d ecid es to treat her symptom s
counseling center d ue to failing grad es. It is w ith a high-potency antipsychotic med ication. While
d eterm ined that he has stop ped attend ing her symptom s appear to improve on the med ication,
classes, preferring to rem ain in his d orm room . she d evelops acu te, painfu l muscle spasm s of her
H is room m ates claim that he stays u p all jaw .
night, w ith little ap parent sleep. The p atient
claim s that he has d iscovered som ething that 38. Which of the follow ing m ed ications shou ld be
w ill m ake him a m illionaire. On m ental sta- ad d ed ?
tu s exam ination, he app ears d iap horetic w ith (A) Benztrop ine (Cogentin)
p sychom otor agitation. H is speech is rap id (B) Cholinergic agonist
and his affect is eu phoric. H e d enies su icid al
(C) Clozap ine
or hom icid al id eation but d isp lays significant
grand iosity. H is insight is p oor. Which of the (D) Methylp henid ate (Ritalin)
follow ing is the m ost likely d iagnosis? (E) Risp erid one (Risp erd al)

(A) Cannabis intoxication 39. Despite compliance with the above regimen, the
(B) Cocaine (stim u lant) intoxication patient continues to suffer with extrapyrami-
(C) MDD w ith p sychotic featu res dal symptoms. As a result of this, she abruptly
(D) Schizoaffective d isord er stops taking her antipsychotic medication. It is
(E) Schizop hrenia decided to switch to another medication. Which
212 7: Pra c tic e Te s t 1

of the following medications would be the most patients mother calls you and reports that her
appropriate choice? son is more agitated and confused. Physical
examination reveals tachycardia, dilated pupils,
(A) H alop erid ol
and flushed skin. Which of the following would
(B) Loxapine (Loxitane) be the most appropriate next step?
(C) Perp henazine (Trilafon)
(A) Discontinu e benztropine and p rescribe
(D) Pim ozid e (Orap )
am antad ine (Sym m etrel).
(E) Qu etiapine
(B) Increase haloperid ol to 5 m g tid .
Questions 40 and 41 (C) Increase benztropine to 2 m g bid .
(D) Discontinu e halop erid ol and p rescribe
A 30-year-old w om an presents w ith a history of risp erid one.
chronic but rapid and intense m ood sw ings, feelings (E) Prescribe lorazep am 1 m g bid .
of em ptiness, low self-esteem , chronic suicid al id e-
ation, as w ell as frequ ent but brief sexu al relation- Questions 43 and 44
ships, anger outbu rsts, and self-m u tilation.
A 25-year-old single w om an w ho carries the p rovi-
40. Which of the follow ing d isord ers is this p atient sional d iagnosis of d epend ent personality d isord er
m ost likely to be su ffering from ? is referred for p sychological testing. You d ecid e to
(A) Avoid ant p ersonality d isord er ad m inister the Rorschach.
(B) Bipolar d isord er
43. Which of the follow ing asp ects of this typ e
(C) Bord erline p ersonality d isord er
of psychological testing w ou ld be the m ost
(D) Depend ent personality d isord er
im p ortant in this case?
(E) N arcissistic personality d isord er
(A) Ask sp ecific qu estions w ith item ized
41. The above p atient is referred for d ialectical resp onses.
behavioral therapy (DBT). Which of the fol- (B) Determ ine how a p atient feels abou t
low ing techniqu es is the m ost im portant in you .
her therapy? (C) Lack of stru ctu re allow s for a variety of
resp onses.
(A) Aversion
(D) Provid e nu m erical scores.
(B) Eye-m ovem ent d esensitization and
rep rocessing (EMDR) (E) Resu lts allow for easy statistical
analysis.
(C) Flood ing
(D) H om ew ork
44. You ad d itionally d ecid e to ad m inister the
(E) System atic d esensitization Minnesota Mu ltiphasic Personality Inventory
2 (MMPI-2). Which of the follow ing aspects of
42. A 21-year-old man presents w ith a 7-month this typ e of p sychological testing w ould be the
history of bizarre behavior, paranoid delusions, m ost im portant in this case?
and auditory hallucinations that comment on
his appearance. After a thorou gh evaluation, (A) Assesses test-taking attitud es.
you d iagnose him w ith schizophrenia and (B) Instru ctions are u nam bigu ou s in natu re.
prescribe haloperid ol 5 m g bid . One w eek later, (C) Item s d esigned to sep arate norm al
the patient retu rns for a follow -up exam ination subjects from those w ith psychiatric
and reports that, althou gh his symptom s illness.
have imp roved , he now experiences mu scle (D) More sensitive in p icking u p gend er-
stiffness in his arms and neck. You p rescribe specific issu es.
benztropine 1 mg bid and sched ule a follow -up (E) Most researched and w ith norm ative
ap pointment in 2 w eeks. One w eek later, the d ata.
Que s tions : 4049 213

45. A 28-year-old m ale m ed ical stu d ent is fou nd are no signs of trau m a or overd ose. A review
to have an enlarged testicle d u ring a rou tine of her chart reveals that the p atient w as w ell
p hysical exam ination. The stu d ent reports the night before and w ent to sleep w ithou t
that it has been grad ually enlarging for several incid ent. You d eterm ine that the p atients
m onths. The physician asks w hy he d id not u nresponsiveness is psychogenic. Which of
report these find ings earlier. Im su re its the follow ing find ings is m ost likely to be
nothing, the stud ent replies. Which of the ap p arent on her exam ination?
follow ing typ es of resp onses d oes this m ost
(A) Abnorm al electroencep halogram (EEG)
likely rep resent?
(B) Cold -caloricind u ced nystagm u s
(A) Confid ence (C) Decreased resp irations
(B) Denial (D) Elevated tem p eratu re
(C) Rationalization (E) N onsaccad ic eye m ovem ents
(D) Repression
(E) Su p p ression 48. A 24-year-old w om an is referred by her fam -
ily p ractice d octor after the Christm as holi-
46. A 45-year-old Asian-Am erican w om an is d ay. She gives a significant history of fatigu e,
brou ght to the em ergency d ep artm ent by her w eight gain, and hyp erp hagia d u ring the w in-
hu sband , w ho rep orts that for the past 3 d ays ter m onths. She d escribes a sad m ood , poor
his w ife has not been sleeping w ell, has been sleep, problem s concentrating, and cant
experiencing bad d ream s, and ap pears in a w ait u ntil her vacation to Florid a. Which of
d aze w ith a sense of feeling nu m b. The the follow ing is the m ost likely d iagnosis?
patient end orses feeling anxiou s bu t d oes not
(A) Acu te stress d isord er
know w hy. She has been u nable to p erform
her u su al activities of d aily living, poorly con- (B) Generalized anxiety d isord er
centrating on tasks and frequently ju m p ing (C) Major d ep ressive d isord er w ith seasonal
w hen startled . One w eek ago, the p atient w as p attern
d ischarged from the hosp ital after exp erienc- (D) Persistent d ep ressive d isord er
ing an anap hylactic reaction to IV contrast d ye (d ysthym ia)
w hile u nd ergoing an im aging p roced u re for (E) Su nd ow ning synd rom e
sinusitis. Althou gh she cannot recall specifics,
her husband verifies the history, ad d ing that 49. A 27-year-old w om an is brou ght to the em er-
the d octors thought she w as going to d ie. gency d ep artm ent by her p arents, w ho rep ort
Upon retu rning to the hosp ital, she exp eri- that their d au ghter is u nable to recall her
ences intense fear abou t revisiting the sam e nam e. The em ergency d ep artm ent p hysician
hospital from w hich she w as recently d is- rep orts that a com p lete neu rologic w orku p is
charged . Which of the follow ing is the m ost w ithin norm al lim its. Collateral inform ation
app rop riate d iagnosis? reveals that the p atient had ep isod es in w hich
(A) Acu te stress d isord er she w ould take u nplanned trips, som etim es
for d ays or w eeks, w ithou t notice, and w ou ld
(B) Ad ju stm ent d isord er
retu rn u nable to recall the ep isod e. A review of
(C) Generalized anxiety d isord er (GAD) her m ed ical chart notes a p ast history of pos-
(D) Major d ep ressive d isord er (MDD) sible sexual abu se as a child . Urine toxicology
(E) Posttrau m atic stress d isord er (PTSD) is negative and she d oes not take any m ed ica-
tions. Which of the follow ing is the m ost likely
47. A 40-year-old w om an w ithou t p ast p sychiatric d iagnosis?
history is ad m itted to the hospital for the treat-
(A) Deliriu m
m ent of d epression. Du ring m orning rou nd s,
the patient appears unresp onsive and d oes (B) Dissociative am nesia w ith d issociative
not, ind eed , respond to verbal stim u li. There fu gu e
214 7: Pra c tic e Te s t 1

(C) Major d ep ressive d isord er (A) Bip olar I d isord er


(D) Partial com p lex seizu res (B) Bip olar II d isord er
(E) Posttrau m atic stress d isord er (C) Cyclothym ic d isord er
(D) Major d ep ressive d isord er
Questions 50 and 51 (E) Persistent d ep ressive d isord er
A 20-year-old college stu d ent is referred for testing (d ysthym ia)
to evaluate poor acad em ic perform ance. H e rep orts
that he has alw ays stru ggled to pass his classes 53. You are asked to evalu ate a 30-year-old m ale
d espite stu d ying for m any hours. H e attend s all of p risoner w ho reports a d ep ressed m ood and
his lectures and is able to pay attention, yet he d oes suicid al id eation. Du ring your interview and
not seem to be able to ad equ ately learn the m aterial. m ental statu s exam ination, you note that
You susp ect he has a learning d isord er. the prisoner d oesnt give correct or sp ecific
answ ers, respond ing instead to you r qu es-
50. Which of the follow ing tests w ou ld be the m ost tions w ith approxim ate answ ers. Which of the
ap prop riate to help d eterm ine this p atients follow ing is the m ost ap p rop riate d iagnosis?
p roblem ? (A) Cap gras synd rom e
(A) Draw -A-Person Test (B) Dissociative am nesia
(B) Minnesota Mu ltip hasic Personality (C) Ganser synd rom e
Inventory 2 (MMPI-2) (D) Major neu rocognitive d isord er
(C) Wechsler Ad u lt Intelligence Scale (d em entia)
Revised (WAIS-R) (E) Mu nchau sen synd rom e
(D) Wechsler Intelligence Scale for Child ren
(WISC) 54. A 75-year-old m an is ad m itted to the hospi-
(E) Wechsler Mem ory Test (WMT) tal follow ing a serious su icid e attem pt. The
p atient exhibits clinical featu res of d ep res-
51. Which of the follow ing form u las is used to cal- sion w ith severe neu rovegetative sym ptom s
cu late the intelligence quotient (IQ)? and has a p ast history of su icid e attem p ts.
The m ed ical chart reveals that he is p rescribed
(A) Actu al IQ/ theoretical IQ 100 levod op a/ carbid op a, atorvastatin, asp irin,
(B) Chronological age/ p erform ance IQ and nifed ip ine for a card iac arrhythm ia. H e
100 rem ains extrem ely help less and hop eless,
(C) Mental age/ chronological age 100 w ith little ap p etite, and ongoing su icid al
(D) Mental age/ full-scale IQ 100 id eation w ith several lethal p lans. Which of
the follow ing treatm ents w ou ld be the m ost
(E) Perform ance IQ/ verbal IQ 100
ap p rop riate?
52. A 26-year-old female graduate student reports to (A) Diazep am (Valiu m )
you a 4-week history of a depressed mood that (B) Electroconvu lsive therap y (ECT)
has caused her significant difficulty in attending (C) N ortrip tyline (Pam elor)
her classes. The patient reports difficulty falling
(D) Risp erid one
asleep at night, red uced appetite and weight
loss, poor energy, and passive suicidal ideation. (E) Su p p ortive p sychotherap y
A careful review of her history reveals that for
the past 2 years she also experienced brief and 55. A 38-year-old w om an presents to your office
distinct periods of an elevated and expansive w ith an 8-w eek history of sym ptom s of
mood, a decreased need for sleep, and an d ep ressed m ood w ith an increased appetite
increase in activities, although she was still able and 10-lb w eight gain, hyp ersom nia, a heavy
to function adequately. Which of the following feeling in her bod y, and rejection sensitivity.
is the most likely diagnosis? Up on fu rther qu estioning she ad m its to being
Que s tions : 5060 215

able to brighten w hen sp end ing tim e in activi- d ie bu t d enies any plan or intent. Upon d is-
ties she u sually enjoys. She rep orts to you that cussion w ith the w ife present, they both con-
she recently enrolled in grad uate school and firm a p ast episod e w here for several w eeks
is having trou ble w ith m any of her classes. last year he w as u p , w ith increased energy,
Which of the follow ing d iagnoses is the m ost little sleep , w anting to have sex several tim es
likely? p er d ay, talking fast, gam bling a large su m of
m oney, and attem pting to rem od el the hom e
(A) Ad ju stm ent d isord er
d esp ite having no experience. H e d enies
(B) Major d epression w ith atypical features hallu cinations or p aranoia. H e occasionally
(C) Major d ep ression w ith m elancholic d rinks one to tw o glasses of w ine on w eek-
featu res end s and d enies any illicit d ru g u se. H e has
(D) Persistent d epressive d isord er not been taking any m ed ications recently as
(d ysthym ia) he thou ght he cou ld tou gh it ou t. Which of
(E) Sleep d isord er the follow ing m ed ications w ou ld likely be the
m ost ap p rop riate to begin?
56. A 33-year-old m an changes his first nam e (A) H alop erid ol
to honor a m u sician w hom he id olizes. H e
(B) Lam otrigine
recently bou ght the sam e gu itar as the m u si-
cian and form ed a rock band to play his m usic. (C) Lithiu m
Du ring p ractice, the m an d resses like his id ol. (D) Sertraline
Which of the follow ing d efense m echanism s (E) Valproic acid
d oes this behavior best rep resent?
Questions 59 and 60
(A) Fixation
(B) Id ealization A 42-year-old healthy m an is u nd ergoing a lot of
(C) Id entification stress at his w ork. As a result, he is having a d if-
(D) Projection ficu lt tim e falling and staying asleep . H is p rim ary
care d octor has since p rescribed him a m ed ication
(E) Regression
to help w ith his insom nia. While he is sleep ing bet-
ter, he calls one m orning several w eeks later w ith
57. A 60-year-old m an w ith schizop hrenia sits
com plaints of a very painfu l penile erection w hich
m otionless in his chair. The patient is m u te and
has lasted throu ghout the evening.
reacts very little to his environm ent. H is eyes
ap pear fixated on a d istant object. At tim es,
59. Which of the follow ing m ed ications has m ost
he assum es bizarre postu res and im itates the
likely been p rescribed ?
m ovem ents of others. Which of the follow ing
is the best d escrip tion of his behavior? (A) Flu oxetine (Prozac)
(A) Absence seizu re (B) N ortrip tyline (Pam elor)
(B) Catalep sy (C) Paroxetine (Paxil)
(C) Catap lexy (D) Sertraline (Zoloft)
(D) Catatonia (E) Trazod one (Desyrel)
(E) Partial com plex seizu re
60. Which of the follow ing is the m ost ap propri-
ate next step ?
58. A 37-year-old m arried m ale presents w ith a
history of m ultiple d epressive ep isod es. For (A) Tell him to d ecrease the d ose.
the past 5 w eeks he has felt sad , w ith crying (B) Tell him to stop the m ed ication and
spells, term inal insom nia, low energy, but restart in 3 d ays.
increased ap p etite and w eight gain. H e has (C) Tell him to stop the m ed ication and
had passive su icid al id eation of w anting to m onitor his cond ition.
216 7: Pra c tic e Te s t 1

(D) Tell him to stop the m ed ication and go d ischarge, she is still exp eriencing significant p ain.
im m ed iately to the em ergency room . She has no other m ed ical problem s or allergies. She
(E) Tell him to stop the m ed ication and is cu rrently taking oral contracep tives and a m ono-
have sexual intercourse or m asturbate. am ine oxid ase inhibitor (MAOI) for her d epression.

Questions 61 and 62 63. Which of the follow ing analgesics shou ld be


m ost avoid ed in this p atient?
A 66-year-old man reports a history of excessive
w orry about his d aughter since she moved aw ay (A) Acetam inophen
from the area 1 year ago. H is w ife of 43 years verifies (B) Cod eine
his complaint, ad d ing he w orries about everything. (C) Ibu p rofen
Recently, his w ife mad e plans to travel abroad to (D) Mep erid ine
visit friend s. The patient is u nable to accomp any his (E) Oxycod one
w ife because of chronic obstructive pulmonary d is-
ease cau sed by years of heavy smoking and is very 64. Tw o m onths later the patient retu rns for a fol-
anxious about her leaving. H e reports su bsequent low -u p visit and rep orts that she is p regnant.
d ifficulty falling asleep, excessive d aytim e fatigue, Which of the follow ing interventions is the
and trou ble concentrating at w ork. m ost app rop riate?

61. Which of the follow ing m ed ications w ou ld be (A) Continu e the MAOI.
the m ost ap propriate to treat his sym p tom s? (B) Discontinu e the MAOI.
(A) Alprazolam (Xanax) (C) Discontinu e the MAOI and initiate
treatm ent w ith flu oxetine after 2 w eeks.
(B) Busp irone (Bu Spar)
(D) Discontinu e the MAOI d u ring the first
(C) Diazep am (Valiu m )
trim ester only.
(D) Lorazep am (Ativan)
(E) Discontinu e the MAOI and begin
(E) Propranolol (Ind eral) m aintenance ECT.

62. Several w eeks later, the patient rep orts that 65. A 62-year-old hom eless m an p resents to
the m ed ication you prescribed has helped the em ergency d epartm ent w ith confu sion,
everything bu t his d ifficu lty falling asleep . H e agitation, im p aired gait, and nystagm u s. H is
rep orts that d u ring the p ast w eek he has slep t vital signs are stable and an ophthalm ologic
only 4 hours p er night. After carefu l consid er- exam ination is w ithin norm al lim its. The
ation, you d ecid e to begin a trial of a 1-w eek p atient is u nable to recall the d ate and has
su pply of m ed ication to help his insom nia. d ifficu lty su staining attention. Which of
Which of the follow ing m ed ications w ou ld be the follow ing is the m ost appropriate initial
the m ost app rop riate to prescribe? intervention?
(A) Diphenhyd ram ine (Benad ryl) (A) Ad m inister IV d extrose.
(B) Lorazepam (Ativan) (B) Ad m inister p arenteral thiam ine.
(C) Oxazep am (Serax) (C) Obtain a com p u ted tom ograp hic scan of
(D) Tem azep am (Restoril) the brain.
(E) Zaleplon (Sonata) (D) Perform a breathalyzer.
(E) Perform a lu m bar p u nctu re.
Questions 63 and 64

A 25-year-old w om an w ith a history of m ajor 66. You are a community psychiatrist who is seeing
d epressive d isord er, single episod e, in rem ission is a patient for the first time. The patient informs
ad m itted to the hosp ital for rem oval of a fibroid . H er that he has ju st m oved into the area and
p roced u re is com pleted w ithout d ifficu lty, but u p on need s to establish him self w ith a p sychiatrist
Que s tions : 6171 217

to continue his treatm ent for d ep ression and ap p ears confu sed and is d iap horetic. A u rine toxi-
anxiety. On fu rther evalu ation, you learn that cology screen is negative.
he w as previou sly arrested for child p ornog-
rap hy and served several years in p rison. H e 68. Which of the follow ing is the m ost im portant
is cu rrently in therap y to ad d ress this issu e. intervention?
You are su rp rised to find that after the d isclo-
(A) Ad m inister brom ocrip tine.
sure, you feel angry tow ard the patient as you
have tw o you ng d au ghters of you r ow n. You (B) Ad m inister IV d antrolene sod iu m .
are concerned abou t w hether it m ay be d iffi- (C) Ad m inister naloxone (N arcan).
cult for you to treat the patient in an u nbiased , (D) Ap p ly cooling blankets.
nonjud gm ental m anner becau se of his crim i- (E) Discontinu e m ed ications.
nal history. Which of the follow ing is the next
m ost ap prop riate step ? 69. Which of the follow ing laboratory abnorm ali-
(A) Continu e to see the patient bu t lim it the ties is m ost likely to be p resent in this patient?
tim e spent w ith him . (A) Anem ia
(B) Im m ed iately refer the patient to another (B) Decreased transam inases
p hysician in you r clinic. (C) Elevated blood u rea nitrogen (BUN )
(C) Inform the p atient that you are u nable (D) Elevated creatine p hosphokinase (CPK)
to see him and refer him back to the
(E) Leu kop enia
com m unity.
(D) Obtain the patients therap y record s so 70. An 18-year-old m an is brou ght to th e
that you are aw are of his progress. emergency department by the paramedics after
(E) Seek a consu ltation w ith an exp erienced being involved in a motor vehicle accident. His
colleague regard ing you r feelings medical chart reports a history of substance use.
tow ard the p atient. Which of the following tests can most likely
confirm the diagnosis of chronic substance use
67. A 39-year-old execu tive w ithout p sychiatric with physical dependence?
history bu t a history of hyp ertension rep orts
d rinking up to six cu ps of caffeinated coffee (A) Breath analysis
p er d ay. H e board s a p lane sched u led for an (B) Elevated heart rate
18-hou r flight on w hich only d ecaffeinated (C) N aloxone injection
beverages are served . Which of the follow ing (D) Seru m liver function stu d ies
sym ptom s is he m ost likely to experience? (E) Urine toxicology screen
(A) Depressed m ood
71. A 72-year-old man without prior psychiatric
(B) H ead ache
history presents to the outpatient clinic with a
(C) Irritability
recent history of memory difficulty. The patient
(D) Mu scle cram p ing has had a stable level of consciousness and
(E) N au sea denies any current or past substance use. He has
not been prescribed any new medications. The
Questions 68 and 69 medical chart reveals a history of megaloblastic
anemia and a subtotal gastrectomy for severe
A 25-year-old m an w ith a history of schizophrenia
peptic ulcer disease. Which of the following ini-
p resents to the em ergency d ep artm ent w ith severe
tial interventions is the most appropriate?
m u scle stiffness and an elevated tem p eratu re. H is
casew orker claim s that he is com pliant w ith his (A) Discu ss long-term extend ed -care facility
m ed ications and d enies that he u ses alcohol or ad m ission.
d rugs. H is vital signs d em onstrate elevated tem - (B) Obtain a forensic exam ination to
p erature, blood p ressu re, and p u lse. The p atient evalu ate com p etency.
218 7: Pra c tic e Te s t 1

(C) Obtain a neu rologic consu lt. (A) BUN and creatinine
(D) Ord er a m agnetic resonance im aging (B) CBC
scan of the brain. (C) ECG
(E) Ord er a thorou gh laboratory w orku p . (D) Liver fu nction stu d ies
(E) Thyroid -stim u lating horm one
72. A 37-year-old w om an presents to the em er-
gency d ep artm ent com p laining of severe d iar- 76. A 27-year-old w om an has been prescribed
rhea, nau sea, and a coarse trem or. The m ed ical flu oxetine for d ep ression for the p ast year. She
chart notes a history of bip olar I d isord er. com es to you com plaining of m ed ication sid e
Which of the follow ing w ou ld be the m ost effects and w ishes to d iscontinu e her antid e-
ap prop riate initial intervention? p ressant. As her p hysician, you d iscu ss w ith
(A) Obtain renal fu nction stu d ies. her the ap p rop riate risks and benefits, and
(B) Obtain a seru m m ed ication level. you d iscontinu e her treatm ent. Tw o w eeks
later, she retu rns com p laining of d ep ressive
(C) Obtain a u rine p regnancy test.
sym p tom s characterized by sleep and appetite
(D) Obtain a u rine toxicology screen. changes, poor concentration, and a d epressed
(E) Prescribe antid iarrheal m ed ication. m ood for m ost of the d ay. You d eterm ine that
she is experiencing a recurrent m ajor d ep res-
Questions 73 and 74 sive episod e and d ecid e to prescribe her a
d ifferent class of m ed ication to treat her sym p -
A 72-year-old m an is brou ght into the em ergency
tom s. After d iscussing the appropriate precau -
room via am bu lance after su ffering w hat is su s-
tions w ith her, you prescribe an MAOI. Within
pected to be a stroke in his right p arietal region.
1 w eek, she begins exp eriencing irritability,
abd om inal pain and d iarrhea, and au tonom ic
73. Which of the follow ing areas w ould be m ost
instability characterized by hyp ertension and
likely to have d eficits d u e to the stroke?
tachycard ia. H er tem p eratu re is 103.5F. The
(A) Long-term im plicit m em ory p atient also rep orts exp eriencing jerking of
(B) Long-term p roced u ral m em ory her m uscles and vivid visu al hallucinations
(C) Tactile m em ory of colorfu l flow ers spinning tow ard the sky.
(D) Verbal m em ory She states that she has follow ed your d irec-
tions carefu lly w hile taking this m ed ication.
(E) Visu al nonverbal m em ory
Which of the follow ing cond itions m ost likely
accou nt for this p resentation?
74. Which of the follow ing tests w ould best id en-
tify the abnorm ality in that region? (A) H allu cinogen intoxication
(A) Minnesota Mu ltip hasic Personality (B) Malignant hyp ertherm ia
Inventory 2 (C) N eu rolep tic m alignant synd rom e (N MS)
(B) ReyOsterrieth Test (D) Serotonin synd rom e
(C) Rorschach Test (E) Tyram ine-ind u ced hyp ertensive crisis
(D) Wechsler Intelligence Scales
Questions 77 and 78
(E) Wisconsin Card Sorting Test
A 17-year-old girl is referred to the school nu rse for
75. A 32-year-old m arried m an is d iagnosed w ith
frequ ent ep isod es of vom iting in the bathroom d u r-
bip olar I d isord er. Variou s treatm ent op tions
ing lu nch breaks. H er friend s rep ort that, d esp ite
are d iscussed , and he agrees to begin lithiu m
alw ays talking abou t w anting to lose w eight, she
carbonate to treat his sym ptom s of m ania.
eats tw ice as m u ch as anybod y else. The p arents
Which of the follow ing tests is the m ost
are called to the school and recall that a recent bill
im p ortant to ord er p rior to initiating treatm ent
from their charge accou nt at the local p harm acy
w ith lithiu m ?
Que s tions : 7282 219

ind icated a large nu m ber of laxative p u rchases. The 80. Which of the follow ing com p lications w ou ld
girl rep orts that her m ood is fine, and her record s be the m ost likely in this p atient?
at school ind icate that she is an above-average
(A) Diarrhea
stu d ent.
(B) H eat intolerance
77. Which of the follow ing m ed ical com p lications (C) Leu kocytosis
w ou ld be m ost likely in this p atient? (D) Menorrhagia
(A) Acid osis (E) Osteop orosis
(B) Dental carries
81. Which of the follow ing w ou ld be the m ost
(C) Diarrhea likely p red iction of a good ou tcom e in this
(D) H yp erkalem ia patient?
(E) H yp erchlorem ia
(A) 10%
78. Which of the follow ing m ed ications w ou ld be (B) 20%
the m ost ap propriate initial treatm ent for her (C) 50%
sym p tom s? (D) 75%
(A) Bu propion (E) 90%
(B) Citalop ram
Questions 82 and 83
(C) Lithiu m
(D) Risp erid one A 70-year-old w om an rep orts a d ep ressed m ood
(E) Valp roic acid an d insom n ia 1 m on th follow in g th e d eath of
her h u sban d . Im m ed iately after h is d eath , sh e
79. A 70-year-old m an w ith m u ltip le m ed ical began to feel th at sh e w ou ld be better off d ead ,
problem s is su sp ected to have had a stroke, alth ou gh d en ies an y active su icid al id eation . For
affecting his ability to speak. Which of the th e first several w eeks sh e h ad a red u ced ap p e-
follow ing tests w ou ld best assess the natu re of tite w ith a several p ou nd w eigh t loss, bu t this h as
his speech d ifficu lty? im p roved . At tim es, sh e believes th at sh e can h ear
his voice calling to her. Sh e d en ies any feelin gs
(A) Bend er Gestalt Test of w orthlessness bu t feels gu ilty abou t n ot bein g
(B) Boston Diagnostic Ap hasia Exam ination able to d o th e right th in gs for h im before h e d ied .
(C) Folstein MMSE Sh e occasionally gets d istracted and forgets that
(D) Sentence Com p letion Test he h as d ied . Sh e is able to brigh ten w hen sp eak-
in g abou t sp en d in g tim e w ith h er gran d ch ild ren .
(E) Stroop Test
Wh en talkin g w ith oth ers, sh e believes th at h er
feelin gs are n orm al.
Questions 80 and 81

A 16-year-old fem ale high school stu d ent is referred 82. Which of the follow ing is the m ost likely
by her parents for an evalu ation. One year prior to d iagnosis?
the evalu ation, the girl began restricting her food (A) Ad ju stm ent d isord er w ith d ep ressed
intake and started a rigorou s exercise program to m ood
im prove her appearance. Aspiring to be a m od el,
(B) Bereavem ent
the girl lost 25 lb bu t rem ained preoccu pied w ith her
app earance d espite w eighing only 85 lb. H er friend s (C) Major d ep ressive ep isod e
rep orted that she constantly referred to herself as (D) Persistent d ep ressive d isord er
being fat and d id not seem interested in d ating. (d ysthym ia)
The girl continu ed to lose w eight and w as relu ctant (E) Schizoaffective d isord er
to d iscu ss her cond ition.
220 7: Pra c tic e Te s t 1

83. Which of the follow ing is the m ost app rop ri- 86. Which of the follow ing d iagnoses w ou ld be
ate course of action? the m ost likely cause of his sym ptom s?
(A) Ad m ission to a hosp ital. (A) Circad ian rhythm sleep w ake d isord er
(B) Evalu ate for a sleep d isord er. (B) Major d ep ressive d isord er
(C) N o treatm ent. (C) Obstru ctive sleep ap nea hyp op nea
(D) Prescribe antid ep ressant m ed ication. (D) Rap id eye m ovem ent (REM) sleep
(E) Prescribe sed ative-hyp notic m ed ication. behavior d isord er
(E) Restless legs synd rom e
84. A 25-year-old m an rep orts a 5-year history of
excessive hand w ashing and a p reoccu p ation 87. Which of the follow ing is the treatm ent of
w ith feeling clean. The thou ght of contracting choice for his sym ptom s?
an infectiou s d isease p ersists throu ghout the
(A) Antid ep ressant m ed ication
d ay even thou gh he m akes attem p ts to ignore
it. H is cond ition has p rogressively w orsened (B) Benzod iazep ine m ed ication
and has cau sed significant im p airm ent w hile (C) Breathing air u nd er p ositive p ressu re
at w ork and at hom e. Which of the follow ing (D) N asal su rgery
m ed ications w ou ld be the best initial choice to (E) Uvu lop alatop lasty
treat his sym ptom s?
(A) Antid epressant Questions 88 and 89
(B) Antiep ilep tic A 60-year-old m ale p atient w ith schizop hrenia w ho
(C) Antip sychotic has been stable for years on a low -p otency antip sy-
(D) Benzod iazep ine chotic agent begins exp eriencing parkinsonian-like
(E) Lithiu m sid e effects. H is p hysician p rescribes a d ru g to alle-
viate som e of these sid e effects. One w eek later, the
85. A 28-year-old w om an w ith no p sychiatric his- p atient is seen in the em ergency d ep artm ent w ith
tory is arrested for shop lifting. She claim s she d ilated pup ils, d ry m ou th, w arm skin, and tachycar-
d oes not rem em ber d etails of the crim e, and d ia. H e is also exp eriencing the new onset of visu al
that for a w hile she has had trou ble recalling hallu cinations.
d etails from things shes recently read . You are
called to assess her, bu t w ant to ensure shell 88. Which of the follow ing w ou ld be the m ost
give good effort on testing. Which of the fol- ap p rop riate m ed ication to ad m inister?
low ing tests w ou ld be the m ost ap p rop riate to (A) Anticholinesterase
ad m inister? (B) Atrop ine
(A) Benton Visu al Retention Test (C) Benzod iazep ine
(B) Clock Draw ing (D) Dantrolene sod iu m
(C) Test of Mem ory Malingering (E) H alop erid ol
(D) Wechsler Intelligence Scale
(E) Wisconsin Card Sorting Test 89. After the ap p rop riate intervention, the p atient
exp eriences nau sea and vom iting and su bse-
Questions 86 and 87 qu ently has a seizu re. Which of the follow ing
m ed ications shou ld be ad m inistered next?
A 52-year-old obese m an exp eriences excessive d ay-
(A) Atrop ine
tim e sleepiness and a d epressed m ood . H is w ife
rep orts that he snores lou d ly and is restless w hile (B) Ep inep hrine
sleeping. There is no evid ence of regu lar alcohol or (C) Lorazep am
illicit su bstance u se, and the p atient d oes not have a (D) Physostigm ine
p rior p sychiatric history. (E) Prochlorp erazine (Com p azine)
Que s tions : 8395 221

90. A 40-year-old w om an com p lain s of 3 to 93. A 48-year-old w oman successfu lly completes
4 m onths of feeling sad , w ith trou ble falling an inpatient program for alcohol d etoxification
asleep , d ecreased ap petite and som e w eight for w hich she w as prescribed chlord iazepoxid e.
loss, d ifficu lty concentrating at w ork, and Upon d ischarge, the patient is prescribed
fatigu e. She ad m its to su icid al id eation w ithou t d isulfiram. Soon after d ischarge, the patient
sp ecific plan or intent. There is no history of attend s an office party w here she ad m its to
m ania. She also m entions longstand ing and having a few d rinks. She has been comp liant
repetitive thoughts w here she w orries abou t w ith her prescribed med ication and d oes not
germ s. As a resu lt, she w ashes her hand s have any active med ical problem s. Which of
very frequ ently, som etim es end ing u p w ith the follow ing symptom s is she most likely to
cracked and chapp ed skin. She realizes that experience?
her w orries are unrealistic, but she cannot
(A) Blu rred vision
keep from d oing it. Which of the follow ing
m ed ications w ou ld be the m ost app rop riate to (B) Eu p horia
p rescribe? (C) H igh blood p ressu re
(D) Urinary retention
(A) Busp irone
(E) Vom iting
(B) Clom ip ram ine (Anafranil)
(C) Doxep in (Sinequ an) 94. A 22-year-old m an w ith a history of bipolar d is-
(D) Flu voxam ine (Lu vox) ord er is prescribed lithium carbonate to treat
(E) Phenelzine (N ard il) his sym p tom s. Du ring a w eekend ru gby tou r-
nam ent, he hu rts his knee and an orthoped ic
Questions 91 and 92 physician p rescribes a m ed ication to red u ce
his sym p tom s of p ain and sw elling. Although
A 56-year-old m an w ith a long history of alcohol u se the patient reports relief from this m ed ication,
d isord er and elevated liver function tests is ord ered he begins to exp erience abd om inal pain, d iar-
by the court to enroll in an inp atient d etoxification rhea, and d row siness. Which of the follow ing
program for his alcohol u se d isord er after nu m erou s m ed ications w ou ld m ost likely contribu te to
DWIs. After 2 d ays in treatm ent, he begins to exp eri- the prod u ction of these sym p tom s?
ence trem ors, sw eating, flu shing, and anxiety.
(A) Acetam inophen
91. Which of the follow ing m ed ications w ou ld be (B) Asp irin
the m ost approp riate to prescribe? (C) Cod eine
(A) Alp razolam (D) Ibup rofen
(B) Chlord iazep oxid e (Libriu m ) (E) Mep erid ine
(C) Disu lfiram (Antabu se)
95. A 32-year-old new ly m arried m an w ith a p ast
(D) Lorazepam
history of m ajor d ep ressive d isord er p resents
(E) Phenobarbital w ith a 4-w eek history of d ep ression, insom -
nia, anergia, p oor concentration, and anhe-
92. Shortly after this tim e, the m an begins to d onia. H e has had p assive su icid al id eation
rep ort visu al hallu cinations and becom es agi- w ithou t a p lan, as w ell. H e has not taken
tated . Which of the follow ing m ed ications citalop ram for several years, and , d esp ite
shou ld be prescribed to treat these sym ptom s? rem ission of his sym p tom s, he had signifi-
(A) Fluoxetine cant sexu al d ysfu nction w hen taking it. H e
(B) H alop erid ol w ishes to restart a m ed ication bu t is greatly
concerned abou t the im p act on his m arriage.
(C) Lithiu m
Which of the follow ing m ed ications w ou ld be
(D) Lorazep am
the m ost ap p rop riate?
(E) Sertraline
222 7: Pra c tic e Te s t 1

(A) Citalop ram 99. Which of the follow ing is the m ost effective
(B) Flu voxam ine treatm ent for this type of d rug abuse?
(C) Paroxetine (A) Abstinence
(D) Mirtazapine (B) Antid ep ressant agents
(E) Venlafaxine (C) Antip sychotic agents
(D) Dialectic behavioral therapy
96. A patient w ho is hu m an im m u nod eficiency
(E) Exp osu re and resp onse p revention
viru s (H IV)-p ositive rep orts 4 w eeks of a
d epressed m ood , low energy, p oor sleep, and
100. A 30-year-old man w ith schizophrenia is
hopelessness. H is app etite is negligible, and
brought in w ith his mother, w ith w hom he
he has been refu sing to eat or d rink anything
lives. His med ical chart d ocuments numerous
for 2 d ays. Which of the follow ing m ed ications
but adequate medication trials including
w ou ld be the m ost appropriate to p rescribe?
risperidone, olanzapine, quetiapine, and halo-
(A) Bupropion perid ol. Despite his compliance he continues
(B) Buspirone to have significant paranoia, d elusions, and
(C) Flu oxetine aud itory hallucinations, telling him that he is
(D) Methylphenid ate going to die. H e has no other medical problems,
and both he and his mother d eny any alcohol or
(E) Sertraline
d rug use. Which of the follow ing medications
w ould be the most appropriate to prescribe for
97. A 43-year-old w om an p resents w ith a new
this patient?
history of d ep ressive sym p tom s, inclu d ing
insom nia, poor ap petite and w eight loss, low (A) Arip ip razole
energy, and d istractibility. On m ental statu s (B) Clozap ine
exam ination she is asked to cou nt backw ard (C) Flu p henazine
by 7s, beginning at 100. Which of the follow ing
(D) Qu etiap ine
asp ects is being assessed by this test?
(E) Zip rasid one
(A) Attention
(B) Fu nd of know led ge 101. You are treating a 32-year-old w om an for bip o-
(C) Mathem atics skills lar I d isord er w ith a com bination of m ed ica-
(D) Rem ote m em ory tions. She d evelops hair loss w hich eventually
resolves on its ow n. Which of the follow ing
(E) Verbal m em ory
m ed ications is m ost likely resp onsible for this
sid e effect?
Questions 98 and 99
(A) Carbam azepine
A 19-year-old college stu d ent rep orts that he and
(B) Clozap ine
his friend s have been experim enting w ith hu ffing
on cam pu s. H is room m ate rep orts that he has been (C) Divalp roex sod iu m
accu mu lating typew riter correction fluid , nail polish (D) Olanzap ine (Zyp rexa)
rem over, and m od el airp lane glu e. (E) Zip rasid one (Geod on)

98. Which of the follow ing w ou ld he m ost likely 102. A 34-year-old man is referred by his internist
exp erience d u ring intoxication? for depression. For the past 2 months he has
(A) Conju nctival injection been suffering from anhedonia, crying spells,
frequent aw akenings, poor appetite, and low
(B) Depressed reflexes
energy. He is subsequently diagnosed w ith a
(C) Dim inished response to pain major d epressive episod e and recommend ed to
(D) Increased ap petite begin citalopram 20 mg daily. After discussion
(E) Staring into space of the risks and benefits of the med ication, he
Que s tions : 96107 223

expresses concern about possible sexual d ys- (C) Ad d an anxiolytic m ed ication.


function as he is currently in a new relationship (D) Discontinu e m ethylp henid ate and
w ith a cow orker. Which of the follow ing symp- p rescribe a m ixed am phetam ine
toms are the most likely w ith this med ication? com pou nd .
(A) Decreased libid o (E) Discontinu e m ethylp henid ate and
(B) Prem ature ejaculation p rescribe an antid ep ressant.
(C) Priap ism
105. A 35-year-old w om an presents w ith ep isod ic
(D) Retrograd e ejaculation anxiety and com p laints that occasionally
(E) Sexually transm itted d isease w hen hearing som ething for the first tim e, that
she has actually heard it before. She expresses
Questions 103 and 104 her concern that she is going crazy. Which
of the follow ing term s best d escribes this
A 21-year-old m ale college stud ent w ithou t p rior
p henom enon?
m ed ical or psychiatric history is evalu ated for p oor
w ork p erform ance. The stu d ent rep orts to the d ean (A) Dj entend u
that he find s it hard to follow through w ith assign- (B) Dj vu
m ents and is easily d istracted by environm ental (C) Folie d eu x
stim uli. The d ean com m ents to the stu d ent abou t
(D) Jam ais vu
how frequ ently he interru p ts others d u ring conver-
sations and observes that it seem s that the stud ent (E) La belle ind iffrence
is not p aying attention to w hat he is saying. Over-
106. A 19-year-old m an is brou ght to the em er-
all, his grad es are p oor excep t in the class he con-
gency d ep artm ent by the p olice for an evalu -
sid ers the m ost interesting class I have ever had .
ation. The w ritten p olice rep ort states that
H e often forgets requ ired m aterials for classes and
the p atient has been calling 911 for the past
his assignm ents are frequ ently late. The stu d ents
5 w eeks, reporting that he is being spied on
p arents report that he w as evaluated for sim ilar d if-
by aliens from a d istant p lanet. The p atient
ficu lties w hile in p u blic school, bu t he seem ed to
rep orts that he is receiving m essages from the
im p rove w hen the fam ily m oved ou t of the area and
aliens throu gh his com pu ter and that he hears
he attend ed p rivate school.
voices in his head com m enting on his appear-
103. Which of the follow ing is the m ost likely ance. H e has not been sleep ing w ell at night
d iagnosis? becau se he has been gu ard ing his bed room .
You note that his affect is flat, and he ap p ears
(A) Attention-d eficit/ hyperactivity d isord er tired d uring your examination. A urine toxicol-
(ADH D) ogy screen is negative. Which of the follow ing
(B) Bipolar I d isord er is the m ost likely d iagnosis?
(C) Bip olar II d isord er (A) Bip olar I d isord er w ith p sychotic
(D) Cond u ct d isord er featu res
(E) Learning d isord er (B) Brief p sychotic d isord er
(C) Major d ep ressive d isord er (MDD) w ith
104. The p atient is p rescribed m ethylp henid ate for p sychotic featu res
the above sym ptom s. After an ad equate trial
(D) Schizoaffective d isord er
and d ose, he continu es to d isp lay som e sym p -
tom s w hich interfere w ith his school fu nction- (E) Schizop hreniform d isord er
ing, althou gh he d enies significant sid e effects.
Which of the follow ing actions w ou ld be the 107. An 18-year-old w oman w ith recently d iag-
m ost ap prop riate next step? nosed schizophrenia is acutely psychotic and
in labor w ith her first child . The obstetrics ser-
(A) Ad d a low -d ose am phetam ine. vice requests a psychiatric consultation for an
(B) Ad d an antid ep ressant. appropriate and safe med ication to use in ord er
224 7: Pra c tic e Te s t 1

to control the patients psychotic symptoms. 110. A 7-year-old boy w ith d ifficu lty in school need s
Which of the follow ing w ould be the best fu rther evalu ation of acad em ic p otential.
choice?
(A) Chlorp rom azine (Thorazine) 111. You w ou ld like to ad d ress the d eficits of a
(B) H aloperid ol (H ald ol) 69-year-old m an w ith nonflu ent sp eech and
form u late a sp ecific treatm ent p lan.
(C) Loxap ine (Loxitane)
(D) Perphenazine (Trilafon)
Questions 112 through 116
(E) Thiorid azine (Mellaril)
Select the d ru g m ost likely to cause the associated
D IRECTION S (Questions 108 through 116): The
sym ptom s.
follow ing group of questions are preceded by a
list of lettered options. For each questions, select (A) Cocaine
the one lettered option that is most closely asso- (B) Lysergic acid d iethylam ine (LSD)
ciated w ith it. Each lettered option may be used (C) Mariju ana
once, multiple times, or not at all.
(D) N icotine
Questions 108 through 111 (E) Op iates
(F) Phencyclid ine (PCP)
Match the clinical presentation w ith the ap propriate
neu ropsychological test. 112. An 18-year-old high school student with con-
junctival redness, increased appetite, dry mouth,
(A) Beck Depression Inventory tachycardia, and a sensation of slowed time.
(B) Bend er Gestalt Test
(C) Blessed Rating Scale 113. A 31-year-old m an w ith m iosis, brad ycard ia,
(D) Boston Diagnostic Aphasia Exam ination hyp otension, hypotherm ia, and constipation.
(E) Mini-Mental state Exam ination (MMSE)
(F) Minnesota Mu ltiphasic Personality 114. An assau ltive 26-year-old m an w ith vertical
Inventory 2 (MMPI-2) nystagm u s, echolalia, paranoid id eation, and
(G) ReyOsterrieth Test hallucinations.
(H ) Rorschach Test
(I) Wad a Test 115. A 16-year-old girl w ith abd om inal cram ps,
(J) Wechsler Ad u lt Intelligence Scale confu sion, palpitations, and m u scle tw itching.
Revised (WAIS-R)
(K) Wechsler Intelligence Scale for Child ren 116. A 21-year-old m an w ith tachycard ia, d ilated
(WISC) p upils, hallucinations, and com p laints of chest
p ain.
108. A 65-year-old fem ale p hysician w ho is now
concerned about m em ory d ifficulties.

109. A 35-year-old w om an w ith a p rotein S d efi-


ciency show s evid ence of left sid e hem i-neglect.
Ans we rs a nd Expla na tions

1. (C) Antip sychotics are ind icated for acu te w ith som atic sym p tom d isord er, it is im p or-
treatm ent of agitation and violence som etim es tant to u nd erstand that sym ptom s are not con-
seen in m anic patients. H aloperid ol w orks sciou sly prod u ced and to let p atients know
relatively qu ickly (2030 m inu tes). Doses of that you realize their sym p tom s are a source of
2 to 5 m g by m ou th (PO) or intram u scu larly great consternation. Regard less of history, any
(IM) are u su al, althou gh it m ay be given IV p atient presenting to a p hysician w ith p hysi-
as w ell, especially in the intensive care u nit cal com plaints d eserves a reasonable physical
(ICU). H yd roxyzine is an antihistam ine; it is investigation; p atients w ith som atic sym p -
not effective in m ania. Lithium , d ivalproex tom d isord er are as likely, if not m ore likely,
sod ium , and carbam azepine have all been to d evelop id entifiable m ed ical cond itions.
show n to control m ood fluctu ations in m anic H ow ever, rather than rep eat tests, it m ay be
p atients. H ow ever, these agents take d ays to necessary to contact previous treaters. Send -
w ork and are not effective in the im m ed iate ing this p atient aw ay for a year w ou ld not be
m anagem ent of this patient. help ful; rather regular, frequ ent checku ps and
fostering a therap eu tic alliance and su p p ort
2. (D ) Lithium is not associated w ith signifi- are in ord er.
cant blood d yscrasias, althou gh it can cau se
a m od est benign increase in the w hite blood 4. (C) Ap p roxim ately 50% of ind ivid u als w ith
cell (WBC) cou nt. Carbam azepine is com - som atic sym p tom d isord er have ad d itional
m only associated w ith a benign red u ction p sychiatric illnesses, esp ecially anxiety and
in WBC cou nt, bu t severe blood d yscrasias d ep ression. Personality traits or a d isord er is
only occu r in app roxim ately 1 in 125,000 not u ncom m on. Obsessive-com pu lsive per-
p atients. Agranulocytosis is also a rare com - sonality d isord er is the m ost likely com orbid
p lication w ith d ivalproex sod iu m , although a p ersonality d isord er in som atic sym ptom -
benign throm bocytop enia is m ore com m on. d isord ered p atients.
Agranulocytosis can also occu r in the setting
of antipsychotic m ed ication u se in rare cases 5. (E) This patient is most likely suffering from
(clozap ine has a higher incid ence). Lorazep am schizotypal personal disorder, which as with all
is not associated w ith WBC abnorm alities bu t personality disorders, represents a lifelong mal-
is not a m ood stabilizer. adaptive approach to life and does not suddenly
develop in ad ulthood . Schizotypal personality
3. (D ) This patient is m ost likely suffering from disorder with its attendant social isolation and
som atic sym ptom d isord er, w hich often coex- subtle distortions of reality may ind eed resem-
ists w ith other p sychiatric illnesses su ch as ble schizophrenia; the diagnosis can be sorted
anxiety, d ep ressive, and p ersonality d isor- out by a thorough history. Frank hallucinations
d ers. The su sp ected p resence of a som atic of any type are uncommon; only subtle dis-
sym ptom d isord er should prom pt a search for tortion of environmental cues is seen. There is
other treatable illnesses. In m anaging patients no evidence of mania to justify a diagnosis of

225
226 7: Pra c tic e Te s t 1

bipolar disorder. While under stress, patients com p liant w ith her m ed ication, her exacerba-
with borderline personality disorder can have tion is not likely d u e to treatm ent refractory
so-called micropsychotic episodes, they dis- schizop hrenia.
play chronic feelings of emptiness, mood/ affect
lability, self-mutilation, and intense anger and 9. (B) Classically, p atients w ith hep atitis second -
impulsivity. Schizoid personality disorder may ary to alcohol use d isord ers have elevated liver
appear similar to schizotypal personality disor- enzym es (su ch as gam m a-glu tam yl transp ep -
der, although it is characterized more by feel- tid ase, aspartate transam inase, or alanine
ings of d etachment, lack of friends, choosing transam inase). H ow ever, in ad vanced alco-
solitary activities, and anhedonia. holism , the liver m ay be bu rnt ou t and liver
fu nction tests m ay reveal low or norm al levels
6. (A) In schizotyp al p ersonality d isord er, the of these enzym es. As a resu lt of liver d am -
subtle d isconnection from reality, w hich m ay age, prothrom bin tim e is typ ically increased .
be exacerbated in tim es of stress as in this case, H ypom agnesem ia, not hyperm agnesem ia, is
can be treated w ith low d oses of antip sychotic m ore likely to be fou nd in alcohol u se d isor-
m ed ication. These patients lack the cap abil- d er, usu ally as a resu lt of d ietary d eficiency. In
ity, stable sense of self, and tru st to be able the p atient having a w ithd raw al seizure, the
to engage in, or benefit from , p sychod ynam ic p roblem is the absence, not presence, of alco-
p sych oth erap y. An tid ep ressant m ed ication hol. H is blood alcohol w ou ld be exp ected to
of any type, includ ing tricyclics and SSRIs, is be low or zero. A p atient w ith alcohol u se d is-
helpful in the schizotypal patient w ho d isplays ord er is m ore likely to have throm bocytop enia
significant affective (mood ) symptoms, w hich rather than throm bocytosis.
are not seen in this case. Benzodiazepines are
not ind icated in this patient as there is no evi- 10. (C) The p atient has alcohol u se d isord er and
d ence of anxiety. has su ffered a w ithd raw al seizu re. The m ain-
stay of alcohol d etoxification rem ains ben-
7. (C) This p atient is m ost likely su ffering from zod iazep ines, w hich m ay be ad m inistered
schizophrenia. Sym ptom s of schizop hrenia p arenterally. Lorazep am is com m only u sed ,
are com m only d ivid ed into positive and nega- since it can be given PO, IM, or IV, and it is
tive sym ptom s. Flat affect, a negative sym p- not d ep end ent on ad equ ate liver fu nctioning
tom , represents an absence of a norm ally (w hich m ay be com p rom ised in alcoholics)
reactive and variable affect. Other negative for its m etabolism . While d iazep am (Valiu m )
sym ptom s inclu d e alogia and avolition. Au d i- can be u sed , it has erratic absorp tion w hen
tory hallucinations, d elu sions (inclu d ing p ar- given IM, and , in a p atient w ith liver d am -
anoid d elu sions), and loose associations are age, its p rolonged half-life can create a large
all positive sym p tom s. bu ild u p w ith su bsequ ent oversed ation and
p otential resp iratory d ep ression. H alop eri-
8. (C) The exacerbation seen is m ost likely p re- d ol is an antip sychotic m ed ication w hich
cip itated by m ed ication noncom p liance. A m ay help w ith agitation seen in alcohol w ith-
viral illness or stress from w ork m ay ind eed d raw al, bu t it d oesnt treat the u nd erlying
contribu te to a relap se, bu t they are less p roblem and m ay fu rther low er the seizu re
strongly p red ictive of a reem ergence of p sy- threshold . While antiseizu re m ed ications
chotic sym p tom s than m ed ication noncom - su ch as carbam azep ine have been show n to
p liance. N icotine has been show n to low er be effective in the m anagem ent of alcohol
neu rolep tic levels, w hich has been offered w ithd raw al, they are not consid ered to be the
as a reason cigarette sm oking is ram p ant first-line treatm ent.
am ong p atients w ith schizop hrenia. H ow -
ever, this p atients sm oking d ecreased , w hich 11. (B) While the p atient d oes not have a history
if anything w ou ld be exp ected to increase of m ental illness, w hich provid es inform ation
antip sychotic levels. Since she has not been abou t p ast behaviors, she is actively su icid al
Answe rs : 617 227

w ith a p lan, and attem p ts to contact collaterals Utilizing an antid ep ressant w ith a d ifferent
have been unsu ccessfu l. Collateral inform a- m echanism of action (affecting d ifferent
tion w ou ld be help fu l to valid ate the p res- neu rotransm itters) m ay be beneficial. Venla-
ent history as w ell as to p rovid e d ata abou t faxine is a serotonin and norepinephrine reu p-
baseline m ental statu s. Given the absence of take inhibitor, w hich, in higher d oses, w ill
collateral inpu t and the fact that she rem ains increase the intrasynap tic levels of both neu -
suicid al, the p atient shou ld not be d ischarged rotransm itters. All the other choices are SSRIs.
from the em ergency d epartm ent and shou ld
be involu ntarily ad m itted u nd er certificate 15. (C) This scenario represents a case of factitiou s
and petition. The p atient d oes not exhibit d isord er im posed on another (form ally facti-
p sychotic behavior and is not a m anagem ent tious d isord er by proxy), in w hich one person
p roblem ; the ad m inistration of an antip sy- falsifies illness in another to vicariously gain
chotic m ed ication can cau se sid e effects and m ed ical attention. Com m only, the victim is a
m ay affect fu tu re com pliance w ith p sycho- child and the perp etrator is the child s m other.
tropic m ed ications. The other choices d o not Ap p arent bleed ing, seizu res, and central ner-
ad equ ately ad d ress her safety. vous system (CN S) d epression are typical pre-
sentations. The d isord er is und erd iagnosed
12. (A) The Beck Dep ression Inventory is a w id ely d ue both to the elu sive and crafty p lanning
u sed test that allow s clinicians to follow the of the perp etrator and the u nw illingness of
severity of previously d iagnosed d ep ression. health care p rovid ers to accu se the ostensibly
The Draw -A-Person Test, Rorschach Test, and caring parent. Given the age of the child , it is
TAT are all typ es of p rojective testing. Projec- extrem ely u nlikely that the child , him self, is
tive tests, although useful clinical tools, often intentionally p rod u cing the sym p tom s, and
suffer low reliability and valid ity. Projective there is no evid ence that he is experiencing
tests requ ire a person skilled at this type of p hysiologic seizu res. Conversion d isord er is
evalu ation and d o not alw ays have rigorou s characterized by the p resence of one or m ore
em p irical d ata and grou p com p arison. The neu rologic sym p tom s that are incom p atible
evalu ation of d ep ression u sing the H alstead - w ith a recognized m ed ical or neu rologic d isor-
Reitan Test is lim ited by the fact that m any d er. Paralysis, blind ness, and pseud oseizu res
d ep ressed patients fail to show d eficits on are com m on conversion d isord er sym p tom s.
such classic neu ropsychological batteries. Sep aration anxiety d isord er is characterized
In ad d ition, these tests, even w hen d em on- by excessive and inap p rop riate anxiety con-
strating d eficits in cognitive d om ains su ch as cerning sep aration from hom e or to w hom
attention and learning, still have very lim ited ind ivid u als are attached .
u sefulness in evaluating the severity of the
d ep ression. 16. (C) Mothers w ith factitiou s d isord er im p osed
on another are m ost likely to have u nd erly-
13. (B) The Brow n-Peterson Task is a test sp ecifi- ing m ajor d ep ressive d isord er. Bord erline
cally d esigned to evalu ate short-term m em ory, p ersonality d isord er is the m ost com m on
a cap acity that can be affected d u ring ECT. The p ersonality d isord er associated w ith facti-
Bulim ia TestRevised and the EDI-2 are both tiou s d isord er im p osed on another. Bip olar
u seful for the evalu ation of bu lim ia and eating d isord er and PTSD have not been clearly
d isord ers, resp ectively. The Beck Depression d escribed in the literatu re. Ep ilep sy is not
Inventory and State-Trait Anxiety Inventory associated w ith the d isord er to any sig-
are u sed to evaluate d epression and anxiety nificant extent, and schizop hrenia has been
d isord ers, respectively. rep orted in only a few cases.

14. (E) The patient has m u ltiple episod es of 17. (E) Panic d isorder is characterized by the sud-
d ep ression, u nfortunately w ithou t rem ission d en unexpected occurrence of panic attacks
d esp ite several m ed ication trials w ith SSRIs. (period s of intense anxiety or fear accompanied
228 7: Pra c tic e Te s t 1

by som atic sym p tom s), com m only cau sing ind icated . N either brom ocrip tine, a d op am ine
the m isd iagnosis of a m ed ical illness such as agonist, nor d antrolene, a m u scle relaxant, is
m yocard ial infarction. The frequ ency of p anic ind icated .
attacks varies w id ely from m any per d ay to a
few per year. Panic d isord er is often associated 20. (D ) Clozap ine is an effective antip sychotic
w ith agorap hobia, the fear of being in p laces m ed ication that has been associated w ith
w here escap e is d ifficu lt. The lifetim e p reva- few er extrapyram id al sid e effects than the con-
lence is up to 4%. Concerns of d eath from car- ventional antipsychotics (w hich prim arily act
d iac or respiratory d isord ers occu r frequ ently. by blocking d op am ine typ e 2 recep tors). Over
Acu te stress d isord er is d iagnosed in ind ivid - 4% of p atients taking m ore than 600 m g/ d
u als w ho have experienced a trau m atic event of clozap ine experience clozapine-associated
and subsequ ently d evelop sym p tom s w ithin seizu res. Tachycard ia, hyp otension, sed ation,
4 w eeks of the event that rem it after 1 m onth. fatigue, and w eight gain have all been asso-
The trau m atic event m u st be reexp erienced ciated w ith clozapine treatm ent. Clozapine,
by the p atient, and this cau ses the p atient to u nlike the conventional antip sychotic agents,
avoid stim uli that arouse recollections of the d oes not affect prolactin secretion and thus
trau m a. Other criteria necessary for the d iag- d oes not cause galactorrhea.
nosis are the presence of d issociative sym p -
tom s and m arked anxiety or increased arou sal 21. (B) Agranu locytosis is a p otentially life-
not better accounted for by other m ed ical or threatening sid e effect of clozapine treatm ent.
p sychiatric illnesses. Delirium is characterized It is d efined as a d ecrease in the nu m ber of
by the su d d en onset of a d istu rbance attention WBCs, w ith a sp ecific d ecrease in the nu m -
and aw areness, as w ell as a change in cogni- ber of neu trop hil granu locytes. It occu rs in
tion, cau sed by another m ed ical cond ition, ap p roxim ately 1% of all p atients treated w ith
substance (alcohol, d ru g, m ed ication), and / or clozapine. Therefore, a com plete blood cou nt
toxin. w ith d ifferential is requ ired w eekly in the
beginning of treatm ent and frequ ently there-
18. (E) Of the choices listed , sertraline, an SSRI, after. Rou tine m onitoring w ith the other tests
is the m ost efficaciou s. In ad d ition, tricyclic are not requ ired or necessary w ith clozap ine
antid ep ressants, MAOIs, and the benzod i- treatm ent.
azep ines are also effective in the treatm ent
of p anic d isord er. Beta-ad renergic d ru gs like 22. (B) The Folstein MMSE is a frequently u sed
p rop ranolol are not effective for the treatm ent screening assessm ent for m ajor neu rocogni-
of p anic d isord er. H alop erid ol, an antip sy- tive d isord er (d em entia), althou gh m ore in
chotic agent; lithiu m , a m ood stabilizer; and d ep th and form al neu ropsychological test-
clonid ine, an alp ha-2-ad renergic agonist, are ing shou ld be p erform ed at som e p oint. It is
ineffective. a 30-p oint scale w ith d ed u ctions for incorrect
answ ers. Scores less than 25 are su ggestive of
19. (A) H yp ertensive crisis is a p otentially life- cognitive im p airm ent. Scores less than 20 ind i-
threatening com plication that occu rs w hen cate d efinitive im pairm ent. The Blessed Rat-
p atients taking an MAOI eat tyram ine-con- ing Scale is a tool that typ ically asks a p atients
taining food s su ch as w ine, beer, pickled food s, friend s or relatives to assess the ability of the
and aged cheese. Clinical featu res includ e p atient to fu nction in his or her cu rrent envi-
hyp ertension, severe occipital head ache, stiff ronm ent. The Geriatric Rating Scale is a rat-
neck, nausea, vom iting, and sw eating. IV ing scale for nonp rofessional staff to evalu ate
p hentolam ine, an alp ha-ad renergic recep tor p atients abilities to p erform their activities of
blocker, is the treatm ent of choice. It has been d aily living and interact w ith others. It m ay be
show n to be m ore effective and safer than beta m ost help ful in evalu ation of the m od erately
blockers or calciu m channel blockers. Ad m is- to severely d em ented ind ivid ual. The Glasgow
sion to an ICU and su pportive m easu res are Com a Scale is an easy-to-perform instrum ent
Answe rs : 1827 229

that evalu ates level of consciou sness. There for themselves. Patients w ith d epend ent per-
are three general categories that the exam - sonality d isord er require an excessive amount
iner tests: eye op ening, verbal resp onse, and of ad vice, reassu rance, and approval from oth-
best m otor resp onse. Each category receives ers. Ind ivid u als w ith histrionic personality d is-
a num ber for p atient resp onse. Overall scores ord er, like those w ith bord erline personality
range from 3 to 15, w ith low er scores reflect- d isord er, may d isplay excessive emotionality
ing m ore severely im p aired consciou sness. A and attention-seeking behavior, but their core
m ental statu s exam ination is the form al p sy- sym ptoms center around su perficial sed u c-
chiatric exam ination that includ es m any item s tiveness and theatricality. Schizoid p ersonal-
such as appearance, behavior, sp eech, assess- ity d isord er represents a pervasive pattern of
m ent of m ood and affect, p resence of p sycho- d etachment from social relationships and a
sis, and evaluation of insight and jud gm ent. It restricted range of expressed emotions.
is not a su bstitu te for the Folstein MMSE.
26. (E) In the WCST, exam inees are asked to sort
23. (B) When evaluating the MMSE score in elderly card s d ep icting variou s p ictu res and sym bols
patients, it is critical to consid er that major accord ing to a variety of d ifferent criteria
d epression (also know n as pseud od ementia) that change over tim e w ithou t the su bjects
may prod uce similar impairments in cognition. know ing. The WCST assesses a p ersons abil-
The other d iagnoses listed w ould not com- ity to sw itch sets, reason abstractly, and solve
monly red uce the Folstein MMSE score. p roblem s. These cap acities are also know n
as execu tive fu nctions and are localized in
24. (E) Schizotyp al p ersonality d isord er is char- the frontal lobes. Cu rrent research regard -
acterized by a p ervasive p attern of social ing the p sychop athology of schizop hrenia
and interp ersonal d eficits. Ind ivid u als d em - su ggests that there are abnorm alities in the
onstrate a red u ced cap acity to establish any frontal lobes, sp ecifically in the d orsolateral
close relationship s, and eccentric behavior is p refrontal cortex, that are reflected in p oor
p resent. The p resence of od d beliefs or m agi- p erform ance on the WCST. Peop le w ith
cal thinking sep arates schizotyp al p ersonality schizop hrenia p erform m ore p oorly on the
d isord er from schizoid p ersonality d isord er. WCST than p eop le w ithou t schizop hrenia;
Paranoid id eation is com m on in both schizo- how ever, ind ivid u als w ith d am age to their
typ al and p aranoid p ersonality d isord ers frontal lobes from a variety of cau ses also
bu t not in the others. Avoid ant p ersonality show execu tive fu nction d eficits. The Bend er
d isord er rep resents a p ervasive p attern of Gestalt Test involves cop ying figu res, w hich
behavior characterized by social inhibition, help s d eterm ine if organic brain d isease is
feelings of inad equ acy, and hyp ersensitivity p resent. The Draw -A-Person Test requ ires
to negative evalu ation. N arcissistic p ersonal- the exam inee to d raw a p erson. It w as ini-
ity d isord er rep resents a p ervasive p attern tially d evised to test intelligence in child ren,
of grand iosity, lack of em p athy, and a need bu t is now u sed p rim arily as a screening test
for ad m iration. Schizoid p ersonality d isord er for brain d amage. The Luria-N ebraska N eu-
rep resents a p ervasive p attern of d etachm ent ropsychological Battery is a com prehensive
from social relationship s and a restricted set of neu ropsychological tests used to assess
range of exp ressed em otions. specific cortical areas and aid s in assessm ent
of hem ispheric d om inance. The Mini-Mental
25. (B) Bord erline p ersonality d isord er is a per- State Examination is a com monly used scale to
vasive pattern of instability of interpersonal assess the possibility of a major neurocognitive
relationships, self-im age, affect, and marked d isord er (d em entia).
impulsivity. Antisocial personality d isord er
d escribes ind ivid uals w ith long histories of 27. (B) The WCST assesses a p ersons ability
d isregard for the rights of others and often to sw itch sets, reason abstractly, and solve
d ishonesty used in attem pts to gain som ething p roblem s. These cap acities are also know n as
230 7: Pra c tic e Te s t 1

execu tive fu nctions and are localized in the m ay be as high as 12%, com p ared w ith a > 1%
frontal lobes. The cerebellum is responsible risk in the general p op u lation.
for coord ination. The occipital lobe is d ed i-
cated to the sense of sight. The parietal lobe 33. (A) The p atient is being evalu ated w ith the
hand les sensory inp u t, m otor outpu t, and Rorschach Test, w hich w as d evelop ed in
visu ospatial processing. The tem p oral lobe is 1921 by H erm ann Rorschach. It consists of
resp onsible for the sense of sou nd , taste, and show ing a su bject a set of 10 inkblot stim u li
m em ory storage. in a sequ ential m anner w hile noting (1) the
resp onses in relationship to the content of the
28. (C) This p atient is m ost likely su ffering from p ercep tion; (2) the area of the blot that form s
bip olar I d isord er. Racing thou ghts, p ressu red the basis of the resp onse; and (3) the asp ects
speech, expansive m ood , a d ecreased need for of the area that are u sed to form the resp onse.
sleep, and an increase in goal-d irected activity In 1961, Lind zey p rop osed a m ethod of clas-
are all com m on m anifestations of m ania. The sifying p rojective tests based on the typ e of
p atient is u su ally energized d u ring the d ay activity. The Rorschach is classified in the
even after only a few hou rs of sleep . Dep ressed category of associations. Another test in this
m ood and w eight loss are m ore characteristic category is the Word Association Test. In the
of a d ep ressive d isord er. Visu al hallu cinations choice of ord ering category, p atients p lace
cou ld be present in severe cases of m ania but objects in a rank ord er w ith resp ect to p refer-
are m ore com m only a sym ptom of a p rim ary ence. Tests su ch as the Sentence Com p letion
p sychotic d isord er su ch as schizophrenia. Test fall into the category of com p letions, in
w hich a p erson com p letes an u ncom p leted
29. (E) Many d isord ers can m im ic sym p tom s of stim u lu s. The constru ction category requ ires
m ania. A com p lete history, physical exam i- the su bject to constru ct content based on
nation, and rou tine laboratory tests can su ffi- a stim u lu s, su ch as a story in the Them atic
ciently ru le ou t m ost m ed ical cau ses of m ania. Ap p ercep tion Test. The self-exp ression cat-
They inclu d e end ocrine d isord ers su ch as egory consists of tests su ch as the Draw -A-
thyrotoxicosis and Cu shing synd rom e, hypo- Person Test, in w hich the su bject p rod u ces a
glycem ia, electrolyte d isord ers, su bstance u se resp onse w ithou t a stim u lu s.
and w ithd raw al, m ed ications su ch as steroid s
and anticholinergic agents, nu tritional d efi- 34. (C) It is not ethical to engage in therapy (i.e.,
ciencies, and CN S insu lts. conversion therapy) to change the sexual orien-
tation of a patient. The American Psychiatric
30. (B) Gingival hyperplasia is associated w ith Association opposes any psychiatric treatment,
ad m inistration of phenytoin. Other, d ose- such as reparative or conversion therapy w hich
related sym p tom s inclu d e nystagm u s, d izzi- is based u p on the assu m p tion that h om osex-
ness, slu rred speech, ataxia, m ental confu sion, u ality p er se is a m en tal d isord er or based
and d ecreased coord ination. Ebstein anom aly u p on th e a p riori assu m p tion that th e p atien t
is associated w ith lithiu m therap y, and hep atic shou ld chan ge his/ h er sexu al h om osexu al
failure is associated w ith d ivalproex sod iu m . orien tation . Ultim ately, p er APAs p rin ci-
p les of m ed ical eth ics a p sych iatrist sh ou ld
31. (C) Before any p harm acologic changes are not be a p arty to any typ e of p olicy that
consid ered , the physician m u st assess com - exclu d es, segregates, or d em ean s the d ign ity
p liance w ith m ed ications. The subtherap eu tic of any p atient becau se of ethn ic origin , race,
p henytoin level m ay be d ue to p atient non- sex, creed , age, socioecon om ic statu s, or sex-
com pliance, especially if sid e effects are being u al orien tation . All the other ch oices w ou ld
exp erienced . be consid ered u nethical.

32. (E) The risk of schizophrenia among first-degree 35. (B) Cocaine (stim u lant) intoxication can
relatives w here one p arent has schizop hrenia ap p ear sim ilar to a m anic ep isod e associated
Answe rs : 2838 231

w ith bip olar I d isord er w ith an increase that w ord . This test seem s to be prim arily
in energy, eu p horia, grand iosity, p ressu red an assessm ent of the ability to concentrate.
speech, and im p aired ju d gm ent. While can- The WCST is u sed to evalu ate set shifting,
nabis intoxication can p resent w ith eup horia, or execu tive fu nctioning of the brain.
p aranoia, and im paired ju d gm ent (in associa-
tion w ith physical sym ptom s of conju nctival 37. (A) Culture-bound synd rom es d enote recu r-
injection, increased ap p etite, d ry m ou th, and rent, locality-sp ecific p atterns of behavior. The
tachycard ia), a d ecreased need for sleep, d ia- synd rom e of am ok is a cu ltu re-bou nd syn-
p horesis, and rap id speech are not consistent d rom e of Malaysian origin that refers to a vio-
w ith cannabis intoxication. The acu te onset lent or fu riou s ou tbu rst w ith hom icid al intent.
of sym ptom s m akes the d iagnosis of m ajor Fou r d efining characteristics are p rod rom al
d ep ression u nlikely. A d iagnosis of schizoaf- brood ing, a hom icid al ou tbu rst, p ersistence in
fective d isord er requires that d u ring the p eriod reckless killing w ithou t an ap p arent m otive,
of illness there are sym ptom s that su ggest both and a claim of am nesia. The attack typ ically
a schizophrenia and a m ood d isord er (m ajor resu lts in m u ltip le casu alties and is m ost com -
d ep ressive or m anic episod e). Schizophrenia m on in you ng m en w hose self-esteem has been
requ ires the p resence of significant p sychotic inju red . Dhat is a Sou th Asian term referring
sym ptom s d uring a 1-m onth p eriod , w hich to anxiety regard ing the loss of sem en. Ganser
is not evid ent in this case; the p atient exp eri- synd rom e is characterized by a patient w ho
enced eu p horia w hich is not characteristic of resp ond s to qu estions by giving ap p roxim ate
schizophrenia. N egative sym ptom s (e.g., flat- or ou tright rid icu lou s answ ers. Ad d itional
tened affect, avolition, alogia) are also m ore features m ay also inclu d e altered consciou s-
com m on in schizophrenia. ness, hallu cinations, conversion p henom enon,
and am nesia for the ep isod e. Koro is another
36. (D ) The Tem poral Orientation Test asks Malaysian term that refers to su d d en anxi-
the patient to id entify the appropriate d ay, ety that the p enis (or vu lva in fem ales) w ill
m onth, d ay of the w eek, and cu rrent tim e. reced e into the bod y and cau se d eath. Latah
Deviation from the correct resp onse is d iffer- is also of Malaysian or Ind onesian origin,
entially scored in each category. Total score m ore frequ ent in m id d le-aged w om en, and
and the d ata incorrectly id entified sep arate the is characterized by su d d en fear, often w ith
p atients into tw o grou ps: p atients w ith brain d issociation and catatonic-like featu res (e.g.,
d am age and patients w ithou t brain d am age. echolalia, echop raxia).
The Tem p oral Orientation Test is also sensi-
tive to cognitive abnorm alities in d em enting 38. (A) Antip sychotic (neu roleptic)-ind u ced extra-
illnesses. The Fargo Map Test assesses recent pyram id al sid e effects d u e to the blockad e
and rem ote sp atial m em ory and visu osp atial of d opamine are comm on in the treatm ent of
orientation by using m aps of the United States psychosis. The higher-potency antipsychotic
and d ifferent regions w ithin it. Patients are med ications are more likely to cau se extrapy-
asked to id entify certain areas. Ed u cation level ram id al sid e effects than the low er-potency
and age influ ence the score on this test. The ones. Extrap yram id al sid e effects includ e acu te
Spatial Orientation Mem ory Test evalu ates d ystonic reactions (as in this case), akathisia
the ability to im m ed iately recall the orienta- (restlessness), pseud oparkinsonism, and tar-
tion of figu res and is u sed to evalu ate im m e- d ive d yskinesia. Acute d ystonia is treated w ith
d iate m em ory. The Stroop Test has a nu m ber anticholinergic agents such as benztropine and
of d ifferent form ats, but the general concep t antihistamines. Other op tions inclu d e red ucing
is that it takes longer to correctly id entify a the d ose or sw itching to a low er-p otency agent
color than to read w ord s and longer yet to cor- or atypical antipsychotic (such as risperid one).
rectly id entify a w ord (e.g., nam e of a color) N either of these are preferable at this point as
w hen that w ord is in a color d ifferent from the patients sym ptoms have im proved .
232 7: Pra c tic e Te s t 1

39. (E) Becau se of this p atients continu ed extra- abou t a stressful event w hile tracking an object
p yram id al sym ptom s d espite treatm ent w ith back and forth across their vision. Flood ing is
an anticholinergic, the m ost app rop riate another behavioral intervention u sed for pho-
choice w ou ld be to sw itch to an atypical anti- bias, w here a p atient is exp osed to a feared
p sychotic m ed ication. Atypical, or second - object d irectly. This is in contrast w ith system -
generation antip sychotics, su ch as qu etiap ine, atic d esensitization, w here a p hobic p atient is
are believed to act on variou s su bp op u lations grad u ally exp osed to increasingly feared situ -
of d opam ine receptor su btyp es as w ell as on ations u sing a hierarchy of anxiety-p rovoking
other neurotransm itter system s, inclu d ing triggers.
serotonergic. They are consid ered atyp ical
agents because they are associated w ith a low er 42. (A) The p atient is exp eriencing anticholinergic
risk of extrapyramid al sym ptoms. Clozapine, toxicity as evid enced by d ilated p up ils, d ry or
introd uced to the United States in 1989, is the flu shed skin, agitation, confu sion, and tachy-
first antipsychotic to be labeled atypical. Other card ia. Ad d itional m anifestations includ e d is-
atyp ical or second -generation m ed ications orientation and u rinary retention. More severe
includ e risperid one, olanzapine, ziprasid one, toxicity m ay resu lt in hypertherm ia or com a.
lu rasid one, asenap ine, and arip ip razole. The In this case, the p atient rep orted benefit from
other choices are all consid ered to be typical halop erid ol bu t experienced extrap yram i-
antipsychotics. d al sid e effects, com m on to the high-p otency
antip sychotic m ed ications. Benztrop ine, an
40. (C) Bord erline p ersonality d isord er is a anticholinergic m ed ication, is p rescribed to
p attern of instability in interpersonal rela- alleviate extrap yram id al sid e effects, bu t in
tionships, self-im age, affect, and m arked this patient it cau sed anticholinergic sid e
im p u lsivity, inclu d ing inap p rop riate anger effects. Am antad ine is an antiviral m ed ication
and self-m u tilation. Avoid ant personality that is also used to help alleviate extrapyra-
d isord er rep resents a p ervasive p attern of m id al sid e effects and should be institu ted to
behavior characterized by social inhibition, rep lace benztrop ine. The other choices w ou ld
feelings of inad equ acy, and hyp ersensitivity not be appropriate in this case.
to negative evaluation. Bipolar d isord er is an
affective illness characterized by alternating 43. (C) Objective tests u sually involve questions
p eriod s of m ania and m ajor d epression. Fea- w ith lists of p ossible resp onses. These tests
tu res of d epend ent personality d isord er are p rovid e num erical scores on w hich statisti-
an excessive need to be taken care of, clinging cal analyses are easily perform ed . Exam ples
behavior, and fears of sep aration. N arcissistic inclu d e the United States Med ical Licensing
p atients d em onstrate a pattern of grand iosity, Exam inations. The m ain ad vantage to p rojec-
need for ad m iration, and lack of em pathy. tive tests (such as the Rorschach) is that there
are a variety of resp onses w ithou t a single
41. (D ) DBT is a m anu alized treatm ent for chroni- correct answ er. The id ea behind projective
cally parasuicid al patients su ch as those w ith tests is that w hen presented w ith an am bigu-
bord erline p ersonality d isord er. It incorp o- ou s stim u lu s, patients cannot help bu t reveal
rates elem ents of cognitive, behavioral, and som ething about them selves both in how they
su pportive therapies, and it includ es tech- ad d ress the stim u lu s as w ell as the content of
niqu es of ad vice, confrontation, and u se of their answ ers. Many of these tests, inclu d ing
hom ew ork. Aversion therapy is a controver- the Rorschach Test, the Them atic Ap percep-
sial techniqu e in behavioral therap y w here tion Test (TAT), the Sentence Com pletion Test,
an aversive stim u li is p aired w ith an u nd e- and the Draw -A-Person Test, requ ire sp ecific
sired behavior, hopefully su ppressing the training in giving the test and interpreting the
u nw anted behavior. EMDR is a cognitive- resu lts. Projective tests d o not necessarily tell
behavioral techniqu e w hereby a p atient w ith the interview er how the patient feels about
p osttrau m atic stress d isord er is told to think him or her.
Answe rs : 3948 233

44. (C) The MMPI and MMPI-2 are objective tests the event in som e w ay, su ch as nightm ares);
that have been u sed for over 50 years in the anxiety or increased arou sal; avoid ance of the
assessm ent of p ersonality stru cture. They con- stim u li that arou se recollections of the trau -
sist of m ore than 500 statem ents, w hich are m atic event. The d iagnosis of PTSD requ ires
cond ensed into clinical subscales. Item s on the m ore than 1 m onth of sim ilar sym p tom s.
su bscales w ere d erived em pirically and w ere Ad ju stm ent d isord er can be consid ered for
chosen becau se d ata show ed they cou ld sepa- those ind ivid u als w ho d o not m eet criteria
rate p sychiatric p atients from norm al control for acu te stress d isord er bu t d evelop sim ilar
su bjects. Ad d itionally, qu estions are asked at sym ptom s in excess of w hat is to be expected
the tim e of exam ination to evaluate attitu d es given the natu re of the stressor. GAD is char-
w hen taking the test. These qu estions help acterized by excessive anxiety and w orry
to p rovid e inform ation abou t the valid ity of that occu r for at least 6 m onths. MDD can
the exam ination. Interpretation of the resu lts exist in the context of an acute stress d isor-
requ ires an exp erienced evalu ator. Together d er. H ow ever, the sym p tom s are not su gges-
w ith the clinical history, this test p rovid es tive of a d ep ressive illness as there is no clear
valu able inform ation abou t personality stru c- d ep ressed m ood , change in ap p etite, energy,
tu re. Its accuracy d oes not d epend on the or concentration.
p atients gend er, and the results d o not reflect
how the exam inee feels about test taking. 47. (B) Psychogenic u nresp onsiveness can be
d elineated from com a by obtaining an EEG,
45. (B) Denial, a narcissistic d efense, is an (uncon- w hich is norm al in a p sychogenic state. On the
sciou s) em otional d efense m echanism used p hysical exam ination of a com atose patient,
to avoid becom ing aw are of a painfu l aspect d eep tend on reflexes m ay be su p p ressed . In
of reality. Denial can be used in both norm al p atients w ho are aw ake, cold w ater intro-
and p athologic states. As a d efense m echa- d u ced into the ear p rod u ces nystagm u s w ith
nism it serves to keep internal or external the fast com p onent aw ay from the ear. In
reality ou t of the consciou s to avert stress and com a, the eyes either d o not react or they m ay
anxiety. Rationalization is a neu rotic d efense slow ly and sm oothly d eviate tow ard the ear
m echanism in w hich unaccep table behavior, in w hich the cold w ater w as introd u ced . Vol-
feelings, or thou ghts are logically ju stified by u ntary eye m ovem ents called saccad es are
elaborate and reassuring answ ers. Regres- rap id and sm ooth. Saccad ic eye m ovem ents
sion, an im m ature d efense, is an em otional are elicited in p atients by asking them to stare
and p hysical retreat from ad u lt stand ard s of at an object at one sid e of the visu al field and
behavior tow ard an infantile level of p assiv- then ask them to shift their gaze to the op po-
ity and d ep end ence. Su p p ression, a m atu re site visual field . Typically, eye m ovem ents in
d efense, is a consciou s act of controlling and com a or persistent vegetative states are sp on-
inhibiting u naccep table im p u lses, em otions, taneou s and rand om . Abnorm al vital signs
or id eas. su ch as tem perature and respiration w ou ld
not likely be affected in a p sychogenic state.
46. (A) The essential featu re of acu te stress d isor-
d er is the d evelop m ent of anxiety and d isso- 48. (C) A seasonal pattern can be associated with
ciative sym p tom s w ithin 1 m onth of exp osu re major d epressive episodes in MDD, bipolar I
to an extrem ely traum atic stressor. Diagnosis disorder, and bipolar II disorder. The essential
shou ld be consid ered if the sym ptom s persist feature is the onset of depressive symptoms at
at least 3 d ays and cau se significant d istress characteristic times of the year, most episod es
or im p airm ent. Du ring or after experiencing beginning in the fall or winter and remitting in
the stressor, the ind ivid u al m ust experience the spring. It occurs more commonly in women
at least nine associated sym ptom s (e.g., sleep and in higher latitudes. Major depressive epi-
d istu rbance; absence of em otional resp onsive- sodes that occur in a seasonal pattern are typi-
ness; d issociative sym ptom s; reexp eriencing cally characterized by anergia, hypersomnia,
234 7: Pra c tic e Te s t 1

overeating, weight gain, and carbohydrate current scale, the WAIS-R. The test is com -
craving. Age is a strong predictor, with young p osed of 11 d ifferent su btests6 verbal and
persons at higher risk. Acute stress disorder is 5 p erform ancethat allow for the calculation
characterized by the development of anxiety, of the fu ll-scale IQ, p erform ance IQ, and ver-
reexperiencing, dissociative, and other symp- bal IQ. The WAIS-R has high reliability, w hich
toms within 1 month of exposure to a traumatic m eans that in norm al su bjects retesting d oes
event. Generalized anxiety d isorder patients not lead to significantly d ifferent evaluations;
describe chronic anxiety over a number of therefore it can be u sed to follow peop le over
events lasting over 6 months. Persistent depres- tim e. The Draw -A-Person Test is u sed to eval-
sive disorder (dysthymia) is a mood disorder u ate for organic brain d isease, and the MMPI-2
where the depressed mood occurs most of the is a test u sed for the assessm ent of p ersonality
time for at least 2 years. Sundowning refers to stru ctu re. The WISC is used to d eterm ine IQ
drowsiness, confusion, and falling, typically in child ren from ages 6 to 16. The WMT w as
seen in patients with major neurocognitive dis- d esigned to evalu ate a variety of aspects of
orders (dementias) when lights or other orient- m em ory fu nction in ad u lts.
ing cues are removed.
51. (C) The IQ is calculated by d ivid ing the m ental
49. (B) The essential featu res of d issociative am ne- age by the chronological age and mu ltip lying
sia w ith fu gu e are the inability to recall im p or- this by 100.
tant autobiographical inform ation, along w ith
p urp oseful travel or w and ering associated 52. (B) While this patient m eets criteria for a
w ith the am nesia. The d isord er is m ore com - m ajor d ep ressive ep isod e, she has a history
m on in w om en and is often associated w ith of sym ptom s consistent w ith a hyp om anic
a history of abu se in child hood . Delirium is ep isod e (a d istinct p eriod of a p ersistently
characterized by a d istu rbance in attention elevated , exp ansive, or irritable m ood that
and aw areness, along w ith a change in cogni- lasts at least 4 d ays). Bipolar II d isord er d iffers
tion that d evelop over a short period of tim e. from bipolar I d isord er in that, although the
Major d ep ression is d iagnosed in ind ivid uals m anic and hyp om anic episod es of these tw o
w ith 2 w eeks of d epression and neu roveg- d isord ers have id entical criteria, bipolar II d is-
etative sym p tom s (changes in sleep , ap p etite, ord er is not severe enou gh to cau se m arked
energy). Partial com p lex seizu res tend to be im p airm ent in social or occu p ational fu nction-
brief and d o not last as long as the clinical ing or to cau se hosp italization, and d oes not
p icture presents. For the d iagnosis of p ost- have p sychotic sym p tom s. Cyclothym ic d isor-
trau m atic stress d isord er, an ind ivid u al m u st d er consists of chronic, flu ctuating m ood ep i-
be exp osed to a trau m atic stressor, along w ith sod es of hyp om ania alternating w ith period s
sym ptom s of intrusion, alterations in cogni- of d epression w hich d o not m eet criteria for
tion and m ood , d istu rbances of arou sal, and m ajor d ep ressive episod es. Persistent d epres-
avoid ance of stim u li. sive d isord er (d ysthym ia) is characterized
by chronically d ep ressed m ood m ost of the
50. (C) The DSM -5 recognizes three sp ecific learn- tim e for 2 years w ithou t hypom anic or m anic
ing d isord ers: im p airm ent in read ing, w ritten ep isod es.
com m u nication, and m athem atics. Learn-
ing d isord ers are id entified w hen a p atients 53. (C) The hallmark of Ganser synd rome is that
acad em ic skills in that area are below those the patient respond s to qu estions by giving
exp ected for the ind ivid u als chronological ap proximate or rid iculous answ ers. It involves
age, as confirm ed by ad m inistered stand ard - the prod u ction of answ ers to qu estions that are
ized achievem ents m easu res and clinical close bu t not qu ite correct (e.g., 4 5 = 19). Gan-
assessm ent. The Wechsler Ad ult Intelligence ser synd rome is primarily d escribed in m ales
Scale w as initially p u blished in 1955 and has in the prison population, most likely related
u nd ergone a series of revisions to reach the to malingering or factitiou s d isord er w ith
Answe rs : 4957 235

p sychological sym ptom s. Capgras synd rom e The essential features of the atypical specifier
d escribes a content-specific d elu sion in w hich are mood reactivity and tw o of the follow ing
the patient believes that a significant other, four features: increased appetite or w eight
u su ally a fam ily m em ber, has been rep laced by gain, hypersom nia, lead en paralysis, and
an id entical im p oster. Major neu rocognitive interpersonal rejection sensitivity. The essen-
d isord ers (d em entias) are cognitive d isord ers tial featu re of a major d epressive episod e w ith
associated w ith m em ory d eficits and p rob- melancholic featu res is a loss of interest or p lea-
lem s w ith execu tive fu nctioning. Althou gh sure in all or almost all activities or a lack of
p atients w ith d em entia m ay confabu late, this reactivity to usually pleasurable stimuli, along
p atients age is too you ng, and he has obvi- w ith ad d itional sym ptoms of d epression (e.g.,
ou s second ary gain. Mu nchausen synd rom e is early morning aw akening, d epression w orse in
a factitious d isord er w ith physical sym p tom s, the morning). The patients d epressive symp -
w here the ind ivid u al feigns an illness in ord er toms are too acute and severe for ad ju stment
to assu m e the sick role, and is often associated d isord er, and d ont last 2 years w hich w ould
w ith traveling to d ifferent hosp itals. be necessary for persistent d epressive d isor-
d er (d ysthymia). Although hypersomnia (and
54. (B) ECT provid es a safe and effective treat- insom nia) may be d ue to a sleep d isord er, the
m ent of MDD in the eld erly, especially in a patients symptom s are better accounted for by
p atient w ith a high risk of su icid e or m ed i- a major d epressive d isord er.
cal contraind ications (e.g., heart d isease) to
the use of certain psychotropic m ed ications. 56. (C) Id entification is an u nconsciou s d efense
It is also ind icated w hen a m ore rap id onset m echanism in w hich the p erson incorporates
of action (several d ays) is im p ortant (com - the characteristics and qu alities of another
p ared w ith several w eeks for antid ep ressant p erson or object into his or her ow n ego sys-
m ed ications). Diazepam is a benzod iazep ine tem . The d efense serves to strengthen the ego.
w hich is u sefu l in the treatm ent of sym p tom - Fixation refers to an overactive attachm ent to
atic anxiety bu t not for d epression. N ortrip ty- a p erson or object; id ealization is the attribu-
line, a tricyclic antid ep ressant (TCA), is lethal tion of near-perfect, u nrealistic attribu tes to
in overd ose and shou ld not be p rescribed in that person or object. Projection is the false
an acutely su icid al p atient. Certain card iac attribu tion of ones ow n u nacceptable feelings
arrhythm ias are also contraind ications to TCA to another. Regression, an im mature d efense,
u se. Risperid one is an antip sychotic m ed ica- is an em otional and physical retreat from ad ult
tion u sed prim arily for psychotic d isord ers stand ard s of behavior tow ard an infantile level
such as schizophrenia and schizoaffective d is- of passivity and d ep end ence.
ord er, as w ell as for bipolar d isord er or m ajor
d epression w ith psychotic features. Supportive 57. (D ) The essential featu re of the catatonia
psychotherapy may be helpful in conjunction observed in p atients w ith schizop hrenia (as
w ith med ications or ECT, but it is not indicated w ell as m ood d isord ers w ith p sychotic fea-
as monotherapy for severe depression w ith tu res) is a m arked p sychom otor d istu rbance
suicid ality. involving m otor im m obility, excessive m otor
activity, mutism, negativism, peculiar volun-
55. (B) This p atient is su ffering from a m ajor tary movements, echolalia (parrot-like senseless
d ep ressive ep isod e w ith atyp ical featu res. repetition of w ord s or phrases), or echopraxia
The sp ecifiers of m elancholic featu res and (im itation of movem ents of another person).
atypical featu res can be ap plied to a cu rrent or Catalepsy, a term for an imm obile position
recent m ajor d epressive episod e that occurs in that is maintained , is one potential symptom
the course of MDD, and to a major d ep ressive of catatonia, and is not a com p lete answ er.
episod e in bipolar I or bipolar II d isord er. The Catap lexy is a su d d en and brief loss of m u scle
atypical featu res sp ecifier can also be applied tone involving either a few m u scle grou p s or
to persistent d epressive d isord er (d ysthymia). m ost of the antigravity m u scles of the bod y;
236 7: Pra c tic e Te s t 1

it is not u ncomm on in patients w ith narco- and constriction of veins. The offend ing m ed -
lepsy. H is d iagnosis of schizophrenia, and the ication shou ld be im m ed iately stop p ed and
fact that the patient occasionally m imics the not restarted , as p rior p riap ism p red isp oses
movements of others, makes seizu re activity to fu tu re ep isod es.
unlikely.
61. (B) The p atient is m ost likely su ffering from
58. (B) This patient w ou ld be d iagnosed w ith generalized anxiety d isord er (GAD). Treat-
bip olar d isord er, typ e I, cu rrent ep isod e m ent of GAD inclu d es antid ep ressants su ch
d ep ressed . The treatm ent of bipolar d epres- as serotonin-sp ecific reup take inhibitors,
sion is tricky. Lam otrigine is likely the m ost benzod iazep ines, bu sp irone, and cognitive-
effective in treating bip olar d ep ression and behavioral therap ies. H ow ever, benzod iaz-
as m aintenance treatm ent to prevent relapse ep ines (su ch as alp razolam , d iazep am , and
of d epression. First-generation antipsychotics lorazep am ) shou ld be avoid ed in p atients
like halop erid ol are not effective in treating w ith resp iratory com p rom ise, becau se these
bip olar d ep ression; how ever, atyp ical anti- m ed ications can exacerbate breathing-related
p sychotics (e.g., qu etiap ine) can be beneficial. d isord ers, and they carry the risk of ad d iction.
While lithiu m is first-line treatm ent for acu te Bu sp irone, a nonbenzod iazep ine anxiolytic,
m ania and p reventing fu rther m anic ep isod es, is a logical alternative and has been d em -
it is not as effective in treating the d ep ressed onstrated to be effective in the treatm ent of
p hase of bipolar d isord er. Monotherapy w ith GAD. Beta-blocking agents such as proprano-
an antid ep ressant like sertraline shou ld be lol are effective in certain anxiety d isord ers,
avoid ed as it can p rovoke a m anic ep isod e in such as social anxiety d isord er (social phobia),
p atients w ith bip olar d isord er. Valp roic acid bu t shou ld not be first-line therap y in p atients
(Depakote) is another m ood stabilizer prim ar- w ith chronic obstru ctive p u lm onary d isease
ily u sed to treat m ania and as m aintenance, or other respiratory d isord ers.
bu t it is not as efficaciou s in the treatm ent of
d ep ression in this popu lation. 62. (E) The long-term u se of benzod iazepines
(lorazepam , oxazepam , and tem azepam ) for
59. (E) Priapism can occu r w ith a num ber of anti- sleep is controversial, especially in p atients
d ep ressant or antip sychotic agents bu t occurs w ith su bstance u se or breathing-related d isor-
m ore frequ ently w ith the u se of trazod one d ers. Zaleplon is not a m em ber of the benzo-
(1 in 1,000 to 1 in 10,000 incid ence). The three d iazepine class but d oes prod u ce its effects at
serotonin-sp ecific reu ptake inhibitors (fluox- the gam m a-am inobu tyric acid (GABA)-ben-
etine, p aroxetine, and sertraline) listed can zod iazepine recep tor and has a low er risk of
cause d ecreased sex d rive and im potence, bu t p hysical d ep end ence. A nonbenzod iazep ine
are u nlikely to cau se priap ism . N ortrip tyline m ed ication is a logical alternative in cases in
is a tricyclic antid ep ressant and not associated w hich resp iratory fu nction is com p rom ised .
w ith p riap ism . Dip henhyd ram ine shou ld be avoid ed in the
old er population d ue to its anticholinergic
60. (D ) While p riap ism u su ally occu rs w ithin effects and p otential d eliriu m .
the first 4 w eeks of treatm ent, it can occu r at
any tim e. It is also ind ep end ent of d ose. Pria- 63. (D ) H yp ertensive crisis is a life-threatening
p ism , esp ecially lasting over several hou rs, is em ergency that m ay resu lt from the com bina-
a m ed ical em ergency and m ay requ ire su r- tion of m ep erid ine (Dem erol) and an MAOI.
gery to p revent p ossible necrosis, scarring, The com bined u se of these m ed ications is
and im p otence. Ejacu lation d oes not alw ays absolu tely contraind icated . The treatm ent of
red u ce the erection. Priap ism m ay requ ire hypertensive crisis is w ith IV alp ha-ad rener-
d raining blood from the corp u s caverno- gic blocking agents and ICU ad m ission and
su m or intracavernosal injection of an alp ha- supp ort. All of the other m ed ications are safe
ad renergic agonist to p rom ote d etu m escence in com bination w ith MAOIs.
Answe rs : 5869 237

64. (C) The use of any m ed ications in pregnancy an u nbiased colleagu e or su p ervisor to d is-
requ ires exam ining the risks and benefits of cuss the issues and ultim ately d eterm ine w hat
treatm ent. The u se of MAOIs is contraind i- w ou ld be in the best interest of the p atient. It
cated in pregnancy partly because they can m ay be that another physician in the clinic or
exacerbate pregnancy-ind u ced hypertension. com m u nity w ou ld better serve the p atients
Althou gh ECT is safe in p regnancy, m ainte- need s, but this w ould best be d eterm ined after
nance ECT is ind icated only after an initial a consu ltation. To lim it the tim e spent w ith
cou rse of ECT is com pleted . The safety of the p atient is not ap prop riate and can consti-
new er antid epressants has not yet been com - tu te d iscrim ination. While it is im portant to
p letely established , bu t stu d ies of flu oxetine be aw are of the p atients p rogress in therap y,
su ggest that it is relatively safe. In this case, the physician m u st first process his or her ow n
after d iscontinu ing the MAOI, there w ou ld p artiality to d eterm ine w hether a therap eu tic
have to be a 14-d ay w ashou t before initiating alliance can be established and m aintained .
flu oxetine. Most recom m end ations regard ing
the course of treatm ent for a single episod e 67. (B) Caffeine withdrawal occurs after the abrupt
of MDD are for 9 to 12 m onths of p harm a- cessation or a m arked red uction in the use
cotherapy. The retu rn of sym ptom s su ggests of caffeinated prod ucts. Stud ies estimate that
a recu rrent MDD that requ ires longer-term approximately 50% of caffeine users w ill expe-
p harm acotherap y. Of cou rse, ongoing d is- rience symptoms of w ithdraw al at some point.
cussion w ith and ed ucation of the patient are Caffeine w ithd raw al sym p tom s have their
essential. onset 12 to 24 hou rs after the last d ose, and
symptoms peak in 24 to 48 hours. Symptoms
65. (B) The patient presents w ith clinical sym p - typ ically rem it in 1 w eek. Althou gh all the
tom s of Wernicke encephalopathy, a m ed ical p ossible choices are associated w ith caffeine
em ergency, characterized by op hthalm op le- w ithdraw al, up to 50% of sufferers experience
gia, ataxia, and d eliriu m , cau sed by acu te thia- head ache, the most common symptom.
m ine d eficiency. If the thiam ine d eficiency is
not corrected , Korsakoff synd rom e can em erge 68. (E) This p atient is m ost likely su ffering from
characterized by chronic p ersisting am nesia. neu roleptic m alignant synd rom e (N MS). N MS
Korsakoff synd rom e resu lts from chronic thi- is characterized by m u scu lar rigid ity, confu -
am ine d eficiency w ith resu ltant p athology in sion, m u tism , agitation, and elevated au to-
the m am m illary bod ies and the thalam u s. A nom ic signs. Death is rep orted in u p to 20% of
history of heavy alcohol use is com m on bu t cases, u su ally from a failu re to recognize the
other cau ses of thiam ine d eficiency su ch as synd rom e or from d elayed treatm ent. Discon-
starvation, p rolonged vom iting, and gastric tinu ation of the offend ing agent and sup port-
carcinom a can cause this synd rom e. The u se ive treatm ent in an ICU setting is im p erative.
of IV d extrose prior to the ad m inistration of IV d antrolene sod iu m and oral brom ocrip tine
thiam ine m ay aggravate Wernicke encepha- have been u sed bu t their benefits are u nclear.
lop athy. The other interventions m ay be Cooling measures may reduce hyperthermia.
ap prop riate if thiam ine d oes not resolve the The exact frequency of NMS remains unknow n,
sym ptom s. w ith some studies suggesting up to 2.4% of all
patients treated w ith antipsychotics experienc-
66. (E) Accord ing to the AMAs cod e of ethics ing this synd rome. NMS may be more frequent
A p hysician shall be d ed icated to provid - in men and younger patients. N aloxone d oes
ing com p etent m ed ical care, w ith com p assion not have a place in the treatment of N MS.
and resp ect for hu m an d ignity and rights.
In the event a p hysician is concerned abou t 69. (D ) An elevated seru m CPK d em onstrates
w hether he or she is able to rem ain im p artial m uscle inju ry. Since liver transam inases (AST,
and p rovid e ad equ ate p sychiatric treatm ent, ALT) are also fou nd in m u scle, these m ay be
the physician shou ld seek a consu ltation w ith elevated . Elevated BUN m ay occu r d u e to
238 7: Pra c tic e Te s t 1

d ehyd ration resu lting from N MS but is not as MRI w ou ld be im portant, it is not necessarily
com m on as creatinine or CPK elevation. Leu - d iagnostic of a p articu lar neu rocognitive d is-
kocytosis is also seen in N MS. Anem ia is not ord er (d em entia).
fou nd in N MS.
72. (B) Lithiu m , com m only u sed to treat bip olar
70. (C) Physical d epend ence emp hasizes the pres- d isord er, can be d angerou s in intoxication
ence of tolerance or w ithd raw al. N aloxone and can lead to d eath or p erm anent d am -
injection can precipitate opiate w ithd raw al in age to the nervou s system , p articu larly the
d epend ent persons. Positive urine toxicology cerebellu m . Card iovascu lar and renal m ani-
or breath analysis can id entify only su bstance festations m ay also be p resent. Lithiu m has
use, not necessarily physiological d ep end ence. a narrow therap eu tic w ind ow w ith a thera-
Elevated heart rate can be p resent in m any p eu tic range of 0.6 to 1.2 m Eq/ L. Plasm a
w ithd raw al synd rom es or m ed ical d isor- concentrations above the therap eu tic range,
d ers and is not sp ecific for any su bstance. esp ecially greater than 2.0 m Eq/ L, p recip itate
Elevated liver fu nction stu d ies can be p res- severe CN S and renal impairm ent. Early clini-
ent in ind ivid u als w ho u se heavy am ou nts of cal signs of intoxication includ e d ysarthria,
alcohol, bu t can also be p resent in other non coarse trem or, and ataxia. Im paired consciou s-
su bstance-related d isord ers. ness, fascicu lations or m yoclonu s, seizu res,
and com a are om inou s m anifestations. The
71. (E) Major neu rocognitive d isord ers (d em en- goal of treatm ent is rem oval of lithiu m from
tias) are characterized by m em ory d eficits the bod y. Gastric lavage, rehyd ration, and
and cognitive d istu rbances w ithou t im p aired hem od ialysis are possible interventions. Given
consciousness. (This d ifferentiates it from the d anger of lithiu m intoxication, a seru m
d elirium characterized by a w axing and w an- d rug level should be obtained before any of
ing consciou sness.) The m ost com m on typ e of the other interventions.
m ajor neu rocognitive d isord er (d em entia) is
d ue to Alzheim er d isease (60%). Major vascu - 73. (E) A right parietal stroke w ould most likely
lar neu rocognitive d isord er is the second m ost d emonstrate abnormalities in visual, nonverbal
com m on typ e of d em entia, occu rring in 10% m em ory.
to 20% of cases. Other d egenerative p rocesses
that cause m ajor neurocognitive d isord ers 74. (B) The ReyOsterrieth Test is a com p lex figu re
(d em entias) are Parkinson d isease, H u n- that the p atient is asked to cop y w hile looking
tington d isease, and Pick d isease. Infectious, at the figu re. The figu re is then taken aw ay
m etabolic, end ocrine, neop lastic, and toxic and the p atient is asked to d raw the p ictu re
p rocesses can also cau se neurocognitive d isor- from im m ed iate m em ory. The p atient is again
d ers (d em entias), so a com p lete w orkup for a asked to d raw the figu re 5 m inu tes and 30 m in-
reversible cau se is essential. This p atients his- u tes after the figu re has been rem oved . The
tory reveals a m egaloblastic anem ia, the cau se tasks assess visu al nonverbal m em ory. Peo-
of w hich m ay be a vitam in B12 d eficiency, p le w ith right p arietal lesions u su ally show
likely second ary to a gastrectom y. Folate d efi- abnorm alities in cop ying the figu re correctly
ciency can also cau se m egaloblastic anem ia. by neglecting the item s in the left visu al field .
Vitam in d eficiencies su ch as B12 and folate can Conversely, a p atient w ith a right tem p oral
cause clinical signs of m ajor neu rocognitive lobectom y m ay have no d ifficu lty in cop ying
d isord ers (d em entias), and they represent one the figu re, bu t show m arked abnorm alities in
of a few possible reversible cau ses of this p ro- d raw in g th e item from m em ory. Th e Min -
cess. A com p lete reversible w orku p for neu- n esota Mu ltip h asic Person ality In ven tory,
rocognitive d isord ers (d em entias) shou ld also Rorschach Test, Wechsler Intelligence Scales,
inclu d e thyroid fu nction stu d ies and a rap id and Wisconsin Card Sorting Test w ou ld not
p lasm a reagin to test for syp hilis. While an be as u sefu l in assessing a right p arietal lesion.
Answe rs : 7080 239

75. (A) Baseline m ed ical tests should be obtained bu lim ia nervosa u su ally m aintain a norm al
and d ocum ented before starting m any psycho- or above-norm al bod y w eight. Med ical com -
tropic med ications, includ ing the mood stabi- p lications of bu lim ia nervosa are u su ally
lizers lithium , carbam azepine, and d ivalproex second ary to either chronic vom iting or laxa-
sod ium. As lithiu m has a narrow therapeu tic tive abu se. They inclu d e flu id and electrolyte
w ind ow and is renally excreted , assessm ent im balances (su ch as alkalosis, hyp okalem ia,
of kid ney function w ith BUN and creatinine hyp ochlorem ia, and d ehyd ration); d ental car-
is essential prior to starting lithium. Lithium ies and enam el loss; gastrointestinal d istu r-
can cause a benign leukocytosis, but a CBC bances su ch as esop hagitis, MalloryWeiss
isnt necessary prior to initiation. Revers- tears, and constip ation; and sore throat.
ible T-w ave changes on the ECG are also
com mon, but an ECG is only necessary in an 78. (B) SSRIs, TCAs, MAOIs, and top iram ate
old er patient or one w ith card iac d isease. Liver have all been show n to be effective in red u c-
function tests are not necessary, but monitor- ing bingeing and p u rging behavior, even in
ing thyroid function during treatment w ith the absence of a m ood d isord er. Bu p rop ion,
lithium is im portant. w hile an antid ep ressant, shou ld be avoid ed
d u e to the increased risk of seizu res from
76. (D ) The serotonin synd rome occurs w hen an electrolyte abnorm alities. N either antim anic
SSRI is combined w ith another d rug that can agents, su ch as lithiu m and valp roic acid , nor
potentiate serotonin, particularly the mono- antip sychotics, su ch as risp erid one, are ben-
amine oxid ase inhibitors. Symptoms of the eficial in treating bu lim ia nervosa.
synd rome includ e abd ominal pain, d iarrhea,
d iaphoresis, hyperpyrexia, tachycard ia, hyper- 79. (B) The Boston Diagn ostic Ap hasia Exam in a-
tension, myoclonus, irritability, agitation, sei- tion is a com p rehensive set of tests given by
zu res, and d eliriu m. Com a, card iovascu lar a skilled interview er to evalu ate ap hasic d is-
collapse, and d eath have been reported . In this ord ers and to help d efine fu rther interven-
case, the serotonergic effects of fluoxetine are tions to im p rove sp eech. The Bend er Gestalt
potentiated by the MAOI. Fluoxetine has a Test is a constru ctional test for evalu ation of
prolonged half-life w ith active metabolites that brain d am age and h as som e ability to d if-
require up to 6 w eeks clearance time. Treat- ferentiate the location of th e lesion. The Fol-
ment w ith the MAOI w as initiated before ad e- stein MMSE is u sed for rap id screening for
quate w ashout of the fluoxetine and resulted in neu rocogn itive d isord ers (d em en tias) and
the serotonin syndrome. Management involves d eliriu m . The Sentence Com p letion Test is
d iscontinuation of the offending agent and a p rojective test u sed to d escribe p ersonality
supportive care. There is no evid ence that the stru ctu re. The Stroop Test aid s in the evalu a-
patient w as not compliant w ith a tyramine-free tion of concentration.
d iet, w as using hallucinogens, or that she w as
taking antipsychotics, w hich w ould be consis- 80. (E) Anorexia nervosa is characterized by a
tent w ith the remaining choices. d eliberate self-im p osed starvation in the p u r-
su it of being thin, d riven by a fear of being
77. (B) Bu lim ia nervosa, m ore com m on than fat. The p atient refu ses to m aintain accep ted
anorexia nervosa, consists of rep eated ep i- norm al bod y w eight appropriate for age and
sod es of binge eating of large am ou nts of height. Und erd eveloped breasts, lanugo (fine,
food associated w ith the feeling of being ou t d ow ny hair), abnorm al insu lin secretion,
of control. The p atient u su ally p u rges by end ocrine d isord ers, and am enorrhea are also
self-ind u ced vom iting, rep eated laxative or associated w ith this d isord er. Com plications
d iu retic u se, or excessive am ou nts of exer- inclu d e osteop orosis, constip ation, cold intol-
cise to p revent w eight gain. Unlike ind ivid u - erance, leu kop enia, thyroid d ysfu nction, and
als w ith anorexia nervosa, ind ivid u als w ith card iac arrhythm ias.
240 7: Pra c tic e Te s t 1

81. (C) Anorexia nervosa has a fair p rognosis risk of overd ose m ake this a second -line m ed i-
overall; app roxim ately 50% of p atients have cation. SSRIs are also very effective in treat-
good ou tcom es (inclu d ing significant w eight ing sym p tom s of OCD, bu t u su ally at d oses
gain), 25% have interm ed iate ou tcom es, and higher than those used to treat d ep ression.
25% have poor ou tcom es. Antip sychotic agents are u sed in conju nction
w ith antid ep ressants in very severe, intrac-
82. (B) Bereavem ent is consid ered to be a nor- table cases, bu t are not u sually ind icated as
m al response to the d eath of a loved one, and ind ivid u al agents. Antiep ilep tics and lithiu m
it is not a m ental illness. It is typ ical for the are not ind icated for the treatm ent of OCD.
bereaved to have intense feelings of em p tiness Anxiolytic agents su ch as benzod iazep ines
that w ax and w ane. It is also not u nu su al to are of little u se in treating the obsessions or
believe they hear the voice of the d eceased , com pu lsions of OCD.
although these experiences are brief and
insight is m aintained . If this p atient m et cri- 85. (C) The concern in this case is that the p atient
teria for m ajor d epression, then she w ou ld be is lying abou t her m em ory p roblem , therefore
d iagnosed as su ch; how ever, in this case she m alingering. When given tests of m em ory,
d oes not m eet the fu ll criteria (e.g., ap p etite m alingerers tend to not put forth genuine
change, poor concentration, anhed onia). Up to effort. The Test of Mem ory Malingering can
30% of bereaved ind ivid u als d evelop a m ajor help assess this effort. It consists of a subject
d ep ressive ep isod e, so it is im p ortant to m oni- being p resented 50 p ictures; they are then
tor sym ptom s closely. Ad ju stm ent d isord er p resented card s and asked if they had seen
cannot be d iagnosed if the ind ivid u al su ffers a given card before. Perform ance below the
from norm al bereavem ent. Persistent d ep res- od d s of chance ind icates high likelihood of
sive d isord er (d ysthym ia) requ ires 2 years intentionally changing a correct answ er to
of sym ptom s m arked by a d epressed m ood . an incorrect one in ord er to falsify resu lts.
There is no evid ence of a p rim ary p sychotic The Benton Visu al Retention Test m easu res
d isord er su ch as schizoaffective d isord er, nor visu al perception and m em ory, and is u sed to
have the fu ll sym p tom s been present for the help id entify learning d isabilities. The Clock
ap prop riate am ou nt of tim e. In ad d ition, her Draw ing is u sed as p art of a screen for cog-
age of onset w ou ld be extrem ely u nu su al for nitive im pairm ent and spatial d ysfu nction in
su ch a d iagnosis. neu rocognitive d isord ers, such as d em entias.
The Wechsler Intelligence Scale is an IQ test
83. (C) Most bereaved d o fine w ithou t treatm ent. for late ad olescents and ad u lts, w hile the Wis-
Sup port and reassurance u sually com e from consin Card Sorting Test is a neuropsycholog-
fam ily, friend s, or clergy. When the bereaved ical test for set-shifting, d etecting frontal lobe
feel u nable to m ove beyond the grief, p sycho- d am age from a variety of cau ses.
therapeu tic m easu res are appropriate. The
treatm ent of anxiety or insom nia w ith m ed i- 86. (C) Obstructive sleep apnea hypopnea is d iag-
cations rem ains controversial, bu t m ed ica- nosed w ith polysomnographic evid ence of
tions m ay be w arranted if cognitive or other either: (1) Five obstructive apneas or hypopneas
higher fu nctions of d aily living are su bstan- per hour w ith snoring/ gasping or d ay times
tially affected . sleepiness/ fatigue or (2) 15 or more obstructive
apneas or hypopneas per hour regard less of
84. (A) The treatm ent of p atients w ith obsessive- associated symptoms. It is increased in males,
com pu lsive d isord er (OCD) requ ires integrat- old er ad ults, and the obese. Circad ian rhythm
ing m u ltip le treatm ent m od alities inclu d ing sleepw ake d isord ers are characterized by pat-
m ed ications, p sychotherapy, and collateral terns of sleep d isruption d ue to alterations of
su pp ort system s. One of the first m ed ications the circad ian rhythm system (e.g., shift w ork).
show n to be effective for OCD w as clom ip- There is no other evidence for major depres-
ram ine, a TCA, althou gh the sid e effects and sion in this patient. The criteria for REM sleep
Answe rs : 8191 241

behavior disorder include repeated episodes of w ould treat the brad ycard ia w ithout treat-
arousal during sleep w ith vocalization and/ or ing the und erlying cond ition. Physostigm ine
motor behaviors occurring only d uring REM w ould w orsen the toxicity. Although prochlor-
sleep. Patients w ith restless legs synd rome perazine w ould com bat vomiting, it w ou ld not
have an urge to move the legs accompanied treat the u nd erlying cond ition and also might
by feelings of uncomfortableness. The urges w orsen the anticholinergic toxicity.
are w orsened at rest and at night, and relieved
w ith movement. 90. (D ) This w om an appears to suffer from both a
major d epressive d isord er (MDD), single epi-
87. (C) N asal continu ou s p ositive airw ay p ressu re sod e, as w ell as obsessive-comp ulsive d isord er
(CPAP) is the treatm ent of choice for obstruc- (OCD). Serotonin-specific reuptake inhibitors
tive sleep apnea. N o m ed ications are consis- (SSRIs) are used to treat a variety of psychiatric
tently effective in norm alizing sleep in these d isord ers, includ ing both MDD and OCD. Flu-
p atients. The use of nasal CPAP ap p ears to voxamine is an SSRI p articu larly ind icated for
p revent long-term m orbid ity and m ortality both d epression and OCD. Buspirone is a non-
and provid es restorative sleep . Su rgical p ro- benzod iazepine m ed ication used to treat gen-
ced ures su ch as those listed are reserved for eralized anxiety d isord er but is not efficacious
CPAP treatm ent failures. for OCD. Clomipramine is a tricyclic antid e-
pressant (TCA) w ith significant serotonergic
88. (A) Anticholinergic toxicity can be cau sed effects and is particu larly u sefu l in OCD; d ox-
by many psychotropic medications. The low - epin is also a TCA, bu t given the tolerability
p otency agen ts chlorp rom azine and thio- and sid e effect profile and safety in overd ose,
rid azine and the TCAs am itrip tyline an d SSRIs are preferred as first-line therapy for
im ip ram ine are p articu larly anticholinergic. MDD and OCD. Phenelzine is a m onoam ine
The synd rom e often resu lts w hen an anticho- oxid ase inhibitor (MAOI) w hich can be u sed
linergic m ed ication, su ch as benztrop ine, is for d epression. H ow ever, the d rugd rug inter-
coad m inistered w ith an antipsychotic m ed i- actions and d ietary restrictions make MAOIs a
cation in ord er to prevent or treat extrap yra- second - or third -line therapy.
m id al sid e effects su ch as p arkinsonism and
d ystonia. Clinical features of anticholinergic 91. (D ) The p atient is beginning to have sym p tom s
toxicity includ e d ecreased secretions, agita- of alcohol w ithd raw al. Because they act at the
tion, d ry skin, flu shing of the skin, hyp er- sam e GABA receptors as alcohol, the benzo-
therm ia, tachycard ia, d ilated p upils, urinary d iazepines are the preferred m ed ications used
retention, constip ation, and hyp otension. Sei- in d etoxification from alcohol. For u ncom -
zu res, hallu cinations, and com a m ay resu lt p licated d etoxification, the long-acting ben-
from severe intoxication. Drugs that inhibit zod iazepines su ch as chlord iazepoxid e and
the enzym e that breaks d ow n acetylcholine d iazepam are appropriate because they are
(anticholinesterase m ed ications such as phy- essentially self-tap ering. The shorter-acting
sostigm ine) reverse the synd rom e and are benzod iazep ines su ch as lorazep am or oxaz-
u su ally ad m inistered IV w ith ap p rop riate ep am are u sed in p atients w ith im p aired liver
m onitoring. Atrop ine w ou ld fu rther increase fu nction, such as in this case, becau se they
the heart rate; benzod iazep ines are reserved are not d ep end ent on liver fu nction for their
for u se in agitation and anxiety, and haloperi- m etabolism and d o not accu m u late. Patients
d ol m ay w orsen anticholinergic sym ptom s. w ith u nstable m ed ical p roblem s, cognitive
im p airm ent, or old age are also cand id ates
89. (A) Anticholinesterase toxicity, m anifesting as for the shorter-acting m ed ications. Disu lfi-
nausea, vom iting, brad ycard ia, and seizu res, ram , w hich interferes w ith the m etabolism of
resu lts from excessive action of acetylcholine at alcohol, is reserved for long-term behavioral
specific receptors. Atropine, an antim uscarinic therapy of alcohol u se d isord er. Phenobarbital
agent, can reverse the symptoms. Epinephrine w as form erly u sed for d etoxification and
242 7: Pra c tic e Te s t 1

has been replaced by benzod iazepines d u e Acetaminophen, aspirin, and the opiates do not
to safety concerns and the risk of respiratory interfere w ith lithium clearance. Meperid ine,
d ep ression. how ever, can cause d angerous interactions
w ith MAOIs.
92. (D ) The patient is experiencing hallucinations
likely due to alcohol w ithd raw al. Benzod iaz- 95. (D ) Given the p atients p rior m ajor d ep ressive
epines w ill treat the underlying w ithdraw al as ep isod e and cu rrent sym p tom s, treatm ent
w ell as the hallucinations. The patient show s w ith antid ep ressants is w arranted . H ow ever,
no evid ence of either a d epressive d isord er that the patient is concerned abou t sexual d ysfunc-
should be treated w ith sertraline or fluoxetine tion, a very com m on sid e effect (up to 80% in
(SSRIs), or a bipolar disord er that w ould bene- som e stu d ies) of serotonin-sp ecific reup take
fit from lithium, a mood stabilizer. An antipsy- inhibitors su ch as citalop ram . Mirtazap ine
chotic, such as haloperid ol, is unnecessary and is an antid ep ressant w hich has the effect of
may also red uce the seizure threshold . increasing both ad renergic and serotonergic
neu rotransm ission, thu s im p roving d epres-
93. (E) Ethanol is metabolized in the liver via alco- sion, bu t w ithout cau sing appreciable sexual
hol d ehyd rogenase to acetald ehyd e, w hich is d ysfunction. All the other m ed ication choices
then enzymatically converted to acetate by the w ou ld likely equ ally cau se p roblem s w ith
enzyme acetaldehyde dehydrogenase. Disul- sexual functioning.
firam is a med ication that irreversibly inhibits
the action of ald ehyd e d ehyd rogenase. Acet- 96. (D ) While the u su al antid ep ressants (e.g.,
ald ehyde accumulates and accounts for the TCAs, SSRIs) are all effective in treating
aversive effects associated w ith the disulfiram d ep ression in a patient w ith H IV, the rapid
reaction. Flushing, sw eating, d yspnea, hyper- onset of action of stim ulants like m ethylphe-
ventilation, tachycard ia, hypotension, nausea, nid ate w ou ld be p articu larly beneficial, esp e-
and vomiting are common symptoms. Extreme cially in a m ed ically com prom ised p atient
but rare reactions can result in respiratory w ho is at risk of d ehyd ration and starvation.
d epression, card iovascular collapse, myocar- Bu p rop ion, flu oxetine, and sertraline m ay take
d ial infarction, seizures, and d eath. Blurred u p to 4 to 6 w eeks for fu ll benefit. Bu sp irone
vision an d u rinary retention are an ticholin - is a nonbenzod iazep ine anxiolytic m ed ication
ergic effects associated w ith an tip sych otic, not ind icated in the treatm ent of d ep ression as
antid epressant, and antip arkinsonian m ed i- m onotherap y.
cations. Eup horia w ou ld not be exp erienced
w hen alcohol is com bined w ith d isu lfiram . 97. (A) Attention refers to the ability to give focu s
H ypertension m ay be associated w ith, bu t is to a cognitive task. Perform ing serial sevens
not characteristic of, the d isu lfiram -ethanol (as in this case) and spelling w orld back-
reaction. Disu lfiram is not recom m end ed for w ard are tests of attention. Althou gh a certain
p atients w ith m od erate to severe liver d isease, facility w ith the rem aining choices is necessary
renal failu re, severe card iac d isease, p reg- to perform each task (no cognitive function is
nancy, or peripheral neu ropathy. tested in absolu te isolation), the serial sev-
ens test p rovid es a w ind ow on a p atients
94. (D ) Many nonsteroid al anti-inflam m atory concentration. Fu nd of know led ge inclu d es
d rugs can d ecrease the clearance of lithium and inform ation the p atient read ily has available
p rod u ce significant increases in serum levels. to him or her; know led ge of cu rrent events
Ibu p rofen, ind om ethacin, ketop rofen, d iclof- is often u sed to assess this. The m ental sta-
enac, nap roxen, and p iroxicam have all been tu s exam ination m ay contain tests of m ath-
rep orted to p rod u ce su ch p otential d anger- em atics skills, bu t testing m athem atics skills
ou s interactions. The sym ptom s of abd om inal is not the p u rp ose of the serial sevens test.
p ain, d iarrhea, and d row siness ind icate m ild Recent m em ory is recall of events occu rring
to m od erate (1.52.0 m Eq/ L) lithiu m toxicity. in the last several m inu tes. Rem ote m em ory
Answe rs : 92103 243

involves the recall of events long p ast, for atyp ical or second -generation antip sychot-
exam p le, inform ation from a p atients child - ics, nor flu p henazine, a typ ical antip sychotic,
hood . Any test of cognitive fu nction m u st w ou ld be as effective as clozap ine in this
take into accou nt the p atients cu ltu ral, ed u - case. Given his ongoing sym p tom s, restarting
cational, and social backgrou nd . qu etiap ine w ou ld not be ap p rop riate in this
p atient.
98. (B) Inhalant use is commonly a social activity
und ertaken in groups. Volatile or toxic solvents 101. (C) Divalp roex sod iu m , ind icated for the treat-
such as toluene (found in glues and ad hesives), m ent of seizu res as w ell as bip olar d isord er,
trichloroethane (found in correction fluid ), or can occasionally cau se significant hair loss
hyd rocarbons (gasoline) are placed in a bag and that is usu ally reversible. Other com m on sid e
the fumes are inhaled . The practice is know n as effects associated w ith d ivalp roex sod iu m
huffing, sniffing, or bagging. Effects are almost are nausea, vom iting, and ind igestion. Sed a-
immed iate because the substances are rapid ly tion and m ild ly elevated seru m transam inase
absorbed . Signs and symptoms of intoxication levels m ay also be seen. Rare occu rrences of
includ e visual d isturbances, d yscoord ination hep atic failu re resu lting in fatalities have hap -
(a d runken appearance), d epressed reflexes, p ened , u sually d u ring the first 6 m onths of
euphoria, and nystagmus. Conjunctival injec- treatm ent. H air loss is not associated w ith car-
tion and increased appetite are consistent w ith bam azep ine, clozap ine, olanzap ine, or zip ra-
cannabis use. A d iminished response to pain, sid one, althou gh these m ed ications have their
euphoria, and staring into space are character- ow n sid e effects.
istic of PCP intoxication.
102. (A) Sexual d ysfu nction is a very com m on sid e
99. (A) Abstinence is the only treatm ent for inhal- effect associated w ith the serotonin-sp ecific
ant u se d isord er. Antid ep ressants have not reu p take inhibitors (SSRIs), su ch as citalo-
show n utility in inhalant u se d isord er. Anti- p ram , w ith an incid ence of ap proxim ately 50%
p sychotic agents are also not effective and to 80%. Decreased libid o and d elayed orgasm
m ay aggravate the p resentations of inhalant are the m ost com m on sym p tom s, bu t erectile
intoxication. Agitation can be m anaged by d ysfunction is also not uncom m on. All SSRIs
benzod iazep ine m ed ications su ch as loraz- ap p ear to be equally as likely to cau se sexu al
ep am . Dialectical-behavioral therap y is a form d ysfunction. Priapism is a rare bu t seriou s sid e
of cognitive-behavioral therap y used in bor- effect of trazod one. Retrograd e ejacu lation is
d erline personality d isord er. Exposu re and a know n sid e effect of certain antipsychotic
resp onse p revention is a cognitive-behavioral m ed ications. Sexually transm itted d iseases are
therapy u sed in the treatm ent of anxiety d isor- not increased w ith SSRI (or any antid ep res-
d ers su ch as obsessive-com pu lsive d isord er. sant) use. Variou s p harm acologic strategies
aim ed at treating sexu al d ysfu nction inclu d e
100. (B) This p atient su ffers from treatm ent refrac- bu p rop ion and p hosp hod iesterase-5 inhibi-
tory schizophrenia. H e has had fou r m ed ica- tors, and less commonly methylphenid ate and
tion trials, includ ing both typical and atypical bu sp irone.
antip sychotics. Given his ad equ ate trials, prior
com pliance, and ongoing sym ptom s, he is an 103. (A) ADH D is a d isord er of u nclear etiology.
ap propriate cand id ate for clozap ine, the first As m any as 5% of child ren su ffer from this
atypical antip sychotic and the only one that d isord er, and ap proxim ately half of those
d em onstrates su p erior efficacy in treatm ent- d iagnosed carry the d iagnosis into ad u lthood .
refractory sch izop hren ia. While taking it The d isord er is characterized by sym p tom s
requ ires frequ ent m onitoring of his com p lete of inattention, hyperactivity, and im p ulsivity
blood cou nts, that fact that he lives w ith his that are inappropriate for age. Although the
m other w ill increase the likelihood of his com - DSM -5 criteria require that sym p tom s m ust
p liance. N either aripiprazole and zip rasid one, have been present before the age of 12 years,
244 7: Pra c tic e Te s t 1

m any cases are not d iagnosed u ntil ad oles- but the episod e lasts betw een 1 and 6 months.
cence or ad u lthood . In these cases, a retro- Distinguishing schizophrenia from mood dis-
spective history from both the patient and ord ers w ith psychotic symptoms (e.g., bipolar
the fam ily often su pport the presence of early I d isorder w ith psychotic features, MDD w ith
life sym p tom s. Althou gh som e sym p tom s are psychotic features) and schizoaffective d isor-
rem iniscent of the m anic com p onent of bip o- d er can often be d ifficult in that a mood dis-
lar d isord er, these d isord ers rep resent tw o turbance is not uncommon d uring episod es
separate entities. A carefu l history by astu te of schizophrenia and schizophreniform d isor-
p hysicians w ill secu re the p rop er d iagnosis. d ers. If psychotic symptoms only occur w ith
Dep ressive, anxiety, and su bstance-u se d is- the presence of mood symptoms, then MDD
ord ers are com m on com orbid cond itions that w ith psychotic features is more likely. Brief
often m ake the d iagnosis of ADH D m ore d if- psychotic d isord er is d efined by the presence of
ficult. The absence of prom inent m ood sym p- d elusions, hallucinations, d isorganized speech
tom s and the relative d egree of im pairm ent or behavior, or catatonic behavior lasting for at
help d istingu ish ADH D from other d isord ers. least 1 day but less than 1 month. Schizoaffec-
tive d isord er d escribes the presence of a major
104. (D ) The app roach to the treatm ent of the core d epressive or manic episod e concurrent w ith
sym ptom s of ADH D has been the initiation of symptoms that are characteristic of schizophre-
a trial of a p sychostim u lant m ed ication, su ch nia: d elusions, hallucinations, d isorganized
as m ethylp henid ate. Often, if one stim u lant speech or behavior, or negative symptoms.
fails to treat sym p tom s, another stim u lant is
p rescribed . Many patients benefit from one 107. (B) In the context of p regnancy and labor,
stim ulant and not another. Given his par- high-p otency typ ical antip sychotics such as
tial response and tolerability, the appropriate halop erid ol are consid ered relatively safe to
choice in this case w ould be to try a second type u se to control psychosis. For a m other su f-
of stimulant before the alternative strategies fering from p sychosis d u ring d elivery, the
are im plem ented . benefit of u sing high-p otency antip sychotics
ou tw eighs the risk. All of the other antipsy-
105. (A) Dj entend u is the feeling that one is hear- chotics are low - or m ed ium -potency and m ay
ing som ething one has heard before. It is u su - low er blood p ressu re significantly d u e to their
ally associated w ith anxiety states or fatigu e. alpha-blocking p rop erties.
Dj vu is a sim ilar exp erience, bu t refers to
the sensation that som ething has been seen 108111. [108 (E), 109 (G), 110 (K), 111 (D )] The
before. Jam ais vu is the op p osite of d j vu in MMSE is a qu ick, easily ad m inistered test
that it refers to som ething that shou ld be fam il- that allow s for im m ed iate screening for neu -
iar bu t seem s qu ite u nfam iliar. Folie d eu x is rocognitive d isord ers (e.g., d em entias). Scores
a shared d elu sion arou sed in one person by of less than 24 are su ggestive of a d em enting
the influ ence of another. La belle ind iffrence p rocess. The ReyOsterrieth figu re is sensitive
is the ind ifference show n tow ard a d eficit or to d eficits in copying and lack of attention to
loss of fu nction, classically (bu t not alw ays) d etail in people w ith right-sid ed parietal lobe
seen in a conversion (fu nctional neu rologic lesions. It ap p ears that this you ng w om an
sym ptom ) d isord er. m ay have had a stroke in this area resu lting
from her protein S d eficiency, a hypercoagu -
106. (E) The ep id em iology and p rognosis of schizo- lable state. The ap p rop riate test to evalu ate the
p hreniform d isord er are p oorly u nd erstood , intelligent qu otient (IQ) for child ren ages 5 to
althou gh the lifetim e p revalence rate is ap prox- 15 is the WISC. The Boston Diagnostic Ap ha-
im ately 0.2% and app roxim ately tw o-third s sia Exam ination is a series of tests given by an
eventu ally go on to d evelop schizop hrenia exp erienced clinician to evalu ate and m ake
or schizoaffective d isord er. The d iagnostic treatm ent recom m end ations for ind ivid u als
criteria are sim ilar to those of schizophrenia, w ith ap hasia. The Beck Dep ression Inventory
Answe rs : 104116 245

(A) is a 21-item test, w ith three responses p er 113. (E) The onset of action for opiates can be almost
item , that is an easily u sed screening tool to immediate when smoked , about 5 minutes
evalu ate for d ep ression. The Bend er Gestalt w hen injected , an d lon ger if taken orally.
Test (B) involves copying figu res, w hich helps Sym p tom s inclu d e m iosis, brad ycard ia,
assess neurological function or brain d isease. hyp otension, hypotherm ia, constipation, and
The Blessed Rating Scale (C) is a tool that asks eu phoria. Intoxication can lead to fatal respi-
friend s or fam ilies of the patient to assess the ratory d ep ression.
ability of the p atient to fu nction in his u su al
environm ent. MMPI-2 (F) is an objective test 114. (F) PCP is a hallu cinogen that p rod u ces a d is-
interp reted by skilled evalu ators u sed in p er- sociative anesthesia. It can cause u npred ictable
sonality assessm ent. It is the m ost w id ely u sed beh avior, assau ltiveness, an d belligeren ce.
and highly stand ard ized test of p ersonality Agitation, nystagm u s (vertical or horizontal),
stru ctu re. The Rorschach Test (H) is a p rojec- tachycard ia, a num bed resp onse to pain, m u s-
tive test u sed to assess p ersonality stru ctu re. cle rigid ity, hyperacu sis (increased sensitivity
The Wad a Test (I) is u sed to evalu ate hem i- to sou nd ), hypertension, echolalia, and anti-
spheric langu age d om inance prior to surgical cholinergic effects are associated sym p tom s.
am elioration of seizu re focu s. Whereas m ost
right-hand ed ind ivid u als show left hem i- 115. (D ) N icotine fou nd in tobacco sm oke p ro-
spheric d om inance for langu age, left-hand ed d u ces sym p tom s of excitem ent. Intoxication
ind ivid u als m ay either be right or left d om i- w ith large am ou nts m ay p rod u ce sym p tom s
nant. The test consists of injecting sod iu m of confu sion, m u scle tw itching, w eakness,
am ytal into the carotid artery and observing abd om inal cram p s, d epression, p alp itations,
the transient effects on sp eech. Injection into com a, and respiratory failu re.
the left carotid artery w ill anesthetize the left
sid e of the brain; those w ith left hem ispheric 116. (A) As a sym p athom im etic agent, cocaine
langu age d om inance w ill show interru p ted increases the heart rate and d ilates the p u p ils.
speech. The WAIS-R (J) is u sed to d eterm ine It can also cau se constriction of the coronary
the IQ for ind ivid u als age 15 and old er. arteries. The hallm ark of cocaine intoxication
is eu p horia. Use of LSD (B) or other hallu ci-
112. (C) Cannabis (m ariju ana) is the m ost com - nogens (e.g., MDMA [3,4-m ethylened ioxy-
m only abu sed illicit d ru g in the United N -m ethylam p hetam ine], or Ecstasy) cau ses
States. The onset of action after sm oking is sym p tom s that u su ally begin 1 hou r after
im m ed iate, and sym p tom s inclu d e conju ncti- ingestion and generally last 8 to 12 hou rs.
val injection, m ild sed ation, d ose-d ep end ent Most hallucinogenic d rugs have stim u lant-
hyp otherm ia, d ry m ou th, increased ap p etite, type effects and prod u ce elevated vital signs
tachycard ia, and eu p horia. A sensation of and increased activity. H allu cinations and a
slow ed tim e and p aranoid id eation can also heightened sense of percep tion of objects and
occu r. colors are typical.
This page intentionally left blank
CHAPTER 8

Prac tic e Te s t 2
Que s tions

D IRECTION S (Questions 1 through 4): For each the lettered answ er that is the one best response in
of the follow ing vignettes, select the one lettered each case
option that is most closely associated w ith it. Each
lettered option may be used once, multiple times, 5. An 18-ye r-ol w o n recently i gnose
or not at all. w ith first e iso e of schizo hreni grees
to t ke n nti sychotic e ic tion to
(A) Cit lo r hel ecre se her h llu cin tions, bu t she is
(B) Flu oxetine ntly o ose to t king ny e ic tion
(C) Mirt z ine th t y c u se her to g in excessive w eight.
(D) P roxetine Which of the follow ing e ic tions w ou l be
(E) Sertr line the ost ro ri te to rescribe?
(F) Venl f xine (A) Cloz ine (Cloz ril)
(B) Ol nz ine (Zy rex )
1. A 29-ye r-ol new ly rrie le w ith (C) Qu eti ine (Seroqu el)
signific nt e ressive sy to s, inclu ing
(D) Ris eri one (Ris er l)
inso ni n w eight loss, w s nonco li nt
w ith e ic tions in the st u e to sexu l (E) Zi r si one (Geo on)
ysfunction.
6. A 40-ye r-ol rrie fe le w ithout st
sychi tric history is referre by her inter-
2. A 45-ye r-ol n w ith severe jor e res- nist. She h s been feeling ow n for sever l
sion on h igh ose of n n ti e ress nt w eeks, w ith little bility to enjoy herself. Since
evelo s incre se bloo ressu re. th t ti e, her slee h s been isru te , her
etite re uce , n shes felt tire . She
3. A 50-ye r-ol ivorce fe le being tre te for h s lso been king ist kes t w ork n is
jor e ression oesnt refill her rescri tion w orrie bou t being fire . She its to h v-
n 2 ys l ter h s he che, u scle ches, ing thou ghts of killing herself, bu t she enies
n n use . ny l n or intent to o so. She enies h llu -
cin tions or elu sions. Which of the follow ing
4. A 33-ye r-ol w o n w ith ersistent e re- e ic tions w ou l be the ost ro ri te
ssive isor er ( ysthy i ) h s history of oor for this tient?
co li n ce w ith e ic tion s, occ sion lly (A) Ari i r zole
issing oses. (B) Cit lo r
(C) Methyl heni te
D IRECTION S (Questions 5 through 41): For each of (D) N ortri tyline
the multiple-choice questions in this section select
(E) Tr nylcy ro ine

247
248 8: Pra c tic e Te s t 2

7. A 25-ye r-ol w o n resents to her ri ry oes not feel like her con ition w ill i rove,
c re octor co l ining of su en onset of she enies ny su ici l i e tion. She rinks
intense fe r n feeling th t she w s going one to tw o rinks once or tw ice er onth,
to ie, w hich occurre w hile she w s sto e bu t enies illicit ru gs. She is su bsequ ently
in tr ffic e rlier in the w eek. At th t ti e she begu n on flu oxetine, w hich is gr u lly
bec e short of bre th, i horetic, n tre - incre se to 60 g. She enies si e effects,
u lous, n she cou l feel her he rt oun ing. lthou gh she h s not ex erience signific nt
H er initi l i u lse w s to rive to n e er- lessening of her e ressive sy to s. H ow
gency e rt ent, bu t her istress su bsi e long shou l her cu rrent os ge be continu e
on its ow n in bou t 20 inu tes. Which of the before sw itching to nother e ic tion?
follow ing shou l be the next ost ro ri te
(A) 3 ys
cou rse of ction?
(B) 1 w eek
(A) Con u ct thorou gh e ic l screening (C) 2 w eeks
for he rt n lu ng ise se.
(D) 4 w eeks
(B) Prescribe selective-serotonin reu t ke
(E) 6 w eeks
inhibitor (SSRI) n sk to see the
tient b ck in 1 w eek.
10. A 56-ye r-ol w o n w ith schizo hreni h s
(C) Prescribe short- cting benzo i ze ine been st ble on h lo eri ol for ny ye rs.
s nee e in c se of nother e iso e. She enies ny current sychotic sy to s
(D) Re ssu re the tient th t her sy to s n h s been co li nt w ith her e ic tions.
re ost likely benign n not c u se She resents for rou tine follow -u oint-
for concern. ent. Which of the follow ing tests shou l be
(E) Refer the tient to sychi trist. inistere ?
(A) Abnor l Involu nt ry Move ent Sc le
8. You re seeing 45-ye r-ol n w ith his-
(AIMS)
tory of schizo hreni . H e is cu rrently co -
li nt w ith his nti sychotic e ic tions (B) Beck De ression Inventory (BDI)
bu t re ins sy to tic, w ith u itory h l- (C) Brief Psychi tric R ting Sc le (BPRS)
lu cin tions n r noi . On ent l st tu s (D) P tient H e lth Qu estionn ire (PH Q)
ex in tion (MSE), he s e ks coherently n (E) Positive n N eg tive Sy to Sc le
rticul tely, bu t kes no sense bec u se ny (PAN SS)
of the w or s he uses re of his ow n invention.
Which of the follow ing ter s best escribes 11. A n brings his w ife to the octor bec u se
this MSE sign? she h s beco e ore sy to tic over the
(A) Cl ng ssoci tions st sever l w eeks. She is i gnose w ith
schizo hreni n h s recently sto e her
(B) Echol li
e ic tions. She h s e thre ts tow r
(C) Flight of i e s her hu sb n bec u se she believes th t he is n
(D) N eologis s i ostor w ho looks ex ctly like her hu sb n .
(E) Wor s l Which of the follow ing ter s best escribes
this sy to ?
9. A 34-ye r-ol w o n resents to sychi -
(A) A ok
trist w ith 6-ye r history of e tiness n
e ression for ost of the ti e, w ith low (B) C gr s syn ro e
self-estee , chronic inso ni , n ifficulty (C) Cot r syn ro e
king ecisions. She enies ny roble s (D) Cou v e syn ro e
w ith her etite or energy, n , w hile she (E) Koro
Que s tions : 717 249

12. A 69-ye r-ol w o n w ithout rior sychi tric (C) Echol li


history is seen by her ri ry c re hysici n (D) P lil li
ue to crying s ells. She st tes th t she h s (E) P r r xis
felt s since the su en e th of her husb n
5 w eeks go. Since th t ti e, she h s h iffi- Questions 15 and 16
culty slee ing n f tigue. She h s lso lost sev-
er l oun s ue to not enjoying her foo . She A w e lthy, ivorce 48-ye r-ol w o n resents
is ble to cheer u w hen s en ing ti e w ith to you fter being rreste for sho lifting. The
her chil ren, but feels es eci lly lonely t night. tient its th t she h s been ste ling for ye rs,
She is so ew h t nxious bec use she occ - lthou gh she is e sily ble to ffor the objects sto-
sion lly he rs her ece se husb n s voice, len. She st tes th t she ste ls on the s u r of the
es eci lly e rly in the orning or l te in the o ent n th t these i u lses see foreign n
evening. While she feels her future is ble k, istressing.
she enies ny suici l i e tion or l n. Which
of the follow ing i gnoses is ost likely? 15. Which of the follow ing i gnoses is the ost
likely?
(A) Bere ve ent
(B) Bi ol r isor er (A) Antisoci l erson lity isor er
(C) M jor e ressive isor er (MDD) (B) Inter ittent ex losive isor er
(D) Persistent e ressive isor er (C) Kle to ni
( ysthy i ) (D) Pyro ni
(E) Schizo ffective isor er (E) Trichotillo ni

Questions 13 and 14 16. Which of the follow ing ter s lies to the
f ct th t these i u lses re istressing to her?
You re ske to ev lu te 12-ye r-ol boy with his-
tory of Tourette isor er n obsessive-co ulsive (A) Delu sion l
isor er. He is brought in to the oint ent with his (B) Ego- ystonic
rents, who escribe the severity of his illness n (C) Ego-syntonic
how he is often te se in school. During the interview (D) Moo congruent
the boy inter ittently kes obscene gestures.
(E) Moo incongru ent
13. Which of the follow ing ter s best escribes
17. A 49-ye r-ol n co es to you co l in-
this sy to ?
ing of he ches, e ory loss, isorient -
(A) Ble h ros s tion , n occ sion l r lysis th t ffects
(B) Bru xis his r s n l sts sever l hou rs. Du ring
(C) Co ro r xi th e MSE, you notice th t th e tient is giv-
ing v gu e nsw ers to ny qu estions (e.g.,
(D) Echo r xi
th ere re six toes on th e foot n 2 + 2 = 5).
(E) Torticollis Which of th e follow in g ter s best escribes
th is resen t tion?
14. While obt ining the history, the tient is lso
note to re e t hr ses you u se i e i tely (A) A ok
fter you s y the . Which of the follow ing (B) Dere liz tion
ter s best escribes this sy to ? (C) G nser syn ro e
(A) Co rol li (D) N eg tivis
(B) Dys rthri (E) Piblokto
250 8: Pra c tic e Te s t 2

18. A sychi trist iscovers th t she is fru str te believe her thoughts coul be he r through the
n e sily ngere w ith one of her tients w lls. She feels th t this is n inv sion of her riv cy.
for no obvious re son. While t lking to col-
le gue, she its th t the tient re in s her 21. Which of the follow ing ter s best escribes
of her busive f ther. Which of the follow ing the belief th t the tients thoughts tow r
best escribes the clinici ns re ction? the neighbor w ere res onsible for the lost
regn ncy?
(A) Countertr nsference
(B) Dis l ce ent (A) Dis l ce ent
(C) Projection (B) I e s of reference
(D) Re ction for tion (C) M gic l thinking
(E) Tr nsference (D) Projection
(E) Re ction for tion
Questions 19 and 20
22. Which of the follow ing ter s best escribes
A 19-ye r-ol w o n resents w ith co l ints the tients fe r th t her thoughts cou l be
of fe r, rehension, n tre bling w ithou t ny overhe r ?
know n re is osing situ tions. On ex in tion,
you note brow n skin, s oky brow n rings on the (A) Echol li
ou ter corne , n occ sion l r i , jerky, u r ose- (B) Thou ght bro c sting
less sw inging of the r s w hich e rs to w orsen (C) Thou ght control
w ith volu nt ry ove ent. (D) Thou ght insertion
(E) Tr nsference
19. Which of the follow ing ter s best escribes
the tients ocul r fin ings?
Questions 23 and 24
(A) Arcus senilis
A 25-ye r-ol n resents to the e ergency roo
(B) Bru shfiel s ots
fter being brou ght in by his girlfrien . She h s
(C) K yserFleischer rings been concerne s he h s not been cting right.
(D) Su bconju nctiv l he orrh ge She st tes he h s been st ying u ll night over
(E) X nthel s the l st sever l ys but still very ctive uring
the yti e. H e h s re rr nge the fu rniture n
20. Which of the follow ing ter s best escribes tte te to re o el their b throo , es ite his
the bnor l ove ents seen in this tient? l ck of ex erience w ith constru ction. H e h snt
show n u for w ork recently, n she is fr i he
(A) Athetoi
w ill be fire . While he its th t he h snt sle t,
(B) Choreifor he cl i s to feel gre t! H e is so ew h t ifficu lt
(C) H e ib llis u s to interview s he w lks roun the roo , h n-
(D) Myoclonu s ling ll the equ i ent. When confronte w ith his
(E) Myotoni not going to his w ork, he st tes Why shou l I . . . I
on the verge of iscovering cu re for AIDS,
Questions 21 and 22 w hich lso h ens to be for u l for econo ics,
w hich w ill lso get ri of overty. H e enies lco-
A 21-ye r-ol w o n is brought to the sychi tric hol or ru g u se, bu t u on qu estioning, its to
e ergency e rt ent fter c lling the olice to erio in his l te teens w hen he felt e resse , w ith
tu rn herself in. She cl i s th t she w s res onsible ifficu lty slee ing, w eight loss, low energy, gu ilt,
for the loss of her neighbors regn ncy. She believes n hel lessness/ ho elessness. H e w s su bse-
her neg tive thoughts tow r the w o n c u se her qu ently tre te w ith so e sort of nti e ress nt
isc rri ge. On further qu estioning, she tells you for roxi tely 18 onths, bu t he never follow e
th t she felt thre tene by her neighbor bec u se she u fter th t ti e.
Que s tions : 1829 251

23. Which of the follow ing is the ost ro ri te (A) Conversion isor er
i gnosis for this tient? (B) F ctitiou s isor er
(A) Bi ol r I isor er (C) Illness nxiety isor er
(B) Bi ol r II isor er (D) M lingering
(C) Cyclothy ic isor er (E) So tic sy to isor er
(D) M jor e ressive isor er w ith
sychotic fe tu res 27. Which of the follow ing ter s best escribes
this tients ini iz tion of the severity of
(E) Schizo hreni
his sy to s?
24. Which of the follow ing e ic tions w ou l be (A) Dj enten u
the ost ro ri te to rescribe? (B) Dj vu
(A) A itri tyline (C) Folie eu x
(B) Cloz ine (D) J is vu
(C) Flu oxetine (E) L belle in iffrence
(D) H lo eri ol
28. A 67-ye r-ol w o n w ith u l on ry c rci-
(E) V l roic ci
no n secon ry br in et st ses recently
r fte w ill in the resence of her f ily
25. A 36-ye r-ol w o n w s begu n on l r -
ttorney. She h s history of jor e ressive
zol (X n x) 3 ye rs go for nic isor er.
isor er (MDD) th t is now in re ission. She
After w tching new s re ort on television,
eci es th t her chil ren w ho re w ell est b-
she bec e frightene bou t iction. She
lishe in their c reers o not nee ny inheri-
bru tly sto e the e ic tion 2 ys go
t nce n th t her est te w oul best serve
n h s since been ex eriencing cu te, intense
ch rity. To secure the v li ity of the w ill, the
nxiety, tre ors, sw e ting, n l it tions.
tient sks her sychi trist to sub it let-
Which of the follow ing e ic tions w ou l be
ter to her ttorney reg r ing her co etency.
the ost ro ri te to rescribe?
Which of the follow ing w ou l be the ost
(A) Di ze i ort nt f ctor in eter ining this w o ns
(B) Flu zenil test ent ry c city?
(C) Pro r nolol (A) Actu s reu s
(D) Sertr line (B) A history of MDD
(E) V l roic ci (C) Know le ge of her n tu r l heirs
(D) The resence of conserv tor of erson
Questions 26 and 27
(E) The resence of ju ge to w itness the
A evou t hu sb n fin s th t his w ife is h ving n signing of the w ill
ff ir w ith his best frien . One w eek l ter, he fin s
th t he c nnot w lk. A thorou gh neurologic w orku 29. You re consu lte to see 77-ye r-ol H is-
f ils to reve l c u se to his su en r legi . H is nic le w ith schizo ffective isor er,
neu rologic ex in tion is not consistent w ith u er e ressive ty e, itte to the hos it l for
or low er otor neuron fin ings. Des ite this r - chest in. H is w orku h s reve le u l o-
tic is bility, he see s qu ite un ffecte by it n ry e bolis , n the e ic l te w ishes
e otion lly. to l ce chest tube in the tient, bu t the
tient refu ses. The ri ry te h s ske
26. Which of the follow ing i gnoses w ou l be you to eter ine his c city to refu se the
the ost ro ri te in this tient? roce u re. Which of the follow ing fe tures is
252 8: Pra c tic e Te s t 2

the ost i ort nt in eter ining ecision- Questions 32 and 33


king c city in this tient?
You re ske to erfor co etency to st n
(A) Ability to co u nic te tri l ev lu tion of n 18-ye r-ol n w ho w s
(B) Age of tient rreste for violently ss u lting his girlfrien . The
(C) Agree ent w ith tre t ent te tient h s no st sychi tric history, n frien s
reco en tions re ort th t he h s never e onstr te ny violent
(D) Presence of sychosis ten encies. The tient re orts th t his rel tion-
shi w ith his girlfrien w ent ow nhill shortly
(E) S nish s e king
fter they gr u te fro high school n his girl-
frien sto e c lling hi . On MSE, his ffect is
30. A e uty sheriff serves you su b oen for
constricte n his oo is re orte s cr zy.
the recor s of one of your tients w ho is the
Thou ght rocesses re go l irecte , n he enies
efen nt in civil li bility l w su it. Which of
ny h llu cin tions or elu sions. The tient is
the follow ing is the ost ro ri te cou rse
u n ble to correctly erfor si le c lcu l tions.
of ction?
E ch ti e he is ske to u lti ly, su btr ct, or
(A) Rele se the tients recor s to the ir of nu bers, his nsw ers re w rong by one
l intiff bec u se the su b oen overri es or tw o igits. For ex le, he res on s 22 w hen
tient consent. ske to u lti ly 7 by 3.
(B) H n over only the infor tion th t is
relev nt to the c se. 32. Which of the follow ing i gnoses is the ost
(C) Cont ct the tient n sk if she or he likely in this c se?
w ou l like the infor tion rele se . (A) Conversion isor er
(D) Rele se the tients recor s irectly to (B) F ctitiou s isor er
the cou rt. (C) M jor neu rocognitive isor er
(E) Refu se to s e k to the sheriff in or er to (D) M lingering
int in confi enti lity.
(E) Schizo hreni
31. A 23-ye r-ol gr u te stu ent you h ve
33. Which of the follow ing st te ents best
been seeing in long-ter sychother y h s
escribes the bove tients ro u ction of
sto e ying his bills, es ite being rei -
sy to s n otiv tion?
bu rse by his insu r nce co ny. Which of
the follow ing is the next ost ro ri te (A) Consciou s ro u ction of sy to s to
cou rse of ction? ssu e the sick role.
(A) Cont ct the tients insu r nce co ny (B) Consciou s ro u ction of sy to s to
n requ est th t they issue you nother obt in secon ry g in.
y ent. (C) Unconsciou s ro u ction of sy to s
(B) Directly ress this issu e w ith u e to u nconsciou s conflicts.
the tient t the next sche u le (D) Unconsciou s ro u ction of sy to s to
oint ent. ssu e the sick role.
(C) Inqu ire w ith f ily e bers w hether (E) Unconsciou s ro u ction of sy to s to
the tient h s fin nci l roble s. obt in secon ry g in.
(D) N otify collection gency to obt in
rei bu rse ent. 34. You re ske to ev lu te 68-ye r-ol n
on the in tient e icine service for incre s-
(E) Sen the tient letter of ter in tion.
ing confu sion. The tient w s itte
2 ys e rlier for neu oni . After erfor -
ing ent l st tu s ev lu tion, you su s ect
eliriu . Fu rther history fro the tients
Que s tions : 3039 253

w ife in ic tes th t the tient ro e ou t 36. Which of the follow ing w ou l be the ost
of school in the seventh gr e. Which of the ro ri te next ste in the n ge ent of
follow ing tests w ou l best ssess his bility to this tient?
int in n focu s ttention?
(A) A inister thi ine or lly (PO) before
(A) Cou nting by 2s to 20 IV flu i s n glu cose.
(B) R n o letter test (B) A inister thi ine intr venou sly (IV)
(C) Seri l sevens before IV flu i s n glu cose.
(D) Seri l threes (C) A inister thi ine IV fter IV flu i s
(E) Si le c lcu l tions n glu cose.
(D) A inister n loxone IV before IV flu i s
35. A 7-ye r-ol girl is brou ght to the e ergency n glu cose.
e rt ent for ev lu tion of sore thro t (E) A inister n loxone IV fter IV flu i s
n fever. H er rents took her to the e i - n glu cose.
trici ns office bou t 1 w eek go n he rec-
o en e flu i s n be rest. Within the 37. The bove tient is st bilize n itte
l st 2 ys, the tient evelo e ys h gi to the ICU. After 3 ys, he begins to beco e
n severe b o in l in. H istory fro the confu se g in, w ith visu l h llu cin tions,
rents in ic tes the tient h s been nxiou s tre ors, i horesis, n elev te bloo res-
n i u lsive over the l st onth. There h s su re n u lse. Which of the follow ing is the
lso been rke ecline in school erfor- ost ro ri te tre t ent for this tient?
nce, n she h s not been intereste in
l ying w ith her frien s. Physic l ex in - (A) A inister b rbitu r te.
tion is re rk ble for signific nt erythe (B) A inister benzo i ze ine.
over the osterior h rynx w ith gr y exu - (C) A inister ition l thi ine.
te. There re br sions in the region of the (D) A inister n nti sychotic.
tients l bi . Co lete bloo cou nt show s (E) A inister hy r l zine.
w hite bloo cell (WBC) cou nt of 14,000/ L
w ith left shift. Which of the follow ing is the 38. A you ng w hite le, ge u nknow n, is brou ght
next ost ro ri te ste ? into the e ergency roo u nres onsive to
(A) Arr nge for f ily eeting to qu estioning. H is vit ls e onstr te nor l
eter ine s fe is osition. te er tu re, low u lse n bloo ressure,
(B) Cont ct the st tes Chil Protective n ecre se res ir tions. H e e rs le,
Services w hile kee ing the tient s fe. w ith u ils th t re constricte n ini lly
res onsive. A inistr tion of w hich of the
(C) Det in the rents w hile you notify the
follow ing w ou l be ost likely to i rove his
olice.
con ition?
(D) N otify the rob te court to h ve the
tient leg lly re ove fro the (A) Disu lfir (Ant bu se)
llege er etr tor. (B) Benzo i ze ines
(E) Refer the tient to the f ilys (C) Flu zenil
e i trici n. (D) N loxone
(E) Thi ine
Questions 36 and 37
A 48-ye r-ol le is brou ght into the e ergency 39. A 77-ye r-ol w o n w ithout rior sychi tric
roo (ER) vi n bu l nce. H e s ells of lcohol, is history is brought into her f ily hysici ns
covere in vo it, n not res onsive to qu estions. office w ith her husb n . H e is concerne th t
H e is unste y n u ncoo er tive w ith the hysic l she is e resse n nee s tre t ent. H e
ex in tion, bu t he is note to h ve isconju g te escribes her w ith r w ing e otion lly over
eye ove ents. the st ye r or tw o, w ith gr u l ecline in
254 8: Pra c tic e Te s t 2

her bility to c re for herself. They h ve been (D) R ce


un ble to t ke rt in their u su l soci l ctivi- (E) Religion
ties, n the tient ju st w n ers roun the
hou se uring the y. H e h s lso notice 41. The tient w its u ntil her f ily n ri ry
th t she is forgetful, often is l cing ite s, c re te le ve the roo . She then sks you to
n ixing u n es of cqu int nces. When hel her co it su ici e, s her rognosis is
ske , the tient enies ny ifficulties, st t- very oor. Wh t is the ost ro ri te next
ing she feels fine, n th t her husb n w or- course of ction?
ries too uch. She ntly refutes roble s
w ith her e ory, inste bl ing hi for (A) Discu ss her re sons for su ici e in ore
oving things roun in their house. She h s et il.
no signific nt e ic l roble s. U on MSE, (B) Ex l in how she c n obt in leth l oses
she is le s nt n coo er tive w ith questions, of e ic tion.
lthough efensive t ti es. H er ffect is neu- (C) Infor the f ily i e i tely.
tr l but full. There is no su ici l or ho ici l (D) Pl ce the tient in restr ints.
i e tion, n she enies ny sychotic sy - (E) Refer her to hysici n w ho erfor s
to s. H er Mini-Ment l St te Ex in tion is eu th n si .
20/ 30. H er hysic l is essenti lly u nre rk-
ble. Which of the follow ing w oul be the D IRECTION S (Questions 42 through 49): Match
ost ro ri te ri ry tre t ent for this the clinical presentation w ith the appropriate neu-
tient? ropsychological test. Each lettered heading may be
(A) Cit lo r selected once, more than once, or not at all.
(B) G l nt ine (A) Beck De ression Inventory (BDI)
(C) Ginkgo bilob (B) Ben er Gest lt Test
(D) Me ntine (C) Blesse R ting Sc le
(E) Zi r si one (D) Boston Di gnostic A h si Ex in tion
(E) Folstein Mini-Ment l St te Ex in tion
Questions 40 and 41 (MMSE)
You re ske to see 37-ye r-ol w hite C tholic (F) Minnesot Mu lti h sic Person lity
fe le w ith en -st ge ov ri n c ncer bec u se she Inventory 2 (MMPI-2)
h s tol her oncologist th t she w nts to ie. When (G) ReyOsterrieth Test
you ro ch the tients be , you see c chectic (H ) Rorsch ch Test
bu t s iling w o n su rrou n e by her hu sb n n (I) Stroo Test
tw o you ng u ghters. She h s history of e res-
(J) W Test
sion bu t is not un er ny tre t ent for it now . I
re y to go, Doctor, she s ys to you . H er oncologist (K) Wechsler A u lt Intelligence Sc le
h s tol you th t he w oul like to try new che o- Revise (WAIS-R)
ther y for w hich the tient is goo c n i te, (L) Wisconsin C r Sorting Test (WCST)
bu t the tient h s refu se ; he is concerne bou t
her suici l otenti l. 42. A 40-ye r-ol w o n w ho scores 26/ 30 on
the Folstein MMSE g ve ny nsw ers of, I
40. Which of the follow ing ch r cteristics incre se ont know, I too tire to nsw er. You w nt
this rticu l r tients risk of co itting to ssess for the ossibility of e ression.
su ici e?
(A) Age 43. A 65-ye r-ol n h s ifficu lty org nizing,
sequ encing, n l nning ctivities but no
(B) Gen er
i ir ent in e ory.
(C) M rit l st tu s
Que s tions : 4051 255

44. The f ily of n 80-ye r-ol n w ith neu - chorionic gon otro in is ositive. The tients
rocognitive isor er ( e enti ) h s ske rents e n to know the resu lts. Which of
you to ev lu te his bility to continu e to live the follow ing is the ost ro ri te cou rse
in his cu rrent environ ent. You w ish to sk of ction?
his f ily n frien s their o inion of how he
(A) Disclose the resu lts in eeting w ith
h s been oing.
the tient n her rents.
(B) Encou r ge the tient to iscu ss the
45. A 16-ye r-ol boy w ith f ily history of resu lts w ith her rents.
intellectu l is bility resents w ith longst n -
(C) Invoke testi oni l rivilege.
ing oor school erfor nce n ggressive
beh vior tow r eers. (D) N otify the rents of the resu lts bec u se
they y her e ic l bills.
(E) Re ort the resu lts to Chil Protective
46. A 37-ye r-ol n h s history of voi ing
Services.
soci l situ tions, no close frien s, n ref-
erence for being lone. H e h s been escribe
Questions 51 and 52
by others s u ne otion l n et che . You
woul like to ev lu te this tients erson lity
The tient is 31-ye r-ol , ivorce le, w ithou t
style w ith rojective test.
rior sychi tric history, referre by his w ork for
ev lu tion. H e st tes he h s felt ow n since his
47. You w oul like to quickly ssess for oten- se r tion n ivorce sever l onths go. H e h s
ti l neurocognitive isor er in 75-ye r-ol been slee ing oorly, frequ ently w king u u ring
wo n itte to the e ergency e rt ent the night, then feeling exh u ste u ring the y.
for f ilure to thrive. He h s h little interest in foo , losing 10 lb, n
he h s been e sily istr cte t w ork, resu lting in
48. A 45-ye r-ol w o n h s long history of ist kes n re ri n . While he feels hel less
u nst ble rel tionshi s, self-injuriou s beh vior, n th t Ill never feel better, he enies ny su i-
n ffective inst bility. She oes not eet cri- ci l i e tion or ho ici l i e tion. H e rinks 1 to
teri for jor e ressive, bi ol r, or nxi- 2 beers on the w eeken s n enies illicit ru g u se.
ety isor er. You w ish to e loy n objective H e s okes 1/ 2 ck of cig rettes er y n h s
test to give fu rther evi ence for erson lity no jor e ic l roble s exce t for bor erline
isor er. hy ertension. H e t kes u ltivit in bu t no other
e ic tions regul rly. After fu rther iscu ssion, he
grees to begin tri l of e ic tion. The tient
49. You re ske to reo er tively ev lu te is begu n on flu oxetine 20 g ily. H e is co li nt
he is heric o in nce in n 18-ye r-ol w ith the e ic tion n follow s u regu l rly. After
left-h n e w o n w ith history of seizu re 4 w eeks he re orts slight i rove ent in his sy -
isor er w ho is bou t to u n ergo su rgery to to s. Of note, he initi lly h nxiety n loose
re ove seizu re focu s in her left he is here. stools, bu t he enies current si e effects.

DIRECTIONS (Questions 50 through 83): For each 51. Which of the follow ing w ou l be the ost
of the multiple-choice questions in this section select ro ri te next ste ?
the lettered answer that is the one best response in
(A) A ri i r zole.
each case.
(B) A lithiu .
50. A 16-ye r-ol girl is brou ght to you r office (C) Discontinu e the flu oxetine.
by her rents to get regn ncy test. She (D) Incre se flu oxetine to 40 g.
consents for the ev lu tion bu t requ ests th t (E) Sw itch to sertr line.
you kee the resu lts confi enti l. Bet hu n
256 8: Pra c tic e Te s t 2

52. H e eventu lly chieves fu ll re ission of his (D) Refer her for electroconvu lsive ther y
e ressive sy to s n continu es on the (ECT).
e ic tion. H ow ever, he is concerne bou t (E) Tell her to sto her e ic tion
his rognosis n otenti lly sto ing the i e i tely.
nti e ress nt in the fu tu re. Wh t o you tell
hi is his roxi te risk of evelo ing 55. A 45-ye r-ol n is brought to the sychi tric
fu rther e iso e in the fu tu re? e ergency roo by olice fter being foun
(A) 10% scre ing w hile cutting ow n tree in the loc l
rk. H e tells you th t the ghost of his w ife,
(B) 20%
w ho ie 3 onths go, is living in the tree.
(C) 30% The n h s no revious sychi tric history,
(D) 40% but he re orts being very e resse since the
(E) 50% e th of his w ife n h s h suici l i e tion.
H e is ishevele n e rs to h ve ignore
Questions 53 and 54 his erson l hygiene for so e ti e. H is vit l
signs re nor l. Which of the follow ing is the
A 35-ye r-ol w o n resents to you r office w ith
ost ro ri te next course of ction?
referr l fro sychologist for sychi tric tre t-
ent. She h s been su ffering the blues for sever l (A) Obt in u rine toxicology screen.
onths, inclu ing crying s ells, low energy, n (B) Prescribe n nti e ress nt.
hy erso ni . She is convince th t her fi nc is (C) Prescribe n nti sychotic.
bout to bre k u w ith her bec use she h s been e t- (D) Prescribe n nti sychotic n n
ing ore th n usu l. So eti es, how ever, she feels nti e ress nt.
ju st fine.
(E) Prescribe oo st bilizer.
53. Wh ich of th e follow in g w ou l be th e ost
Questions 56 and 57
ro ri te tre t en t of choice for th is
tient?
You re seeing 56-ye r-ol n w ith history of
(A) A itri tyline (El vil) lcohol u se isor er. H e co l ins bou t not being
(B) Div l roex so iu (De kote) ble to slee n requ ests slee ing i . H e enies
(C) Flu oxetine (Proz c) erv sive e ression or ny ch nge in his etite,
energy, or concentr tion. H e cl i s th t otherw ise
(D) Phenelzine (N r il)
everything is going w ell.
(E) Zi r si one (Geo on)
56. Which of the follow ing e ic tions w ou l
54. The tient res on s w ell to tre t ent, n
be the ost ro ri te to rescribe for this
fter sever l onths retu rns to you r office for
tient?
follow -u visit. She tells you she h s just
been rrie n w ou l like to beco e reg- (A) Al r zol (X n x)
n nt. She n her hu sb n h ve been h ving (B) Di ze (V liu )
u n rotecte sex for onth. Which of the fol- (C) Tr zo one (Desyrel)
low ing w ou l be the ost ro ri te next (D) Z le lon (Son t )
cou rse of ction?
(E) Zol i e (A bien)
(A) A nother nti e ress nt to revent
ost rtu e ression. 57. H e grees to begin the ro ri te bove e -
(B) Ask how i ort nt regn ncy is to her. ic tion. Which of the follow ing si e effects
(C) Discu ss the risks n benefits of w oul be ost i ort nt to w rn hi bout?
continu ing her tre t ent u ring (A) Anorg s i
regn ncy. (B) I otence
Que s tions : 5261 257

(C) Incontinence Questions 60 and 61


(D) Pri is
A 38-ye r-ol tient w ith history of bi ol r isor-
(E) Retrogr e ej cul tion er h his first ( n only) nic e iso e 18 onths
go. H e w s tre te for 1 ye r w ith oo st bi-
Questions 58 and 59 lizer, n h s not t ken ny sychotro ics since th t
58. You re seeing 9-ye r-ol boy w ith history ti e. H e h s co e in for n ev lu tion t the urging
of e ression n suici l i e tion. H e h s of his w ife, w ho is concerne bout his frequent cry-
been t king flu oxetine (Proz c) for 6 onths ing s ells. The tient its th t he h s felt ore
n h s h goo res onse. H is other e resse over the l st sever l w eeks, w ith ssoci-
sks to see you fter his oint ent n tells te inso ni , low energy, oor etite n 5-lb
you th t she is concerne bou t new s re orts w eight loss, ifficu lty concentr ting, n nhe o-
s ying th t serotonin-s ecific reu t ke inhi- ni . H e enies ny su ici l i e tion. Fortu n tely,
bitors (SSRIs) incre se the risk of su ici e in these sy to s h ve not interfere w ith his w ork.
chil ren. She sks you to consi er sto ing H e h s no jor e ic l roble s n is on no
the e ic tion. Which of the follow ing is the e ic tions. H e rinks one to tw o gl sses of w ine
best cou rse of ction? sever l ti es er ye r n enies illicit ru g u se.

(A) Ask the other, Wh ts re lly 60. Which of the follow ing e ic tions w ou l be
bothering you bou t you r son being on the ost ro ri te to rescribe this tient?
nti e ress nts?
(A) Bu ro ion
(B) Discu ss the risks n benefits of
flu oxetine n other SSRIs in the (B) Flu oxetine
e i tric o u l tion. (C) H lo eri ol
(C) Sto the flu oxetine i e i tely s the (D) L otrigine
tient is inor n it is his others (E) V l roic ci
w ish.
(D) Sw itch to itri tyline (El vil). 61. Which of the follow ing si e effects w ou l be
(E) Sw itch to roxetine (P xil). the ost i ort nt to iscu ss w ith the tient
rior to st rting?
59. A onth l ter, the boy n his other return to (A) At xi
your office. She infor s you th t fter her sons (B) N u se
l st oint ent, she h s been giving hi
(C) N eu tro eni
St. John wort bec use she believes he nee s
n tur l re e y. Which of the following (D) R sh
woul be the ost ro ri te course of ction? (E) Se tion

(A) Incre se the ose of his nti e ress nt Questions 62 and 63


s St. John w ort w ill low er bloo
concentr tions of the nti e ress nt. A 60-ye r-ol w hite fe le w i ow is brought to you r
(B) Refer the c se to soci l services for chil clinic by her ughter, w ho re orts th t her other
bu se. s i to her this fternoon th t she w nte to en it
(C) Tell the other there is no evi ence th t ll. She re orts thou ghts of over osing on her ills,
St. John w ort is n effective tre t ent w hich she h s stock ile . This tient h s no rior
for e ression. history of su ici e tte ts bu t h s h thou ghts of
su ici e on ulti le rior occ sions over the cou rse
(D) Tell the other to sto the St. John w ort
of her ne r-lifelong history of e ression. She lost
i e i tely.
her hu sb n to su en he rt tt ck 1 ye r go, but
(E) Tell the other to w tch ou t for si e her u ghter s ys th t she still h s ny very su -
effects inclu ing hotosensitivity n ortive frien s w ith w ho she s en s signific nt
izziness.
258 8: Pra c tic e Te s t 2

ou nts of ti e. She still c rries on her vi hobby (C) 25%


of g r ening, lthough t slow er ce bec u se of (D) 50%
her w orsening rthritis. She ex resses w orries over (E) 75%
recent big ro in the v lu e of retire ent stocks
she h s. 66. A 24-ye r-ol gr u te stu ent resents to her
University He lth C re Syste w ith the chief
62. Which of the follow ing is the ost signific nt co l int of e ression. U on et ile ques-
f ctor itig ting her risk for su ici e? tioning she reve ls history of intense oo
(A) G r ening hobby sw ings since e rly olescence, w ith short
(B) N o rior su ici e tte ts (hours to ys) erio s of ys hori , ltern t-
ing w ith ti es w here she feels nor l. She
(C) R ce
its th t she is e sily ngere , occ sion lly
(D) Slow er ove ents secon ry to rthritis bre king ite s n su erfici lly cutting on her-
(E) Soci l su orts self w hen or bore . She h s been un ble to
kee boyfrien for ore th n sever l onths,
63. Which of the follow ing e ic l con itions but st tes th t she w ill frequently rtici te
w ou l be the ost i ort nt to ru le ou t s in one-night st n s bec use she often feels
otenti l c u se of e ression in this tient? lonely n e ty. She enies ersistent inso -
(A) H y ercholesterole i ni , nergi , or nhe oni , lthough gets e s-
ily istr cte w hile in lectures. While w hen
(B) Phos holi i et bolis isor ers
cutely u set she y feel ssively suici l,
(C) Thin b se ent e br ne ise ses she enies ny st or current l n. Which of
(D) Thyroi illness the follow ing tre t ents w oul be the ost
(E) Try to h n eficiency ro ri te to begin for this tient?
(A) Ari i r zole
Questions 64 and 65
(B) Cit lo r
A 20-ye r-ol college so ho ore is brou ght to the (C) Di lectic l beh vior l ther y (DBT)
e ergency e rt ent fter being fou n sse ou t
(D) Lithiu
in her b throo . She h been vo iting, w hich she
(E) V l roic ci
its now w s self-in u ce . H er history is not ble
for bre ku 4 onths go w ith her boyfrien of
67. A 24-ye r-ol n w ith history of schizo-
2 ye rs. H er w eight is in the 82n ercentile for her
hreni resents to the e ergency e rt-
height. She is enorrheic. There is no history of
ent. H e h reviou sly t ken h lo eri ol
binge e ting.
(H l ol), but now refuses to t ke it. Bec use of
this, you eci e to try flu hen zine (Prolixin),
64. Which of the follow ing o you ost ex ect to
w hich see s to itig te his sy to s. One
fin on her hysic l ex in tion?
w eek l ter, he le ves the hos it l. H e contin-
(A) C f u l it s ots ues the e ic tions you rescribe n on
(B) Dent l ec y follow -u visit you notice he oves slow ly
(C) P l ble s leen n h s festin ting g it. Block e of w hich
(D) Pectus exc v tu of the follow ing rece tors ost likely ccounts
for the bove si e effect?
(E) Pitting fingern ils
(A) B s l g ngli cetylcholine rece tors
65. Wh t is the likelihoo th t with tre t ent she (B) D 4 rece tors
will ke full recovery over the next 10 ye rs? (C) 5-hy roxytry t ine-2 (5-H T2)
(A) 5% (D) Mesoli bic D 2 rece tors
(B) 10% (E) N igrostri t l D 2 rece tors
Que s tions : 6274 259

68. A 16-ye r-ol boy beg n h ving re etitive eye (D) Psycho yn ic sychother y
blinking t the ge of 6. By ge 12, he beg n (E) Pu nish ent
blu rting stereoty e hr ses su ch s B lls!
n Shitty! s w ell s king kissing noises. 71. The chil un ergoes lengthy tri l of the rec-
Conco it nt w ith this, he h incre sing if- o en e ther y s bove, but his con ition
ficu lty w ith ying ttention in school n continues without signific nt i rove ent.
occ sion lly is l ye w il , isinhibite Which of the following woul be the ost
beh vior. Which of the follow ing w ou l be ro ri te h r cologic tre t ent of choice?
the ost effective tre t ent for the ri ry
(initi l) con ition? (A) C rb ze ine (Tegretol)
(B) Cloni ine (C t res)
(A) Cloni ine (C t res)
(C) Des o ressin (D v )
(B) Flu oxetine (Proz c)
(D) Di henhy r ine (Ben ryl)
(C) Methyl heni te (Rit lin)
(E) I i r ine (Tofr nil)
(D) Ris eri one (Ris er l)
(E) To ir te (To x) Questions 72 and 73

69. A 12-ye r-ol boy h s recently been rreste An 8-ye r-ol boy is referre by his e i trici n for
for setting fire to neighbor s b rn. H e h s fu rther ev lu tion. H is history is not ble for being
been in trou ble before for l ying w ith fire- o te t ge 6 onths. H is f ily history is sig-
w orks in the neighborhoo , n w hen he w s nific nt for biologic l other w ith severe lcohol
10 ye rs ol he w s su s en e for lighting use isor er. H e h s stru ggle w ith ll of his su b-
tches t his school. Which of the follow ing jects n h s been in s eci l e u c tion cl sses s
i gnoses w ou l ost likely be co orbi in resu lt. H e cts ore concrete th n his eers, n h s
this tient? few frien s. Of note, his intelligence qu otient (IQ)
h s recently been e su re t 60.
(A) Au tis s ectru isor er
(B) Con u ct isor er 72. Which of the follow ing w ou l be this chil s
(C) Intellectu l is bility ost ro ri te i gnosis?
(D) M jor e ressive isor er (A) Au tistic s ectru isor er
(E) Tic isor er (B) Mil intellectu l is bility
(C) Mo er te intellectu l is bility
Questions 70 and 71
(D) Severe intellectu l is bility
A 9-ye r-ol boy w ith history of n or l evel- (E) Profou n intellectu l is bility
o ent l ilestones h s been w etting his be for
the l st 6 on ths. H is ren ts re ort th t th e be 73. Given the bove tients history, w hich of the
w etting initi lly occu rre s or ic lly, bu t for the follow ing hysic l fin ings w ou l ost likely
l st 2 onths it h s been h enin g bou t every be ex ecte ?
oth er n ight. This h s c u se hi sign ific nt
ou nt of sh e n h s been li iting his soci l (A) Cleft l te
inter ctions s he is u n ble to go to slee overs or (B) Congenit l blin ness
w y to c . (C) H y erextensible joints
(D) Microce h ly
70. Which of the follow ing interventions w ou l (E) Pro inent j w
be the ost ro ri te?
(A) Beh vior l ther y 74. A 5-ye r-ol girl h s co e un er the c re of
(B) Cognitive ther y st te Chil Protection Services. H er other w s
know n to be using cr ck coc ine n IV heroin,
(C) Di lectic l beh vior l ther y
n su orting her h bit w ith rostitution.
260 8: Pra c tic e Te s t 2

The girls te or ry foster rents re orte been ski ing cl sses n not turning in ssign ents
to st te w orkers th t she w s in iscri in tely s he believes he w ill u to tic lly gr u te n
soci l w ith u lts bu t often oesnt e r to ss the b r bec use of his bilities n intelligence.
seek or res on to co forting. In her soci l
inter ctions w ith other 5-ye r-ol chil ren t 76. Which of the follow ing w ou l be the ost
her reschool, she is note to often st y by her- likely i gnosis for this tient?
self but is escribe s being in ro ri tely
(A) Bi ol r isor er, cu rrent e iso e
ngry w ith her eers. Which of the follow ing
e resse
w ou l be the ost ro ri te i gnosis in
(B) Bi ol r isor er, cu rrent e iso e nic
this c se?
(C) Cyclothy ic isor er
(A) Attention- eficit/ hy er ctivity isor er (D) Persistent e ressive isor er
(ADH D) ( ysthy i )
(B) Au tis s ectru isor er (E) Schizo ffective isor er, bi ol r ty e
(C) Con u ct isor er
(D) L ngu ge isor er 77. Which of the follow ing w ou l be the ost
(E) Re ctive tt ch ent isor er ro ri te h r cologic tre t ent for this
tient?
75. A 39-ye r-ol n is rreste fter being (A) Cit lo r (Celex )
fou n in e rt ent store, stu rb ting by
(B) Div l roex so iu (De kote)
ru bbing w o ns shoe long his enis. H e
relu ct ntly its th t he h s co itte this (C) Flu oxetine
ct on nu erou s occ sions lthou gh h s not (D) I i r ine
gotten c u ght or rreste . H e feels gre t e l (E) Lor ze (Ativ n)
of sh e n guilt, bu t he is u n ble to sto the
beh vior. Which of the follow ing is the ost Questions 78 and 79
likely i gnosis?
A 36-ye r-ol w o n resents w ith 7 w eeks of
(A) Fetishistic isor er e resse oo ost of the ti e ( lthou gh w s
(B) Frotteu ristic isor er h y for short erio w hen she w on $150 in the
(C) Pe o hilic isor er lottery), sense of ho elessness, n thou ghts of
(D) Tr nsvestic isor er su ici e for the l st onth. She lso re orts incre se
etite n h s been slee ing u to 16 hou rs y.
(E) Voyeu ristic isor er
On ent l st tus ex in tion, her oo n ffect
re ys horic ost of the ti e, but she l u ghs in
Questions 76 and 77
res onse to hu orou s st te ents.
A 27-ye r-ol l w stu ent resents to you s
referr l for the tre t ent of first e iso e of jor 78. Which of the follow ing i gnoses w ou l be
e ression, initi lly i gnose by his ri ry c re the ost ro ri te in this tient?
hysici n 8 w eeks go. On his ent l st tu s ex i- (A) Bi ol r I isor er, cu rrent e iso e
n tion, the tients s eech is extre ely r i , if- e resse
ficult to interru t, n he quickly ju s fro one
(B) Bi ol r II isor er, cu rrent e iso e
to ic to nother. H e s ys th t his oo h s i rove
e resse
gre tly since he l st s w his ri ry c re octor: In
f ct, I on to of the w orl . The tient re orts 3 to (C) M jor e ressive isor er, w ith ty ic l
4 hou rs of slee er night for the l st w eek, not th t fe tu res
I nee th t uch slee nyw y. The tient h s (D) M jor e ressive isor er, w ith
been s en ing ll of his oney buying first e ition el ncholic fe tu res
textbooks n re rr nging the fu rniture in the l w (E) Persistent e ressive isor er
school inistr tion buil ing. H ow ever, he h s ( ysthy i )
Que s tions : 7582 261

79. Which of the follow ing cl sses of e ic tions the hos it l, n she h s been telling her f -
w ou l be the ost effective in this tient? ily bout her concerns reg r ing the govern-
ent controlling her for the l st 2 w eeks. The
(A) Anticonvu ls nts
tients st sychi tric history is signific nt
(B) Lithiu for three rior hos it liz tions over the l st
(C) Mono ine oxi se inhibitors (MAOIs) 20 ye rs. At the ti e of her first ission,
(D) Tricyclic nti e ress nts (TCAs) she resente w ith 3-w eek history of nic
(E) Ty ic l (first-gener tion) nti sychotics sy to s n sychosis. During her secon
hos it liz tion, she w s suici l n note to
80. A 42-ye r-ol ccount nt resents for long- be e resse w ith sychotic sy to s. During
ter sychother y tre t ent. In the first ses- her thir ission, she w s g in e resse
sion, you fin th t he h s been ivorce three w ith ro inent isorg nize beh vior. Her
ti es. The oor qu lity of his ro ntic n ent l st tus ex in tion u on resent tion
other rel tionshi s re the focus of his concerns. to the hos it l reve ls looseness of ssoci tion
Over the course of onths, you fin out th t he n elusion l sy to s w ithout ro inent
is erfectionist n inflexible w ithin rel tion- oo co onent. Which of the follow ing is
shi s. He ty ic lly i e lizes his rtner, then the ost ro ri te i gnosis for this tient?
ev lues her, before en ing the rel tionshi (A) Bi ol r isor er, w ith sychotic fe tu res
su enly. At ti es, he loses sense of self in
(B) M jor e ressive isor er (MDD) w ith
rel tionshi s, not know ing w ho he is or w h t
sychotic fe tu res
he w nts. During this ti e, he relies on others
to vise hi on w h t he shoul o. H ow ever, (C) Schizo ffective isor er
if he t kes this vice n his l ns f il, he uses (D) Schizo hreni
this to ev lue the one w ho vise hi . The (E) Uns ecifie sychotic isor er
tient kes it very cle r th t he sought your
tre t ent s sychother ist bec use Ill Questions 82 and 83
only cce t the best in the city. The tient lso
The e ic l te on n in tient service c lls you
re orts to you th t he h s so e unique bilities
to hel the w ith 72-ye r-ol fe le tient w ho
of erce tion th t llow hi to etect w h t or
w s itte for congestive he rt f ilu re. They
w ho w ill rovi e the best ossible outco e for
re ort th t t night she beco es confu se , think-
hi . For ex le, he icke you s ther ist
ing th t she is in hotel. She kes u nre son ble
not only bec use of your re ut tion, but lso
e n s of the nu rsing st ff n beco es isru -
bec use of your office hone nu ber, w hich
tive on the w r , interfering in the nu rses w ork.
h s igits in scen ing v lue or er. Which of
She h s the ost ifficu lty fro 10 pm u ntil bou t
the following i gnoses is the ost ro ri te
2 a m, w hen she u su lly f lls slee . In the orning,
for this tient?
she is better oriente n u ch ore coo er tive. A
(A) Bor erline erson lity isor er review of her e ic tions reve ls th t she is t king
(B) N rcissistic erson lity isor er ci eti ine 400 g PO bi , fu rose i e 40 g PO q ,
(C) Obsessive erson lity isor er tenolol 50 g PO q , igoxin 0.125 g PO q , n
i henhy r ine 50 g PO qhs n 25 g PO qhs
(D) Other s ecifie erson lity isor er
(secon ose 3 hou rs ost first ose for inso ni ).
(E) Schizoty l erson lity isor er
82. Which of the follow ing e ic tions is ost
81. A 43-ye r-ol w o n is hos it lize uring likely to c u se this tients confu sion?
sychotic e iso e in w hich she h the elu-
sion th t she is being controlle w ith co - (A) Atenolol
uter rogr evelo e by the De rt ent (B) Ci eti ine
of Ho el n Security. Accor ing to her f ily, (C) Digoxin
she sto e t king her regi en of queti ine (D) Di henhy r ine
n lithiu 4 w eeks before this ission to (E) Fu rose i e
262 8: Pra c tic e Te s t 2

83. Which of the follow ing neu rotr ns itter sys- bu t she re ins sychotic, w ith u itory
te s is ost irectly i lic te in the etiol- h llu cin tions telling her to kill herself.
ogy of her confu sion?
(A) Acetylcholine 87. A 53-ye r-ol rrie le with history of
(B) Do ine jor e ressive isor er n oor co li nce
ue to signific nt sexu l ysfunction now res-
(C) G - inobu tyric ci (GABA)
ents with recurrence of e ressive sy to s
(D) Serotonin with rke hy erso ni n low energy.
(E) N ore ine hrine
88. A 22-ye r-ol fe le gr u te stu ent w ith
D IRECTION S (Questions 84 through 89): The fol-
bi ol r isor er beco es regn nt.
low ing question is preceded by a list of lettered
options. For each of the questions in this section
match the clinical vignette with the most appropriate 89. A 37-ye r-ol n w ith history of nic
medication to prescribe. e iso es now h s 3 w eeks of e ression, w ith
inso ni , oor etite, w eight loss, low
M tch the clinic l vignette w ith the ost ro ri te energy, n ecre se concentr tion.
e ic tion to rescribe.
DIRECTIONS (Questions 90 through 112): For each
(A) Ari i r zole
of the multiple-choice questions in this section select
(B) Bu ro ion
the lettered answer that is the one best response in
(C) Chlor ro zine each case.
(D) Cloz ine
(E) H lo eri ol 90. A clinic l rese rch sychi trist t u niversity
(F) L otrigine A h s been running ou ble-blin l cebo-
(G) Lithiu controlle tri l of new nti e ress nt.
The sychi trist conclu es fter st tistic l
(H ) N ortri tyline
n lysis of the t th t there is no better
(I) P roxetine effect of the ru g co re to l cebo. The
(J) Ris eri one sychi trists colle gu es t five se r te
(K) Ris eri one Const institu tions (u niversities B, C, D, E, n F) h ve
(L) Sertr line ru n i entic l stu ies n eter ine th t there
(M) V l roic ci is tre t ent effect of the ru g co re
to l cebo. Given this infor tion, one y
(N ) Venl f xine
conclu e th t the rese rch tri l t u niversity A
resu lte in w hich of the follow ing?
84. A 32-ye r-ol n with schizo ffective isor er
h s goo benefit with nti sychotic e ic tions (A) H igh v ri nce
but is oorly co li nt ue to l ck of insight (B) Low - re ictive v lu e
n re uce cognition. (C) St n r error
(D) Ty e I error
85. A 48-ye r-ol ivorce w o n h s history (E) Ty e II error
of schizo hreni n is co li nt w ith her
e ic tion, bu t she resents to the e er- Questions 91 and 92
gency roo fter evelo ing severe su nbu rn
es ite only being ou tsi e for sever l hou rs. A 56-ye r-ol n w ith sever l ec e history
of schizo hreni h s been t king nu erou s if-
86. A 41-ye r-ol w o n w ith chronic schizo hre- ferent nti sychotics, lbeit regu l rly, for 27 ye rs.
ni h s been trie on sever l first- n secon - H is sy to s re re son bly w ell controlle , w ith
gener tion nti sychotics t ther eu tic oses, occ sion l, nonco n h llu cin tions n so e
Que s tions : 8396 263

r noi . H e h s been ble to w ork s volu nteer 94. A 29-ye r-ol rrie n resents to you r
n lives in grou ho e. Abou t 5 ye rs go, he out tient sychi tric clinic co l ining of
evelo e w rithing ove ents of his w rists n nxiety n e ression. H is soci l history
fingers th t is e r w hen he goes to slee . reve ls th t he is ju nior f cu lty e ber of
loc l u niversity, w orking bout 60 hours er
91. Which of the follow ing si e effects is ost w eek. H e enjoys his job but re orts stress ris-
consistent w ith this c se? ing fro his rit l rel tionshi bec use his
w ife ccuses hi of not s en ing qu lity
(A) Ak thisi
ti e w ith her n his being rrie to the
(B) Dystoni job. Accor ing to Erik Erikson, in w hich st ge
(C) N eu role tic lign nt syn ro e (N MS) of evelo ent oes this tient currently
(D) P rkinsonis h ve conflict?
(E) T r ive yskinesi (A) Au tono y versu s sh e n ou bt
(B) I entity versu s role iffu sion
92. Which of the follow ing n to ic stru ctu res
is ost likely i lic te in the etiology of his (C) In u stry versu s inferiority
ove ents? (D) Inti cy versu s isol tion
(E) Tru st versu s istru st
(A) B s l g ngli
(B) Cerebellu Questions 95 and 96
(C) Front l cortex
(D) Mi br in A 46-ye r-ol n resents w ith e iso es of bl ck-
ing out, w here he w ill not re e ber erio s l sting
(E) Motor cortex
for sever l inutes. During those ti es, he is tol by
others th t he h s n o ex ression on his f ce n
93. You re seeing 66-ye r-ol Vietn veter n
oesnt res on to his n e. In ition, he co -
t Veter ns A inistr tion hos it l. H e
l ins of ifficulty in rel tionshi s u e to ecre se
h s been hos it lize nu erous ti es w ith
sex rive n irrit bility. H e s en s gre t e l of
si il r resent tions. H e co l ins of ongo-
ti e w riting ow n his thoughts in his e oirs.
ing night res reg r ing fighting in Vietn ,
w ith frequ ent fl shb cks, intru sive thou ghts,
95. Which of the follow ing ition l sy to s
oor slee , n incre se st rtle. Other th n
w ou l be ost likely resent in this tient?
co ing in for his oint ents, he h s been
u n ble to go out in ublic bec u se he often (A) De ression
feels th t he is u n er tt ck, fe ring th t so e- (B) H y erreligiosity
one is going to ju ou t of the bu shes n (C) Obsessive-co u lsive beh viors
bush hi . H e h s been tre te w ith the
(D) Slee isor ers
follow ing e ic tions, ll w ith only o er-
te i rove ent in his sy to s: flu oxetine, (E) Urin ry incontinence
sertr line, roxetine, cit lo r , irt z -
ine, i i r ine, n itri tyline. H e h s 96. A slee - e rive electroence h logr is
been co li nt w ith venl f xine 375 g ily. erfor e on the tient s rt of his co -
H e h s been sober fro lcohol n heroin for lete w orku . Which of the follow ing lobes
20 ye rs. Which of the follow ing e ic tions of the br in w oul ost likely e onstr te
w ou l be the ost benefici l for this tient? bnor lities in this tient?

(A) Bu ro ion (A) Front l


(B) H lo eri ol (B) Cerebell r
(C) Lor ze (C) Occi it l
(D) Pr zosin (D) P riet l
(E) V l roic ci (E) Te or l
264 8: Pra c tic e Te s t 2

97. After being c lle to the e ergency e rt- (A) Beets


ent to ev lu te you ng w o n w ho over- (B) Li be ns
ose on b rbitu r tes, you fin her in be w ith (C) Liver
her eyes close . She o ens her eyes briefly in
(D) Pe nu ts
res onse to in n e onstr tes flexion
fro in bu t kes no sou n s. Which of the (E) White w ine
follow ing best re resents her Gl sgow Co
Sc le score? 100. A 66-ye r-ol n w ith history of schizo hre-
ni h been int ine on thiori zine (Mel-
(A) 7 l ril) 700 g q for ny ye rs, fter nu erous
(B) 6 unsuccessful tri ls w ith v rious nti sychotics.
(C) 5 Over the l st 8 onths, he h s been co l in-
(D) 4 ing of he ring erog tory voices, n his ose
h s been gr u lly incre se to 1,000 g q .
(E) 3
H e h s no signs of t r ive yskinesi . Which
of the follow ing si e effects w oul be the ost
Questions 98 and 99
concerning t this ose?
You h ve been tre ting 34-ye r-ol n for jor (A) Consti tion
e ression w ith serotonin-s ecific reu t ke inhibi-
(B) Dry eyes
tor (SSRI) for the l st 2 onths. H e w s initi lly
st rte on flu oxetine 20 g/ for the first onth, (C) N e hrogenic i betes insi i u s
n then you r ise the ose to 40 g/ for the l st (D) Pig ent ry retino thy
onth. While he h s toler te the e ic tion w ith- (E) Urin ry retention
out signific nt si e effects, his e ressive sy to s
h ve re ine refr ctory es ite the incre se in os- 101. A 24-ye r-ol n w ithou t rior sychi tric
ge. You h ve eci e to sw itch his e ic tion to history is itte to the sychi tric w r
henelzine (N r il), ono ine oxi se inhibitor w ith isorg niz tion, r noi , git tion,
(MAOI), in ho es of eliciting better nti e ress nt n co n u itory h llu cin tions tell-
res onse. ing hi to kill hi self. H e is given tw o oses
of h lo eri ol 5 g or lly. A roxi tely
98. Which of the follow ing w ou l be the best 8 hou rs l ter, he evelo s torticollis. Which of
str tegy in king the tr nsition fro the the follow ing w ou l be the ost ro ri te
SSRI to the MAOI? tre t ent?
(A) Begin t ering the ose of the SSRI (A) Acet ino hen
w hile incre sing the ose of the MAOI (B) Benztro ine
si ult neou sly. (C) Cyclobenz rine
(B) St rt the MAOI u ntil ther eu tic levels (D) Ibu rofen
h ve been re che , then t er the SSRI.
(E) Pro r nolol
(C) Sto the SSRI bru tly n begin
the MAOI t n equi otent ose the 102. A 28-ye r-ol w o n w ith history of bi o-
follow ing y. l r isor er is itte to the e ic l service
(D) Sto the SSRI bru tly n i e i tely bec u se of w e kness, ent l st tu s ch nges,
begin the MAOI. n seru so iu of 154 ol/ L. Accor -
(E) T er the SSRI n 5 w eeks fter the ing to her boyfrien , for the l st 2 w eeks she
l st ose begin the MAOI. h s been rinking lo s n lo s of w ter
bu t co l ining of being thirsty often, n
99. After he is sw itche to the henelzine, w hich lso frequ ently u rin ting. Which of the follow -
of the follow ing foo s/ rink shoul the ing e ic tions w ou l be ost likely res on-
tient ost voi ? sible for this tients resent tion?
Que s tions : 97107 265

(A) C rb ze ine enies ny suici l i e tion. H is st e ic l his-


(B) H lo eri ol tory is signific nt for i betes ellitu s, tw o yo-
(C) Lithiu c rbon te c r i l inf rctions, tri l fibrill tion, n coron ry
rtery by ss gr ft. The tients w ife w ishes hi to
(D) Qu eti ine
st rt e ic tion.
(E) V l roic ci
105. Which of the follow ing nti e ress nts shou l
Questions 103 and 104 be ost voi e in this rticu l r tient?
A 58-ye r-ol n w ith long history of o i te u se (A) Bu ro ion
isor er h s been u sing intr venou s heroin ily,
(B) I i r ine
roxi tely $5060 er y. H e h s not been
(C) Mirt z ine
ble to int in ny signific nt sobriety es ite
nu erous in tient n resi enti l reh bilit tion (D) Sertr line
rogr s. H e l st u se heroin 12 hou rs go n h s (E) Venl f xine
su bsequ ently been itte to the in tient sychi-
tric w r for etoxific tion. 106. Which of the follow ing nti e ress nts w ou l
ost likely resu lt in cognitive ecline in this
103. Which of the follow ing e ic tions w ou l be tient?
the ost ro ri te to tre t his w ith r w l (A) Bu ro ion
sy to s?
(B) Cit lo r
(A) A nt ine (Sy etrel) (C) Flu oxetine
(B) Cloni ine (D) P roxetine
(C) H lo eri ol (E) Venl f xine
(D) Lor ze
(E) N loxone 107. A 35-ye r-ol w o n w ith bi ol r isor er
n frequ ent hos it liz tions resents to the
104. After the tient co letes his etoxific tion e ergency e rt ent fter t king n over-
on the u nit, he continu es to be otiv te to ose of lithiu in su ici e tte t. Which
bst in fro heroin u se. H e is referre to n of the follow ing signs w ou l be ost likely to
out tient ictions rogr , bu t w ishes occur?
to t ke e ic tion to ecre se his heroin (A) Ab o in l in
u se fter isch rge. Which of the follow ing
(B) Acu te ystoni
e ic tions w ou l be the ost ro ri te to
rescribe? (C) Leg in
(D) P r noi elu sions
(A) D ntrolene
(E) P resthesi s
(B) Disulfir
(C) Flu zenil Questions 108 and 109
(D) Meth one
(E) Sertr line A 67-ye r-ol w o n resents to you r office w ith
long history of jor e ression. She h s trie
Questions 105 and 106 nu ber of ifferent nti e ress nt e ic tions
in the st bu t h s not h re ission w ith ny
A 75-ye r-ol n w ith jor neu rocognitive is- of the . Currently, she is on lithiu , venl f xine
or er u e to Alzhei er ise se resents w ith his (Effexor), nortri tyline (P elor), lor ze , ris-
w ife to you r clinic for ev lu tion. She is concerne eri one, n benztro ine. She is st rting to believe
bec u se he h s been ow n recently, co l ining th t her next- oor neighbors re ou t to h r her.
of e ression. H is slee h s been w orse, n his You eci e to begin tri l of electroconvulsive ther-
etite n energy h ve been oor. The tient y (ECT).
266 8: Pra c tic e Te s t 2

108. Which of the follow ing ini u ti es shou l 112. A 45-ye r-ol w o n w ith rior history of
the seizure l st for o ti l effectiveness in this jor e ression is itte to the sychi-
tient? tric in tient w r . She h s been severely
e resse , w ith oor slee , ecre se e-
(A) 10 secon s
tite, low energy, n su ici l i e tion. She
(B) 15 secon s now believes th t her neighbors re cons ir-
(C) 60 secon s ing to u r er her, n occ sion lly c n he r
(D) 120 secon s ns voice th t tells her she shou l ie.
(E) 180 secon s After s e king w ith the neighbors, you fin
ou t th t they brou ght her to the hos it l. They
109. Which of the follow ing verse effects w ou l h ve been goo frien s for 20 ye rs n h ve
be the ost likely u ring the cou rse of ECT? notice the tient h s not been le ving the
hou se uch, h s t ken to looking out of her
(A) Br in ge w in ow s w ith su s iciou s gl nces, n h s not
(B) C t toni cle ne her hou se in the l st 2 onths. Which
(C) Fr ctu res of the follow ing e ic tion co bin tions
(D) Me ory i ir ent w ou l be the ost ro ri te tre t ent for
(E) Vo iting this tient?
(A) Flu oxetine n ris eri one
Questions 110 and 111
(B) Lor ze n cloz ine
(C) P roxetine n thyroi u g ent tion
A 26-ye r-ol n w ithou t rior sychi tric history
resents to sychi tric e ergency e rt ent (D) Sertr line n lithiu
w ith r noi , visu l h llu cin tions, feelings of (E) Sertr line n lor ze
u nre lity, e erson liz tion, n extre e git tion.
A u rine toxicology screen is ositive for hencycli- Questions 113 through 116
ine (PCP).
M tch the clinic l vignette w ith the ost likely
su bst nce-in uce isor er. E ch lettere o tion
110. Which of the follow ing w ou l be the best
y be u se once, u lti le ti es, or not t ll.
i e i te tre t ent for the git tion in this
tient? (A) Alcohol intoxic tion
(A) Chlor ro zine (B) Alcohol w ith r w l
(B) Flu oxetine (C) C nn bis intoxic tion
(C) Lor ze (D) Coc ine intoxic tion
(D) Tr zo one (E) Coc ine w ith r w l
(E) Trihexy heni yl (Art ne) (F) H eroin intoxic tion
(G) H eroin w ith r w l
111. Which of the follow ing tre t ents w ou l be (H ) Inh l nt intoxic tion
the ost ro ri te if this tient w ere not (I) Meth het ine intoxic tion
git te ? (J) N icotine w ith r w l
(A) Aci ific tion of the urine (K) N itrou s oxi e intoxic tion
(B) Cheese (L) PCP intoxic tion
(C) Su ortive c re (M) Psilocybin w ith r w l
(D) Vit in B12 (N ) 3,4-Methylene ioxy eth het ine
(E) Vit in E (MDMA) (Ecst sy) intoxic tion
Que s tions : 108118 267

113. A 23-ye r-ol n is brou ght to the e ergency lettere o tion y be u se once, u lti le ti es,
e rt ent by the olice. H e show s git tion, or not t ll.
vertic l nyst g us, n n lgesi .
(A) Benztro ine
(B) Cloz ine
114. A 62-ye r-ol ho eless n itte to sy-
(C) Desi r ine (N or r in)
chi tric unit begins h ving elev te vit l signs,
visu l h llucin tions, i horesis, tre or, n (D) Flu oxetine
seizu res. (E) H lo eri ol
(F) Lithiu
115. A 43-ye r-ol w o n resents to the sychi- (G) Lor ze
tric e ergency e rt ent co l ining of (H ) Ol nz ine
e ression n suici lity. She is observe to be (I) Phenelzine
f tigue , irrit ble, n with ys horic ffect. (J) V l roic ci
(K) Venl f xine
116. A 30-ye r-ol u sici n resents in the
ou t tient clinic. H e co l ins of fever n 117. A 34-ye r-ol n w ith schizo hreni
chills, runny nose, n use , bo y ches, i rrhe , resents w ith fever n chills. H e is foun to
n b o in l cr s for the l st 24 hours. be b ctere ic n h s WBC cou nt of 900/ L.

Questions 117 and 118 118. A 41-ye r-ol w o n on e ic tions for her
bi ol r isor er resents w ith f tigu e, w eight
For the follow ing scen rios, choose the e ic tion g in, col intoler nce, consti tion, n
ost likely ssoci te w ith the si e effects. E ch ecre se concentr tion.
Ans we rs a nd Expla na tions

1. (C) Mirt z ine is n ty ic l nti e ress nt there re ny effic ciou s nti e ress nts
w ith nor renergic n serotonergic ctivity. to choose fro , serotonin-s ecific reu t ke
Its ech nis of ction inclu es block e of inhibitors (SSRIs) su ch s cit lo r re con-
the 2- renergic uto- n heterorece tors, si ere first-line tre t ent. Ari i r zole is
s w ell s block e of the 5-H T2 n 5-H T3 secon -gener tion (or ty ic l) nti sychotic.
serotonin rece tors n hist inergic (H 1) It w ou l be ro ri te to u se ri i r zole
rece tors. This unique co bin tion results in if this tient h sychotic sy to s (su ch
incre sing serotonin, n , in higher oses, nor- s h llu cin tions or elu sions) or tre t ent
e ine hrine; how ever, irt z ine oesnt cre- refr ctory e ression (w hich is not the c se in
te sexu l ysfunction n , es eci lly t low er this tient). Methyl heni te is sti ul nt
oses, c uses se tion n incre se etite. u se for ttention- eficit hy er ctivity isor-
er, lthou gh it y occ sion lly be u se s
2. (F) Venl f xine is n nti e ress nt th t n ju nct in the tre t ent of severe e res-
inhibits the reu t ke of both serotonin, n , sions or in cert in c ses w here r i onset is
in higher oses, nore ine hrine. Bec u se of necess ry (such s ehy r te tient). N or-
this, it y c u se incre se bloo ressure, tri tyline is tricyclic nti e ress nt (TCA).
rticu l rly in oses bove 300 g ily. While TCAs re ju st s effic cious s SSRIs,
their si e effects n leth lity in over ose
3. (D ) Although ost nti e ress nts c n c use resu lt in their not being u se s first-line
w ith r w l syn ro e (ch r cterize s flu- e ic tions for jor e ression. Tr nylcy-
like sy to s) if bru tly iscontinue , those ro ine is ono ine oxi se inhibitor
w ith shorter h lf-lives, es eci lly roxetine, (MAOI), rticu l rly u sefu l in MDD w ith
re ore likely to o so. ty ic l fe tu res (incre se etite, w eight
g in, n oo re ctivity), lthou gh the ru g
4. (B) Flu oxetine h s signific ntly longer h lf- ru g inter ctions n iet ry restrictions
life th n the other liste nti e ress nts. This of MAOIs li it their use.
tr nsl tes into few er w ith r w l sy to s
if su enly sto e n y lso be n v n- 7. (A) This tient h s escribe nic tt ck.
t ge in c ses of occ sion l oor co li nce. P nic tt cks re ch r cterize by su en
onset of intense fe r, re , or nxiety, n
5. (E) Most of the secon -gener tion nti sy- cco nie by v riety of hysic l sy -
chotics c u se w eight g in, inclu ing cloz - to s. Althou gh virtu lly ny org n syste
ine, ol nz ine, qu eti ine, n ris eri one. c n see to be the source of istress in nic
H ow ever, zi r si one, lu r si one, n ri i - isor er, co on sy to s inclu e i ho-
r zole re w eight neu tr l. resis, l it tions, shortness of bre th, tre u -
lou sness, flu shing, chest in, n izziness.
6. (B) This tient is su ffering fro jor e res- Reg r less of the clinic l setting or sychi tric
sive isor er (MDD), single e iso e. While history, tients resenting w ith sy to s

268
Answe rs : 112 269

suggestive of nic isor er ust be thor- BPRS is short sc le evelo e to ssess sy-
ou ghly w orke u to ru le ou t hysiologic chi tric sy to tology ri rily in tients
thology s the sou rce of the sy to s. w ith sychosis or severe i ir ent. It is
Re ssu r nce y be re tu re u ntil other ore often use in rese rch settings. The PH Q
e ic l c u ses re ru le ou t. If no other e i- is self- inistere screening tool focu se
c l con itions re c u sing the sy to s n on co on ent l roble s th t often res-
the nic tt cks continu e, rescribing n SSRI ent to the ri ry c re hysici n (e.g., e res-
n / or referring her to sychi trist y be sive, nxiety, lcohol use, e ting, n so tic
ro ri te. Since nti e ress nts t ke sev- sy to isor ers). The PAN SS is st n r
er l w eeks to re ch fu ll benefit, benzo i ze- test to ssess tre t ent ou tco es in stu ies of
ines y be u se in the interi . Untre te , schizo hreni n other sychotic illnesses; it
nic isor er ten s to ru n chronic cou rse u tilizes three subsc les: ositive sy to s,
n c n c u se severe is bility. neg tive sy to s, n gener l sycho -
thology. Like the BPRS, it is ostly use for
8. (D ) N eologistic s eech, ositive sy to rese rch r ther th n clinic l r ctice.
of schizo hreni , consists of w or s eu
by the tient, h ving no est blishe e n- 11. (B) P tients w ith C gr s syn ro e su ffer
ing. Cl ng ssoci tions re w or s or hr ses fro the elusion th t so eone f ili r is
stru ng together bec u se of the soun s they n i entic l- e ring re l ce ent. A ok is
ke, not bec u se of the e ning they con- M l ysi n ter for issoci tive e iso e
vey. Echol li is sy to of c t toni , ch r cterize by erio of broo ing le -
w here the tient w ill re e t w h t is s i by ing to fit of violence follow e by ne-
so eone else, reg r less of the context, n si . Cot r syn ro e is nihilistic elu sion
often in re etitive nner. Flight of i e s is involving believing one is e , or th t the
the r i shifting of one i e to nother, n is bo y or org ns re ying. Cou v e syn ro e
ch r cteristic of ni . Wor s l is n illog- occurs w hen the husb n of regn nt
ic l, incoherent collection of w or s or hr ses. w o n h s si il r sy to s of regn ncy
n / or l bor. Koro is tr u tic fe r th t the
9. (E) This w o n suffers fro ersistent e res- enis is shrinking into the bo y c vity, seen in
sive isor er ( ysthy i ), low -gr e, chronic so e Asi n cu ltu res.
e ression. In ults, the sy to s ust be
resent for t le st 2 ye rs. Like jor e res- 12. (A) Bere ve ent is the sychologic l re c-
sive isor er (MDD), other th n subjective tion to loss, often the e th of signific nt
feeling of e resse oo , the key fe tures of other. It is not consi ere ent l isor er or
ysthy i re lter tions in etite n slee , thologic l re ction. While sy to s y
feelings of ho elessness, ifficulty concentr t- overl w ith MDD ( e ression, etite/
ing or king ecisions, low self-estee , n slee / energy ch nges), the intense sy -
low energy or f tigue. While ersistent e res- to s of e ression in bere ve ent re often
sive isor er y not res on s robustly to short-live n self-li iting, n there is little
e ic tions s MDD, n equ te ther eutic over ll fu nction l i ir ent. Occ sion l illu -
tri l (on xi u ose) shoul l st t le st sions or h llucin tions of the ece se re not
6 w eeks before eci ing th t it is f ilure. u nco on, bu t the in ivi u l ret ins insight
reg r ing these. Th t being s i , if the e res-
10. (A) The AIMS is st n r clinic l ex in - sive sy to s continue or w orsen, consi er-
tion n r ting sc le th t shoul be in- tion for MDD shou l be given. There is no
istere to tients receiving nti sychotics evi ence of rior or cu rrent nic e iso es to
(neu role tics) su ch s h lo eri ol. It is u sefu l ju stify the i gnosis of bi ol r isor er. Per-
for etecting the evelo ent of t r ive ys- sistent e ressive isor er ( ysthy i ) is
kinesi s w ell s tr cking ch nges over ti e. chronic, low -level e ression th t l sts t le st
The BDI is e ression screening tool. The 2 ye rs in u lts. There is lso no evi ence of
270 8: Pra c tic e Te s t 2

ri ry sychotic illness consistent w ith or not istressing to the tient, res ectively.
schizo ffective isor er. Delu sions re fixe f lse beliefs th t re in
es ite evi ence to the contr ry n re not
13. (C) Co ro r xi is co lex otor tic occ - cultu r lly s nctione . Moo congru ent n
sion lly seen in Tou rette isor er, ch r cter- oo incongru ent refer to elu sions or h llu -
ize by king obscene gestu res. Bru xis is cin tions in sychi tric isor ers th t re con-
the grin ing of the teeth. Ble h ros s is the sistent w ith or not consistent w ith the ri ry
s s of the eriorbit l u scles c using su s- oo istu rb nce, res ectively.
t ine or ex gger te blinking. Echo r xi is
lso co lex otor tic w here ove ents of 17. (C) G nser syn ro e w s thou ght to be ty e
nother re re e te . It is cl ssic sy to of issoci tive isor er, origin lly escribe
of c t toni . Torticollis is s s of the neck in risoners; how ever, it is now believe to be
u scles su ch s the sternoclei o stoi on consistent w ith lingering. It is ch r cterize
one or both si es, n c n be si e effect of by giving roxi te nsw ers. A ok n
nti sychotic e ic tions. iblokto re issoci tive tr nce isor ers th t
re escribe s istu rb nces in consciou s-
14. (C) Echol li refers to re e ting nothers ness n re in igenou s to rticu l r cu ltu res.
w or s or hr ses. Co rol li is so eti es Dere liz tion is n ex erience of u nre lity or
seen in Tou rettes, w here tient w ill blu rt et ch ent, often seen in nic tt cks n
ou t obscenities s rt of voc l tic. Dys rthri issoci tive isor ers. N eg tivis is ch r c-
refers to roble of w or rticu l tion, u su - terize by o osition to su ggestion or vice,
lly secon ry to cerebell r or otor control inclu ing irect resist nce to being ove ; it
bnor lities. P lil li is lso re etition of is co on sy to of c t toni .
w or s bu t is the re etition of ones ow n w or s
s if the erson gets stu ck on the s e w or 18. (A) Cou ntertr nsference refers to feelings the
or hr se. P r r xis is the ter for sli of tient elicits in the ther ist, versu s tr nsfer-
the tongu e or Freu i n sli , ch r cterize by ence, w hich refers to feelings the ther ist elic-
n error in s eech occu rring u e to the inter- its in the tient. Dis l ce ent is consi ere
ference of the u nconsciou s. neu rotic efense ech nis w here feeling
is irecte tow r nother in ivi u l w ho
15. (C) Kle to ni is n i u lse control isor- is less thre tening th n the origin l object of
er w here n in ivi u l ste ls objects not out th t feeling. Projection is ri itive efense
of nee or onet ry v lu e. Antisoci l er- ech nis ch r cterize by re cting to u n c-
son lity isor er is ch r cterize by l ck of ce t ble feelings or i u lses by l cing the
e thy n chronic, l tive ttern in nother erson. Re ction for tion is
of viol ting the rights of others; in this c se, nother neurotic efense ech nis w here
the tient is istu rbe by the ste ling n n u n cce t ble i u lse is tr nsfor e into
show s no other evi ence of ntisoci l beh v- its o osite.
ior. Inter ittent ex losive isor er is lso n
i u lse control isor er w here ggressive 19. (C) This tient likely h s Wilson ise se, n
i u lses resu lt in ss u lt or the estru ction utoso l recessive error of co er et bo-
of ro erty. Pyro ni is n i u lse control lis . Wilson ise se c n resent with ulti le
isor er of eliber te, recurrent fire setting. hysic l s well s sychi tric sy to s n
Trichotillo ni (h ir- u lling isor er) is signs, inclu ing e ression, irrit bility, l bility,
isor er rel te to obsessive-co ulsive is- n erson lity ch nges. K yserFleischer rings
or er, w here tients co u lsively u ll their re one such hysic l sign: gol en brown or
h ir resu lting in reci ble h ir loss. gr y-green rings of ig ent t the corne l li -
bus. Arcus senilis is light gr y ring beginning
16. (B) Ego- ystonic n ego-syntonic refer to su eriorly n exten ing to the li bus. It c n
w hether thou ghts or beh viors re istressing rese ble corne l rcus, which is ssoci te
Answe rs : 1324 271

with hy ercholesterole i , but in n el erly others c n control the tients thoughts or


tient it is often nor l fin ing. Brushfiel insert thoughts into tients in , res ec-
s ots re lighter-colore re s in the outer thir tively. Tr nsference, in strict ter s, is the reex-
of the iris th t c n be ssoci te with Down syn- eriencing of st ex eriences with the ther ist
ro e, but c n lso be nor l v ri nt. Sub- in the setting of sychother y. In gener l, this
conjunctiv l he orrh ge is the fin ing of bloo ter h s co e to e n the tr nsferring of e o-
in the re s of the eye surroun ing the iris. Also tions n feelings th t one h s fro ones st
ssoci te with hy ercholesterole i is x nthe- to the hysici n or c re rovi er.
l s , r ise yellow re s roun the eyeli s.
23. (A) This tient is cu rrently ex eriencing
20. (C) H e ib llis us is n u ncontrolle sw ing- nic e iso e. While this tient likely h
ing of n extre ity ssoci te w ith Wilson is- rior jor e ressive e iso e (MDE), w ith
e se. It is u su lly su en, n once initi te one nic e iso e he eets criteri for bi o-
it c nnot be controlle . Athetoi ove ents, l r I isor er. Bi ol r II isor er consists of
or thetosis, re slow , sn ke-like ove ents n MDE n hy o nic e iso e. Criteri
of the fingers n h n s. Choreifor ove- for hy o ni re the s e s for ni ,
ents re lso involu nt ry, irregu l r, n bu t the severity is less (i.e., the tient is not
jerky bu t l ck the b llistic-like n tu re of he i- sychotic, oes not nee hos it liz tion, or
b llis u s. Choreo thetoi ove ents re oes not h ve signific nt soci l or occu -
cl ssic lly seen in t r ive yskinesi , long- tion l i ir ent). This tient h s sychotic
ter si e effect of nti sychotic e ic tions. sy to s (gr n iose elusions) n sig-
Myoclonus is su en u scle s s , n nific nt i ir ent (re o eling hou se, not
yotoni is rolonge u scle contr ction. going to w ork). Cyclothy ic isor er consists
of inor e ressive e iso es (i.e., th t o not
21. (C) M gic l thinking n i e s of reference eet criteri for MDE) ltern ting w ith hy o-
re tw o ty es of elusions th t c n be fou n nic e iso es. While this tient h s his-
in sychotic isor ers, su ch s schizo hre- tory of n MDE, he is currently su ffering fro
ni . M gic l thinking is the belief th t ones nic e iso e n not e ressive e iso e.
thoughts c n control outsi e events. I e s of While he is sychotic ( elu sion l), his rior
reference re beliefs th t other in ivi u ls fu nctioning n signific nt ffective sy -
(e.g., govern ent, entert iners, the e i ) to s ke ri ry sychotic isor er su ch
re referring to or t lking bou t the erson s schizo hreni u ch less likely.
ex eriencing the elu sion. Dis l ce ent, ro-
jection, n re ction for tion re ll ty es 24. (E) First-line h r cother y for tre ting
of efense ech nis s. Dis l ce ent is the nic e iso e of bi ol r isor er is either
tr nsferring of feeling tow r n object th t oo st bilizer (such s v l roic ci or
is less thre tening, su ch s the f ily et or lithiu ) or secon -gener tion ( ty ic l)
ones s ou se or chil ren. Projection is the f lse nti sychotic (e.g., ris eri one, queti ine,
ttribution of ones ow n u n cce t ble feelings ol nz ine). Anti e ress nts such s itri -
to nother. Re ction for tion is the for - tyline ( tricyclic) or flu oxetine ( serotonin-
tion of th ou gh ts th t re o osite to nxiety- s ecific reu t ke inhibitor) shou l be voi e ,
rovoking feelings. s they y w orsen the nic e iso e. While
cloz ine is secon -gener tion nti sychotic,
22. (B) Thou ght bro c sting is the elu sion it is not first-line e ic tion given the risk
th t ones thou ghts c n be he r by others. of gr nu locytosis n nee for rou tine bloo
It is often seen in schizo hreni . Echol li is onitoring. Although first-gener tion (ty i-
the re etition of nothers w or s or hr ses, c l) nti sychotics such s h lo eri ol c n be
so eti es seen in c t toni . Thou ght con- use , they re not consi ere to be s effic -
trol n thou ght insertion lso c n be seen ciou s in cute ni s oo st bilizers or the
in schizo hreni , n re both elu sions th t secon -gener tion nti sychotics.
272 8: Pra c tic e Te s t 2

25. (A) This tient is su ffering fro benzo i z- on the resence of ll of the follow ing: (1)
e ine w ith r w l. The ost effic cious tre t- n u n erst n ing of the n tu re of the w ill;
ent for benzo i ze ine w ith r w l is to use (2) know le ge of ones ssets; (3) know le ge
benzo i ze ine w ith long h lf-life (e.g., of n tu r l heirs; (4) bsence of cu te sycho-
i ze ) n to slow ly t er it over ti e. sis (i.e., elusions) w hich ight co ro ise
Flu zenil is nons ecific co etitive nt g- r tion l ecision king; n (5) free o
onist of the benzo i ze ine rece tor, u se in fro u n u e influ ence or coercion. The v li -
c ses of benzo i ze ine over ose. Giving it to ity of the w ill y be u n er ine by e -
this tient w oul severely w orsen her sy - onstr ting th t the in ivi u l f ile to eet
to s of w ith r w l n ut her t risk of sei- ny of the bove criteri . Actu s reu s refers to
zu res. The other choices re not consi ere to the volu nt ry ct of co itting cri e n
be s effective s benzo i ze ines in tre ting is n ele ent u se in eter ining cri in l
w ith r w l. res onsibility. A history of ent l illness is
not su fficient to inv li te ones test ent ry
26. (A) A conversion isor er (fu nction l neu ro- c city. The resence of conserv tor of er-
logic l sy to isor er) is often r tic son n history of ent l illness re i or-
set of neu rologic sy to s th t likely result t nt only insof r s they be r u on ny of the
fro u nconsciou s conflicts. F ctitiou s isor- bove note f ctors. The resence of ju ge t
ers involve the consciou s f bric tion of n the signing of w ill is not requ ire .
illness in or er to resent oneself s ill (i.e.,
ssu e the sick role n be t ken c re of). Ill- 29. (A) Co etency is leg l eter in tion e
ness nxiety isor er (hy ochon ri sis) is the by the cou rt. C city, w hich is situ tion- n
reoccu tion w ith fe rs of h ving seriou s ti e-s ecific, is frequ ent sychi tric consu lt
ise se b se on isinter ret tion of bo ily qu estion. Ele ents of c city inclu e the fol-
sens tions. M lingering is the conscious f bri- low ing bilities: to co u nic te ones w ishes,
c tion of n illness in or er to obt in secon ry to u n erst n ones illness, to u n erst n the
g in (e.g., voi ing ilit ry u ty, w ork or j il, risks n benefits of tre t ent n nontre t-
or obt ining is bility), not i entifie in this ent, n to re son bou t ones o tions. The
vignette. So tic sy to isor er is ch r c- ge of tient h s no be ring on the eter i-
terize by one or ore so tic sy to s th t n tion of c city. Whether the tient grees
re istressing or resu lt in isru tion of life; it w ith the tre t ent te reco en tions is
is likely ore rev lent in w o en. not the issu e; how ever, qu estions of c c-
ity o not u su lly co e u u nless the tient
27. (E) L belle in iffrence is the cl ssic l ck of is grees w ith the tre t ent te . The res-
concern show n tow r eficit or loss of func- ence of sychotic illness oes not, in n
tion so eti es seen in conversion isor er of itself, eter ine c city. A tient being
(fu nction l neu rologic l sy to isor er). too isorg nize to co unic te equ tely
Dj enten u is the feeling th t one is he ring or h ving elu sions involving the tre t ent
w h t one h s he r before. It is u su lly ssoci- ro ose y in ic te l ck of c city. A l n-
te w ith nxiety st tes or f tigu e. Dj vu is gu ge b rrier w ou l not e onstr te l ck of
si il r ex erience but refers to the sens tion c city, so long s the tient int ins the
th t so ething h s been seen before. J is bility to co u nic te his or her references
vu is the o osite of j vu in th t it refers to n re sons.
so ething th t shou l be f ili r but see s
qu ite unf ili r. Folie eux is sh re elu- 30. (C) Confi enti lity is n integr l rt of the
sion rou se in one erson by the influ ence of hysici n tient rel tionshi n is es e-
nother. ci lly i ort nt in est blishing tru st in
sychother eutic rel tionshi . The hysi-
28. (C) Test ent ry c city (the level of co - ci n tient rivilege shou l be bre che
etence requ ire to ke v li w ill) is b se only in s ecific inst nces, such s the tients
Answe rs : 2535 273

w iving of the rivilege (e.g., tient initi- (fu nction l neurologic l sy to isor er)
tes litig tion, consent to rele se infor tion escribes the ro u ction of neu rologic sy -
is obt ine , there is u ty to rotect, e er- to s thought to resu lt fro intr sychic
gency situ tion, or in c ses in w hich the cou rt nee s or conflicts. F ctitiou s isor er c n be
or ers rele se). In this c se, the infor tion ifferenti te fro lingering b se on the
shou l not be rele se unless the tient h s bsence of extern l incentives. In f ctitiou s
w ive his hysici n tient confi enti lity isor er, sy to s re intention lly feigne
rivilege. Lying to or isle ing the cou rt is to s tisfy n intr sychic incentive to ssu e
u nethic l n y su bject one to rosecu tion. the sick role. A jor neu rocognitive isor er
The re ining choices f il to int in confi- ( e enti ) is unlikely in this c se given the
enti lity. ge n resenting co l ints of the tient.
There is no evi ence to su ggest isorg niz -
31. (B) When tient h s sto e ying his or tion or sychotic sy to s consistent w ith
her bill, the hysici n shoul irectly ress schizo hreni .
the issu e w ith the tient. F ilu re to y ones
bills y reflect u n erlying issu es for w hich 33. (B) In ivi u ls w ith lingering consciou sly
the tient h s entere into sychother y feign sy to s for fin nci l incentives or to
or th t re rel te to sychi tric sy to s. voi w ork/ soci l oblig tions (i.e., secon ry
Cont cting collection gency before ress- g in). The consciou s ro u ction of sy to s
ing the issu e w ith the tient is in ro ri te. to ssu e the sick role is seen in f ctitiou s is-
If tient continu es to refu se y ent fter or er. Conversion isor er is the u nconsciou s
iscu ssing the issu e, the tient shou l be or involu nt ry ro uction of sy to s likely
notifie in w riting of the ou tst n ing b l nce, ue to intr sychic, or u nconsciou s, nee s
n referr l to collection gency ight be or conflicts. The unconscious ro u ction of
e. To reserve confi enti lity, c re shou l sy to s to ssu e the sick role or to obt in
be t ken to isclose the ini u infor - secon ry g in is not consistent w ith sychi-
tion nee e for collection (i.e., te of service tric illnesses.
n ch rge) w ithou t referring to i gnosis
or tre t ent ren ere . Asking the insu r nce 34. (B) In this inst nce, the r n o letter test,
co ny to y you fter check h s been w hich relies on concentr tion, coo er tion,
isbu rse to the tient is fu tile bec u se the n the bility to he r, is the test of choice. It
insu r nce co nys oblig tion tow r the consists of telling tient letter n then in
insu re h s been fu lfille . Disclosing the exis- onotone voice listing r n o string of
tence of rofession l rel tionshi w ith the letters. The tient res on s by r ising fin-
tient to his f ily is bre ch of confi enti- ger to in ic te w hen he or she he rs the key
lity. Ter in ting the ther eu tic rel tionshi letter. The other tests rely not only on ttention
before ex loring the yn ics behin the bu t lso on c lcu l tion bilities, n therefore
tients y ent elinqu ency ight h r e u c tion l level.
ny fu tu re ther eu tic lli nce.
35. (B) Sexu l bu se is esti te to occu r w ith n
32. (D ) M lingering is the intention l ro u ction inci ence of 1% of chil ren er ye r. Know n
of sy to s for secon ry g in (e.g., to voi cqu int nces (e.g., f thers, ste f thers, n
w ork, to ev e cri in l rosecu tion, or to g in le rel tives) re often the er etr tors. In
fin nci l rew r s). A i gnosis of linger- this vignette, the tient likely h s cqu ire
ing shou l be su s ecte in e ico-leg l c ses, N eisseria gonorrhoeae fro le er etr tor.
in in ivi u ls w ith ntisoci l erson lity is- Other hysic l fin ings y inclu e inju ries
or er, w hen sy to s re ou t of ro ortion to the genit li (i.e., hy en, v gin ) or the
to objective fin ings ( s in this c se), or w hen erineu . Psychi tric nifest tions of sexu l
tient in istress oes not coo er te w ith buse y inclu e nxiety, git tion, ggres-
ev lu tion or tre t ent. Conversion isor er sive or i u lsive beh vior, n exhibitionis .
274 8: Pra c tic e Te s t 2

When sexu l buse of chil is su s ecte , the benzo i ze ine nt gonist use in the e er-
first ste is to ensure the s fety of the chil gency tre t ent of over ose w ith benzo i-
n i e i tely notify the st tes Chil Pro- ze ines, w hich oes not c use u ill ry
tective Services. Re orting is n tory in constriction. Thi ine w oul be given in c ses
c ses of hysic l n sexu l chil bu se. The of Wernicke ence h lo thy, c u se ost
other choices f il to rotect the chil n o co only by chronic, he vy lcohol con-
not s tisfy st te- n te re orting st tu tes. su tion. It resents w ith sy to s of con-
fusion, t xi , n o hth l o legi .
36. (B) This c se re resents Wernicke ence h -
lo thy, con ition seen in chronic lcohol 39. (B) This w o n is show ing signs of jor
u se isor ers. The etiology is cute thi ine neu rocognitive isor er ( e enti ), likely
eficiency n it resents w ith the cl s- ue to Alzhei er ise se, the ost co on
sic tri of confu sion, t xi , n o hth l- c use of neu rocognitive isor ers ( e enti ).
o legi . Althou gh it is u su lly reversible The ri ry tre t ent o lity is to begin
w ith thi ine re l ce ent, it is i ort nt to n nticholinester se inhibitor, su ch s g l n-
inister the thi ine renter lly, rior to t ine. Me ntine, n N - ethyl d - s rt te
giving flu i s n glu cose. A inistr tion of (N MDA) rece tor nt gonist, is e ic tion
glu cose n flu i s first y rovoke w ors- th t y be e to the regi en ition-
ening of sy to s resu lting in er nent lly, bu t it is not s effective in il e en-
ge. The inistr tion of n loxone is ti s. Anti e ress nts (e.g., cit lo r ) n
u se for o i te w ith r w l. nti sychotic e ic tions (e.g., zi r si one)
y be u se for ssoci te e ression or gi-
37. (B) The tient is ex eriencing sy to s t tion/ sychosis, res ectively, bu t they re
consistent w ith severe lcohol w ith r w l, not ri ry tre t ents. Ginkgo bilob h s
n ely eliriu tre ens (DTs), ch r cterize not been show n to be effective for the e ory
by eliriu , confu sion, co b tiveness, n eficits in neu rocognitive isor ers.
elev te vit l signs. DTs c rry signific nt
ort lity if not tre te . The tre t ent of choice 40. (D ) Risk f ctors for co lete su ici e inclu e
is IM or IV benzo i ze ines. Although b r- (1) ge > 45, (2) le gen er, (3) never rrie
bitur tes y lso tre t lcohol w ith r w l, > w i ow e / se r te / ivorce > rrie ,
they h ve n rrow er ther eu tic in ex, w ith (4) w hite r ce > bl ck r ce, n (5) religiou s
higher risk of se tion n res ir tory e res- > not religiou s. Wo en tte t su ici e tw ice
sion. N either thi ine nor n nti sychotic s frequently s en; how ever, en re three
w ill tre t the un erlying lcohol w ith r w l, ti es ore likely to co lete su ici e th n
n giving hy r l zine y be ngerou s s re w o en.
it y sk the u tono ic sy to s of the
lcohol w ith r w l. 41. (A) Physici n- ssiste suici e involves the
octors f cilit ting ersons e th by rovi -
38. (D ) The tient is resenting w ith evi ence of ing the infor tion or equi ent necess ry to
over ose w ith o i tes, w hich is ch r cterize en his or her life. The AMAs Co e of Me ic l
by the tri of so nolence, res ir tory e res- Ethics strongly con e ns such ction s fun-
sion, n in oint u ils. O i te intoxic tion ent lly inco tible w ith the hysici ns
c n be r i ly reverse w ith IV inistr - role s he ler. R ther, the ethic l oblig tion
tion of n o i te nt gonist, su ch s n loxone. of the hysici n is to equ tely res on to
Ant bu se blocks cet l ehy e ehy roge- tients en -of-life issues. Choices (B) n (E)
n se n is given to lcoholics s n versive w oul both be ssisting the tient to co it
sti u lu s to voi lcohol consu tion. Ben- suici e n re u nethic l. Infor ing the f -
zo i ze ines w ou l be u se in c ses of severe ily w oul be bre ch of confi enti lity n
lcohol w ith r w l, w hich w oul e on- not ro ri te t this oint, n l cing the
str te elev te vit l signs. Flu zenil is tient in restr ints is not in ic te .
Answe rs : 3651 275

4249. [42 (A), 43 (L), 44 (C), 45 (K), 46 (H), 47 (E), 48 Injection into the left c roti rtery nesthe-
(F), 49 (J)] The Beck De ression Inventory (A) tizes the left si e of the br in, n those w ith
is 21-ite test w ith three res onses er ite left he is heric l ngu ge o in nce show
th t is n e sily use screening tool to ev lu te interru te s eech. The Ben er Gest lt Test
for e ression. Bec u se it w ou l be u nu su l (B) involves co ying figu res, w hich hel s to
for n in ivi u l so you ng to h ve neu ro- screen for evelo ent l isor ers or br in
cognitive isor er ( e enti ) n bec u se ge. The Boston Di gnostic A h si
ny of the nsw ers reflect l ck of interest, Ex in tion (D ) is series of tests given by
the Beck De ression Inventory, in conju nction n ex erience clinici n to ev lu te n ke
w ith the Folstein MMSE, y hel istingu ish tre t ent reco en tions for in ivi u -
e ression fro e enti . The WCST (L) ls w ith h si . The ReyOsterrieth figure
ssesses execu tive fu nctions of the br in su ch (G) is sensitive to eficits in co ying n l ck
s org niz tion l bilities, ent l flexibility, of ttention to et il in in ivi u ls w ith right-
n the bility to bstr ct n re son. These si e riet l lobe lesions. The Stroo Test
c cities re believe to be loc te in the (I) ssesses execu tive function s the tient
front l lobes. D ge to the front l lobes c n h s to e l w ith istr ctions n h s to su -
le to bnor lities on this test. The Blesse ress n incorrect re ly before rovi ing the
R ting Sc le (C) is tool th t sks frien s or correct one.
f ily of the tient to ssess the bility of the
tient to fu nction in his or her u su l environ- 50. (B) Physici n tient confi enti lity in the
ent. The WAIS-R (K) is n ssess ent of IQ tre t ent of inors u st be int ine
u se in ges 16 onw r ; it c n be e loye to u nless otherw ise n te by l w (e.g., chil
hel better u n erst n the in ivi u ls intel- bu se or c ses of bortion in so e st tes)
lectu l level of fu nctioning n ru le ou t n or w hen rent l involve ent is necess ry
intellectu l is bility. IQ ssess ent w ill lso in king co lex n life-thre tenin g
i the school in evelo ing n in ivi u l- e ic l ecision s. In this c se, th e hysici n
ize stu y rogr for the stu ent. The Ror- u st res ect the tients confi enti lity bu t
sch ch Test (H) is rojective test th t y shou l re in cogniz nt of the i lic tions
be u se to ssess erson lity stru ctu re. The this w ill h ve on the f ily syste . Therefore,
tient h s ch r cteristics of schizoi er- it is ost ro ri te to encou r ge the inor
son lity isor er, w hich y be better elu ci- to iscu ss the issu e w ith her rents. Testi o-
te w ith skille ssess ent in the context ni l rivilege refers to the rivilege invoke
of his clinic l history. The Folstein MMSE to rotect hysici n tient confi enti lity
(E) is qu ick, e sily inistere test th t w hen e ic l infor tion is su b oen e
llow s for i e i te ssess ent of jor w ithou t tient consent. At this oint, there
neu rocognitive isor er ( e enti ). Scores is no in ic tion of chil h r or neglect th t
of less th n 24 re su ggestive of e enti . w ou l n te re orting to Chil Protective
The MMPI-2 (F) is n objective test consisting Services. The re ining tw o choices f il to
of sever l hu n re tru e/ f lse qu estions u se int in hysici n tient confi enti lity.
to ssess n in ivi u ls erson lity. It is the
ost w i ely use n highly st n r ize 51. (D ) This tient is ex eriencing jor e res-
test of erson lity. The W Test (J) is u se sive isor er (MDD), single e iso e. Anti e-
to ev lu te he is heric l ngu ge o in nce ress nts re the tre t ent of choice. As this
rior to surgic l elior tion of seizure focus. tient is show ing so e i rove ent n
Where s ost right-h n e in ivi u ls show toler ting the e ic tion, the ost ro ri-
left he is heric o in nce for l ngu ge, te next ste w ou l first be to xi ize the
left-h n e in ivi u ls y be either right ose. While u g enting w ith n ty ic l nti-
or left o in nt. The test consists of inject- sychotic (e.g., ri i r zole) or lithiu c n be
ing so iu yt l into the c roti rtery hel ful in tre t ent refr ctory e ression, this
n observing the tr nsient effects on s eech. tient h s not h n equ te ose or tri l
276 8: Pra c tic e Te s t 2

of his cu rrent nti e ress nt. Discontinuing Asking the tient how i ort nt reg-
his fluoxetine w ou l gre tly incre se his risk n ncy is to her w ou l be in ro ri te, since
of rel se. While sw itching to nother nti- it is erson l choice, n , ccor ing to the
e ress nt (e.g., sertr line) y be in ic te , infor tion rovi e in the vignette, it is not
this w ou l only be reserve in c ses w here so ething th t u ts her in nger of ny kin .
he either i not res on to n equ te tri l ECT y be n o tion s it is s fe in reg-
of xi l ose of n nti e ress nt, or if n ncy, bu t not before iscu ssion of the risks
he suffere signific nt si e effects fro the n benefits of her cu rrent tre t ent. While
current nti e ress nt. fluoxetine oes not h ve the w ith r w l syn-
ro e know n to occu r w ith roxetine, sto -
52. (E) MDD ten s to be chronic illness. P tients ing it i e i tely shou l be voi e u e to
w ith one e iso e of e ression h ve 50% the risk of rel se.
risk of h ving fu tu re e iso es, lthou gh nti-
e ress nts hel to ini ize the risk. 55. (A) This tient is exhibiting sychosis (cu t-
ting ow n tree th t his w ifes ghost lives
53. (C) The tient is su ffering fro jor e res- in) n e ression. There re ny ossible
sive isor er (MDD), w ith ty ic l fe tures etiologies, inclu ing jor e ressive isor-
(i.e., incre se etite/ slee , oo re ctiv- er w ith sychotic fe tu res, bi ol r isor er,
ity). Bec u se of their si e effect rofile, the schizo ffective isor er, n schizo hre-
tre t ent of choice in jor e ression is the ni . Reg r less of the eventu l i gnosis, it
serotonin-s ecific reu t ke inhibitors (SSRIs), is i ort nt to first ru le ou t other c u ses of
such s flu oxetine (Proz c). A itri tyline n his beh vior su ch s illicit or rescribe rug
other tricyclic nti e ress nts (TCAs) inclu e intoxic tion. Therefore, it is re tu re t this
the risk of rrhyth i s n c n be leth l in oint to rescribe ny e ic tions for hi .
over ose, u nlike the SSRIs. While the tient
re orts erio s u ring w hich she is ju st fine, 56. (C) Tr zo one, w hich w s evelo e s n
there is nothing to in ic te cycling ttern nti e ress nt, h s been show n to be n effec-
(i.e., bi ol r isor er) th t w oul require tive slee ing i n w ou l be useful in this
oo st bilizer su ch s iv l roex so iu . tient. The benzo i ze ines, such s l r -
Si il rly, being convince th t her fi nc is zol n i ze , overl w ith lcohol
bou t to le ve her is not likely sychotic, bu t in bin ing to GABA rece torshence their
is inste istortion th t oes not requ ire n u se in lcohol w ith r w l n shoul be
nti sychotic su ch s zi r si one. Althou gh voi e in tients w ith history of lcohol
MAOIs such s henelzine re thou ght to be u se isor er. Z le lon is rel tively contr in-
so ew h t ore effective (th n TCAs) in ty - ic te in those w ith severe liver ysfu nction,
ic l e ression, they c rry the risk of hy er- w hich is otenti l issu e in tient w ith
tensive crisis w hen co bine w ith foo s history of chronic lcohol u se. Zol i e h s
cont ining try to h n. lso been show n to h ve si il r ictive
otenti l to the benzo i ze ines.
54. (C) Flu oxetine is cl ss C ru g, e ning th t
it is otenti lly h r fu l to the evelo ing 57. (D ) Tr zo one c rries 1 in 1,000 to 1 in 10,000
fetu s, ccor ing to ni l stu ies, lthou gh risk of ri is . The other sexu l si e effects
SSRIs s cl ss re not believe to be ter to- ong the choices re know n to occur w ith the
genic or c u se birth efects. A iscu ssion of serotonin-s ecific reu t ke inhibitors, but not
the risks n benefits of tre t ent is therefore rticul rly w ith tr zo one. Incontinence is
ost ro ri te. There is no in ic tion th t not frequent si e effect of ny nti e ress nt.
she is t rticu l r risk for ost rtu e res-
sion n no evi ence in the liter ture th t 58. (B) All of the SSRIs h ve been in the new s
ing secon nti e ress nt in regn ncy bec u se of controversy over t on incre se
w ill revent this isor er. suici l gestures ong chil ren t king these
Answe rs : 5265 277

e ic tions. As resu lt of nu erou s stu ies e ressive e iso es of bi ol r isor er. Anti-
n review , the FDA h s l ce bl ck box e ress nts su ch s bu ro ion n fluox-
w rning for ll nti e ress nts u e to the etine shou l be voi e (es eci lly w ithou t
s ll incre se risk of suici l thou ghts or n ition l oo st bilizer) s they y
beh vior in chil ren t king nti e ress nts reci it te nic e iso e or le to r i
co re w ith l cebo. H ow ever, flu ox- cycling (> 3 oo e iso es er ye r). While
etine is not only the ost stu ie in chil ren, secon -gener tion nti sychotics (e.g., qu e-
bu t, long w ith escit lo r (Lex ro), re ti ine) re likely benefici l in the tre t ent
the only e ic tions FDA rove for the of bi ol r e ression, first-gener tion nti-
tre t ent of chil hoo e ression, w hich sychotics (neu role tics), su ch s h lo eri ol,
kes sw itching to roxetine or itri ty- re not effective n y w orsen e ressive
line in ro ri te in this setting. Asking the sy to s. V l roic ci is consi ere first-
chil s other w h t is re lly bothering her line tre t ent of bi ol r ni , bu t the t
is u nlikely to be ro u ctive, s it is so ew h t re f r less convincing for e resse bi ol r
ccu s tory n not ressing her legiti te tients.
concerns. Fluoxetine oes not h ve the w ith-
r w l syn ro e know n to occu r w ith rox- 61. (D ) L otrigine u se h s 7% risk of r sh;
etine, bu t sto ing it i e i tely shou l still u nfortu n tely, StevensJohnson syn ro e
be voi e . ( life-thre tening r sh) occu rs in 0.1% of
tients, n bec u se of this l otrigine u st
59. (D ) Althou gh ny in ivi u ls think of n t- be titr te slow ly. Other signific nt bu t less
u r l re e ies su ch s St. John w ort s s fe, concerning verse events inclu e t xi ,
these su le ents c n h ve signific nt si e n u se , n se tion. N eu tro eni is oten-
effects. In rticu l r, St. John w ort y c u se ti lly ngerou s si e effect of c rb ze ine,
incre se hotosensitivity, sto ch u set, not l otrigine.
r shes, f tigu e, restlessness, he che, ry
outh, izziness, n confu sion. These si e 62. (B) H er l ck of rior su ici e tte ts is her
effects re ore likely w hen St. John w ort is ost signific nt rotective f ctor; lesser ro-
co bine w ith SSRIs. The recise h r - tective f ctor w ou l be her soci l su orts.
cokinetics of St. John w ort re u nknow n, bu t The other f ctors o not l y s u ch of
St. John w ort is believe to ct like ono- role.
ine oxi se inhibitor (MAOI) n h s
conflicting evi ence reg r ing its effic cy 63. (D ) There re ny otenti l e ic l c u ses
in tre ting il e ression. H ow ever, oses of e ression, inclu ing thyroi ysfunction,
of SSRIs shou l be ecre se w hile tients vit in B12 eficiency, ren l ise se, n
re t king the su le ent bec u se of os- v scu l r ise se. The other choices o not
sible otenti tion of si e effects. As w ith ny likely c u se e ressive illness.
nti e ress nt, it is not goo i e to sto it
bru tly. There is no re son to think th t chil 64. (B) This tient is ost likely su ffering fro
buse is involve , s the chil s other is not norexi nervos . Excessive ent l ec y is
utting her son in nger. co on in tients w ith norexi or bu li-
i w ho urge by self-in u ce vo iting.
60. (D ) This tient is resenting w ith bi ol r Althou gh the other choices y be resent
isor er, ost recent e iso e e resse . in severe st rv tion, excessive ent l ec y is
Des ite the l ck of t reg r ing bi ol r the ost co on n e rs the e rliest.
e ression, the i ct of e ressive e i-
so es y be signific ntly w orse th n th t of 65. (D ) Anorexi nervos h s v rie course,
nic e iso es. L otrigine, n ntie ile - lthough oor rognosis over ll. Stu ies
tic e ic tion, h s e onstr te effic cy in ocu ent th t only bout 50% of tients h ve
( n h s been rove for) the tre t ent of goo outco e; 25% h ve oor ou tco e.
278 8: Pra c tic e Te s t 2

Unfortun tely, the ort lity r te of norexi h ve not been fou n to be s ecific lly hel fu l
nervos is roxi tely 4%. for this ri ry con ition. Althou gh eth-
yl heni te y hel this tients rent
66. (C) This tient likely h s bor erline er- ttention- eficit hy er ctivity isor er sy -
son lity isor er. The first-line tre t ent for to s, it y lso ex cerb te his tic isor er.
bor erline erson lity isor er re ins sy-
chother y, such s DBT. DBT is w ell-stu ie 69. (B) Fire setting in re e te ttern in chil-
v ri tion of CBT th t focu ses on suici l ren often cco nies other beh viors th t
beh vior, consisting of in ivi u l sychother- re efi nt of ru les n u thority, s in con-
y, grou ther y, n ho ew ork ssign- u ct isor er. Althou gh fire setting y occu r
ents. A v riety of e ic tions h ve been in chil ren i gnose w ith u tis s ectru
stu ie in bor erline erson lity isor er, isor er, intellectu l is bility, or tic isor-
inclu ing nti sychotics (e.g., ri i r zole), er, it is u ch ore u nu su l. Chil ren w ith
nti e ress nts (e.g., cit lo r ), n oo jor e ressive isor er o not eng ge in
st bilizers (e.g., lithiu n v l roic ci ). fire-setting s co on beh vior.
H ow ever, w hile the liste e ic tions h ve
e onstr te so e benefit in so e of the 70. (A) This chil is suffering fro nocturn l
sy to s of bor erline erson lity isor er, enu resis. Utilizing bell/ bu zzer n
in the bsence of nother sychi tric isor er evice ( for of beh vior l ther y) is the
(e.g., MDD, bi ol r isor er), sychother y ost effective intervention, hel fu l in gre ter
is the inst y of tre t ent. th n 50% of chil ren. Cognitive ther y, i -
lectic l beh vior l ther y ( for of cog-
67. (E) N igrostri t l D 2 rece tors th t re blocke nitive-beh vior l ther y esigne for the
by nti sychotic e ic tions c use n i b l- tre t ent of bor erline erson lity isor er),
nce betw een o ine n cetylcholine, n sycho yn ic ther y h ve not been
w hich ccounts for the rkinsoni n sy - foun to be effective in the tre t ent of enure-
to s seen in this c se. Block e of cetylcholine sis. Pu nish ent is not ty e of beh vior l con-
rece tors c n hel tre t extr yr i l itioning use to tre t sychi tric illnesses.
sy to s (EPS). D 4 or 5-H T2 rece tor block-
e oes not c u se EPS. Mesoli bic D 2 71. (C) While i i r ine w s consi ere the
rece tor block e is believe to ccou nt for tre t ent of choice, bec use of its si e effects
the nti sychotic effect of nti sychotics n (e.g., c r iotoxicity) es o ressin is consi -
oes not c u se extr yr i l sy to s. ere first-line h r cother y for enu resis.
Ty ic l nti sychotics block both nigros- Althou gh rel se is high fter sto ing the
tri t l n esoli bic o ine, w here s e ic tion, roxi tely 30% of tients
ty ic l (secon -gener tion) nti sychotics chieve tot l ryness, w ith nother 40% exhib-
referenti lly block esoli bic o ine, iting re u ction in be w etting. C rb ze-
ccou nting for the over ll ecre se extr y- ine is n nticonvu ls nt so eti es use s
r i l sy to li bility. oo st bilizer, n cloni ine is n ntihy er-
tensive often use to tre t o i te w ith r w l
68. (D ) This tient is ost likely suffering fro n ADH D; neither re ro ri te for the
Tou rette isor er. Aty ic l nti sychotic tre t ent of enuresis. Di henhy r ine is n
e ic tions (such s ris eri one) w hich block ntihist ine w ith nticholinergic ro erties,
o ine rece tors re consi ere the ost often use for inso ni or to tre t extr yr -
effic ciou s t ecre sing voc l n otor tics. i l sy to s, n r rely use for enuresis.
Cloni ine is benefici l in tre ting Tou rettes,
es eci lly in il c ses, lthough it y c u se 72. (B) B se on his c e ic ifficu lties n
excessive se tion n izziness. Flu oxetine IQ, this tient w ou l be cl ssifie s h ving
n to ir te y be effective for co orbi il intellectu l is bility (intellectu l evel-
con itions foun w ith Tourette isor er, but o ent l isor er). Intellectu l is bility is
Answe rs : 6677 279

ch r cterize by eficits in intellectu l fu nc- is lso r hili w hereby the in ivi u l


tions confir e by clinic l ssess ent n s tisfies sexu l u rges by ru bbing g inst non-
st n r ize intelligence testing (e.g., IQ test- consenting in ivi u ls. Pe o hilic isor er
ing), s w ell s eficits in tive fu nctioning involves sexu l ctivity w ith re u bes-
th t resu lt in f ilure to eet evelo ent l cent chil . The focu s of tr nsvestic isor er
st n r s. While intellectu l is bilities re involves cross- ressing, n is u su lly seen in
co on in tients w ith u tistic s ectru heterosexu l les. The r hili of observ-
isor er, there is no history of restrictive, ing u nsu s ecting eo le n ke or involve
re etitive tterns of beh vior or interests in sexu l ctivity is i gnose s voyeu ristic
consistent w ith u tis . isor er.

73. (D ) Microce h ly, short l ebr l fissu res, 76. (B) Des ite his recent i gnosis of jor
fl t i f ce, n thin u er li re ll sso- e ressive isor er (MDD), this tient h s
ci te w ith fet l lcohol syn ro e (FAS), sy to s consistent w ith cu rrent nic
w hich w ou l be su s ecte in this tient w ith e iso e (e.g., eu hori , ecre se nee for
tern l history of chronic, severe, lcohol slee , i u lsivity, ressu re s eech, i ir-
u se. Cleft l te is not s ecific lly ssoci te ent in fu nctioning), n therefore w oul be
w ith FAS. Congenit l blin ness is not sso- i gnose w ith bi ol r isor er, current e i-
ci te w ith FAS, lthou gh ethyl lcohol so e nic. Bi ol r isor er, cu rrent e iso e
consu tion c n c use blin ness in u lts. e resse w oul be i gnose in tient
H y erextensible joints n ro inent j w w ith history of ni bu t cu rrent jor
re ssoci te w ith fr gile X syn ro e. e ressive e iso e. Criteri for cyclothy ic
isor er re e ressive e iso es (w hich o
74. (E) Chil ren w ith re ctive tt ch ent is- not eet criteri l for MDD) ltern ting w ith
or er is l y rke ly istu rbe n in - hy o nic e iso es (sy to s of ni
ro ri te tt ch ent beh viors ssoci te w ithou t i ir ent in fu nctioning, sychotic
w ith thologic l c re, su ch s seen in this sy to s, or the nee for hos it liz tion).
c se. Althou gh ADH D n con u ct isor- Persistent e ressive isor er ( ysthy i ) is
er re often co orbi in those chil ren w ho chronic, e ressive illness, l sting for t le st
h ve histories of re ctive tt ch ent isor- 2 ye rs. Althou gh the tient h s nic sy -
er, there is not enou gh evi ence to i gno- to s n qu estion ble sychosis ( gr n iose
sis either t this ti e. P tients w ith u tis elusion), there is no in ic tion th t he h s
s ectru isor er e onstr te ersistent h erv sive sychotic sy to s in the
i ir ent in reci roc l soci l co u nic - bsence of his oo sy to s, w hich w ou l
tion (si il r to re ctive tt ch ent isor er) be consistent w ith schizo ffective isor er,
bu t lso h ve restricte , re etitive tterns of bi ol r ty e.
beh vior, interests, or ctivities, not evi ent
in this c se. The criteri for l ngu ge isor- 77. (B) As this tient h s sy to s consistent
er inclu e ifficu lties in the cqu isition n w ith bi ol r isor er, ni , he w ou l ost
u se of l ngu ge u e to eficits, n o not benefit fro lithiu or nother oo st bi-
inclu e bnor lities in e otion l co u- lizer su ch s v l roic ci , or n ty ic l nti-
nic tion (e.g., is l ys of ggression tow r sychotic (e.g., ol nz ine). Anti e ress nts
other chil ren) fou n in re ctive tt ch ent such s cit lo r , flu oxetine, n i i r -
isor er. ine c n ex cerb te ni n shou l be
voi e . Lor ze , benzo i ze ine, y
75. (A) This tient w ou l ostly likely be i g- be u se to tre t sycho otor git tion n
nose w ith fetishistic isor er, r hili inso ni , bu t it oes not tre t the u n erlying
ch r cterize by the use of nonliving objects isor er n h s risk of iction if u se in
for sexu l gr tific tion. Frotteuristic isor er the long ter .
280 8: Pra c tic e Te s t 2

78. (C) A jor e ressive e iso e w ith ty i- erson lity isor er re erfectionists, inflex-
c l fe tu res is ch r cterize by oo re ctiv- ible, n overly e n ing of the selves
ity, rejection sensitivity, hy erso nolence, n others. Schizoty l tients re know n
incre se etite, n le en r lysis for their qu irkiness n see ingly gic l
(i.e., he vy feeling in extre ities). Mel ncholic thinking styles.
fe tures inclu e signific nt nhe oni , e res-
sion w orse in the orning, ter in l inso ni , 81. (C) The essenti l fe tu res of schizo ffective
rke sycho otor bnor lities, signifi- isor er re ro inent oo e iso es ( jor
c nt w eight loss, n in ro ri te guilt. To e ression, ni , or ixe e iso e) concu r-
i gnose bi ol r I isor er there ust be t rent w ith sy to s consistent w ith schizo-
le st one nic e iso e. Bi ol r II isor er is hreni . In ition, the i gnosis requ ires
ch r cterize by n e iso e of jor e res- erio of 2 w eeks of sychotic sy to s
sion n hy o nic e iso e. Persistent w ithou t ffective sy to s w hich ru les ou t
e ressive isor er ( ysthy i ) is chronic, bi ol r isor er or MDD w ith sychotic fe -
often less-intense e ression, l sting 2 ye rs. tu res. While tients w ith schizo hreni y
( n often o) beco e e resse , the oo
79. (C) Althou gh not co only u se bec u se sy to s re not resent for su bst nti l
of the risk of si e effects, MAOIs re consi - ortion of the illness. Uns ecifie sychotic
ere to be the ost effective cl ss of ru gs isor er is u su lly reserve for situ tions
for tre ting ty ic l e ression, es eci lly in w hich the clinici n oes not h ve enou gh
co re to TCAs. SSRIs re lso quite effec- historic l t to know w hether the tient
tive for tients w ith ty ic l fe tures. So e h s schizo hreni , oo isor er w ith
nticonvu ls nts n lithiu re u se for the sychotic fe tu res, su bst nce-in uce sy-
tre t ent of bi ol r isor er, bu t not ty i- chotic isor er, or sychotic isor er u e to
c lly for uni ol r e ression. First-gener tion nother e ic l con ition.
nti sychotics (e.g., h lo eri ol) re u se to
tre t sychotic isor ers su ch s schizo hre- 82. (D ) This w o n is su ffering fro eliriu ,
ni n schizo ffective isor er, s w ell s likely c u se by u lti le e ic tions. The
jor e ression w ith sychotic fe tu res, but el erly re rticu l rly vu lner ble to e i-
they re not ro ri te s onother y for c tions know n to h ve sycho ctive ro -
MDD. erties or si e effects. This is in rt ue to
the ecre se bility of geri tric tients to
80. (D ) This c se eliber tely brings together et bolize e ic tions n the ten ency of
ny ch r cter tr its r w n fro u re the rugs to bu il u to higher levels. A i-
i gnoses of erson lity (ch r cter) isor ers, tion lly, el erly ersons re gener lly ore
inclu ing bor erline, n rcissistic, e en ent, vu lner ble to the effects of e ic tions th t
obsessive-co ulsive, n schizoty l er- ct on the centr l nervou s syste (CN S). The
son lities. In clinic l r ctice, ny tients tient in this c se is on nu ber of e i-
o not eet criteri for ny one s ecific c tions th t y h ve CN S si e effects t
erson lity isor er; su ch ixe c ses re high oses. H ow ever, the signific nt nti-
referre to s other s ecifie erson lity cholinergic ro erties of i henhy r ine
isor er. P tients w ith i gnose bor er- ut this tient t rticu l r risk of e ory
line erson lity isor er re know n for their i ir ent n confu sion. Ci eti ine n
intense erson l rel tionshi s, oo l bility, fu rose i e re lso otenti l contribu tors
self- estru ctive beh viors, nger n ge ent to eliriu , bu t re uch less likely th n
issues, n oor sense of self-i ge. N rcissis- i henhy r ine. Bet blockers like tenolol
tic erson lity isor er is rke by gr n i- h ve been ssoci te w ith e ression, re-
ose sense of self-w orth, entitle ent, l ck of re l su bly ssoci te w ith block e of centr l
e thy, n ten ency to use others for ones renergic rece tors. A ition lly, in so e
ow n en s. P tients w ith obsessive-co ulsive vu lner ble tients, they y re u ce c r i c
Answe rs : 7891 281

ou t u t enou gh to c u se function l hy o er- s ecific reu t ke inhibitors), n venl f x-


fu sion of the br in c using confusion n iz- ine (N ) (co bine serotonin-nore ine hrine
ziness. Digoxin y ctu lly ecre se so iu reu t ke inhibitor), only bu ro ion h s
ion con u ction enou gh in CN S neu rons to re u ce risk of sexu l ysfu nction, n in f ct
ro uce eliriu in so e tients. y be use to tre t nti e ress nt-in u ce
sexu l ysfu nction. While no sychotro ic
83. (A) Deliriu rel te to block e of cholinergic e ic tions re roven s fe in regn ncy,
rece tors is the ost co on c u se of e i- nti nic gents su ch s c rb ze ine,
c tion-in uce eliriu in the el erly. Ace- lithiu (G), n v l roic ci (M) re ssoci-
tylcholine is u niqu ely involve w ith e ory te w ith signific nt fet l bnor lities n
rocesses in the br in, n its involve ent in shou l be voi e . H lo eri ol (E) h s been
the tho hysiology of jor neu rocognitive u se s fely in regn nt tients w ith bi ol r
isor er ( e enti ) ue to Alzhei er is- isor er. L otrigine (F) is n ntie ile -
e se h s been he vily rese rche . Dru gs th t tic e ic tion in ic te for the inten nce
lter o ine neu rotr ns ission re ost tre t ent of bi ol r isor er (to el y the
ssoci te w ith ove ent isor ers n ti e to occurrence of oo e iso es) n
sychosis-rel te si e effects. Me ic tions for the cute tre t ent of bi ol r e res-
ffecting GABA rece tors (su ch s benzo i z- sion. Ari i r zole (A) n ris eri one (J) re
e ines n b rbitu r tes) y c u se se tion secon -gener tion nti sychotic e ic tions
n t xi s w ell s e ory istu rb nces. use for schizo hreni , schizo ffective isor er,
Dru gs cting on the serotonergic syste y n bi ol r isor er.
lter slee , etite, n oo , lthou gh few
non sychi tric ru gs w ork on this syste . 90. (E) Stu ies th t fin no st tistic l ifference
Me ic tions ffecting centr l nore ine hrine betw een grou s w hen there ctu lly is if-
tr ns ission (e.g., ntihy ertensives) y ference resu lt in ty e II error. In this c se,
r rely be ssoci te w ith e ression. the ositive fin ings t the other u niversities
ke the fin ing t u niversity A less likely
8489. [84 (K), 85 (C), 86 (D ), 87 (B), 88 (E), 89 (F)] to be correct. University A y h ve nee e
P tients w ith ent l illnesses y be non- l rger nu ber of su bjects in the stu y to
co li nt for v riou s re sons. Long- cting fin the tre t ent effect. Ty e I errors occu r
nti sychotics su ch s ris eri one Const ore often th n ty e II errors n c u se the
(K) re goo choices in sychotic tients rese rcher to conclu e th t there is if-
w ho benefit fro n toler te nti sychot- ference betw een grou s w hen in f ct there
ics, bu t w ho re nonco li nt u e to l ck of is none. The v ri nce is the su of the if-
insight, confu sion, or forgetfu lness (not u e ferences of e ch t oint fro the e n.
to si e effects). Chlor ro zine (C) is ty i- Positive re ictive v lu e is the ro ortion of
c l nti sychotic (neu role tic) w hich h s been bnor l test resu lts th t re tru e ositive,
ssoci te w ith sensitivity to su nlight, w hich w hile neg tive re ictive v lu e is the ro or-
c n c u se n ex gger te sunburn. Cloz - tion of nor l test resu lts th t re true neg -
ine (D ) w s the first ty ic l nti sychotic tive. St n r error is the egree to w hich
evelo e ; it is rove for the tre t ent the e ns of sever l ifferent s les w oul
of tients w ith refr ctory schizo hreni n v ry if they w ere t ken re e te ly fro the
h s lso e onstr te effic cy in re u cing s e o u l tion.
suici lity in schizo hreni or schizo ffective
isor er. Bu ro ion (B) is n nti e ress nt 91. (E) This tient h s signs ost consistent w ith
th t is believe to block reu t ke of o - t r ive yskinesi , n extr yr i l syn-
ine in the br in. Althou gh ll nti e ress nt ro e th t usu lly ffects erior l or li b
e ic tions h ve si il r effic cy, inclu ing u scu l tu re n c u ses choreifor ove-
nortri tyline (H) (tricyclic nti e ress nt), ents. Its onset is u su lly sever l ye rs fter
roxetine (I) n sertr line (L) (serotonin- t king nti sychotics n it is ore likely to
282 8: Pra c tic e Te s t 2

ffect ol er tients. Ak thisi is escribe is n nti e ress nt u se to tre t jor


s sycho otor restlessness th t y h ve e ressive isor er, but its ctiv ting ro er-
n onset of ys to w eeks fter beginning n ties ke it not s u sefu l for the tre t ent of
nti sychotic. Dystoni is n cu te re ction PTSD. Anti sychotics like h lo eri ol shou l
to nti sychotics (neu role tics) in w hich r- only be use in severe c ses of PTSD w ith
ticu l r uscle grou s (neck or ocu l r uscles ro inent sychotic sy to s or git tion/
co only) contr ct involu nt rily. It c n be ggression. Lor ze n other benzo i ze-
infu l n shou l be tre te i e i tely ines shou l be voi e in tients w ith PTSD
w ith nticholinergic e ic tions. N MS is ue to their l ck of effic cy n otenti l for
otenti lly leth l bu t r re e ic l e ergency iction. Moo st bilizers su ch s v l roic
in w hich tients y h ve glob l rigi ity, ci y be benefici l for so e tients w ith
ent l st tu s ch nges, fever, c r iov scu l r PTSD, bu t re not likely s effic ciou s s SSRIs
inst bility, elev te cre tine hos hokin ses, or l h -blockers like r zosin.
n risk of rh b o yolysis. P rkinsonis y
look i entic l to P rkinson ise se (tre or, 94. (D) Erik Erikson w s sycho n lyst best
br ykinesi , ske f ces) n usu lly h s known for his escri tion of eight st ges of
n onset w ithin ys to w eeks fter beginning hu n sychologic l ex erience s nning the
n nti sychotic e ic tion. entire life s n. He believe th t successful
co letion of ll e rlier st ges w s necess ry
92. (A) The b s l g ngli , res onsible for the for successful co letion of future st ges. The
tr ns ission of thought to ction n for con- tient in this ex le is ble to ro uctively
trolling the initi tion n qu lity of otor work but h s ifficulty with inti cy in his
ction, is centr l to the tho hysiology of rel tionshi , hence the conflict between inti-
extr yr i l syn ro es, inclu ing ys- cy n isol tion. All of Eriksons eight st ges
toni , rkinsonis , k thisi , n t r ive center on st ge- ro ri te evelo ent l
yskinesi . The cerebellu is i ort nt in conflicts: b sic trust versus istrust (birth to
controlling the coor in tion of otor ove- 1 ye r); utono y versus sh e n oubt
ents n ostu re. The front l cortex is gen- (13 ye rs); initi tive versus guilt (35 ye rs);
er lly consi ere to be i ort nt for ecision in ustry versus inferiority (611 ye rs); i entity
king, i u lse control, n ffect regu l - versus role iffusion (11 ye rs to en of oles-
tion. The i br in cont ins nuclei th t hel to cence); inti cy versus isol tion (2140 ye rs);
ensu re centr l nervou s syste ho eost sis by gener tivity versus st gn tion (4065 ye rs);
regu l ting neu roveget tive, u tono ic, n n integrity versus es ir (65 n ol er).
rou s l functions. The otor cortex serves s
the l st st ge of cerebr l rocessing of otor 95. (B) H y erreligiou s thinking or reoccu tion
infor tion before it escen s into the s in l w ith or l beh vior, ltere sexu l beh viors,
cor . An int ct otor cortex is requ ire for hy ergr hi or over el bor tive co u-
initi tion of ove ent. nic tion styles ( lso referre to s viscosity),
n heightene ex erience of e otions for
93. (D ) This tient is suffering fro chronic ost- cl ssic constell tion of erson lity tr its
tr u tic stress isor er (PTSD) sy to s, ssoci te w ith co lex rti l e ile sy (or
only rti lly tre te es ite co li nce te or l lobe seizu res). De ression, obses-
w ith e i tions n bstinence. Serotonin- sive-co u lsive sy to s, n slee isor-
s ecific reu t ke inhibitors (SSRIs) re con- ers y be ssoci te w ith structur l br in
si ere first-line tre t ent bec u se of their inju ry in front l n su bcortic l re s, bu t
effic cy n toler bility. Pr zosin, n l h -1 they h ve not been escribe s being rel te
renergic rece tor blocker h s e onstr te s ecific lly to co lex rti l e ile sy. Uri-
signific nt effic cy in tre ting ny of the n ry incontinence is ore often ssoci te
sy to s of PTSD (e.g., night res, fl sh- w ith nor l ressu re hy roce h lu s or jor
b cks, hy er rou s l, voi nce). Bu ro ion neu rocognitive isor er ( e enti ).
Answe rs : 92102 283

96. (E) This tient ost likely w oul e on- thiori zine h s been ssoci te w ith ig en-
str te seizure ctivity in his te or l lobes, t ry retino thy, lso know n s retinitis ig-
consistent w ith te or l lobe e ile sy. entos . Therefore, oses shoul not excee
800 g/ . Pig ent ry retino thy c n c use
97. (B) The Gl sgow Co Sc le e su res loss of retin l res onse to contr ction of the
level of rou s l n r nges fro sc le of 3 visu l fiel . An e rly sign y be noctu rn l
( ee co ) to 14 (fu lly lert). The c tegories confu sion. Consti tion, ry eyes, n uri-
ssesse re eye o ening, best otor res onse, n ry retention re ll si e effects fro the
n best verb l res onse. Eye o ening r nges nticholinergic ro erties of thiori zine, bu t
fro score of 4 (o ening s ont neou sly) to 1 ty ic lly re not so severe s to co ro ise
(not o ening t ll). In this c se, the eyes o en he lth er nently. N e hrogenic i betes
to in, so score of 2 is given. Best otor insi i u s is not ssoci te w ith thiori zine.
res onse is s follow s: obeys co n s, 5;
loc lizes in, 4; flexion, 3; extension, 2; n 101. (B) This tient is ex eriencing n cu te
no res onse, 1. In this c se, score of 3 is ystonic re ction fro the h lo eri ol,
given for the res onse. Best verb l res onse is high- otency first-gener tion (ty ic l) nti-
lso b se on sc le r nging fro 5 to 1. An sychotic, ty ic lly seen hours to ys fter
oriente tient receives score of 5 n no initi ting or incre sing the nti sychotic.
res onse receives score of 1, s in this c se. Acu te ystonic re ctions y resent s u s-
The scores re then e , n in this c se re cle s s s, torticollis, oculogyric crisis, n
2 + 3 + 1 = 6. l rynge l s s . Benztro ine 1 to 2 g IM is
u seful in the tre t ent of cu te ystonic re c-
98. (E) When ch nging fro n SSRI to n MAOI, tions. Altern tively, i henhy r ine 50 g
one u st llow w shout erio of bout IM c n lso be u se . If the sy to s o not
five h lf-lives for the SSRI to be ore th n resolve w ithin 20 inu tes, l rger oses c n be
90% eli in te ; in the c se of flu oxetine, this given. Benzo i ze ines c n lso be u se bu t
is roxi tely 5 w eeks. This rec u tion re not first-line tre t ent. For cu te l ryn-
voi s the serotonin syn ro e, w hich y ge l ystoni , l rger oses of benztro ine
occur w hen signific nt bloo levels of both shou l be given i e i tely, follow e by
ru gs re resent. Serotonin syn ro e, w hich lor ze IV if nee e . N one of the other
c n be f t l, is rke by sy to s of tre or, choices re in ic te for the tre t ent of cute
i horesis, rigi ity, yoclonu s, u tono ic ystoni . While n lgesics su ch s cet ino-
ysregu l tion, hy erther i , rh b o yoly- hen n ibu rofen y hel w ith in, they
sis, ren l f ilu re, n co . The other choices w ont resolve the u n erlying etiology. Cyclo-
w ou l ose signific nt risk of evelo ing benz rine is u scle rel xer w hich w ill not
serotonin syn ro e. equ tely tre t the ystoni . Pro r nolol is
B-blocker, u se for hy ertension, bu t it c n
99. (C) MAOIs revent the eri her l bre k ow n lso be use to tre t k thisi , nother si e
of tyr ine, w hich y be ingeste w ith cer- effect of nti sychotics, ch r cterize by n
t in foo s su ch s liver, beer, re w ine, ge inner feeling of restlessness n nee to ove.
cheeses, s oke fish, ry s u s ge, n f v
be ns. If too u ch tyr ine bu il s u in the 102. (C) The isor er escribe is ne hrogenic i -
bloo , it y s tu r te eri her l sy thetic betes insi i u s, ost likely c use by lithiu .
nerve rece tors n c use hy ertensive crisis, Lithiu inhibits the effect of nti iu retic hor-
otenti lly f t l con ition. The other choices one on the ki ney, c using olyuri , thirst,
o not contribu te to hy ertensive crisis. oly i si , n otenti l hy ern tre i n
eliriu . Although c rb ze ine n v l-
100. (D ) Thiori zine is ty ic l nti sychotic, roic ci (other oo st bilizers), h lo eri ol
r rely u se now u e to the risk of c r i c ( first-gener tion nti sychotic), n queti -
rrhyth i . At oses of 1,000 g/ or higher, ine ( secon -gener tion nti sychotic) re ll
284 8: Pra c tic e Te s t 2

use to tre t bi ol r isor er, none re ssoci- inhibitors (such s irt z ine n venl f x-
te w ith i betes insi i us. ine), n serotonin-s ecific reu t ke inhibitors
(SSRIs) like sertr line y ll be s fely use in
103. (B) Cloni ine, centr l l h -2- utorece tor tients w ith c r i c ise se; how ever, tten-
gonist, h s been roven useful in the tre t ent tion shoul be i to otenti l hy ertension
of the utono ic hy er ctivity ssoci te with c use or w orsene by venl f xine n ny
o ioi with r w l. A nt ine is use to tre t c r i c e icines et bolize through the
P rkinson ise se n lso s n nti-influenz P-450 cytochro e syste , s so e SSRIs (e.g.,
gent. H lo eri ol, n nti sychotic, n lor z- fluoxetine) y lter the e ic tion levels.
e , benzo i ze ine, y hel with signifi-
c nt git tion occ sion lly seen in with r w l, 106. (D ) Acetylcholine is the neu rotr ns itter
but they will not tre t the un erlying sy - ost i lic te in cognitive fu nctioning.
to s. N loxone, n o i te nt gonist, is use in Anticholinergic effects of e ic tions re fre-
o i te over ose n will signific ntly worsen qu ently i lic te in cognitive ecline n
o i te with r w l sy to s in this tient. ru g-in u ce eliriu . Of the e ic tions
liste , roxetine h s the ost nticholinergic
104. (D ) Meth one h s been roven to signifi- effect, n therefore shou l be voi e in this
c ntly re uce the use of heroin w hen use s tient w ith jor neu rocognitive isor er
inten nce e ic tion. P tients int ine ( e enti ).
on oses low er th n 40 g/ of eth one
re f r ore likely to rel se th n those on 107. (A) Lithiu toxicity is ch r cterize e rly on by
higher oses. D ntrolene is uscle-rel x nt ys rthri , t xi , co rse tre or, n b o i-
u se in the n ge ent of neu role tic lig- n l in. L ter nifest tions inclu e seizures,
n nt syn ro e, r re si e effect of nti sy- neuro uscul r irrit bility, n i ire con-
chotic tre t ent. Disulfir is e ic tion sciousness ( eliriu to co ). Acute ystoni s
u se to revent lcohol consu tion by re ssoci te w ith the use of ty ic l nti sy-
c using n versive re ction if t ken in con- chotics. Leg in n resthesi s re not sso-
ju nction w ith lcohol. Flu zenil is in ic te ci te w ith lithiu . P r noi elusions re
for benzo i ze ine over ose. Sertr line is rt of the sy to rofile of schizo hreni
serotonin-s ecific reu t ke inhibitor, u se in n so eti es of bi ol r isor er.
the tre t ent of e ressive n nxiety isor-
ers, ong other sychi tric illnesses. While 108. (B) For ECT to be effective, the seizu re shou l
it y be hel fu l for co orbi e ression l st t le st 15 secon s. Prolonge seizu res
or nxiety, it h s not been show n to ecre se gre ter th n 180 secon s shou l be ter in te
o i te u se in in ivi u ls. w ith n nesthetic (e.g., ro ofol) or intr ve-
nou s benzo i ze ines (e.g., i ze ).
105. (B) I i r ine is tricyclic nti e ress nt
(TCA). TCAs re consi ere cl ss 1 A nti r- 109. (D ) Me ory i ir ent ( ostly u ring the
rhyth ics bec use they ossess quini ine-like tre t ent) is quite co on, lthou gh ost
effects th t ecre se con uction ti e through tients retu rn to their b seline by 6 onths.
the bun le of His. They h ve been show n to There is no in ic tion th t ECT c u ses br in
incre se ort lity in c r i c tients. TCAs ge. ECT y ctu lly be use to tre t
c n lso incre se the he rt r te, n so tient tients w ith c t toni . Fr ctu res n vo it-
w ith co ro ise c r i c function y suf- ing re u nco only seen in ECT since the u se
fer fro incre se oxygen e n . Fin lly, of u scle rel x nts n ntie etics h ve been
TCAs re lso ssoci te w ith signific nt i le ente .
orthost tic hy otension, w hich y be further
ex cerb te in tients w ith c r i c ise se. 110. (C) For extre e git tion, the benzo i ze ine
Wellbutrin, serotonin-nore ine hrine reu t ke lor ze c n be effective. Another ltern tive
Answe rs : 103116 285

is the butyro henone nti sychotic h lo eri ol. DTs re ssoci te w ith signific nt risk of
Phenothi zine nti sychotics su ch s chlor- ort lity.
ro zine c n c u se u tono ic inst bility
w hen given to tient w ith PCP intoxic tion. 115. (E) Sy to s ssoci te w ith coc ine w ith-
Tr zo one, use co only for inso ni , n r w l ten to be il n l st sever l ys
fluoxetine, n SSRI, re nti e ress nts, n fter the l st u se. Occ sion lly, how ever, the
w ill not hel w ith git tion. Trihexy heni yl e ression c n be severe enough to r ise
is use to co b t extr yr i l sy to s the risk of self-inju rious beh vior or su ici e
ssoci te w ith nti sychotic use n not for tte ts.
git te st tes.
116. (G) H eroin w ith r w l e ks t bout
111. (C) Su ortive c re is the best tre t ent for 72 hou rs fter the l st u se in chronic u sers. It is
PCP intoxic tion th t is not co lic te by ex erience s v riou s flu -like sy to s n
extre e git tion or violence. Lor ze , g strointestin l co l ints, su ch s fever n
benzo i ze ine, or h lo eri ol, ty ic l chills, ru nny nose, n u se , bo y ches, i r-
nti sychotic, c n be u sefu l for extre e git - rhe , n b o in l cr s. The e r nce
tion. Aci ific tion of the u rine oesnt signifi- of iloerection in the syn ro e g ve rise to
c ntly incre se the excretion of PCP. Des ite the sl ng ter col tu rkey to escribe the
u rb n yths, neither cheese nor vit ins ction of tot l cess tion of u se of the ru g.
h ve rticu l r ther eu tic benefits for PCP Alcohol intoxic tion (A) involves ch r cter-
intoxic tion. istic beh vior l ch nges, s w ell s slu rre
s eech, t xi , n other neurologic fin ings.
112. (A) This tient is ost likely su ffering fro It c n rogress to co if severe. C nn bis
jor e ression w ith sychotic fe tu res. intoxic tion (C) heightens sensitivity to exter-
N either nti e ress nts nor nti sychotics n l sti u li n i irs otor skills, long
lone w ill s r i ly or effectively tre t jor w ith t chyc r i , conju nctiv l injection, n
e ression w ith sychotic fe tu res s the incre se etite (the u nchies). Coc ine
co bin tion. The co bin tion of lor ze intoxic tion (D ) is rke by el te oo ,
n cloz ine ( n ty ic l nti sychotic) w ill ecre se slee , h llu cin tions, n git tion.
not ro ri tely tre t the e ression. Si i- H eroin intoxic tion (F) is rke by n ltere
l rly, the co bin tion of sertr line n lor z- oo , sycho otor ret r tion, n row si-
e w ill not equ tely tre t the sychotic ness, long w ith in oint u ils n consti-
sy to s. Lith iu n synthroi c n both tion. Intoxic tion by inh l nts (H) (vol tile
effectively u g en t nti e ress nts su ch subst nces su ch s g soline fu es) is rke
s roxetine n sertr line. Lor ze , by eu hori , isorient tion, fe r, n f ci l
benzo i ze ine, y hel w ith nxiety, bu t r sh, lthou gh the sy to s re short-live .
w ill not be effective g inst sychosis or The syn ro es of eth het ine (I) n
e ression. MDMA (N ) intoxic tion re si il r to those
of coc ine intoxic tion. MDMA u sers ty i-
113. (L) PCP ( k ngel u st) intoxic tion ro- c lly re ort n incre se sense of closeness
uces git tion, vertic l nyst g us, n w ith other eo le. N icotine w ith r w l (J) is
n lgesi s w ell s hy erther i , e erson- ch r cterize by e resse oo , inso ni ,
liz tion, n h llu cin tions in u itory, t c- irrit bility, ifficu lty concentr ting, ecre se
tile, n visu l o lities. he rt r te, n incre se etite. N itrou s
oxi e intoxic tion (K) ro u ces eu hori n
114. (B) Alcohol w ith r w l y begin w ithin light-he e ness, n u su lly su bsi es w ithin
24 hou rs fter the l st rink, n c n rogress hou rs w ithou t tre t ent. With r w l fro
to seizures by 24 to 48 hou rs n eliriu silocybin (M), h llu cinogen erive fro
tre ens (DTs) fter roxi tely 72 hou rs. u shroo s, is not w ell escribe .
286 8: Pra c tic e Te s t 2

117. (B) Cloz ine ( n ty ic l nti sychotic) 118. (F) Lithiu is ssoci te w ith clinic l hy othy-
c uses gr nu locytosis in bou t 1% of tients. roi is in t le st 5% of tients, signific ntly
For this re son, w eekly bloo cou nts re e - ore often in w o en. Thyroi su le ent -
su re for the first 6 onths of tre t ent, every tion y be e to cou nter this si e effect.
2 w eeks for the next 6 onths, then every 4 A roxi tely 25% to 30% of tients y
w eeks there fter (u nless there is signific nt show elev te thyroi -sti u l ting hor one
ro in w hite bloo cell cou nts). levels.
Biblio g raphy

Adelson SL; American Academy of Ch ild and Adolescent Psych iatry (AACAP) Com m ittee on Qu ality Issues
(CQI). Practice parameter on gay, lesbian, or bisexu al sexu al orientation, gend er noncon form ity, and gender
d iscord ance in ch ild ren and adolescents. J A m A cad Child A dolesc Psychiatry. 2012;51(9):957974.
AMAs Code of Med ical Ethics. Opinions on Confidentiality, A dvertising, and Communications M edia Relations.
The American Med ical Association. Available at http:// w w w.am a-assn.org/ am a/ pub/ physician-resou rces/
med ical-ethics/ code-med ical-ethics/ opinion504.page? Accessed March 1, 2016.
AMAs Code of Med ical Ethics. Opinions on Confidentiality, A dvertising, and Communications M edia Relations.
The American Med ical Association. Available at http:// w w w.am a-assn.org/ am a/ pub/ physician-resou rces/
med ical-ethics/ code-med ical-ethics/ opinion505.page? Accessed February 28, 2016.
AMAs Code of Med ical Ethics. Opinions on Confidentiality, A dvertising, and Communications M edia Relations.
The American Med ical Association. Available at http:// w w w.am a-assn.org/ am a/ pub/ physician-resou rces/
med ical-ethics/ code-med ical-ethics/ opinion505.page? Accessed March 2, 2016.
AMAs Code of Med ical Ethics. Opinions on Confidentiality, A dvertising, and Communications M edia Relations.
The American Med ical Association. Available at http:// w w w.am a-assn.org/ am a/ pub/ physician-resou rces/
med ical-ethics/ code-med ical-ethics/ opinion505.page? Accessed March 8, 2016
AMAs Code of Med ical Ethics. Opinions on Confidentiality, A dvertising, and Communications M edia Relations.
The American Med ical Association. Available at http:// w w w.am a-assn.org/ am a/ pub/ physician-resou rces/
med ical-ethics/ code-med ical-ethics/ opinion5055.page? Accessed February 26, 2016.
AMAs Code of Med ical Eth ics. Opinions on Practice M atters. The American Med ical Association. Available
at http:/ / w w w.am a-assn.org/ am a/ pu b/ physician-resou rces/ m ed ical-eth ics/ cod e-m ed ical-eth ics.p age?
Accessed Febru ary 14, 2016.
AMAs Code of Med ical Eth ics. Opinions on Practice M atters. The American Med ical Association. Available
at http:// w w w.am a-assn.org/ am a/ pu b/ physician-resou rces/ med ical-eth ics/ cod e-med ical-eth ics/ opin ion
8115.page. Accessed March 13, 2016.
AMAs Code of Medical Ethics. Opinions on Practice M atters. The American Med ical Association. Available
at http:// w w w.ama-assn.org/ ama/ pub/ physician-resou rces/ med ical-ethics/ code-med ical-ethics/ opinion808.
page. Accessed February 29, 2016.
AMAs Code of Medical Ethics. Opinions on Practice M atters. The American Med ical Association. Available
at http:// w w w.ama-assn.org/ ama/ pub/ physician-resou rces/ med ical-ethics/ code-med ical-ethics/ opinion808.
page. Accessed March 12, 2016.
AMAs Code of Med ical Eth ics. Opinions on Practice M atters. The American Med ical Association. Available at
http:// w w w.am a-assn.org/ am a/ pu b/ physician-resou rces/ med ical-eth ics/ code-med ical-eth ics/ opin ion808.
page? Accessed Febru ary 28, 2016.
AMAs Code of Medical Ethics. Opinions on Practice M atters. The American Medical Association. Available at
http:/ / w w w.am a-assn.org/ am a/ pub/ physician-resou rces/ med ical-eth ics/ code-med ical-eth ics/ opin ion8081.
page. Accessed March 2, 2016.
AMAs Code of Med ical Ethics. Opinions on Practice M atters. The American Medical Association. Available
at http:// w w w.ama-assn.org/ ama/ pub/ physician-resou rces/ med ical-ethics/ code-med ical-ethics/ opinion8121.
page. Accessed February 29, 2016.

287
288 Bib liog ra p hy

AMAs Code of Medical Ethics. Opinions on Practice M atters. The American Med ical Association. Available
at http:// w w w.am a-assn.org/ ama/ pub/ physician-resou rces/ med ical-ethics/ code-med ical-ethics/ opinion814.
page. Accessed March 7, 2016.
AMAs Code of Medical Ethics. Opinions on Practice M atters. The American Med ical Association. Available
at http:// w w w.ama-assn.org/ ama/ pub/ physician-resou rces/ med ical-ethics/ code-med ical-ethics/ opinion9031.
page. Accessed March 13, 2016.
AMAs Code of Med ical Eth ics. Opinions on Professional Rights and Responsibilities. The American Med ical
Association. Available at http:// w w w.am a-assn.org/ am a/ pu b/ physician-resou rces/ med ical-eth ics/ code-
med ical-eth ics/ opin ion905.page. Accessed March 12, 2016.
AMAs Code of Med ical Eth ics. Opinions on Professional Rights and Responsibilities. The American Med ical
Association. Available at http:// w w w.am a-assn.org/ am a/ pu b/ physician-resou rces/ med ical-eth ics/ code-
med ical-eth ics/ opin ion907.page. Accessed March 12, 2016.
AMAs Code of Med ical Eth ics. Opinions on Professional Rights and Responsibilities. The American Med ical
Association. Available at http:// w w w.am a-assn.org/ am a/ pu b/ physician-resou rces/ med ical-eth ics/ code-
med ical-eth ics/ opin ion9141.page. Accessed March 8, 2016.
AMAs Code of Med ical Eth ics. Opinions on Social Policy Issues. The American Med ical Association. Available
at http:// w w w.am a-assn.org/ am a/ pu b/ physician-resou rces/ m ed ical-eth ics/ cod e-med ical-eth ics/ opin ion
223.page? Accessed Febru ary 26, 2016.
AMAs Code of Medical Ethics. Opinions on Social Policy Issues. The American Med ical Association. Available
at http:// w w w.ama-assn.org/ ama/ pub/ physician-resou rces/ med ical-ethics/ code-med ical-ethics/ opinion2211.
page? Accessed March 8, 2016.
AMAs Code of Med ical Ethics. Opinions on Social Policy. The American Med ical Association. Available
at http:// w w w.am a-assn.org/ am a/ pu b/ physician-resou rces/ m ed ical-eth ics/ cod e-med ical-eth ics/ opin ion
2215.page? Accessed March 1, 2016.
American Psych iatric Association. A PA Commentary on Ethics in Practice. Topic 3.2.6 Therap eutic Bou nd ary
Keeping. 2015:6.
American Psych iatric Association. Diagnostic and Statistical M anual of M ental Disorders. 5th ed. (DSM -5). 2013:
186187.
The American Psychiatric Association. Principles of Medical Ethics with Annotations Especially Applicable to Psychiatry.
2013:6
Axelson DA, Birm aher B. Relation between an xiety and d epressive d isorders in ch ild hood and adolescence.
Depress A nxiety. 2001;14(2):6778.
Barsky AJ, Orav EJ, Bates DW. Som atization increases med ical utilization and costs independent of p sych iatric
and med ical comorbid ity. A rch Gen Psychiatry. 2005;62(8):903910.
Bobes J, Rejas J, Garcia-Garcia M, et al; EIRE Study Group. Weight gain in patients w ith sch izoph ren ia
treated w ith risperidone, olan zapine, quetiapine or haloperidol: resu lts of the EIRE study. Schizophr Res.
2003;62(1-2):7788.
Boud en A, Krebs MO, Loo H, Olie JP. Mu nchau sen synd rome by proxy: a challenge for med icine. Presse M ed.
1996;25:567569.
Garcia-Campayo J, Ald a M, Sobrad iel N, Olivan B, Pascu al A. Personality d isorders in som atization d isorder
patients: a controlled study in Spain. J Psychosom Res. 2007;62:675680.
Gartleh ner G, H ansen RA, Th ied a P, et al. Comparative Effectiveness of Second-Generation A ntidepressants in
the Pharmacologic Treatment of A dult Depression. Comparative Effectiveness Review No. 7. (Prepared by RTI
International-Un iversity of North Carolina Evid ence-based Practice Center u nd er Contract No. 290-02-
0016.) Rockville, MD: Agency for Healthcare Research and Qu ality. Accessed Janu ary, 2007.
Habeas Corpu s. Legal Information Institute. Cornell University Law School. Available at https:// w w w.law.cornell.
edu/ wex/ habeas_corpu s. Accessed March 12, 2016
Herxheimer A, Petrie KJ. Melaton in for the prevention and treatment of jet lag. Cochrane Database Syst Rev.
2002;(2):CD001520.
Bib liog ra p hy 289

Kellogg N; American Academy of Ped iatrics Com m ittee on Ch ild Abu se and Neglect. The evalu ation of
sexu al abu se in child ren. Pediatrics. 2005;116(2):506512.
Kieler H, Artam a M, Engeland A, et al. Selective seroton in reuptake in hibitors du ring pregnancy and risk of
persistent pu lmonary hyp erten sion in the new born: p opu lation based cohort study from the five Nord ic
cou ntries. BM J. 2012;344:d8012.
Lam b EJ, Steven s PE, Nashef L. Topiram ate increases biochem ical risk of neph rolith iasis. A nn Clin Biochem.
2004;41(Pt 2):166169.
Lee T, Fou ras G, Brow n R; American Academy of Ch ild and Adolescent Psych iatry (AACAP) Com m ittee on
Qu ality Issues (CQI). Practice parameter for the assessment and m anagement of youth involved w ith the
ch ild welfare system. J A m A cad Child A dolesc Psychiatry. 2015;54(6):502517. Available at http:// w w w.jaacap.
com/ article/ S0890-8567(15)00148-3/ pd f
Lock J, La Via MC; American Academy of Ch ild and Adolescent Psych iatry (AACAP) Com m ittee on Qu ality
Issues (CQI). Practice parameter for the assessment and treatment of child ren and adolescents w ith eat-
ing d isorders. J A m A cad Child A dolesc Psychiatry. 2015;54(5):412425. Available at http:// w w w.jaacap.com/
article/ S0890-8567(15)00070-2/ p d f
McClellan J, Stock S; American Academy of Child and Adolescent Psychiatry (AACAP) Com m ittee on Quality
Issues (CQI). Practice parameter for the assessment and treatment of child ren and adolescents w ith schizo-
phrenia. J Am Acad Child A dolesc Psychiatry. 2013;52(9):976990. Available at http:// w w w.jaacap.com/ article/
S0890-8567(13)00112-3/ pd f
Mu rphy TK, Lew in AB, Storch EA, Stock S; American Academy of Ch ild and Adolescent Psych iatry (AACAP)
Com m ittee on Qu ality Issues (CQI). Practice parameter for the assessment and treatment of ch ild ren and
adolescents w ith tic d isorders. J A m A cad Child A dolesc Psychiatry. 2013;52(12):13411359. Available at http://
w w w.jaacap.com/ article/ S0890-8567(13)00695-3/ pd f
Mu rray J, Farrington DP. Risk factors for conduct d isorder and d elinquency: Key find ings from longitud inal
stud ies. Can J Psychiatry. 2010;55(10): 633642.
Neveu s T, Eggert P, Evan s J, et al; International Ch ild rens Continence Society. Evalu ation of and treatment
for monosymptom atic enu resis: a stand ard ization docu ment from the International Ch ild rens Continence
Society. J Urol. 2010;183(2):441447.
Nicole B, Brahm NC, Kissack J. Which psychotropics carry the greatest risk of QTc prolongation? Curr Psychiatr.
2012;11(10):3639. Available at http://www.currentpsychiatry.com/home/article/which-psychotropics-carry-the-great-
est-risk-of-qtc-prolongation/60d07ce6dd7c885cae85d bb19b9dd3c4.html
Olatu nji BO, Mim iaga MJ, OCleirigh C, Safren SA. A review of treatment stud ies of depression in H IV. Top
HIV M ed. 2006;14(3):112124.
Rsner S, H ackl-Herrwerth A, Leucht S, Vecch i S, Srisu rapanont M, Soyka M. Opioid antagon ists for alcohol
dep endence. Cochrane Database Syst Rev. 2010;(12):CD001867.
Sadock BJ, Sadock VA, Ru iz P. Kaplan & Sadocks Synopsis of Psychiatry: Behavioral Sciences/Clinical Psychiatry.
11th ed. Lippincott William s & Wilkin s; 2014.
Serretti A, Ch iesa AJ. Treatment-emergent sexu al dysfu nction related to antidepressants: a meta-analysis. Clin
Psychopharmacol. 2009;29(3):259266.
Singh T, William s K. Atypical Depression. Psychiatry (Edgmont). 2006;3(4);3339.
Stein hau sen HC. The outcome of anorexia nervosa in the 20th centu ry. A m J Psychiatry. 2002;159(8):12841293.
Stone J, Smyth R, Carson A, et al. System atic review of m isd iagnosis of conversion symptom s and hysteria.
BM J. 2005;331(7523):989.
Tish ler CL, Reiss NS, Rhodes AR. Su icid al behavior in ch ild ren you nger than twelve: A d iagnostic challenge
for emergency department person nel. A cad Emerg M ed. 2007;14(9):810818.
This page intentionally left blank
Inde x

A Alcohol w ithd raw al (Cont.): ArgyllRobertson p u p il, 196


Abnorm al Involu ntary Movem ent Scale seizu res and , 170, 189 Arip ip razole, 262, 281
(AIMS), 248, 269 treatm ent, 170, 189 Arnold -Chiari synd rom e, 62
Acetylcholine, 262, 265, 281, 284 Alexia, 13, 28 Ativan. See Lorazep am
Acetylcysteine, 70 Alp ha-blocker, 208, 228 Atropine, 220, 241
Acu te stress d isord er, 42, 66 Alzheim er d isease Attention, assessing, 222, 242243
d ifferential d iagnosis, 158, 168, 187188, d ifferential d iagnosis, 178, 197 Attention-d eficit/ hyperactivity d isord er
213, 233 treatm ent, 253254, 274 (ADH D)
Ad d erall, 24 AMA. See Am erican Med ical Association d ifferential d iagnosis, 223, 243244
ADH D. See Attention-d eficit/ hyp eractivity AMA Cod e of Med ical Ethics, 158, 159160, treatm ent of, 223, 244
d isord er 162163 behavioral therapy, 127
Ad ju stm ent d isord er, 39, 63 Am antad ine (Sym m etrel), 212, 232 sym p tom s, 15
w ith d epressed m ood , 173, 192193 Am erican Law Institute test (Mod el Penal m ethylp henid ate for, 8, 26, 181, 201
d iabetes m ellitu s w ith, 6, 22 Cod e), 142, 155156. See also Legal m ania v., 10, 26
MDD d istinguished , 6, 22 issu es Autistic d isord er, 15
treatm ent, 173, 193 Am erican Med ical Association (AMA), 157 Autonom y, 144, 156157
Ad olescents Am itrip tyline Avoid ant p ersonality d isord er, 184, 203
antisocial personality d isord er, 13, 28 card iac effects of, 93, 114
bip olar d isord er in, 3, 18 m ania and , 50, 73 B
bulim ia nervosa, 20 Am nesia, d issociative, 183, 202 Basal ganglia, 263, 282
d ep ression in, 1, 17 anterograd e, 62 BDI. See Beck Dep ression Inventory
MDD in, 6, 18, 20 ECT and , 67 Beck Dep ression Inventory (BDI), 207, 227,
panic d isord er, 5, 19, 22, 27, 29 w ith fu gue, 213214, 234 254, 275
su icid e risk in, 1, 17 localized , 73 Behavioral therap y, enu resis, 259, 278
Ad ult psychop athology, 3358 Am ok, 211, 231 Benad ryl. See Dip henhyd ram ine
Ad vance d irective, 147, 159 Am phetam ine, 56, 79 Bend er Gestalt Test, 254, 275
Agitation, 266, 284285 Anom ia, 28 Benzod iazep ine
Agnosia, 28 Anorexia nervosa ad verse effects, 110
Agorap hobia, panic d isord er w ith com p lications, 258, 277 alcohol w ithd raw al, 116
d ifferential d iagnosis of, 173174, 193 d ifferential d iagnosis, 175, 195 bip olar d isord er, 167, 187
treatm ent of, 174, 193 m ed ical com p lications, 219, 239 DTs, 253, 274
Agranu locytosis, 109, 113, 116, 120, 209, m ortality rate, 219, 240 high-p otency, 105
228 p rognosis, 258, 277278 Benztrop ine (Cogentin)
AIMS. See Abnorm al Involu ntary Anterograd e am nesia, 62 anticholinergic toxicity and , 211, 232
Movem ent Scale Anticholinergic toxicity, 211, 232 torticollis and , 264, 283
Akathisia Anticholinesterase, 220, 241 Bereavem ent, 249, 269270
antip sychotics and , 104 Antidepressants. See also specific antidepressants d ifferential d iagnosis, 219, 240
risperid one and , 98, 118 OCD and , 220, 240 MDE v., 173, 192
Alcohol abu se Antim otivational synd rom e, 100 treatm ent, 173, 192, 220, 240
d ep ression and , 179, 199 Antip sychotic m ed ications Bipolar d isord er
d isulfiram and , 221, 242 akathisia and , 104105 genetics, 180181, 200
Alcohol d ep end ence, 66 bipolar d isord er, 167, 187 lam otrigine for, 257, 277
Alcohol intoxication, 70, 79, 266, 285 d opam ine recep tors and , 114 m anic, 167, 187, 260, 279
Alcohol u se d isord er, 39, 63 N MS and , 105 treatm ent of, 260, 279
Alcoholism schizotypal p ersonality d isord er, 206, 226 Bipolar I d isord er, 210, 218, 230, 239,
evalu ation, 178179, 198 sid e effects, 51, 75, 92, 113 250251, 271
m ed ications, 97, 118 TD and , 106 Bipolar II d isord er, 37, 40, 61, 64, 105, 165,
thiam ine d eficiency and , 112, 116 Antisocial p ersonality d isord er, 183, 203 185, 214, 234, 250251, 271
Alcohol u se d isord er ad olescent, 22, 28, 29, 203, 229 Bipolar m ania, 113, 277
elevated / d epressed liver enzym es and , cond u ct d isord er and , 20, 22, 24, 28 Blessed Rating Scale, 228, 245, 255, 275
206, 226 Anxiety Bord erline p ersonality d isord er (BPD), 184,
m ed ications for, 207, 226 brain activity and , 55 203, 208, 212, 227, 232
Alcohol w ithd raw al, 267, 285 p erform ance, 118 DBT, 258, 278
elevated / d epressed liver enzym es and , Anxiety d isord ers d ialectical behavioral therap y and , 171,
206, 226 MDD and , 5, 20 191, 212, 232
hallucinations d u e to, 221, 242 Aphasia, 28 d ifferential d iagnosis, 171, 191, 209, 229
lorazepam for, 189, 199, 221, 241242 Apraxia, 28 treatm ent, 258, 278

291
292 Ind ex

Boston Diagnostic Aphasia Exam ination, Cod e of Ethics, AMA, 158, 159160, 162163 Depressive d isord ers
219, 224, 239, 244245, 254, 275 Cogentin. See Benztrop ine d ifferential d iagnosis, 179, 199
BPD. See Bord erline p ersonality d isord er Cognitive-behavioral therap y (CBT), OCD, m ed ically ill p atients w ith, 222, 242
Brain 174, 194 treatm ent, 179, 199, 248, 269
activity in, 55 Cognitive therap y, d ep ressive d isord er, Desm op ressin (Dd avp), 259, 278
d am age, Kl ver-Bucy synd rom e and , 62 179, 199 Desyrel. See Trazod one
frontal lobes, 209, 229230 Cold -caloricind u ced nystagm u s, 213, 233 Dialectical behavioral therap y (DBT)
receptors Com p etency, 155, 251252, 272 BPD and , 171, 212, 232, 258, 278
antip sychotics and , 113 Com p etency to stand trial, 143, 149150, Diazep am (Valiu m ), 251, 272
clozap ine and , 88 156, 160 p ostp artu m d epression, 185
d op am ine, 94 Com p liance, m ed ication, 210, 230 Differential d iagnosis, 165203
haloperid ol and , 94, 115 Cond u ct d isord er, 16, 259, 278 Diphenhyd ram ine (Benad ryl)
m ed ications and , 60 antisocial personality d isord er and , 13 confu sion and , 261, 280281
Brief psychotic d isord er, 59, 62, 66, 71, 172, Confid entiality, 148, 158, 252, 272273 insom nia, 181, 201
191192 m inors and , 145, 157 Discharging p atient, 149, 160
Brow n-Peterson Task, 207, 227 p hysicianp atient confid entiality, 146, from care, 152, 162163
Bu limia nervosa 158, 255, 275 Disulfiram , alcohol abu se and , 221,
ad olescent, 4, 20 violating, 145, 157 242
CBT, 20 w aivers, 156, 158 Divalp roex sod iu m , sid e effect, 222,
com p lications, 277 Continu ou s p ositive airw ay pressu re 243
m ed ical com plications, 219, 239 (CPAP), 220, 241 Divalp roex sod iu m (Dep akote), bip olar
m ed ications, 113 Conversion d isord er, 174175, 194, 211, 230, d isord er, 260, 279
p sychotherap y, 20 251, 272 DTs. See Deliriu m trem ens
signs, 20 Coprop raxia, 249, 270 Du loxetine, 119
sym ptom s, 31 Cou ntertransference, 250, 270. See also Dusky v. United States, 143, 156, 160
treatm ent, 219, 239 Patient-d octor relationship
Bu p rop ion, 6, 22, 26, 37, 262, 281 CPAP. See Continuou s p ositive airw ay E
Bu Spar. See Bu sp irone p ressu re Echolalia, 249, 270
Bu spirone (Bu Sp ar), 51, 75, 107, 109, 167, Crim inal resp onsibility, 142, 156 ECT. See Electroconvu lsive therap y
169, 187, 216, 236 Cu ltu re-bou nd d elu sions, 77 Effexor. See Venlafaxine
Cu ltu re-bou nd synd rom es, 211, 231 Ego-d ystonic, 249, 270
C Cyclothym ic d isord er, 43, 61 Ego-syntonic, 249, 270
Caffeine, w ithd raw al, 217, 237 Cyp rohep tad ine, 103, 107 Electroconvu lsive therap y (ECT)
Cannabis, intoxication, 266, 285 am nesia, 43, 67
Capgras synd rom e, 33, 54, 58, 77, 248, 269 D d ep ression, 36, 60, 214, 235
Card iac arrhythm ia, TCAs and , 169, 189 DBT. See Dialectical behavioral therap y p regnancy and , 94, 115
Card iac effects Dd avp . See Desm op ressin seizure length and , 265, 284
am itriptyline, 93, 113 Defense m echanism s, 11, 12, 35, 59, 70, 80. seizures, 266, 284
lithiu m , 64, 71 See also specific defense mechanisms sid e effects, 284
Catalepsy, 74, 235 Dj entend u , 223, 244 Enu resis
Catap lexy, 18, 51, 69, 74, 200 Delirium , 262, 281 com orbid m ental illness, 8, 24
Catap res. See Clonid ine d ifferential d iagnosis, 172, 177, 178, Dd AVP for, 24
Catatonia, 215, 235236 190191, 197, 198 m ed ication, 8, 24
Charcot, Jean-Martin, 131, 140 m ed ications, 171, 191 second ary noctu rnal, 259, 278
Child abu se, 157 Delirium trem ens (DTs), treatm ent, 253, treatm ent, 259, 278
legal issu es, 12, 30, 144145, 157, 203 274 Ep ilep sy
PTSD and , 19 Delusional d isord er com p lex partial, 263, 282
Child cu stod y, 152, 163 p resentation, 166, 186 d ifferential d iagnosis, 176, 195196
Child ren treatm ent, 166, 186 Erikson, Erik, 263, 282
anxiety d isord ers, 19 Delusions Ethical issu es, 141163
bipolar d isord er in, 3, 11, 19, 26 Fregoli, 33 Extrap yram id al sid e effects (EPS)
clinical interview , 4 MDD, 180, 200 benztrop ine and , 211, 231
d evelopm ent, 15 m ood congru ent, 180, 200
MDD in, 5 p aranoid , 142, 155 F
treatm ent Dem entia Factitiou s d isord er
parental refu sal of, 145146, 158 d ifferential d iagnosis, 209, 217218, d ifferential d iagnosis, 175, 194195
Chlorp rom azine, 262, 281 228229, 238 by proxy, 13, 208, 227 (See also Child
Citalopram , 247, 268 Folstein MMSE, 209, 228229 abu se)
bulim ia nervosa, 219, 239 Lew y bod y, 178, 198 Fetishistic d isord er, 260, 279
Clonid ine (Catapres), 59 Dem erol. See Mep erid ine Fire setting, 259, 278
ad verse effects, 11, 27 Denial, 213, 233 Flat affect, 206, 226
alcohol w ithd raw al, 189 Dep akote. See Divalp roex sod iu m Flu oxetine (Prozac), 247, 268
opiate w ithd raw al, 265, 284 Dep end ent p ersonality d isord er, 183184, d ose, 255, 275276
Tou rette d isord er, 6, 11, 22, 176, 195 203 insom nia and , 172173, 192
Clozapine (Clozaril), 262, 281 Dep ressed reflexes, 222, 243 MDD, 172, 192, 256, 266, 276, 285
agranulocytosis and , 209, 228 Depression p ostp artu m d epression, 166, 185
schizop hrenia, 222, 243 alcohol abu se and , 179, 199 p regnancy and , 256, 276
sid e effects, 208209, 228, 267, 286 electroconvu lsive therapy for, 214, 235 and risp erid one, 266, 285
Clozaril. See Clozap ine factitiou s d isord er by p roxy and , 208, sid e effects, 173, 192
Cocaine 227 Flu voxam ine (Lu vox)
intoxication, 211, 224, 230231, 245, 266, 285 m ed ical cau ses of, 258, 277 MDD, 221, 241
w ithd raw al, 267, 285 w ith psychotic featu res, 266, 285 OCD, 221, 241
Ind ex 293

Fregoli d elusion, 33 Inhalant, abu se, 222, 243 Malingering, 152, 162, 252, 273
Freu d , Sigm u nd , 140 Inhalant intoxication, 266, 285 conscious p rod u ction of sym ptom s, 252,
Frotteurism , 169, 188 Inp atient facility, ad m ittance, 207, 226227 273
Insanity, 144, 157 d ifferential d iagnosis, 220, 240
G Insanity d efense, 142, 144, 155156, 157 Malp ractice
Galantam ine, 253254, 274 voluntary intoxication and , 150151, 161 harm or d am age in, 141, 154
Ganser synd rom e, 214, 234235, 249, 270 Insom nia, SSRIs and , 173, 192 p hysicians p rotection against, 142, 155
Gend er, su icid e and , 17 Insu rance, 149, 160 stand ard of care, 142, 145
Generalized anxiety d isord er (GAD) Intellectu al d isability, 259, 278279 su icid e and , 150, 161
buspirone for, 216, 236 Intelligence qu otient (IQ), calcu lation, 214, Managem ent, 165203
d ifferential d iagnosis, 174, 179, 194, 234 Mariju ana, 224, 245
198199 Interm ittent exp losive d isord er, 183, Med ical exp ert w itnesses, 151, 161162
Genetics 202203 Mellaril. See Thiorid azine
bip olar d isord er, 180181, 200 IQ. See Intelligence qu otient Mens rea, 142, 156
schizop hrenia, 210, 230 Iron d eficiency, restless legs synd rom e and , Meperid ine (Dem erol), 216, 236
Geod on. See Ziprasid one 183, 202 Methad one, op iate w ithd raw al, 265, 284
Gingival hyperp lasia, 210, 230 Metham phetam ine intoxication, 266, 285
Glasgow Com a Scale, 264, 283 J 3,4-Methylened ioxym etham p hetam ine
Jehovahs Witness, 146, 158 (MDMA) (Ecstasy) intoxication, 266,
H 285
H abeas corp u s, 151, 162 K Methylp henid ate (Ritalin)
H allu cinations KayserFleischer rings, 250, 270271 ADH D, 181, 201, 223, 244
alcohol w ithd raw al and , 221, 242 Klep tom ania, 249, 270 d ep ressive d isord er, 222, 242
com m and au d itory, 167, 181, 187, 200 Korsakoff synd rom e, 180, 199200 Microcep haly, 259, 279
lorazepam for, 221, 242 Mini-m ental state exam ination (MMSE),
H aloperid ol (H ald ol), 205, 225, 262, 281 L 209, 224, 228229, 244, 254, 255, 275
d elirium , 171, 191 La belle ind iffrence, 251, 272 Minnesota Multip hasic Personality
d op am ine recep tors and , 94, 115 Lam otrigine, 262, 281 Inventory 2 (MMPI-2), 212, 233, 255,
forcing treatm ent, 144, 156157 bipolar d ep ression, 215, 236 275
lithiu m and , 168, 187 bipolar d isord er, 257, 277 Minors, confid entiality and , 145, 157
PCP intoxication, 285 sid e effects, 257, 277 Mirtazap ine, 247, 268
postpartum d ep ression, 165, 185 Learning d isord ers MDD, 221222, 242
pregnancy and , 223224, 244 d ifferential d iagnosis, 214, 234 MMPI-2. See Minnesota Mu ltip hasic
torticollis and , 264, 283 Legal issu es, 141163 Personality Inventory 2
H ead ache, 217, 237 child abu se, 144, 157 MMSE. See Mini-m ental state exam ination
H em iballism u s, 250, 271 p aranoid d elu sions, 142, 155 Monoam ine oxid ase inhibitor (MAOI)
H epatic encephalopathy, 177, 196197 w ills, 142, 155 MDD, w ith atyp ical featu res, 261, 280
H eroin Lew y bod y d em entia, 178, 198 p regnancy and , 216, 237
d isord er of, 64 Lithiu m (Eskalith), 205, 225, 262, 281 SSRIs and changing to, 264, 283
intoxication, 266, 285 haloperid ol and , 168, 187 tyram ine and , 264, 283
overd ose, 46, 70 ibup rofen, 221, 242
w ithd raw al, 267, 285 laboratory tests before treatm ent w ith, N
H istrionic personality d isord er, 184, 203 218, 238 N aloxone
H oard ing d isord er, 182, 202 nep hrogenic d iabetes insip id u s, 264265, opiate overd ose, 253, 274
H om icid al threats, confid entiality and , 141, 283284 su bstance d ep end ence and , 217, 238
154 sid e effects, 267, 286 N arcissistic p ersonality d isord er, 184,
H osp italization toxicity, 265, 284 203
involuntary, 143144, 156 Living w ill, 151152, 162 N arcolepsy, d ifferential d iagnosis, 181,
p ostpartum d ep ression, 147148, 159 Lorazep am (Ativan) 200201
partial hosp ital program s, 151, 162 agitation and , 266, 284285 N eologism s, 248, 269
H uffing, 222, 243 for alcohol u se d isord er, 207, 226 N eu rolep tic m alignant synd rom e (N MS),
H um an im m unod eficiency viru s (H IV) alcohol w ithd raw al, 170, 179, 189, 199, 217, 237
d ifferential d iagnosis, 177, 197 221, 241242 characteristics, 166167, 186187
inform ed consent and testing for, 145, for hallucinations, 221, 242 laboratory abnorm alities, 217, 237238
157158 Lu vox. See Flu voxam ine treatm ent, 167, 187
H untington d isease, 178, 198 N eu rosyp hilis, 176, 196
H yperp lasia, gingival, 210, 230 M N icotine, intoxication, 224, 245
H yperreligiou s thinking, 263, 282 Magical thinking, 250, 271 N igrostriatal D 2 recep tors, 258, 278
H ypertensive crisis, 208, 216, 228, 236 Major d ep ressive d isord er (MDD), 208, N itrous oxid e intoxication, 266, 285
H yperthyroid ism , 177, 197 227 N ortriptyline, 262, 281
H ypnotherap y, 123, 132 w ith atyp ical featu res, 214215, 235, 260, N ot gu ilty by reason of insanity, 144, 157
H ypoglycem ia, 177, 197 280
bereavem ent v., 173, 192 O
I d elu sions, 180, 200 Obsessive-com p u lsive d isord er (OCD)
Ibup rofen, 221, 242 d ifferential d iagnosis, 168, 188, 209, 229 antid ep ressants and , 220, 240
Id entification, 215, 235 ECT for, 214, 235 d ifferential d iagnosis, 174, 193194
Im ipram ine (Tofranil) m irtazap ine for, 221222, 242 MDD w ith, 221, 241
contraind ications, 265, 284 OCD w ith, 221, 241 treatm ent, 174, 194
Inform ed consent, 143, 156 w ith p sychotic featu res, 171172, 191 Obsessive-com p u lsive p ersonality d isord er,
cap acity to give, 151152, 162 recu rrent ep isod es of, 256, 276 184, 203, 205, 225
H IV testing, 145, 157158 w ith seasonal pattern, 213, 233234 Obstru ctive sleep ap nea hyp opnea, 220,
obtaining, 151, 162 treatm ent of, 172, 191, 255, 256, 275276 240241
294 Ind ex

OCD. See Obsessive-com pu lsive d isord er Q Sexu al abu se, 273274


Ocu logyric crisis, 168, 188 Qu etiapine (Seroqu el), 211212, 232 Sexu al d ysfu nction, SSRIs and , 169, 188,
Olanzap ine (Zyp rexa) p ostp artu m d ep ression, 166, 185 222223, 243
d elu sional d isord ers, 166, 186 SLE. See System ic lu pu s erythem atosus
hallucinations, 167, 187 R Sleep ap nea, 220, 240241
Opiates. See also specific opiates Racing thou ghts, 210, 230 Som atic sym p tom d isord er, 177, 197
intoxication, 170, 190, 224, 245 Rand om letter test, 252253, 273 Sonata. See Zalep lon
overd ose, treatm ent of, 253, 274 Reactive attachm ent d isord er, 259260, SSRI. See Serotonin-specific reuptake inhibitor
w ithd raw al, 170, 181182, 190, 201 279 St. John w ort, 257, 277
Opp ositional d efiant d isord er (ODD), 73 Refu sal of treatm ent, 143, 147, 156 Steroid -ind u ced p sychotic d isord er
child ren and , 145146, 158 d ifferential d iagnosis, 182, 201
P Jehovahs Witness and , 146, 158 m anagem ent of, 182, 201
Panic attacks Rep orting im p aired colleagu es, 152153, StevensJohnson synd rom e, 257, 277
d ifferential d iagnosis, 168169, 188 163 Stress from w ork, 206, 226
p hysiologic pathology, 248, 268269 Restless legs synd rom e Stroke, visual nonverbal m em ory and , 218,
Panic d isord er d ifferential d iagnosis, 183, 202 238
w ith agoraphobia treatm ent, 183, 202 Stroop Test, 254, 275
d ifferential d iagnosis of, 173174, 193 Retinitis p igm entosa, 264, 283 Subp oena to release m ed ical inform ation,
treatm ent of, 174, 193 ReyOsterrieth Test, 218, 224, 238, 244, 254, 149, 160
d ifferential d iagnosis, 208, 227228 275 Substance intoxication, insanity d efence
sertraline, 207, 227 Risp erid one, 262, 281 and , 150151, 161
treatm ent of, 208, 228 and flu oxetine, 266, 285 Suicid e. See also Physician-assisted su icid e
Paranoid d elu sions, 142, 155 Risp erid one Consta, 262, 281 m alp ractice and , 150, 161
Paranoid personality d isord er, 184, 203 Risp erid one (Risperd al) risk factors, 150, 161, 254, 258, 274, 277
Paroxetine (Paxil), 247, 262, 268, 281 MDD SSRI and , 257, 276277
cognitive d ecline and , 265, 284 w ith p sychotic featu res, 172, 191 Supportive care, for PCP intoxication, 266, 285
Partial hospital p rogram s, 151, 162 Tourette d isord er, 259, 278 Sym m etrel. See Am antad ine
Patient-d octor relationship, 216217, 237 Rorschach Test, 210, 212, 230, 232, 255, 275 Systemic lupus erythematosus (SLE), 177, 197
p aym ent and , 150, 161, 252, 273
sexual, 148, 159 S T
Paxil. See Paroxetine Schizoaffective d isord er Tarasoff v. Regents of University of California,
Personality d isord ers, 183, 203, 261, 280. d iagnosis for, 261, 280 141, 154
See also specific personality disorders m ed ications, 165, 185 Tard ive d yskinesia (TD), 262263, 281282
Phencyclid ine (PCP) intoxication, 224, Schizoid p ersonality d isord er, 180, 184, 200, TD. See Tard ive d yskinesia
245, 267, 285 203 Tem p oral Orientation Test, 211, 231
supportive care, 266, 285 Schizop hrenia Testam entary cap acity, 142, 155, 251, 272
treatm ent for, 266, 285 flat affect and , 206, 226 Test of Mem ory Malingering, 220, 240
Pheochrom ocytom a, 176177, 196 genetics, 210, 230 Thiam ine, Wernicke encep halopathy and ,
Physician-assisted su icid e, 148, 160, 254, negative sym p tom , 206, 226 166, 180, 186, 199, 216, 237, 253, 274
274 risk of, 210, 230 Thiorid azine (Mellaril), 264, 283
Physicianp atient confid entiality, 146, 158, treatm ent, 222, 243, 248, 269 Thou ght broad casting, 250, 271
252, 255, 272273, 275 Schizophreniform d isord er Thyroid d isord er, 210, 230
Pick d isease. See also Dem entia d ifferential d iagnosis, 169, 189, 210, 223, Tofranil. See Im ip ram ine
d ifferential d iagnosis, 178, 198 230, 244 Torticollis, 264, 283
Postp artu m d epression Schizotyp al p ersonality d isord er, 184, 203 Tou rette d isord er, 259, 278
involuntary com m itm ent and , 147148, d ifferential d iagnosis, 206, 209, 225226, cop rop raxia, 249, 270
159 229 echolalia and , 249, 270
w ith p sychosis, 165166, 185 treatm ent, 206, 226 treatm ent, 176, 195
treatm ent, 166, 185 Seizures Toxicology screen, 170, 189190
Postp artu m psychosis, 165166, 185 alcohol w ithd raw al and , 170, 189 brief p sychotic d isord er and , 172, 192
Posttraum atic stress d isord er (PTSD) ECT, 266, 284 u rine, 256, 276
treatm ent, 263, 282 tem p oral lobe, 263, 283 Trazod one (Desyrel), 256, 276
Pow er of attorney, 147, 159 Selective m u tism , 1315 p riap ism and , 215, 236, 256257, 276
Prazosin, 263, 282 Serotonin-sp ecific reu p take inhibitor (SSRI), Triazolam (H alcion), 170, 190
Pregnancy 167, 187 Tricyclic antid ep ressants (TCAs)
fluoxetine and , 256, 276 insom nia and , 173, 192 card iac arrhythm ia and , 169, 189
haloperid ol and , 223224, 244 MAOIs and changing from , 264, 283 Type II error, 262, 281
MAOI and , 216, 237 sexu al d ysfu nction and , 169, 188,
Priapism , 215216, 236, 256, 276 222223, 243 V
Prion (Creu tzfeld tJacob) d isease, 198 sid e effects, 169, 188 Valp roic acid , 262, 281
Pseud od em entia, 178, 198. See also su icid e and , 257, 276277 bip olar I d isord er, 251, 271
Dem entia Serotonin synd rom e, 218, 239 Venlafaxine (Effexor), 247, 262, 268, 281
Psilocybin w ithd raw al, 266, 285 d ifferential d iagnosis, 182, 201202
Psychiatric illnesses, 205, 225 m anagem ent of, 182, 202 W
Psychogenic unresponsiveness, 213, 233 Sertraline (Zoloft) Wad a Test, 255, 275
Psychosis, postpartum , 165166, 185 MDD WAIS-R. See Wechsler Ad u lt Intelligence
Psychotherapy. See also specific w ith p sychotic featu res, 172, 191 ScaleRevised
psychotherapies p anic d isord er, 207, 227 WCST. See Wisconsin Card Sorting Test
ad ju stm ent d isord er, 173, 193 w ith agorap hobia, 174, 193 Wechsler Ad ult Intelligence ScaleRevised
Psychotic d isord er, brief, 172, 191192 for p anic d isord er, 208, 228 (WAIS-R), 214, 234, 255, 275
Psychotic featu res, MDD w ith, 171, 191 schizoaffective d isord er, 165, 185 Wechsler Intelligence Scale for Child ren
treatm ent of, 172, 192 Seru m liver fu nction stu d ies, 217, 238 (WISC), 175176, 195, 224, 244
Ind ex 295

Weight gain, 247, 268 WISC. See Wechsler Intelligence Scale for Z
Wernicke encephalop athy, 166, 186 Child ren Zalep lon (Sonata), 216, 236
m anagem ent, 166, 186 Wisconsin Card Sorting Test (WCST), 209, Zip rasid one (Geod on)
thiam ine and , 166, 180, 186, 199, 216, 237, 229, 254, 275 schizoaffective d isord er, 165,
253, 274 Withd raw al 185
WernickeKorsakoff synd rom e, 65 alcohol, 267, 285 w eight gain and , 247, 268
Wills caffeine, 217, 237 Zoloft. See Sertraline
legality, 142, 155 cocaine, 267, 285 Zolp id em (Am bien), 181, 201, 256,
living w ill, 151152, 162 heroin, 267, 285 276
Wilson d isease, 250, 270271 opiates, 170, 181182, 190, 201

You might also like